Está en la página 1de 812

Modulo de Renal

• La PTH promueve todo lo siguiente excepto:


o EL aumento de la reabsorción tubular de fosfato.
La PTH a nivel renal es eminentemente fosfatúrica, promueve la excreción de fosfato y la
reabsorción de Calcio.
• Síndrome nefrósico de cambios mínimos se caracteriza por:
o Fusión de los pedicelos de los podocitos.
La fusión de los pedicelos no se puede ver en microscopia de luz, hay que usar
microscopia electrónica, pero se puede hacer el dx con la proteinuria masiva
característica del síndrome.
• En un paciente con eclampsia se le ordeno un litro de solución de dextrosa al 5%,
conteniendo 20 ml de MgSO4 al 50% (PM= 100 g), ¿cuantos ml de sal se necesita
para preparar la solución?
o 10 ml.
• El porcentaje de solución es:
o 1%
• ¿Cuantos mEq de Mg tiene la venoclisis?
o 83.3
• La hipocalcemia de la falla renal crónica puede cursar:
o Asintomática.
• La velocidad de administración de KCL en un caso de alto riesgo puede ser:
o 40 mEq/L
En los pacientes con una hipokalemia muy severa se usa 40, pero se debe pasar a una
velocidad de 20, porque si se pasa muy rápido puede ocasionar un bloqueo AV completo.
• Paciente urémico, con K sérico de 6, Academia, EKG normal se trata con:
o Kayexalate
Ya que la hiperkalemia no es tan severa se puede tratar con el Kayexalate, que es una
resina de intercambio catiónico, la misma remueve el potasio del cuerpo.
• Paciente femenina de 50 Kg presenta diarrea, presión arterial un poco baja, sin signos
neurológicos, Na=120, K=2, Cl=100, HCO3=10, asuma 60% de agua, cuantos
gramos de Sal para elevar el Na a 130
o 17.5
Calculo: (60*0.5)*10=300 se necesita dos soluciones salinas (154*2=308), cada solución
salina tiene 58.5 g NaCl y cada gramo de solución tiene 17.5 g de Na.
• ¿Qué solución salina usted escogería?
o 0.9 %
• La hipofosfatemia produce:
o Fosfaturia
La disminución del calcio, produce un aumento de la PTH y esta a su vez un aumento de
la eliminación del fósforo por la orina.
• La hipokalemia puede predisponer, excepto:
o Disminución de la amoniogenesis.
La hipokalemia por lo general cursa con acidosis metabólica, en donde va a estar
aumentada la formación de amonio.
• Un inhibidor de la anhidrasa carbónica es causa de:
o Hipokalemia
Ellos producen un aumento de la excreción de bicarbonato al ser este un anión, arrastra el
potasio que es un catión.
• Un hombre llega al hospital comatoso, con fractura de cráneo, orina 175, suero de
Na=178, K=4, Cl=130, HCO3=25 Osm=350 en la orina 800 (no c ve bien).
o Diabetes insípida central.
Recordar que una de las causas de hipernatremia es la diabetes insípida.
• Confirmaría su dx así:
o Prueba de restricción y administración de vasopresina.
• Causa de HTA con hipokalemia excepto:
o Síndrome de Bartter.
El síndrome de Bartter no cursa con hipertensión arterial (HTA maligna,
hiperaldosteronismo primario y síndrome de hiperprostaglandismo entre otros).
• Se relaciona con hipofosfatemia, excepto:
o Disminución de la afinidad de la hemoglobina por el O2.
La hipofosfatemia produce una disminución del 2,3-DPG, y hay una desviación a la
izquierda, lo que produce una aumento de la afinidad por el oxigeno.
• Paciente con vómitos severos, hipotensión arterial, la orina al azar muestra Na mas de
40, osmolaridad de 800, puede tener:
o Falla suprarrenal.
El paciente al tener una hipovolemia, se activa el SRAA, y aumenta la reabsorción de Na
y disminuye su cantidad en orina.
• Un hombre de 30 Kg padece de Ca pulmonar, tiene 2 semanas de estar letargico y
obnubilado, suero de Na 105, Cl 72, K 4, HCO3 21, Osm 122 y en la orina Na 78,
Osm 604, el dx mas probable:
o SIADH.
Hay una hiponatremia, hipotónica y euvolémica.
• Tratamiento:
o Solución salina al 3%
El tratamiento de SIADH, es restricción de agua y suplir el déficit de sodio.
• Causas de enfermedad tubular intersticial crónica excepto:
o Nefritis alérgica.
En su mayoría medicamentos y medio de contraste.
• La administración de solución salina isotónica produce:
o Aumento del volumen extracelular.
• Sospecha de obstrucción de las vías urinarias en caso de:
o Anuria.
• En la insuficiencia renal aguda siempre da, excepto:
o Retensión nitrogenada
Es un parámetro a tomar en cuenta para dializar al paciente cuando es mayor de 100.
• Siempre habrá insuficiencia renal en caso de:
o Glomérulo nefritis rápidamente progresiva
La proliferación extracapilar comprime el glomérulo comprimiéndolo y dañándolo
severamente.
• En caso de glomérulo nefritis post estreptocócica la filtración glomerular puede
disminuir por:
o Alteración del coeficiente de ultrafiltración.
• En la nefritis túbulo intersticial puede dar lo siguiente, excepto:
o Proteinuria de 4 gr/día
Ese dato es de síndrome nefrósico.
• Es la causa de falla prerrenal.
o Pancreatitis aguda.
• En la insuficiencia renal aguda siempre habrá:
o Aumento de la creatinina.
• Causa de hipmagnesemia:
o Alcoholismo.
Otra causa importante son las pérdidas gastrointestinales.
• La hipofosfatemia causa, excepto:
o Aumento de PTH.
• Paciente con diuresis de 250 ml al día con Osm plasmática y osmolaridad urinaria de
300, el Dx probable:
o FRA, de causa renal.
• Paciente con oliguria, hiperkalemia, hiponatremia, creat 0.8 mg/dl y cpk normal, esto
puede corresponer a:
o Falla renal aguda, de causa prerrenal.
• En falla renal crónica:
o Hipofosfatemia.
• Urémico crónico con K 6.5 con leve academia, sin trastorno muscular y EKG normal,
tratamiento:
o Kayexalate.
• Urémico agudo, acidótico, oligúrico, deshidratado, todo es cierto, excepto:
o Densidad urinaria 1010.
Esto indica isostenuria, el riñón no tiene la capacidad de concentrar y es indicativo de
riñones muy maltratados.
• Falla renal aguda, oliguria, todo es cierto, excepto:
o Osmolaridad urinaria elevada.
• Síndrome nefrósico de cambios mínimos se caracteriza:
o Fusión de los pedicelos de los podocitos.
• En el shock hipovolémico, el uso de dextrosa en agua puede causar:
o Hiponatremia sintomática.
Primero siempre corregir la volemia con SSN 0.9.
• En la deshidratación por perdida fundamental de solutos, se da la siguiente, excepto:
o Entrada de electrolitos al cerebro.

• En el SIADH se da lo siguiente excepto:


o Edema de los miembros inferiores.
• Paciente con gastroenteritis, hipovolémico, cuyo suero contiene Na 155, K 2, Cl 117:
o SSN 0.45 + 40 mEq KCl.
• Una medida general en el tx de un hipercalcémico:
o SSN 0.9%
Para expandir el volumen.
• La velocidad de adminis tracion de K en caso de riesgo para la vida puede ser:
o 40
• Un ejemplo de hipernatremia hipervolemica es:
o Hiperaldosteronismo primario.
• Una dieta de 3gr de NaCl corresponde a:
o 17 gr de Sodio.
• La IRC ocaciona hiperfosfatemia permanente cuando la TFG esta por debajo de:
o 25 ml/min
El valor es por debajo de 30.
• Paciente con alcalosis metabólica e hipokalemia severa debe ser tratado con:
o KCl
• No esta indicada en la hipercalcemia:
o Aldactone
Es un diurético ahorrador de K, ósea produce hipercalcemia, se utiliza en el
hiperaldosteronismo primaria y edema.
• Metabolismo del calcio están implicados menos:
o Insulina.
• Hiponatremia hipotónica normovolémica:
o SIADH.
• Causa de enuresis
o Necrosis cortical difusa.
• La situación clínica de hiperfosfatemia con hipocalcemia con cirrosis hepática es:
o Cirrosis alcohólica.
• Factor que altera el balance hídrico en el periodo post operatorio excepto:
o Disminución de ADH.
• Ejemplo de hipernatremia hipervolemica:
o Hiperaldosteronismo primario.
• Ejemplo de hipokalemia con aldosterona alta y osm baja:
o Hiperaldosteronismo primario.
También con renina baja.
• Hiperkalemia sin alteración en el EKG no es necesario administrar:
o Gluconato de calcio.
• Urémico crónico con deshidratación hipertónica severa y acidótico presentara lo
siguiente excepto:
o Osmolaridad urinaria de 800.
• La fracción de excreción de Na esta aumentada en:
o Falla cardiaca.
También esta aumentada en FRC, de causas renales.
• Paciente con vomito abundante, HTA, Una 6, oligúrico y osmolaridad de 800,
presentara:
o Insuficiencia suprarrenal.
• En cirrótico con ascitis se recomienda, excepto:
o Dieta con 8.5 mEq de Na.
60 to 90 mEq per day, equivalent to approximately 1500 to 2000 mg of salt per day
(NEJM).
• Factor que predispone a la hipernatremia son, excepto:
o Intoxicación hídrica.
• En la hiperkalemia:
o Hay menos diferencia entre el potencial de reposo y el umbral.
• Los indicios de falla renal aguda son:
Pueden ser cualquiera de las siguientes:
o Alteraciones en la osmolaridad urinaria.
o Alteraciones sen la relación Uosm/Posm.
o Alteraciones en el Una.
o Alteraciones en el FENA.
• Se presenta en acidosis tubular renal proximal, excepto:
o NaHCO3 y los alpha agonista.
• Na urinario menor de 10:
o Falla cardiaca.

Examen de Renal
1. Potencia los efectos de Hiperkalemia
a. Hipocalcemia b.hipercalcemia c. alcalemia d.
hipermagnesemia

2. La mayor cantidad de magnesio se encuentra en:


a. Hueso b. hígado c. Músculo d. eritrocito e. neurona

3. Hipokalemia con hipertensión arterial, aldosterona elevada y renina


plasmática baja
a. Tumor secretor de renina b. Hipertensión acelerada c. estenosis de
la arteria renal d. hiperaldosteronismo primario e. síndrome de
Liddle.

4. En el tratamiento de la hipermagnesemia se puede utilizar


a. Cloruro de Calcio b. gluconato de calcio c.solución salina al 0.9%
d. A y B e. B y C

5. La ausencia de proteínas y elementos celulares en la orina es más


común excepto:
a. Litiasis renal unilateral b.hiperplasia prostática benigna
c. deshidratación d. uropatía obstructiva bilateral e. falla renal
parenquimatosa.

6. En la glomerulonefritis aguda pos infecciosa, la filtración


glomerular…. No alcanzo a leer que más sigue
a. Proteinuria masiva b .Alteración del coeficiente de filtración
glomerular c. aumento de la presión oncótica d. disminución de la
presión arterial e. A y D

7. Hipermagnesemia de 10 meq/l es de alto riesgo porque puede darse


a. Abolición de reflejos osteotendinosos b. cuadriplejía c.
vasodilatación periférica d. bloqueo cardiaco e. depresión
respiratoria

8. En el tratamiento general de la urgencia hipercalcémica se utiliza con


prioridad
a. D/A 5% b. Solución salina 17.7% c. solución salina al 0.9%
d. bifosfonatos e. calcitonina

9. Paciente con uremia crónica, deshidratación severa puede presentar


excepto:
a. Oliguria b. Densidad urinaria 1.010 c. Osmolaridad urinaria de
800 meq/l d. hiperkalemia e. hiperfosfatemia

10. Paciente con diuresis de 12 ml/hora, potasio sérico 7 mm/l,


osmolaridad 300 mosm/l diagnostico más probable
a. SIADH b. insuficiencia suprarrenal c. Insuficiencia renal d.
deshidratación hipertónica e. todas las anteriores

11. Paciente con vómitos abundantes con hipotensión arterial ph:7.40


Pco2: 4i mmhg HC03: 25mm/l Na:132 meq/l K:3.2 Cl-: 86mm/l N
de urea: 40 mg/dl
a. No tiene trastorno ácido-base b. alcalosis metabólica severa
c.Acidosis metabólica con acidosis respiratoria s. acidosis
metabolica con alcalosis metabólica e. brecha anionica elevada
12. En la hipokalemia se presenta
a. Disminución de la sístole eléctrica b. menor diferencia entre el
potencial umbral y de reposo c. la onda P alta d. potencial umbral
es menos negativo e. potencial de reposo transmembrana más
negativo.
13. El shock séptico puede presentar:
a. Alcalosis respiratoria con acidosis metabólica con brecha aniónica
normal
b. .alcalosis metabólica con alcalosis respiratoria
c. Alcalosis respiratoria con acidosis normoclororémica
d. Alcalosis respiratoria con acidosis metabólica con brecha aniónica
elevada
e. Ninguna de las anteriores

14. Puede causar hipocloremia… la foto se cortó, pero en este cuadro


estab las causas

15. Causas de hipercalcemia excepto:


a. Uso de laxantes que contienen fosfato b. enemas que contiene
fosfatos c. administración de fosfato intravenoso d. falla renal
crónica e. enfermedades granulomatosas.

16. PH 7,20 HCO3: 8mm/l PCO2: 10 mmhg


a. Acidosis metabólica + alcalosis respiratoria b. acidosis mixta
c. acidosis metabólica pura d.acidosis metabólica + acidosis
respiratoria aguda e. acidosis metabólica + acidosis respiratoria
crónica

17. Paciente con Sodio serico 115 mm/l, glucosa 90 mg/dl y urea 300
mg/dl podría presentar:
a. Agitación b. letargia c.calambres musculares d. todas las
anteriores e ninguna de las anteriores

18. En la hipokalemia puede darse excepto


a. Rabdomiolisis b.una onda accesoria c. intoxicación digitálica d.
QT prolongado e. disminución de la amniogenesis

19. Cuando el nivel de K de 10mm/l, debido a pseudohiperkalemia debe


administrarse.
a. Kayexalate b. solución hipertónica de glucosa c. bicarbonato de
sodio d. gluconato de calcio 10% e. ninguna de las anteriores.

20. Causas de falla renal aguda excepto:


a. Medios de contraste b.cilindros hialinos c. aminoglucósidos d.
cadenas Kappa e.mioglobina

21. Es indicación para dializar:


a. Potasio sérico mayor 6mm b. Urea sérica de 50 mm/l c. náuseas y
vómitos incontrolables d. síntomas neurológicos e. C y D

22. Cuando disminuye el calcio sérico:


a. Disminuye fosfaturia. b. disminuye la resorción ósea c. inhibe la
paratohormona d. aumenta la absorción intestinal de fosfato e.
disminuye la reabsorción intestinal de fosfato.

23. Calcule en déficit de agua en paciente de 70 kg con 50% de agua


corporal y sodio serico 160 mml
a. 3L b. 10L c. 7L d. 5L e. 6L
24. Enfermo de 60 kg y 60% de agua corporal Posm 250 mOsm/kg
¿Cuántos gramos de Nacl elevarían el Na a 10 mm/l?
a. 21 b.30 c. 19 d. 26 e.15

25. En hipokalemia
a. Aumenta ruidos intestinales b. siempre produce cambios
electrocardiográficos c. disminuye la diferencia entre potencial de
membrana y umbral d. aumenta irritabilidad neuromuscular e.
aumenta magnitud del potencial de acción.

26. Cuando disminuye el fosfato sérico:


a. Disminuye PTH b. Disminuye absorción intestinal de calcio
c. Aumenta fosfaturia d. disminuye calcitriol e. C y D

27. Es una causa de hipocalcemia excepto


a. Esprue Tropical b. pancreatitis crónica c. cirrosis biliar d. factor
de necrosis tumoral e. desnutrición

28. Hallazgos de laboratorios en nefritis tutbuloinsterstricial


a. Cilindros hemáticos b. hematuria c. proteína masiva d. A Y B e.
hipostenuria.

29. En toda alcalosis metabólica se da lo siguiente


a. PH urinario disminuido b. fase de mantenimiento c. brecha
aniónica normal d. disminución sérica de lactato e. resistencia a
cloruro

30. Se utiliza para extraer potasio del cuerpo


a. Bicarbonato de sodio b. solución hipertónica de glucosa c.
solución hipotónica de glucosa d. sulfonato de poliestireno
e.albuterol

31. Alcalosis respiratoria


a. NH4 y H3PO4 disminuidos en orina b. NH4 disminuido y H2PO4
aumentado en orina c. NH4 y H2PO4 aumentados en orina d.
NH4 aumentados y H2PO4 disminuidos en orina e. Ph urinario de
10

32. La mayor reabsorción de magnesio ocurre en:


a. Porción contorneada del túbulo proximal b. rama gruesa
ascendente del asa de henle c. porción recta del túbulo proximal
d. túbulo distal e. túbulo colector

33. Causa de brecha aniónica disminuida


a. Hipomagnesemia b. intoxicación por litio c. hipocalcemia
d.hipokalemia e. intoxicación por metanol

34. Causa de hipofosfatemia grave excepto


a. Falla renal b. recuperación de cetoacidosis diabética c.alcalosis
respiratoria d. quemaduras graves e. nutrición parenteral

35. Acidosis respiratoria


A. disminución de reabsorción proximal del bicarbonato b. aumento
de la reabsorción proximal del bicarbonato c.disminucion el
bicarbonato del eritrocito d. disminución de la secreción de protones
del túbulo distal e. no alcanzo a leer lo que dice sale excreción
urinaria de NH4.

36. En una falla renal aguda usted espera encontrar


a. Densidad urinaria 1010 b. densidad urinaria 1020 c. osmolaridad
urinaria 300 mOsm/kg d.osmolaridad urinaria 800 mOsm/kg e. A
yC

37. Neoplasias malignas comúnmente asociadas con hipercalcemia,


excepto
a. Carcinoma mamario b. hemagioma c. carcinoma pulmonar d.
leucemia e. linforma

38. Ph normal con bicarbonato elevado


a. Alcalosis metabolica + acidosis respiratoria
b. Alcalosis metabolica + alcalosis respiratoria
c. Acidosis metabolica + acidosis respiratoria aguda
d. Alcalosis respiratoria +acidosis respiratoria
e. Ninguna de las anteriores

39. Alcalosis metabólica con hipertensión anterial e hipokalemia excepto


a. Hipertension maligna b.deficiencia de 11 beta hidroxilasa c.
deficiencia de 17 beta hidroxilasa d. Síndrome de Gittelman

40. Característica de …. Túbulo instersticial (no se entiende bien en la


foto lo que dice sale borroso).
a. Perdida de la capacidad para concentrar la orina b. hipertensión
arterial c. edema d. proteinuria mayor a 3 gramos e.
macroalbuminuria.

41. Plasma: creatinina: 1 meq/l ; sodio140 mm/l Orina: creatinina: 150


mm/l ; sodio 40 mm/l. la fracción de excreción de sodio:
a. 2% b.0.9% c.19% d.0.3% e. 4%

UNIVERSIDAD LATINA
VIII SEMESTRE JUNIO 2008

1. Riesgo de la hipermagnesemia
a) Cianosis
b) Hipertensión arterial
c) Aumento de la irritabilidad neuromuscular
d) Paro respiratorio

2. Ejemplo de hiponatremia hipotónica normovolémica


a) Diarrea
b) Cirrosis hepática
c) Síndrome nefrósico
d) SIADH

3. La hipocalcemia provoca
a) Acortamiento del QT
b) Disminución de la irritabilidad neuromuscular
c) Afecta el potencial de reposo
d) Potencializa los efectos de la hiperkalemia

4. Tratamiento de la hiperfosfatemia con tasa de filtración de 5 ml/min


a) Furosemida
b) Expansión con solución salina al 0,9%
c) Gel hidróxido de aluminio
d) Diálisis

5. Calcule el déficit de agua en una paciente que pesa 70Kg y tiene sodio sérico en 168 mM/L
¿Cuánto es el déficit de agua, en litros? (Utilice 50% de agua corporal y el sodio normal de 140
mM/L).
a) 10
b) 5
c) 7
d) 3

6. La hipokalemia puede provocar en un cirrótico


a) Acortamiento del QT
b) Aumento de la amoniogénesis
c) Disminución de la amoniogénesis
d) Ninguna de las anteriores

7. En una deshidratación intracelular severa, se espera lo siguiente, excepto


a) UOsm > POsm
b) UNa < 10 mM/L
c) LIC disminuido
d) FENA > 3%

8. Para un calcio total sérico de 7mg/dl y albúmina de 2,2 g/dl ¿Cuál será el calcio total corregido?
Tome el valor de la albúmina normal de 4 g/dl
a) 7,44 mg/dl
b) 8,44 mg/dl
c) 9,44 mg/dl
d) 8,90 mg/dl

9. Velocidad de administración de KCl en condiciones normales, en mEq/hora


a) 8
b) 120
c) 20
d) 40

10. Enfermo de 60Kg con 60% de agua corporal, POsm 260 mOsm/Kg ¿Cuántos gramos de NaCl
aumentarían el sodio en 10mM/L/día
a) 30
b) 15
c) 21
d) 19
11. Parte del síndrome nefrósico es:
a) Proteinuria menor de 2 gramos
b) Albúmina sérica de 4 g/dl
c) Hiperglobulinemia
d) Proteinuria mayor de 3,5 gramos por día

12. El sedimento urinario en uropatía obstructiva puede mostrar:


a) Cilindros cereos
b) Cilindros tubulares anchos
c) Cilindros de eritrocitos y leucocitos
d) Cilindros granulares

13. La pielonefritis aguda se caracteriza por:


a) Poliuria
b) Polaquiuria
c) Urgencia urinaria
d) Fiebre

14. Paciente con diarrea severa, oliguria de 3 días de evolución. Examen físico: hipotensión
arterial, deshidratación. Plasma: creatinina 9 mg/dl, N de urea 110 mg/dl Na+ 126 mM/L K+ 3mM/L
UNa 40 mM/L UOsm 300 mOsm/Kg. Diagnóstico probable:
a) Falla cardiaca
b) SIADH
c) Insuficiencia prerrenal
d) Insuficiencia renal

15. Sugiere glomerulitis


a) Eritrocituria
b) Leucocituria
c) Cilindros de eritrocitos
d) a y b

16. La solución salina al 23.4% tiene osmolaridad (mOsm/L) de:


a) 80
b) 800
c) 8000
d) 80000

17. Uno de los índices de falla renal aguda es:


a) Oferta tubular de sodio / Excreción urinaria de sodio X 100
b) Reabsorción de sodio / Oferta tubular de sodio X 100
c) UNa / PNa X 100
d) Excreción urinaria de sodio / Oferta tubular de sodio X 100
18. Paciente con oliguria y en el suero presenta creatinina, CPK normales, sodio disminuido y
potasio elevado; esto puede corresponder a:
a) Rabdomiolisis
b) Síndrome hepatorrenal
c) Insuficiencia suprarrenal
d) Insuficiencia renal

19. Es una indicación para dializar:


a) Potasio sérico de 6 mM/L
b) Creatinina sérica de 4 mg/dl
c) Etapa 3 de la enfermedad renal crónica
d) Encefalopatía urémica
20. La mielinolisis pontina puede presentar
a) Calambres musculares
b) Espasticidad de las piernas
c) Cuadriplejía flácida
d) No compromete los pares craneales

21. Urémico crónico con deshidratación hipertónica severa, con acidemia, puede presentar lo
siguiente, excepto:
a) Densidad urinaria 1,010
b) Hiperkalemia
c) UOsm 800 mOsm/Kg
d) Oliguria

22. En la secreción inapropiada de ADH se da lo siguiente, excepto:


a) Disminución de la POsm
b) Aumento de peso
c) No hay expansión de volumen
d) UNa > 20 mM/L

23. Causa falla prerrenal


a) Obstrucción de ambos uréteres
b) Falla cardiaca
c) Glomérulonefritis aguda
d) Síndrome nefrósico

24. Vasculitis que afecta los pequeños vasos es:


a) Arteritis de células gigantes
b) Poliarteritis microscópica
c) Poliarteritis nodosa
d) Enfermedad de Kawasaki

25. La falla renal aguda se caracteriza por:


a) Necrosis papilar
b) Proteinuria masiva
c) Cese súbito de la función renal
d) Aumento del ácido úrico

26. Situaciones en que se puede presentar simultáneamente síndrome nefrósico y síndrome


urémico, excepto:
a) Diabetes mellitus
b) Lupus eritematoso sistémico
c) Amiloidosis
d) Hipertensión renovascular

27. El tratamiento más adecuado en un paciente con falla renal crónica, oligúrico, con edema e
hipertensión arterial y sodio sérico de 120 mM/L es:
a) Restricción de líquidos
b) Restricción de sal y suministrarle diurético
c) Restricción de sal
d) a y b

28. Produce necrosis en la falla renal crónica


a) Calcificación periarticular
b) Calcificación visceral
c) Calcifilaxia
d) Osteítis fibrosa quística

29. La anemia en la falla renal crónica


a) Es frecuente
b) Responde a la eritropoyetina
c) Es normocítica normocrómica
d) Todas las anteriores

30. Etapa 3 de la enfermedad renal crónica. Filtración glomerular en ml/min


a) 15-29
b) 60-89
c) 30-59
d) Menor de 15
1. Favorece la penetración de amoniaco al cerebro situaciones de:
a) Acidosis metabólica
b) Alcalosis con Hipopotasemia
c) Acidosis con Hiperpotasemia
d) Alcalosis con hiperpotasemia

1. Paciente que pesa 50Kg, con sodio sérico de 160 mEq/dL , con ACT =40%
, su déficit de agua es:
a) 5L b) 3L c) 4L d) 6L

2. Para un calcio total sérico de 7mg/dl y albúmina de 2,2 g/dl ¿Cuál será
el calcio total corregido? Tome el valor de la albúmina normal de 4 g/dl
a) 7,44 mg/dl b) 8,44 mg/dl c)9,44 mg/dl d) 8,90 mg/dl

3. Velocidad de administración de KCl en condiciones normales, en


mEq/hora
a) 8 b) 120 c) 20 d) 40

4. Causa falla prerrenal


a)Obstrucción de ambos uréteres b)Falla cardiaca c)
Glomérulonefritis aguda d) Síndrome nefrósico

5. Manifestación en el EKG de un paciente con Hipokalemia:


a) Onda T picuda b) Supradesnivel del segmento ST
c) Intervalo QT deprimido d) Onda S profunda
e) Prolongación del intervalo PR f) N/A

6. Cuál es la concentración sérica de fósforo por encima de la cual se


considera que una persona tiene Hiperfosfatemia (en mg/dL):
a) 3.0 b) 4.0 c) 5.0 d) 6.0 e) 7.0
7.. Diurético de uso común en el tratamiento del edema en los
síndromes nefrítico y nefrótico:
a) Indapamida b) Espironolactona c) Furosemida d)
Acetazolamida
e) Hidroclorotiazida f) N/A

8. Cuál es el principal tratamiento para pacientes con nefritis


tubulointersticial
alérgica:
a) Prednisona b) Enalapril c) Ibuprofeno d) Diálisis e) N/A

9. Cuál de las siguientes enfermedades puede producir un Síndrome


Nefrítico
mediado por inmunocomplejos
a) Granulomatosis de Wegener b) Síndrome de Goodpasture
c) Púrpura Trombocitopénica Trombótica d) Amiloidosis e)
N/A

10. Ejemplo de hiponatremia hipotónica normovolémica


a)Diarrea B)Cirrosis hepática c) Síndrome nefrósico d) SIADH

11. Mencione cuál de las siguientes condiciones puede ser causa de


Hiperkalemia:
a) Uso de IECA b) Hiperaldosteronismo primario c)
Hiperinsulinemia d) Alcalosis Metabólica e) N/A

12. Mencione cuál de las siguientes condiciones cursa con el


desarrollo de
Enfermedad Crónica de los Riñones:
a) Nefrolitiasis unilateral b) Enfermedad de Cambios Mínimos
c) Nefropatía Diabética d) Carcinoma de Riñón e) N/A

13. Oliguria implica una diuresis menor de (mL/día):


a) 50 b) 400 c) 800 d) 1000 e) N/A

14. Señale la aseveración Incorrecta acerca de la Hormona


Antidiurética:
a) Polipéptido sintetizado en el hipotálamo
b) También se le conoce como Arginina Vasopresina
c) El efecto neto de su acción a nivel de la nefrona es la
reabsorción pasiva de agua y sodio
e) El mayor estímulo para su secreción es la hipertonicidad

15. Generalmente la proporción BUN / creatinina plasmática en


hiperaazoemia prerrenal es de:
a) <10 b) > 20 c) 100% d) d) 1%

16. El sodio urinario en hiperazoemia renal intrínseca es de:


a) <10 b) >20 c) > 30 d) >1

17. La causa principal del síndrome nefrítico agudo es:


a) Hipertensión arterial b) GN postestreptocóccica c) Síndrome
deGoodpasture
d) Nefritis lúpica

18. Cuál es el componente esencial en el síndrome nefrótico?


a) hipoalbuminemia b) hiperlipidemia c) proteinuria d)
edema

19. Cuadro clínico clásico del síndrome nefrítico incluye lo siguiente


excepto:
a) lipiduria b)hipertensión c) Oliguria d) Hematuria

20. El tx en cuarto de urgencias de hipercalcemia intensa es:


a) Bifosfonatos b) Furosemida y SSisotónica c) Furosemida y
Bifosfonatos
d) SSisotónica +Calcitriol.

Toxicología
1. Paciente intoxicado con medicamento para los ojos presenta: rubor,
mucosas secas, Intranquilo y retención urinaria su tx incluiría:
a)Fisostigmina b) Atropina c) Propranolol d) Flumazenil

2. Paciente rociado con un tóxico llamado Malatión podrá presentar los


siguientes síntomas muscarínicos excepto:
a) lagrimeo b) sialorrea c) broncoespasmo d) rubicundez
b)
3. Paciente en coma porque se tomó 30 pastillas de alprazolam, enojado con su
novia. Usted utilizaría el siguiente antídoto:
a) Atropina b) Flumazenil c) Protamina d) Fisostigmina

4. Cuál de las siguientes no es una medida para evitar absorción del tóxico:
a) Descontaminación oral b) D. digestiva c)Diuresis forzada d) D.
Cutánea

5. Pte mordido por una culebra , no presenta signos al inicio, 24 horas después
presenta ptosis Palpebral. La intoxicación ofídica la clasificaría como:
a) Síndrome Botrops b) S. Elapídico c) S. Clotalico d) S. Botulínico

6. Pte mordido por una culebra en el pie , con aparición de equimosis , edema
hasta 1/3 medio de la pierna , con TPT y TP alterados ligeramente la
intoxicación ofídica se clasifica como:
a) Grado 0 b) leve c) moderada d) Severa

7. El tx con suero antiofídico del paciente anterior sería así:


a) < 5frascos b) 5-9 frascos c) 15 frascos d) > 15 frascos.

8. El tx de las arritmias ventriculares por intoxicación con cocaína incluye:


a) Digoxina b) Amiodarona c) Nifedipina d) Propranolol

9. Pte de 18 años, llega con sialorrea, miosis pupilar, broncoespasmo, dice


que tomó “Baygon”, usted lo trataría con:
a) Fisotigmina b) Atropina c) Flumazenil d) N- acetilcisteína

10. Tx sugerido para el intoxicado por Paraquat:


a) O2 b) Tierra fuller c) Diazepam d) Atropina

11. Intoxicaciones que cursan con excitación todo lo siguiente excepto:


a) Atropina b)Opiaceos c) Abstinencia d) Teofilina

12. Antídoto para Heparina es:


a) Protamina b) Coumadin c) Vitamina K d) Plasma fresco

13. Intoxicación por Malatión usted usaría como Antídoto


a)Fisostigmina b) Atropina c) Propranolol d) Flumazenil
14. Prioridad del manejo general en un intoxicado:
a) Canalizar vena b) Oxigenoterapia c) Glasgow d) Permeabilidad de
vías aéreas

3 Parcial de MEDICINA INTERNA


Renal y Gastro
ENERO 2006

1. Causa de hiperfosfatemia:
a. Falla hepatica
b. Falla renal cronica
c. Falla cardiaca
d. Insuficiencia suprarrenal

2. Causas de hipofosfatemia grave, excepto:


a. recuperacion de cetoacidosis diabetica.
b. Alcalosis respiratoria
c. Acidosis respiratoria.

3. Paciente con vómitos severos, oligurico con hipotensión arterial Una> 40……
800 mOsm/Kg puede tener:
a. Insuciencia renal aguda.
b. Insuficiencia suprarrenal
c. Falla pre-renal
d. Exceso de mineralocorticoides

4. Paciente urémico crónico con deshidratación severa puede presentar lo siguiente


excepto:
a. Densidad urinaria 1010.
b. Osmolaridad urinaria de 800 mOsm/Kg
c. K sérico aumentado
d. Oliguria.

5. La hipokalemia produce:
a. Aumento de la magnitud del potencial de reposo

6. El marcador tumoral mas importante del Ca de páncreas:


a. AFP
b. CA 19-9
c. ACE
d. K-ras

7. El siguiente no es factor de riesgo para cancer de colon:


a. AINES.
b. Acidosis biliar
c. Tabaquismo
d. Dietas altas en grasas

8. En cuanto a la AFP, todo es cierto, excepto:


c. Suele estar aumentada en colangiosarcoma
9. El paciente con esófago de Barret:
a. Generalmente se presenta con disfagia
b. Necesita vigilancia endoscopica cada 24 meses.
c. Necesita resección esofágica
d. Tiene que tratarse con fundoplicatura de Nissen

10. En la acalasia la siguiente aseveración es falsa:


a. Hay disfagia a sólidos y líquidos
b. Hay riesgo aumentado de carcinoma esofágico
c. El tratamiento restaura la peristalsis
d. La tox. Botulínica causa mejoría transitoria

11. Son factores en la clasificacion de Child-Pugh excepto:


a.Transaminasa
b. ascitis
c. bilirrubina
d. albumina

12. Un adulto de 25 años asintomático con ALT 60, AST 50, HBsAg(+), HBcAb
IgM(-) HAV IgG(+):
c. Hepatitis B cronica

13. El se debe vacunar contra la hepatitis tipo:??


a. A
b. B
c. C
d. Ninguna de las anteriores(no es esta)

14. Un Paciente hemofilico con 22 años con ictericia y ALT 1000, AST 900, HBsAg
(+), HBc AbIgM(+), HBCAb(+)
a. Hep. C cronica agravado x Hep B aguda (no es esta)
b. Hep B aguda
c. Inmunidad natural contra la Hep B
d. Hep B cronica

15. Se debe vacunar contra la Hep:


A

16. Falso acerca del H. pylori:


La mayoria de los pacientes con H pylori tienen ulcera

17. Son drogas hepatotoxicas excepto:


a. Ticlopidina
b. Metil dopa
c. Fluoxetina
d. Pravastatina

18. En la cirrosis hepatica generalmente se encuentra:


Anemia normocitica, leucopenia, trombocitopenia.

19. Causa de HT portal sinusal:


a. Esquistosomiasis
b. Síndrome de Budd Chiari
c. Colestasis calculosa cronica
d. Trombosis de vena porta

20. Se encuentra gradiente >1,1:


b. Síndrome de Budd Chiari

21. Son causa de encefalopatia hepatica, excepto:


a. síndrome febril

22. paciente de 35 años, ud sospecha de enfermedad de colon ya q consulta x diarrea


cronica, cual de las siguientes enfermedad no forma parte del diagnostico
diferencial:
a. Colitis por tuberculosis.
b. Colitis amebiana
c. Colitis inespecifica
d. Síndrome de colon irritable

23. Cual de los siguientes fármacos es util en la prevencion de la recidiva de la


enfermedad inflamatoria intestinal no es activo en el brote agudo?
Azatriopina

24. Síntoma inicial de la cirrosis biliar primaria:


a. Icteria
b. Hiperpigmentacion cutanea
c. Prurito
d. Xantelasma

25. El aumento de -fetoproteina serica puede detectar , excepto


c. Carcinoma pancreatico

26. Primera*****Falla renal crónica:


a.HTA
b.Glomerulopatias
c.Diabetes Mellitus
d.Nefritis tubulo intersticial
27. Uno de los indices de falla renal aguda es:
Uosm/Posm

28. Hormona producida en los riñones que disminuye en la falla renal cronica:
a. ADH
b. Noradrenalina
c. Peptido natriuretico (segun arjona)
d. 1,25 (OH)2 Vit D.

29. Las calcificaciones metastásicas se deben:??


a. Aumento del producto Ca, P
b. Aumento del producto Ca, Mg
c. Hipoparatiroidismo
d. Hiperparatiroidismo 1ario

30. Causa de anuria:


a. Pielonefritis unilateral
b. Necrosis tubular aguda
c. Enf cambios minimo
d. Necrosis cortical

31. la etapa de la insuficiencia renal cronica con tasa de filtración glomerular de 60-
89 ml/min es la:??
a. 1
b. 2
c. 3
d. 4

32. La FENA es la relación entre:


a. Oferta tubular/Na reabsorbido x 100
b. depuración de Na/ depuración de creatinina x 100
c. oferta tubular/ Na excretado x 100.
d. Na excretado/ Na reabsorvido x 100.

33. En la nefritis tubular intersticial aguda se pierde la capacidad de poder concentrar


la orina debida a:??
a. Daño medular
b. Daño glomerular
c. Daño tubular proximal
d. Daño de la porcion cortical del tubulo colector

34. La solucion de NaCl al 23,4 % tiene:


8000 mOsm/l
4000 mOsm/l
35. Se asocia con el SIAD, excepto:
lipoma

36. El efecto biologico mas importante de la calcitonina es, excepto:


a. Responder a la hipocalcemia
b. Aumenta la calciuria
c. Estimular la resorcion osea osteoclastica
d. Utilizada en el tx de la hipercalcemia.

37. Favorece la entrada de K al espacio intracelular, excepto:


a. Agonista adrenergico
b. Estimulacion adrenergico
c. Insulina
d. Alcalosis metabolica

38. Factores que afectan el balance hídrico en el post-operatorio, excepto:


a. Fármacos
b. Calcitriol
c. Administración de liquidos hipotonico
d. Aumento de la ADH

39. Las primeras manifestaciones clínicas de la hiponatremia son:??


a. neurológica
b. gastrointestinal
c. pulmonares
d. musculares

40. En la hipernatremia hipertonica se da lo siguiente excepto:


a. Entrada de Na
b. Salida de aa
c. Ganancia de iones
d. Disminución del agua cerebral

41. Manifestaciones clinicas de la hipercalcemia, excepto:


a. Taquicardia
b. HTA
c. Constipación
d. Vomitos

42. En falla renal aguda por nefrotoxicidad:


a. Se conserva la membrana basal
b. Se destruye la membrana basal
c. Frecuente hay oliguria
d. La creatinina no aumenta
43. Causa de hipomagnesemia:
alcoholismo

44. Na serico de 168 mEq/l 70 kg (utilize como % de agua al 50% y Na normal 140
mEq/l
7 litros

45. Indicaciones de diálisis en falla renal aguda, excepto:


a. Encefalopatia uremica
b. Hiperkalemia refractaria
c. Ac. Metabolica refractaria
d. hipocalcemia

46. El KCL IV se puede asministrar asi, excepto:


a.infusion por linea periferica de 10 meq en 100 ml en 1hora
b. “ “ central de 20 meq em 50ml en 1 hora
c. “ “ central de 100 meq em 50ml en 1 hora
d. “ “ periferica de 20 meq en 20ml en 2 horas

47. Aumenta la excrecion urinaria de K, excepto:


a. Triamtereno
b. Bumetanida
c. Furosemida
d. Metolazona

48. Factores que regulan la secrecion distal de K, excepto:


a. Flujo tubular distal y aporte distal de Na
b. Excrecion de aniones no reabsorbibles
c. Aldosterona
d. PTH

49. Enfermo de 60 Kg con 60% de agua corporal, Posm 260 mOsm/kg, ¿Cuántos
gramos de NaCl aumentaria el Na en 10 mEq?
a. 21 gramos

50. Conteniendo 10 g de MgSO4 al %, cuantos ml de sal necesitaria?


20 ml
3er Parcial
Medina Interna

1Mencione dos enfermedades que se asocien a síndrome de mal absorción:


a.Sprue Celiaco
b.Intolerancia a lactosa

2. En relación a los factores etiológicos y manifestaciones clinicas del carcinoma


hepatocelular las siguientes afirmaciones son correctas Excepto:
a. Asienta sobre una hepatopatia crónica en el 90% de los casos y se ha descrito la
implantación tumoral sobre cirrosis de cualquier etiologia

3.Un varón de 70 años de edad tiene una historia larga de estreñimiento y episodios breves
y recurrentes de dolor abdominal inferior intenso. En la actualidad esta asintomático, el
enema de bario muestra enfermedad diverticular de colon, cual de los siguientes es el tx
adecuado:
a. coloide productor de masa, como salvado

4. De las siguientes afirmaciones con respecto a la colitis ulcerosa. Cual es falsa?


a. no esta relacionada con alteraciones inmunológicas

5. Ante un individuo de 25 años, asintomático y con elevación de la concentración serica


de bilirrubina no conjugada, ausencia de hemólisis y normalidad de la función hepática,
cual seria el dx mas probable:
c. Síndrome de Gilbert

6. Mujer de 58 años sin hábitos ni antecedentes patológicos de interes, que desde hace dos
meses refiere astenia franca y ningún síntoma digestivo. Toma ibuprofeno de forma
ocasional por dolores lumbares inespecíficos. La analítica revela Hb 9, Ht 29, VCM 79 y
ferritina 14, cual de las entidades que a continuación de mencionan NOO debe incluirse en
el diagnostico diferencial de este caso?
a. lesiones agudas o cronica de la mucosa GI secundarias al consumo de AINES
b. Cancer colorrectal
c. Angiodisplasia de Colon
d. Gastritis atrofica
e. ulcera gastrica en hernia parahiatal

7. Que actuación posee MENOR utilidad en el tx de la acalasia de esófago?


d. Tx endoscopico con Argon cc

8. En un px de 66 años, diabetico no insulinodepend. Al que se le ha realizado una


endoscopia, se ha detectado una lesion extensa en el cuerpo gastrico. La biopsia viene
informada como adenocarcinoma de tipo intestinal. Cual de las siguientes afirmaciones es
la correcta?
c. El siguiente paso es la realización de ecografia y TAC abdomino pelvico.

9. Hombre de 45 años acude a urgencias porque lleva 6 hrs con dolor abdominal continuo
con exacerbaciones localizadas en epigastrio e hipocondrio derecho, junto con nauseas y
dos episodios de vómitos alimentario bilioso. No es bebedor habitual. En los últimos meses
ha tenido molestias similares pero mas ligeras y transitorias, que no pone en relación con
ningún factor desencadenante. Ex físico: obesidad, dolor a la palpación profunda bajo
reborde costal derecho, signo de Murphy negativo, ruidos intestinales disminuidos.
Analisis: AST (GTP)183 y amilasemia 390, Bilirub Total de 2.4, resto normal. Ecografia:
colelitiasis multiple sin signos de colecistitis, coledoco dilatado (14 mm de diámetro) hasta
su porción distal, donde hay una imagen de 5mm bien delimitada que deja sombra acústica.
Cual seria su actitud?
a.Tx conservador hasta que resuelva la pancreatitis
b. Laparotomía con colecistectomia y exploracion del coledoco
c. Colangiopancreatografia retrogada endoscopia (CPRE) con papilotomía para
tratar de extraer calculo y posterior colecistectomia programada.
d. Ac ursodesoxicalico a dosis de 300 8hrs VO
e. Litotricia biliar con ondas de choque

10. Mujer de 45 años con episodios de pancreatitis aguda documentada radiologicamente


con TAC. No ingiere alcohol, ni esta sometida a medicación alguna, las cifras de colesterol
y TG normales, en el ultimo ingreso, exceptuando la hipoamilasemia y una leve alteración
de la bioquímica hepática, el resto de los parámetros anilíticos fueron normales. La
ecografia abd como en las ocasiones previas normales. Cual de las que se expresan cree q
es la etiología mas probable de la pancreatitis aguda a esta paciente?
d. Microlitiasis Vesicular

11Cual de las siguientes afirmaciones respecto a la valoración de dolor abd agudo en el


anciano es cierta?
e. En los ancianos, la forma atípica de presentación de la enfermedad y el retraso en
el dx conducen a una mayor mortalidad que en los jóvenes.

12. La pauta habitual de 3 dosis de vacuna anti hepatitis B a los 0.1 y 6 meses asegura una
respuesta de anticuerpos protectores que se mide a través de:
a. Anti HBc
c. anti HBs
d. Anti HC
e Anti HA

13. Hombre de 45 años de edad diagnosticado de ulcera peptica duodenal a los 25 años y
que presenta regurgitaciones ocasionales se trara con Ranitidina. Coincidiendo con una
fase aguda se realiza endoscopia oral que demuestra una ulcera bulbar en cara posterior, de
1cm, de diámetro. La prueba de ureasa astral es +. Cual le parece el tx mas conveniente?
d. Metronidazol + Claritromicina+ Pantoprazol 10dias
14. La exploración inicial idónea en el dx de la hemorragia digestiva x hipertensión portal
es:
d. endoscopia alta

15.Cual de los siguientes no es un efecto secundario de los antiácidos?


d. Hiperfosfatemia

16. Cual de los siguientes afirmaciones es cierta?


d. La hepatitis A es de corto periodo de incubación y se transmite por via fecal oral

17. Solo una de las siguientes características es propia del síndrome de Budd Chiari:
c. su frecuencia es elevada en sujetos anticoagulados con dicumarinicos.

Parcial de Medicina Interna


Nefrología

1. Potencia los efectos de la hiperkalemia:


a. Hipocalcemia b. Hipercalcemia c. Alcalemia d. Hipernatremia
e. Hiponatremia

2. La mayor cantidad de magnesio se encuentra en:


a. Hueso b. Hígado c. Musculo d. Eritrocito e. Neurona

3. Hipokalemia con hipertensión arterial, aldosterona elevada y renina baja:


a. Tumor secretor de renina b. Hipertensión Acelerada c. Estenosis de la arteria
renal
d. Hiperaldosteronismo primario e. Síndrome de Uddle

4. En el tratamiento de la hipermagnesemia se puede utilizar:


a. Cloruro de Potasio b. Gluconato de calcio c. Solución Salina al 0.9% con
KCL
d. A y B e. B y C

5. La ausencia de proteínas y elementos celulares en la orina es más compatible con,


excepto:
a. Litiasis Renal unilateral b. Hiperplasia Prostática c. Deshidratación
hipotónica
d. Uropatía obstructiva bilateral e. Falla renal parenquimatosa
6. En la glomerulonefritis aguda post – infecciosa, la filtración glomerular disminuye
por:
a. Proteinuria masiva b. Alteración del coeficiente de ultrafiltración c.
Aumento de la presión oncótica d. Disminución de la presión oncótica
e. A y D

7. Hipermagnesemia de 10 mEq/L es de alto riesgo porque puede darse:


a. Abolición de los reflejos osteotendinosos b. Cuadriplejía flácida c. Vasodilatación
periférica d. Bloqueo cardiaco e. Depresión respiratoria

8. En el tratamiento general de la urgencia hipercalcémica se utiliza con prioridad:


a. D/A 5% B. Solución Salina 17,7% c. Solución salina 0.9% d. Bifosfonatos e.
Calcitonina
9. Paciente con uremia crónica, deshidratación severa puede presentar, excepto:
a. Oliguria b. Densidad Urinaria 1.010 c. osmolaridad urinaria de 800 mosm/kg d.
Hipercalcemia e. Hiperfosfatemia
10. En la hipokalemia puede darse, excepto:
a. Rabdomiolisis b. Una onda accesoria c. Intoxicación digitalica d. QT prolongado e.
Disminución por amonio génesis
11. Con un nivel de K de 10 mM/L debido a pseudohiperkalemia debe administrarse:
a. kayexalate b. solución hipertónica de glucosa c. bicarbonato de sodio d. gluconato de
calcio al 10% e. Ninguna de las anteriores.
12. Causas de falla renal aguda, excepto:
a. Medios de contraste b. Cilindros Hialinos c. Amino glucósidos d. Cadenas Kappa
e. Mioglobina
13. Es indicador para dializar:
a. Potasio Sérico de 6 mM/L b. Urea sérica de 50 mg/dl c. Náuseas y vómitos
incontrolables
d. Síntomas neurológicos e. C y D
14. cuando disminuye el calcio sérico:
a. disminuye la fosfaturia b. disminuye la resorción ósea c. se inhibe la parathormona
d. aumenta la absorción intestinal de fosfato e. Disminuye la absorción intestinal de
fosfato

15. Calcule el déficit de agua en una paciente de 70 kg con 50% de agua corporal y sodio
sérico de 160 mM/L:
a. 3 L b. 10 L c. 7L d. 5L E. 6L
16. enfermo de 60 kg y 60% de agua corporal, POsm 260 mOms/kg ¿Cuántos gramos
de Na Cl elevarían el sodio en 10 mM/L?
a. 21 b.30 c. 19 d. 26 e. 15
17. La hipocalemia:
a. Aumenta los ruidos intestinales
b. siempre produce cambios electrocardiográficos
c. disminuye la diferencia entre el potencial de membrana y el de umbral
d. aumenta la irritabilidad muscular
e. aumenta la magnitud del potencial de reposo

18. cuando disminuye el fosfato sérico:


a. Disminuye la PTH b. Disminuye la absorción de calcio intestinal c. Aumenta la
fostaturia d. Disminuye el calcitriol e. C y D
19. en causa de hipocalcemia, excepto:
a. Esprue tropical b. parcratitis crónica c. cirrosis biliar d. factor de necrosis tumoral
e. desnutrición
20.
28. paciente con diuresis de 12ml/h, potasio serico 7mM/l, osmolaridad 300 mOsm/l, el
diagnostico mas probable es:
a) SIADH
b) insuficiencia suprarrenal
c) insuficiencia renal
d) deshidratación hipertónica
e) todas las anteriores

29. paciente con vomitos abundantes con hipotensión arterial, pH 7.40; pCO2=41mmHg,
HCO3= 25mM/l; sodio 132mM/l; potasio 3.2m/l; cloro 86mM/l; nitrógeno de urea
40mg/dl:
a) No tiene trastorno acido/base

b) Alcalosis metabolicA severa

c) Acidosis metabolica con acidosis respiratoria

d) Acidosis metabolica con alcalosis metabolica

e) La brecha anionica esta elevada

30. en la hipokalemia se presenta:


a) Disminución de la sístole eléctrica

b) Menor diferencia entre el potencial de umbral y reposo

c) La onda P alta (la onda p esta aplanada)

d) El potencial de umbral menos negativa

31. el shock séptico puede presentar:


a) Alcalosis respiratoria con acidosis metabolica con brecha anionica normal

b) Alcalosis metabolica con alcalosis respiratoria

c) Alcalosis respiratoria con acidosis normocloremica

d) Acidosis respiratoria con acidosis metabolica con brecha anionica elevada

e) Ninguna de las anteriores


32. causas de hipercalcemia, excepto:
a) Uso de laxantes que contiene fosfatos

b) Enemas que contienen fosfatos

c) Administración de fosfato intravenoso

d) Falla renal crónica

e) Enfermedades granulomatosas

33. pH 7.20; HCO3= 8Mm/l; pCO2= 10mmHg:


a) Acidosis metabolica +alcalosis respiratoria

b) Acidosis mixya

c) Acidosis metabolica pura

d) Acidosis metabolica +acidosis respiratoria aguda

e) Acidosis metabolica + acidosis respiratoria crónica

34. paciente con sodio serico de 115mM/l, glucosa 90mg/dl, urea 300mg/dl. Podría
presentar:
a) Agitación

b) Letargia

c) Calambres musculares

d) Todas las anteriores

e) Ninguna de las anteriores

36. hallazgos de laboratorio de nefritis tubulo intersticial


a) Cilindros hemáticos

b) Hematuria

c) Proteinuria masiva (si se presenta es MUY leve)

d) A y b

e) Hipostenuria
37. en toda alcalosis metabolica se da lo siguiente:
a) pH urinario disminuido

b) fase de mantenimiento

c) brecha anionica normal

d) disminución serica de lactato

e) resistencia al cloruro

38. se utiliza para extraer el potasio del cuerpo:


a) bicarbonato de sodio

b) solución hipertónica de glucosa

c) solución hipotónica de glucosa

d) sulfonato de poliestireno

e) albuterol

39. alcalosis respiratoria


a) NH4+ y H2PO4 disminuidos en la orina

b) NH4+ disminuido y H2PO4 aumentado en la orina

c) NH4+ y H2PO4 aumentados en orina

d) NH4+ aumentado y H2PO4 disminuido en orina

e) pH urinario de 10

40. la mayor reabsorción de magnesio ocurre en:


a) la porción contorneada del tubulo proximal

b) la rama gruesa ascendente del asa de henle

c) la porción recta del tubulo proximal

d) el tubulo distal

e) el tubulo colector
41. causa de brecha anionica disminuida:
a) hipomagnesemia

b) intoxicación por litio

c) hipocalcemia

d) hipokalemia

e) intoxicación por metanol

42. causas de hipofosfatemia grave, EXCEPTO


a) falla renal

b) recuperación de cetoacidosis diabética

c) alcalosis respiratoria

d) quemaduras graves

e) nutrición parenteral total

43. acidosis respiratoria


a) disminución en la reabsorción proximal del bicarbonato

b) aumento en la reabsorción proximal del bicarbonato

c) disminución del bicarbonato en el eritrocito

d) disminución en la secreción de protones en el tubulo distal

e) ……………urinaria del NH4+

44. sugiere trastorno de tubulo intersticial


a) Perdida de la capacidad para concentrar la orina

b) Hipertensión arterial

c) Edema

d) Proteinuria mayor de 2g

e) Macrohematuria

45. alcalosis metabolica con hipertensión arterial e hipokalemia, EXCEPTO


a) Hipertension maligna

b) Deficiencia de 11 betahidroxilasa

c) Ingestión de biconice

d) Deficiencia de 17 betahidroxilasa

e) Síndrome de Gitelman

46. en una falla renal aguda espero encontrar:


a) Densidad urinaria 1010
b) Densidad urinaria 1020
c) Osmolaridad urinaria 100mOsm/kg
d) Osmolaridad urinaria 8000mOsm/kg
e) A y c

47. Aciduria paradójica se da en


a) Cetoacidosis diabética
b) Alcalosis metabolica leve
c) Acidosis tubular renal
d) Alcalosis metabolica con severa depleción de volumen
e) Acidosis láctica

48. una neoplasia maligna mas comúnmente asociada con hipercalcemia son,
EXCEPTO.
a) carcinoma de mama
b) Hemangioma
c) Carcinoma pulmonar
d) Leucemia
e) Linfoma

49. ph normal con bicarbonato leve:


a) Alcalosis metabolica + acidosis respiratoria
b) Alcalosis metabolica + alcalosis respiratoria
c) Acidosis metabolica + acidosis respiratoria
d) Alcalosis respiratoria + acidosis respiratoria
e) Ninguna de las anteriores

50. plasma: creatinina 1mg/dl; sodio 140mM/l; ORINA: creatinina


150mg/dl; sodio 40mM/l; la fracción de excreción del sodio es:
a) 2% 𝑠𝑜𝑑𝑖𝑜 𝑜𝑟𝑖𝑛𝑎 𝑐𝑟𝑒𝑎𝑡𝑖𝑛𝑖𝑛𝑎 𝑒𝑛 𝑝𝑙𝑎𝑠𝑚𝑎
+ x 100
𝑠𝑜𝑑𝑖𝑜 𝑝𝑙𝑎𝑠𝑚𝑎 𝑐𝑟𝑒𝑎𝑡𝑖𝑛𝑖𝑛𝑎 𝑒𝑛 𝑜𝑟𝑖𝑛𝑎
b) 0.5%
c) 0.19%
d) 0.3%
e) 4%

2. Favorece la penetración de amoniaco al cerebro situaciones de:


a) Acidosis metabólica
b) Alcalosis con Hipopotasemia
c) Acidosis con Hiperpotasemia
d) Alcalosis con hiperpotasemia
3. Paciente que pesa 50Kg, con sodio sérico de 160 mEq/dL , con ACT =40%
, su déficit de agua es:
a) 5L b) 3L c) 4L d) 6L

2. Para un calcio total sérico de 7mg/dl y albúmina de 2,2 g/dl ¿Cuál será
el calciototal
corregido? Tome el valor de la albúmina normal de 4 g/dl
a) 7,44 mg/dl b) 8,44 mg/dl c)9,44 mg/dl d) 8,90 mg/dl

3. Velocidad de administración de KCl en condiciones normales, en


mEq/hora
a) 8 b) 120 c) 20 d) 40

4. Causa falla prerrenal


a)Obstrucción de ambos uréteres b)Falla cardiaca c)
Glomérulonefritis aguda d) Síndrome nefrósico

5. Manifestación en el EKG de un paciente con Hipokalemia:


a) Onda T picuda b) Supradesnivel del segmento ST
c) Intervalo QT deprimido d) Onda S profunda
e) Prolongación del intervalo PR f) N/A

6. Cuál es la concentración sérica de fósforo por encima de la cual se


considera que una persona tiene Hiperfosfatemia (en mg/dL):
a) 3.0 b) 4.0 c) 5.0 d) 6.0 e) 7.0

7.. Diurético de uso común en el tratamiento del edema en los


síndromes
nefrítico y nefrótico:
a) Indapamida b) Espironolactona c) Furosemida d)
Acetazolamida
e) Hidroclorotiazida f) N/A

8. Cuál es el principal tratamiento para pacientes con nefritis


tubulointersticial
alérgica:
a) Prednisona b) Enalapril c) Ibuprofeno d) Diálisis e) N/A

9. Cuál de las siguientes enfermedades puede producir un Síndrome


Nefrítico
mediado por inmunocomplejos
a) Granulomatosis de Wegener b) Síndrome de Goodpasture
c) Púrpura Trombocitopénica Trombótica d) Amiloidosis e)
N/A

10. Ejemplo de hiponatremia hipotónica normovolémica


a)Diarrea B)Cirrosis hepática c) Síndrome nefrósico d) SIADH

11. Mencione cuál de las siguientes condiciones puede ser causa de


Hiperkalemia:
a) Uso de IECA b) Hiperaldosteronismo primario c)
Hiperinsulinemia d) Alcalosis Metabólica e) N/A

12. Mencione cuál de las siguientes condiciones cursa con el


desarrollo de Enfermedad Crónica de los Riñones:
a) Nefrolitiasis unilateral b) Enfermedad de Cambios Mínimos
c) Nefropatía Diabética d) Carcinoma de Riñón e) N/A

13. Oliguria implica una diuresis menor de (mL/día):


a) 50 b) 400 c) 800 d) 1000 e) N/A

14. Señale la aseveración Incorrecta acerca de la Hormona


Antidiurética:
a) Polipéptido sintetizado en el hipotálamo
b) También se le conoce como Arginina Vasopresina
c) El efecto neto de su acción a nivel de la nefrona es la
reabsorción pasiva de agua y sodio
d) El mayor estímulo para su secreción es la hipertonicidad
15. Generalmente la proporción BUN / creatinina plasmática en
hiperaazoemia prerrenal es de:
a) <10 b) > 20 c) 100% d) d) 1%

16. El sodio urinario en hiperazoemia renal intrínseca es de:


a) <10 b) >20 c) > 30 d) >1

18. La causa principal del síndrome nefrítico agudo es :


a) Hipertensión arterial b) GN postestreptocóccica c) Síndrome
deGoodpasture
d) Nefritis lúpica

18. Cuál es el componente esencial en el síndrome nefrótico?


a) hipoalbuminemia b) hiperlipidemia c) proteinuria d)
edema

19. Cuadro clínico clásico del síndrome nefrítico incluye lo siguiente


excepto:
a) lipiduria b)hipertensión c) Oliguria d) Hematuria

20. El tx en cuarto de urgencias de hipercalcemia intensa es:


a) Bifosfonatos b) Furosemida y SS isotónica c) Furosemida y
Bifosfonatos
d) SSisotónica +Calcitriol.

1. Causa de hipomagnesemia:
a. estreñimiento
b. alcoholismo
c. falla pre-renal
d. IRA (insuficiencia renal aguda)

2. La hipocalcemia en la falla renal aguda puede ser asintomática por la:


a. acidemia
b. hipoalbuminemia
c. Deficiencia de PTH
d. Alcalemia

3. La hipofosfatemia causa, excepto:


a. Aumento de vit. D
b. Aumento de PTH
c. Disminución de la excresión de fosfato
d. Aumento de la absorción de fosfato a nivel intestinal
4. Un pte con diuresis de 250 ml al día con osmolaridad plasmática (presión osmótia) y
osmolaridad urinaria de 300 mosm/l, el dx probable e:
a. Insuficiencia suprarrenal
b. Insuficiencia renal
c. Nefritis túbulo intersticial crónica
d. Deshidratación hipertónica

5. Pte con oliguria, hiperkalemia, hiponatremia, creatinina 0.8 mg/dl y CPK normal: Esto
puede corresponder a:
a. IRA
b. Insuficiencia suprarrenal
c. Síndrome hepatorrenal
d. Rabdomiolisis

6. No está indicado en caso de hiperkalemia:


r. Aldactone

7. Mujer con hiperemesis grávida:


r. Alcalosis hipoclórica

8. En causa de hipomagnesemia:
r. Alcoholismo.

9. En el metabolismo del calcio está implicado, menos:


r. Insulina
10. Hiponatremia, hipotónico, normovolémico:
r. SIADH
11. Fórmula del clearence de creatinina:
r. clearence = 140 – edad x peso en Kg
72 x creatinina

12. Fórmula de irritabilidad muscular:


r. IR = Na + K + OH
( ) + Mg + H
13. Indica falla renal aguda:
a. U.N. = 4.1
b.
U.N. = 1
c.
U.N. = ¼
d.
Gravedad específica: 1.026
e.
Ninguna.

14. Deshidratación por pérdida de agua y densidad urinaria 1.003, creatinina 6:


a. falla renal aguda parenquimatosa
b.
Falla pre-renal
c.
Falla renal obstructiva
d.
Falla renal crónica
e.
No hay falla renal.

15. Uno de los índices de falla renal es:


a. Reabsorción de Na/ oferta tubular de Na x 100
b.
Excreción urinaria de Na/ oferta tubular de Na x 100
c.
Oferta tubular de Na/ excresión urinaria de Na x 100 **
d.
Una / pna x 100
e.
Ninguna

16. El síndrome hepatorrenal es causa de falla:


a. pre-renal
b.
renal
c.
posrenal
d.
falla cardiaca
e.
daño tubular

17. Causa más comun de falla renal cronica:


a. Diabetes melitus
b.
HTA
c.
Glomerulonefritis
d.
Enfermedad poliquística renal
e.
Vasculitis

18. En la glomerulnefritis aguda por infección, la causa, excepto:


a. proteínas mayor de 10g
b. osmolaridad urinaria normal
c. hematuria
d. HTA
e. Edema

18. A pesar de la diversidad de enfermedades indicadas abajo, ellas tienen muchos


síntomas en común. Con frecuencia, los síntomas y signos observados incluyen:
disminución en el volumen de orina, proteína en la orina (proteinuria), sangre en la
orina microscópica o macrscópica (hematuria), hinchazón (edema), presión sanguínea
alta (HTA) y una disminución en la capacidad del riñón de extraer eficazmente los
residuos.

19. Falla renal crónica, excepto:


a. PTH aumentada
b. Hipofosfatemia
c. Acidemia
d. Hipocalcemia
e. 1.25 OH (D3) disminuída.

20. En la falla renal aguda siempre habrá:


a. aumento de creatinina
b. fracción de excresión de Na aumentado
c. Hiperkalemia
d. Una aumentado
e. Hipernatremia

21. Causa de falla pre-renal:


a. Sd. Nefrótico
b. Obstrucción de ambos uréteres
c. Falla cardiaca
d. Glomerulonefritis aguda pos- infecciosa
e. Calculo renal
22. En la nefritis tubular intersticial se puede dar, excepto:
a. hipernatremia
b. proteinuria de 1.5 g
c. normotension arterial
d. hipostenuria
e. hipocalcemia

23. Sugiere glomerulonefritis


a. leucocituria
b. cilindros eritrocitarios
c. anuria
d. ……

24. Causas de falla renal parenquimatosa:


a. papilitis
b. calculo en pelvis derecha
c. estenosis unilateral de arteria renal
d. cistitis hemorragica
e. ………

25. Causa de anuria:


a. Enfermedad de cambios mínimos
b. Necrosis cortical
c. Pielonefritis aguda leve bilateral
d. Necrosis tubular aguda
e. Calculo coloriforme bilateral

26. La hipofosfatemia causa todo, excepto:


r. Aumento de vit. D

27. En pte con falla renal crónica que ha sido transplantado esperamos encontrar:
r. osteomalacia.

1) Cuál de las siguientes medicinas pueden dar hiperkalemia


a) inhibidor de la enzima convertidota
b) bloqueador alfa adrenérgico
c) bloqueador beta adrenérgico
d) ayb
e) ayc

28. Hormonas que disminuyen en la falla renal crónica, excepto


a) 1,25 dihidroxi vitatamina D
b) eritropoyetina
c) paratohormona
d) foliculo estimulante

29. MgSO4 (PM 120) 1 gramo contiene de magnesio en meq


a) 10
b) 7.9
c) 8.3
d) 6

30. Factores que conducen a la progresión de falla renal crónica


a) hiperlipidemia
b) toxinas urémicas
c) proteinuria
d) b y c
c) todas

31. Paciente de 20 años presenta abundantes vômitos y diarrea de 3 días de evolución,


examen físico depleción de volumen, suero muestra Na 155, K 3meq/l, Cl 117meq/l, HCO3 25
meq/l, tratamiento de elección:
a) D/A 5% + KCl
b) solución salina 3% + KCl
c) solución salina 23.4% + KCl
d) solución salina 0.45% + KCl

32. En la enfermedad ateroembólica renal se da lo siguiente excepto:


d) Velocidad de eritrosedimentación

33. Pseudohiperkalemia de 8 mEq/l que se ve en el EKG:


Onda T picuda
Aplanamiento de la onda P
Ay B
Ninguna

34. Criterios para iniciar diálisis excepto:


Alcalosiss Metabolica

35. Hipokalemia con normotension excepto


Síndrome de liddle

36. Plasama creatinina 8mg/dl N de urea 100mg/dl U na 40 Meq/l :


Insuficiência renal

37. Factores reversibles responsables de deterioro de la funcion renal excepto:


Fibrosis intersticial

38. Causa hipokalemia con renina y aldosterona disminuida:


Síndrome de Liddle

39. Sugiere trastorno túbulo intersticial:


.Proteinuria mayor a 3.5 g /24 horas
.Notrmotensión
.* Edema
.Osmolarida urinaria normal

40. Glomérulonefritis rápidamente progresiva


.Engrosamiento de la membrana basal
Fusión de los pedículos de los podocitos
prolñiferacion intracapilar
proliferación extracapilar

41. Una de las siguientes causas no produce hematuria


.enfermedad de cambios mínimos
42. Plasma Na:115, Posm240 mOsm,Uosm 680,Una 60:meq/l:
.Deshidratación hipotónica
.* secreción inapropiada de ADH
.Deshidratación isotónica
.Deshidratación primaria

43. .en la falla renal crónica produce necrosis


calcifilaxia

44. No produce hipokalemia:


Pancreatitis aguda
rabdomiolisis
neoplasia
Def. Vit D

44. No produce hipocalcemia:


Pancreatitis aguda
rabdomiolisis
neoplasia
Def. Vit D

45. Paciente con glucosa 90mg/dl, Na+= 115mEq/L, Urea en 140mg/dl


a. Síndrome hipoosmolar
b. * Síndrome hiperosmolar
c. Diarrea
d. Ninguna de las anteriores.

46. Causa de seudohiponatremia con osmolaridad elevada


a. Hiperproteinemia
b. * Diabetes mellitus
c. Hiperlipidemidemia
d. Reserción transuretral prostática.

47. Manifestaciones de hipermagnesemia excepto


Normotensión

48. Piel en uremia crónica excepto


Paño blanco

49. No es causa de necrosis papilar aguda


a.Diabetes mellitas
b. Uropatía obstructiva
c. Pielonefritis
d. Trombosis de venas renal.

50. Puede presentarse hiperuricemia excepto


d. Adenocarcinoma gástrico

51. Compromete riñones y pulmones:


a. Enf de GoodPasture
b. Granulomatosis de Wegener
c. LES
d. Ay C
TODAS

52. Sugiere glomerulopatia:


-anasarca

53. TFG < 15ml/min corresponde al estadio de la enfermedad renal cronica:


* 5

54. Causa obstetrica de necrosis cortical bilateral


- desprendimiento prematuro de placenta

55. Promueve amoniogenesis en cirróticos


- hipokalemia

56. Criterio para diagnostico de síndrome nefrosico


a. Proteinuria en 24 hr de 3.5 gr para superficie corporal de 1. 73
b. Albumina serica < 3g/dl
c.Hiperlipidemia y edema
d. Todas las anteriores
e. ay b

57. Urémico crónico con potasio sérico de 6.5mEq/L con leve academia, sin trastorno muscular
y EKG normal, puede ser tratado con:
a. kayaxelate
b. gluconato de Calcio al 10%
c. NaHCO3
d. Diálisis

58. La insuficiencia renal crónica ocasiona hiperfosfatemia permanentemente cuando la tasa


de filtración glomerular esta por debajo de:
a. 60ml/min
b. 50 ml/min
c. 25 ml/min
d. 40 ml/min

59. La hipofosfatemia es causa de excepto


a. inotropismo positivo
b. disminución de 2,3 DPG
c. hemólisis
d. curva de HbO2 desplazada hacia la izquierda

60.La solución de NaCl 23,4% tiene de osmolaridad:


a. 8000 mosm/l
b. 800 mosm/l
c. 80 mosm/l
d. 0,8 mosm/l

61. En la hiperkalemia:
a. aumenta la sístole eléctrica
b. hay menor diferencia entre el potencial de reposo y el umbral
c. el potencial de reposo transmembrana se hace mas negativo
d. el potencial umbral se hace menos negativo
62. La velocidad de administración de potasio en caso de gran riesgo para la vida puede ser
de:
a. 10-20 mEq/L
b. 8 mEq/L
c. 110 mEq/L
d. 40 mEq/L

63. Un ejemplo de hiponatremia hipotónica normovolemica es:


a. síndrome nefrósico
b. SIADH
c. Diarrea
d. Cirrosis

64. A una embarazada con eclampsia se le ordena 1 litro de D/A 5% con 10g de MgSO4.
¿Cuántos ml de MgSO4 al 50% se han utilizado?
a. 20
b. 5
c. 12,5
d. 30

65. Causa hipomagnesemia:


a. estreñimiento
b. alcoholismo
c. deshidratación hipertónica
d. insuficiencia renal aguda

66. Paciente con diuresis de 300ml/día, K sérico 7mEq/L, Posm y Uosm 300mosm/L, el
diagnóstico más probable es:
a. insuficiencia suprarrenal
b. insuficiencia renal
c. SIADH
d. Deshidratación primaria

67. La mielinosis pontina se caracteriza por:


a. pares craneales intactos
b. cuadriplejía flácida
c. comportamiento normal
d. hipertonía de los miembros inferiores

68. Calcule el déficit de agua en una paciente con sodio sérico 168mEq/L y que pesa 70Kg
(utilice como porcentaje de agua 50% y sodio normal mEq/L):
a. 5L
b. 3L
c. 10L
d. 7L

69. En la SIADH la concentración de sodio urinario es de:


a. menos de 10mEq/L
b. 20mEq/L
c. 15mEq/L
d. Mas de 40mEq/L
70. En paciente con vómitos abundantes y prolongados con hipokalemia severa debe
administrarse:
a. NaCl 0.9% con KHCO3
b. NaCl 0.9% con KCl
c. NaCl 0.9% con gluconato de potasio
d. NaCl 0.9% con NaHCO3

71. La hipokalemia en un paciente con cirrosis puede contribuir:


a. coma
b. alcalosis respiratoria
c. acidosis metabolica
d. aumento de la síntesis de urea

72. El plasma de un paciente contiene Na 125mEq/L, glucosa 108mg/dl, urea 300mg/dl, podrá
presentar:
a. cirrosis
b. síndrome nefrósico
c. síntomas de hipertonicidad
d. síntomas de hipotonicidad

73. La hiponatremia aguda sintomática es debida a, excepto:


a. exceso de glucocorticoides
b. SIADH
c. post-operatorio
d. intoxicación hídrica

74. La administración de solución salina 0.9% produce:


a. disminución del volumen intracelular y aumento del extracelular
b. aumento del volumen intra y extracelular
c. aumento del volumen extracelular
d. aumento del volumen intracelular

75. Alcalosis metabólica con hipertensión arterial, hipokalemia, aldosterona normal y renina
normal:
a. Hipertensión renovascular
b. Hipertensión maligna
c. Sindroma de Cushing
d. Administración de licor

76. La hipokalemia puede predisponer, excepto:


a. intoxicación digitálica
b. disminución de la amoniogénesis
c. rabdomiolisis
d. ondas U en el EKG

77. Aldosterona aumentada, Renina dism. → Hiperaldosteronismo I

78. Limites de Hipo e Hiperkalemia

79. Px con IRC tine una taza de filtración glomerular de menor de 25 ml/min ;aunque hay yuna
tabla de las etapas en esta enfemedad crónica.
Estas osn las TFG:
1. 90 en riesgo
2. 20 ( con TFG normal o incrementada)
3. 60 -89
4. 30 – 59
5. 15-29
Menor a 15 diálisis.

80. Factor que altera el balance hídrico en el periódo post- operatorio:


Disminución de la ADH (Excepto)

81. Un ejemplo de hipokalemia con aldosterona alta y osmolaridad baja es:


Hiperaldosteronismo primario

82.Ejemplode hipernatremia hipervolémica: Hiperaldosteronismo primario

83.Hiperkalemioa son alteraciones del EKG no es necesario administrar gluconato de calcio se


colocará un antagonista de K asíe l Px no cae en paro y estoes si tiene manifestación en el EKG.

84. La fracción de excreción de sodio estará disminuida en falla cardíaca.

85.En la hiperkalemia: hay menos diferencia entre el potencial de reposo y el umbral.

Manifestaciones cardíacas. El problema médico más grave de la hiperkalemia es la


cardiotoxicidad. Los cambios en el ECG producidos por los niveles altos de potasio son bastante
constantes. A medida que aumentan los niveles se aprecian los siguientes cambios:

Ondas T picudas (con intervalo QT normal o ligeramente reducido)


Prolongación del intervalo PR con depresión de ST
Desaparición progresiva de la onda P
Bloqueo cardíaco progresivo
Arritmias ventriculares
Paro cardíaco

Las ondas T picudas constituyen el dato en el ECG más constante en la hiperkalemia.

Efectos neuromusculares. El primer signo neuromuscular de la hiperkalemia suele ser la


aparición de parestesias seguidas de debilidad progresiva de varios grupos musculares. Si el
cuadro se agrava se observa cuadriplejia fláccida. Las funciones cerebrales y de los pares
craneanos se conservan y la parálisis de la musculatura respiratoria puede ocurrir, pero es
excepcional.

86. La infusión de potasio sin riesgo para la vida del PX: es de 40 meq; porque

I lb de solución salina: debe llevr de 20 a40 mEq de KCl a una velocidad de 10 a 20 mEq por
hora; si aumenta la velocidad puede provocar un bloqueo A-V. Se recomienda una Velocidad
mayor si la hipokalemia es muy severa.

Sis e usa venas priféricas es:

10 meq en 100 cc de sol.salina para pasar en una hora

20 meq en 200 cc de sol.salina para pasar en una hora


En vena central:

20 meql en 50 cc pasarla en una hora

40 meq en 100cc pasar la en una hora

31.Los indicios de falla renal aguda son:

(Na filtrado/Na excretado) x 100%

87.Na urinario menor de 10: falla cardíaca, presentará edema estará hinchado y lo puede
llevar a una I.C.C. ( Esto lo saque de Medline: Los valores normales generalmente son de 15 a 250
mEq/L/día, dependiendo del estado de deshidratación y la ingesta diaria de sodio en la dieta.
Los rangos de los valores normales pueden variar ligeramente entre diferentes laboratorios.
Nota: mEq/L/día = miliequivalentes por litro por día.)

88.Causa de enuresis: necrosis cortical difusa.

89.En SIDH la concentración urinaria es menor de 20.

90. Deshidratación pura de agua excepto: hay salida de electrolitos del cerebro.

91.Urémico crónico con deshidratación hipertónica severa y acidótico presentará lo siguiento


excepto: osmolaridad urinaria de 8mm mosmol/kg.

92. Px con vómitos abundante, hipertensión arterial; Na urinario=6, oliguria, osmolaridad de 800
presenta: insuficiencia suprarrenal.

93. Factor que predispone a la hipernatremia son intoxicación hídrica.

94. Se presenta en acidosis tubular renal proximal excepto: NaHCO3 y lo alfa antagonistas.

95. La hipokalemia produce:


- mayor negatividad del potencial de reposo.
96. La hipokalemia en un cirrótico puede precipitar COMA por:
- aumento de la amniogénesis
97. Exceso del kayaxelate en el tratamiento de hiperkalemia puede producir:
- constipación.

98. Peligro de la hipermagnesemia severa:


- paro respiratorio.
99. Hipofosfatemia:
- aumenta el calcitriol.
100. Consecuencia metabólica de hipofosfatemia severa:
- miocardiopatía
101. Causa de hiperfosfatemia:
- insuficiencia renal
102. El Síndrome Nefrósicose caracteriza por:
- proteinuria mayor o igual de 3g y albúmina sérica menor o igual de 3g/dl.
103. En el Síndrome Nefrósico pueden encontrarse:
- cilindros céreos
104. Urémico agudo, oligúrico, acidótico, deshidratado puede presentar:
- osmolaridad urinaria de 250mosm/kg
105. La falla renal crónica avanzada cursa con:
brecha aniónica elevada
106. Cuál de los siguientes antibióticos produce nefritis intersticial:
- meticilina
107. El factor más determinante en el tratamiento de la hipokalemia renal es:
- la función cardiovascular.
108. El tratamiento de inicio de secreción inapropiada de ADH es:
- restricción hídrica
109.Causa de hiperfosfatemia:
e. Falla hepatica
f. Falla renal cronica
g. Falla cardiaca
h. Insuficiencia suprarrenal

110. Causas de hiperfosfatemia grave, excepto:


d. recuperacion de cetoacidosis diabetica.
e. Alcalosis respiratoria
f. Acidosis respiratoria.

111.Paciente con vómitos severos, oligurico con hipotensión arterial Una> 40…… 800 mOsm/Kg
puede tener:
e. Insuciencia renal aguda.
f. Insuficiencia suprarrenal
g. Falla pre-renal
h. Exceso de mineralocorticoides

112. Paciente urémico crónico con deshidratación severa puede presentar lo siguiente excepto:
a. Densidad urinaria 1010.
b. Osmolaridad urinaria de 800 mOsm/Kg
c. K sérico aumentado
d. Oliguria.

113.La hipokalemia produce:


a. Aumento de la magnitud del potencial de reposo

114. Primera*****Falla renal crónica:


a.HTA
b.Glomerulopatias
c.Diabetes Mellitus
d.Nefritis tubulo intersticial

50. Uno de los indices de falla renal aguda es:


Uosm/Tosm

51. Hormona producida en los riñones que disminuye en la falla renal cronica:
a. ADH
b. Noradrenalina
c. Peptido natriuretico (segun arjona)
d. 1,25 (OH)2 Vit D.
52. Las calcificaciones metastásicas se deben:??
a. Aumento del producto Ca, P
b. Aumento del producto Ca, Mg
c. Hipoparatiroidismo
d. Hiperparatiroidismo 1ario

53. Causa de anuria:


a. Pielonefritis unilateral
b. Necrosis tubular aguda
c. Enf cambios minimo
d. Necrosis cortical

54. la etapa de la insuficiencia renal cronica con tasa de filtración glomerular de 60-89
ml/min es la:??
a. 1
b. 2
c. 3
d. 4

55. La FENA es la relación entre:


a. Oferta tubular/Na reabsorbido x 100
b. depuración de Na/ depuración de creatinina x 100
c. oferta tubular/ Na excretado x 100.
d. Na excretado/ Na reabsorvido x 100.

56. En la nefritis tubular intersticial aguda se pierde la capacidad de poder concentrar la


orina debida a:??
a. Daño medular
b. Daño glomerular
c. Daño tubular proximal
d. Daño de la porcion cortical del tubulo colector

57. La solucion de NaCl al 23,4 % tiene:


8000 mOsm/l

58. Se asocia con el SIAD, excepto:


Lipoma

59. El efecto biologico mas importante de la calcitonina es, excepto:


a. Responder a la hipocalcemia
b. Aumenta la calciuria
c. Estimular la resorcion osea osteoclastica
d. Utilizada en el tx de la hipercalcemia.
60. Favorece la entrada de K al espacio intracelular, excepto:
a. Agonista adrenergico
b. Estimulacion adrenergico
c. Insulina
d. Alcalosis metabolica

61. Factores que afectan el balance hídrico en el post-operatorio, excepto:


a. Fármacos
b. Calcitriol
c. Administración de liquidos hipotonico
d. Aumento de la ADH

62. Las primeras manifestaciones clínicas de la hiponatremia son:??


a. neurológica
b. gastrointestinal
c. pulmonares
d. musculares

63. En la hipernatremia hipertonica se da lo siguiente excepto:


a. Entrada de Na
b. Salida de aa
c. Ganancia de iones
d. Disminución del agua cerebral

64. Manifestaciones clinicas de la hipercalcemia, excepto:


a. Taquicardia
b. HTA
c. Constipación
d. Vomitos

65. En falla renal aguda por nefrotoxicidad:


a. Se conserva la membrana basal
b. Se destruye la membrana basal
c. Frecuente hay oliguria
d. La creatinina no aumenta

66. Causa de hipomagnesemia:


alcoholismo

67. Na serico de 168 mEq/l 70 kg (utilize como % de agua al 50% y Na normal 140 mEq/l
7 litros

68. Indicaciones de diálisis en falla renal aguda, excepto:


a. Encefalopatia uremica
b. Hiperkalemia refractaria
c. Ac. Metabolica refractaria
d. hipocalcemia

69. El KCL IV se puede asministrar asi, excepto:


a.infusion por linea periferica de 10 meq en 100 ml en 1hora
b. “ “ central de 20 meq em 50ml en 1 hora
c. “ “ central de 100 meq em 50ml en 1 hora
d. “ “ periferica de 20 meq en 20ml en 2 horas

70. Aumenta la excrecion urinaria de K, excepto:


a. Triamtereno
b. Bumetanida
c. Furosemida
d. Metolazona
71. Factores que regulan la secrecion distal de K, excepto:
a. Flujo tubular distal y aporte distal de Na
b. Excrecion de aniones no reabsorbibles
c. Aldosterona
d. PTH

72. Enfermo de 60 Kg con 60% de agua corporal, Posm 260 mOsm/kg, ¿Cuántos gramos
de NaCl aumentaria el Na en 10 mEq?
a. 21 gramos

17. La PTH promueve todo lo siguiente excepto:


a aumento de la absorción intestinal de calcio
b- aumento de la absorción tubulár de fosfato.
c. aumento de la resorción ósea de fosfato
d- aumento de la resorción osea de calcio
e, disminución de la reabsorción de fosfato

18. Un varon de 57 esta con hemodiálisis de mantenimiento para insuficiencia renal cronica, ¿Cuál
de las sig anomalías metabolicas podría ser anticipáda?
a. hipernatremia
b. hiponatremia
c.
d. exceso de vitamina D
e. hipoparatiroidismo

19. Un px de 25años fue ingresado a UCI por lesiones graves en cabeza y fractura de la base del
Graneo, aprox X horas después de la lesion manifestaba poliuria, la OSMU 150, y la de suero 350.
Los líquidos IV fueron detenidos y 3 hors después la producción de orina y la osm urinaria
permanecieron sin cambio. Se administraron 5 unidades de vasopresina IV, la osm de la orina se
incremento a 300. ¿Cuál es el dx mas probable?
a- Diabetes insípida central
b. diabetes insípida nefrogenica
c. intoxicación acuosa
d. sobrecarga de solutos
e. S IADH

55. indica falla renal aguda


a- U..N.. = 4.1
b- UN = 1
c. U..N.. = 1/4i/
d. Gravedad especifica 1.026
e. Ninguna

56. deshidratación por perdida de agua y densidad urinaria 1.003, creatinina 6 /


a. falla renal aguda parenquimatosa
b- falla prerenal
c. falla renal obstructiva
d. falla renal cronica
e. no hay falla renal

57. uno de los indices de falla renales


a. reabsorción de Na/oferta tubular de Na x 100
b. excresion urinaria deNa/oferta tubular de x 100
c.- oferta tubular de Na/excrecion urinaria de Na x 100 ~.--~
d- Una/Pna x 100
e. Ninguna

58. el Sdr hepatorenal es causa de falla:


a- prerrenal
b. renal
c. póstrenal
d. falla cardiaca
e. daño tubular

59. causa mas común de falla renal crónica:


A- DM

60- en la glomendonefritis aguda por infeccion, la causa, excepto


a- proteina mayor de 10 g
b- osmolaridad urinaria normal
c. hematuria
d. HT A
e. Edema

61. falla renal cronica, excepto


a. PTH aumentado
b- Hipofosfatemia
c- Acidemia
d. Hipocalcemia
e. 1.25 OH (D3) disminuida

62. EN la falla renal aguda siempre habra


a- Aumento de creatinina
b- fraccion de excrecion de Na aumentado
c- hiperkalemia
d- Una aumentado
e- Hipernatremia

63. causa de falla prerenal


a. Sdr nefrotico
b- obstrucción de ambos uréteres
c- falla cardiaca
d. Grlomerulonefritis aguda post-infeciosa
e. Calculo renal

64. en la nefritis tubular interticial se puede dar, excepto


a. hipernatremia
b. proteinuria de 1.5 g
c. normotension arterial
d. hipostenuria
e. hipocalcemia

65. Sugiere glomerulonefritis:


a. leucocituia
b-cilindros eritrocitarios
c- anuria
d. -

66 -causa de falla renal parenquimatosa


a) papilitis
b) calculo en pelvis derecha
c) estenosis unilateral de arteria renal
d) cistitis hemorrágica

60- Causa de hiperkalemia:


a Aumento del volumen arterial eficaz
b. Hipercatabolismo tisular
c Hiperaldosteronismo
d. Alcalosis Inetabolica
e. Cirrosis hepatica

61. Causa de hipomagnesemia


a. cirrosis avanzada
b. falla renal cronica
c. falla cardiaca
d. constipacion
e. acidosis respiratoria

62. Causa renal, de hipomagnesemia


a. falla renal crônica
b. esteatorreaa
c desnutricion severa
d. usó de diureticos de asa
e. infeccion urinaria

63. en caso de hipercalcemia pensar en:


a. neoplasia
b: hiperfosfatemia severa
c. nefrolitiasis
d. nefrocalcinosis
e. diuréticos

64. En el síndrome nefrosico puede encontrarse cilindros


a. cereos
b. hematicos
c. leucocitarios
d. epiteliales
e. mixtos leucocitario y hematicos,

2--En el riñón la hormona paratiroides:


Aumenta la reabsorción tubular de calcio y disminuye la de fósforo

3- La hipercalcemia produce
Potencialización de los efectos de la hipercalemia

36- La hipomagnesemia produce.


Vasodilatación periférica
36-De los factores: nefrotoxicidad./ -La buena hidratación

37 - En la concentración sérica para calcular:: 8,8

38- la velocidad de administración de potasio: 40 meq

39-hipocalcemia produce : fosfaturia


18- La hipofosfatemia cansa excepto: aumento de vitamina D
20. La renal aguda, oligurica todo es cierto excepto: Osmolaridad urinaria aumentada
21 Paciente can alcalosis metabolica e hipokalemia debe ser tratada con KCL

24. Usted tiene una paciente con hiperhemesis gravidica. El trastorno metabolico que espera
encontrar es:
A - acidosis hipocloremica
b, acidosis hipercloremica'
c- alcalosis hipocloremica
d. alcalosis hipercloremica
e. no hay alteración acido base

25. Cual de las siguientes opciones esta contraindicada en el tratamiento de la hiperkalcmia


a, gluconato de calcio
b. aldactone
c, kayaxelate
d. Bicarbonato de sodio e. Todas las anteriores

43. Rotando en sala de medicina interna le piden que evalue un paciente con HIV el cual
porpresentar una meningitis por criptococci recibe anfotericina B; dicho este medicamento
buscaría como complicación:
a. aparicion de diarrea
b. aumento de la presion arterial
c- hipokalemia
d . cefalea
e. dolor toráxico

81- donde se halla el calcio en el organismo:


a- 50% sangre
b- 20 % proteínas
c- 99% hueso

2- cual de los siguientes datos refleja vieja gravedad


a- distension abdominal
b- hipocalcemia
c- hiperamilasemia
d- aumento de la intensidad de dolor

4- en la hipercalemia
a- hay una menor diferencia entre el potencial de reposo y el umbral

5—la hipercalemia en un cirrótico puede desencadenar


a- coma hepatico

6- la hipermagnesemia puede producir


a- paro respiratorio
7- tx de hipercalcemia
a- SSN 0.9 %

8- la hipofosfatemia son causas excepto:


a- aumento de PTH
b- aumento de la absorción intestinal de fosfato
c- disminuye fosfatemia
d- aumento de vitamina D

9- una solucion isotonica


a- aumenta el volumen extracell

10- la vel de administración de potasio en caso de riesgo para la vida


a—40 meq por hora

11- Px cronico con deshidratación severa puede presentar los siguiente excepto:
a-osmolaridad urinaria de 800 mosmol/Kg

12- Px con vomito abundante, HTA, sodio urinario en 6, oligurico, Osmolaridad de 800
presente,
a- insuficiencia suprarrenal

13- Px uremico cronico con vomito severo hipotension arterial orina al azar, muestra
sodio mas de 40, Osmolaridad de 800 puede tener :
a- Falla suprarrenal

14- En quien predomina la enfermedad de higado graso:


A- Sexo masculino

15- La hipocalemia predispone excepto:


a- amoniogenesis

16- Cual de las siguientes es causa de hiperfosfatemia


a- insuficiencia renal

17- en la glomerulonefritis aguda post infecciosa disminuye la tasa de filtración:


a- por alteración en el coeficiente de ultrafiltracion

18- causa de insuficiencia prerenal excpto:.


a- Aneurisma disecante de la aorta
b- Cuándo disminuye el fosfato?
R. Aumento de PTH

c- Ejemplo de hipernatremia hipervolémica:


R. Aldosteronismo primario

d- La hipocalemia produce:
R. Aumento de la magnitud del potencial de reposo

e- La depleción de agua produce, excepto:


R. Salida de electrolitos del cerebro
f- Velocidad de administración de KCl en caso de urgencia:
R. 40 mEq/h

g- Cambios en EKG de la hipermagnesemia es igual:


R. Hipercalemia

h- Enfermo de 60 Kg con 60% de agua corporal, osmolaridad plasmática 260


mosmol/Kg, ¿cuántos gramos de NaCl aumenta el Na en 10 mEq en 24 horas?
R. a) 15
b) 21
c) 30
d) 19

i- Uso de Calcio en tratamiento de hipercalemia para:


R. Modificar el Potencial de Membrana

j- Hipermagnesemia severa produce:


R. Paro respiratorio

k- Venoclisis de 1L de D/A 5% conteniendo 10 mg de MgSO4 y al 50%:


R. 20mL

l- La hipocalcemia:
R. Prolongo QT a expensas del segmento ST

m- Tratamiento de urgencia de hipercalcemia:


R. SSN 0.9%

n- Causa de hiperfosfatemia:
R. Insuficiencia renal

o- Adaptación cerebral, hipertonicidad, excepto:


R. Pérdida de iones

p- En la adaptación cerebral, la hipotensión da lo siguiente, excepto:


R. Ganancia de aminoácidos
q- En ausencia de síntomas de hipernatremia, el sodio debe ser disminuido a razón de:
R. 0.5 mEq/L/h

r- Para corregir un sodio cerca de 185 mEq se administró en 86 horas, 66 letras de D/SS
5%. ¿cuál es la velocidad de administración de la venoclisis?
R. 69 mL ¿?????

s- Manifestación clínica de hipercalcemia, excepto:


R. a) Hipotensión arterial
b) Bradicardia
c) Nauseas
d) Anorexia

t- Consecuencia de hipofosfatemia, excepto:


R. a) No afecta al SNC
b) Disfunción plaquetaria
c) Cardiomiopatía
d) Osteomalacia

u- Causa de hipofosfatemia, excepto:


R. a) Falla renal
b) Recuperación de cetoacidosis diabética
c) Alcalosis respiratoria
d) Quemaduras graves

v- Causa de HTA con hipocalemia, aldosterona-renina baja:


R. Síndrome de Cushing

w- Causas de HTA con hipocalemia y renina alta:


R. Hipertensión maligna

x- En una deshidratación severa hipertónica con hipotensión arterial severa, la prioridad


inicial es administrar:
R. SSN 0.9%

y- En la secreción inapropiada de ADH se da lo siguiente, excepto:


R. No hay expansión de volumen

z- Tejido con mayor contenido de K+:


R. a) Hígado
b) Músculo
c) Eritrocito
d) Hueso
La PTH promueve todo lo siguiente excepto:
Aumento de la absorción tubular de fosfato

8 Varón de 57a esta con hemodiálisis de mantenimiento para insuficiencia renal


crónica. Cual de las siguientes anormalidades metabólicas podría ser anticipada?
Osteomalacia
50 S Nefrosico:
Cilindros cereos

51 Falla renal crónica avanzada:


Brecha aniónica elevada
24. Usted tiene una paciente con hiperemesis gravidica. El trastorno metabolico que espera
encontrar es:
a. acidosis hipocloremica
b. acidosis hipercloremica
c. alcalosis hipocloremica
d. alcalosis hipercloremica
e. no hay alteración acido base

25. Cual de las siguientes opciones esta contraindicada en el tratamiento de la hiperkalemia


a. gluconato de calcio
b. aldactone
c. kayaxelate
d. Bicarbonato de sodio
e. Todas las anteriores
40. Paciente de 84 anos quien recibe diuretico tiazidico para la hipertensión arterial es
admitido por presentar diarrea y trastornos del sensorio. Tiene disminución del turgor
de la piel y la presion arterial es normal, Pna 174mEq/L, Una 5mEq/L, Uosm
606mEq/kg. La hipertensión se debe a:
a. perdidas insensibles
b. polidipsia
c. uso de diuretico
d. diabetes insipida
e. secrecion inapropiada de hormona antidiuretica

41. Hipernatremia Hipervolemica


a. diuresis osmótica
b. diarrea
c. aldosteronismo primario
d. diabetes insipida
e. SIADH

42. La hipokalemia en un cirrotico puede precipitar COMA por:


a. Hipermagnesemia asociada
b. Aumento de la amoniogenesis
c. Hiperfosfatemia asociada
d. Rabdomiolisis
e. Induce vomitos
43. Causa de hiperkalemia:
a. Aumento del volumen arterial eficaz
b. Hipercatabolismo tisular
c. Hiperaldosteronismo
d. Alcalosis metabolica
e. Cirrosis hepatica

44. Causa de hipomagnesemia


a. cirrosis avanzada
b. falla renal cronica
c. falla cardiaca
d. constipacion
e. acidosis respiratoria

45. Causa renal de hipomagnesemia


a. falla renal cronica
b. esteatorrea
c. desnutricion severa
d. uso de diureticos de asa
e. infeccion urinaria

46. en caso de hipercalcemia pensar en:


a. neoplasia
b. hiperfosfatemia severa
c. nefrolitiasis
d. nefrocalcinosis
e. diureticos

47. En el síndrome nefrosico puede encontrarse cilindros:


a. cereos
b. hematicos
c. leucocitarios
d. epiteliales
mixtos leucocitario y hematic
31) En hiponatremia el daño principal es:

a) Neurológica
b) Respiratoria
c) Renal

32) Adaptación del cerebro ante solución hipotónica:

a) Pérdida de iones
b) Pérdida de aminoácidos
c) Edema cerebral
d) Ganancia de iones
11.habia una de renal de un man q estaba en UCI con fractura de base de craneo:
Diabetes insidia central
25.En el riñon la PTH
Aumenta la reabsorción tubular de Ca y disminuye la de fosforo

73. Causa de hiperfosfatemia:


i. Falla hepatica
j. Falla renal cronica
k. Falla cardiaca
l. Insuficiencia suprarrenal

74. Causas de hiperfosfatemia grave, excepto:


g. recuperacion de cetoacidosis diabetica.
h. Alcalosis respiratoria
i. Acidosis respiratoria.

75. Paciente con vómitos severos, oligurico con hipotensión arterial Una> 40…… 800
mOsm/Kg puede tener:
i. Insuciencia renal aguda.
j. Insuficiencia suprarrenal
k. Falla pre-renal
l. Exceso de mineralocorticoides

76. Paciente urémico crónico con deshidratación severa puede presentar lo siguiente
excepto:
a. Densidad urinaria 1010.
b. Osmolaridad urinaria de 800 mOsm/Kg
c. K sérico aumentado
d. Oliguria.

77. La hipokalemia produce:


a. Aumento de la magnitud del potencial de reposo
78. Uno de los indices de falla renal aguda es:
Uosm/Tosm

79. Hormona producida en los riñones que disminuye en la falla renal cronica:
a. ADH
b. Noradrenalina
c. Peptido natriuretico (segun arjona)
1,25 (OH)2 Vit D.
80. Causa de anuria:
a. Pielonefritis unilateral
b. Necrosis tubular aguda
c. Enf cambios minimo
d. Necrosis cortical

81. la etapa de la insuficiencia renal cronica con tasa de filtración glomerular de 60-89
ml/min es la:??
a. 1
b. 2
c. 3
d. 4

82. La FENA es la relación entre:


a. Oferta tubular/Na reabsorbido x 100
b. depuración de Na/ depuración de creatinina x 100
c. oferta tubular/ Na excretado x 100.
d. Na excretado/ Na reabsorvido x 100.
83. En la nefritis tubular intersticial aguda se pierde la capacidad de poder concentrar la
orina debida a:??
a. Daño medular
b. Daño glomerular
c. Daño tubular proximal
d. Daño de la porcion cortical del tubulo colector

84. La solucion de NaCl al 23,4 % tiene:


8000 mOsm/l

85. Se asocia con el SIAD, excepto:


lipoma

86. El efecto biologico mas importante de la calcitonina es, excepto:


a. Responder a la hipocalcemia
b. Aumenta la calciuria
c. Estimular la resorcion osea osteoclastica
d. Utilizada en el tx de la hipercalcemia.

87. Favorece la entrada de K al espacio intracelular, excepto:


a. Agonista adrenergico
b. Estimulacion adrenergico
c. Insulina
d. Alcalosis metabolica

88. Factores que afectan el balance hídrico en el post-operatorio, excepto:


a. Fármacos
b. Calcitriol
c. Administración de liquidos hipotonico
d. Aumento de la ADH

89. Las primeras manifestaciones clínicas de la hiponatremia son:??


a. neurológica
b. gastrointestinal
c. pulmonares
d. musculares

90. En la hipernatremia hipertonica se da lo siguiente excepto:


a. Entrada de Na
b. Salida de aa
c. Ganancia de iones
d. Disminución del agua cerebral

91. Manifestaciones clinicas de la hipercalcemia, excepto:


a. Taquicardia
b. HTA
c. Constipación
d. Vomitos

92. En falla renal aguda por nefrotoxicidad:


a. Se conserva la membrana basal
b. Se destruye la membrana basal
c. Frecuente hay oliguria
d. La creatinina no aumenta

93. Causa de hipomagnesemia:


alcoholismo

94. Na serico de 168 mEq/l 70 kg (utilize como % de agua al 50% y Na normal 140 mEq/l
7 litros

95. Indicaciones de diálisis en falla renal aguda, excepto:


a. Encefalopatia uremica
b. Hiperkalemia refractaria
c. Ac. Metabolica refractaria
d. hipocalcemia

96. El KCL IV se puede asministrar asi, excepto:


a.infusion por linea periferica de 10 meq en 100 ml en 1hora
b. “ “ central de 20 meq em 50ml en 1 hora
c. “ “ central de 100 meq em 50ml en 1 hora
d. “ “ periferica de 20 meq en 20ml en 2 horas

97. Aumenta la excrecion urinaria de K, excepto:


a. Triamtereno
b. Bumetanida
c. Furosemida
d. Metolazona

98. Factores que regulan la secrecion distal de K, excepto:


a. Flujo tubular distal y aporte distal de Na
b. Excrecion de aniones no reabsorbibles
c. Aldosterona
d. PTH

99. Enfermo de 60 Kg con 60% de agua corporal, Posm 260 mOsm/kg, ¿Cuántos gramos
de NaCl aumentaria el Na en 10 mEq?
a. 21 gramos

19. Un px de 25años fue ingresado a UCI por lesiones graves en cabeza y fractura de la base del
Graneo, aprox X horas después de la lesion manifestaba poliuria, la OSMU 150, y la de suero 350.
Los líquidos IV fueron detenidos y 3 hors después la producción de orina y la osm urinaria
permanecieron sin cambio. Se administraron 5 unidades de vasopresina IV, la osm de la orina se
incremento a 300. ¿Cuál es el dx mas probable?
a- Diabetes insípida central
b. diabetes insípida nefrogenica
c. intoxicación acuosa
d. sobrecarga de solutos
e. S IADH

55. indica falla renal aguda


a- U..N.. = 4.1
b- UN = 1
c. U..N.. = 1/4i/
d. Gravedad especifica 1.026
e. Ninguna

56. deshidratación por perdida de agua y densidad urinaria 1.003, creatinina 6 /


a. falla renal aguda parenquimatosa
b- falla prerenal
c. falla renal obstructiva
d. falla renal cronica
e. no hay falla renal

57. uno de los indices de falla renales


a. reabsorción de Na/oferta tubular de Na x 100
b. excresion urinaria deNa/oferta tubular de x 100
c.- oferta tubular de Na/excrecion urinaria de Na x 100 ~.--~
d- Una/Pna x 100
e. Ninguna

58. el Sdr hepatorenal es causa de falla:


a- prerrenal
b. renal
c. póstrenal
d. falla cardiaca
e. daño tubular

59. causa mas común de falla renal crónica:


A- DM

60- en la glomendonefritis aguda por infeccion, la causa, excepto


a- proteina mayor de 10 g
b- osmolaridad urinaria normal
c. hematuria
d. HT A
e. Edema

61. falla renal cronica, excepto


a. PTH aumentado
b- Hipofosfatemia
c- Acidemia
d. Hipocalcemia
e. 1.25 OH (D3) disminuida

62. EN la falla renal aguda siempre habra


a- Aumento de creatinina
b- fraccion de excrecion de Na aumentado
c- hiperkalemia
d- Una aumentado
e- Hipernatremia

63. causa de falla prerenal


a. Sdr nefrotico
b- obstrucción de ambos uréteres
c- falla cardiaca
d. Grlomerulonefritis aguda post-infeciosa
e. Calculo renal

64. en la nefritis tubular interticial se puede dar, excepto


a. h.ipernatremia
b. proteinuria de 1.5 g
c. normotension arterial
d. hipostenuria
e. hipocalcemia

65. Sugiere glomerulonefritis:


a. leucocituia
b-cilindros eritrocitarios
c- anuria
d. -

66 -causa de falla renal parenquimatosa


e) papilitis
f) calculo en pelvis derecha
g) estenosis unilateral de arteria renal
h) cistitis hemorragica

98 – Hep C periodo de incubación:


a- 50 dias

60- Causa de hipercalemia:


a Aumento del volumen arterial eficaz
b. Hipercatabolismo tisular
c Hiperaldosteronismo
d. Alcalosis Inetabolica
e. Cirrosis hepatica

61. Causa de hipomagnesemia


a. cirrosis avanzada
b. falla renal cronica
c. falla cardiaca
d. constipacion
e. acidosis respiratoria

62. Causa renal, de hipomagnesemia


a. falla renal crônica
b. esteatorreaa
c desnutricion severa
d. usó de diureticos de asa
e. infeccion urinaria

63. en caso de hipercalcemia pensar en:


a. neoplasia
b: hiperfosfatemia severa
c. nefrolitiasis
d. nefrocalcinosis
e. diuréticos
64. En el síndrome nefrosico puede encontrarse cilindros
a. cereos
b. hematicos
c. leucocitarios
d. epiteliales
e. mixtos leucocitario y hematicos,
11- Px cronico con deshidratación severa puede presentar los siguiente excepto:
a-osmolaridad urinaria de 800 mosmol/Kg

12- Px con vomito abundante, HTA, sodio urina rio en 6, oligurico, Osmolaridad de 800
presente,
a- insuficiencia suprarrenal

13- Px uremico cronico con vomito severo hipotension arterial orina al azar, muestra
sodio mas de 40, Osmolaridad de 800 puede tener :
a- Falla suprarrenal

14- En quien predomina la enfermedad de higado graso:


A- Sexo masculino

19- La hipocalemia predispone excepto:


b- amoniogenesis

20- Cual de las siguientes es causa de hiperfosfatemia


b- insuficiencia renal

21- en la glomerulonefritis aguda post infecciosa disminuye la tasa de filtración:


b- por alteración en el coeficiente de ultrafiltracion

22- causa de insuficiencia prerenal excpto:.


aa- Aneurisma disecante de la aorta
Urémico crónico con potasio sérico de 6.5mEq/L con leve academia, sin trastorno muscular y
EKG normal, puede ser tratado con:
e. kayaxelate
f. gluconato de Calcio al 10%
g. NaHCO3
h. Diálisis

La insuficiencia renal crónica ocasiona hiperfosfatemia permanentemente cuando la tasa de


filtración glomerular esta por debajo de:
e. 60ml/min
f. 50 ml/min
g. 25 ml/min
h. 40 ml/min

La hipofosfatemia es causa de excepto


e. inotropismo positivo
f. disminución de 2,3 DPG
g. hemólisis
h. curva de HbO2 desplazada hacia la izquierda

La solución de NaCl 23,4% tiene de osmolaridad:


e. 8000 mosm/l
f. 800 mosm/l
g. 80 mosm/l
h. 0,8 mosm/l

En la hiperkalemia:
e. aumenta la sístole eléctrica
f. hay menor diferencia entre el potencial de reposo y el umbral
g. el potencial de reposo transmembrana se hace mas negativo
h. el potencial umbral se hace menos negativo

La velocidad de administración de potasio en caso de gran riesgo para la vida puede ser de:
e. 10-20 mEq/L
f. 8 mEq/L
g. 110 mEq/L
h. 40 mEq/L

Un ejemplo de hiponatremia hipotónica normovolemica es:


e. síndrome nefrósico
f. SIADH
g. Diarrea
h. Cirrosis

A una embarazada con eclampsia se le ordena 1 litro de D/A 5% con 10g de MgSO4. ¿Cuántos
ml de MgSO4 al 50% se han utilizado?
a. 20
b. 5
c. 12,5
d. 30

Causa hipomagnesemia:
e. estreñimiento
f. alcoholismo
g. deshidratación hipertónica
h. insuficiencia renal aguda

Paciente con diuresis de 300ml/día, K sérico 7mEq/L, Posm y Uosm 300mosm/L, el diagnóstico
más probable es:
e. insuficiencia suprarrenal
f. insuficiencia renal
g. SIADH
h. Deshidratación primaria

La mielinosis pontina se caracteriza por:


e. pares craneales intactos
f. cuadriplejía flácida
g. comportamiento normal
h. hipertonía de los miembros inferiores

Calcule el déficit de agua en una paciente con sodio sérico 168mEq/L y que pesa 70Kg (utilice
como porcentaje de agua 50% y sodio normal mEq/L):
e. 5L
f. 3L
g. 10L
h. 7L

En la SIADH la concentración de sodio urinario es de:


e. menos de 10mEq/L
f. 20mEq/L
g. 15mEq/L
h. Mas de 40mEq/L

En paciente con vómitos abundantes y prolongados con hipokalemia severa debe


administrarse:
e. NaCl 0.9% con KHCO3
f. NaCl 0.9% con KCl
g. NaCl 0.9% con gluconato de potasio
h. NaCl 0.9% con NaHCO3

La hipokalemia en un paciente con cirrosis puede contribuir:


e. coma
f. alcalosis respiratoria
g. acidosis metabolica
h. aumento de la síntesis de urea

El plasma de un paciente contiene Na 125mEq/L, glucosa 108mg/dl, urea 300mg/dl, podrá


presentar:
e. cirrosis
f. síndrome nefrósico
g. síntomas de hipertonicidad
h. síntomas de hipotonicidad

La hiponatremia aguda sintomática es debida a, excepto:


e. exceso de glucocorticoides
f. SIADH
g. post-operatorio
h. intoxicación hídrica

La administración de solución salina 0.9% produce:


e. disminución del volumen intracelular y aumento del extracelular
f. aumento del volumen intra y extracelular
g. aumento del volumen extracelular
h. aumento del volumen intracelular

Alcalosis metabólica con hipertensión arterial, hipokalemia, aldosterona normal y renina


normal:
e. Hipertensión renovascular
f. Hipertensión maligna
g. Sindroma de Cushing
h. Administración de licor

La hipokalemia puede predisponer, excepto:


e. intoxicación digitálica
f. disminución de la amoniogénesis
g. rabdomiolisis
h. ondas U en el EKG
Plasma: creatinina 1mg/dl,140 mEq/L. Orina: creatinina 150 mg/dl, Na 40 mEq/L la FE Na es:
b) 0.19%

Tratamiento de inicio de SIADH es:


a) restricción hidrica

24. U Na 115, Uosm 240. P osm 680, P Na 62


SIADH

25. Paciente de 84 años que recibe diureticos para la hipertensión arterial. Presenta
diarrea y trastornos del sensorio y disminución del rubor de la piel. Na 174 U
Na 5 U osm 606. La hipernatremia se debe a:
Perdidas insensibles

26. hipernatremia hipervolemica


Aldosteronismo primario

27. causa de hipomagnesemia


Alcoholismo

28. hipocalcemia:
Provoca potencializacion de efectos de hiperkalemia

29. En hipocalcemia
Prolonga QT a expensas de segmento ST

30. Tratamiento de urgencia hipercalcemica


Solucion salina isotonica 0.9% 4 a 5 L

31. Síndrome nefrotico


Edema, proteinuria, hipertensión arterial, hematuria
2) Cuál de las siguientes medicinas pueden dar hiperkalemia
f) inhibidor de la enzima convertidota
g) bloqueador alfa adrenérgico
h) bloqueador beta adrenérgico
i) a y b
j) a y c

2) Hormonas que disminuyen en la falla renal crónica, excepto


a) 1,25 dihidroxi vitatamina D
b) eritropoyetina
c) paratohormona
d) foliculo estimulante

3) MgSO4 (PM 120) 1 gramo contiene de magnesio en meq


a) 10
b) 7.9
c) 8.3
d) 6

4) Factores que conducen a la progresión de falla renal crónica


a) hiperlipidemia
b) toxinas urémicas
c) proteinuria
d) b y c
c) todas

5) Paciente de 20 años presenta abundantes vômitos y diarrea de 3 días de evolución,


examen físico depleción de volumen, suero muestra Na 155, K 3meq/l, Cl 117meq/l, HCO3 25
meq/l, tratamiento de elección:
a) D/A 5% + KCl
b) solución salina 3% + KCl
c) solución salina 23.4% + KCl
d) solución salina 0.45% + KCl

6) En la enfermedad ateroembólica renal se da lo siguiente excepto:


d) Velocidad de eritrosedimentación

7) Pseudohiperkalemia de 8 mEq/l que se ve en el EKG:


Onda T picuda
Aplanamiento de la onda P
Ay B
Ninguna

8)Criterios para iniciar diálisis excepto:


Alcalosiss Metabolica

9)Hipokalemia con normotension excepto


Síndrome de liddle

10) Plasama creatinina 8mg/dl N de urea 100mg/dl U na 40 Meq/l :


Insuficiência renal

11) Factores reversibles responsables de deterioro de la funcion renal excepto:


Fibrosis intersticial

12) Causa hipokalemia con renina y aldosterona disminuida:


Licorice

13. Sugiere trastorno túbulo intersticial:


.Proteinuria mayor a 3.5 g /24 horas
.Notrmotensión
.Edema
.Osmolarida urinaria normal

14. Glomérulonefritis rápidamente progresiva


.Engrosamiento de la membrana basal
Fusión de los pedículos de los podocitos
prolñiferacion intracapilar
proliferación extracapilar ( creo que es esa verificar)

15.Una de las siguientes causas no produce hematuria


.enfermedad de cambios mínimos

16.Plasma Na:115, Posm240 mOsm,Uosm 680,Una 60:meq/l:


.Deshidratación hipotónica
.secreción inapropiada de ADH
.Deshidratación isotónica
.Deshidratación primaria

17.en la falla renal crónca produce necrosis


calcifilaxia

18. No produce hipokalemia:


Pancreatitis aguda
rabdomiolisis
neoplasia
Def. Vit D

19. Paciente con glucosa 90mg/dl, Na+= 115mEq/L, Urea en 140mg/dl


a. Síndrome hipoosmolar
b. Síndrome hiperosmolar
c. Diarrea
d. Ninguna de las anteriores.

20. Causa de seudohiponatremia con osmolaridad elevada


a. Hiperproteinemia
b. Diabetes mellitus
c. Hiperlipidemidemia
d. Reserción transuretral prostática.

21. Manifestaciones de hipermagnesemia excepto


Normotensión

22. Piel en uremia crónica excepto


Paño blanco

23. No es causa de necrosis papilar aguda


a.Diabetes mellitas
b. Uropatía obstructiva
c. Pielonefritis
d. Trombosis de venas renal.

24. Puede presentarse hiperuricemia excepto


d. Adenocarcinoma gástrico

25. Compromete riñones y pulmones:


a. Enf de GoodPasture
b. Granulomatosis de Wegener
c. LES
d. Ay C

26. Sugiere glomerulopatia:


-anasarca

27. TFG < 15ml/min corresponde al estadio de la enfermedad renal cronica:


* 4

28. Causa obstetrica de necrosis cortical bilateral


- desprendimiento prematuro de placenta

29. Promueve amoniogenesis em cirróticos


- hipokalemia

30. Criterio para diagnostico de síndrome nefrosico


a. Proteinuria en 24 hr de 3.5 gr para superficie corporal de 1. 73
b. Albumina serica < 3g/dl
c.Hiperlipidemia y edema
d. Todas las anteriores
e. ay b
24. Factor que altera el balance hídrico en el periódo post- operatorio:
Disminución de la ADH (Excepto)

25. Un ejemplo de hipokalemia con aldosterona alta y osmolaridad baja es:


Hiperaldosteronismo primario

26.Ejemplode hipernatremia hipervolémica: Hiperaldosteronismo primario

27.Hiperkalemioa son alteraciones del EKG no es necesario administrar gluconato de calcio se


colocará un antagonista de K asíe l Px no cae en paro y estoes si tiene manifestación en el EKG.

28. La fracción de excreción de sodio estará aumentada en falla cardíaca.

29.En la hiperkalemia: hay menos diferencia entre el potencial de reposo y el umbral.

30. La infusión de potasio sin riesgo para la vida del PX: es de 40 meq; porque

I lb de solución salina: debe llevr de 20 a40 mEq de KCl a una velocidad de 10 a 20 mEq por
hora; si aumenta la velocidad puede provocar un bloqueo A-V. Se recomienda una Velocidad
mayor si la hipokalemia es muy severa.

Sis e usa venas priféricas es:

10 meq en 100 cc de sol.salina para pasar en una hora

20 meq en 200 cc de sol.salina para pasar en una hora

En vena central:

20 meql en 50 cc pasarla en una hora

40 meq en 100cc pasar la en una hora

31.Los indicios de falla renal aguda son:

(Na filtrado/Na excretado) x 100%

33.Causa de enuresis: necrosis cortical difusa.


34.En SIDH la concentración urinaria es menor de 20.

35. Deshidratación pura de agua excepto: hay salida de electrolitos del cerebro.

36.Urémico crónico con deshidratación hipertónica severa y acidótico presentará lo siguiento


excepto: osmolaridad urinaria de 8mm mosmol/kg.

37. Px con vómitos abundante, hipertensión arterial; Na urinario=6, oliguria, osmolaridad de 800
presenta: insuficiencia suprarrenal.

38. Factor que predispone a la hipernatremia son intoxicación hídrica.

39. Se presenta en acidosis tubular renal proximal excepto: NaHCO3 y lo alfa antagonistas.
1. Cual de las siguientes medicinas puede dar hiperkalemia:
A. inhibidor de la enzima convertidora
B. bloqueador alfa adrenérgico
C. bloqueador beta adrenérgico
D. A Y B
E. A Y C
“Los betabloqueantes suprimen la liberación de renina estimulada por catecolaminas, con lo cual se reduce
la síntesis de aldosterona. Además, y más importante aún, estos fármacos disminuyen la captación celular
de potasio. ”
2. Hormonas que disminuyen en la falla renal crónica, excepto:
A. 1,25 vit D
B. eritropoyetina
C. paratohomona
D. folículo estimulante
3. MgSO4 (PM 120) 1g contiene Mg en meq:
A. 1.0
B. 7.9
C. 8.3
D. 6
4. Factores que conducen a la progresión de falla renal crónica:
A. hiperlipidemia
B. toxinas urémicas
C. proteinuria
D. B y C
E. Todas
5. Paciente de 20 años presenta abundantes vómitos y diarrea de 3 días de
evolución, examen físico depleción de volumen, suero muestra Na 155, K 3
meq/l, Cl 117 meq /l , HCO3 25 meq /l tratamiento de elección:
A. D/A 5% + KCl
B. Sol. Salina 3% + KCl
C. Sol. Salina 23.4% + KCl
D. Sol. Salina 0.45% + KCl
6. En la enfermedad ateroembolica renal se da lo siguiente excepto:
D. velocidad de eritrosedimentación
7. Pseudohiperkalemia de 8 meq / l se ve en el EKG:
A. onda T picuda
B. aplanada la onda P
C. A y B
D. Ninguna
8. Criterios para iniciar diálisis, excepto:
A. alcalosis metabólica
9. Hipokalemia con normotensión excepto:
A. síndrome de Liddle
10. Plasma creatinina 8 meq/dl N de urea 100 mg/dl Urea 40 meq/l :
A. insuficiencia renal
11. Factores reversibles responsables del deterioro de la función renal, excepto:
A. fibrosis intersticial
12. Causa Hipokalemia con renina y aldosterona disminuida:
A. Síndrome de Liddle
13. Sugiere trastorno túbulo intersticial
- proteinuria > 3.5g/ 24 hrs
- normotensión
- no edema
- osmolaridad urinaria normal
14. Glomerulonefritis rápidamente progresiva:
D. hay proliferación extracapilar
15. Una de la siguientes causas no produce hematuria:
A. enfermedad de cambios mínimos
16. Plasma: Na 115, Posm 240 mosm, Uosm 680, Urea 60 meq:
A. deshidratación hipotónica
B. secreción inapropiada de ADH
C. deshidratación isotónica
D. deshidratación primaria
17. falla renal crónica produce necrosis:
A. Calcifilaxia
18. no produce Hipokalemia:
A. pancreatitis aguda
B. rabdomiólisis
C. neoplasia
D. def. vitamina D
19. Paciente con glucosa 90mg/dl, Na 115 meq, urea en 40 mg/dl:
A. Síndrome hipoosmolar
B. Síndrome hiperosmolar
C. Diarrea
D. Ninguna de las anteriores
20. Causa de pseudohiponatremia con osmolaridad elevada:
A. hiperproteinemia
B. diabetes mellitus
C. hiperlipidemia
D. reserción transuretral prostática
21. Manifestación de hipermagnesemia excepto:
A. Normotensión
22. Piel en uremia crónica excepto:
A. Paño blanco
23. No es una causa de necrosis papilar aguda:
A. diabetes mellitus
B. uropatía obstructiva
C. pielonefritis
D. trombosis de vena renal
24. Puede presentar hiperuricemia excepto :
A. Adenocarcinoma gástrico
25. Compromete riñones y pulmones:
A. Enf de GoodPasture
B. Granulomatosis de Wegener
C. LES
D. A y C
E. TODAS
26. Sugiere glomerulopatia:
A. Anasarca
27. TFG < 15ml/min corresponde al estadio de la enfermedad renal cronica:
*5

27. Causa obstétrica de necrosis cortical bilateral


A. desprendimento prematuro de placenta
28. Promueve amoniogenesis en cirróticos
A. Hipokalemia
29. Criterio para diagnostico de síndrome nefrosico
a. Proteinuria en 24 hr de 3.5 gr para superficie corporal de 1. 73
b. Albumina serica < 3g/dl
c.Hiperlipidemia y edema
d. Todas las anteriores
e. a y b
46. el antigeno carcinoembrionario es util en el estudio de pacientes:
c. es eficaz para su detección

40. El ciclo de la urea tiene como finalidad fundamental:


a. sintetizar protrombina
b. transformar el amoniaco
c. producir sales biliares
d. sintetizar colesterol
e. producir acido folico

1. Gradiente sérico de albumina menor de 2.1, se observa en:


r. hipoalbuminemia severa.

2. Causa de hipomagnesemia:
a. estreñimiento
b. alcoholismo
c. falla pre-renal
d. IRA (insuficiencia renal aguda)
3. La hipocalcemia en la falla renal aguda puede ser asintomática por la:
a. Acidemia
b. hipoalbuminemia
c. Deficiencia de PTH
d. Alcalemia

4. La Hipofosfatemia causa, excepto:


a. Aumento de vit. D
b. Aumento de PTH
c. Disminución de la excreción de fosfato
d. Aumento de la absorción de fosfato a nivel intestinal

5. Un Pte. con diuresis de 250 ml al día con osmolaridad plasmática (presión osmótica)
y osmolaridad urinaria de 300 mosm/l, el dx probable e:
a. Insuficiencia suprarrenal
b. Insuficiencia renal
c. Nefritis túbulo intersticial crónica
d. Deshidratación hipertónica

6. Pte. con oliguria, hiperkalemia, hiponatremia, creatinina 0.8 mg/dl y CPK normal:
Esto puede corresponder a:
a. IRA
b. Insuficiencia suprarrenal
c. Síndrome hepatorrenal
d. Rabdomiólisis

7. No está indicado en caso de hiperkalemia:


r. Aldactone

8. Mujer con hiperémesis grávida:


r. Alcalosis hipoclórica

9. En causa de hipomagnesemia:
r. Alcoholismo.

10. En el metabolismo del calcio está implicado, menos:


r. Insulina

11. Hiponatremia, hipotónico, normovolémico:


r. SIADH

12. Fórmula del clearence de creatinina:


r. clearence = 140 – edad x peso en Kg
72 x creatinina

13. Fórmula de irritabilidad muscular:


r. IR = Na + K + OH
( ) + Mg + H

14. Con respecto al síndrome diarreico, nombre 2 tipos osmóticos:


a. Laxantes osmóticos (Mg2+, PO4, SO4)
b.
Carbohidratos no absorbibles (sorbitol, lactulosa).
15. Nombre dos síndromes diarreicos tipo secretora:
a. Consumo crónico de etanol.
b.
Obstrucción intestinal incompleta.

16. Nombre 2 síndromes diarreicos motores con aumento de motilidad:


a. Hipertiroidismo
b.
Prostaglandinas.

17. Indica falla renal aguda:


a. U.N. = 4.1
b.
U.N. = 1
c.
U.N. = ¼
d.
Gravedad específica: 1.026
e.
Ninguna.

18. Deshidratación por pérdida de agua y densidad urinaria 1.003, creatinina 6:


a. falla renal aguda parenquimatosa
b.
Falla pre-renal
c.
Falla renal obstructiva
d.
Falla renal crónica
e.
No hay falla renal.

19. Uno de los índices de falla renal es:


a. Reabsorción de Na/ oferta tubular de Na x 100
b.
Excreción urinaria de Na/ oferta tubular de Na x 100
c.
Oferta tubular de Na/ excreción urinaria de Na x 100 **
d.
Una / pna x 100
e.
Ninguna

20. El síndrome hepatorrenal es causa de falla:


a. pre-renal
b.
renal
c.
posrrenal
d.
falla cardiaca
e.
daño tubular

21. Causa más común de falla renal crónica:


a. Diabetes mellitus
b.
HTA
c.
Glomerulonefritis
d.
Enfermedad poliquística renal
e.
Vasculitis

36. En la Glomerulonefritis aguda por infección, la causa, excepto:


a. proteínas mayor de 10g
b. osmolaridad urinaria normal
c. hematuria
d. HTA
e. Edema

• A pesar de la diversidad de enfermedades indicadas abajo, ellas tienen muchos


síntomas en común. Con frecuencia, los síntomas y signos observados incluyen:
disminución en el volumen de orina, proteína en la orina (proteinuria), sangre en la
orina microscópica o macroscópica (hematuria), hinchazón (edema), presión
sanguínea alta (HTA) y una disminución en la capacidad del riñón de extraer
eficazmente los residuos.

22. Falla renal crónica, excepto:


a. PTH aumentada
b. Hipofosfatemia
c. Acidemia
d. Hipocalcemia
e. 1.25 OH (D3) disminuida.

23. En la falla renal aguda siempre habrá:


a. aumento de creatinina
b. fracción de excreción de Na aumentado
c. Hiperkalemia
d. Una aumentado
e. Hipernatremia

24. Causa de falla pre-renal:


a. Síndrome Nefrótico
b. Obstrucción de ambos uréteres
c. Falla cardiaca
d. Glomerulonefritis aguda pos- infecciosa
e. Calculo renal

25. En la nefritis tubular intersticial se puede dar, excepto:


a. Hipernatremia
b. proteinuria de 1.5 g
c. normotensión arterial
d. hipostenuria
e. hipocalcemia

26. Sugiere Glomerulonefritis


a. leucocituria
b. cilindros eritrocitarios
c. anuria

27. Causas de falla renal parenquimatosa:


a. papilitis
b. calculo en pelvis derecha
c. estenosis unilateral de arteria renal
d. cistitis hemorrágica

28. Causa de anuria:


a. Enfermedad de cambios mínimos
b. Necrosis cortical
c. Pielonefritis aguda leve bilateral
d. Necrosis tubular aguda
e. Calculo coloriforme bilateral

29. La Hipofosfatemia causa todo, excepto:


r. Aumento de vit. D - MAL

30. En Pte. con falla renal crónica que ha sido trasplantado esperamos encontrar:
r. osteomalacia.

Calcule el déficit de agua en una paciente con sodio sérico 168mEq/L y que pesa 70Kg
(utilice como porcentaje de agua 50% y sodio normal mEq/L):
a. 5L
b. 3L
c. 10L
d. 7L

En la SIADH la concentración de sodio urinario es de:


a. menos de 10mEq/L
b. 20mEq/L
c. 15mEq/L
d. Mas de 40mEq/L
En paciente con vómitos abundantes y prolongados con hipokalemia severa debe
administrarse:
a. NaCl 0.9% con KHCO3
b. NaCl 0.9% con KCl
c. NaCl 0.9% con gluconato de potasio
d. NaCl 0.9% con NaHCO3

La hipokalemia en un paciente con cirrosis puede contribuir:


a. coma
b. alcalosis respiratoria
c. acidosis metabolica
d. aumento de la síntesis de urea

El plasma de un paciente contiene Na 125mEq/L, glucosa 108mg/dl, urea 300mg/dl,


podrá presentar:
a. cirrosis
b. síndrome nefrósico
c. síntomas de hipertonicidad
d. síntomas de hipotonicidad

La hiponatremia aguda sintomática es debida a, excepto:


a. exceso de glucocorticoides
b. SIADH
c. post-operatorio
d. intoxicación hídrica

La administración de solución salina 0.9% produce:


a. disminución del volumen intracelular y aumento del extracelular
b. aumento del volumen intra y extracelular
c. aumento del volumen extracelular
d. aumento del volumen intracelular

Alcalosis metabólica con hipertensión arterial, hipokalemia, aldosterona normal y


renina normal:
a. Hipertensión renovascular
b. Hipertensión maligna
c. Sindroma de Cushing
d. Administración de licor

La hipokalemia puede predisponer, excepto:


a. intoxicación digitálica
b. disminución de la amoniogénesis
c. rabdomiolisis
d. ondas U en el EKG

¿Cuál de los siguientes no es un efecto secundario de los antiácidos?


a. Estreñimiento
b. Diarrea
c. Hipercalcemia
d. Hiperfofatemia
e. Retención de sodio

La PTH promueve todo lo siguiente excepto:


a. aumento de la absorción intestinal de calcio
b. aumento de la absorción tubular de fosfato
c. aumento de la resorción ósea de fosfato
d. aumento de la resorción ósea de calcio
e. disminución de la reabsorción de fosfato

Un varón de 57 esta con hemodiálisis de mantenimiento para insuficiencia renal


crónica, ¿Cuál de las sig anomalías metabólicas podría ser anticipáda?
a. hipernatremia
b. hiponatremia
c. exceso de vitamina D
d. hipoparatiroidismo

Un px de 25años fue ingresado a UCI por lesiones graves en cabeza y fractura de la base
del Graneo, aprox X horas después de la lesion manifestaba poliuria, la OSMU 150, y la de
suero 350. Los líquidos IV fueron detenidos y 3 hors después la producción de orina y
la osm urinaria permanecieron sin cambio. Se administraron 5 unidades de
vasopresina IV, la osm de la orina se
incremento a 300. ¿Cuál es el dx mas probable?
a. Diabetes insípida central
b. diabetes insípida nefrogenica
c. intoxicación acuosa
d. sobrecarga de solutos
e. S IADH

Deshidratación por perdida de agua y densidad urinaria 1.003, creatinina 6 /


a. falla renal aguda parenquimatosa
b. Falla prerenal
c. falla renal obstructiva
d. falla renal crónica
e. no hay falla renal

Uno de los indices de falla renales


a. reabsorción de Na/oferta tubular de Na x 100
b. excresion urinaria deNa/oferta tubular de x 100
c. oferta tubular de Na/excrecion urinaria de Na x 100 ~.--~
d. Una/Pna x 100
e. Ninguna

El Sdr hepatorenal es causa de falla:


a. prerrenal
b. renal
c. póstrenal
d. falla cardiaca
e. daño tubular

Causa más común de falla renal crónica:


a. DM

En la glomendonefritis aguda por infección, causa, excepto


a. proteina mayor de 10 g
b. osmolaridad urinaria normal
c. hematuria
d. HT A
e. Edema

Falla renal crónica, excepto


a. PTH aumentado
b. Hipofosfatemia
c. Acidemia
d. Hipocalcemia
e. 1.25 OH (D3) disminuida

EN la falla renal aguda siempre habrá


a. Aumento de creatinina
b. fraccion de excrecion de Na aumentado
c. hiperkalemia
d. Una aumentado
e. Hipernatremia

Causa de falla prerenal


a. Sdr nefrotico
b. obstrucción de ambos uréteres
c. falla cardiaca
d. Grlomerulonefritis aguda post-infeciosa
e. Calculo renal

En la nefritis tubular interticial se puede dar, excepto


a. hipernatremia
b. proteinuria de 1.5 g
c. normotension arterial
d. hipostenuria
e. hipocalcemia

Sugiere glomerulonefritis:
a. leucocituia
b. cilindros eritrocitários
c. anuria

Causa de falla renal parenquimatosa


a. papilitis
b. calculo en pelvis derecha
c. estenosis unilateral de arteria renal
d. cistitis hemorragica

Causa de hipercalemia:
a. Aumento del volumen arterial eficaz
b. Hipercatabolismo tisular
c. Hiperaldosteronismo
d. Alcalosis Inetabolica
e. Cirrosis hepatica

Causa de hipomagnesemia
a. cirrosis avanzada
b. falla renal cronica
c. falla cardiaca
d. constipacion
e. acidosis respiratoria

Causa renal, de hipomagnesemia


a) falla renal crónica
b) esteatorreaa
c) desnutrición severa
d) usó de diuréticos de asa
e) infección urinaria

En caso de hipercalcemia pensar en:


a) neoplasia
b) hiperfosfatemia severa
c) nefrolitiasis
d) nefrocalcinosis
e) Diuréticos

En el síndrome nefrósico puede encontrarse cilindros


a) Cereos (Cereos y grasos)
b) Hemáticos (en S. nefrítico)
c) leucocitarios
d) epiteliales
e) mixtos leucocitario y hematicos,

Cual de estas afirmaciones es falsa:


a) Las transaminasas se elevan cuando hay daño celular hepático
b) Las transaminasas se elevan en las hepatopatias alcoholicas
c) El cociente GOT/GPT normal es de 1.3
d) Las transaminasas se elevan en las colestasis
e) Unas transaminasas normales excluyen enfermedad hepática

En el riñón la hormona paratiroides:


a. Aumenta la reabsorción tubular de calcio y disminuye la de fósforo

La velocidad de administración de potasio: 40 meq

Hipocalcemia produce: fosfaturia

En paciente con vómito severo hipotensión arterial o más de 40 meq


a. Insuficiencia suprarrenal

Glomerulonefritis post estreptococcica A


a. ltera el cociente de ultrafiltración

La renal aguda, oligurica todo es cierto excepto: Osmolaridad urinaria aumentada

Paciente can alcalosis metabólica e hipokalemia debe ser tratada con KCL

Usted tiene una paciente con hiperhemesis gravídica. El trastorno metabólico que
espera encontrar es:
a. acidosis hipocloremica
b. acidosis hipercloremica
c. alcalosis hipocloremica
d. alcalosis hipercloremica
e. no hay alteración acido base

Cuál de las siguientes opciones esta contraindicada en el tratamiento de la


hiperkalemia
a. gluconato de calcio
b. aldactone
c. kayaxelate
d. Bicarbonato de sodio
e. Todas las anteriores

Rotando en sala de medicina interna le piden que evalué un paciente con HIV el cual
porpresentar una meningitis por criptococci recibe anfotericina B; dicho este
medicamento buscaría como complicación:
a. aparicion de diarrea
b. aumento de la presion arterial
c. c- hipokalemia
d. d . cefalea
e. dolor toráxico

Donde se halla el calcio en el organismo:


a. 50% sangre
b. 20 % proteínas
c. 99% hueso

Cual de los siguientes datos hacen dudar del diagnostico de colon irritable:
a. aparicion en la vejez
b. rectorragia
c. esteatorrea
d. todas las anteriores

Distensión simultanea de antiHBC y HBs es signo de


a. infeccion prodromica
b. infección activa
c. infeccion pasada
d. ninguna

En la hipercalemia
a. hay una menor diferencia entre el potencial de reposo y el umbral

La hipermagnesemia puede producir


a. paro respiratorio

Tx de hipercalcemia
a. SSN 0.9 %

La hipofosfatemia son causas excepto:


a. aumento de PTH
b. aumento de la absorción intestinal de fosfato
c. disminuye fosfatemia
d. aumento de vitamina D

Una solución isotónica


a. aumenta el volumen extracelular

La vel de administración de potasio en caso de riesgo para la vida


a. 40 meq por hora

Px crónico con deshidratación severa puede presentar los siguiente excepto:


a. osmolaridad urinaria de 800 mosmol/Kg

Px con vómito abundante, HTA, sodio urinario en 6, oligurico, Osmolaridad de


800 presente,
a. insuficiencia suprarrenal

Px urémico crónico con vómito severo hipotensión arterial orina al azar,


muestra sodio mas de 40, Osmolaridad de 800 puede tener :
a. Falla suprarrenal

En quien predomina la enfermedad de higado graso:


a. Sexo masculino

Cual de las siguientes es causa de hiperfosfatemia


a. insuficiencia renal

En la glomerulonefritis aguda post infecciosa disminuye la tasa de filtración:


a. por alteración en el coeficiente de ultrafiltracion

El magnesio esta aumentado


a. eritrocito

Cuál de las siguientes medicinas pueden dar hiperkalemia


a. inhibidor de la enzima convertidota
b. bloqueador alfa adrenérgico
c. bloqueador beta adrenérgico
d. a y b
e. a y c
Hormonas que disminuyen en la falla renal crónica, excepto
a. 1,25 dihidroxi vitatamina D
b. eritropoyetina
c. paratohormona
d. foliculo estimulante

MgSO4 (PM 120) 1 gramo contiene de magnesio en meq


a. 10
b. 7.9
c. 8.3
d. 6

Factores que conducen a la progresión de falla renal crónica


a. hiperlipidemia
b. toxinas urémicas
c. proteinuria
d. b y c
e. todas

Paciente de 20 años presenta abundantes vômitos y diarrea de 3 días de evolución,


examen físico depleción de volumen, suero muestra Na 155, K 3meq/l, Cl 117meq/l,
HCO3 25 meq/l, tratamiento de elección:
a. D/A 5% + KCl
b. solución salina 3% + KCl
c. solución salina 23.4% + KCl
d. solución salina 0.45% + KCl

En la enfermedad ateroembólica renal se da lo siguiente excepto:


a. Velocidad de eritrosedimentación

Pseudohiperkalemia de 8 mEq/l que se ve en el EKG:


a. Onda T picuda
b. Aplanamiento de la onda P
c. Ay B
d. Ninguna

Criterios para iniciar diálisis excepto:


a. Alcalosiss Metabolica

Hipokalemia con normotension excepto


a. Síndrome de liddle

Plasama creatinina 8mg/dl N de urea 100mg/dl U na 40 Meq/l :


a. Insuficiência renal

Factores reversibles responsables de deterioro de la funcion renal excepto:


a. Fibrosis intersticial

Causa hipokalemia con renina y aldosterona disminuida:


a. Liddle

Sugiere trastorno túbulo intersticial:


a. Proteinuria mayor a 3.5 g /24 horas
b. Normotensión
c. Edema
d. Osmolaridad urinaria normal

Glomérulonefritis rápidamente progresiva


a. Engrosamiento de la membrana basal
b. Fusión de los pedículos de los podocitos
c. proliferacion intracapilar
d. proliferación extracapilar

Plasma Na:115, Posm240 mOsm,Uosm 680,Una 60:meq/l:


a. Deshidratación hipotónica
a. Secreción inapropiada de ADH
b. .Deshidratación isotónica
c. .Deshidratación primaria

En la falla renal crónca produce necrosis


a. calcifilaxia

No produce hipokalemia:
a. Pancreatitis aguda
b. rabdomiolisis
c. neoplasia
d. Def. Vit D

Paciente con glucosa 90mg/dl, Na+= 115mEq/L, Urea en 140mg/dl


a. Síndrome hipoosmolar
b. Síndrome hiperosmolar
c. Diarrea
d. Ninguna de las anteriores.

Causa de seudohiponatremia con osmolaridad elevada


a. Hiperproteinemia
b. Diabetes mellitus
c. Hiperlipidemidemia
d. Reserción transuretral prostática.

Manifestaciones de hipermagnesemia excepto


a. Normotensión

Piel en uremia crónica excepto


a. Paño blanco

No es causa de necrosis papilar aguda


a. a.Diabetes mellitas
b. Uropatía obstructiva
c. Pielonefritis
d. Trombosis de venas renal.

Puede presentarse hiperuricemia excepto


a. Adenocarcinoma gástrico

Compromete riñones y pulmones:


a. Enf de GoodPasture
b. Granulomatosis de Wegener
c. LES
d. Ay C
e. TODAS

Sugiere glomerulopatia:
a. anasarca

TFG < 15ml/min corresponde al estadio de la enfermedad renal cronica:


a. 5

Causa obstetrica de necrosis cortical bilateral


a. Desprendimiento prematuro de placenta
Promueve amoniogenesis en cirróticos
a. hipokalemia

Criterio para diagnostico de síndrome nefrosico


a. Proteinuria en 24 hr de 3.5 gr para superficie corporal de 1. 73
b. Albumina serica < 3g/dl
c. Hiperlipidemia y edema
d. Todas las anteriores
e. ay b

El aumento de -fetoproteina serica puede detectar , excepto


a. Carcinoma pancreático

Cuando usted evalua a un paciente con diagnostico de cirrosis hepática y le da


seguimiento se preocupa de las siguientes situaciones que le puede llevar a una
encefalopatía hepática; menos una:
a. hiponatremia
b. ingesta baja en proteinas
c. hipokalemia
d. sangrado digestivo alto
e. aparicion de una infeccion

Cual de los siguientes datos refleja vieja gravedad de una pancreatitis - esta es la que
es!
a. distensión abdominal
b. hipocalcemia
c. hiperamilasemia
d. aumento de la intensidad del dolor
e. disnea

El ciclo de la urea tiene como finalidad fundamental:


f. sintetizar protrombina
g. transformar el amoniaco
h. producir sales biliares
i. sintetizar colesterol
j. producir acido folico

La transformación de la Vit D2 en 25(OH)D3 tiene lugar en


a. hígado
b. piel
c. riñón
d. intestino
e. hueso

La elevación de la alfafetoproteina serica puede detectarse excepto:


a. carcinoma pancreático

La elevación de la alfafetoproteteina renca puede detestarse en todas excepto:


a. carcinoma primitivo de hígado
b. carcinoma germinal de ovario
c. Carcinoma pancreático
d. D. carcinoma testicular
e. hepatitis aguda

Las perdidas renales excesivas de potasio ....


a. Colitis ulcerativa
b. Enf de Cronh
c. colon irritable
d. tumor velloso
Gradiente sérico de albumina menor de 1.1, se observa en:
a. hipoalbuminemia severa.

Las fosfatasas alcalinas no aparecen nunca elevadas en una de las siguientes


circunstancias:
a. enf de Hodgkin
b. raquitismo
c. cirrosis biliar primaria
d. sarcoidosis
e. gota

Gradiente sérico de albumina menor de 2.1, se observa en:


a. hipoalbuminemia severa.

Causa de hipomagnesemia:
e. estreñimiento
f. alcoholismo
g. falla pre-renal
h. IRA (insuficiencia renal aguda)

La hipocalcemia en la falla renal aguda puede ser asintomática por la:


e. acidemia
f. hipoalbuminemia
g. Deficiencia de PTH
h. Alcalemia

La hipofosfatemia causa, excepto:


e. Aumento de vit. D
f. Aumento de PTH
g. Disminución de la excresión de fosfato
h. Aumento de la absorción de fosfato a nivel intestinal

Un pte con diuresis de 250 ml al día con osmolaridad plasmática (presión osmótia) y
osmolaridad urinaria de 300 mosm/l, el dx probable e:
e. Insuficiencia suprarrenal
f. Insuficiencia renal
g. Nefritis túbulo intersticial crónica
h. Deshidratación hipertónica

Pte con oliguria, hiperkalemia, hiponatremia, creatinina 0.8 mg/dl y CPK normal: Esto
puede corresponder a:
e. IRA
f. Insuficiencia suprarrenal
g. Síndrome hepatorrenal
h. Rabdomiolisis

No está indicado en caso de hiperkalemia:


a. Aldactone

Mujer con hiperemesis grávida:


a. Alcalosis hipoclórica

En causa de hipomagnesemia:
a. Alcoholismo.

En el metabolismo del calcio está implicado, menos:


a. Insulina

Hiponatremia, hipotónico, normovolémico:


a. SIADH

Fórmula del clearence de creatinina:


a. clearence = 140 – edad x peso en Kg
72 x creatinina

Fórmula de irritabilidad muscular:


a. IR = Na + K + OH
( ) + Mg + H

Con respecto al síndrome diarreico, nombre 2 tipos osmóticos:


a. Laxantes osmóticos (Mg2+, PO4, SO4)
b. Carbohidratos no absorbibles (sorbitol, lactulosa).

Nombre 2 síndromes diarreicos motores con aumento de motilidad:


c. Hipertiroidismo
d. Prostaglandinas.

Indica falla renal aguda:


f. U.N. = 4.1
g. U.N. = 1
h. U.N. = ¼
i. Gravedad específica: 1.026
j. Ninguna.

Deshidratación por pérdida de agua y densidad urinaria 1.003, creatinina 6:


f. falla renal aguda parenquimatosa
g. Falla pre-renal
h. Falla renal obstructiva
i. Falla renal crónica
j. No hay falla renal.

Uno de los índices de falla renal es:


f. Reabsorción de Na/ oferta tubular de Na x 100
g. Excreción urinaria de Na/ oferta tubular de Na x 100
h. Oferta tubular de Na/ excresión urinaria de Na x 100 **
i. Una / pna x 100
j. Ninguna

El síndrome heatorrenal es causa de falla:


f. pre-renal
g. renal
h. posrenal
i. falla cardiaca
j. daño tubular

Causa más comun de falla renal cronica:


f. Diabetes melitus
g. HTA
h. Glomerulonefritis
i. Enfermedad poliquística renal
j. Vasculitis

En la glomerulnefritis aguda por infección, la causa, excepto:


a. proteínas mayor de 10g
b. osmolaridad urinaria normal
c. hematuria
d. HTA
e. Edema
• A pesar de la diversidad de enfermedades indicadas abajo, ellas tienen muchos
síntomas en común. Con frecuencia, los síntomas y signos observados
incluyen: disminución en el volumen de orina, proteína en la orina
(proteinuria), sangre en la orina microscópica o macrscópica (hematuria),
hinchazón (edema), presión sanguínea alta (HTA) y una disminución en la
capacidad del riñón de extraer eficazmente los residuos.

Falla renal crónica, excepto:


f. PTH aumentada
g. Hipofosfatemia
h. Acidemia
i. Hipocalcemia
j. 1.25 OH (D3) disminuída.

En la falla renal aguda siempre habrá:


f. aumento de creatinina
g. fracción de excresión de Na aumentado
h. Hiperkalemia
i. Una aumentado
j. Hipernatremia

Causa de falla pre-renal:


f. Sd. Nefrótico
g. Obstrucción de ambos uréteres
h. Falla cardiaca
i. Glomerulonefritis aguda pos- infecciosa
j. Calculo renal

En la nefritis tubular intersticial se puede dar, excepto:


f. hipernatremia
g. proteinuria de 1.5 g
h. normotension arterial
i. hipostenuria
j. hipocalcemia

Sugiere glomerulonefritis
d. leucocituria
e. cilindros eritrocitarios
f. anuria

Causa de anuria:
f. Enfermedad de cambios mínimos
g. Necrosis cortical
h. Pielonefritis aguda leve bilateral
i. Necrosis tubular aguda
j. Calculo coloriforme bilateral

En paciente con falla renal crónica que ha sido trasplantado esperamos encontrar:
a. osteomalacia.

1. Causa de hiperfosfatemia:
a. Falla hepatica
b. Falla renal cronica
c. Falla cardiaca
d. Insuficiencia suprarrenal

2. Causas de hiperfosfatemia grave, excepto:


a. recuperacion de cetoacidosis diabetica.
b. Alcalosis respiratoria
c. Acidosis respiratoria.

3. Paciente con vómitos severos, oligurico con hipotensión arterial Una> 40…… 800
mOsm/Kg puede tener:
a. Insuciencia renal aguda.
b. Insuficiencia suprarrenal
c. Falla pre-renal
d. Exceso de mineralocorticoides

4. Paciente urémico crónico con deshidratación severa puede presentar lo siguiente


excepto:
a. Densidad urinaria 1010.
b. Osmolaridad urinaria de 800 mOsm/Kg
c. K sérico aumentado
d. Oliguria.

5. La hipokalemia produce: → celula menos excitable


a. Aumento de la magnitud del potencial de reposo

6. El marcador tumoral mas importante del Ca de páncreas:


a. AFP
b. CA 19-9
c. ACE
d. K-ras

7. El siguiente no es factor de riesgo para cancer de colon:


a. AINES.
b. Acidosis biliar
c. Tabaquismo
d. Dietas altas en grasas

8. En cuanto a la AFP, todo es cierto, excepto: → es un marcador tumoral en adultos, y


en liquido amniotico dice prbably el feto tenga alguna anormalidad
c. Suele estar aumentada en colangiosarcoma

9. Primera*****Falla renal crónica:


a.HTA
b.Glomerulopatias
c.Diabetes Mellitus
d.Nefritis tubulo intersticial

10. Uno de los indices de falla renal aguda es:


Uosm/Tosm

11. Hormona producida en los riñones que disminuye en la falla renal cronica:
a. ADH
b. Noradrenalina
c. Peptido natriuretico (segun arjona)
d. 1,25 (OH)2 Vit D.

12. Causa de anuria:


a. Pielonefritis unilateral
b. Necrosis tubular aguda
c. Enf cambios minimo
d. Necrosis cortical

13. la etapa de la insuficiencia renal cronica con tasa de filtración glomerular de 60-89
ml/min es la:??
a. 1
b. 2
c. 3
d. 4

14. La FENA es la relación entre:


a. Oferta tubular/Na reabsorbido x 100
b. depuración de Na/ depuración de creatinina x 100
c. oferta tubular/ Na excretado x 100.
d. Na excretado/ Na reabsorvido x 100.

15. En la nefritis tubular intersticial aguda se pierde la capacidad de poder concentrar la


orina debida a:??
a. Daño medular
b. Daño glomerular
c. Daño tubular proximal
d. Daño de la porcion cortical del tubulo colector

16. La solucion de NaCl al 23,4 % tiene:


8000 mOsm/l

17. El efecto biologico mas importante de la calcitonina es, excepto:


a. Responder a la hipocalcemia
b. Aumenta la calciuria
c. Estimular la resorcion osea osteoclastica
d. Utilizada en el tx de la hipercalcemia.

18. Favorece la entrada de K al espacio intracelular, excepto:


a. Agonista adrenergico
b. Estimulacion adrenergico
c. Insulina
d. Alcalosis metabolica

19. Factores que afectan el balance hídrico en el post-operatorio, excepto:


a. Fármacos
b. Calcitriol
c. Administración de liquidos hipotonico
d. Aumento de la ADH

20. Las primeras manifestaciones clínicas de la hiponatremia son:??


a. neurológica
b. gastrointestinal
c. pulmonares
d. musculares

21. En la hipernatremia hipertonica se da lo siguiente excepto:


a. Entrada de Na
b. Salida de aa
c. Ganancia de iones
d. Disminución del agua cerebral

22. Manifestaciones clinicas de la hipercalcemia, excepto:


a. Taquicardia
b. HTA
c. Constipación
d. Vomitos

23. En falla renal aguda por nefrotoxicidad:


a. Se conserva la membrana basal
b. Se destruye la membrana basal
c. Frecuente hay oliguria
d. La creatinina no aumenta

24. Causa de hipomagnesemia:


alcoholismo

25. Na serico de 168 mEq/l 70 kg (utilize como % de agua al 50% y Na normal 140
mEq/l
7 litros

26. Indicaciones de diálisis en falla renal aguda, excepto:


a. Encefalopatia uremica
b. Hiperkalemia refractaria
c. Ac. Metabolica refractaria
d. hipocalcemia

27. El KCL IV se puede asministrar asi, excepto:


a.infusion por linea periferica de 10 meq en 100 ml en 1hora
b. “ “ central de 20 meq em 50ml en 1 hora
c. “ “ central de 100 meq em 50ml en 1 hora → este tampoco me convence
por →El ritmo de infusión habitual no ha de ser superior a 20 mEq por hora. Si bien en
casos urgentes se puede llegar hasta 40 mEq por hora

d. “ “ periferica de 20 meq en 20ml en 2 horas

28. Aumenta la excrecion urinaria de K, excepto:


a. Triamtereno
b. Bumetanida
c. Furosemida
d. Metolazona

29. Factores que regulan la secrecion distal de K, excepto:


a. Flujo tubular distal y aporte distal de Na
b. Excrecion de aniones no reabsorbibles
c. Aldosterona
d. PTH

30. Enfermo de 60 Kg con 60% de agua corporal, Posm 260 mOsm/kg, ¿Cuántos
gramos de NaCl aumentaria el Na en 10 mEq?
a. 21 gramos

50. Conteniendo 10 g de MgSO4 al %, cuantos ml de sal necesitaria?


20 ml

11. La hipocalcemia en la falla renal aguda puede ser asintomática por la:
acidemia 

hipoalbuminemia 

Deficiencia de PTH 

Alcalemia 


12. La hipofosfatemia causa, excepto:


Aumento de vit. D 

Aumento de PTH 

Disminución de la excresión de fosfato 

Aumento de la absorción de fosfato a nivel intestinal 

13. Un pte con diuresis de 250 ml al día con osmolaridad plasmática (presión osmótia) y
osmolaridad urinaria de 300 mosm/l, el dx probable e:
Insuficiencia suprarrenal 

Insuficiencia renal 

Nefritis túbulo intersticial crónica 

Deshidratación hipertónica 


14. Pte con oliguria, hiperkalemia, hiponatremia, creatinina 0.8 mg/dl y CPK normal: Esto
puede corresponder a:
IRA 

Insuficiencia suprarrenal 

Síndrome hepatorrenal 

Rabdomiolisis 


15. No está indicado en caso de hiperkalemia:


r. Aldactone

16. Mujer con hiperemesis grávida:


r. Alcalosis hipoclórica

17. En causa de hipomagnesemia:


r. Alcoholismo.

25. Fórmula del clearence de creatinina: r. clearence = 140 – edad x peso en Kg


72 x creatinina

26. Su infección produce litiasis renal: r. proteus sp.

27. Fórmula de irritabilidad muscular: r. IR = Na + K + OH


( ) + Mg + H

31. Indica falla renal aguda:


U.N. = 4.1 

U.N. = 1 

U.N. = 1⁄4 

Gravedad específica: 1.026 

Ninguna. 


32. Deshidratación por pérdida de agua y densidad urinaria 1.003, creatinina 6:


falla renal aguda parenquimatosa 

Falla pre-renal 

Falla renal obstructiva 

Falla renal crónica 

No hay falla renal. 


33. Uno de los índices de falla renal es:


Reabsorción de Na/ oferta tubular de Na x 100 

Excreción urinaria de Na/ oferta tubular de Na x 100 

Oferta tubular de Na/ excresión urinaria de Na x 100 ** 

Una / pna x 100 

Ninguna 


34. El síndrome heatorrenal es causa de falla:


pre-renal 

renal 

posrenal 

falla cardiaca 

daño tubular 


35. Causa más comun de falla renal cronica:


Diabetes melitus 

HTA 

Glomerulonefritis 

Enfermedad poliquística renal 

Vasculitis

36. En la glomerulnefritis aguda por infección, la causa, excepto: a. proteínas mayor de
10g
b. osmolaridad urinaria normal
c. hematuria
d. HTA e. Edema

A pesar de la diversidad de enfermedades indicadas abajo, ellas tienen muchos síntomas


en común. Con frecuencia, los síntomas y signos observados incluyen: disminución en el
volumen de orina, proteína en la orina (proteinuria), sangre en la orina microscópica o
macrscópica (hematuria), hinchazón (edema), presión sanguínea alta (HTA) y una
disminución en la capacidad del riñón de extraer eficazmente los residuos.
36. Falla renal crónica, excepto:
PTH aumentada 

Hipofosfatemia 

Acidemia 

Hipocalcemia 

e. 1.25 OH (D3) disminuída.

37. En la falla renal aguda siempre habrá:


aumento de creatinina 

fracción de excresión de Na aumentado 

Hiperkalemia 

Una aumentado 

Hipernatremia 


38. Causa de falla pre-renal:


Sd. Nefrótico 

Obstrucción de ambos uréteres 
creo que puede ser esta e caso que la
obstrucción no lleve much tiempo
Falla cardiaca 

Glomerulonefritis aguda pos- infecciosa 

Calculo renal 


39. En la nefritis tubular intersticial se puede dar, excepto:


hipernatremia 

proteinuria de 1.5 g 
→ es masiva
normotension arterial 

hipostenuria 

hipocalcemia 


40. Sugiere glomerulonefritis


leucocituria 

cilindros eritrocitarios 

anuria 

...... 


41. Causas de falla renal parenquimatosa:


papilitis 
ESTA NO, amenos que hayan querido escribir vasculitis
calculo en pelvis derecha 

estenosis unilateral de arteria renal 
→ tiene que ser obstruccion
cistitis hemorragica 

......... 


42. Causa de anuria:


Enfermedad de cambios mínimos 

Necrosis cortical 

Pielonefritis aguda leve bilateral 

Necrosis tubular aguda 

Calculo coloriforme bilateral 


43. La hipofosfatemia causa todo, excepto:


r. Aumento de vit. D

1. Potencia los efectos de la hiperkalemia:


a. Hipocalcemia b. Hipercalcemia c. Alcalemia d. Hipernatremia
e. Hiponatremia

2. La mayor cantidad de magnesio se encuentra en:


a. Hueso b. Hígado c. Musculo d. Eritrocito e. Neurona

3. Hipokalemia con hipertensión arterial, aldosterona elevada y renina baja:


a. Tumor secretor de renina b. Hipertensión Acelerada c. Estenosis de la
arteria renal
d. Hiperaldosteronismo primario e. Síndrome de Uddle

4. En el tratamiento de la hipermagnesemia se puede utilizar:


a. Cloruro de Potasio b. Gluconato de calcio c. Solución Salina al 0.9% con
KCL
d. A y B e. B y C

5. La ausencia de proteínas y elementos celulares en la orina es más compatible


con, excepto:
a. Litiasis Renal unilateral b. Hiperplasia Prostática c. Deshidratación
hipotónica
d. Uropatía obstructiva bilateral e. Falla renal parenquimatosa
6. En la glomerulonefritis aguda post – infecciosa, la filtración glomerular disminuye por:
a. Proteinuria masiva b. Alteración del coeficiente de ultrafiltración
c. Aumento de la presión oncótica d. Disminución de la presión
oncótica e. A y D

7. Hipermagnesemia de 10 mEq/L es de alto riesgo porque puede darse:


a. Abolición de los reflejos osteotendinosos b. Cuadriplejía flácida c. Vasodilatación
periférica d. Bloqueo cardiaco e. Depresión respiratoria

8. En el tratamiento general de la urgencia hipercalcémica se utiliza con prioridad:


a. D/A 5% B. Solución Salina 17,7% c. Solución salina 0.9% d. Bifosfonatos e.
Calcitonina
ambas son correctas
9. Paciente con uremia crónica, deshidratación severa puede presentar, excepto:
a. Oliguria b. Densidad Urinaria 1.010 c. osmolaridad urinaria de 800 mosm/kg d.
Hipercalcemia e. Hiperfosfatemia
10. En la hipokalemia puede darse, excepto:
a. Rabdomiolisis b. Una onda accesoria c. Intoxicación digitalica d. QT prolongado e.
Disminución por amonio génesis
11. Con un nivel de K de 10 mM/L debido a pseudohiperkalemia debe administrarse:
a. kayexalate b. solución hipertónica de glucosa c. bicarbonato de sodio d. gluconato de
calcio al 10% e. Ninguna de las anteriores.
12. Causas de falla renal aguda, excepto:
a. Medios de contraste b. Cilindros Hialinos c. Amino glucósidos d. Cadenas Kappa e.
Mioglobina
13. Es indicador para dializar:
a. Potasio Sérico de 6 mM/L b. Urea sérica de 50 mg/dl c. Náuseas y vómitos
incontrolables
d. Síntomas neurológicos e. C y D
14. cuando disminuye el calcio sérico:
a. disminuye la fosfaturia b. disminuye la resorción ósea c. se inhibe la parathormona
d. aumenta la absorción intestinal de fosfato e. Disminuye la absorción intestinal de
fosfato
15. Calcule el déficit de agua en una paciente de 70 kg con 50% de agua corporal y sodio
sérico de 160 mM/L:
a. 3 L b. 10 L c. 7L d. 5L E. 6L
16. enfermo de 60 kg y 60% de agua corporal, POsm 260 mOms/kg ¿Cuántos gramos de
Na Cl elevarían el sodio en 10 mM/L?
a. 21 b.30 c. 19 d. 26 e. 15
17. La hipocalemia:
a. Aumenta los ruidos intestinales
b. siempre produce cambios electrocardiográficos
c. disminuye la diferencia entre el potencial de membrana y el de umbral
d. aumenta la irritabilidad muscular
e. aumenta la magnitud del potencial de reposo
18. cuando disminuye el fosfato sérico:
a. Disminuye la PTH b. Disminuye la absorción de calcio intestinal c. Aumenta la
fostaturia d. Disminuye el calcitriol e. C y D
19. en causa de hipocalcemia, excepto:
a. Esprue tropical b. parcratitis crónica c. cirrosis biliar d. factor de necrosis tumoral
e. desnutrición
20.
28. paciente con diuresis de 12ml/h, potasio serico 7mM/l, osmolaridad 300 mOsm/l, el
diagnostico mas probable es:
a) SIADH
b) insuficiencia suprarrenal
c) insuficiencia renal
d) deshidratación hipertónica
e) todas las anteriores

29. paciente con vomitos abundantes con hipotensión arterial, pH 7.40; pCO2=41mmHg,
HCO3= 25mM/l; sodio 132mM/l; potasio 3.2m/l; cloro 86mM/l; nitrógeno de urea
40mg/dl:
a) No tiene trastorno acido/base

b) Alcalosis metabolicA severa

c) Acidosis metabolica con acidosis respiratoria

d) Acidosis metabolica con alcalosis metabolica

e) La brecha anionica esta elevada

30. en la hipokalemia se presenta:


a) Disminución de la sístole eléctrica

b) Menor diferencia entre el potencial de umbral y reposo (RESPUESTA


PROFETICA)

c) La onda P alta ( la onda p esta aplanada)

d) El potencial de umbral menos negativa


31. el shock séptico puede presentar:
a) Alcalosis respiratoria con acidosis metabolica con brecha anionica normal

b) Alcalosis metabolica con alcalosis respiratoria

c) Alcalosis respiratoria con acidosis normocloremica

d) Acidosis respiratoria con acidosis metabolica con brecha anionica elevada

e) Ninguna de las anteriores

32. causas de hipercalcemmia, excepto:


a) Uso de laxantes que contiene fosfatos

b) Enemas que contienen fosfatos

c) Administración de fosfato intravenoso

d) Falla renal crónica

e) Enfermedades granulomatosas

33. pH 7.20; HCO3= 8Mm/l; pCO2= 10mmHg:


a) Acidosis metabolica +alcalosis respiratoria

b) Acidosis mixya

c) Acidosis metabolica pura

d) Acidosis metabolica +acidosis respiratoria aguda

e) Acidosis metabolica + acidosis respiratoria crónica

34. paciente con sodio serico de 115mM/l, glucosa 90mg/dl, urea 300mg/dl. Podría
presentar:
a) Agitación

b) Letargia

c) Calambres musculares

d) Todas las anteriores

e) Ninguna de las anteriores

36. hallazgos de laboratorio de nefritis tubulo intersticial


a) Cilindros hemáticos

b) Hematuria

c) Proteinuria masiva (si se presenta es MUY leve)

d) A y b

e) Hipostenuria

37. en toda alcalosis metabolica se da lo siguiente:


a) pH urinario disminuido

b) fase de mantenimiento
c) brecha anionica normal

d) disminución serica de lactato

e) resistencia al cloruro

38. se utiliza para extraer el potasio del cuerpo:


a) bicarbonato de sodio

b) solución hipertónica de glucosa

c) solución hipotónica de glucosa

d) sulfonato de poliestireno (KAYEXALATE™)

e) albuterol

39. alcalosis respiratoria


a) NH4+ y H2PO4 disminuidos en la orina

b) NH4+ disminuido y H2PO4 aumentado en la orina

c) NH4+ y H2PO4 aumentados en orina

d) NH4+ aumentado y H2PO4 disminuido en orina

e) pH urinario de 10

40. la mayor reabsorción de magnesio ocurre en:


a) la porción contorneada del tubulo proximal

b) la rama gruesa ascendente del asa de henle

c) la porción recta del tubulo proximal

d) el tubulo distal

e) el tubulo colector

41. causa de brecha anionica disminuida:


a) hipomagnesemia

b) intoxicación por litio

c) hipocalcemia

d) hipokalemia

e) intoxicación por metanol

42. causas de hipofosfatemia grave, EXCEPTO


a) falla renal

b) recuperación de cetoacidosis diabética

c) alcalosis respiratoria

d) quemaduras graves

e) nutrición parenteral total


43. acidosis respiratoria
a) disminución en la reabsorción proximal del bicarbonato

b) aumento en la reabsorción proximal del bicarbonato

c) disminución del bicarbonato en el eritrocito

d) disminución en la secreción de protones en el tubulo distal

e) ……………urinaria del NH4+

44. sugiere trastorno de tubulo intersticial


a) Perdida de la capacidad para concentrar la orina

b) Hipertensión arterial

c) Edema

d) Proteinuria mayor de 2g

e) Macrohematuria

45. alcalosis metabolica con hipertensión arterial e hipokalemia, EXCEPTO


a) Hipertension maligna

b) Deficiencia de 11 betahidroxilasa

c) Ingestión de biconice

d) Deficiencia de 17 betahidroxilasa

e) Síndrome de Gitelman

46. en una falla renal aguda espero encontrar:


a) Densidad urinaria 1010
b) Densidad urinaria 1020
c) Osmolaridad urinaria 100mOsm/kg
d) Osmolaridad urinaria 8000mOsm/kg
e) A y c

47. Aciduria paradójica se da en


a) Cetoacidosis diabética
b) Alcalosis metabolica leve
c) Acidosis tubular renal
d) Alcalosis metabolica con severa depleción de volumen
e) Acidosis láctica

48. una neoplasia maligna mas comúnmente asociada con hipercalcemia son,
EXCEPTO.
a) carcinoma de mama
b) Hemangioma
c) Carcinoma pulmonar
d) Leucemia
e) Linfoma

49. ph normal con bicarbonato leve:


a) Alcalosis metabolica + acidosis respiratoria
b) Alcalosis metabolica + alcalosis respiratoria
c) Acidosis metabolica + acidosis respiratoria
d) Alcalosis respiratoria + acidosis respiratoria
e) Ninguna de las anteriores

50. plasma: creatinina 1mg/dl; sodio 140mM/l; ORINA: creatinina 150mg/dl;


sodio 40mM/l; la fracción de excresion del sodio es:
a) 2% 𝑠𝑜𝑑𝑖𝑜 𝑜𝑟𝑖𝑛𝑎 𝑐𝑟𝑒𝑎𝑡𝑖𝑛𝑖𝑛𝑎 𝑒𝑛 𝑝𝑙𝑎𝑠𝑚𝑎
b) 0.5% + x 100
𝑠𝑜𝑑𝑖𝑜 𝑝𝑙𝑎𝑠𝑚𝑎 𝑐𝑟𝑒𝑎𝑡𝑖𝑛𝑖𝑛𝑎 𝑒𝑛 𝑜𝑟𝑖𝑛𝑎
c) 0.19%
d) 0.3%
e) 4%
1. un paciente diabético conocido, de 2 años de evolución y 64 años de edad, consulta
por anemia, proteinuria de 3 gramos/24 horas junto con hematuria, hipertensión
arterial y discreta insuficiencia renal con creatinina plasmática de 2.3 mg/dl. El
diagnóstico MENOS probable es:
A. Nefropatía diabética.
B. Angeítis necrotizante del tipo poliangeítis microscópica.
C. Glomerulonefritis rápidamente progresiva.
D. Granulomatosis de Wegener.
E. Síndrome úremico-hemolítico.

2. Si un paciente con una pérdida neuronal progresiva, la presencia de hiperpotasemia


indica que ha perdido al menos:
1) Un 25% del filtrado glomerular.
2) Un 35% del filtrado glomerular.
3) Un 50% del filtrado glomerular.
4) Un 75% del filtrado glomerular.
5) Un 100% del filtrado glomerular.

3. Un paciente de 35 años con insuficiencia renal crónica secundaria a pielonefritis


crónica recibe un trasplante renal de cadáver con el que compartía dos identidades en
Ay B y una en DR. Recibe tratamiento inmunosupresor con ciclosporina A y
corticoides a dosis estándar. En el postoperatorio inmediato se observa buena
diuresis y no es necesario el tratamiento sustitutivo con hemodiálisis. En el 5º día de
evolución, el paciente presenta fiebre de 38º. TA de 180/ 110, oliguria y disminución
en la concentración urinaria de sodio. El diagnóstico más probable sería:
1) Crisis hipertensiva.
2) Infección respiratoria.
3) Pielonefritis aguda del injerto renal.
4) Recidiva de su enfermedad renal.
5) Rechazo agudo del injerto renal.

4. Una paciente de 15 años de edad consulta por poliuria y nicturia, cansancio fácil y
astenia. Presenta dichos síntomas desde hace años, y tienden a intensificarse
durante los veranos muy calurosos, en los que se asocia hormigueo lingual y
peribucal. A la exploración física sólo destaca una tensión de 100/ 50 mmHg. El
ionograma muestra Na 135 mEq/l, K 2 mEq/l. CI 105 mEq/l, pH 7.45, bicarbonato 30
mEq/l. La determinación de renina y aldosterona muestra cifras elevadas, tanto
basales como tras estímulo. De los procesos que siguen ¿cuál es compatible con el
cuadro clínico descrito?:
1) Hipoaldosteronismo primario.
2) Estenosis de la arterial renal.
3) Síndrome de Liddle.
4) Enfermedad de Addison.
5) Síndrome de Bartter.

5. ¿Cuál de las siguientes medidas terapéuticas NO está indicada en el tratamiento de la


Hiperpotasemia?:
1) Glucosa e insulina.
2) Bicarbonato sódico.
3) Gluconato cálcico.
4) Resinas de intercambio iónico.
5) Glucocorticoides.
6. ¿Cuál de las siguientes nefropatías glomerulares no se relaciona con
hipocomplementemia?:
1) Glomerulonefritis aguda postinfecciosa.
2) Glomerulonefritis membrano-proliferativa.
3) Nefropatía lúpica.
4) Nefropatía diabética.
5) Nefropatía de la crioglobulinemia.

7. Un paciente de 28 años presenta hematuria macroscópica al día siguiente de una


infección faríngea y edemas maleolares. Dos años antes había presentado un cuadro
similar que desapareció con rapidez, por lo que no había consultado previamente. En
la exploración física se objetiva una T.A. de 180/110, una Creatinina Plasmática de 2
mgrs/dl e Hipocomplementemia. ¿Cuál de los siguientes es el diagnóstico más
probable?:
1) Glomerulonefritis membrano-proliferativa.
2) Glomerulonefritis post-estreptocócica aguda.
3) Glomerulonefritis rápidamente progresiva.
4) Glomerulonefritis por depósitos mesangiales de IgA (Enfermedad de Berger).
5) Hialinosis Segmentaria y Focal.

8. Niña de 3 años, que presenta afectación brusca del estado general dentro del
contexto de un cuadro febril con diarrea mucosanguinolenta. A la exploración aparece
pálida y soñolienta, la auscultación cardiopulmonar es normal, el abdomen doloroso y
el resto del examen físico sólo revela la presencia del petequias puntiformes
diseminadas. La orina es hematúrica y se constata hipertensión arterial. El
hemograma muestra HB 7 g/dl, 17.000 leucocitos con neutrofilia, 37.000 plaquetas
con normalidad de las pruebas de coagulación. ¿Cuál es el diagnóstico más
probable?:
1) Sepsis por Salmonella.
2) Púrpura de Schönlein-Henoch.
3) Glomerulonefritis aguda post-infecciosa.
4) Síndrome hemolítico-urémico.
5) Coagulación intravascular diseminada.

9. Se trata de un niño de 7 años, con retraso estaturo-ponderal, que presenta signos


radiológicos de raquitismo y ecográficos de nefrocalcinosis. Los análisis demuestran
acidosis metabólica con PH inferior a 7.32 y Bicarbonato plasmático inferior a 17 mEq/
litro. El PH de la orina en 3 determinaciones es siempre superior a 5.5. Otras
alteraciones metabólicas consisten en hipercalciuria, hipocitruria y discreta
disminución de la resorción del fósforo. No se detecta glucosuria ni aminoaciduria. De
las siguientes posibilidades, señale el diagnóstico correcto:
1) Síndrome de Bartter.
2) Acidosis tubular distal (Tipo I).
3) Acidosis tubular proximal (Tipo II).
4) Síndrome completo de Fanconi.
5) Enfermedad de Harnup.

10. Paciente de 50 años de edad, que consulta por dolor en la fosa renal, polaquiuria,
disuria y hematuria. En el análisis de la orina se observa piuria y pH ácido con cultivos
repetidamente negativos. ¿Cuál sería la primera posibilidad diagnóstica, de entre las
siguientes?:
1) Pielonefritis aguda.
2) Síndrome nefrítico.
3) Tuberculosis genitourinaria.
4) Prostatitis aguda.
5) Carcinoma renal de células claras.

11. ¿En qué tipo de pacientes está indicado realizar detección y tratamiento de bacteriuria
asintomática?:
1) Pacientes con sondaje vesical permanente.
2) Gestantes.
3) Pacientes mayores de 65 años.
4) Pacientes ingresados en el hospital.
5) Paciente prostatectomizados.

12. Sobre colangitis Esclerosante I asociada con Enfermedad Inflamatoria Intestinal es


falsa:
Se presenta con mayor frecuencia con Enf. De Crohn que con Colitis Ulcerativa
13. En un paciente alcohólico con ascitis por cirrosis hepatica, tiene hiponatremia
serica con hiperadolesteronismo, la fisiopatología de esta enfermedad es por:
Aumento de producción de renina por hipovolemia

14. Px de 60 años con dx de colangitis se le inicia tx con antibióticos empírico contra


bacterias entericas y la fiebre no cede en 48 horas, la conducta seria:
proceder inmediatamente al tx quirúrgico

15. Las pérdidas renales excesivas de potasio:


tumor velloso

16. Px de 60 años con ascitis por ingesta de alcohol, investigar si se trata de cirrosis
hepática, para su dx es necesario:
TP

17. Px de 30ª acude por diarrea crónica ligeramente, al examen físico esta normal
excepto que la región anal presenta fistulas, debido a esto ud pensaría:
Enfermedad de Crohn

18. La PTH promueve todo lo siguiente excepto:


Aumento de la absorción tubular de fosfato

19. Varón de 57a esta con hemodiálisis de mantenimiento para insuficiencia renal
crónica. Cual de las siguientes anormalidades metabólicas podría ser anticipada?
Osteomalacia

20. Virus de dengue:


Arbovirus

21. En la intoxicación de OF cual de los siguientes efectos no es:


Anticolinergico

22. Estadio dx de Churg Strauss:


Ninguna

23. Indica falla renal aguda:


UN = 1

24. Síndrome hepatorenal es causa de falla:


Prerenal

25. Síndromes Hipercoagulabes, excepto:


Sepsis

26. Antibiótico contra Listeria Monocitogenes:


Ampicilina

27. Hepatitis C, periodo de incubación;


20- 110 días

28. Cristal de birrefringencia negativa en forma de aguja a la luz polarizada:


Pirofosfato de calcio
Cristales de urato/ Monourato Sodico

29. Factor que predispone a infecciones en pacientes falcémico:


Falla de opsonizacion

30. Factor que predispone a infección en pacientes diabéticos


Alteración en la adhesión de los granulocitos

31. Señale la enzima que no es de utilidad en el dx de polimiositis:


Transaminasa glutaminopiruvica

32. La crisis de gota se debe:


Precipitación de uratos
33. Indicaciones de densitometría ósea, excepto:
Menopausia

34. Los marcadores bioquímicas para osteoporosis están indicados solamente en:
seguimiento de la efectividad del tratamiento de osteoporosis

35. Las Ac antifosfolipidos producen sobre las plaquetas:


Aumento de la actividad de plaquetas

36. Es cierto par a los Ac antifosfolìpidos:


Test de Coombs directo positivos

37. Los nódulos de Bouchard son denominación de artrosis de:


Interfalangica distal

38. En el LES:
EL ANA es positivo en 95% de los pacientes

39. En artritis reactiva:


Suele estar asociada a HLA-DR27

40. Paciente con artritis, leucopenia y esplenomegalia:


Síndrome de Felty

41. Droga que es síntesis de laboratorio:


Tramadol

42. En Wegener:
C-ANCA es de 90%

43. Paciente de 84 anos, con erupciones vesiculares en forma de cinturon, su nieto


puede tener:
Varicela

44. Pte. con dengue dar:


Acetaminofen, reposo, nutrición parenteral y trato ambulatorio

45. Tratamiento de Pneumocystis Carinii:


TMP-SMX

46. Hombre con fractura de cráneo, osm 800:


Diabetes insípida central

47. Raro en dengue:


Epistaxis- tos

48. Penicilina sigue siendo efectiva para:


peumococos

49. Para prevenir necrosis:


Alta hidratación parenteral

50. el dengue hemorragico es una enfermedad grave con trombocitopenia y shock


que se diferencia del dengue clásico por no presentar:
Mialgia y dolores articulares

51. Marcados de LES:


Anti-DNA y anti-smith

52. FR- se une a porción Fc de IgM

53. Anemia de las vasculitis:


normocitica normocromica

54. Polimialgia reumática:


Artritis Temporal
55. Hombre de 57 años con entumecimiento…. Livedo reticular:
Poliarteritis nodosa

56. Exámenes:
Velocidad de Conducción Nerviosa
Transaminasas
Sedimento Urinario

57. Tx:
Corticoides

58. Si no evoluciona:
Ciclofosfamida

59. S Nefrosico:
Cilindros cereos

60. Falla renal crónica avanzada:


Brecha aniónica elevada

61. Factor mas importante de estimulacion osteoclastica:


IL-1

62. Disentería Amebiana:


Acidosis e hipokalemia

63. Fenómeno de Raynaud:


Esclerosis Sistémica

64. Criterios de Lupus:


ANA +,
Rash malar
Discoide

65. Hiperemesis Gravidica:


Alcalosis hipoclorica

66. Efectos simpaticomimeticos de cocaína:


hipoglicemia

67. Con respecto a la adicción de las drogas uno de los siguientes enunciados es
falso:
La cafeína se considera un estimulante mayor

68. Con respecto a la neurobiología de la adicción es correcto:


Las anfetaminas son dopaminergicas

69. Agente de S Diarreico tipo 1


Staph Aureus

70. La elevación de alfafetoproteina serica puede detectarse en todas excepto:


Carcinoma testicular

71. La poliarteritis nodosa generalmente respeta: Pulmón

72. Paciente de 30 años mordido por serpiente venenosa y mitad de la pierna con
edema: Síndrome bothropico
.
73. Cual de los siguientes antibioticos tiene mejor cobertura contra klebsiella
cefotaxima

74. Periodo de incubacion de chlamidia: 7 a 21 días

75. No es subjetivo de hantavirus: VES normal

76. En la encefalopatía hepática existe excepto: Inhibición gabaergica


77. Nicotina cierta: Tiene receptores específicos

78. paciente de 25 que llega al cuarto de urgencia con sangrado digestivo bajo con 3
dias de evolución hubo febricula, usted piensa en los siguientes diagnosticos
excepto: Síndrome de colon irritable
:
79. Es cierta para osteoporosis excepto: Se inicia la perdida a los 60 años en el hueso
cortical

80. Cual de estos no es betalactamico: Vancomicina

81. Arterias musculares de mediano calibre: Granulomatosis de Wegener

82. Fracturas más frecuentes en osteoporosis: Vertebras

83. Cual es la meta para el tratamiento de pacientes con VIH: Suprimir la carga viral

84. Falso positivo en VDRL a los 6 meses: Malaria

85. 76 trastorno metabolico causado por vibrio cólera: Acidosis metabolica:


Clindamicina

86. el paciente con esófago de barret es cierto

Tiene años con reflujo tiene un riesgo de adenocarcinoma del cardias de 10%
necesita vigilancia endoscopica con biopsia de 24 meses.

87. Dosis de los fármacos para la TBC


Izoniacida: 5 mg
Rifampicina: 10 mg
Etambutol: 15 a 25 mg
Pirazinamida: 25 a 30 mg

Pareo
• Pirazinamida: hiperurecemia y artralgia
• Etambutol: neuritis óptica
• Anfotericina B: hipokalemia
• Izoniacida: hepatitis y colestasis
• Rifampicina: compromiso hepático

Llena espacio
88. Factores predisponentes a candidemia:
Nutrición parenteral
Neutropenia
Inmunosuprimidos
Uso prolongado de antibioticos

89. Microscopia con luz polarizada:


Gota
Pseudogota

90. Para que se utiliza el analisis del líquido sinovial


Artritis séptica
Artritis por micro cristales
La hipokalemia produce:
- mayor negatividad del potencial de reposo.
- Aumento de la magnitud del potencial de reposo

La hipokalemia en un cirrótico puede precipitar COMA por:


- aumento de la amniogénesis

Exceso del kayaxelate en el tratamiento de hiperkalemia puede producir:


- constipación.

Peligro de la hipermagnesemia severa:


- paro respiratorio.
Hipofosfatemia:
- aumenta el calcitriol.

Consecuencia metabólica de hipofosfatemia severa:


- miocardiopatía

Causa de hiperfosfatemia:
- insuficiencia renal

El Síndrome Nefrósicose caracteriza por:


- proteinuria mayor o igual de 3g y albúmina sérica menor o igual de 3g/dl.

En el Síndrome Nefrósico pueden encontrarse:


- cilindros céreos

Urémico agudo, oligúrico, acidótico, deshidratado puede presentar:


- osmolaridad urinaria de 250mosm/kg

La falla renal crónica avanzada cursa con:


- brecha aniónica elevada

Cuál de los siguientes antibióticos produce nefritis intersticial:


- meticilina

El factor más determinante en el tratamiento de la hipokalemia renal es:


- la función cardiovascular.

El tratamiento de inicio de secreción inapropiada de ADH es:


- restricción hídrica

. Plasma: creatinina 1mg/dl, 140 mEq/L de Na. Orina: creatinina 150 mg/dl, Na 40
mEq/L la FE Na es:
0.19% (La fórmula es: FE Na= (Na Orina x Creatinina plasmática) / (Na plasmático x Creatinina
Orina) Y lo multiplicas por 100 para que de en %
FE Na= (40 x 1)/ (140 x 150) TODO por 100 = 0.190%
2. Factor determinante en tratamiento de hipokalemia real es:
R: función cardiovascular.
Recordar que el potasio es el principal actor en el potencial de relajación transmembrana así
que el paciente corre el gran riesgo de padecer arritmias; por ende en los pacientes con
insuficiencia cardiaca congestiva se da IECA para atenuar la hipokalemia inducida por
diuréticos y brinda protección contra arritmias cardiacas.
3. Tratamiento de inicio de SIADH es:
R: restricción hídrica
El tratamiento de inicio de la hiponatremia euvolémica es la restricción de H2O hasta <
1L/día, sin embargo a veces es inefectiva o mal tolerada. EXTRA: El tratamiento óptimo es
la corrección del trastorno subyacente.
4. Paciente con alcalosis metabolica e hipokalemia debe ser tratada con:
a) KCl
Esto se debe a que en el LEC hay una disminución de H+ y salen hidrogeniones del LIC al
LEC para compensar la alcalosis metabólica (disminución en [HCO3]) posterior a esto entra
un K+ del LEC para mantener el equilibrio de las cargas en ambos lugares.
Sabiendo esto el tratamiento con KCL es complementado con la corrección patológica
subyacente (diarrea) o retiro del medicamento nocivo (diurético de asa o tiazídico).
Entonces si no es posible o tolerable la complementación oral, puede administrarse KCL
I.V. a través de una vena central con vigilancia cardiaca, a un ritmo < 20 mmol/h. El KCL
siempre debe administrarse en soluciones salinas, no con glucosas porque el aumento en la
insulina inducido por glucosa causa exacerbación aguda de hipopotasemia porque la
insulina en su metabolismo introduce el potasio sérico a nivel intracelular lo que agrava aún
más el asunto.
5. Causa de hiperkalemia
a) aumento de volumen arterial eficaz
b) hipercatabolismo tisular
c) hiperaldosteronismo
d) alcalosis metabolica
e) cirrosis hepática
R: Hay una amplia clasificación de las etiologías de la hiperkalemia y aquí redacto las
principales y sus sub-divisiones:
I. Pseudohiperkalemia
Salida de K+ de la célula (trombosis, eritrocitosis, leucocitosis y hemolisis in vitro.
(Aquí es donde está la respuesta de arriba)
Defectos hereditarios en la membrana de transporte eritrocitaria.
II. Desplazamiento del interior al exterior de la célula.
Acidosis
Hiperosmolalidad, medio de contraste radiográfico
Antagonistas adrenérgicos B
Digoxina y Glucósidos relacionados
III. Excreción inadecuada
Inhibición del SRAA. Aumenta el riesgo con IECA y ARA.
Disminución del aporte distal
Hipoaldosteronismo hiporreninémico
Resistencia renal a mineralcorticoides
Insuficiencia renal avanzada con velocidad de filtración glomerular disminuida.
Insuficiencia suprarrenal primaria
6. Causa de hipomagnesemia
a) cirrosis avanzada
b) falla renal cronica
c) falla cardiaca
d) constipación
e) acidosis respiratoria
Esta alteración no es muy común y sus causas son: falta de aporte (desnutrición), trastornos
del tracto digestivo (S. mal absorción, S. intestino corto, etc.) y los alcohólicos crónicos en
donde se presenta cirrosis. Y en estos casos no se presenta de forma aislada, sino junto a la
hipocalemia, hipofostatemia, hiponatremia.
7. Causa renal de hipomagnesemia
a) falla renal cronica
b) esteatorrea
c) desnutrición severa
d) uso de diuréticos de tiazidas. También Alcoholismo
e) infeccion urinaria
R: Las tiazidas causan hipomagnesemia por el siguiente mecanismo: Este medicamento
trabaja a nivel del túbulo contorneado distal inhibiendo el simportador de Na+/CL- en la
membrana apical de la célula; por ende lo que sucede es que al no permitir el paso de sodio
por su canal este se acumula en la membrana apical gormando una barrera catiónica y
obstruyendo a los canales de Mg y Ca por consiguiente todos estos iones son positivos y los
positivos se repelen es por eso que no se puede reabsorber el Mg y Ca por la obstrucción de
la barrera catiónica formada por el sodio y además a este nivel no se reabsorbe Mg ni Ca
por los espacios intercelulares pues carecen de canales y de gradiente eléctrico para poder
ser reabsorbidos.
8. En caso de hipercalcemia pensar en:
a) neoplasia
b) hiperfosfatemia severa
c) nefrolitiasis
d) nefrocalcinosis
e) diureticos
Primero saber que: Hiperparatiroidismo primario (más frecuente en pacientes
ambulatorios), la hipercalcemia de origen tumoral y la hipervitaminosis D (por iatrogenia),
abarcan el 90% de las causas de hipercalcemia.
La hipercalcemia se relaciona con el hiperparatiroidismo secundario a un adenoma de la
glándula tiroides que hipersecreta PTH, entonces es cierto que la PTH sintetiza calcitonina
que su trabajo es disminuir la concentraciones de Ca, sin embargo solo la PTH trabaja a
nivel renal reabsorbiendo calcio y como hay una hipersecreción de la misma el calcio sérico
aumenta y se tiende al riesgo de formar cálculos renales calcio.
9. El síndrome nefrótico puede encontrarse cilindros:
a) cereos
b) hematicos
c) leucocitarios
d) epiteliales
e) mixtos
R: Aparte el síndrome nefrótico se caracteriza por albuminuria (>3.5 g/día) e
hipoalbuminemia (<30g/L) asociado a edema, hiperlipidemia y lipiduria (En respuesta a la
hipoproteinemia, el hígado pone en marcha un mecanismo compensatorio consistente en la
síntesis de una mayor cantidad de proteínas, tales como alfa-2
macroglobulina y lipoproteínas, siendo estas últimas las causantes de la hiperlipemia que
incrementa el riesgo cardiovascular en estos pacientes)
10. U Na 115, Uosm 240. P osm 680, P Na 62
SIADH
Esto está así solo estos datos: Bueno lo que se sabe del SIADH es que el UNa debe ser > 20
mmol/L y que la Uosm debe ser > 600 mosm.
11. Paciente de 84 años que recibe diureticos para la hipertensión arterial. Presenta
diarrea y trastornos de sensoriales y disminución del rubor de la piel. Na 174 U Na 5
U osm 606.
La hipernatremia se debe a: Perdidas insensibles
12. Hipernatremia hipervolemica
Hiperaldosteronismo primario o Síndrome de Cohn
R: Aumento de Na+ más que el aumento de agua. Corrección de una hiponatremia,
NaHCO3, solución usada en diálisis. Tratamiento Diurético + Agua. Acidosis metabólica
con bicarbonato.
Se presenta un adenoma a nivel de las glándulas suprarrenales que hipersecreta aldosterona,
por consiguiente se retiene más sodio y agua y se da una gran pérdida de potasio. Esto
conlleva hipernatremia con hiperosmolalidad e hipervolemia y trastornos cardiacos como
las arritmias.
13. causa de hipomagnesemia
Alcoholismo. También uso de diuréticos
R: Esta alteración no es muy común y sus causas son: falta de aporte (desnutrición),
trastornos del tracto digestivo (S. mal absorción, S. intestino corto, etc.) y los alcohólicos
crónicos en donde se presenta cirrosis. Y en estos casos no se presenta de forma aislada,
sino junto a la hipocalemia, hipofostatemia, hiponatremia.
14. hipocalcemia
Provoca potencializacion de efectos de hiperkalemia
15. En hipocalcemia
Prolonga QT a expensas de segmento ST
Lo de arriba mencionado solo sucede cuando la hipocalciemia es intensa además se asocia a
convulsiones, espasmo carpopedal, broncoespasmo, laringoespasmo
Aparte la hipocalciemia puede no mostrar síntomas si la disminución del calcio sérico es
relativamente leve y crónica. La hipocalciemia moderada o grave se acompaña de
parestesias, por lo común de dedos de manos y pies y zonas peribucales, y es causada por
una mayor irritabilidad neuromuscular.
16. Tratamiento de urgencia hipercalcemica
Solucion salina isotonica 0.9% 4 a 5 L
Saber además lo siguiente: La hipercalciemia asintomática leve por lo común no necesita
tratamiento inmediato y éste debe depender del diagnóstico principal. A diferencia de la
sintomática que obliga a una intervención terapéutica, sea cual sea el origen del exceso de
calcio. Las medidas iniciales contra la hipercalciemia notable comienzan con la expansión
volumétrica, porque el problema invariablemente culmina en deshidratación se necesitan en
las primeras 24 h 4 a 6 L de solución salina intravenosa, sin olvidar que cuadros
coexistentes primarios (como insuficiencia cardiaca congestiva) obligan a veces a emplear
diuréticos con acción en asa de Henle para intensificar la excreción de sodio y calcio esto
pasa por que los diuréticos de asa actúan inhibiendo el cotransportador de Na/K/2CI lo que
forma en una barrera catiónica en la membrana apical de la célula del asa ascendente de
Henle e impide que el Ca y Mg sean reabsorbidos por la tight junctions (zonas de unión)
porque ambos son positivos y estos con el sodio se repelen evitando así su reabsorción.
31. Síndrome nefrotico
Edema, proteinuria (>3.5 g/día), hipertensión arterial, hematuria (cilindros céreos),
hiperlipidemia y lipiduria
Las últimas dos se deben a lo ya explicado del siguiente proceso hepático: En respuesta a la
hipoproteinemia, el hígado pone en marcha un mecanismo compensatorio consistente en la
síntesis de una mayor cantidad de proteínas, tales como alfa-2
macroglobulina y lipoproteínas, siendo estas últimas las causantes de la hiperlipidemia que
incrementa el riesgo cardiovascular en estos pacientes.

1) Cuál de las siguientes medicinas pueden dar hiperkalemia


a) inhibidor de la enzima convertidota
b) bloqueador alfa adrenérgico
c) bloqueador beta adrenérgico
d) a y b
e) a y c

Los betabloqueantes no selectivos han sido asociados con el desarrollo de


hiperkalemia, aunque ésta no suele ser grave. Los betabloqueantes suprimen la
liberación de renina estimulada por catecolaminas, con lo cual se reduce la síntesis de
aldosterona. Además, y más importante aún, estos fármacos disminuyen la captación
celular de potasio. Normalmente, la unión de un agonista a un receptor b-adrenérgico
estimula la formación de AMP cíclico; éste activa la bomba Na+-K+-ATPasa que
promueve la entrada de potasio a la célula. La inhibición competitiva del receptor b-
adrenérgico reduce la función de esta bomba y, por consiguiente, la captación celular
de potasio.
Los inhibidores de la enzima convertidora de angiotensina (ECA) producen
hiperkalemia al inducir un estado de hipoaldosteronismo. La inhibición de la ECA
también causa deterioro -en la excreción renal de potasio al reducir la velocidad
efectiva de filtración glomerular en pacientes con depleción de volumen, estenosis
arterial renal o insuficiencia renal crónica. En estos pacientes, explica el autor, los
inhibidores de la ECA suprimen la constricción arteriolar posglomerular inducida
por la angiotensina II. Esto conduce a una reducción en el aporte de sodio y agua al
nefrón distal, lo que -sumado al hipoaldosteronismo- puede promover hiperkalemia.
Los inhibidores de la ECA producen entre el 9% y el 38% de los casos hospitalarios
de hiperkalemia.

2) Hormonas que disminuyen en la falla renal crónica, excepto


a) 1,25 dihidroxi vitatamina D
b) eritropoyetina
c) paratohormona
d) foliculo estimulante

El diagnóstico se basa en la clínica que presente el paciente así como en las


alteraciones analíticas. En la analítica de sangre se observa un aumento de la urea por
encima de 40 mg/dl, un aumento de la creatinina por encima de 1,2 mg/dl, un descenso
de los niveles de hemoglobina, hematocrito, sodio y calcio, y un aumento del potasio, el
fósforo y el magnesio en sangre, así como de la PTH.
3) MgSO4 (PM 120) 1 gramo contiene de magnesio en meq
a) 10
b) 7.9
c) 8.3
d) 6
4) Factores que conducen a la progresión de falla renal crónica
a) hiperlipidemia
b) toxinas urémicas
c) proteinuria
d) b y c
c) todas
5) Paciente de 20 años presenta abundantes vômitos y diarrea de 3 días de evolución,
examen físico depleción de volumen, suero muestra Na 155, K 3meq/l, Cl 117meq/l,
HCO3 25 meq/l, tratamiento de elección:
a) D/A 5% + KCl
b) solución salina 3% + KCl
c) solución salina 23.4% + KCl
d) solución salina 0.45% + KCl

6) En la enfermedad ateroembólica renal se da lo siguiente excepto:


d) Velocidad de eritrosedimentación (esta aumentada)

En un 50% de los pacientes se observa afectación renal en forma de insuficiencia renal


aguda, generalmente no oligúrica. En el momento del diagnóstico el 25% de los
pacientes tienen una creatinina sérica mayor de 5 mg/dl, precisando diálisis casi el
40% de ellos, y hasta un 80% mayor de 2 mg/dl1 . El análisis de orina no suele ser
diagnóstico, aunque en algunos casos se observa proteinuria discreta (1 g/día) y un
sedimento con hematuria, leucocituria y cilindros gránulo-hialinos1,8. En los
primeros días se puede observar eosinofiluria con tinción de Hansel en el sedimento y,
en la sangre, eosinofilia, leucocitosis, anemia y aumento de la velocidad de
sedimentación eritrocitaria.
7) Pseudohiperkalemia de 8 mEq/l que se ve en el EKG:
Onda T picuda
Aplanamiento de la onda P
Ay B
Ninguna

8)Criterios para iniciar diálisis excepto:


Alcalosiss Metabolica

Indicaciones agudas para Diálisis o hemofiltración:

• 1) Hiperpotasemia
• 2) Acidosis metabólica
• 3) Sobrecarga de fluido (que usualmente se manifiesta como un edema pulmonar)
• 4) Pericarditis urémica, una potencial complicación que amenaza la vida en una falla
renal
• 5) Y en pacientes sin falla renal, envenenamiento agudo con toxinas dializables, como
el litio

9)Hipokalemia con normotension excepto


Síndrome de liddle
El síndrome de Liddle, también llamado pseudoaldoesteronismo,1 es una enfermedad
con herencia autosómica dominante caracterizada por hipertensión temprana y
severa, asociada a bajos niveles de renina y de aldosterona además de alcalosis
metabólica e hipocalemia.
Normotensión: diuréticos, síndrome de Bartter, deplección grave de potasio

10) Plasma creatinina 8mg/dl N de urea 100mg/dl U na 40 Meq/l :


Insuficiência renal
11) Factores reversibles responsables de deterioro de la funcion renal excepto:
Fibrosis intersticial
12) Causa hipokalemia con renina y aldosterona disminuida:
Licorice
13. Sugiere trastorno túbulo intersticial:
.Proteinuria mayor a 3.5 g /24 horas
.Notrmotensión
.Edema
.Osmolarida urinaria normal
14. Glomérulonefritis rápidamente progresiva
.Engrosamiento de la membrana basal
Fusión de los pedículos de los podocitos
prolñiferacion intracapilar
proliferación extracapilar
Anatomopatológicamente, el concepto de GNRP se asocia a la presencia de proliferación
extracapilar como consecuencia de la ruptura de la membrana basal glomerular que
permite el paso de fibrina y monocitos al espacio de Bowman
15.Una de las siguientes causas no produce hematuria
.enfermedad de cambios mínimos
16.Plasma Na:115, Posm240 mOsm,Uosm 680,Una 60:meq/l:
.Deshidratación hipotónica
.secreción inapropiada de ADH
.Deshidratación isotónica
.Deshidratación primaria
17.en la falla renal crónca produce necrosis
calcifilaxia
18. No produce hipokalemia:
Pancreatitis aguda
rabdomiolisis
neoplasia
Def. Vit D
19. Paciente con glucosa 90mg/dl, Na+= 115mEq/L, Urea en 140mg/dl
a. Síndrome hipoosmolar
b. Síndrome hiperosmolar
c. Diarrea
d. Ninguna de las anteriores.
20. Causa de seudohiponatremia con osmolaridad elevada
a. Hiperproteinemia
b. Diabetes mellitus
c. Hiperlipidemidemia
d. Reserción transuretral prostática.
21. Manifestaciones de hipermagnesemia excepto
Normotensión
22. Piel en uremia crónica excepto
Paño blanco
23. No es causa de necrosis papilar aguda
a.Diabetes mellitas
b. Uropatía obstructiva
c. Pielonefritis
d. Trombosis de venas renal.
24. Puede presentarse hiperuricemia excepto
d. Adenocarcinoma gástrico
25. Compromete riñones y pulmones:
a. Enf de GoodPasture
b. Granulomatosis de Wegener
c. LES
d. Ay C
26. Sugiere glomerulopatia:
-anasarca
27. TFG < 15ml/min corresponde al estadio de la enfermedad renal cronica:
*5
Etapa 1 — Normal o alta TFG (TFG> 90 ml/min)
Etapa 2 — ERC leve (TFG = 60-89 ml/min)
Etapa 3 — ERC moderada (TFG = 30-59 ml/min)
Etapa 4 — ERC grave (TFG = 15-29 ml/min)
Etapa 5 — ERC terminal (TFG <15 ml/min)
28. Causa obstetrica de necrosis cortical bilateral
- desprendimiento prematuro de placenta
29. Promueve amoniogenesis em cirróticos
- hipokalemia
30. Criterio para diagnostico de síndrome nefrosico
a. Proteinuria en 24 hr de 3.5 gr para superficie corporal de 1. 73
b. Albumina serica < 3g/dl
c.Hiperlipidemia y edema
d. Todas las anteriores
e. ay b

Criterios diagnósticos del SN

• Proteinuria >3-3,5g/24 horas o una relación proteína:creatinina >3-3,5 (ambas en mg/dL)


en una muestra aislada
• Albuminemia <25 g/L
• Edemas periféricos
• Es frecuente la hiperlipidemia severa (colesterol total >385 mg/dL)

31. Uno de los siguientes factores no es de riesgo del higado graso alcoholico:
-Ingesta alta de carbohidratos
Causas
No hay una causa clara para la EHNA. Sin embargo, ciertos factores pueden ponerlo
en riesgo:

• Sobrepeso u obesidad. Cuanto mayor sea su sobrepeso, mayor será su riesgo.


• Prediabetes (resistencia a la insulina)
• Diabetes tipo 2
• Colesterol alto
• Triglicéridos altos

• Hipertensión arterial

Otros factores de riesgo pueden incluir:

• Una pérdida rápida de peso y una mala dieta

• Cirugía de derivación gástrica

• Enfermedad intestinal

• Ciertas medicinas como los bloqueadores de los canales de calcio y algunos


medicamentos para tratar el cáncer
La EHNA también se presenta en personas que no tienen factores de riesgo conocidos.

32. El siguiente no es un factor de riesgo para cancer de colon:


-acido alcohol salicilico
33. Uno de los siguientes síntomas es menos probable en pancreatitis aguda:
-diarrea
¿Cuáles son los síntomas más habituales?

Dolor abdominal
Náuseas y vómitos
Fiebre
Distensión abdominal
34. Cual de los siguientes no es un factor de riesgo para hepatocarcinoma:
-helicobacter pylori
35.Child – Pugh, excepto:
-Transaminasas
36. Cirrosis hepatica, un examen que debe de realizarse siempre es:
-endoscopia alta

En los pacientes con cirrosis la endoscopia digestiva alta sirve para lo


siguiente:

Diagnóstico de las varices esofágicas

Tratamiento de la hemorragia por varices

Prevención de la primera o de sucesivas hemorragias por varices

37. …..de Insuficiencia hepatica aguda grave, en la prod. por intoxicación con
paracetamol, hay que administar:
-n-acetil-cisteina
se llegó a la publicación en 1974 de artículos que indicaban sustancias regeneradoras
del glutation (donantes de grupos sulfidrilo), como la L-cisteina, L-metionina,
cisteamina y la N-acetilcisteina (NAC). En años posteriores se demostraría su utilidad
clínica.

38.Todo es cierto de cancer gastrico , excepto:


-La mayoria de los pacientes tienen gastritis cronica atrofica
39. Generalmente la lipasa pancreatica no esta aumentada en:
-colitis amebiana
40.Cual es el factor responsable de la duracion de la secrecion pancreatica:
-Duracion del vaciamiento gastrico
41.Es falso con respecto al higado graso no alcoholico
-no evoluciona a cirrosis hepatica
42. En cuanto al cancer gástrico:
La endoscopia digestiva alta es el mejor estudio diagnóstico
43.falta era profeta
44. La esteatorrea en pancreatitis cronica aparece cuando el páncreas…… en un :
-90%
45. el flapping tremor:
c) es un signo de encefalopatia hepática
Término en desuso que se emplea para denominar los movimientos aleteantes que
aparecen en pacientes con encefalopatía hepática, al extender las muñecas y los dedos, y
que son debidos a la presencia de asterixis… La asterixis (del Griego a + stērig, "no
sujetar") es un término médico usado para describir una alteración neuromuscular que
consiste en la aparición involuntaria de interrupciones rítmicas de una contracción
muscular voluntaria. Adecuadamente llamado temblor

46. el antigeno carcinoembrionario es util en el estudio de pacientes:


c)es eficaz para su detección
47. La hipertrofia paratiroidea se observa:
c)cirrosis hepática
48. la enfermedad de Crohn :
b) snd de colon irritable
49 y 50 Nombre 2 manifestaciones extrahepaticas de hepatitis viral:
1. Crioglobulinemia
2. Anemia
1. La PTH promueve todo lo siguiente excepto:
o EL aumento de la reabsorción tubular de fosfato. (Estimula la reabsorción de
calcio en el túbulo distal e inhibe la de fósforo en el túbulo proximal)La PTH a
nivel renal es eminentemente fosfatúrica, promueve la excreción de fosfato y la
reabsorción de Calcio.
2. Síndrome nefrósico de cambios mínimos se caracteriza por:
o Fusión de los pedicelos de los podocitos. (La anormalidad característica del
podocito es la completa fusión de los pedicelos, . La fusión de los pedicelos no
se puede ver en microscopia de luz, hay que usar microscopia electrónica, pero
se puede hacer el dx con la proteinuria masiva característica del síndrome.
SNCM se caracteriza por alteraciones en los podocito)
3. En un paciente con eclampsia se le ordeno un litro de solución de dextrosa al 5%,
conteniendo 20 ml de MgSO4 al 50% (PM= 100 g), ¿cuantos ml de sal se necesita para
preparar la solución?
o 10 ml.
4. El porcentaje de solución es:
o 1%
5. ¿Cuantos mEq de Mg tiene la venoclisis?
o 83.3
6. La hipocalcemia de la falla renal crónica puede cursar:
o Asintomática.
7. La velocidad de administración de KCL en un caso de alto riesgo puede ser:
o 40 mEq/L (En los pacientes con una hipokalemia muy severa se usa 40, pero se
debe pasar a una velocidad de 20, porque si se pasa muy rápido puede ocasionar
un bloqueo AV completo. )
8. Paciente urémico, con K sérico de 6, Academia, EKG normal se trata con:
o Kayexalate. (Ya que la hiperkalemia no es tan severa se puede tratar con el
Kayexalate, que es una resina de intercambio catiónico, la misma remueve el
potasio del cuerpo.)
9. Paciente femenina de 50 Kg presenta diarrea, presión arterial un poco baja, sin signos
neurológicos, Na=120, K=2, Cl=100, HCO3=10, asuma 60% de agua, cuantos gramos
de Sal para elevar el Na a 130
o 17.5
Calculo: (60*0.5)*10=300 se necesita dos soluciones salinas (154*2=308), cada solución
salina tiene 58.5 g NaCl y cada gramo de solución tiene 17.5 g de Na.
10. ¿Qué solución salina usted escogería?
o 0.9 %
11. La hipofosfatemia produce:
o Fosfaturia (La disminución del calcio, produce un aumento de la PTH y esta a
su vez un aumento de la eliminación del fósforo por la orina. La PTH regula la
reabsorción de fosfato, pero su función principal es mantener la homeostasis del
calcio. La PTH aumenta la excreción urinaria de fosfato a través de la reducción
de la expresión del trasportador Na-Pi.)
12. La hipokalemia puede predisponer, excepto:
o Disminución de la amoniogenesis. (La hipokalemia por lo general cursa con
acidosis metabólica, en donde va a estar aumentada la formación de amonio.)
13. Un inhibidor de la anhidrasa carbónica es causa de:
o Hipokalemia ( Ellos producen un aumento de la excreción de bicarbonato al ser
este un anión, arrastra el potasio que es un catión. )
14. Un hombre llega al hospital comatoso, con fractura de cráneo, orina 175, suero de
Na=178, K=4, Cl=130, HCO3=25 Osm=350 en la orina 800 .
o Diabetes insípida central. Recordar que una de las causas de hipernatremia es la
diabetes insípida.
15. Confirmaría su dx así:
o Prueba de restricción y administración de vasopresina.
16. Causa de HTA con hipokalemia excepto:
o Síndrome de Bartter. (El síndrome de Bartter no cursa con hipertensión arterial.
HTA maligna, hiperaldosteronismo primario y síndrome de
hiperprostaglandismo entre otros).
17. Se relaciona con hipofosfatemia, excepto:
o Disminución de la afinidad de la hemoglobina por el O2 (La hipofosfatemia
produce una disminución del 2,3-DPG, y hay una desviación a la izquierda, lo
que produce una aumento de la afinidad por el oxigeno.)
18. Paciente con vómitos severos, hipotensión arterial, la orina al azar muestra Na mas de
40, osmolaridad de 800, puede tener:
o Falla suprarrenal. (El paciente al tener una hipovolemia, se activa el SRAA, y
aumenta la reabsorción de Na y disminuye su cantidad en orina.)
19. Un hombre de 30 Kg padece de Ca pulmonar, tiene 2 semanas de estar letargico y
obnubilado, suero de Na 105, Cl 72, K 4, HCO3 21, Osm 122 y en la orina Na 78, Osm
604, el dx mas probable:
o SIADH. (Hay una hiponatremia, hipotónica y euvolémica.Se define como el
conjunto de síntomas causado por el exceso de vasopresina (ADH) en la sangre
en relación a la osmolalidad del plasma, con un volumen normal de sangre
circulante. Un exceso de ADH condiciona una retención de agua y la
eliminación de sodio por la orina, lo que produce hiponatremia, hipoosmolalidad
del plasma e hiperosmolaridad de orina)
20. Tratamiento:
o Solución salina al 3%. (El tratamiento de SIADH, es restricción de agua y suplir
el déficit de sodio.)
21. Causas de enfermedad tubular intersticial crónica excepto:
o Nefritis alérgica. (En su mayoría medicamentos y medio de contraste. )
22. La administración de solución salina isotónica produce:
o Aumento del volumen extracelular.
23. Sospecha de obstrucción de las vías urinarias en caso de:
o Anuria.
24. En la insuficiencia renal aguda siempre da, excepto:
o Retensión nitrogenada. (Es un parámetro a tomar en cuenta para dializar al
paciente cuando es mayor de 100.)
25. Siempre habrá insuficiencia renal en caso de:
o Glomérulo nefritis rápidamente progresiva (La proliferación extracapilar
comprime el glomérulo comprimiéndolo y dañándolo severamente. El hallazgo
patognomónico es la presencia de semilunas epiteliales dentro del espacio de
Bowman, que comprimen y colapsan el ovillo glomerular. )
26. En caso de glomérulo nefritis post estreptocócica la filtración glomerular puede
disminuir por:
o Alteración del coeficiente de ultrafiltración.
27. En la nefritis túbulo intersticial puede dar lo siguiente, excepto:
o Proteinuria de 4 gr/día (Ese dato es de síndrome nefrósico. La cuantía de la
proteinuria, si existe, es moderada y la mayoría de las veces en rango inferior al
nefrótico (< 3.5 g/24h) salvo en algunos casos de NIA por AINE)
28. Es la causa de falla prerrenal.
o Pancreatitis aguda.
29. En la insuficiencia renal aguda siempre habrá:
o Aumento de la creatinina.
30. Causa de hipmagnesemia:
o Alcoholismo.(Otra causa importante son las pérdidas gastrointestinales. )
31. La hipofosfatemia causa, excepto:
o Aumento de PTH.
32. Paciente con diuresis de 250 ml al día con Osm plasmática y osmolaridad urinaria de
300, el Dx probable:
o Falla Renal Aguda
33. Paciente con oliguria, hiperkalemia, hiponatremia, creat 0.8 mg/dl y cpk normal, esto
puede corresponer a:
o Falla renal aguda, de causa prerrenal.
34. En falla renal crónica:
o Hipofosfatemia.
35. Urémico crónico con K 6.5 con leve academia, sin trastorno muscular y EKG normal,
tratamiento:
o Kayexalate.
36. Urémico agudo, acidótico, oligúrico, deshidratado, todo es cierto, excepto:
o Densidad urinaria 1010.(Esto indica isostenuria, el riñón no tiene la capacidad
de concentrar y es indicativo de riñones muy maltratados. )
37. Falla renal aguda, oliguria, todo es cierto, excepto:
o Osmolaridad urinaria elevada.
38. Síndrome nefrósico de cambios mínimos se caracteriza:
o Fusión de los pedicelos de los podocitos.
39. En el shock hipovolémico, el uso de dextrosa en agua puede causar:
o Hiponatremia sintomática. (Primero siempre corregir la volemia con solución
salina al 0.9.)
40. En la deshidratación por perdida fundamental de solutos, se da la siguiente, excepto:
o Entrada de electrolitos al cerebro.
41. En el SIADH se da lo siguiente excepto:
o Edema de los miembros inferiores.
42. Paciente con gastroenteritis, hipovolémico, cuyo suero contiene Na 155, K 2, Cl 117:
o Solucion Salina 0.45 + 40 mEq KCl.
43. Una medida general en el tx de un hipercalcémico:
o Solución salina al 0.9% (Para expandir el volumen.)
44. La velocidad de administracion de K en caso de riesgo para la vida puede ser:
o 40
45. Un ejemplo de hipernatremia hipervolemica es:
o Hiperaldosteronismo primario.
46. Una dieta de 3gr de NaCl corresponde a:
o 17 gr de Sodio.
47. La IRC ocaciona hiperfosfatemia permanente cuando la TFG esta por debajo de:
o 25 ml/min (El valor es por debajo de 30.)
48. Paciente con alcalosis metabólica e hipokalemia severa debe ser tratado con:
o KCl
49. No esta indicada en la hipercalcemia:
o Aldactone (Es un diurético ahorrador de K, ósea produce hipercalcemia, se
utiliza en el hiperaldosteronismo primaria y edema.)
50. Metabolismo del calcio están implicados menos:
o Insulina.
51. Hiponatremia hipotónica normovolémica:
o SIADH.
52. Causa de enuresis
o Necrosis cortical difusa.
53. La situación clínica de hiperfosfatemia con hipocalcemia con cirrosis hepática es:
o Cirrosis alcohólica.
54. Factor que altera el balance hídrico en el periodo post operatorio excepto:
o Disminución de ADH.
55. Ejemplo de hipernatremia hipervolemica:
o Hiperaldosteronismo primario.
56. Ejemplo de hipokalemia con aldosterona alta y osm baja:
o Hiperaldosteronismo primario. (También con renina baja.)
57. Hiperkalemia sin alteración en el EKG no es necesario administrar:
o Gluconato de calcio.
58. Urémico crónico con deshidratación hipertónica severa y acidótico presentara lo
siguiente excepto:
o Osmolaridad urinaria de 800.
59. La fracción de excreción de Na esta aumentada en:
o Falla cardiaca. (También esta aumentada en FRC, de causas renales.)
60. Paciente con vomito abundante, HTA, Una 6, oligúrico y osmolaridad de 800,
presentara:
o Insuficiencia suprarrenal.
61. En cirrótico con ascitis se recomienda, excepto:
o Dieta con 8.5 mEq de Na.
62. Factor que predispone a la hipernatremia son, excepto:
o Intoxicación hídrica.
63. En la hiperkalemia:
o Hay menos diferencia entre el potencial de reposo y el umbral.
64. Los indicios de falla renal aguda son:
Pueden ser cualquiera de las siguientes:
o Alteraciones en la osmolaridad urinaria.
o Alteraciones sen la relación Uosm/Posm.
o Alteraciones en el Una.
o Alteraciones en el FENA.
65. Se presenta en acidosis tubular renal proximal, excepto:
o NaHCO3 y los alpha agonista.
66. Na urinario menor de 10:
o Falla cardiaca.

1. La policia encuentra en la calle, incosciente e inmóvil, a altas horas de la madrugada a un indigente


que presenta múltiples hematomas y fetor etílico. En el hospital se le detecta urea de 200 mg/dl,
creatinina de 6 mg/dl, ácido úrico de 10 mg/dl y CPK de 1500 U/l. El diagnóstico probable es:

a. Necrosis tubular aguda alcohólica.


b. Fracaso renal agudo por urato.
c. Necrosis tubular aguda por hemólisis.
d. Infarto agudo de miocardio en paciente con insuficiencia renal crónica.
e. Fracaso renal agudo por radbomiolisis.
2. Ante el cuadro descrito en la pregunta anterior, la actitud más adecuada, de las siguientes, sería:

a. Restricción de líquidos a 800-1000 ml/día.


b. Tratamiento antibiótico empírico en espera de los cultivos.
c. Reposición de líquidos con suero salino isotónico ajustando el ritmo a la diuresis.
d. Iniciar hemoperfusión.
e. Reposición de líquidos con suero salino hiperosmolar.
a.
b. 3. ¿Cuál cree que es el mejor método para disminuir las infecciones urinarias asociadas a la
colocación de catéteres urinarios en paciente hospitalizados?:
a. Profilaxis antibióticas.
b. Válvulas sin retorno en los sistemas de drenaje.
c. Medidas antisépticas en su colocación.
d. La obligatoriedad del uso de sistemas cerrados de drenaje.
e. Uso de catéteres impregnados en antimicrobiano
4. Ante una sospecha de enfermedad renal poliquística del adulto, el paso diagnóstico siguiente más
razonable, ente los que se señalan, es:

Tomografía.
Urografía intravenosa.
Ecografía.
Tomografía axial (TAC).
Angiografía.

5. ¿Qué entenderemos por "riñón de mieloma"?: Cualquier fracaso renal en un paciente portador de un
mieloma.

La infiltración renal por el mieloma.


La precipitación de cadenas ligeras en los túbulos renales.
El depósito de proteína de Bence-Jones en el glomérulo.
El fracaso renal causado por la hipercalcemia del mieloma.

6. Un paciente de 68 años presenta progresivo deterioro del nivel de conciencia en las horas siguientes a
haber sido operado de herniorrafia inguinal con anestesia general, de la que despertó normalmente. Su
situación hemodinámica es estable, no hay signos de insuficiencia cardíaca, no tiene fiebre ni focalidad
neurológica. El hemograma y el estudio de coagulación son normales. La bioquímica sérica muestra:
urea 36 mg/dl, Glu 120 mg/dl, Na+ 118 mEq/l, K+ 4,5 mEq/l, úrico 2,3 mg/dl, osmolaridad urinaria 500
mOsm/ kg, Na+ urinario 35 mEq/l. Señale, de las siguientes, la causa más probable de este cuadro:

Insuficiencia hepática aguda por anestésicos.


Deshidratación.
Hiponatremia ADH-dependiente.
Tubulopatía pierde-sal.
Sepsis en fase inicial.
7. ¿Cuál de las siguientes afirmaciones es cierta con respecto a la enfermedad de Hodgkin?:

Es de comienzo extranganglionar con mayor frecuencia que los linfomas no Hodgkin.


La célula de Redd-Sternberg, aunque característica de la enfermedad es un acompañante no
tumoral.
A diferencia de los linfomas no Hodgkin, es característica su extensión a otros territorios
linfoides por contigüidad.
La variante de esclerosis nodular corresponde al 1-5% de todos los casos de enfermedad de Hodgkin.
Los hallazgos moleculares sugieren que se trata de una neoplasia de origen histiocítico.

8. En la uremia pre-renal:

El sodio en orina es superior a 60 mEq/l, la osmolaridad urinaria es superior a 500 mOsm/ kg H2O y la
relación urea en orina/urea en plasma es superior a 8.
El sodio en la orina es inferior a 20, la osmolaridad urinaria es inferior a 200 mOsm/ kg H2O y la
relación entre urea en orina / urea es plama es inferior a 2.
El sodio en orina es inferior a 20 mEq/l, la osmolaridad en orina es inferior a 200 mOsm/ Kg H2O y
la relación urea en orina / urea en plama es superior a 8.
El sodio en orina es inferior a 20 mEq/l, la osmolaridad urinaria es superior a 500 mOsm/ Kg H2O,
y la relación entre la urea en orina y la urea en plasma es superior a 8.
El sodio en orina es superior a 60, la osmolaridad urinaria es superior a 500 mOsm/Kg H2O, y la relación
urea en orina/urea en plasma es superior a 8.

9. Señale la respuesta correcta referida a las alteraciones del examen de la orina:

La presencia de cilindros hialinos es siempre patológica.


Los cilindros granulosos contienen albúmina e Inmnoglobulinas.
Los cilindros leucocitarios son típicos de Glomerulonefritis postestreptocócica.
Los cilindros hemáticos se presentan en cualquier discrasia sanguínea.
Un resultado negativo de presencia de nitritos en tira reactiva, excluye la existencia de baterias.
10. Mujer de 32 años de edad, embarazada de 11 semanas, sin antecedentes personales de interés
salvo alergia a las penicilinas, acude a su médico de familia con el objeto de recoger los
resultados de la analítica del primer trimestre, en el que se evidencia una bacteriuria, estando
la paciente asintomática. Una vez comprobada la bacteriuria, ¿qué actuación terapéutica y de
control debería ser aconsejada en este caso?:
Buena hidratación y vigilancia de síntomas urinarios o fiebre.
Cefalexina 500 mg/6h durante 3-7 días y cultivo urinario a la semana de haber finalizado el tratamiento.
Cotrimoxazol 800/160mg/12h durante 3-7 días y cultivo urinario mensual hasta el final de la
gestación.
Nitrofurantoína 100 mg/6h durante 3-7 días y cultivo urinario mensual hasta el final de la
gestación.
Ciprofloxacino 500 mg/12h durante 10 días y cultivo urinario a la semana de haber finalizado el
tratamiento.
11. En un paciente con síndrome nefrótico, las medidas generales no específicas, para corregir la
proteinuria incluye uno de los siguientes procedimientos:

Dieta hiperproteica.
Diuréticos y/o ?-bloqueantes.
Inhibidores del enzima conversor de la Angiotensina (IECAs).
Calcioantagonistas no dihidropiridínicos.
Corticoides por vía sistémica.

12. Paciente de 68 años diabética, que consulta por malestar general. En la gasometría venosa destaca
pH 7.25, Bicarbonato 15 mmol/ l (normal 24-28 mmol/l). Hiato anionico (anión GAP): 11 mmol/l
(normal 10-12 mmol/l). ¿Cúal de las siguientes entidades NO descartaría como diagnóstico?:

Cetoacidosis diabética.
Insuficiencia renal crónica.
Acidosis tubular renal.
Ingesta de salicilatos.
Acidosis láctica.

13. Un paciente de 48 años, con antecedentes de hepatitis C (anti VHC +), presenta edemas maleolares,
proteinuria 3,5 g/día, creatinina sérica 1,6 mg/dL y microhematuria en el sedimento. ¿Qué proceso
glomerular de los siguientes se encontrará con más probabilidad en la biopsia renal?:

Cambios mínimos.
Glomeruloesclerosis focal y esclerosante.
Glomerulonefritis extracapilar.
Glomerulonefritis membranosa.
Glomerulonefritis membranoproliferativa.

14. Una mujer de 68 años acude al Servicio de Urgencias por malestar general que ha ido progresando
en los últimos 15 días, a partir de un episodio gripal. Ha notado disminución progresiva del volumen de
diuresis, edemas maleolares y dificultad respiratoria. Es hipertensa. En la analítica destaca una
creatinina plamática de 5 mg/dl, urea 180 mg/dl, Na 138 mEq/l, K 4.9 mEq/l. Las cifras de
complemento son normales. Los anticuerpos anti-membrana basal son negativos. En la orina presenta
cilindros hemáticos, proteinuria de 1 g/l y microhematuria. Aporta una analítica de un mes antes, sin
alteraciones. ¿Cuál de los siguientes diagnósticos es más probable?: poliangeítis microscópica
9. Señale la respuesta correcta referida a las alteraciones del examen de la orina:

La presencia de cilindros hialinos es siempre patológica.


Los cilindros granulosos contienen albúmina e Inmnoglobulinas.
Los cilindros leucocitarios son típicos de Glomerulonefritis postestreptocócica.
Los cilindros hemáticos se presentan en cualquier discrasia sanguínea.
Un resultado negativo de presencia de nitritos en tira reactiva, excluye la existencia de baterias.

1. Un paciente asintomático, hipertenso de 65 años en tratamiento farmacológico, acude a su médico


que le encuentra en la analítica los siguientes parámetros: Hb 14,1 g/dl, VCM 88 fl, Urea 75 mg/dl,
Creatinina 1,4 mg/dl, Sodio sérico 128 mEq/l, Potasio sérico 2,8 mEq/l, Cloro 89 mEq/l. Lo más
probable sería:

f. Que tenga una anomalía en la absorción de cloro, tipo Bartter.


g. Que la hipertensión sea secundaria a una tubolopatía perdedora de potasio.
h. Que en el tratamiento que reciba exista un diurético de asa.
i. Que en el tratamiento se incluya un inhibidor de la enzima convertidora de la angiotensina.
j. Que no tome fruta en la dieta.

2. Un paciente de 35 años con insuficiencia renal crónica secundaria a pielonefritis crónica recibe un
trasplante renal de cadáver con el que compartía dos identidades en A y B y una en DR. Recibe
tratamiento inmunosupresor con ciclosporina A y corticoides a dosis estándar. En el postoperatorio
inmediato se observa buena diuresis y no es necesario el tratamiento sustitutivo con hemodiálisis. En el
5º día de evolución, el paciente presenta fiebre de 38ª, TA de 180/110, oliguria y disminución en la
concentración urinaria de sodio. El diagnóstico más probable sería:

f. Crisis hipertensiva.
g. Infección respiratoria.
h. Pielonefritis aguda del injerto renal.
i. Recidiva de su enfermedad renal.
j. Rechazo agudo del injerto renal.

3. ¿A cuál de los siguientes factores NO se asocia la Pielonefritis Aguda por Pseudomonas


Aeuruginosa?:

f. Embarazo.
g. Sonda urinaria.
h. Nefrolitiasis.
i. Manipulación urológica.
j. Estenosis de la vía urinaria.
a.
4. Paciente diagnosticado de hiperplasia benigna de próstata que presenta como sintomatología
dificultad para el inicio de la micción, disminución de la fuerza y volumen del chorro miccional,
micción entrecortada y goteo terminal; la aparición de polaquiuria, y urgencia miccional se debe
generalmente a:

Presencia de infección urinaria.


Inestabilidad vesical.
Prostatitis (adenomitis).
Existencia de residuo vesical.
Presencia de divertículos vesicales.

6. Una paciente de 15 años de edad consulta por poliuria y nicturia, cansancio fácil y astenia. Presenta
dichos síntomas desde hace años, y tienden a intensificarse durante los veranos muy calurosos, en los
que se asocia hormigueo lingual y peribucal. A la exploración física sólo destaca una tensión de 100/50
mmHg. El ionograma muestra Na 135 mEq/l, K 2 mEq/l, Cl 105 mEq/l, pH 7.45, bicarbonato 30 mEq/l.
La determinación de renina y aldosterona muestra cifras elevadas, tanto basales como tras estímulo. De
los procesos que siguen ¿cuál es compatible con el cuadro clínico descrito?:

Hipoaldosteronismo primario.
Estenosis de la arteria renal.
Síndrome de Liddle.
Enfermedad de Addison.
Síndrome de Bartter.

8. Un hombre, adicto a drogas por vía parenteral, está ingresado por endocarditis infecciosa. Durante
su enfermedad presenta un cuadro de glomerulonefritis aguda. ¿Cuál de las respuestas es
INCORRECTA?:

Suele ser debida a inmunocomplejos.


No suele presentar piuria.
El complemento está descendido.
A veces produce síndrome nefrótico.
Suele evolucionar favorablemente al controlar la infección cardiaca.

9. En la poliquistosis renal del adulto, ¿cuál de las siguientes afirmaciones es FALSA?:

Es una enfermedad hereditaria, autosómica dominante.


Habitualmente se detecta en la primera infancia con ecografía.
Es causa de deterioro progresivo de la función renal.
Frecuentemente cursa con hipertensión arterial.
Se le asocia litiasis renal en un 15-20% de los casos.

10. Señalar la respuesta correcta en relación con la fisiopatología del calcio en la insuficiencia renal
crónica:

La hormona paratiroidea se eleva precozmente y de forma progresiva.


Mecanismos compensadores consiguen mantener a la hormona paratiroidea dentro de límites
normales hasta estadíos avanzados de la insuficiencia renal.
Disminuye la síntesis de 25-hidroxi-vitamina D.
El calcio sanguíneo aumenta paralelamente a la disminución del filtrado glomerular.
Aumenta la síntesis de 1,25-dihidroxi-vitamina D.
incorrectas => Pregunta difícil

11. La actitud más correcta en el caso planteado en la pregunta anterior, debe ser:

Retirar el enalapril, indicar esteroides y controlar la evolución de la función renal.


Sustituir el enalapril por otro hipotensor de distinto mecanismo de acción.
Iniciar tratamiento dialítico.
Sustituir el enalapril por ramipril.
Disminuir la dosis de enalapril.

12. Una mujer de 68 años acude al Servicio de Urgencias por malestar general que ha ido progresando
en los últimos 15 días, a partir de un episodio gripal. Ha notado disminución progresiva del volumen de
diuresis, edemas maleolares y dificultad respiratoria. Es hipertensa. En la analítica destaca una
creatinina plamática de 5 mg/dl, urea 180 mg/dl, Na 138 mEq/l, K 4.9 mEq/l. Las cifras de
complemento son normales. Los anticuerpos anti-membrana basal son negativos. En la orina presenta
cilindros hemáticos, proteinuria de 1 g/l y microhematuria. Aporta una analítica de un mes antes, sin
alteraciones. ¿Cuál de los siguientes diagnósticos es más probable?:

PAN microscópica. poliangeítis microscópica


Brote lúpico.
14. Un varón de 45 años llega comatoso a Urgencias. Presenta PaCO2 basal 25 mmHg, pH 7,15,
hipocalcemia moderada con Gap aniónico y osmolar elevados, leucocitosis y cristaluria. Con más
probabilidad este paciente tiene una intoxicación por:

Barbitúricos.
Monóxido de carbono.
Etilenglicol.
Benzodiacepinas.
Salicilatos.

15. Mujer de 63 años que es diagnosticada de carcinoma de células escamosas del trígono vesical, con
invasión de la capa muscular. ¿Cuál sería su actitud terapéutica en este caso?:

Radioterapia externa con 7000 rads.


Quimioterapia adyuvante seguida de Cistectomía radical.
Resección transuretral seguida de inmunoterapia intravesical (BCG).
Cistectomía radical con extirpación de cara anterior de vagina.
Radioterapia externa seguida de quimioterapia con Cisplatino.

17. Hombre de 25 años que presenta tumoración testicular derecha indolora de 1 mes de evolución. La
alfafetoproteína (AFP) está elevada. ¿Cuál de los siguientes tipos histológicos de cáncer de testículo es
MENOS probable?:

Seminoma puro.
Carcinoma embrionario.
Tumor del saco vitelino.
Tumores mixtos.
Teratocarcinoma.
18. Cuando un paciente bajo tratamiento diurético con tiazidas o furosemida incumple la dieta y come
más sal de la prescrita, el resultado análitico esperable es:

Mayor hipernatremia.
Mayor hiponatremia.
Mayor hiperpotasemia.
Mayor hipopotasemia.
Mayor acidosis.
20. ¿En cuál de las siguientes glomerulonefritis hay activación del complemento por la vía alternativa?:
Aguda secundaria a endocarditis.
Aguda postestreptocócica.
Difusa lúpica.
Por crioglobulinemia.
Membranosa.

21. En un enfermo con un síndrome de Goodpasture que le ha conducido a una insuficiencia renal
crónica:

No debe realizarse un trasplante renal, pues la enfermedad le dañará el riñón trasplantado.


El trasplante puede realizarse si la producción de anticuerpos anti-membrana basal ha cesado.
Debe seguir siendo tratado indefinidamente con inmunosupresores para proteger su pulmón.
Debe seguir siendo tratado indefinidamente con esteroides para proteger su pulmón.
La función renal se recuperará cuando dejen de producirse autoanticuerpos anti-membrana basal.

22. En relación a la hemoglobinuria paroxística nocturna, señale cuál de las siguientes


afirmaciones es FALSA:
Es un defecto adquirido.
Falta la molécula de anclaje glicosilfosfoinositol en el membrana.
Cursa con un aumento de la resistencia del hematíe a la lisis por el complemento.
Se acompaña de un aumento de frecuencia de trombosis venosas.
Presenta a menudo leucopenia y trombopenia.

23. Con respecto a la nefropatía diabética, señalar la respuesta FALSA:

La diabetes tipo 2 es la etiología más frecuente de insuficiencia renal terminal en el mundo occidental.
Más del 90% de los diabéticos tipo 1 desarrollan nefropatía a los 30 años del diagnóstico de
diabetes.
La alteración renal más temprana es la hiperfiltración.
La existencia de microalbuminuria predice el desarrollo de nefropatía clínica.
La gran mayoría de los diabéticos tipo 1 con nefropatía tienen también retinopatía.

24. El diagnóstico de la anemia hemolítica se realiza gracias a cinco signos biológicos característicos:
elevación de los reticulocitos, hiperregeneración eritroblástica, hiperbilirrubinemia no conjugada,
incremento de la lácticodeshidrogenasa sérica (LDH) y descenso de la haptoglobina. ¿Cuáles de estos
signos biológicos pueden observarse también en las pérdidas de sangre por hemorragia?:

Descenso de la haptoglobina e hiperregeneración eritroblástica.


Elevación de LDH y bilirrubina no conjugada.
Hiperregeneración eritroblástica y elevación de la cifra de reticulocitos.
Elevación de la bilirrubina no conjugada y descenso de la haptoglobina.
Descenso de la haptoglobina y elevación de la LDH.
26. En el tratamiento de la hiperpotasemia grave de un paciente urémico NO está indicado
administrar:

Resinas de intercambio catiónico (sodio, calcio).


Bicarbonato sódico i.v.
Glucosa con insulina i.v.
Gluconato cálcico o cloruro cálcico i.v.
Tiacidas por vía oral.

27. Paciente de 82 años de edad, que refiere síndrome constitucional de tres semanas de evolución, con
astenia, anorexia y pérdida de peso con oligoanuria progresiva en las veinticuatro horas previas al
ingreso hospitalario. No signos de hiperhidratación. Creatinina plasmática 6 mg/dl. Proteinuria
1gr/24h. Sedimiento: microhematuria. Determinación de ANCA positivo, patrón p-ANCA anti MPO.
ECO renal que muestra riñón derecho pequeño y riñón izquierdo de tamaño normal. ¿Cuál cree que es
el procedimiento más adecuado y prioritario?:

Iniciar tratamiento sustitutivo con diálisis.


Proceder a practicar biopsia renal.
Iniciar tratamiento con pulsos intravenosos de metilprednisolona y Ciclofosfamida oral.
Plantear plasmaféresis.
Iniciar tratamiento con Prednisona oral.

28. Un niño de 4 años de edad muestra un importante retraso de crecimiento, lesiones de raquitismo
resistentes al tratamiento con dosis habituales de vitamina D y poliuria. ¿Cuál de las siguientes
asociaciones considera que permite el diagnóstico de síndrome de Fanconi?:

Glucosuria + hiperaminoaciduria + alcalosis + hiperfosforemia.


Glucosuria + hipoglucemia + acidosis metabólica + hipofosforemia.
Glucosuria + hiperaminoaciduria + acidosis metabólica + hipofosforemia.
Glucosuria + hiperaminoaciduria + alcalosis metabólica + hipofosforemia.
Hipoglucemia + hiperaminoaciduria + alcalosis metabólica + hipofosforemia.

29. La aparición de carencia de vitamina B12 no es infrecuente en la población geriátrica. Todas las
siguientes afirmaciones son ciertas, EXCEPTO una, señálela:

Su déficit se relaciona con defectos de absorción secundarios a una gastritis atrófica con aquilia.
Puede presentarse sólo como alteraciones neuropsiquiátricas.
Pueden no existir alteraciones hematológicas.
Siempre existen niveles de cobalaminas (B12) séricas disminuidas.
Los niveles de ácido metil-malónico plasmáticos están elevados.
30. De los siguientes tipos de linfomas, señale cuál es el que tiene el peor pronóstico:

Linfoma no Hodgkin de tipo folicular.


Linfoma de tejido linfoide asociado a mucosas (linfoma tipo MALT).
Linfoma no Hodgkin linfocítico difuso.
Enfermedad de Hodgkin tipo esclerosis nodular.
Linfoma no Hodgkin de células del manto.

31. Hombre de 35 años que consulta por la aparición de una masa en el testículo izquierdo sin
antecedentes traumáticos ni infecciosos; la ecografía testicular demuestra que se trata de una
masa sólida. ¿Cuál, entre las siguientes, es la actitud a seguir más adecuada?:
PAAF de la masa.
Biopsia quirúrgica de la masa.
Orquiectomía por vía inguinal previa toma de muestra sanguínea para determinar -fetoproteína y -
gonadotropina coriónica.
Orquiectomía transescrotal con resección de hemiescroto.
Vigilancia mediante ecografías cada 2 meses para valorar aumento de tamaño de la masa.

32. En el caso de un paciente con hiperplasia prostática benigna, ¿cuál, entre las siguientes
circunstancias, NO establece por sí misma indicación de cirugía?:

Hematuria severa recurrente.


Retención urinaria que requiere sondaje vesical permanente.
Infecciones de orina de repetición.
Nicturia de dos veces.
Residuo postmiccional mayor de 200 cc.

33. De las siguientes afirmaciones respecto a la enfermedad renal poliquística del adulto, ¿cuál es la
correcta?:

Es más prevalente en mujeres, por el uso de anovulatorios.


Se puede acompañar de quistes hepáticos.
Sólo desarrolla hipertensión arterial si reciben antiinflamatorios.
Rara vez evolucionan a insuficiencia renal crónica, excepto en presencia de hipertensión arterial.
La cirugía precoz sobre los quistes es fundamental.

35. Paciente de 70 años, que hace 2 semanas fue sometido a una coronariografía, acude al hospital por
aparición de lesiones purpúricas palpables en miembros inferiores, elevación de la creatinina sérica a 3
mg/dl, proteinuria de 1g/24h, hipocomplementemia y microhematuria y leucocituria en el sedimento
urinario. ¿Cuál es el diagnóstico más probable?:

Glomerulonefritis aguda rápidamente progresiva.


Glomerulonefritis aguda postestreptocócica.
Síndrome hemolítico-urémico.
Enfermedad atero-embólica.
Glomerulonefritis membrano-proliferativa.

36. ¿De cuál de los siguientes cuadros es característica la aparición de cilindros hemáticos en el
sedimento urinario?:
Necrosis tubular aguda.
Lesión a cualquier nivel de las vías urinarias.
Daño glomerular severo.
Daño tubular.
Cualquier lesión de la nefrona.

37. Paciente de 63 años, fumador importante, que refiere hematuria total intermitente y síndrome
miccional irritativo desde hace 1 mes. La citología de orina es positiva. Se realiza una evaluación vesical
bajo anestesis con biopsias vesicales múltiples, siendo diagnosticado un carcinoma vesical “in situ”.
¿Cuál de las siguientes estrategias terapéuticas es la más apropiada?:

Instilaciones endovesicales con Mitomicina C.


Instilaciones endovesicales con BCG.
Cistectomía radical.
Quimioterapia sistémica.
Radioterapia pelviana.

38. En los pacientes diagnosticados de leucemia linfática crónica, a lo largo de su evolución presentan
complicaciones infecciosas bacterianas y víricas así como segundas neoplasias. ¿Qué otras
complicaciones presentan frecuentemente?:

Hipercalcemia y lesiones osteolíticas.


Fenómenos autoinmunes.
Insuficiencia renal crónica.
Transformación en leucemia aguda.
Mielofibrosis con metaplasia mieloide.

39. Ante una sospecha de enfermedad renal poliquística del adulto, el paso diagnóstico siguiente más
razonable, ente los que se señalan, es:

Tomografía.
Urografía intravenosa.
Ecografía.
Tomografía axial (TAC).
Angiografía.

1. Causa de hiperfosfatemia:
a. Falla renal cronica

2. Causas de hiperfosfatemia grave, excepto:


a. Alcalosis respiratoria

3. Paciente con vómitos severos, oligurico con hipotensión arterial Una>


40…… 800 mOsm/Kg puede tener:
a. In suficienci a sup rarren al

4. Paciente urémico crónico con deshidratación severa puede presentar lo


siguiente excepto:
a. Osmolarida d urinaria de 800 m Osm/Kg

5. La hipokalemia produce:
a. Aumento de la magnitud del potencial de reposo

6. Uno de los indices de falla renal aguda


es: Uosm/Tosm

7. Hormona producida en los riñones que disminuye en la falla renal cronica:


a. Peptido natriuretico (segun arjona)

8. Las calcificaciones metastásicas se deben:??


a. Hip erp aratiroi dism o 1ario

9. Causa de anuria:
a. Necrosis cortical

10. la etapa de la insuficiencia renal cronica con tasa de filtración glomerular


de 60-89 ml/min es la:??
a. 2

11. La FENA es la relación entre:


a. depuración de Na/ depuración de creatinina x 100

12. En la nefritis tubular intersticial aguda se pierde la capacidad de poder


concentrar la orina debida a:??
a. Daño medular

13. La solucion de NaCl al 23,4 %


tiene: 8000 mOsm/l

14. Se asocia con el SIAD, excepto:


lipoma

15. El efecto biologico mas importante de la calcitonina es, excepto:


a. Estimular la resorcion osea osteoclastica

16. Favorece la entrada de K al espacio intracelular, excepto:


a. Agonista adrenergico
17. Factores que afectan el balance hídrico en el post-operatorio, excepto:
a. Calcitriol

18. Las primeras manifestaciones clínicas de la hiponatremia son:??


a. neurológica

19. En la hipernatremia hipertonica se da lo siguiente excepto :


a. Salida de aa

20. Manifestaciones clinicas de la hipercalcemia, excepto :


a. Taquicardia

21. En falla renal aguda por nefrotoxicidad:


a. Se destruye la membrana basal

22. Causa de
hipomagnesemia:
alcoholismo

23. Na serico de 168 mEq/l 70 kg (utilize como % de agua al 50% y Na normal


140 mEq/l 7 litros

24. Indicaciones de diálisis en falla renal aguda, excepto :


a. Ac. Meta bolica refra ctaria

25. El KCL IV se puede asministrar asi, excepto:


d. “ “ periferica de 20 meq en 20 ml en 2 horas

26. Aumenta la excrecion urinaria de K, excepto:


a. Triamtereno

27. Factores que regulan la secrecion distal de K, excepto:


a. PTH

28. Enfermo de 60 Kg con 60% de agua corporal, Posm 260 mOsm/kg,


¿Cuántos gramos de NaCl aumentaria el Na en 10 mEq?
a. 21 gramos

50. Conteniendo 10 g de MgSO4 al %, cuantos ml de sal necesitaria?


20 ml

1. Cuándo disminuye el
fosfato? R. Aumento de
PTH

2. Ejemplo de hipernatremia
hipervolémica: R. Aldosteronismo
primario

3. La hipocalemia produce:
R. Aumento de la magnitud del potencial de reposo

4. La depleción de agua produce, excepto:


R. Salida de electrolitos del cerebro
5. Velocidad de administración de KCl en caso de
urgencia: R. 40 mEq/h

6. Cambios en EKG de la hipermagnesemia es


igual: R. Hipercalemia

7. Enfermo de 60 Kg con 60% de agua corporal, osmolaridad plasmática 260


mosmol/Kg,
¿cuántos gramos de NaCl aumenta el Na en 10 mEq en 24 horas?
b) 21

8. Uso de Calcio en tratamiento de


hipercalemia para: R. Modificar el
Potencial de Membrana

9. Hipermagnesemia severa produce:


R. Paro respiratorio

10. Venoclisis de 1L de D/A 5% conteniendo 10 mg de MgSO4 y


al 50%: R. 20 mL

11. La hipocalcemia:
R. Prolongo QT a expensas del segmento ST

12. Tratamiento de urgencia de


hipercalcemia: R. SSN 0.9%

13. Causa de
hiperfosfatemia: R.
Insuficiencia renal

14. Adaptación cerebral, hipertonicidad,


excepto: R. Pérdida de iones

15. En la adaptación cerebral, la hipotensión da lo siguiente, excepto:


R. Ganancia de aminoácidos

16. En ausencia de síntomas de hipernatremia, el sodio debe ser


disminuido a razón de: R. 0.5 mEq/L/h

17. Para corregir un sodio cerca de 185 mEq se administró en 86 horas, 66 litros de
D/SS 5%.
¿cuál es la velocidad de administración de la venoclisis?
R. 69 mL ¿?????

18. Manifestación clínica de hipercalcemia, excepto:


R. a) Hipotensión arterial

19. Consecuencia de hipofosfatemia, excepto:


R. a) No afecta al SNC

20. Causa de hipofosfatemia, excepto:


R. a) Falla renal

21. Causa de HTA con hipocalemia, aldosterona-


renina baja: R. Síndrome de Cushing
22. Causas de HTA con hipocalemia y
renina alta: R. Hipertensión maligna

23. En una deshidratación severa hipertónica con hipotensión arterial severa, la


prioridad inicial es administrar:
R. SSN 0.9%

24. En la secreción inapropiada de ADH se da lo siguiente, excepto:


R. No hay expansión de volumen

25. Tejido con mayor contenido


de K+: b) Músculo

14) Paciente con Oliguria,…… muchos síntomas :


a) Insuficiencia suprarrenal ( creo que era la respuesta)

15) En IRA se mide todo menos:


a) Uosm/Posm x100
b) Na excretado/ Na filtrado x200
c) Na filtrado/ Na excretado x100
d) Na urinario >40
e) Ninguna de las anteriores

16) Paciente con hipertensión , hipokalemia, cretinina nor mal t iene:


a) Hipertensión arterial maligna
b) Cushing
c) Algo relacionado a licorice

31) En hiponatremia el daño principal


es: a) Neurológica

32)Adaptación del cerebro ante solución


hipotónica: c) Edema cerebral

53) Paciente con hipercalcemia puede ser por todo menos: d)


Medicamentosa

62) La PTH promueve:


a) Reabsorción tubular de calcio y excreción de fosfato

66) Paciente que sufre golpe en la cabeza y t iene fractura de cráneo.


Tiene osmolaridad urinaria disminuída , pero al darle tratamiento con
vasopresina ésta aumenta. El paciente t iene:
a) Diabetes insípida central

24. Usted tiene una paciente con hiperemesis gravidica. El trastorno


metabolico que espera encontrar es:
a. alcalosis hipocloremica

25. Cual de las siguientes opciones esta contraindicada en el tratamiento de la


hiperkalemia
a. aldactone

40. La accion toxica mas importante del ganciclovir es:


a. Insuficiencia Renal
41. Rotando en sala de medicina interna le piden que evalue un paciente con
HIV el cual por presentar una meningitis por criptococci recibe
anfotericina B; debido a este medicamento buscaria como complicación:
a. hipokalemia

42. Paciente de 84 anos quien recibe diuretico tiazidico para la hipertensión


arterial es admitido por presentar diarrea y trastornos del sensorio.
Tiene disminución del turgor de la piel y la presion arterial es normal,
Pna 174mEq/L, Una 5mEq/L, Uosm 606mEq/kg. La hipertensión se debe
a:
a. diabetes insipida

43. Hipernatremia Hipervolemica


a. aldostero nismo primario

44. La hipokalemia en un cirrotico puede precipitar COMA por:


a. Aumento de la amoniogenesis

Causa de hiperkalemia:
a. Hipercatabolismo t isular

45. Causa de hipomagnesemia


a. cirrosis avanzada

46. Causa renal de hipomagnesemia


a. uso de diureticos de asa

47. en caso de hipercalcemia pensar en:


a. neoplasia

48. En el síndrome nefrosico puede encontrarse cilindros:


a. cereos

1. Causa de hipomagnesemia:
a. alcoholismo

2. La hipocalcemia en la falla renal aguda puede ser asintomática por la:


a. acidemia

3. La hipofosfatemia causa, excepto:


a. Aumento de vit. D
b. Aumento de PTH

4. Un pte con diuresis de 250 ml al día con osmolaridad plasmática (presión


osmótica) y osmolaridad urinaria de 300 mosm/l, el dx probable e:
a. Insuficiencia renal

5. Pte con oliguria, hiperkalemia, hiponatremia, creatinina 0.8 mg/dl y CPK


normal: Esto puede corresponder a:
a. IRA <de 30 mosm es ira

6. No está indicado en caso de hiperkalemia:


r. Aldactone

7. Mujer con hiperemesis grávida:


r. Alcalosis hipoclórica

8. En causa de hipomagnesemia:
r. Alcoholismo.

9. En el metabolismo del calcio está implicado, menos:


r. Insulina

10. Hiponatremia, hipotónico, normovolémico:


r. SIADH

11. Fórmula del clearence de creatinina:


r. clearence = 140 – edad x peso en Kg
72 x creatinina

12. Su infección produce litiasis renal:


r. proteus sp.

13. Fórmula de irritabilidad muscular:


r. IR = Na + K + OH
(Ca) + Mg + H

14. Con respecto al síndrome diarreico, nombre 2 tipos osmóticos:


a. Laxantes osmóticos (Mg2+, PO4, SO4)
b. Carbohidratos no absorbibles (sorbitol, lactulosa).

15. Nombre dos síndromes diarréicos tipo secretora:


a. Consumo crónico de etanol.
b. Obstrucción intestinal incompleta.

16. Nombre 2 sindromes diarréicos motores con aumento de motilidad:


a. Hipertiroidismo
b. Prostaglandinas.

17. Indica falla renal aguda:


a. U.N. = 1

18. Deshidratación por pérdida de agua y densidad urinaria 1.003, creatinina 6:


a. Falla pre-renal

19. Uno de los índices de falla renal es:


a. Oferta tubular de Na/ excresión urinaria de Na x 100 **

20. El síndrome hepatorrenal es causa de falla:


a. pre-renal
b. renal

21. Causa más comun de falla renal cronica:


a. Diabetes melitus

36. En la glomerulnefritis aguda por infección, la causa, excepto:


b. osmolaridad urinaria normal

A pesar de la diversidad de enfermedades indicadas abajo, ellas tienen muchos síntomas en


común. Con frecuencia, los síntomas y signos observados incluyen: disminución en el volumen
de orina, proteína en la orina (proteinuria), sangre en la orina microscópica o macrscópica
(hematuria), hinchazón (edema), presión sanguínea alta (HTA) y una disminución en la
capacidad del riñón de extraer eficazmente los residuos.

22. Falla renal crónica, excepto:


a. Hipofosfatemia
b. Acidemia

23. En la falla renal aguda siempre habrá:


a. aumento de creatini na

24. Causa de falla pre-renal:


a. Obstrucción de ambos uréteres
b. Falla cardiaca

25. En la nefritis tubular intersticial se puede dar, excepto:


a. hipernatremia

26. Sugiere glomerulonefritis


a. cilindro s eritrocitarios

27. Causas de falla renal parenquimatosa:


a. papilitis

28. Causa de anuria:


a. Necrosis cortical

29. La hipofosfatemia causa todo,


excepto: r- aumento de PTH

30. Hepatitis C periodo de incubación:


r. 15-150 media de 5º (7 semanas)

31. En pte con falla renal crónica que ha sido transplantado esperamos
encontrar:
r. osteomalacia.

Urémico crónico con potasio sérico de 6.5mEq/L con leve acalemia, sin trastorno muscular y
EKG normal, puede ser tratado con:
a. kayaxelate

La insuficiencia renal crónica ocasiona hiperfosfatemia permanentemente cuando la tasa de


filtración glomerular esta por debajo de:
a. 25 ml/ min

La hipofosfatemia es causa de excepto


a. inotropismo positivo
La solución de NaCl 23,4% tiene de osmolaridad:
a. 8000 mosm/l

En la hiperkalemia:
a. hay menor diferencia entre el potencial de reposo y el umbral

La velocidad de administración de potasio en caso de gran riesgo para la vida puede ser de:
a. 40 mEq/ L

Un ejemplo de hiponatremia hipotónica normovolemica es:


a. SIADH

A una embarazada con eclampsia se le ordena 1 litro de D/A 5% con 10g de MgSO4. ¿Cuántos
ml de MgSO4 al 50% se han utilizado?
a. 20
Causa hipomagnesemia:
a. alcoholismo

Paciente con diuresis de 300ml/día, K sérico 7mEq/L, Posm y Uosm 300mosm/L, el diagnóstico
más probable es:
a. insuficiencia renal <350

La mielinosis pontina se caracteriza por:


a. cuadriplejía flácida

Calcule el déficit de agua en una paciente con sodio sérico 168mEq/L y que pesa 70Kg (utilice
como porcentaje de agua 50% y sodio normal mEq/L):
a. 7L (70) (0.50) (168/140-1)

En la SIADH la concentración de sodio urinario es de:


a. Mas de 40 mEq/L

En paciente con vómitos abundantes y prolongados con hipokalemia severa debe administrarse:
a. Na Cl 0.9% con KCl

La hipokalemia en un paciente con cirrosis puede contribuir:


a. coma

El plasma de un paciente contiene Na 125mEq/L, glucosa 108mg/dl, urea 300mg/dl, podrá


presentar:
a. síntomas de hipotonicidad

La hiponatremia aguda sintomática es debida a, excepto:


a. exceso de glucocorticoides

La administración de solución salina 0.9% produce:


a. aumento del volumen extracelular

Alcalosis metabólica con hipertensión arterial, hipokalemia, aldosterona normal y renina


normal:
a. Sindroma de Cushing

La hipokalemia puede predisponer, excepto:


a. disminución de la

amoniogénesis Compendio renal

1) Cuál de las siguientes medicinas pueden dar hiperkalemia


a) a y c

2) Hormonas que disminuyen en la falla renal crónica,


excepto c) paratohor mona

3) MgSO4 (PM 120) 1 gramo contiene de magnesio


en meq c) 8.3

4) Factores que conducen a la progresión de falla renal crónica


a) hiperlipidemia
b) toxinas urémicas
c) proteinu
ria c)
todas

5) Paciente de 20 años presenta abundantes vômitos y diarrea de 3 días de


evolución, examen físico depleción de volumen, suero muestra Na 155, K
3meq/l, Cl 117meq/l, HCO3 25 meq/l, tratamiento de elección:
d) solución salina 0.45% + KCl

6) En la enfermedad ateroembólica renal se da lo


siguiente excepto: d) Velocidad de
eritrosedimentación

7) Pseudohiperkalemia de 8 mEq/l que se ve en


el EKG: Onda T picuda
Aplanamiento de la onda P
Ay B
Ninguna

8) Criterios para iniciar diálisis


excepto: Alcalosis Metabolica

9) Hipokalemia con normotension


excepto Síndrome de liddle

10)Plasma creatinina 8mg/dl N de urea 100mg/dl U na


40 Meq/l : Insuficiencia renal

11)Factores reversibles responsables de deterioro de la funcion


renal excepto: Fibrosis intersticial

12)Causa hipokalemia con renina y aldosterona


disminuida: Licorice

13. Sugiere trastorno túbulo intersticial:


.Proteinuria mayor a 3.5 g /24 horas
.Notrmotensión
.Edema
.Osmolarida urinaria normal
14. Glomérulonefritis rápidamente progresiva
proliferación extracapilar ( creo que es esa verificar)

15. Una de las siguientes causas no produce hematuria


.enfer medad de cambios mínimos

16. Plasma Na:115, Posm240 mOsm,Uosm 680,Una 60:meq/l:


.secreción inapropiada de ADH
.
17. en la falla renal crónca produce
necrosis calcifilaxia

18. No produce
hipokalemia: Def. Vit D

19. Paciente con glucosa 90mg/dl, Na+= 115mEq/L, Urea en


140mg/dl b. Síndrome hiperosmolar

20. Causa de seudohiponatremia con osmolaridad


elevada b. Diabetes mellitus

21. Manifestaciones de hipermagnesemia


excepto Nor motensión

22. Piel en uremia crónica


excepto Paño blanco

23. No es causa de necrosis papilar aguda


a.Diabetes mellitas ESTA NO ES CAUSA AGUDA

24. Puede presentarse hiperuricemia


excepto d. Adenocarcinoma
gástrico

25. Compromete riñones y


pulmones: c. LES

26. Sugiere glomerulopatia:


-anasarca

27. TFG < 15ml/min corresponde al estadio de la enfermedad renal cronica:


* 4

28. Causa obstetrica de necrosis cortical bilateral


- desprendimiento prematuro de placenta

29. Promueve amoniogenesis em cirróticos


- hipokalemia

30. Criterio para diagnostico de síndrome nefrosico


a. Proteinuria en 24 hr de 3.5 gr para superficie corporal de 1. 73
b. Albumina serica <
3g/dl c.Hiperlipidemia
y edema d. Todas las
anteriores
1. En la Hiperkalemia:
R/. Hay una menor diferencia entre el potencial de reposo y de umbral

2. La hipokalemia en un cirrotico puede


desencadenar: R/. Coma Hepático

3. La hipermagnesemia puede
producir: R/. Paro
respiratorio

7. Tratamiento de
hipercalcemia: R/.
SSN 0.9%

8. La hipofosfatemia son causas,


excepto: a. Aumento de la
PTH

9. Una solución isotónica:


R/. Aumenta el volumen extracelular

13. La velocidad de administración de potasio en caso de gran riesgo para la


vida puede ser: d. 40 mEq/ h

14. Paciente crónico con deshidratación severa puede presentar lo


siguiente excepto: b. Osmolaridad urinaria de 800
mOsm/kg

15. Paciente con vómito abundante, hipertensión arterial, sodio urinario de 6,


oligúrico, osmolaridad de 800 presenta:
R/.Insuficiencia suprarrenal

Paciente urémico crónico con vómitos severos, hipotensión arterial, orina al azar muestra Na
mas de 40, osmolaridad de 800 puede tener:
Falla suprarrenal

18. Paciente con Insuficiencia renal crónica, con K sérico de 6, leve acidemia, EKG normal, se
trata con:
R/. Kayexalate

22. La hipokalemia predispone,


excepto: R/. Amoniogénesis

25. Cual de las siguientes es causa de


hiperfosfatemia: R/. Insuficiencia
renal

26. Paciente con deshidratación hipertónica e


hipertensión: R/. SSN 0.9%

29. En la Glomerulonefritis Aguda post-infecciosa disminuye la tasa de


filtración: R/. Por alteración en el coeficiente de ultrafiltración

23. Px con IRC tine una taza de filtración glomerular de menor de 25 ml/min ;aunque hay una
tabla de las etapas en esta enfemedad crónica.
Estas osn las TFG:
1. 90 en riesgo
2. 60 -89
3. 30 – 59
4. 15-29
5. Menor a 15 diálisis.
24. Factor que altera el balance hídrico en el periódo
post- operatorio: Disminución de la ADH (Excepto creo que
es calcit riol)

25. Un ejemplo de hipokalemia con aldosterona alta y


osmolaridad baja es: Hiperaldosteronismo primario

26. Ejemplo de hipernatremia hipervolémica: Hiperaldosteronismo primario

27. Hiperkalemia son alteraciones del EKG no es necesario administrar


gluconato de calcio se colocará un antagonista de K así el Px no cae en paro y esto
es si tiene manifestación en el EKG.

28. La fracción de excreción de sodio estará aumentada en falla cardíaca .

29. En la hiperkalemia: hay menos diferencia entre el potencial de reposo y el


umbral. Manifestaciones cardíacas. El problema médico más grave de la
hiperkalemia es la cardiotoxicidad. Los cambios en el ECG producidos por los
niveles altos de potasio son bastante constantes. A medida que aumentan los
niveles se aprecian los siguientes cambios:
Ondas T picudas (con intervalo QT normal o ligeramente reducido)
• Prolongación del intervalo PR con depresión de ST
• Desaparición progresiva de la onda P
• Bloqueo cardíaco progresivo
• Arritmias ventriculares
• Paro cardíaco
Las ondas T picudas constituyen el dato en el ECG más constante en la hiperkalemia.
Efectos neuromusculares. El primer signo neuromuscular de la hiperkalemia suele ser la
aparición de parestesias seguidas de debilidad progresiva de varios grupos musculares. Si el
cuadro se agrava se observa cuadriplejia fláccida. Las funciones cerebrales y de los pares
craneanos se conservan y la parálisis de la musculatura respiratoria puede ocurrir, pero es
excepcional.
30. La infusión de potasio sin riesgo para la vida del PX: es de 40 meq; porque
I lb de solución salina: debe llevr de 20 a40 mEq de KCl a una velocidad de 10 a 20 mEq por
hora; si aumenta la velocidad puede provocar un bloqueo A-V. Se recomienda una Velocidad
mayor si la hipokalemia es muy severa.
Sis e usa venas priféricas es:
10 meq en 100 cc de sol.salina para pasar en una hora
20 meq en 200 cc de sol.salina para pasar en una hora
En vena central:
20 meql en 50 cc pasarla en una hora
40 meq en 100cc pasar la en una hora
31.Los indicios de falla renal aguda son:
(Na filtrado/Na excretado) x 100%
32. Na urinario menor de 10: falla cardíaca , presentará edema estará hinchado y
lo puede llevar a una I.C.C. ( Esto lo saque de Medline: Los valores normales
generalmente son de 15 a 250 mEq/L/día, dependiendo del estado de
deshidratación y la ingesta diaria de sodio en la dieta. Los rangos de los valores
normales pueden variar ligeramente entre diferentes laboratorios. Nota:
mEq/L/día = miliequivalentes por litro por día.)
33. Causa de enuresis: necrosis cortical difusa.
34. En SIDH la concentración urinaria es menor de 20.
35. Deshidratación pura de agua excepto: hay salida de electrolitos del cerebro.
36. Urémico crónico con deshidratación hipertónica severa y acidótico presentará
lo siguiento excepto: osmolaridad urinaria de 8mm mosmol/kg.
37. Px con vómitos abundante, hipertensión arterial; Na urinario=6, oliguria,
osmolaridad de 800 presenta: insuficiencia suprarrenal.
38. Factor que predispone a la hipernatremia son intoxicación hídrica.
39. Se presenta en acidosis tubular renal proximal excepto: NaHCO3 y lo alfa
antagonistas. La hipokalemia produce:
- mayor negatividad del potencial de reposo.

La hipokalemia en un cirrótico puede precipitar COMA por:


- aumento de la amniogénesis

Exceso del kayaxelate en el tratamiento de hiperkalemia puede producir:


- constipación.

Peligro de la hipermagnesemia severa:


- paro respiratori o.

Hipofosfatemia:
- aumenta el calcitriol.

Consecuencia metabólica de hipofosfatemia severa:


- miocardiopatía

Causa de hiperfosfatemia:
- insuficiencia renal

El Síndrome Nefrósicose caracteriza por:


- proteinuria mayor o igual de 3g y albúmina sérica menor o
igual de 3g/dl.

En el Síndrome Nefrósico pueden encontrarse:


- cilindro s céreos

Urémico agudo, oligúrico, acidótico, deshidratado puede presentar:


- osmolaridad urinaria de 250 mosm/ kg

La falla renal crónica avanzada cursa con:


- brecha aniónica elevada

Primera*****Falla renal
crónica: c. Diabetes
Mellitus

- Uno de los indices de falla renal


aguda es: Uosm/Tosm

- Hormona producida en los riñones que disminuye en la falla renal cronica:


Peptido natriuretico (segun arjona)

- Las calcificaciones metastásicas se deben:??


Hip erp aratiroi dism o 1ario

- Causa de anuria:
Necrosis cortical

- la etapa de la insuficiencia renal cronica con tasa de filtración


glomerular de 60-89 ml/min es la:??
2

- La FENA es la relación entre:


depuración de Na/ depuración de creatinina x 100

- En la nefritis tubular intersticial aguda se pierde la capacidad de poder


concentrar la orina debida a:??
Daño medular

- La solucion de NaCl al 23,4 %


tiene: 8000 mOsm/l

- Se asocia con el SIAD,


excepto: lipoma

- El efecto biologico mas importante de la calcitonina es, excepto:


Estimular la resorcion osea osteoclastica

- Favorece la entrada de K al espacio intracelular, excepto:


Agonista adrenergico

- Factores que afectan el balance hídrico en el post-operatorio, excepto:


Calcitriol

- Las primeras manifestaciones clínicas de la hiponatremia son:??


neurológica

- En la hipernatremia hipertonica se da lo siguiente excepto :


Salida de aa

- Manifestaciones clinicas de la hipercalcemia, excepto :


Taquicardia

- En falla renal aguda por nefrotoxicidad:


Se destruye la membrana basal

- Causa de
hipomagnesemia:
alcoholismo

- Na serico de 168 mEq/l 70 kg (utilize como % de agua al 50% y Na normal


140 mEq/l 7 litros

- Indicaciones de diálisis en falla renal aguda, excepto :


Ac. Meta bolica refra ctaria

- El KCL IV se puede asministrar asi, excepto:


a.infusion por linea periferica de 10 meq en 100 ml en 1hora
d. “ “ periferica de 20 meq en 20 ml en 2 horas

- Aumenta la excrecion urinaria de K, excepto:


o Triamtereno

- Factores que regulan la secrecion distal de K, excepto:


o PTH

- Enfermo de 60 Kg con 60% de agua corporal, Posm 260 mOsm/kg,


¿Cuántos gramos de NaCl aumentaria el Na en 10 mEq?
a. 21 gramos

50. Conteniendo 10 g de MgSO4 al %, cuantos ml de sal necesitaria?


20 ml

29. Causa de hiperfosfatemia:


b. Falla renal cronica

30. Causas de hiperfosfatemia grave, excepto:


b. Alcalosis respiratoria

31. Paciente con vómitos severos, oligurico con hipotensión arterial Una>
40…… 800 mOsm/Kg puede tener:
b. Insuficiencia suprarrenal

32. Paciente urémico crónico con deshidratación severa puede presentar


lo siguiente excepto:
a. Osmolarida d urinaria de 800 m Osm/Kg

33. La hipokalemia produce:


a. Aumento de la magnitud del potencial de reposo

17. La PTH promueve todo lo siguiente excepto:


b- au men t o de l a a b s o r c i ó n t u b u l á r d e f o s f a t o . .

18. Un varon de 57 esta con hemodiálisis de mantenimiento para insuficiencia


renal cronica,
¿Cuál de las sig anomalías metabolicas podría ser
anticipáda? b. hi po natr emia

19. Un px de 25años fue ingresado a UCI por lesiones graves en cabeza y fractura de
la base del Graneo, aprox X horas después de la lesion manifestaba poliuria, la OSMU
150, y la de suero 350. Los líquidos IV fueron detenidos y 3 hors después la
producción de orina y la osm urinaria permanecieron sin cambio. Se
administraron 5 unidades de vasopresina IV, la osm de la orina se incremento a
300. ¿Cuál es el dx mas probable?
a- Diabetes insípida central

55. indica falla renal


aguda b- UN = 1

56. deshidratación por perdida de agua y densidad urinaria 1.003,


creatinina 6 b- falla prerenal

57. uno de los indices de falla renales


b. excresion urinaria deNa/oferta tubular de x 100
58. el Sdr hepatorenal es causa de
falla: a - p r er r en a l
59. causa mas común de falla renal
crónica: A- DM

60- en la glomendonefritis aguda por infeccion, la causa,


excepto b- osmolaridad urinaria normal

61. falla renal cronica,


excepto b-
Hipofosfatemia

62. EN la falla renal aguda siempre


habra a- Aumento de
creatinina

63. causa de falla


prerenal c- falla
cardiaca

64. en la nefritis tubular interticial se puede dar,


excepto a. h.ipernatremia

65. Sugiere
glomerulonefritis: b-
cilind ros eritr
ocitarios

66 -causa de falla renal parenquimatosa


a) papilitis

2--En el riñón la hormona paratiroides:


Aumenta la reabsorción tubular de calcio y disminu ye la de fósfor o

3- La hipercalcemia produce
Potencializacióndelosefectosdelahipercalemia

36- La hipomagnesemia produce.


Vasodilatación periférica

36-De los factores: nefrotoxicidad./ -La buena

hidratación 37 - En la concentración sérica para calcular::

8,8

38- la velocidad de administración de potasio: 40 meq

39-hipocalcemia produce : fosfaturia

40-La causa de hipertensión con hipocalemia: Síndrome de Bartter=:,

41-En paciente con vomito severo hipotensión arterial o ir a la sar con


mas de 40 meq Insuficiencia suprarrenal;
42-La ,administración de sol isotónica aumento de vol extracelulares

43-Glomerulonefritis post estreptococcica Altera el cociente de

ultrafiltración 44-En-el túbulo` intersticial se puede dar: La

proteinuria de 4 g al día.
25. Cual de las siguientes opciones esta contraindicada en el tratamiento de la hiperkalcmia
a, gluconato de calcio
b. aldactone
c,
kayaxelate
d. Bicarbonato de sodio e. Todas las anteriores

43. Rotando en sala de medicina interna le piden que evalue un paciente con HIV el cual
porpresentar una meningitis por criptococci recibe anfotericina B; dicho este medicamento
buscaría como complicación:
c- hipokalemia

81- donde se halla el calcio en el organismo:


c- 99 % hueso

aumento de PTH
b - a u me n t o d e l a a b so r c i ó n i n t es t i n a l d e f os f a t o

9- una solucion isotonica


a- aumenta el volumen extracell

10- la vel de administración de potasio en caso de riesgo para


la vida a—40 meq por hora

11- Px cronico con deshidratación severa puede presentar los siguiente


excepto: a-osmolaridad urinaria de 800 mosmol/Kg

12- Px con vomito abundante, HTA, sodio urinario en 6, oligurico,


Osmolaridad de 800 presente,
a- insuficiencia suprarrenal

13- Px uremico cronico con vomito severo hipotension arterial orina al


azar, muestra sodio mas de 40, Osmolaridad de 800 puede tener :
a- Falla suprarrenal

26. Cuándo disminuye el


fosfato? R. Aumento de
PTH

27. Ejemplo de hipernatremia


hipervolémica: R. Aldosteronismo
primario

28. La hipocalemia produce:


R. Aumento de la magnitud del potencial de reposo

29. La depleción de agua produce, excepto:


R. Salida de electrolitos del cerebro

30. Velocidad de administración de KCl en caso de


urgencia: R. 40 mEq/h

31. Cambios en EKG de la hipermagnesemia es


igual: R. Hipercalemia

32. Enfermo de 60 Kg con 60% de agua corporal, osmolaridad plasmática 260


mosmol/Kg,
¿cuántos gramos de NaCl aumenta el Na en 10 mEq en 24 horas?
b) 21

33. Uso de Calcio en tratamiento de


hipercalemia para: R. Modificar el
Potencial de Membrana

34. Hipermagnesemia severa produce:


R. Paro respiratorio

35. Venoclisis de 1L de D/A 5% conteniendo 10 mg de MgSO 4 y


al 50%: R. 20 mL

36. La hipocalcemia:
R. Prolongo QT a expensas del segmento ST

37. Tratamiento de urgencia de


hipercalcemia: R. SSN 0.9%

38. Causa de
hiperfosfatemia: R.
Insuficiencia renal

39. Adaptación cerebral, hipertonicidad,


excepto: R. Pérdida de iones

40. En la adaptación cerebral, la hipotensión da lo siguiente, excepto:


R. Ganancia de aminoácidos
41. En ausencia de síntomas de hipernatremia, el sodio debe ser
disminuido a razón de: R. 0.5 mEq/L/h

42. Para corregir un sodio cerca de 185 mEq se administró en 86 horas, 66 letras de
D/SS 5%.
¿cuál es la velocidad de administración de la venoclisis?
R. 69 mL ¿?????

43. Manifestación clínica de hipercalcemia, excepto:


R. a) Hipotensión arterial

44. Consecuencia de hipofosfatemia, excepto:


R. a) No afecta al SNC

Causa de hipofosfatemia, excepto:


R. a) Falla renal

45. Causa de HTA con hipocalemia, aldosterona-


renina baja: R. Síndrome de Cushing

46. Causas de HTA con hipocalemia y


renina alta: R. Hipertensión maligna

47. En una deshidratación severa hipertónica con hipotensión arterial severa, la


prioridad inicial es administrar:
R. SSN 0.9%

48. En la secreción inapropiada de ADH se da lo siguiente, excepto:


R. No hay expansión de volumen
49. Tejido con mayor contenido
de K+: b) Músculo

50. La insuficiencia renal funcional de la cirrosis hepática es


debida a: R. Vasoconstricción de las arterias renales
RENAL:

23. Px con IRC tine una taza de filtración glomerular de menor de 25 ml/min ;aunque hay yuna
tabla de las etapas en esta enfemedad crónica.
Estas osn las TFG:
1. 90 en riesgo
2. 20 ( con TFG normal o
incrementada) 3. 60 -89
4. 30 – 59
5. 15-29
Menor a 15 diálisis.

24. Factor que altera el balance hídrico en el periódo post-


operatorio: Disminución de la ADH (Excepto)

25. Un ejemplo de hipokalemia con aldosterona alta y


osmolaridad baja es: Hiperaldosteronismo primario

26. Ejemplode hipernatremia hipervolémica: Hiperaldosteronismo primario

27. Hiperkalemioa son alteraciones del EKG no es necesario administrar gluconato


de calcio se colocará un antagonista de K asíe l Px no cae en paro y estoes si tiene
manifestación en el EKG.

28. La fracción de excreción de sodio estará aumentada en falla cardíaca.

29. En la hiperkalemia: hay menos diferencia entre el potencial de reposo y el


umbral. Manifestaciones cardíacas. El problema médico más grave de la
hiperkalemia es la cardiotoxicidad. Los cambios en el ECG producidos por los
niveles altos de potasio son bastante constantes. A medida que aumentan los
niveles se aprecian los siguientes cambios:
• Ondas T picudas (con intervalo QT normal o ligeramente reducido)
• Prolongación del intervalo PR con depresión de ST
• Desaparición progresiva de la onda P
• Bloqueo cardíaco progresivo
• Arritmias ventriculares
• Paro cardíaco
Las ondas T picudas constituyen el dato en el ECG más constante en la hiperkalemia.
Efectos neuromusculares. El primer signo neuromuscular de la hiperkalemia suele ser la
aparición de parestesias seguidas de debilidad progresiva de varios grupos musculares. Si el
cuadro se agrava se observa cuadriplejia fláccida. Las funciones cerebrales y de los pares
craneanos se conservan y la parálisis de la musculatura respiratoria puede ocurrir, pero es
excepcional.
30. La infusión de potasio sin riesgo para la vida del PX: es de 40 meq; porque I lb
de solución salina: debe llevr de 20 a40 mEq de KCl a una velocidad de 10 a 20
mEq por hora; si aumenta la velocidad puede provocar un bloqueo A-V. Se
recomienda una Velocidad mayor si la hipokalemia es muy severa.
Sis e usa venas priféricas es:
10 meq en 100 cc de sol.salina para pasar en una hora
20 meq en 200 cc de sol.salina para pasar en una hora
En vena central:
20 meql en 50 cc pasarla en una hora
40 meq en 100cc pasar la en una hora
31.Los indicios de falla renal aguda son:
(Na filtrado/Na excretado) x 100%
32. Na urinario menor de 10: falla cardíaca , presentará edema estará hinchado y
lo puede llevar a una I.C.C. ( Esto lo saque de Medline: Los valores normales
generalmente son de 15 a 250 mEq/L/día, dependiendo del estado de
deshidratación y la ingesta diaria de sodio en la dieta. Los rangos de los valores
normales pueden variar ligeramente entre diferentes laboratorios. Nota:
mEq/L/día = miliequivalentes por litro por día.)
33. Causa de enuresis: necrosis cortical difusa.
34. En SIDH la concentración urinaria es menor de 20.
35. Deshidratación pura de agua excepto: hay salida de electrolitos del cerebro.
36. Urémico crónico con deshidratación hipertónica severa y acidótico presentará
lo siguiento excepto: osmolaridad urinaria de 8mm mosmol/kg.
37. Px con vómitos abundante, hipertensión arterial; Na urinario=6, oliguria,
osmolaridad de 800 presenta: insuficiencia suprarrenal.
38. Factor que predispone a la hipernatremia son intoxicación hídrica.
39. Se presenta en acidosis tubular renal proximal excepto: NaHCO3 y lo alfa
antagonistas.
1. En la Hiperkalemia:
R/. Hay una menor diferencia entre el potencial de reposo y de umbral

2. La hipokalemia en un cirrotico puede


desencadenar: R/. Coma Hepático

3. La hipermagnesemia puede
producir: R/. Paro
respiratorio

7. Tratamiento de
hiprecalcemia: R/.
SSN 0.9%

8. La hipofosfatemia son causas,


excepto: a. Aumento de la
PTH

9. Una solución isotónica:


R/. Aumenta el volumen extracelular

13. La velocidad de administración de potasio en caso de gran riesgo para la


vida puede ser: d. 40 mEq/ h

14. Paciente crónico con deshidratación severa puede presentar lo


siguiente excepto: b. Osmolaridad urinaria de 800
mOsm/kg

15. Paciente con vómito abundante, hipertensión arterial, sodio urinario de 6,


oligúrico, osmolaridad de 800 presenta:
R/.Insuficiencia suprarrenal

Paciente urémico crónico con vómitos severos, hipotensión arterial, orina al azar muestra Na
mas de 40, osmolaridad de 800 puede tener:
Falla suprarrenal

16. Todo lo siguiente es cierto acerca de Colitis Ulcerosa, EXCEPTO:


R/. No está en relación las alteraciones inmunológicas.

17. En quien predomina la enfermedad de


Hígado Graso: R/. Sexo masculino

18. Paciente con Insuficiencia renal crónica, con K sérico de 6, leve academia, EKG
normal, se trata con:
R/. Kayetzalate

22. La hopikalemia predispone,


excepto: R/. Amoniogénesis

25. Cual de las siguientes es causa de


hiperfosfatemia: R/. Insuficiencia
renal

26. Paciente con deshidratación hipertónica e


hipertensión: R/. SSN 0.9%

29. En la Glomerulonefritis Aguda post-infecciosa disminuye la tasa de


filtración: R/. Por alteración en el coeficiente de
ultrafiltración

30. Causa de Insuficiencia preerenal, excepto:


R/. Aneurisma Disecante de la Aorta

2) Cuál de las siguientes medicinas pueden dar hiperkalemia


b) inhibidor de la enzima convertidota
c) bloqueador alfa adrenérgico
d) bloqueador beta adrenérgico
e) ayb
f) a yc

2) Hormonas que disminuyen en la falla renal crónica,


excepto c) paratohor mona

3) MgSO4 (PM 120) 1 gramo contiene de magnesio


en meq c) 8.3

4) Factores que conducen a la progresión de falla renal crónica


a) hiperlipidemia
b) toxinas urémicas
c) proteinu
ria c)
todas

5) Paciente de 20 años presenta abundantes vômitos y diarrea de 3 días de


evolución, examen físico depleción de volumen, suero muestra Na 155, K
3meq/l, Cl 117meq/l, HCO3 25 meq/l, tratamiento de elección:
d) solución salina 0.45% + KCl

6) En la enfermedad ateroembólica renal se da lo


siguiente excepto: d) Velocidad de
eritrosedimentación

7) Pseudohiperkalemia de 8 mEq/l que se ve en


el EKG: Onda T picuda
Aplanamiento de la onda P
Ay B
Ninguna

8)Criterios para iniciar diálisis


excepto: Alcalosiss
Metabolica

9)Hipokalemia con normotension


excepto Síndrome de liddle

10) Plasama creatinina 8mg/dl N de urea 100mg/dl U na


40 Meq/l : Insuficiência renal

11) Factores reversibles responsables de deterioro de la funcion


renal excepto: Fibrosis intersticial

12) Causa hipokalemia con renina y aldosterona


disminuida: Licorice

13. Sugiere trastorno túbulo intersticial:


.Proteinuria mayor a 3.5 g /24 horas
.Notrmotensión
.Edema
.Osmolarida urinaria normal

14. Glomérulonefritis rápidamente progresiva


proliferación extracapilar ( creo que es esa
verificar)

15. Una de las siguientes causas no produce hematuria


.enfer medad de cambios mínimos

16. Plasma Na:115, Posm240 mOsm,Uosm 680,Una 60:meq/l:


.secreción inapropiada de ADH

17. en la falla renal crónca produce


necrosis calcifilaxia

18. No produce
hipokalemia: Def. Vit D

19. Paciente con glucosa 90mg/dl, Na+= 115mEq/L, Urea en 140mg/dl


b. Síndrome hiperosmolar
20. Causa de seudohiponatremia con osmolaridad elevada
b. Diabetes mellitus

21. Manifestaciones de hipermagnesemia


excepto Nor motensión

22. Piel en uremia crónica


excepto Paño blanco

23. No es causa de necrosis papilar


aguda a.Diabetes mellitas
b. Uropatía obstructiva
c. Pielonefritis
d. Trombosis de venas renal.
TODAS SON

24. Puede presentarse hiperuricemia


excepto d. Adenocarcinoma
gástrico

25. Compromete riñones y


pulmones: c. LES

26. Sugiere glomerulopatia:


-anasarca

27. TFG < 15ml/min corresponde al estadio de la enfermedad renal cronica:


* 4

28. Causa obstetrica de necrosis cortical bilateral


- desprendimiento prematuro de placenta

29. Promueve amoniogenesis em cirróticos


- hipokalemia

30. Criterio para diagnostico de síndrome nefrosico


a. Proteinuria en 24 hr de 3.5 gr para superficie corporal de 1. 73
b. Albumina serica <
3g/dl c.Hiperlipidemia
y edema
d. Todas las anteriores
e. ay b

101. Paciente masculino de 33 años llega a su consulta por presentar fiebre, un


pequeño rash, signo de Jordano positivo; el medico de turno le envía un urinalisis
que revela proteinuria, hematuria y múltiples leucocitos; dentro de su diagnostico
diferencial, debe estar una de las siguientes patologías, cual:

a. Nefritis Intersticial Aguda. Rash, leucositosis puede ser un LES

102. Dentro de las Glomerulopatias, existen algunas, que se pueden presentar con
Síndrome Nefrosico, y otras con Síndrome Nefrítico, cual de las siguientes se suele
presentar con Síndrome Nefrítico:

a. Nefropatia IgA.

103. En un paciente de 65 años, que desarrolla un Sd. Nefrítico Agudo, la etiología


mas común causando de la Glomerulopatia es:
a. Vasculitis.

104. Cual de las siguientes Glomerulopatia, siempre desarrolla Insufiencia Renal:


a. Rápidamente Prog resiva.

105. En un paciente con SIADH, al medirle la osmolaridad urinaria, la podemos


encontrar:
a. Elevada.

106. La hiponatermia, se hace clínicamente evidente cuando el sodio, ha disminuido


a:
a. 125 mEq/lt.
107-111. Mencione 5 medicamentos que puedan causar hiperkalemia:
a. aldactone
b. captopril
c. losartan
d. carvedilol
e. Lisinopril

112-117. Las características del Síndrome Nefrotico son:


a. edema
b. hipoalbuminemia
c. hipercoagulabilidad
d. proteinuria >3.5
e. hiperlipidemia
f. cilindros lipid
g. icos en orina .

118-120. Los tres puntos en los que se basa la corrección de la hiperkalemia son:
a. aumento de excresion renal.
b. Disminucion de trasvase celular de K (salida de celulas
c. disminucion d eposicion de K.

121-123. Para cada uno de estos puntos, se suele usar: se le solicita al estudiante ser lo mas
explicito posible en su respuesta, en cuanto a dosis y tratamiento exacto.
a. SSN 0.9, gluconato cálcico y aument de Na renal
b. Insulina,dism relajantes musculares
c. Dieta sin K platano, tomate, naranja, espinaca. Quitar fármaco que
aumente K

124-126. Debemos sospechar, de alteraciones del Magnesio, en que pacientes:


a. ALCOHOLICOS,QUE USEN DIURETICOS DE ASA
b. CIRROTICOS, SD DE MALA ABSORCION
c. USEN LAXANTES, TX DE PREECLAMSIA.

127. Paciente con hipokalemia + alcalosis metabólica + hipertensión arterial +


disminución de la renina plasmática, esta desarrollando un:
HIPERTENSION MALIGNA CREO.

128. Única causa de diarrea con alcalosis metabólica es:


HIPOPOTASEMIA, SD DE GITELMAN O BARTTER

129. Si un paciente, tiene un potasio de 3 mEq/lt, decimos que ha perdido ya:

130. Cual es la principal preocupación que se tiene, cuando un paciente tiene un


aumento en los niveles plasmáticos de Magnesio:

a. Bloqueo AV complete depression respiratoria


131-132. Causas de acidosis metabólica con hipokalemia:

a. diarrea con perdida de K.


b. INTOXICACION POR TOLUENO,
c. EXCRESION DE ANION NO REABSSORBIBLE CAD.
d. Acidosis tubular renal

133. Que examen le ayudaria para diferenciarlas:

a. POTASIO EN ORINA, ESTADO AC BASE,

134. Que esperaria encontrar en una ATR tipo 2:

a. HIPOCALEMIA Y AC METABOLICA

135-136. Principales causas de hipercalcemia:

a. HIPERPARATIROIDISMO 1
b. NEOPLASIA MALIGNA POR TUMOR SECRETOR DE PTH
c. Rabdomiolisis

d. Cetoacidosis diabetica

e. 136-137. Principales causas de hipocalcemia:

a. HIPOPARATIROIDISMO SD DI GEORGE,
CONGEITO, QUIRURGICO
b. DEFICIENCIA DE VIT D SD NEFROTICO IRC
c. Déficit de Vitamina D.
d. Insuficiencia renal crónica.
e. Déficit de magnesio.
f. Alcoholismo.
g. Tratamiento con bifosfonatos: medicamentos para tratar niveles
altos de calcio en sangre o píldoras para tratar la osteoporosis.
h. Ciertos tipos de leucemia o trastornos sanguíneos.
i.

138-139. Consecuencias de la hiperfosfatemia:

a. HIPERPARATIROIDISMO
b. CALCIFICACION ECTOPICA
140-143. El Sd. De Barter, se caracteriza clínicamente por:

a. HIPOPOTASEMIA
b. ALCALOCIS MET
c. HIPERCALCIURIA
d. hipocloremia, nefrocalcinosis
e. hypotension o normotension

144. En un paciente con diagnostico de Glomerulonefritis post estreptococcica, el


diagnostico serologico se basa en el hallazgo de anticuerpos:

a. Antiestreptolisina O.

145. En la Nefiritis secundaria al LES, se suelen encontrar positivos los:


a. ANA.

146. Si un paciente es diagnosticado con Falla Renal Crónica, el siguiente punto es


el estadiage del mismo, si tiene una TFG en 82 ml/minuto, esta en estadio:
a. II

147-148. La Osteodistrofia Renal, se va a caracterizar clínicamente por la presencia de:


a. DOOR OSEO, FRACTURAS, HIPERPTH 2
b. Osteitis fibrosa quistica
c. Osteomalacia
149.Normalmente la Kps entre el Fósforo y el calcio es de:
a. .

150-154. Las características de una muestra de orina con sedimento de tipo nefrítico, son:

a. HEMATURIA_>5
b. PROTEINURIA <3.5
c. CILINDROS_ERITROCITARIOS

155-158. Mencione algunos parámetros de laboratorio, para pensar en una IRC Renal:
a. ALBUMINURIA
b. hipercloremia
c. Hiperfosfatemia
d. BUN AUMENTADO
e. proteinuria, hematuria
f. Cilindros hialinos con sedimento normal, TFG
DISMINUIDA, CRET AUMENTADA

159. El Gasto cardiaco normal es de 5 – 6 litros por minuto; de los cuales, un 25% le
corresponde al riñón, en una relación de:
a. 1300 ml/ min.

160. Si un paciente, diagnosticado con FRC, presenta ademas una acidosis


metabolica hipercloremica, usted sabe que este paciente debe tener una Brecha
Anionica:
a. Aumentada.

161. Este mismo paciente, por tener estas características clínicas, debe encontrarse
en estadios de la FRC:
a. Avanzados.

162. Para que por una obstrucción, aumente la creatinina en IRA, se deben haber
perdido:
a. 4/4 partes de la masa renal .

163. La lesión histologica mas común de IRA es:


a. Necrosis tubular aguda.!!!

164. Y la mas peligrosa y temida es:


a. Necrosis cortical bilateral.

165. Cual de las siguientes, es la principal causa de IRA, en la maternidad.


a. Desprendimiento prematuro de mem brana.

Tercer Parcial de Medicina Interna

1. Cuándo disminuye el fosfato?


R. Aumento de PTH

2. Ejemplo de hipernatremia hipervolémica:


R. Aldosteronismo primario

3. La hipocalemia produce:
R. Aumento de la magnitud del potencial de reposo

4. La depleción de agua produce, excepto:


R. Salida de electrolitos del cerebro

5. Velocidad de administración de KCl en caso de urgencia:


R. 40 mEq/h

6. Cambios en EKG de la hipermagnesemia es igual:


R. Hipercalemia

88
7. Enfermo de 60 Kg con 60% de agua corporal, osmolaridad plasmática 260 mosmol/Kg,
¿cuántos gramos de NaCl aumenta el Na en 10 mEq en 24 horas?
R. a) 15
b) 21
c) 30
d) 19

8. Uso de Calcio en tratamiento de hipercalemia para:


R. Modificar el Potencial de Membrana

9. Hipermagnesemia severa produce:


R. Paro respiratorio

10. Venoclisis de 1L de D/A 5% conteniendo 10 mg de MgSO4 y al 50%:


R. 20mL

11. La hipocalcemia:
R. Prolongo QT a expensas del segmento ST

12. Tratamiento de urgencia de hipercalcemia:


R. SSN 0.9%

13. Causa de hiperfosfatemia:


R. Insuficiencia renal

14. Adaptación cerebral, hipertonicidad, excepto:


R. Pérdida de iones

15. En la adaptación cerebral, la hipotensión da lo siguiente, excepto:


R. Ganancia de aminoácidos
16. En ausencia de síntomas de hipernatremia, el sodio debe ser disminuido a razón de:
R. 0.5 mEq/L/h

17. Para corregir un sodio cerca de 185 mEq se administró en 86 horas, 66 letras de D/SS
5%. ¿cuál es la velocidad de administración de la venoclisis?
R. 69 mL ¿?????

18. Manifestación clínica de hipercalcemia, excepto:


R. a) Hipotensión arterial
b) Bradicardia
c) Nauseas
d) Anorexia

19. Consecuencia de hipofosfatemia, excepto:


R. a) No afecta al SNC
b) Disfunción plaquetaria
c) Cardiomiopatía
d) Osteomalacia

20. Causa de hipofosfatemia, excepto:


R. a) Falla renal
b) Recuperación de cetoacidosis diabética
c) Alcalosis respiratoria
d) Quemaduras graves

21. Causa de HTA con hipocalemia, aldosterona-renina baja:


R. Síndrome de Cushing

22. Causas de HTA con hipocalemia y renina alta:


R. Hipertensión maligna

89
23. En una deshidratación severa hipertónica con hipotensión arterial severa, la prioridad
inicial es administrar:
R. SSN 0.9%

24. En la secreción inapropiada de ADH se da lo siguiente, excepto:


R. No hay expansión de volumen

25. Tejido con mayor contenido de K+:


R. a) Hígado
b) Músculo
c) Eritrocito
d) Hueso

Produce hepatitis crónica, cirrosis y hepatocarcinoma:


1) __F__HAV 2) __C__HBV
3) __C__HCV 4) __F__HEV

Son virus ADN:


5) __F__ HAV 6) __C__ HBV
7) __F__ HCV 8) __F__HEV

Se transmite por la ruta fecal-oral


9) __C__HCV 10) __F__HBV
11) __F__HCV 12) __C__HEV

En la encefalopatía hepática existe:


a. Estimulación dopaminergica
b. Estimulación serotoninergica
c. Estimulación gabaergica
d. Estimulación noradrenergica
e. Ninguna de las anteriores

8) La enfermedad de Crohn se puede confundir con excepto:


a. Colitis ulcerativa
b. Síndrome del colon irritable
c. Colitis amebiana
d. Colitis inespecífica
e. Colitis por tuberculosis

9) ¿Cuál de las siguientes afirmaciones sobre la colangitis esclerosante primaria asociada a la


enfermedad inflamatoria intestinal es falsa?
a. Se presenta con mayor frecuencia en paciente con enfermedad de Crohn que con
colitis ulcerativa
b. Es más frecuente en pacientes con colitis extensa que en aquellos con colitis distales o
proctitis
c. La colangiografía retrógrada endoscópica es la prueba diagnóstica más útil para esta
enfermerdad
d. Alrededor del 70% de los pacientes presentan títulos elevados de anticuerpos
anticitoplasma del neutrófilo
e. La aparición de cierto grado de colestasis suele ser el primer signo de la enfermedad antes
de la aparición de cualquier manifestación clínica

10) Período de incubación de la hepatitis A


a. 5-10 días
b. 10-15 días
c. 15-45 días
d. 45-60 días
e. Mas de 60 días

11) ¿Cuál de los siguientes fármacos es útil en la prevención de la recidiva de la enfermedad


inflamatoria intestinal y no es activo en el brote agudo?
a. Prednisona
b. Ciclosporina
c. 5-aminosalicilato

90
d. Budenonida
e. Azatioprina

12) El hiperaldoteronismo de la cirrosis hepática se debe:


a. Aumento de la producción suprarenal
b. Disminución de la metabolización hepática
c. Disminución de la eliminación renal
d. Todas la anteriores
e. Ninguna de las anteriores

13) La elevación de la alfafetoproteína sérica puede dectetarse en excepto:


a. Carcinoma primitivo de higado
b. Carcinoma germinal del ovario
c. Carcinoma pancreático
d. Carcinoma testicular
e. Hepatitis aguda

14) La mejor técnica diagnóstica para la coledocolitiasis es:


a. Colangiografía intravenosa
b. Colecistografía oral
c. Ecografía
d. Colangiografía transparietohepàtica
e. Colangiografía retrógrada

15) ¿A que edad suele aparecer con mayor frecuencia la enfermedad de Crohn?
a. Antes de los 10 años
b. Entre 20 y los 30 años
c. Entre los 40 y 50 años
d. Entre los 60 y 70 años
e. Puede aparecer a cualquier edad, sin preferencia alguna

16) Las perdidas fecales excesivas de potasio evocan:


a. Colitis ulcerativa
b. Enfermedad de Crohn
c. Colon irritable
d. Poliposis rectocólica familiar
e. Tumor velloso

17) De las complicaciones anorrectales de la enfermedad de Crohn la más frecuente:


a. Fistula anal
b. Fisura anal
c. Absceso perianal
d. Carcinoma anal
e. Prolapso rectal

18) De todas las siguientes alteraciones, ¿cuál no es mejorada por el tratamiento esteroideo en la
enfermedad de Crohn?
a. Fiebre b. Diarrea c. Dolor abdominal d. Número de recurrencia e. Ninguna de las
anteriores

19) El antigeno carcinoembrionario es útil en el estudio de los pacientes con cánceres colorectales:
a. Es específico para estos tumores
b. Detecta precozmente la aparición del cáncer
c. Es eficaz para la detección de recidivas después de la extirpación del tumor
colorrectal
d. Aumenta después de estudios endoscópicos
e. Aumenta después de estudios radiológicos

20) La clasificación de Dukes del cáncer de colon tiene en cuenta los siguientes parámetros:
a. Invasión del tumor en la pared intestinal
b. Estado de los ganglios regionales
c. Afección de órganos a distancia del colon
d. Todas las anteriores
e. Ningunas de las anteriores

91
21) Paciente Polo Gomex de 30 años con ictericia ALT 2200, AST 2000, HBcAb IgG post, HBsAb
IgG post, HAV Ig M post. Qué tiene. Nota: valores normales de ALT y AST 15-35 U.I.
a. Hepatitis E
b. Inmune contra la hepatitis A
c. Hepatitis A
d. Hepatitis B crónica
e. Hepatitis B aguda

22) Además el paciente Polo Gomez tiene:


a. Vacunado contra la hepatitis B
b. Inmune contra la hepatitis A
c. Inmunidad natural contra hepatitis B
d. hepatitis B crónica
e. Hepatitis B aguda

23) La exploración inicial idónea en el diagnóstico de la hemorragia digestiva por hipertensión


portal es:
a. La anamnesis y exploración clínica detallada b. La arteriografía selectiva
c. Serie esófago-gastro-duodenal d. La endoscopia alta e. Songrama de hígado, vías
biliares.

24) El flapping tremor:


a. Es un signo de hepatopatía crónica
b. Indica la existencia de un delirium tremens
c. Es un signo de encafalopatía hepática
d. Su ausencia descarta la existencia de una encefalopatía hepática
e. Consiste en un temblor intencional de las manos

25) Pueden ser característica de transformación maligna de un pólipo excepto:


a. Tamaño b. Tipo histológico c, Presencia de atipías epiteliales
d. Presencia de un pedicuro inferior a 1 cm e. Ningunas de las anteriores

26) De la siguientes afirmaciones con respecto a la colitis ulcerosa. ¿Cuál es falsa?


a. No se debe a deficiencias nutricionales
b. No se debe a un agente infeccioso
c. No está en relación con alteraciones inmunológicas
d. No se debe a alteraciones vasculares
e. No está en relación con tóxicos ambientales

27) Todo es cierto con respecto al cáncer gástrico, excepto:


El tumor temprano puede afectar hasta la submucosa
a) Los polipos gástricos tienen potencial bajo de malignidad
b) La mayoria de los pacientes tienen gastritis crónica atrófica
c) Histologicamente, la mayoria son adenocarcinomas

28) Cual de los siguientes síntomas es menos probable encontrar en pancreatitis:


a) Dolor abdominal epigástrico b) Diarrea c) Taquipnea d) Vómitos incohercible

29) Todo lo siguiente es cierto para el cáncer de páncreas, excepto:


a) La mayoría se encuentra en la cabeza
b) La ictericia es su principal síntoma
c) La mayoria de los casos no son operable al momento del diagnóstico
d) La Diabetes mellitus es su principal factor de riesgo

30) Factores en clasificación Child-Pugh, excepto:


a) Transaminasas
b) Ascitis
c) Bilirrubina
d) Albúmina
e) INR

31) Abunda el potasio en el


a. Hueso b. Eritrocito
c. Hígado d. Músculo
e. Espacio intravascular

92
32) Hipernatremia normovolémica
a. Diuresis osmótica
b. Quemaduras
c. Hiperaldosteronismo primario
d. Síndrome de Cushing
e. Diabetes insípida

33. Abunda el magnesio en el


a. Hueso b. Eritrocito
c. Hígado d. Músculo e. Espacio intracelular

34. Hiponatremia hipervolémica de causa extrarrenal


a. Pancreatitis
b. Quemaduras
c. Deficiencia de mineralocorticoides
d. Deficiencia de glucocorticoides
e. Cirrocis

35. Consecuencias de la hipokalemia a nivel hormonal


a. Aumento renal del amonio
b. Disminución en la liberación de insulina
c. Aumento de la TSH
d. Aumento de la liberación de insulina
e. Aumento de la ADH

36. Son manifestaciones de la hipermagnesemia, excepto:


a. Insuficiencia respiratoria
b. Letargia
c. Hiperreflexia
d. Inhibición de la transmisión neuromuscular
e. Bloqueo cardíaco completo

37. Se considera hipercalcemia severa, una concentración de calcio de:


a. 2,6 mM/L b. 14 mg/dl
c.4 mEq/L d. 3,5 mM/L
e. b y d

38. La diminución del fosfato sérico produce, excepto:


a. Aumento del calcitriol
b. Aumento de la PTH
c. Aumento de la absorción intestinal de fosfato
d. Aumento de la absorción intestinal de calcio
e. Disminución de la excreción urinaria de calcio

39. Inhibe la resorción ósea


a. Kayexalate b. Calcitonina
c. Bisfosfonatos d. Ciclosporina
e. Ácido ascórbico

40. Tratamiento de elección en la alcalosis metabólica con hipokalemia severa


a. D/A 5% + KCl b. Solución salina al 3% + KCl
c. Inhibidor de la anhidrasa carbónica d. Solución salina al 0,9% + KCl
e. Lactato Ringer

41. Se puede presentar en la hipercalcemia, excepto


a. Constipación b. Hipotensión
c. Náuseas d. Vómitos
e. Acortamiento del QT

42. Se puede presentar en la hipocalcemia, excepto


a. Parestesia b. Hipotensión
c. Laringoespasmo d. Depresión
e. Hipertensión

43. Consecuencia de la hipofosfatemia, excepto


a. Cardiomiopatía b. Osteomalacia
c. No afecta el sistema nervioso central d. Hemólisis
e. Raquitismo

44. En la rabdomiólisis se puede dar lo siguiente, excepto


a. Hipocalcemia b. Hiperkalemia
c. Hipercalcemia d. Hiperuricemia
e. Hiperfosfatemia

45. En la desmielinización osmótica se da lo siguiente, excepto

93
a. Monoparesia b. Disartria
c. Paraparesia d. Cuadriparesia
e. Coma

46. Hiperkalemia con tasa de filtración glomerular menor de 20 ml/min


a. Enfermedad de Addison b. Obstrucción del uréter derecho
c. Hipotensión arterial leve d. Uso de furosemida
e. Falla renal aguda

47. Aumento de la osmolaridad sérica con sodio disminuido


a. Hiperproteinemia severa b.Hiperlipidemia severa
c. Intoxicación por etilenglicol d. Uso de manitol
e. Intoxicación por metanol

48. Hipokalemia con aldosterona alta y renina baja


a. Deficiencia de 17 α hidroxilasa b. Exceso de glucocorticoides
c. Estenosis de la arterial renal d. Hiperaldosteronismo primario
e. Hiperaldosteronismo secundario

49. Tratamiento de urgencia de la hipercalcemia


a. D/A 5% b.Solución salina al 0,9%
c. Bisfosfonatos d. Solución salina al 3%
e. Lactato Ringer

50. En ausencia de síntomas de hipernatremia, el sodio debe disminuirse, en mEq/L, a razón de


a. 1
b. 2
c. 3
d. 0.5
e. 0.8

EXAMEN PARCIAL
OCTUBRE 2005

Un Px q acude con Ictericia, transaminasas elevadás por lo cual sospecha que tiene una
hepatitis viral. Si el período es de 20 a 110 días; sospecharía de hepatitis. C- Hep C

Fármaco útil en prevención de la recidivada de la enfermedad inflamatoria intestinal y no es


activo en brote agudo: C- azatriopina

7. En un pte alcoholico con ascitis por cirrosis Hepatica, hiperaldosteronismo, en la fisiopatología


de esta enfermedad esto se
A- aumento de la producción de renina por hipovolemia
b. disminución de la producción de renina por hipovolemia
c, aumento de la producción de renina por hipernvolemia
d. disminución de la producción de renina por hipervolemia
e. secreción de resina normal por normovolemía

8. Síntoma iñicial de la cirrosis biliar: B- Prurito

9. Un paciente de 43 años llega al ER con dolor abdominal y a la palpación profunda presenta dolor
en hipocondrio derecho- Maniobra que emplearía: '
a. signo de W ilson
b- signo de Hugo-Curtir
c. signo de Murphy
10. Px 60 años con dx de colangitis se le inicia tx con antibióticos empírico contra bacterias
lent as y la fiebre no cede en 48 horas, la conducta siguiente serla: D. Procede inmediatamente
al tx quirúrgico

12- La mejor técnica diagnostica para la colédoco litiasis es:


R la colangiografia Retrograda endoscopica

14. Px 60 años con ascitis por ingesta de alcohol, investigar si se trata de cirrosis hepática, para
su Dx es necesario
a- TP
b- TPT
c. recuento de leucocitos
d. Factor VIII
e- azatriopina

16. Un pte de 30a acude por diarrea crónica ligeramente... -al examen fisico normal excepto q la
region anal presenta fístulas, debido a esto ud pensaría q este paciente tiene:
a) TBC intestinal
b) enfermedad de Crohn
c) t. colitis ulcerativa

94
d) d. carcinoma anal
e) t. prolapso anal

17. La PTH promueve todo lo siguiente excepto:


a aumento de la absorción intestinal de calcio
b- aumento de la absorción tubulár de fosfato.
c. aumento de la resorción ósea de fosfato
d- aumento de la resorción osea de calcio
e, disminución de la reabsorción de fosfato

18. Un varon de 57 esta con hemodiálisis de mantenimiento para insuficiencia renal cronica, ¿Cuál
de las sig anomalías metabolicas podría ser anticipáda?
a. hipernatremia
b. hiponatremia
c. exceso de vitamina D
d. hipoparatiroidismo

19. Un px de 25años fue ingresado a UCI por lesiones graves en cabeza y fractura de la base del
Graneo, aprox X horas después de la lesion manifestaba poliuria, la OSMU 150, y la de suero 350.
Los líquidos IV fueron detenidos y 3 hors después la producción de orina y la osm urinaria
permanecieron sin cambio. Se administraron 5 unidades de vasopresina IV, la osm de la orina se
incremento a 300. ¿Cuál es el dx mas probable?
a- Diabetes insípida central
b. diabetes insípida nefrogenica
c. intoxicación acuosa
d. sobrecarga de solutos
e. S IADH

55. indica falla renal aguda


a- U..N.. = 4.1
b- UN = 1
c. U..N.. = 1/4i/
d. Gravedad especifica 1.026
e. Ninguna

56. deshidratación por perdida de agua y densidad urinaria 1.003, creatinina 6 /


a. falla renal aguda parenquimatosa
b- falla prerenal
c. falla renal obstructiva
d. falla renal cronica
e. no hay falla renal

57. uno de los indices de falla renales


a. reabsorción de Na/oferta tubular de Na x 100
b. excresion urinaria deNa/ oferta tubular de x 100
c.- oferta tubular de Na/excrecion urinaria de Na x 100 ~.--~
d- Una/Pna x 100
e. Ninguna

58. el Sdr hepatorenal es causa de falla:


a- prerrenal
b. renal
c. póstrenal
d. falla cardiaca
e. daño tubular

59. causa mas común de falla renal crónica: A- DM

60- en la glomendonefritis aguda por infeccion, la causa, excepto


a- proteina mayor de 10 g
b- osmolaridad urinaria normal
c. hematuria
d. H T A
e. Edema

61. falla renal cronica, excepto


a. PTH aumentado
b- Hipofosfatemia
c- Acidemia
d. Hipocalcemia
e. 1.25 OH (D3) disminuida

62. EN la falla renal aguda siempre habra


a- Aumento de creatinina
b- fraccion de excrecion de Na aumentado

95
c- hiperkalemia
d- Una aumentado
e- Hipernatremia

63. causa de falla prerenal


a. Sdr nefrotico
b- obstrucción de ambos uréteres
c- falla cardiaca
d. Grlomerulonefritis aguda post-infeciosa
e. Calculo renal

64. en la nefritis tubular interticial se puede dar, excepto


a. h.ipernatremia
b. proteinuria de 1.5 g
c. normotension arterial
d. hipostenuria
e. hipocalcemia

65. Sugiere glomerulonefritis:


a. leucocituia
b-cilindros eritrocitarios
c- anuria

66 -causa de falla renal parenquimatosa


e. papilitis
f. calculo en pelvis derecha
g. estenosis unilateral de arteria renal
h. cistitis hemorragica

98 – Hep C periodo de incubación:


a- 50 dias

60- Causa de hipercalemia:


a Aumento del volumen arterial eficaz
b. Hipercatabolismo tisular
c Hiperaldosteronismo
d. Alcalosis Inetabolica
e. Cirrosis hepatica

61. Causa de hipomagnesemia


a. cirrosis avanzada
b. falla renal cronica
c. falla cardiaca
d. constipacion
e. acidosis respiratoria

62. Causa renal, de hipomagnesemia


a. falla renal crônica
b. esteatorreaa
c desnutricion severa
d. usó de diureticos de asa
e. infeccion urinaria

63. en caso de hipercalcemia pensar en:


a. neoplasia
b: hiperfosfatemia severa
c. nefrolitiasis
d. nefrocalcinosis
e. diuréticos

64. En el síndrome nefrosico puede encontrarse cilindros


a. cereos
b. hematicos
c. leucocitarios
d. epiteliales
e. mixtos leucocitario y hematicos,

65. Cual de estas afirmaciones es falsa:


a- Las transaminasas se elevan cuando hay daño celular hepatico
b- Las transaminasas se elevan en las hepatopatias alcoholicas
c. El cociente GOT/GPT normal es de 1.3
d. Las transaminasas se elevan en las colestasis
e. Unas transaminasas normales excluyen enfermedad hepática

96
66. La positividad de los anticuerpos ant imitocondriales permite establecer el
diagnostico de:
a. hepatitis viral
b cirrosis alcoholica
c. cirrosis biliar primaria
d. cancer primitivo de higado
e. enfermedad de carola

HAV 25 dias
HBV 75 dias
HCV 50 dias
HEV 35 dias
HDV 30 dias

1- De las enf intestinales inflamatorias inespecífica crónica la forma de artropatía ^ asociada es


la:
Espondilitis y fibrólisis

2--En el riñón la hormona paratiroides:


Aumenta la reabsorción tubular de calcio y disminuye la de fósforo

3- La hipercalcemia produce
Potencialización de los efectos de la hipercalemia

36- La hipomagnesemia produce. Vasodilatación periférica

36-De los factores: nefrotoxicidad -La buena hidratación

37 - En la concentración sérica para calcular: 8,8

38- la velocidad de administración de potasio: 40 meq

39-hipocalcemia produce: fosfaturia

40-La causa de hipertensión con hipocalemia: Síndrome de Bartter

41-En paciente con vomito severo hipotensión arterial: Insuficiencia suprarrenal

42-La administración de sol isotónica aumento de vol extracelulares

43-Glomerulonefritis post estreptococcica Altera el cociente de ultrafiltración

44-En-el túbulo` intersticial se puede dar: La proteinuria de 4 g al día.

15- Enfermedad que puede causar falsos positivos por FTA-ABS: cirrosis biliar

16: Es una señal de haber sufrido de pancreatitis: las opciones eran hiperamilasemia, aumento de
dolor, hipercalemia, aumento de tamaño.

17. ALFA- feto proteína no está elevada en: CA de colon

18- La hipofosfatemia cansa excepto: aumento de vitamina D

19. La renal aguda, oligurica todo es cierto excepto: Osmolaridad urinaria aumentada

20. Paciente can alcalosis metabolica e hipokalemia debe ser tratada con KCL

21-Todas son complicaciones de la cirrosis hepatica exceplo


a. peritonitis espontanea
b. perforación del ileo
c. sindrome hepatorenal
d. asc:itis
e. hiperesplenismo

19. Cuando usted evalua a un paciente con diagnostico de cirrosis hepatica y le da seguimiento) se
preocupa de la siguientes situaciones que le puede llevar a una encealopatia hepatica como, una:
a. hiponatremia
b. ingesta baja en proteínas
c. hipokalcemia
d, sangrado digestivo alto
e- aparición de una infección

97
20. Un paciente con que sufre de cirrosis hepática, acude a consulta por Fiebre, al ver su
hemograma tiene leucocitos normales y.observa que tiene trombocitopenia por lo tanto se asume
que:
á. • que ademas tiene dengue..
b- hiperesplenismo
c. trombocitopenia autoinrnune
d. que la pierde por sangrado digestivo asintomático
e. debe ser estudiado por esta trómbocitopenia

24. Usted tiene una paciente con hiperhemesis gravidica. El trastorno metabolico que espera
encontrar es:
A - acidosis hipocloremica
b, acidosis hipercloremica'
c- alcalosis hipocloremica
d. alcalosis hipercloremica
e. no hay alteración acido base

25. Cual de las siguientes opciones esta contraindicada en el tratamiento de la hiperkalcmia


a, gluconato de calcio
b. aldactone
c, kayaxelate
d. Bicarbonato de sodio e. Todas las anteriores

2T.-Son manifistaciones de hipertension portal excepto.


a. varices abdominales
b. ascitis
c: telangectasia
d. esplenomegal.ia
e. ictericia

-28. Cual de los siguientes datos refleja vieja gravedad de una pancreatitis:
a. distensión abdominal
b. hipocalcemia
c. hiperamilasemia
d. aumento de la intensidad .del dolor
e. disnea

29. Cual de los siguientes síntomas es menos probables encontrar en pancreatitis


a. dolor abdominal epigastrico
b. diarrea
c. taquipnea
d. vomitos incoercible
e, hipovolemia

30.Lo mas importante que se puede realizarse para evitar el desarrollo de necrosis en
pancrectis es,.
a..' Hidratación energica del paciente por vía parenteral
b. iniciar antibiotico terapia en forma temprana
c. Admitir al paciente a la unidad de cuidados intensivos
d. Colocar tubo nasogastrico para respiración
e. Tratamiento temprano del dolor

31 _Todo lo siguiente es cierto para el cancer de páncreas, excepto~


a. La mayoría se encuentra en la cabeza
b. La ictericia es su principal síntoma
c. La mayoría d los casos no son operable al momento del diagnostico
d. la diabetes mellitus es su principal factor de riesgo

32- el marcador tumoral más importante del cáncer de páncreas es C- CA-19-9


33- un paciente de 60 años llega al cuarto de urgencia con dolor abdominal y usted
sospecha de pancreatitis , del lab se informa que no se realizara la amilasemia por
escaces de reactivo . usted le solicita
a- un BHC
b- bilirrubina
c- lipasa pancretica
d- transaminasa
e- CPK

43. Rotando en sala de medicina interna le piden que evalue un paciente con HIV el cual
porpresentar una meningitis por criptococci recibe anfotericina B; dicho este medicamento buscaría
como complicación:
a. aparicion de diarrea
b. aumento de la presion arterial

98
c- hipokalemia
d . cefalea
e. dolor toráxico

81- donde se halla el calcio en el organismo:


a- 50% sangre
b- 20 % proteínas
c- 99% hueso

1- cual de los siguientes datos hacen dudar del diagnostico de colon irritable:
a- aparicion en la vejez
b- rectorragia
c- esteatorrea
d- todas las anteriores

2- cual de los siguientes datos refleja vieja gravedad


a- distension abdominal
b- hipocalcemia
c- hiperamilasemia
d- aumento de la intensidad de dolor

3- distensión simultanea de antiHBC y HBs es signo de


a-infeccion prodromica
b- infección activa
c- infeccion pasada
4- en la hipercalemia a- hay una menor diferencia entre el potencial de reposo y el umbral
5—la hipercalemia en un cirrótico puede desencadenar a- coma hepatico
6- la hipermagnesemia puede producira- paro respiratorio
7- tx de hipercalcemiaa- SSN 0.9 %

8- la hipofosfatemia son causas excepto:


a- aumento de PTH
b- aumento de la absorción intestinal de fosfato
c- disminuye fosfatemia
d- aumento de vitamina D

9- una solucion isotonica a- aumenta el volumen extracell


10- la vel de administración de potasio en caso de riesgo para la vida a—40 meq por hora
11- Px cronico con deshidratación severa puede presentar los siguiente excepto:
a-osmolaridad urinaria de 800 mosmol/Kg
12- Px con vomito abundante, HTA, sodio urinario en 6, oligurico, Osmolaridad de 800
presente,
a- insuficiencia suprarrenal
13- Px uremico cronico con vomito severo hipotension arterial orina al azar, muestra sodio
mas de 40, Osmolaridad de 800 puede t ener :
a- Falla suprarrenal

14- En quien predomina la enfermedad de higado graso:


A- Sexo masculino

a- La hipocalemia predispone excepto: amoniogenesis

a- Cual de las siguientes es causa de hiperfosfatemia insuficiencia renal

15- en la glomerulonefritis aguda post infecciosa disminuye la tasa de filtración:


a- por alteración en el coeficiente de ultrafiltracion

16- causa de insuficiencia prerenal excpto:.


a- Aneurisma disecante de la aorta

17- cual es cierta


A- DNA hepatitis B y RNA de hepatitis A

18- Cual no es un signo de encefalopatia hepática Rta:monoparesia

19- Anticuerpo músculo liso es especifico en : Rta: autoinmune

20- Parámetro que se evalua según clasificacion Child -pugs


A-tiempo de prototrombina
B- ascitis
c- bilirrubina
d- albumina
e- encefalopatia hepatica
f- todas las anteriores

99
21- hombre de 25 años que acude con sangrado digestivo bajo de 3 dias de
evolucion con febricula se pensaria los siguiente excepto a - colon irritable
22- hemorragia alta por hipertensión portal para el diagnostico Rta: endoscopia alta
23- hipertensión portal y cirrosis alcoholica la resistencia se da a nivel de
A- respuesta sinusoidal
24- el magnesio esta aumentado Rta: eritrocito

101. Paciente masculino de 33 años llega a su consulta por presentar fiebre, un pequeño
rash, signo de Jordano positivo; el medico de turno le envía un urinalisis que revela
proteinuria, hematuria y múltiples leucocitos; dentro de su diagnostico diferencial, debe
estar una de las siguientes patologías, cual:
b.Nefritis Intersticial Aguda.

102. Dentro de las Glomerulopatias, existen algunas, que se pueden presentar con
Síndrome Nefrosico, y otras con Síndrome Nefrítico, cual de las siguientes se suele
presentar con Síndrome Nefrítico:
b.Nefropatia IgA.

103. En un paciente de 65 años, que desarrolla un Sd. Nefrítico Agudo, la etiología mas
común causando de la Glomerulopatia es:
b.GN rápidamente progresiva.

104. Cual de las siguientes Glomerulopatia, siempre desarrolla Insufiencia Renal:


c.Rápidamente Progresiva.

105. En un paciente con SIADH, al medirle la osmolaridad urinaria, la podemos


encontrar:
a. Elevada.

106. La hiponatermia, se hace clínicamente evidente cuando el sodio, ha disminuido a:


d.125 mEq/lt.

107-111. Mencione 5 medicamentos que puedan causar hiperkalemia:


- Espironolactona
- Trimetropin
- Pentamidina
- IECA
- AINES

112-117. Las características del Síndrome Nefrotico son:


- hipoalbuminemia
- 3.5 g de proteína en orina en 24 h (proteinuria)
- hiperlipidemia
- edema generalizado
- hipertensión
- cilindro céreo
118-120. Los tres puntos en los que se basa la corrección de la hiperkalemia son:
- excreción
- absorción
- resorcion

121-123. Para cada uno de estos puntos, se suele usar: se le solicita al estudiante ser lo
mas explicito posible en su respuesta, en cuanto a dosis y tratamiento exacto.
- expandir volumen con ssn+kcl cuando no hay IC + furosemida
- quelante HPO4 o EDTA
- bifosfonatos pamidronato 1 dosis por semana

124-126. Debemos sospechar, de alteraciones del Magnesio, en que pacientes:


- trastornos de tubo digestivo por malabsorcion
- cirrosis

100
- embarazadas con THE

127. Paciente con hipokalemia + alcalosis metabólica + hipertensión arterial +


disminución de la renina plasmática, esta desarrollando un:
- aldosteronismo primarios
128. Única causa de diarrea con alcalosis metabólica es:
- …
129. Si un paciente, tiene un potasio de 3 mEq/lt, decimos que ha perdido ya:
-…
130. Cual es la principal preocupación que se tiene, cuando un paciente tiene un
aumento en los niveles plasmáticos de Magnesio:
- bloqueo av completo

131-132. Causas de acidosis metabólica con hipokalemia:


- acidosis tubular renal
- …
133. Que examen le ayudaria para diferenciarlas:
-…
134. Que esperaria encontrar en una ATR tipo 2:
-…
135-136. Principales causas de hipercalcemia:
- rabdomiolisis
- exceso de PTH

136-137. Principales causas de hipocalcemia:


- insuficiencia renal crónica
- poca ingesta

138-139. Consecuencias de la hiperfosfatemia:


- hipocalcemia por disminución de paratohomrona
- hiperparatiroidismo
140-143. El Sd. De Barter, se caracteriza clínicamente por:
- Hipopotasemia
- Presión arterial normal o baja
- alcalosis metabolica
- hiperaldosteronismo hipereninémico

144. En un paciente con diagnostico de Glomerulonefritis post estreptococcica, el


diagnostico serologico se basa en el hallazgo de anticuerpos:
a. Antiestreptolisina O.

145. En la Nefiritis secundaria al LES, se suelen encontrar positivos los:


a. ANA.

146. Si un paciente es diagnosticado con Falla Renal Crónica, el siguiente punto es el


estadiage del mismo, si tiene una TFG en 82 ml/minuto, esta en estadio:
b.II

147-148. La Osteodistrofia Renal, se va a caracterizar clínicamente por la presencia de:


- osteitis fibrosa quistica
- osteomallacia u osteopatia adinamica

149.Normalmente la Kps entre el Fósforo y el calcio es de:


- …
150-154. Las características de una muestra de orina con sedimento de tipo nefrítico,
son:
- microhematuria
- cilindro hemático y/o hematíes dismorficos
- oliguria

101
- proteinuria
- disminución del filtrado glomerular

155-158. Mencione algunos parámetros de laboratorio, para pensar en una IRC Renal:
- BUN
- Creatinina
- Hemoglobina
- Presión arterial
159. El Gasto cardiaco normal es de 5 – 6 litros por minuto; de los cuales, un 25% le
corresponde al riñón, en una relación de:
d.1500 ml/min.

160. Si un paciente, diagnosticado con FRC, presenta ademas una acidosis metabolica
hipercloremica, usted sabe que este paciente debe tener una Brecha Anionica:
b.Aumentada.

161. Este mismo paciente, por tener estas características clínicas, debe encontrarse en
estadios de la FRC:
a. Avanzados.

162. Para que por una obstrucción, aumente la creatinina en IRA, se deben haber
perdido:
c.¾ partes de la masa renal.

163. La lesión histologica mas común de IRA es:


b.Necrosis tubular aguda.

164. Y la mas peligrosa y temida es:


b.Necrosis tubular aguda.

165. Cual de las siguientes, es la principal causa de IRA, en la maternidad.


a. Abortos infectados.

14 de agosto de 2015
1. De la vasopresina se puede decir lo siguiente, excepto:
a) Se produce en el hipotálamo anterior.
b) Se produce en la neurohipófisis.

102
c) Esta relacionada con los receptores V2 del túbulo colector.
d) Está relacionada con acuaporina 2
e) Se estimula su producción en caso de deshidratación. Si
2. Causa de hipernatremia
a) Diabetes insípida central
b) Diabetes insípida netrogenica
c) Perdidas gastrointestinales
d) Ay b
a) Todas las anteriores
3. En caso de una pseudohiperkalemia de 7.5 Mm/L, la conducta a seguir
a) Hacer un electrocardiograma urgente
b) Administrar inmediatamente 2 ampollas i.v de gluconato de calcio al 10%
c) Ofrecer el tratamiento completo para la hiperkalemia
d) A y b
e) Ninguna de las anteriores
4. Defecto genético en el túbulo distal se da en el síndrome de:
a. Barter
b. Liddle
c. Gittelman
d. Fanconi
e. Con
5. Manifestaciones clínicas de la hipermagnesemia, excepto:
a. Letargia
b. dificultad para deglutir
c. hiperreflexia
d. paro respiratorio
e. hipotensión arterial
6. Para un calcio serico total de 6mg/dl, con albumina 2g/dl y tomando en
cuenta como albumina sérica normal 4g/dl, ¿Cuánto será el calcio
corregido?
a) 6.6 mg/dl
b) 7.6
c) 8.6
d) 9.6
7. Puede presentarse en hipofosfatemia muy severa, excepto:
a. Hemolisis
b. Rabdomiolisis
c. Trombocitosis
d. Depleción de ATP
e. Miopatia proximal
8. Causa de hipomagnesemia
a. Estreñimiento
b. Hiperkalemia
c. Falla renal crónica
d. Alcoholismo
e. Deshidratación hipertónica
9. Son quelantes de calcio serico
a. Fosfatos intravenosos
b. Fosfatos orales
c. EDTA
d. A y b
e. Todas las anteriores
10.Aumenta la excresion urinaria de fosfatos, excepto:
a) Diuresis osmótica.
b) Expansión de volumen
c) Acetazolamida
d) Espironolactona

103
e) Acidosis tubular renal proximal
11.La sudoración excesiva produce, excepto:
a) Aumento en la excreción urinaria de sodio
b) Perdida de liquido diluido
c) Perdida del liquido concentrado
d) Disminución en la excreción urinaria de sodio
e) B y d
12.Calcule el déficit de agua con la siguiente información: hombre de 60 kg
con 50% de agua corporal total y sodio serico de 168Mm/l
a) 2L
b) 3
c) 4
d) 5
e) 6
13.Sedimento urinario en un síndrome nefrítico
a. Cilindros anchos
b. Eritrocitos dismorficos
c. Cilindros de eritrocitos
d. B y c
e. Todas las anteriores
14.El síndrome nefrosico de cambios minimos se caracteriza por:
a. Proliferación intracapilar
b. Engrosamiento de la membrana basal
c. Hiperplasia mesangial
d. Proliferación endotelial
e. Fusión de los pedicelos de los podocitos
15.En una nefritis tubulointersticial alérgica, se puede encontrar en la orina:
a. Abundantes eritrocitos
16.En la hipokalemia puede presentarse excepto:
e. disminución de amniogenesis
17. se utiliza para eliminar potasio del cuerpo:
d. sulfonato de poliestireno sódico
18. calcule el déficit de sodio con la siguiente información: mujer de 60 kg con 50% de
agua corporal total y sodio serico de 108 Mm/L:
c. 360
19. en la falla renal crónica con oliguria se puede presentar excepto:
b. normokalemia
20. indicación para dializar:
d. pericarditis
21. pensar en una obstrucción de las vías urinarias en caso de:
c. anuria
22. paciente con hipercalcemia de 18 mg e insuficiencia cardiaca, el tartamiento
apropiado es:
c. hidratación con solución salina al 0.9%
23. la disfunción renal debida a hipercalcemia consiste en, excepto:
b. acidosis tubular renal tipo IV
24. Un índice de falla renal aguda en la fracción de excresion de sodio, calcule la
misma con la siguiente información: suero: creatinina 3.2 mg/dl ,sodio 140mM/L.
Orina: creatinina 160mM/L, sodio 35Mm/L.
b. 0.5
25. la síntesis de calcitriol esta estimulada por:
e) todas las anteriores (hipocalcemia, el
Calcule el déficit de agua en una paciente con sodio sérico 168mEq/L y que pesa 70Kg
(utilice como porcentaje de agua 50% y sodio normal mEq/L):
e. 5L
f. 3L
g. 10L
h. 7L

En la SIADH la concentración de sodio urinario es de:

104
e. menos de 10mEq/L
f. 20mEq/L
g. 15mEq/L
h. Mas de 40mEq/L

En paciente con vómitos abundantes y prolongados con hipokalemia severa debe


administrarse:
e. NaCl 0.9% con KHCO3
f. NaCl 0.9% con KCl
g. NaCl 0.9% con gluconato de potasio
h. NaCl 0.9% con NaHCO3

La hipokalemia en un paciente con cirrosis puede contribuir:


e. coma
f. alcalosis respiratoria
g. acidosis metabolica
h. aumento de la síntesis de urea

El plasma de un paciente contiene Na 125mEq/L, glucosa 108mg/dl, urea 300mg/dl,


podrá presentar:
e. cirrosis
f. síndrome nefrósico
g. síntomas de hipertonicidad
h. síntomas de hipotonicidad

La hiponatremia aguda sintomática es debida a, excepto:


e. exceso de glucocorticoides
f. SIADH
g. post-operatorio
h. intoxicación hídrica

La administración de solución salina 0.9% produce:


e. disminución del volumen intracelular y aumento del extracelular
f. aumento del volumen intra y extracelular
g. aumento del volumen extracelular
h. aumento del volumen intracelular

Alcalosis metabólica con hipertensión arterial, hipokalemia, aldosterona normal y


renina normal:
e. Hipertensión renovascular
f. Hipertensión maligna
g. Sindroma de Cushing
h. Administración de licor

La hipokalemia puede predisponer, excepto:


e. intoxicación digitálica
f. disminución de la amoniogénesis
g. rabdomiolisis
h. ondas U en el EKG

¿Cuál de los siguientes no es un efecto secundario de los antiácidos?


f. Estreñimiento
g. Diarrea
h. Hipercalcemia
i. Hiperfofatemia
j. Retención de sodio

La PTH promueve todo lo siguiente excepto:


a. aumento de la absorción intestinal de calcio
b. aumento de la absorción tubular de fosfato

105
c. aumento de la resorción ósea de fosfato
d. aumento de la resorción ósea de calcio
e. disminución de la reabsorción de fosfato

Un varón de 57 esta con hemodiálisis de mantenimiento para insuficiencia renal


crónica, ¿Cuál de las sig anomalías metabólicas podría ser anticipáda?
a. hipernatremia
b. hiponatremia
c. exceso de vitamina D
d. hipoparatiroidismo

Un px de 25años fue ingresado a UCI por lesiones graves en cabeza y fractura de la base
del Graneo, aprox X horas después de la lesion manifestaba poliuria, la OSMU 150, y la de
suero 350. Los líquidos IV fueron detenidos y 3 hors después la producción de orina y
la osm urinaria permanecieron sin cambio. Se administraron 5 unidades de
vasopresina IV, la osm de la orina se
incremento a 300. ¿Cuál es el dx mas probable?
f. Diabetes insípida central
g. diabetes insípida nefrogenica
h. intoxicación acuosa
i. sobrecarga de solutos
j. S IADH

Deshidratación por perdida de agua y densidad urinaria 1.003, creatinina 6 /


f. falla renal aguda parenquimatosa
g. Falla prerenal
h. falla renal obstructiva
i. falla renal crónica
j. no hay falla renal

Uno de los indices de falla renales


f. reabsorción de Na/oferta tubular de Na x 100
g. excresion urinaria deNa/oferta tubular de x 100
h. oferta tubular de Na/excrecion urinaria de Na x 100 ~.--~
i. Una/Pna x 100
j. Ninguna

El Sdr hepatorenal es causa de falla:


f. prerrenal
g. renal
h. póstrenal
i. falla cardiaca
j. daño tubular

Causa más común de falla renal crónica:


b. DM

En la glomendonefritis aguda por infección, causa, excepto


f. proteina mayor de 10 g
g. osmolaridad urinaria normal
h. hematuria
i. HT A
j. Edema

Falla renal crónica, excepto


f. PTH aumentado
g. Hipofosfatemia
h. Acidemia
i. Hipocalcemia
j. 1.25 OH (D3) disminuida

106
EN la falla renal aguda siempre habrá
f. Aumento de creatinina
g. fraccion de excrecion de Na aumentado
h. hiperkalemia
i. Una aumentado
j. Hipernatremia

Causa de falla prerenal


f. Sdr nefrotico
g. obstrucción de ambos uréteres
h. falla cardiaca
i. Grlomerulonefritis aguda post-infeciosa
j. Calculo renal

En la nefritis tubular interticial se puede dar, excepto


f. hipernatremia
g. proteinuria de 1.5 g
h. normotension arterial
i. hipostenuria
j. hipocalcemia

Sugiere glomerulonefritis:
d. leucocituia
e. cilindros eritrocitários
f. anuria

Causa de falla renal parenquimatosa


i. papilitis
j. calculo en pelvis derecha
k. estenosis unilateral de arteria renal
l. cistitis hemorragica

Causa de hipercalemia:
f. Aumento del volumen arterial eficaz
g. Hipercatabolismo tisular
h. Hiperaldosteronismo
i. Alcalosis Inetabolica
j. Cirrosis hepatica

Causa de hipomagnesemia
f. cirrosis avanzada
g. falla renal cronica
h. falla cardiaca
i. constipacion
j. acidosis respiratoria

Causa renal, de hipomagnesemia


f) falla renal crónica
g) esteatorreaa
h) desnutrición severa
i) usó de diuréticos de asa
j) infección urinaria

En caso de hipercalcemia pensar en:


f) neoplasia
g) hiperfosfatemia severa
h) nefrolitiasis
i) nefrocalcinosis
j) Diuréticos

107
En el síndrome nefrósico puede encontrarse cilindros
f) Cereos (Cereos y grasos)
g) Hemáticos (en S. nefrítico)
h) leucocitarios
i) epiteliales
j) mixtos leucocitario y hematicos,

Cual de estas afirmaciones es falsa:


f) Las transaminasas se elevan cuando hay daño celular hepático
g) Las transaminasas se elevan en las hepatopatias alcoholicas
h) El cociente GOT/GPT normal es de 1.3
i) Las transaminasas se elevan en las colestasis
j) Unas transaminasas normales excluyen enfermedad hepática

En el riñón la hormona paratiroides:


b. Aumenta la reabsorción tubular de calcio y disminuye la de fósforo

La velocidad de administración de potasio: 40 meq

Hipocalcemia produce: fosfaturia

En paciente con vómito severo hipotensión arterial o más de 40 meq


b. Insuficiencia suprarrenal

Glomerulonefritis post estreptococcica A


b. ltera el cociente de ultrafiltración

La renal aguda, oligurica todo es cierto excepto: Osmolaridad urinaria aumentada

Paciente can alcalosis metabólica e hipokalemia debe ser tratada con KCL

Usted tiene una paciente con hiperhemesis gravídica. El trastorno metabólico que
espera encontrar es:
f. acidosis hipocloremica
g. acidosis hipercloremica
h. alcalosis hipocloremica
i. alcalosis hipercloremica
j. no hay alteración acido base

Cuál de las siguientes opciones esta contraindicada en el tratamiento de la


hiperkalemia
f. gluconato de calcio
g. aldactone
h. kayaxelate
i. Bicarbonato de sodio
j. Todas las anteriores

Rotando en sala de medicina interna le piden que evalué un paciente con HIV el cual
porpresentar una meningitis por criptococci recibe anfotericina B; dicho este
medicamento buscaría como complicación:
f. aparicion de diarrea
g. aumento de la presion arterial
h. c- hipokalemia
i. d . cefalea
j. dolor toráxico

Donde se halla el calcio en el organismo:


d. 50% sangre
e. 20 % proteínas
f. 99% hueso

108
Cual de los siguientes datos hacen dudar del diagnostico de colon irritable:
e. aparicion en la vejez
f. rectorragia
g. esteatorrea
h. todas las anteriores

Distensión simultanea de antiHBC y HBs es signo de


e. infeccion prodromica
f. infección activa
g. infeccion pasada
h. ninguna

En la hipercalemia
b. hay una menor diferencia entre el potencial de reposo y el umbral

La hipermagnesemia puede producir


b. paro respiratorio

Tx de hipercalcemia
b. SSN 0.9 %

La hipofosfatemia son causas excepto:


e. aumento de PTH
f. aumento de la absorción intestinal de fosfato
g. disminuye fosfatemia
h. aumento de vitamina D

Una solución isotónica


b. aumenta el volumen extracelular

La vel de administración de potasio en caso de riesgo para la vida


b. 40 meq por hora

Px crónico con deshidratación severa puede presentar los siguiente excepto:


b. osmolaridad urinaria de 800 mosmol/Kg

Px con vómito abundante, HTA, sodio urinario en 6, oligurico, Osmolaridad de


800 presente,
b. insuficiencia suprarrenal

Px urémico crónico con vómito severo hipotensión arterial orina al azar,


muestra sodio mas de 40, Osmolaridad de 800 puede tener :
b. Falla suprarrenal

En quien predomina la enfermedad de higado graso:


b. Sexo masculino

Cual de las siguientes es causa de hiperfosfatemia


b. insuficiencia renal

En la glomerulonefritis aguda post infecciosa disminuye la tasa de filtración:


b. por alteración en el coeficiente de ultrafiltracion

El magnesio esta aumentado


b. eritrocito

Cuál de las siguientes medicinas pueden dar hiperkalemia

109
f. inhibidor de la enzima convertidota
g. bloqueador alfa adrenérgico
h. bloqueador beta adrenérgico
i. ayb
j. ayc

Hormonas que disminuyen en la falla renal crónica, excepto


e. 1,25 dihidroxi vitatamina D
f. eritropoyetina
g. paratohormona
h. foliculo estimulante

MgSO4 (PM 120) 1 gramo contiene de magnesio en meq


e. 10
f. 7.9
g. 8.3
h. 6

Factores que conducen a la progresión de falla renal crónica


f. hiperlipidemia
g. toxinas urémicas
h. proteinuria
i. b y c
j. todas

Paciente de 20 años presenta abundantes vômitos y diarrea de 3 días de evolución,


examen físico depleción de volumen, suero muestra Na 155, K 3meq/l, Cl 117meq/l,
HCO3 25 meq/l, tratamiento de elección:
e. D/A 5% + KCl
f. solución salina 3% + KCl
g. solución salina 23.4% + KCl
h. solución salina 0.45% + KCl

En la enfermedad ateroembólica renal se da lo siguiente excepto:


a. Velocidad de eritrosedimentación

Pseudohiperkalemia de 8 mEq/l que se ve en el EKG:


e. Onda T picuda
f. Aplanamiento de la onda P
g. Ay B
h. Ninguna

Criterios para iniciar diálisis excepto:


b. Alcalosiss Metabolica

Hipokalemia con normotension excepto


b. Síndrome de liddle

Plasama creatinina 8mg/dl N de urea 100mg/dl U na 40 Meq/l :


b. Insuficiência renal

Factores reversibles responsables de deterioro de la funcion renal excepto:


b. Fibrosis intersticial

Causa hipokalemia con renina y aldosterona disminuida:


b. Liddle

Sugiere trastorno túbulo intersticial:


e. Proteinuria mayor a 3.5 g /24 horas
f. Normotensión
g. Edema

110
h. Osmolaridad urinaria normal

Glomérulonefritis rápidamente progresiva


e. Engrosamiento de la membrana basal
f. Fusión de los pedículos de los podocitos
g. proliferacion intracapilar
h. proliferación extracapilar

Plasma Na:115, Posm240 mOsm,Uosm 680,Una 60:meq/l:


b. Deshidratación hipotónica
d. Secreción inapropiada de ADH
e. .Deshidratación isotónica
f. .Deshidratación primaria

En la falla renal crónca produce necrosis


b. calcifilaxia

No produce hipokalemia:
e. Pancreatitis aguda
f. rabdomiolisis
g. neoplasia
h. Def. Vit D

Paciente con glucosa 90mg/dl, Na+= 115mEq/L, Urea en 140mg/dl


e. Síndrome hipoosmolar
f. Síndrome hiperosmolar
g. Diarrea
h. Ninguna de las anteriores.

Causa de seudohiponatremia con osmolaridad elevada


e. Hiperproteinemia
f. Diabetes mellitus
g. Hiperlipidemidemia
h. Reserción transuretral prostática.

Manifestaciones de hipermagnesemia excepto


b. Normotensión

Piel en uremia crónica excepto


b. Paño blanco

No es causa de necrosis papilar aguda


e. a.Diabetes mellitas
f. Uropatía obstructiva
g. Pielonefritis
h. Trombosis de venas renal.

Puede presentarse hiperuricemia excepto


b. Adenocarcinoma gástrico

Compromete riñones y pulmones:


f. Enf de GoodPasture
g. Granulomatosis de Wegener
h. LES
i. Ay C
j. TODAS

Sugiere glomerulopatia:
b. anasarca

TFG < 15ml/min corresponde al estadio de la enfermedad renal cronica:

111
b. 5

Causa obstetrica de necrosis cortical bilateral


b. Desprendimiento prematuro de placenta

Promueve amoniogenesis en cirróticos


b. hipokalemia

Criterio para diagnostico de síndrome nefrosico


f. Proteinuria en 24 hr de 3.5 gr para superficie corporal de 1. 73
g. Albumina serica < 3g/dl
h. Hiperlipidemia y edema
i. Todas las anteriores
j. ay b

El aumento de -fetoproteina serica puede detectar , excepto


b. Carcinoma pancreático

Cuando usted evalua a un paciente con diagnostico de cirrosis hepática y le da


seguimiento se preocupa de las siguientes situaciones que le puede llevar a una
encefalopatía hepática; menos una:
f. hiponatremia
g. ingesta baja en proteinas
h. hipokalemia
i. sangrado digestivo alto
j. aparicion de una infeccion

Cual de los siguientes datos refleja vieja gravedad de una pancreatitis - esta es la que
es!
f. distensión abdominal
g. hipocalcemia
h. hiperamilasemia
i. aumento de la intensidad del dolor
j. disnea

El ciclo de la urea tiene como finalidad fundamental:


k. sintetizar protrombina
l. transformar el amoniaco
m. producir sales biliares
n. sintetizar colesterol
o. producir acido folico

La transformación de la Vit D2 en 25(OH)D3 tiene lugar en


f. hígado
g. piel
h. riñón
i. intestino
j. hueso

La elevación de la alfafetoproteina serica puede detectarse excepto:


b. carcinoma pancreático

La elevación de la alfafetoproteteina renca puede detestarse en todas excepto:


f. carcinoma primitivo de hígado
g. carcinoma germinal de ovario
h. Carcinoma pancreático
i. D. carcinoma testicular
j. hepatitis aguda

112
Las perdidas renales excesivas de potasio ....
e. Colitis ulcerativa
f. Enf de Cronh
g. colon irritable
h. tumor velloso

Gradiente sérico de albumina menor de 1.1, se observa en:


b. hipoalbuminemia severa.

Las fosfatasas alcalinas no aparecen nunca elevadas en una de las siguientes


circunstancias:
f. enf de Hodgkin
g. raquitismo
h. cirrosis biliar primaria
i. sarcoidosis
j. gota

Gradiente sérico de albumina menor de 2.1, se observa en:


b. hipoalbuminemia severa.

Causa de hipomagnesemia:
i. estreñimiento
j. alcoholismo
k. falla pre-renal
l. IRA (insuficiencia renal aguda)

La hipocalcemia en la falla renal aguda puede ser asintomática por la:


i. acidemia
j. hipoalbuminemia
k. Deficiencia de PTH
l. Alcalemia

La hipofosfatemia causa, excepto:


i. Aumento de vit. D
j. Aumento de PTH
k. Disminución de la excresión de fosfato
l. Aumento de la absorción de fosfato a nivel intestinal

Un pte con diuresis de 250 ml al día con osmolaridad plasmática (presión osmótia) y
osmolaridad urinaria de 300 mosm/l, el dx probable e:
i. Insuficiencia suprarrenal
j. Insuficiencia renal
k. Nefritis túbulo intersticial crónica
l. Deshidratación hipertónica

Pte con oliguria, hiperkalemia, hiponatremia, creatinina 0.8 mg/dl y CPK normal: Esto
puede corresponder a:
i. IRA
j. Insuficiencia suprarrenal
k. Síndrome hepatorrenal
l. Rabdomiolisis

No está indicado en caso de hiperkalemia:


b. Aldactone

Mujer con hiperemesis grávida:


b. Alcalosis hipoclórica

En causa de hipomagnesemia:
b. Alcoholismo.

113
En el metabolismo del calcio está implicado, menos:
b. Insulina

Hiponatremia, hipotónico, normovolémico:


b. SIADH

Fórmula del clearence de creatinina:


b. clearence = 140 – edad x peso en Kg
72 x creatinina

Fórmula de irritabilidad muscular:


b. IR = Na + K + OH
( ) + Mg + H

Con respecto al síndrome diarreico, nombre 2 tipos osmóticos:


c. Laxantes osmóticos (Mg2+, PO4, SO4)
d. Carbohidratos no absorbibles (sorbitol, lactulosa).

Nombre 2 síndromes diarreicos motores con aumento de motilidad:


e. Hipertiroidismo
f. Prostaglandinas.

Indica falla renal aguda:


k. U.N. = 4.1
l. U.N. = 1
m. U.N. = ¼
n. Gravedad específica: 1.026
o. Ninguna.

Deshidratación por pérdida de agua y densidad urinaria 1.003, creatinina 6:


k. falla renal aguda parenquimatosa
l. Falla pre-renal
m. Falla renal obstructiva
n. Falla renal crónica
o. No hay falla renal.

Uno de los índices de falla renal es:


k. Reabsorción de Na/ oferta tubular de Na x 100
l. Excreción urinaria de Na/ oferta tubular de Na x 100
m. Oferta tubular de Na/ excresión urinaria de Na x 100 **
n. Una / pna x 100
o. Ninguna

El síndrome heatorrenal es causa de falla:


k. pre-renal
l. renal
m. posrenal
n. falla cardiaca
o. daño tubular

Causa más comun de falla renal cronica:


k. Diabetes melitus
l. HTA
m. Glomerulonefritis
n. Enfermedad poliquística renal
o. Vasculitis

En la glomerulnefritis aguda por infección, la causa, excepto:


a. proteínas mayor de 10g

114
b. osmolaridad urinaria normal
c. hematuria
d. HTA
e. Edema

• A pesar de la diversidad de enfermedades indicadas abajo, ellas tienen muchos


síntomas en común. Con frecuencia, los síntomas y signos observados
incluyen: disminución en el volumen de orina, proteína en la orina
(proteinuria), sangre en la orina microscópica o macrscópica (hematuria),
hinchazón (edema), presión sanguínea alta (HTA) y una disminución en la
capacidad del riñón de extraer eficazmente los residuos.

Falla renal crónica, excepto:


k. PTH aumentada
l. Hipofosfatemia
m. Acidemia
n. Hipocalcemia
o. 1.25 OH (D3) disminuída.

En la falla renal aguda siempre habrá:


k. aumento de creatinina
l. fracción de excresión de Na aumentado
m. Hiperkalemia
n. Una aumentado
o. Hipernatremia

Causa de falla pre-renal:


k. Sd. Nefrótico
l. Obstrucción de ambos uréteres
m. Falla cardiaca
n. Glomerulonefritis aguda pos- infecciosa
o. Calculo renal

En la nefritis tubular intersticial se puede dar, excepto:


k. hipernatremia
l. proteinuria de 1.5 g
m. normotension arterial
n. hipostenuria
o. hipocalcemia

Sugiere glomerulonefritis
g. leucocituria
h. cilindros eritrocitarios
i. anuria

Causa de anuria:
k. Enfermedad de cambios mínimos
l. Necrosis cortical
m. Pielonefritis aguda leve bilateral
n. Necrosis tubular aguda
o. Calculo coloriforme bilateral

En paciente con falla renal crónica que ha sido trasplantado esperamos encontrar:
b. osteomalacia.

31) En hiponatremia el daño principal es:

a) Neurológica
b) Respiratoria
c) Renal

115
32) Adaptación del cerebro ante solución hipotónica:

a) Pérdida de iones
b) Pérdida de aminoácidos
c) Edema cerebral
d) Ganancia de iones

41. Rotando en sala de medicina interna le piden que evalue un paciente con HIV el
cual por presentar una meningitis por criptococci recibe anfotericina B; debido a
este medicamento buscaria como complicación:
a. aparicion de diarrea
b. aumento de la presion arterial
c. hipokalemia
d. cefalea
e. dolor toráxico

42. Paciente de 84 anos quien recibe diuretico tiazidico para la hipertensión arterial es
admitido por presentar diarrea y trastornos del sensorio. Tiene disminución del
turgor de la piel y la presion arterial es normal, Pna 174mEq/L, Una 5mEq/L,
Uosm 606mEq/kg. La hipertensión se debe a:
a. perdidas insensibles
b. polidipsia
c. uso de diuretico
d. diabetes insípida no es esta
e. secrecion inapropiada de hormona antidiuretica

43. Hipernatremia Hipervolemica


a. diuresis osmótica
b. diarrea
c. aldosteronismo primario
d. diabetes insipida
e. SIADH

44. La hipokalemia en un cirrotico puede precipitar COMA por:


a. Hipermagnesemia asociada
b. Aumento de la amoniogenesis
c. Hiperfosfatemia asociada
d. Rabdomiolisis
e. Induce vomitos

116
Causa de hiperkalemia:
a. Aumento del volumen arterial eficaz
b. Hipercatabolismo tisular
c. Hiperaldosteronismo
d. Alcalosis metabolica
e. Cirrosis hepatica

45. Causa de hipomagnesemia


a. cirrosis avanzada
b. falla renal cronica
c. falla cardiaca
d. constipacion
e. acidosis respiratoria

46. Causa renal de hipomagnesemia


a. falla renal cronica
b. esteatorrea
c. desnutricion severa
d. uso de diureticos de asa
e. infeccion urinaria

47. en caso de hipercalcemia pensar en:


a. neoplasia
b. hiperfosfatemia severa
c. nefrolitiasis
d. nefrocalcinosis
e. diureticos

48. En el síndrome nefrosico puede encontrarse cilindros:


a. Cereos
b. hematicos
c. leucocitarios
d. epiteliales
e. mixtos leucocitario y hematicos
Cilindros Cereos (síndrome nefrítico) grasos (síndrome nefrótico)

49. cual de estas afirmaciones es falsa:


a. Las transaminasas se elevan cuando hay dano celular hepatico
b. Las transaminasas se elevan en las hepatopatias alcoholicas
c. El cociente GOT/GPT normal es de 1.3
d. Las transaminasas se elevan en las colestasis
e. Unas transaminasas normales excluyen enfermedad hepatica

50. La positividad de los anticuerpos antimitocondriales permite establecer el


diagnostico de:
a. hepatitis viral
b. cirrosis alcoholica
c. cirrosis biliar primaria
d. cancer primitivo de higado
e. enfermeda de carola

51. El ciclo de la urea tiene como finalidad fundamental:


p. sintetizar protrombina
q. transformar el amoniaco
r. producir sales biliares
s. sintetizar colesterol
t. producir acido folico

52. Cual de las siguientes afirmaciones es cierta


a. la hepatitis A es de corto periodo de incubacion y se transmite por via parenteral
b. la hepatitis B es de largo periodo de incubacion y se transmite por via fecal-oral
c. L a hepatitis B es de corto perido de incubacion y se transmite por via parenteral

117
d. La hepatitis A es de corto periodo de incubacion y se transmite por via fecal-
oral
e. La hepatitis A es de largo periodo de incubacion y se transmite por via fecal-oral
La Hepatitis A y E se transmite por via fecal-oral y es de corto periodo fecal.
Hepatitis B y D (via parenteral)

53. Cual de estos datos biologicos es sugestivo de gravedad en una hepatitis virica
aguda
a. Bilirrubina superior a 15mg/dL
b. Transaminasas superior a 2000 mU/mL
c. Tasa de protrombina inferior al 40%
d. HBsAg positivo
e. Todos los anteriores

54. El signo clinico que refleja mejor grado de obstrucción biliar en una colestasis es:
a. intensidad de la ictericia
b. presencia de coluria
c. prurito intenso
d. existencia de xantelasma
e. existencia de acolia

55. La hipertensión portal en la hepatitis alcoholica aguda se atribuye a


a. fibrosis periportal
b. necrosis hepatocelular
c. esteatosis hepatica
d. constriccion de las venas centrolobulillares
e. nodulos de regeneracion

56. La transformación de la Vit D2 en 25(OH)D3 tiene lugar en


k. higado
l. piel
m. rinon
n. intestino
o. hueso

57. Donde se halla el calcio del organismo


a. el 50% en la sangre
b. el 20% en las proteinas
c. el 99% en el hueso
d. el 50% en el hueso
e. el 50% en el hueso y el 50% en la sangre

2. En un paciente alcohólico con ascitis por cirrosis hepatica, tiene hiponatremia


serica con hiperadolesteronismo, la fisiopatología de esta enfermedad es por:
• Aumento de producción de renina por hipovolemia

3. Px de 60 años con dx de colangitis se le inicia tx con antibióticos empírico contra


bacterias entericas y la fiebre no cede en 48 horas, la conducta seria:
• proceder inmediatamente al tx quirurgico

4. Las pérdidas renales excesivas de potasio:


• tumor velloso

5. Px de 60 años con ascitis por ingesta de alcohol, investigar si se trata de cirrosis


hepática, para su dx es necesario:
Conteo sanguíneo completo, Tiempo de protrombina, Pruebas de la función hepática, Nivel de
albúmina en la sangre

7. La PTH promueve todo lo siguiente excepto:


Aumento de la absorción tubular de fosfato

118
8. Varón de 57a esta con hemodiálisis de mantenimiento para insuficiencia renal
crónica. Cual de las siguientes anormalidades metabólicas podría ser anticipada?
Osteomalacia

11 Estadio dx de Churg Strauss:


un primer estadio prodrómico en el que predominan las manifestaciones alérgicas (rinitis,
asma, pólipos nasales, alergia a fármacos -antibióticos fundamentalmente-, alergia
alimentaria, al polvo o al polen), un segundo estadio que cursa con hipereosinofilia
sanguínea e infiltrados tisulares de eosinófilos (neumonía eosinófila de Löffler, infiltración
miocárdica y gastroenteritis eosinofílica) y un tercer estadio de vasculitis sistémica con
afectación de diferentes órganos (sobre todo piel, sistema nervioso periférico y riñones).

12 Indica falla renal aguda:


BUN

13 Síndrome hepatorenal es causa de falla:


Prerenal

14 Síndromes Hipercoagulabes, excepto:


Sepsis

16 Hepatitis C, periodo de incubación;


20- 110 días
El período de incubación de la hepatitis C puede variar de dos semanas a seis meses
35 Hombre con fractura de cráneo, osm 800:
Diabetes insípida central

38 Para prevenir necrosis:


Alta hidratación parenteral

51 Falla renal crónica avanzada:


Brecha aniónica elevada

53 Disentería Amebiana:
Acidosis e hipokalemia

56 Hiperemesis Gravidica:
Alcalosis hipoclorica

61 La elevación de alfafetoproteina serica puede detectarse en todas excepto:


o Carcinoma testicular

67 En la encefalopatía hepática existe excepto:


o Inhibición gabaergica

69 paciente de 25 que llega al cuarto de urgencia con sangrado digestivo bajo con 3
dias de evolución hubo febricula, usted piensa en los siguientes diagnosticos excepto:
o Síndrome de colon irritable

78 el paciente con esófago de barret es cierto


o Tiene años con reflujo tiene un riesgo de adenocarcinoma del cardias de 10%
necesita vigilancia endoscopica con biopsia de 24 meses.

Modulo de Renal
67. La PTH promueve todo lo siguiente excepto:
o EL aumento de la reabsorción tubular de fosfato.
La PTH a nivel renal es eminentemente fosfatúrica, promueve la excreción de fosfato y la
reabsorción de Calcio.
68. Síndrome nefrósico de cambios mínimos se caracteriza por:
o Fusión de los pedicelos de los podocitos.

119
La fusión de los pedicelos no se puede ver en microscopia de luz, hay que usar microscopia
electrónica, pero se puede hacer el dx con la proteinuria masiva característica del síndrome.
69. En un paciente con eclampsia se le ordeno un litro de solución de dextrosa al 5%,
conteniendo 20 ml de MgSO4 al 50% (PM= 100 g), ¿cuantos ml de sal se necesita para
preparar la solución?
o 10 ml.
70. El porcentaje de solución es:
o 1%
71. ¿Cuantos mEq de Mg tiene la venoclisis?
o 83.3
72. La hipocalcemia de la falla renal crónica puede cursar:
o Asintomática.
73. La velocidad de administración de KCL en un caso de alto riesgo puede ser:
o 40 mEq/L
En los pacientes con una hipokalemia muy severa se usa 40, pero se debe pasar a una
velocidad de 20, porque si se pasa muy rápido puede ocasionar un bloqueo AV completo.
• Paciente urémico, con K sérico de 6, Academia, EKG normal se trata con:
o Kayexalate
Ya que la hiperkalemia no es tan severa se puede tratar con el Kayexalate, que es una resina
de intercambio catiónico, la misma remueve el potasio del cuerpo.
• Paciente femenina de 50 Kg presenta diarrea, presión arterial un poco baja, sin signos
neurológicos, Na=120, K=2, Cl=100, HCO3=10, asuma 60% de agua, cuantos gramos
de Sal para elevar el Na a 130
o 17.5
Calculo: (60*0.5)*10=300 se necesita dos soluciones salinas (154*2=308), cada solución
salina tiene 58.5 g NaCl y cada gramo de solución tiene 17.5 g de Na.
• ¿Qué solución salina usted escogería?
o 0.9 %
• La hipofosfatemia produce:
o Fosfaturia
La disminución del calcio, produce un aumento de la PTH y esta a su vez un aumento de la
eliminación del fósforo por la orina.
• La hipokalemia puede predisponer, excepto:
o Disminución de la amoniogenesis.
La hipokalemia por lo general cursa con acidosis metabólica, en donde va a estar
aumentada la formación de amonio.
• Un inhibidor de la anhidrasa carbónica es causa de:
o Hipokalemia
Ellos producen un aumento de la excreción de bicarbonato al ser este un anión, arrastra el
potasio que es un catión.
• Un hombre llega al hospital comatoso, con fractura de cráneo, orina 175, suero de
Na=178, K=4, Cl=130, HCO3=25 Osm=350 en la orina 800 (no c ve bien).
o Diabetes insípida central.
Recordar que una de las causas de hipernatremia es la diabetes insípida.
• Confirmaría su dx así:
o Prueba de restricción y administración de vasopresina.
• Causa de HTA con hipokalemia excepto:
o Síndrome de Bartter.
El síndrome de Bartter no cursa con hipertensión arterial (HTA maligna,
hiperaldosteronismo primario y síndrome de hiperprostaglandismo entre otros).
• Se relaciona con hipofosfatemia, excepto:
o Disminución de la afinidad de la hemoglobina por el O2.
La hipofosfatemia produce una disminución del 2,3-DPG, y hay una desviación a la
izquierda, lo que produce una aumento de la afinidad por el oxigeno.
• Paciente con vómitos severos, hipotensión arterial, la orina al azar muestra Na mas de
40, osmolaridad de 800, puede tener:
o Falla suprarrenal.
El paciente al tener una hipovolemia, se activa el SRAA, y aumenta la reabsorción de Na y
disminuye su cantidad en orina.
• Un hombre de 30 Kg padece de Ca pulmonar, tiene 2 semanas de estar letargico y
obnubilado, suero de Na 105, Cl 72, K 4, HCO3 21, Osm 122 y en la orina Na 78, Osm
604, el dx mas probable:

120
o SIADH.
Hay una hiponatremia, hipotónica y euvolémica.
• Tratamiento:
o Solución salina al 3%
El tratamiento de SIADH, es restricción de agua y suplir el déficit de sodio.
• Causas de enfermedad tubular intersticial crónica excepto:
o Nefritis alérgica.
En su mayoría medicamentos y medio de contraste.
• La administración de solución salina isotónica produce:
o Aumento del volumen extracelular.
• Sospecha de obstrucción de las vías urinarias en caso de:
o Anuria.
• En la insuficiencia renal aguda siempre da, excepto:
o Retensión nitrogenada
Es un parámetro a tomar en cuenta para dializar al paciente cuando es mayor de 100.
• Siempre habrá insuficiencia renal en caso de:
o Glomérulo nefritis rápidamente progresiva
La proliferación extracapilar comprime el glomérulo comprimiéndolo y dañándolo
severamente.
• En caso de glomérulo nefritis post estreptocócica la filtración glomerular puede
disminuir por:
o Alteración del coeficiente de ultrafiltración.
• En la nefritis túbulo intersticial puede dar lo siguiente, excepto:
o Proteinuria de 4 gr/día
Ese dato es de síndrome nefrósico.
• Es la causa de falla prerrenal.
o Pancreatitis aguda.
• En la insuficiencia renal aguda siempre habrá:
o Aumento de la creatinina.
Otra causa importante son las pérdidas gastrointestinales.
• La hipofosfatemia causa, excepto:
o Aumento de PTH.
• Paciente con diuresis de 250 ml al día con Osm plasmática y osmolaridad urinaria de
300, el Dx probable:
o FRA, de causa renal.
• Paciente con oliguria, hiperkalemia, hiponatremia, creat 0.8 mg/dl y cpk normal, esto
puede corresponer a:
o Falla renal aguda, de causa prerrenal.
• En falla renal crónica:
o Hipofosfatemia.
• Urémico crónico con K 6.5 con leve academia, sin trastorno muscular y EKG normal,
tratamiento:
o Kayexalate.
• Urémico agudo, acidótico, oligúrico, deshidratado, todo es cierto, excepto:
o Densidad urinaria 1010.
Esto indica isostenuria, el riñón no tiene la capacidad de concentrar y es indicativo de
riñones muy maltratados.
• Falla renal aguda, oliguria, todo es cierto, excepto:
o Osmolaridad urinaria elevada.
• Síndrome nefrósico de cambios mínimos se caracteriza:
o Fusión de los pedicelos de los podocitos.
• En el shock hipovolémico, el uso de dextrosa en agua puede causar:
o Hiponatremia sintomática.
Primero siempre corregir la volemia con SSN 0.9.
• En la deshidratación por perdida fundamental de solutos, se da la siguiente,
excepto:
o Entrada de electrolitos al cerebro.
• En el SIADH se da lo siguiente excepto:
o Edema de los miembros inferiores.
• Paciente con gastroenteritis, hipovolémico, cuyo suero contiene Na 155, K 2, Cl 117:
o SSN 0.45 + 40 mEq KCl.

121
• Una medida general en el tx de un hipercalcémico:
o SSN 0.9%
Para expandir el volumen.
• La velocidad de adminis tracion de K en caso de riesgo para la vida puede ser:
o 40
• Un ejemplo de hipernatremia hipervolemica es:
o Hiperaldosteronismo primario.
• Una dieta de 3gr de NaCl corresponde a:
o 17 gr de Sodio.
• La IRC ocaciona hiperfosfatemia permanente cuando la TFG esta por debajo de:
o 25 ml/min
El valor es por debajo de 30.
• Paciente con alcalosis metabólica e hipokalemia severa debe ser tratado con:
o KCl
• No esta indicada en la hipercalcemia:
o Aldactone
Es un diurético ahorrador de K, ósea produce hipercalcemia, se utiliza en el
hiperaldosteronismo primaria y edema.
• Metabolismo del calcio están implicados menos:
o Insulina.
• Hiponatremia hipotónica normovolémica:
o SIADH.
• Causa de enuresis
o Necrosis cortical difusa.
• La situación clínica de hiperfosfatemia con hipocalcemia con cirrosis hepática es:
o Cirrosis alcohólica.
• Factor que altera el balance hídrico en el periodo post operatorio excepto:
o Disminución de ADH.
• Ejemplo de hipernatremia hipervolemica:
o Hiperaldosteronismo primario.
• Ejemplo de hipokalemia con aldosterona alta y osm baja:
o Hiperaldosteronismo primario.
También con renina baja.
• Hiperkalemia sin alteración en el EKG no es necesario administrar:
o Gluconato de calcio.
• Urémico crónico con deshidratación hipertónica severa y acidótico presentara lo
siguiente excepto:
o Osmolaridad urinaria de 800.
• La fracción de excreción de Na esta aumentada en:
o Falla cardiaca.
También esta aumentada en FRC, de causas renales.
• Paciente con vomito abundante, HTA, Una 6, oligúrico y osmolaridad de 800,
presentara:
o Insuficiencia suprarrenal.
• En cirrótico con ascitis se recomienda, excepto:
o Dieta con 8.5 mEq de Na.
60 to 90 mEq per day, equivalent to approximately 1500 to 2000 mg of salt per day
(NEJM).
• Factor que predispone a la hipernatremia son, excepto:
o Intoxicación hídrica.
• En la hiperkalemia:
o Hay menos diferencia entre el potencial de reposo y el umbral.
• Los indicios de falla renal aguda son:
Pueden ser cualquiera de las siguientes:
o Alteraciones en la osmolaridad urinaria.
o Alteraciones sen la relación Uosm/Posm.
o Alteraciones en el Una.
o Alteraciones en el FENA.
• Se presenta en acidosis tubular renal proximal, excepto:
o NaHCO3 y los alpha agonista.
• Na urinario menor de 10:

122
o Falla cardiaca.

La esteatorrea en la pancreatitis crónica aparece cuando el páncreas exocrino funcionante


tiene destruido:
a. 40%
b. 50%
c. 60%
d. 70%
e. 90%

El flapping tremor:
a. es un signo de hepatopatía crónica
b. indica la existencia de un delirium tremens
c. es un signo de encefalopatía hepática
d. su ausencia descarta la existencia de una encefalopatía hepática
e. consiste en un temblor intencional de las manos

La insuficiencia renal funcional de la cirrosis hepática es debida a:


a. lesión glomerular
b. lesión tubular
c. hipovolemia
d. compresión de la ascitis sobre los grandes vasos renales
e. vasoconstricción de las arterias renales

La presencia de transformación maligna de un pólipo no depende de:


a. tamaño
b. tipo histológico
c. presencia de atipias epiteliales
d. presencia de un pólipo inferior a 1cm
e. ninguna de las anteriores

El antígeno carcinoembrionario es útil en el estudio de los pacientes con cánceres


colorrectales:
a. porque es específico para estos tumores
b. detecta precozmente la aparición del cáncer
c. es eficaz para la detección de recidivas después de la extirpación del tumor
colorrectal
d. aumenta después de estudios radiológicos
e. solo aparece en el cáncer de colon

De las siguientes afirmaciones con respecto a la colitis ulcerosa, ¿Cuál es falsa?


a. no se debe a deficiencias nutricionales
b. no se debe a un agente infeccioso
c. no debe a alteraciones vasculares
d. no esta en relación con tóxicos ambientales
e. no esta en relación con alteraciones inmunlógicas

Cuál es el factor responsable de la duración de la secreción pancreática?


a. pH duodenal
b. concentración proteica del contenido duodenal
c. duración del vaciamiento gástrico
d. concentración grasa del contenido duodenal
e. ninguna de las anteriores

Son factores de riesgo del hígado graso no alcohólico excepto:


a. obesidad
b. sexo masculino
c. alcohol
d. hiperlipidemia
e. DM tipo 2

123
Son medicamentos que se pueden utilizar en la colitis ulcerativa excepto:
a. aminosalicilatos
b. acido acetil salicilico
c. corticoides
d. inmunomoduladores
e. agentes biológicos (infliximab)

En el carcinoma de esófago las metástasis más frecuentes en órganos ajeno al mismo son:
a. peritoneales
b. pulmón e hígado
c. estómago
d. páncreas
e. cerebrales

Cuál de los siguientes no es un efecto secundario de los antiácidos?


a. estreñimiento
b. diarrea
c. hipercalcemia
d. hiperfosfatemia
e. retención de sodio

El vaciamiento gástrico se acelera por la acción de:


a. somatostatina
b. glucagon
c. péptido intestinal vasoactivo (VIP)
d. secretina
e. prostaglandinas

Cuál de las siguientes hormonas no inhibe la secreción gástrica?


a. VIP
b. Glucagon
c. Calcitonina
d. Gastrina
e. Somatostatina duodenal

La gastritis aguda erosiva o hemorrágica no esta relacionada con la toma de:


a. antibióticos de amplio espectro
b. alcohol
c. acetilsalicilatos
d. fenilbutazona
e. reserpina

Una de las siguientes alteraciones neurológicas no es habitual en la encefalopatía hepática:


a. flapping tremor
b. coma
c. apraxia construccional
d. bradipsiquia y bradilalia
e. monoparesia

En qué tipo de hepatopatía es más frecuente la hipertrofia parotidea?


a. cirrosis biliar primaria
b. cirrosis posthepática
c. cirrosis alcohólica
d. cirrosis biliar secundaria
e. hepatitis crónica activa

Cuál de las siguientes afirmaciones es cierta?


a. el VHB posee RNA y el VHA posee DNA
b. el VHB posee DNA y el VHA posee RNA
c. el VHB posee DNA y el VHA posee DNA
d. EL VHB posee RNA y el VHA posee RNA
e. Los dos virus carecen de ácido nucleico

124
La breve duración del periodo de excreción fecal del virus A de la hepatitis se atribuye:
a. escasa eliminación biliar
b. inactividad del virus por jugo pancreático
c. activación de los linfocitos T supresores
d. fagocitación por marófagos intestinales
e. formación local intestinal de IgA

Junto al resto de medidas terapéuticas habituales de la insuficiencia hepática aguda grave,


en la producción por intoxicación por paracetamol, hay que administrarle:
a. hidrocortisona
b. N-acetil-cisteína
c. Acido paraaminobenzoico
d. Medroxiprogesterona
e. Fenobarbital

Las fosfatasas alcalinas no aparecen nunca elevadas en una de las siguientes circunstancias:
k. enf de Hodgkin
l. raquitismo
m. cirrosis biliar primaria
n. sarcoidosis
o. gota

La positividad de los anticuerpos antimitocondriales permite establecer el diagnóstico de:


a. Hepatitis viral
b. Cirrosis alcohólica
c. Cirrosis biliar primaria
d. Cáncer primitivo de hígado
e. Enf. De Carola

Las sales biliares proceden de:


a. bilirrubina
b. ácidos grasos
c. fosfolípidos
d. colesterol
e. lecitina

Enuncia 4 parámetros que utiliza la evaluación de Chile-Pug en la cirrosis hepática:


a. encefalopatía hepática→
b. ascitis→
c. albúmina sérica→
d. bilirrubina→
e. actividad de protrombina→

En el síndrome de malabsorción encontramos excepto:


a. alteración del tiempo de protrombina
b. hipocalcemia
c. grasas en heces disminuidas
d. hipoalbuminemia
e. anemia

El siguiente no es un factor de riesgo para cáncer de colon:


a. AINES
b. Ácidos biliares
c. Tabaquismo
d. Dietas altas en grasas
e. Dieta baja en fibras

Lo más importante que se puede realizar para evitar el desarrollo de necrosis en la


pancreatitis aguda es:
a. hidratación enérgica del paciente por vía parenteral

125
b. iniciar antibiótico terapia en forma temprana
c. admitir al paciente en unidad de cuidados intensivos
d. colocar tubo nasogástrico para aspiración
e. ofrecer oxígeno al paciente

Un paciente de 25 años de edad llega al cuarto de urgencia con sangrado digestivo bajo, de
tres días de evolución, hubo febrícula, usted piensa en los siguientes diagnósticos excepto:
a. Enf de Crohn
b. Colitis ulcerativa
c. Colon irritable
d. Amebiasis intestinal
e. TBC intestinal

Cuál de estos datos biológicos es sugestivo de gravedad en una hepatitis vírica aguda?
a. bilirrubina superior a 15mg/dl
b. transaminasas superior a 2000mU/ml
c. tasa de protrombina inferior a 40%
d. HbsA positivo
e. Todas las anteriores

Urémico crónico con potasio sérico de 6.5mEq/L con leve academia, sin trastorno muscular
y EKG normal, puede ser tratado con:
a. kayaxelate
b. gluconato de Calcio al 10%
c. NaHCO3
d. Diálisis

La insuficiencia renal crónica ocasiona hiperfosfatemia permanentemente cuando la tasa de


filtración glomerular esta por debajo de:
a. 60ml/min
b. 50 ml/min
c. 25 ml/min
d. 40 ml/min

La hipofosfatemia es causa de excepto


a. inotropismo positivo
b. disminución de 2,3 DPG
c. hemólisis
d. curva de HbO2 desplazada hacia la izquierda

La solución de NaCl 23,4% tiene de osmolaridad:


a. 8000 mosm/l
b. 800 mosm/l
c. 80 mosm/l
d. 0,8 mosm/l

En la hiperkalemia:
a. aumenta la sístole eléctrica
b. hay menor diferencia entre el potencial de reposo y el umbral
c. el potencial de reposo transmembrana se hace mas negativo
d. el potencial umbral se hace menos negativo

La velocidad de administración de potasio en caso de gran riesgo para la vida puede ser de:
a. 10-20 mEq/L
b. 8 mEq/L
c. 110 mEq/L
d. 40 mEq/L

Un ejemplo de hiponatremia hipotónica normovolemica es:


a. síndrome nefrósico
b. SIADH
c. Diarrea

126
d. Cirrosis

A una embarazada con eclampsia se le ordena 1 litro de D/A 5% con 10g de MgSO4.
¿Cuántos ml de MgSO4 al 50% se han utilizado?
a. 20
b. 5
c. 12,5
d. 30

Causa hipomagnesemia:
a. estreñimiento
b. alcoholismo
c. deshidratación hipertónica
d. insuficiencia renal aguda

Paciente con diuresis de 300ml/día, K sérico 7mEq/L, Posm y Uosm 300mosm/L, el


diagnóstico más probable es:
a. insuficiencia suprarrenal
b. insuficiencia renal
c. SIADH
d. Deshidratación primaria

La mielinosis pontina se caracteriza por:


a. pares craneales intactos
b. cuadriplejía flácida
c. comportamiento normal
d. hipertonía de los miembros inferiores

Calcule el déficit de agua en una paciente con sodio sérico 168mEq/L y que pesa 70Kg
(utilice como porcentaje de agua 50% y sodio normal mEq/L):
i. 5L
j. 3L
k. 10L
l. 7L

En la SIADH la concentración de sodio urinario es de:


i. menos de 10mEq/L
j. 20mEq/L
k. 15mEq/L
l. Mas de 40mEq/L

En paciente con vómitos abundantes y prolongados con hipokalemia severa debe


administrarse:
i. NaCl 0.9% con KHCO3
j. NaCl 0.9% con KCl
k. NaCl 0.9% con gluconato de potasio
l. NaCl 0.9% con NaHCO3

La hipokalemia en un paciente con cirrosis puede contribuir:


i. coma
j. alcalosis respiratoria
k. acidosis metabolica
l. aumento de la síntesis de urea

El plasma de un paciente contiene Na 125mEq/L, glucosa 108mg/dl, urea 300mg/dl, podrá


presentar:
i. cirrosis
j. síndrome nefrósico
k. síntomas de hipertonicidad
l. síntomas de hipotonicidad

La hiponatremia aguda sintomática es debida a, excepto:

127
i. exceso de glucocorticoides
j. SIADH
k. post-operatorio
l. intoxicación hídrica

La administración de solución salina 0.9% produce:


i. disminución del volumen intracelular y aumento del extracelular
j. aumento del volumen intra y extracelular
k. aumento del volumen extracelular
l. aumento del volumen intracelular

La hipokalemia puede predisponer, excepto:


i. intoxicación digitálica
j. disminución de la amoniogénesis
k. rabdomiolisis
l. ondas U en el EKG

Tercer Parcial de Medicina Interna

26. Cuándo disminuye el fosfato?


R. Aumento de PTH

27. Ejemplo de hipernatremia hipervolémica:


R. Aldosteronismo primario

28. La hipocalemia produce:


R. Aumento de la magnitud del potencial de reposo

29. La depleción de agua produce, excepto:


R. Salida de electrolitos del cerebro

30. Velocidad de administración de KCl en caso de urgencia:


R. 40 mEq/h

31. Cambios en EKG de la hipermagnesemia es igual:


R. Hipercalemia

32. Enfermo de 60 Kg con 60% de agua corporal, osmolaridad plasmática 260 mosmol/Kg,
¿cuántos gramos de NaCl aumenta el Na en 10 mEq en 24 horas?
R. a) 15
b) 21
c) 30
d) 19

33. Uso de Calcio en tratamiento de hipercalemia para:


R. Modificar el Potencial de Membrana

34. Hipermagnesemia severa produce:


R. Paro respiratorio

35. Venoclisis de 1L de D/A 5% conteniendo 10 mg de MgSO4 y al 50%:


R. 20mL

36. La hipocalcemia:
R. Prolongo QT a expensas del segmento ST

37. Tratamiento de urgencia de hipercalcemia:


R. SSN 0.9%

38. Causa de hiperfosfatemia:


R. Insuficiencia renal

128
39. Adaptación cerebral, hipertonicidad, excepto:
R. Pérdida de iones

40. En la adaptación cerebral, la hipotensión da lo siguiente, excepto:


R. Ganancia de aminoácidos
41. En ausencia de síntomas de hipernatremia, el sodio debe ser disminuido a razón de:
R. 0.5 mEq/L/h

42. Para corregir un sodio cerca de 185 mEq se administró en 86 horas, 66 letras de D/SS
5%. ¿cuál es la velocidad de administración de la venoclisis?
R. 69 mL

43. Manifestación clínica de hipercalcemia, excepto:


R. a) Hipotensión arterial
b) Bradicardia
c) Nauseas
d) Anorexia

44. Consecuencia de hipofosfatemia, excepto:


R. a) No afecta al SNC
b) Disfunción plaquetaria
c) Cardiomiopatía
d) Osteomalacia

45. Causa de hipofosfatemia, excepto:


R. a) Falla renal
b) Recuperación de cetoacidosis diabética
c) Alcalosis respiratoria
d) Quemaduras graves

46. Causa de HTA con hipocalemia, aldosterona-renina baja:


R. Síndrome de Cushing

47. Causas de HTA con hipocalemia y renina alta:


R. Hipertensión maligna

48. En una deshidratación severa hipertónica con hipotensión arterial severa, la prioridad
inicial es administrar:
R. SSN 0.9%

49. En la secreción inapropiada de ADH se da lo siguiente, excepto:


R. No hay expansión de volumen

50. Tejido con mayor contenido de K+:


R. a) Hígado
b) Músculo
c) Eritrocito
d) Hueso

2. El siguiente no es un criterio de Ranson:


ALT mayor de 250U/L

3. Antibiotico de eleccion en sepsis por pancreatitis de origen biliares


Amoxicilina ac. Clavulanico y cefoxima

4. Marcador de tumor de páncreas:


Ca-199

5. Tres causas de cirrosis de origen metabolico:


Enfermedad de Gaucher, Enfermedad de Wilson
Intolerancia a la fructosa, galactosemia, déficit de antitripsina alfa 1

129
6. En laboratorio de cirrosis hepatica encontramos:
Elevación importante de transaminasas

24. U Na 115, Uosm 240. P osm 680, P Na 62


SIADH

25. Paciente de 84 años que recibe diureticos para la hipertensión arterial. Presenta
diarrea y trastornos del sensorio y disminución del rubor de la piel. Na 174 U
Na 5 U osm 606. La hipernatremia se debe a:
Perdidas insensibles

28. hipocalcemia:
Provoca potencializacion de efectos de hiperkalemia

30. Tratamiento de urgencia hipercalcemica


Solucion salina isotonica 0.9% 4 a 5 L

31. Síndrome nefrotico


Edema, proteinuria, hipertensión arterial, hematuria

32. síntomas de reflujo gastroesofagico


Regurgitación, pirosis, disfagia

33. Paciente con esófago de Barrett


Tiene reflujo, riesgo de adenocarcinoma, vigilancia endoscopica

34. paciente de 60 años con difagia pregresicva y perdida de peso.


CA de esófago
35. factor de riesgo de carcinoma escamoso
Tabaquismo alcohol, radiación ionizante, diverticulos de esófago, Plummer-Vinson

36. terapia optima para hepatitis


Interferon peg y rivabirina

37. hepatitis fulminante en mujeres embarazadas


Hepatitis E

38. hepatitis mas comun en panama por trasmisión fecal oral


Hepatitis A

39. produce hepatitis cronica, cirrosis y cancer


Hepatitis B, C y alcohol

2--En el riñón la hormona paratiroides:


Aumenta la reabsorción tubular de calcio y disminuye la de fósforo

3- La hipercalcemia produce
Potencialización de los efectos de la hipercalemia

36- La hipomagnesemia produce.


Vasodilatación periférica

36-De los factores: nefrotoxicidad./ -La buena hidratación

37 - En la concentración sérica para calcular:: 8,8

38- la velocidad de administración de potasio: 40 meq

39-hipocalcemia produce : fosfaturia

130
40-La causa de hipertensión con hipocalemia: Síndrome de Bartter=:,

41-En paciente con vomito severo hipotensión arterial o ir a la sar con mas de 40
meq
Insuficiencia suprarrenal;

42-La ,administración de sol isotónica aumento de vol extracelulares

43-Glomerulonefritis post estreptococcica Altera el cociente de ultrafiltración

44-En-el túbulo` intersticial se puede dar: La proteinuria de 4 g al día.

15- Enfermedad que puede causar falsos positivos por FTA-ABS: cirrosis biliar

16: Es una señal de haber sufrido de pancreatitis: las opciones eran


hiperamilasemia, aumento de dolor, hipercalemia, aumento de tamaño.

17. ALFA- feto proteína no está elevada en: CA de colon

18- La hipofosfatemia cansa excepto: aumento de vitamina D


20. La renal aguda, oligurica todo es cierto excepto: Osmolaridad urinaria
aumentada

18-Todas son complicaciones de la cirrosis hepatica exceplo


a. peritonitis espontanea
b. perforación del ileo
c. sindrome hepatorenal
d. asc:itis
e. hiperesplenismo

19. Cuando usted evalua a un paciente con diagnostico de cirrosis hepatica y le


da seguimiento) se preocupa de la siguientes situaciones que le puede llevar a una
encealopatia hepatica como, una:
a. hiponatremia
b. ingesta baja en proteínas
c. hipokalcemia
d, sangrado digestivo alto
e- aparición de una infección

20. Un paciente con que sufre de cirrosis hepática, acude a consulta por Fiebre, al
ver su hemograma tiene leucocitos normales y.observa que tiene trombocitopenia
por lo tanto se asume que:
á. • que ademas tiene dengue..
b- hiperesplenismo
c. trombocitopenia autoinrnune
d. que la pierde por sangrado digestivo asintomático
e. debe ser estudiado por esta trómbocitopenia

24. Usted tiene una paciente con hiperhemesis gravidica. El trastorno metabolico
que espera encontrar es:
A - acidosis hipocloremica
b, acidosis hipercloremica'
c- alcalosis hipocloremica
d. alcalosis hipercloremica
e. no hay alteración acido base

25. Cual de las siguientes opciones esta contraindicada en el tratamiento de la


hiperkalcmia
a, gluconato de calcio

131
b. aldactone
c, kayaxelate
d. Bicarbonato de sodio e. Todas las anteriores

2T.-Son manifistaciones de hipertension portal excepto.


a. varices abdominales
b. ascitis
c: telangectasia
d. esplenomegal.ia
e. ictericia

-28. Cual de los siguientes datos refleja vieja gravedad de una pancreatitis:
a. distensión abdominal
b. hipocalcemia
c. hiperamilasemia
d. aumento de la intensidad .del dolor
e. disnea

29. Cual de los siguientes sintornas es menos probables encontrar en pancreatitis


a. dolor abdominal epigastrico
b. diarrea
c. taquipnea
d. vomitos incoercible
e, hipovolemia

30.Lo mas importante que se puede realizarse para evitar el desarrollo de


necrosis en pancrectis es,.
a..' Hidratación energica del paciente por vía parenteral
b. iniciar antibiotico terapia en forma temprana
c. Admitir al paciente a la unidad de cuidados intensivos
d. Colocar tubo nasogastrico para respiración
e. Tratamiento temprano del dolor

31 _Todo lo siguiente es cierto para el cancer de páncreas, excepto~


a. La mayoría se encuentra en la cabeza
b. La ictericia es su principal síntoma
c. La mayoría d los casos no son operable al momento del diagnostico

32- el marcador tumoral más importante del cáncer de páncreas es C - CA-


19-9
33- un paciente de 60 años llega al cuarto de urgencia con dolor abdominal
y usted sospecha de pancreatitis , del lab se informa que no se realizara la
amilasemia por escaces de reactivo . usted le solicita
a- un BHC
b- bilirrubina
c- lipasa pancretica
d- transaminasa
e- CPK
31. Causa de hiperfosfatemia:
e. Falla hepatica
f. Falla renal cronica
g. Falla cardiaca
h. Insuficiencia suprarrenal

32. Causas de hiperfosfatemia grave, excepto:


d. recuperacion de cetoacidosis diabetica. SI
e. Alcalosis respiratoria
f. Acidosis respiratoria. SI

33. Paciente con vómitos severos, oligurico con hipotensión arterial Una>
40…… 800 mOsm/Kg puede tener:

132
e. Insuciencia renal aguda.
f. Insuficiencia suprarrenal
g. Falla pre-renal
h. Exceso de mineralocorticoides

34. Paciente urémico crónico con deshidratación severa puede presentar lo


siguiente excepto:
a. Densidad urinaria 1010.
b. Osmolaridad urinaria de 800 mOsm/Kg
c. K sérico aumentado
d. Oliguria.

35. La hipokalemia produce:


a. Aumento de la magnitud del potencial de reposo

36. Uno de los indices de falla renal aguda es:


Uosm/Tosm

37. Hormona producida en los riñones que disminuye en la falla renal cronica:
a. ADH
b. Noradrenalina
c. Peptido natriuretico (segun arjona)
d. 1,25 (OH)2 Vit D.

38. Las calcificaciones metastásicas se deben:??


a. Aumento del producto Ca, P
b. Aumento del producto Ca, Mg
c. Hipoparatiroidismo
d. Hiperparatiroidismo 1ario

39. Causa de anuria:


a. Pielonefritis unilateral
b. Necrosis tubular aguda
c. Enf cambios minimo
d. Necrosis cortical

40. la etapa de la insuficiencia renal cronica con tasa de filtración glomerular de


60-89 ml/min es la:??
a. 1
b. 2
c. 3
d. 4

41. La FENA es la relación entre:


a. Oferta tubular/Na reabsorbido x 100
b. depuración de Na/ depuración de creatinina x 100
c. oferta tubular/ Na excretado x 100.
d. Na excretado/ Na reabsorvido x 100.

42. En la nefritis tubular intersticial aguda se pierde la capacidad de poder


concentrar la orina debida a:??
a. Daño medular
b. Daño glomerular
c. Daño tubular proximal
d. Daño de la porcion cortical del tubulo colector

43. La solucion de NaCl al 23,4 % tiene:


8000 mOsm/l

44. Se asocia con el SIAD, excepto:


lipoma

133
45. El efecto biologico mas importante de la calcitonina es, excepto:
a. Responder a la hipocalcemia
b. Aumenta la calciuria
c. Estimular la resorcion osea osteoclastica
d. Utilizada en el tx de la hipercalcemia.

46. Favorece la entrada de K al espacio intracelular, excepto:


a. Agonista adrenergico
b. Estimulacion adrenergico
c. Insulina
d. Alcalosis metabolica

47. Factores que afectan el balance hídrico en el post-operatorio, excepto:


a. Fármacos
b. Calcitriol
c. Administración de liquidos hipotonico
d. Aumento de la ADH

48. Las primeras manifestaciones clínicas de la hiponatremia son:??


a. neurológica
b. gastrointestinal
c. pulmonares
d. musculares

49. En la hipernatremia hipertonica se da lo siguiente excepto:


a. Entrada de Na
b. Salida de aa
c. Ganancia de iones
d. Disminución del agua cerebral

50. Manifestaciones clinicas de la hipercalcemia, excepto:


a. Taquicardia
b. HTA
c. Constipación
d. Vomitos

51. En falla renal aguda por nefrotoxicidad:


a. Se conserva la membrana basal
b. Se destruye la membrana basal
c. Frecuente hay oliguria
d. La creatinina no aumenta

52. Causa de hipomagnesemia:


alcoholismo

53. Na serico de 168 mEq/l 70 kg (utilize como % de agua al 50% y Na normal


140 mEq/l
7 litros

54. Indicaciones de diálisis en falla renal aguda, excepto:


a. Encefalopatia uremica
b. Hiperkalemia refractaria
c. Ac. Metabolica refractaria
d. hipocalcemia

55. El KCL IV se puede asministrar asi, excepto:


a.infusion por linea periferica de 10 meq en 100 ml en 1hora
b. “ “ central de 20 meq em 50ml en 1 hora

134
c. “ “ central de 100 meq em 50ml en 1 hora
d. “ “ periferica de 20 meq en 20ml en 2 horas

56. Aumenta la excrecion urinaria de K, excepto:


a. Triamtereno
b. Bumetanida
c. Furosemida
d. Metolazona

57. Factores que regulan la secrecion distal de K, excepto:


a. Flujo tubular distal y aporte distal de Na
b. Excrecion de aniones no reabsorbibles
c. Aldosterona
d. PTH

58. Enfermo de 60 Kg con 60% de agua corporal, Posm 260 mOsm/kg,


¿Cuántos gramos de NaCl aumentaria el Na en 10 mEq?
a. 21 gramos

50. Conteniendo 10 g de MgSO4 al %, cuantos ml de sal necesitaria?


20 ml

51. Cuándo disminuye el fosfato?


R. Aumento de PTH

52. Ejemplo de hipernatremia hipervolémica:


R. Aldosteronismo primario

53. La hipocalemia produce:


R. Aumento de la magnitud del potencial de reposo

54. La depleción de agua produce, excepto:


R. Salida de electrolitos del cerebro

55. Velocidad de administración de KCl en caso de urgencia:


R. 40 mEq/h

56. Cambios en EKG de la hipermagnesemia es igual:


R. Hipercalemia

57. Enfermo de 60 Kg con 60% de agua corporal, osmolaridad plasmática 260


mosmol/Kg, ¿cuántos gramos de NaCl aumenta el Na en 10 mEq en 24 horas?
R. a) 15
b) 21
c) 30
d) 19

58. Uso de Calcio en tratamiento de hipercalemia para:


R. Modificar el Potencial de Membrana

59. Hipermagnesemia severa produce:


R. Paro respiratorio

60. Venoclisis de 1L de D/A 5% conteniendo 10 mg de MgSO 4 y al 50%:


R. 20mL

61. La hipocalcemia:
R. Prolongo QT a expensas del segmento ST

62. Tratamiento de urgencia de hipercalcemia:


R. SSN 0.9%

135
63. Causa de hiperfosfatemia:
R. Insuficiencia renal

64. Adaptación cerebral, hipertonicidad, excepto:


R. Pérdida de iones

65. En la adaptación cerebral, la hipotensión da lo siguiente, excepto:


R. Ganancia de aminoácidos
66. En ausencia de síntomas de hipernatremia, el sodio debe ser disminuido a
razón de:
R. 0.5 mEq/L/h

67. Para corregir un sodio cerca de 185 mEq se administró en 86 horas, 66 letras
de D/SS 5%. ¿cuál es la velocidad de administración de la venoclisis?
R. 69 mL ¿?????

68. Manifestación clínica de hipercalcemia, excepto:


R. a) Hipotensión arterial
b) Bradicardia
c) Nauseas
d) Anorexia

69. Consecuencia de hipofosfatemia, excepto: no sè


R. a) No afecta al SNC
b) Disfunción plaquetaria
c) Cardiomiopatía
d) Osteomalacia

70. Causa de hipofosfatemia, excepto:


R. a) Falla renal
b) Recuperación de cetoacidosis diabética
c) Alcalosis respiratoria
d) Quemaduras graves

71. Causa de HTA con hipocalemia, aldosterona-renina baja:


R. Síndrome de Cushing

72. Causas de HTA con hipocalemia y renina alta:


R. Hipertensión maligna

73. En una deshidratación severa hipertónica con hipotensión arterial severa, la


prioridad inicial es administrar:
R. SSN 0.9%

74. En la secreción inapropiada de ADH se da lo siguiente, excepto:


R. No hay expansión de volumen

75. Tejido con mayor contenido de K+:


R. a) Hígado
b) Músculo
c) Eritrocito
d) Hueso

14) Paciente con Oliguria,……muchos síntomas :

a) Insuficiencia suprarrenal ( creo que era la respuesta)

15) En IRA se mide todo menos:

a) Uosm/Posm x100
b) Na excretado/ Na filtrado x200

136
c) Na filtrado/ Na excretado x100
d) Na urinario >40
e) Ninguna de las anteriores

16) Paciente con hipertensión , hipokalemia, cretinina normal tiene:

a) Hipertensión arterial maligna


b) Cushing
c) Algo relacionado a licorice
TODO creo

31) En hiponatremia el daño principal es:

a) Neurológica
b) Respiratoria
c) Renal

32) Adaptación del cerebro ante solución hipotónica:

a) Pérdida de iones
b) Pérdida de aminoácidos
c) Edema cerebral
d) Ganancia de iones

53) Paciente con hipercalcemia puede ser por todo menos:

a) Aumento de secreción de TSH


b) Hiperparatioidismo
c) Toxicidad de Vitamina D
d) Medicamentosa
e) Leche de álcalis

62) La PTH promueve:

a) Reabsorción tubular de calcio y excreción de fosfato

66) Paciente que sufre golpe en la cabeza y tiene fractura de cráneo. Tiene
osmolaridad urinaria disminuída , pero al darle tratamiento con vasopresina
ésta aumenta. El paciente tiene:

a) Diabetes insípida central


b) Diabetes insípida nefrógena

24. Usted tiene una paciente con hiperemesis gravidica. El trastorno metabolico
que espera encontrar es:
a. acidosis hipocloremica
b. acidosis hipercloremica
c. alcalosis hipocloremica
d. alcalosis hipercloremica
e. no hay alteración acido base

25. Cual de las siguientes opciones esta contraindicada en el tratamiento de la


hiperkalemia
a. gluconato de calcio
b. aldactone
c. kayaxelate
d. Bicarbonato de sodio
e. Todas las anteriores

137
58. La accion toxica mas importante del ganciclovir es:
a. Nauseas y vomitos
b. Depresion de la medula osea
c. Diarrea
d. Insuficiencia Renal
e. Insuficiencia Respiratoria

59. Rotando en sala de medicina interna le piden que evalue un paciente con
HIV el cual por presentar una meningitis por criptococci recibe anfotericina
B; debido a este medicamento buscaria como complicación:
f. aparicion de diarrea
g. aumento de la presion arterial
h. hipokalemia
i. cefalea
j. dolor toráxico

60. Paciente de 84 anos quien recibe diuretico tiazidico para la hipertensión


arterial es admitido por presentar diarrea y trastornos del sensorio. Tiene
disminución del turgor de la piel y la presion arterial es normal, Pna
174mEq/L, Una 5mEq/L, Uosm 606mEq/kg. La hipertensión se debe a:
f. perdidas insensibles
g. polidipsia
h. uso de diurético (yo pienso que es esta)
i. diabetes insipida
j. secrecion inapropiada de hormona antidiuretica

61. Hipernatremia Hipervolemica


f. diuresis osmótica
g. diarrea
h. aldosteronismo primario
i. diabetes insipida
j. SIADH

62. La hipokalemia en un cirrotico puede precipitar COMA por:


f. Hipermagnesemia asociada
g. Aumento de la amoniogenesis
h. Hiperfosfatemia asociada
i. Rabdomiolisis
j. Induce vomitos

Causa de hiperkalemia:
f. Aumento del volumen arterial eficaz
g. Hipercatabolismo tisular
h. Hiperaldosteronismo
i. Alcalosis metabolica
j. Cirrosis hepatica

63. Causa de hipomagnesemia


f. cirrosis avanzada
g. falla renal cronica
h. falla cardiaca
i. constipacion
j. acidosis respiratoria

64. Causa renal de hipomagnesemia (LA PUSE EN ROJA XQ PIENSO QUE


ES UN EXCEPTO, Y EN TAL CASO ES LA FALLA RENAL CRÒNICA)
f. falla renal cronica
g. esteatorrea
h. desnutricion severa
i. uso de diureticos de asa
j. infeccion urinaria

138
65. en caso de hipercalcemia pensar en:
f. neoplasia
g. hiperfosfatemia severa
h. nefrolitiasis
i. nefrocalcinosis
j. diureticos

66. En el síndrome nefrosico puede encontrarse cilindros:


f. cereos
g. hematicos
h. leucocitarios
i. epiteliales
j. mixtos leucocitario y hematicos

31. Causa de hipomagnesemia:


m. estreñimiento
n. alcoholismo
o. falla pre-renal
p. IRA (insuficiencia renal aguda)

32. La hipocalcemia en la falla renal aguda puede ser asintomática por la:
m. acidemia
n. hipoalbuminemia
o. Deficiencia de PTH
p. Alcalemia

33. La hipofosfatemia causa, excepto:


m. Aumento de vit. D
n. Aumento de PTH
o. Disminución de la excresión de fosfato
p. Aumento de la absorción de fosfato a nivel intestinal

34. Un pte con diuresis de 250 ml al día con osmolaridad plasmática (presión
osmótia) y osmolaridad urinaria de 300 mosm/l, el dx probable e:
m. Insuficiencia suprarrenal
n. Insuficiencia renal
o. Nefritis túbulo intersticial crónica
p. Deshidratación hipertónica

35. Pte con oliguria, hiperkalemia, hiponatremia, creatinina 0.8 mg/dl y CPK
normal: Esto puede corresponder a:
m. IRA <de 30 mosm es ira
n. Insuficiencia suprarrenal<--
o. Síndrome hepatorrenal
p. Rabdomiolisis

36. No está indicado en caso de hiperkalemia:


s. Aldactone

37. Mujer con hiperemesis grávida:


s. Alcalosis hipoclórica

38. En causa de hipomagnesemia:


s. Alcoholismo.

39. En el metabolismo del calcio está implicado, menos:


s. Insulina

139
40. Hiponatremia, hipotónico, normovolémico:
s. SIADH

41. Fórmula del clearence de creatinina:


s. clearence = 140 – edad x peso en Kg
72 x creatinina

42. Su infección produce litiasis renal:


r. proteus sp.

43. Fórmula de irritabilidad muscular:


s. IR = Na + K + OH
(Ca) + Mg + H

44. Con respecto al síndrome diarreico, nombre 2 tipos osmóticos:


c. Laxantes osmóticos (Mg2+, PO4, SO4)
d.
Carbohidratos no absorbibles (sorbitol, lactulosa).

45. Nombre dos síndromes diarréicos tipo secretora:


c. Consumo crónico de etanol.
d.
Obstrucción intestinal incompleta.

46. Nombre 2 sindromes diarréicos motores con aumento de motilidad:


g. Hipertiroidismo
h.
Prostaglandinas.

47. Indica falla renal aguda:


p. U.N. = 4.1
q.
U.N. = 1
r.
U.N. = ¼
s.
Gravedad específica: 1.026
t.
Ninguna.

48. Deshidratación por pérdida de agua y densidad urinaria 1.003, creatinina 6:


p. falla renal aguda parenquimatosa
q.
Falla pre-renal
r.
Falla renal obstructiva
s.
Falla renal crónica
t.
No hay falla renal.

49. Uno de los índices de falla renal es:


p. Reabsorción de Na/ oferta tubular de Na x 100
q.
Excreción urinaria de Na/ oferta tubular de Na x 100
r.
Oferta tubular de Na/ excresión urinaria de Na x 100 **

s.
Una / pna x 100
t.
Ninguna

50. El síndrome heatorrenal es causa de falla:


p. pre-renal
q.
renal
r.
posrenal
s.
falla cardiaca
t.
daño tubular

51. Causa más comun de falla renal cronica:


p. Diabetes melitus
q.
HTA
r.
Glomerulonefritis
s.
Enfermedad poliquística renal

140
t.
Vasculitis

36. En la glomerulnefritis aguda por infección, la causa, excepto:


a. proteínas mayor de 10g
b. osmolaridad urinaria normal
c. hematuria
d. HTA
e. Edema

• A pesar de la diversidad de enfermedades indicadas abajo, ellas tienen


muchos síntomas en común. Con frecuencia, los síntomas y signos
observados incluyen: disminución en el volumen de orina, proteína en la
orina (proteinuria), sangre en la orina microscópica o macrscópica
(hematuria), hinchazón (edema), presión sanguínea alta (HTA) y una
disminución en la capacidad del riñón de extraer eficazmente los residuos.

52. Falla renal crónica, excepto:


p. PTH aumentada
q. Hipofosfatemia
r. Acidemia
s. Hipocalcemia
t. 1.25 OH (D3) disminuída.

53. En la falla renal aguda siempre habrá:


p. aumento de creatinina
q. fracción de excresión de Na aumentado NO
r. Hiperkalemia
s. Una aumentado
t. Hipernatremia NO XQ HAY HIPONATREMIA

54. Causa de falla pre-renal:


p. Sd. Nefrótico
q. Obstrucción de ambos uréteres
r. Falla cardiaca
s. Glomerulonefritis aguda pos- infecciosa
t. Calculo renal

55. En la nefritis tubular intersticial se puede dar, excepto:


p. hipernatremia
q. proteinuria de 1.5 g SI
r. normotension arterial
s. hipostenuria
t. hipocalcemia

56. Sugiere glomerulonefritis


j. leucocituria
k. cilindros eritrocitarios
l. anuria
m. ……

57. Causas de falla renal parenquimatosa:


e. papilitis
f. calculo en pelvis derecha
g. estenosis unilateral de arteria renal
h. cistitis hemorragica
i. ………

58. Causa de anuria:


p. Enfermedad de cambios mínimos
q. Necrosis cortical
r. Pielonefritis aguda leve bilateral
s. Necrosis tubular aguda

141
t. Calculo coloriforme bilateral

59. La hipofosfatemia causa todo, excepto:


r- aumento de PTH

60. Hepatitis C periodo de incubación:


r. 15-150 media de 5º (7 semanas)

61. En pte con falla renal crónica que ha sido transplantado esperamos
encontrar:
s. osteomalacia.

Urémico crónico con potasio sérico de 6.5mEq/L con leve academia, sin trastorno
muscular y EKG normal, puede ser tratado con:
e. kayaxelate
f. gluconato de Calcio al 10%
g. NaHCO3
h. Diálisis

La insuficiencia renal crónica ocasiona hiperfosfatemia permanentemente cuando


la tasa de filtración glomerular esta por debajo de:
e. 60ml/min
f. 50 ml/min
g. 25 ml/min
h. 40 ml/min

La hipofosfatemia es causa de excepto


e. inotropismo positivo
f. disminución de 2,3 DPG
g. hemólisis
h. curva de HbO2 desplazada hacia la izquierda

La solución de NaCl 23,4% tiene de osmolaridad:


e. 8000 mosm/l
f. 800 mosm/l
g. 80 mosm/l
h. 0,8 mosm/l

En la hiperkalemia:
e. aumenta la sístole eléctrica
f. hay menor diferencia entre el potencial de reposo y el umbral
g. el potencial de reposo transmembrana se hace mas negativo
h. el potencial umbral se hace menos negativo

La velocidad de administración de potasio en caso de gran riesgo para la vida


puede ser de:
e. 10-20 mEq/L
f. 8 mEq/L
g. 110 mEq/L
h. 40 mEq/L

Un ejemplo de hiponatremia hipotónica normovolemica es:


e. síndrome nefrósico
f. SIADH
g. Diarrea
h. Cirrosis

A una embarazada con eclampsia se le ordena 1 litro (1000ml) de D/A 5% con 10g
de MgSO4. ¿Cuántos ml de MgSO4 al 50% se han utilizado?
a. 20
b. 5

142
c. 12,5
d. 30
(IMAGINO QUIEREN QUE LES RESUELVA ESTO CIERTO? OK)

50% = 50/100=0.5 x 1000ml(1L de solución)= 500g

500g/1000ml= 0,5g/ml

10g MgSO4 x (1ml/0,5g MgSO4)= 20ml

Causa hipomagnesemia:
e. estreñimiento
f. alcoholismo
g. deshidratación hipertónica
h. insuficiencia renal aguda

Paciente con diuresis de 300ml/día, K sérico 7mEq/L, Posm y Uosm 300mosm/L,


el diagnóstico más probable es:
e. insuficiencia suprarrenal
f. insuficiencia renal <350
g. SIADH
h. Deshidratación primaria

La mielinosis pontina se caracteriza por:


e. pares craneales intactos
f. cuadriplejía flácida
g. comportamiento normal
h. hipertonía de los miembros inferiores

Calcule el déficit de agua en una paciente con sodio sérico 168mEq/L y que pesa
70Kg (utilice como porcentaje de agua 50% y sodio normal mEq/L):
m. 5L
n. 3L
o. 10L
p. 7L

En la SIADH la concentración de sodio urinario es de:


m. menos de 10mEq/L
n. 20mEq/L
o. 15mEq/L
p. Mas de 40mEq/L

En paciente con vómitos abundantes y prolongados con hipokalemia severa debe


administrarse:
m. NaCl 0.9% con KHCO3
n. NaCl 0.9% con KCl
o. NaCl 0.9% con gluconato de potasio
p. NaCl 0.9% con NaHCO3

La hipokalemia en un paciente con cirrosis puede contribuir:


m. coma
n. alcalosis respiratoria
o. acidosis metabolica
p. aumento de la síntesis de urea

El plasma de un paciente contiene Na 125mEq/L, glucosa 108mg/dl, urea


300mg/dl, podrá presentar:
m. cirrosis
n. síndrome nefrósico
o. síntomas de hipertonicidad
p. síntomas de hipotonicidad

143
La hiponatremia aguda sintomática es debida a, excepto:
m. exceso de glucocorticoides
n. SIADH
o. post-operatorio
p. intoxicación hídrica

La administración de solución salina 0.9% produce:


m. disminución del volumen intracelular y aumento del extracelular
n. aumento del volumen intra y extracelular
o. aumento del volumen extracelular
p. aumento del volumen intracelular

Alcalosis metabólica con hipertensión arterial, hipokalemia, aldosterona normal y


renina normal:
i. Hipertensión renovascular
j. Hipertensión maligna
k. Sindroma de Cushing
l. Administración de licor

La hipokalemia puede predisponer, excepto:


m. intoxicación digitálica
n. disminución de la amoniogénesis
o. rabdomiolisis
p. ondas U en el EKG

Compendio renal

1) Cuál de las siguientes medicinas pueden dar hiperkalemia


a) inhibidor de la enzima convertidota
b) bloqueador alfa adrenérgico
c) bloqueador beta adrenérgico
d) ayb
e) ayc

2) Hormonas que disminuyen en la falla renal crónica, excepto


a) 1,25 dihidroxi vitatamina D
b) eritropoyetina
c) paratohormona
d) foliculo estimulante

3) MgSO4 (PM 120) 1 gramo contiene de magnesio en meq


a) 10
b) 7.9
c) 8.3
d) 6

4) Factores que conducen a la progresión de falla renal crónica


a) hiperlipidemia
b) toxinas urémicas
c) proteinuria
d) b y c
c) todas

5) Paciente de 20 años presenta abundantes vômitos y diarrea de 3 días de


evolución, examen físico depleción de volumen, suero muestra Na 155, K
3meq/l, Cl 117meq/l, HCO3 25 meq/l, tratamiento de elección:
a) D/A 5% + KCl
b) solución salina 3% + KCl
c) solución salina 23.4% + KCl
d) solución salina 0.45% + KCl

144
6) En la enfermedad ateroembólica renal se da lo siguiente excepto:
d) Velocidad de eritrosedimentación

7) Pseudohiperkalemia de 8 mEq/l que se ve en el EKG:


Onda T picuda
Aplanamiento de la onda P
Ay B
Ninguna

8)Criterios para iniciar diálisis excepto:


Alcalosiss Metabolica

9)Hipokalemia con normotension excepto


Síndrome de liddle

10) Plasma creatinina 8mg/dl N de urea 100mg/dl U na 40 Meq/l :


Insuficiência renal

11) Factores reversibles responsables de deterioro de la funcion renal excepto:


Fibrosis intersticial

12) Causa hipokalemia con renina y aldosterona disminuida:


Licorice

13. Sugiere trastorno túbulo intersticial:


.Proteinuria mayor a 3.5 g /24 horas
.Notrmotensión
.Edema
.Osmolarida urinaria normal

14. Glomérulonefritis rápidamente progresiva


.Engrosamiento de la membrana basal
Fusión de los pedículos de los podocitos
prolñiferacion intracapilar
proliferación extracapilar ( creo que es esa verificar)

15.Una de las siguientes causas no produce hematuria


.enfermedad de cambios mínimos

16.Plasma Na:115, Posm240 mOsm,Uosm 680,Una 60:meq/l:


.Deshidratación hipotónica
.secreción inapropiada de ADH
.Deshidratación isotónica
.Deshidratación primaria

17.en la falla renal crónca produce necrosis


calcifilaxia

18. No produce hipokalemia:


Pancreatitis aguda
rabdomiolisis
neoplasia
Def. Vit D

19. Paciente con glucosa 90mg/dl, Na+= 115mEq/L, Urea en 140mg/dl


a. Síndrome hipoosmolar
b. Síndrome hiperosmolar
c. Diarrea
d. Ninguna de las anteriores.

20. Causa de seudohiponatremia con osmolaridad elevada


a. Hiperproteinemia

145
b. Diabetes mellitus
c. Hiperlipidemidemia
d. Reserción transuretral prostática.

21. Manifestaciones de hipermagnesemia excepto


Normotensión

22. Piel en uremia crónica excepto


Paño blanco

23. No es causa de necrosis papilar aguda


a.Diabetes mellitas
b. Uropatía obstructiva
c. Pielonefritis
d. Trombosis de venas renal.

24. Puede presentarse hiperuricemia excepto


d. Adenocarcinoma gástrico

25. Compromete riñones y pulmones:


a. Enf de GoodPasture
b. Granulomatosis de Wegener
c. LES
d. Ay C
TODASSSSS
26. Sugiere glomerulopatia:
-anasarca (locos)

27. TFG < 15ml/min corresponde al estadio de la enfermedad renal cronica:


* 4

28. Causa obstetrica de necrosis cortical bilateral


- desprendimiento prematuro de placenta

29. Promueve amoniogenesis em cirróticos


- hipokalemia

30. Criterio para diagnostico de síndrome nefrosico


a. Proteinuria en 24 hr de 3.5 gr para superficie corporal de 1. 73
b. Albumina serica < 3g/dl
c.Hiperlipidemia y edema
d. Todas las anteriores
e. ay b

1. En la Hiperkalemia:
R/. Hay una mayor diferencia entre el potencial de reposo y de umbral

2. La hipokalemia en un cirrotico puede desencadenar:


R/. Coma Hepático

3. La hipermagnesemia puede producir:


R/. Paro respiratorio

7. Tratamiento de hiprecalcemia:
R/. SSN 0.9%

8. La hipofosfatemia son causas, excepto:


a. Aumento de la PTH
b. Aumento de la absorción intestinal de fosfato
c. Disminuye la fosfatemia
d. Aumento de la vitamina D

146
9. Una solución isotónica:
R/. Aumenta el volumen extracelular

13. La velocidad de administración de potasio en caso de gran riesgo para la vida


puede ser:
a. 10-20 mEq/L
b. 8mE/L
c. 110mE/L
d. 40 mEq/h

14. Paciente crónico con deshidratación severa puede presentar lo siguiente


excepto:
a. Densidad urinaria 1010
b. Osmolaridad urinaria de 800 mOsm/kg
c. K sérico aumentado
d. Oliguria

15. Paciente con vómito abundante, hipertensión arterial, sodio urinario de 6,


oligúrico, osmolaridad de 800 presenta:
R/.Insuficiencia suprarrenal (fumadoooooos)
Paciente urémico crónico con vómitos severos, hipotensión arterial, orina al azar
muestra Na mas de 40, osmolaridad de 800 puede tener:
Falla suprarrenal

16. Todo lo siguiente es cierto acerca de Colitis Ulcerosa, EXCEPTO:


R/. No está en relación las alteraciones inmunológicas.

17. En quien predomina la enfermedad de Hígado Graso:


R/. Sexo masculino

18. Paciente con Insuficiencia renal crónica, con K sérico de 6, leve academia,
EKG normal, se trata con:
R/. Kayetzalate

22. La hopikalemia predispone, excepto:


R/. Amoniogénesis

25. Cual de las siguientes es causa de hiperfosfatemia:


R/. Insuficiencia renal

26. Paciente con deshidratación hipertónica e hipertensión:


R/. SSN 0.9%

29. En la Glomerulonefritis Aguda post-infecciosa disminuye la tasa de filtración:


R/. Por alteración en el coeficiente de ultrafiltración

23. Px con IRC tine una taza de filtración glomerular de menor de 25 ml/min
;aunque hay yuna tabla de las etapas en esta enfemedad crónica.
Estas osn las TFG:
1. 90 en riesgo
2. 20 ( con TFG normal o incrementada)
3. 60 -89
4. 30 – 59
5. 15-29
Menor a 15 diálisis.

25. Un ejemplo de hipokalemia con aldosterona alta y osmolaridad baja es:


Hiperaldosteronismo primario

147
26.Ejemplode hipernatremia hipervolémica: Hiperaldosteronismo primario

59. La fracción de excreción de sodio estará


aumentada en falla cardíaca.

29.En la hiperkalemia: hay mayor diferencia entre el potencial de reposo y el


umbral.

Manifestaciones cardíacas. El problema médico más grave de la hiperkalemia es


la cardiotoxicidad. Los cambios en el ECG producidos por los niveles altos de
potasio son bastante constantes. A medida que aumentan los niveles se aprecian
los siguientes cambios:

Ondas T picudas (con intervalo QT normal o ligeramente reducido)


Prolongación del intervalo PR con depresión de ST
Desaparición progresiva de la onda P
Bloqueo cardíaco progresivo
Arritmias ventriculares
Paro cardíaco

Las ondas T picudas constituyen el dato en el ECG más constante en la


hiperkalemia.

Efectos neuromusculares. El primer signo neuromuscular de la hiperkalemia


suele ser la aparición de parestesias seguidas de debilidad progresiva de varios
grupos musculares. Si el cuadro se agrava se observa cuadriplejia fláccida. Las
funciones cerebrales y de los pares craneanos se conservan y la parálisis de la
musculatura respiratoria puede ocurrir, pero es excepcional.

30. La infusión de potasio sin riesgo para la vida del PX: es de 40 meq; porque

I lb de solución salina: debe llevr de 20 a40 mEq de KCl a una velocidad de 10 a


20 mEq por hora; si aumenta la velocidad puede provocar un bloqueo A-V. Se
recomienda una Velocidad mayor si la hipokalemia es muy severa.

Sis e usa venas priféricas es:

10 meq en 100 cc de sol.salina para pasar en una hora

20 meq en 200 cc de sol.salina para pasar en una hora

En vena central:

20 meql en 50 cc pasarla en una hora

40 meq en 100cc pasar la en una hora

31.Los indicios de falla renal aguda son:

(Na filtrado/Na excretado) x 100%

32.Na urinario menor de 10: falla cardíaca, presentará edema estará hinchado y
lo puede llevar a una I.C.C. ( Esto lo saque de Medline: Los valores normales
generalmente son de 15 a 250 mEq/L/día, dependiendo del estado de
deshidratación y la ingesta diaria de sodio en la dieta. Los rangos de los valores
normales pueden variar ligeramente entre diferentes laboratorios. Nota: mEq/L/día
= miliequivalentes por litro por día.)

33.Causa de enuresis: necrosis cortical difusa.

148
34.En SIDH la concentración urinaria es menor de 20. (es mayor de 20)

35. Deshidratación pura de agua excepto: hay salida de electrolitos del cerebro.

36.Urémico crónico con deshidratación hipertónica severa y acidótico presentará


lo siguiento excepto: osmolaridad urinaria de 8mm mosmol/kg.

37. Px con vómitos abundante, hipertensión arterial; Na urinario=6, oliguria,


osmolaridad de 800 presenta: insuficiencia suprarrenal. (fumaaaaadooooos, ta
mala)

38. Factor que predispone a la hipernatremia son intoxicación hídrica.

39. Se presenta en acidosis tubular renal proximal excepto: NaHCO3 y lo alfa


antagonistas. (¿??????)

__________________________________________________________________

La hipokalemia produce:
- mayor negatividad del potencial de reposo.

La hipokalemia en un cirrótico puede precipitar COMA por:


- aumento de la amniogénesis

Exceso del kayaxelate en el tratamiento de hiperkalemia puede producir:


- constipación.

Peligro de la hipermagnesemia severa:


- paro respiratorio.

Hipofosfatemia:
- aumenta el calcitriol.

Consecuencia metabólica de hipofosfatemia severa:


- miocardiopatía

Causa de hiperfosfatemia:
- insuficiencia renal

El Síndrome Nefrósicose caracteriza por:


- proteinuria mayor o igual de 3g y albúmina sérica menor o igual de
3g/dl.

En el Síndrome Nefrósico pueden encontrarse:


- cilindros céreos

Urémico agudo, oligúrico, acidótico, deshidratado puede presentar:


- osmolaridad urinaria de 250mosm/kg

La falla renal crónica avanzada cursa con:


- brecha aniónica elevada

- Primera*****Falla renal crónica:


a.HTA
b.Glomerulopatias
c.Diabetes Mellitus
d.Nefritis tubulo intersticial

- Uno de los indices de falla renal aguda es:


Uosm/Tosm

149
- Hormona producida en los riñones que disminuye en la falla renal cronica:
▪ ADH
▪ Noradrenalina
▪ Peptido natriuretico (segun arjona)
▪ 1,25 (OH)2 Vit D.

- Las calcificaciones metastásicas se deben:??


▪ Aumento del producto Ca, P
▪ Aumento del producto Ca, Mg
▪ Hipoparatiroidismo
▪ Hiperparatiroidismo 1ario

- Causa de anuria:
▪ Pielonefritis unilateral
▪ Necrosis tubular aguda
▪ Enf cambios minimo
▪ Necrosis cortical

- la etapa de la insuficiencia renal cronica con tasa de filtración glomerular de


60-89 ml/min es la:??
▪ 1
▪ 2
▪ 3
▪ 4

- La FENA es la relación entre:


▪ Oferta tubular/Na reabsorbido x 100
▪ depuración de Na/ depuración de creatinina x 100
▪ oferta tubular/ Na excretado x 100.
▪ Na excretado/ Na reabsorvido x 100.

- En la nefritis tubular intersticial aguda se pierde la capacidad de poder


concentrar la orina debida a:??
▪ Daño medular
▪ Daño glomerular
▪ Daño tubular proximal
▪ Daño de la porcion cortical del tubulo colector

- La solucion de NaCl al 23,4 % tiene:


8000 mOsm/l

- Se asocia con el SIAD, excepto:


lipoma

- El efecto biologico mas importante de la calcitonina es, excepto:


▪ Responder a la hipocalcemia
▪ Aumenta la calciuria
▪ Estimular la resorcion osea osteoclastica
▪ Utilizada en el tx de la hipercalcemia.

- Favorece la entrada de K al espacio intracelular, excepto:


▪ Agonista adrenergico
▪ Estimulacion adrenergico
▪ Insulina
▪ Alcalosis metabolica

- Factores que afectan el balance hídrico en el post-operatorio, excepto:


▪ Fármacos
▪ Calcitriol
▪ Administración de liquidos hipotonico

150
▪ Aumento de la ADH

- Las primeras manifestaciones clínicas de la hiponatremia son:??


▪ neurológica
▪ gastrointestinal
▪ pulmonares
▪ musculares

- En la hipernatremia hipertonica se da lo siguiente excepto:


▪ Entrada de Na
▪ Salida de aa
▪ Ganancia de iones
▪ Disminución del agua cerebral

- Manifestaciones clinicas de la hipercalcemia, excepto:


▪ Taquicardia
▪ HTA
▪ Constipación
▪ Vomitos

- En falla renal aguda por nefrotoxicidad:


▪ Se conserva la membrana basal
▪ Se destruye la membrana basal
▪ Frecuente hay oliguria
▪ La creatinina no aumenta

- Causa de hipomagnesemia:
alcoholismo

- Na serico de 168 mEq/l 70 kg (utilize como % de agua al 50% y Na normal


140 mEq/l
7 litros

- Indicaciones de diálisis en falla renal aguda, excepto:


▪ Encefalopatia uremica
▪ Hiperkalemia refractaria
▪ Ac. Metabolica refractaria
d. hipocalcemia

- El KCL IV se puede asministrar asi, excepto:


a.infusion por linea periferica de 10 meq en 100 ml en 1hora
b. “ “ central de 20 meq em 50ml en 1 hora
c. “ “ central de 100 meq em 50ml en 1 hora
d. “ “ periferica de 20 meq en 20ml en 2 horas

- Aumenta la excrecion urinaria de K, excepto:


o Triamtereno
o Bumetanida
o Furosemida
o Metolazona

- Factores que regulan la secrecion distal de K, excepto:


o Flujo tubular distal y aporte distal de Na
o Excrecion de aniones no reabsorbibles
o Aldosterona
o PTH

- Enfermo de 60 Kg con 60% de agua corporal, Posm 260 mOsm/kg,


¿Cuántos gramos de NaCl aumentaria el Na en 10 mEq?
a. 21 gramos
30 gramos

151
50. Conteniendo 10 g de MgSO4 al %, cuantos ml de sal necesitaria?
20 ml

60. Causa de hiperfosfatemia:


i. Falla hepatica
j. Falla renal cronica
k. Falla cardiaca
l. Insuficiencia suprarrenal

61. Causas de hiperfosfatemia grave, excepto:


g. recuperacion de cetoacidosis diabetica.
h. Alcalosis respiratoria
i. Acidosis respiratoria.

62. Paciente con vómitos severos, oligurico con hipotensión arterial Una>
40…… 800 mOsm/Kg puede tener:
i. Insuciencia renal aguda.
j. Insuficiencia suprarrenal
k. Falla pre-renal
l. Exceso de mineralocorticoides

63. Paciente urémico crónico con deshidratación severa puede presentar lo


siguiente excepto:
a. Densidad urinaria 1010.
b. Osmolaridad urinaria de 800 mOsm/Kg
c. K sérico aumentado
d. Oliguria.

64. La hipokalemia produce:


a. Disminuciòn de la magnitud del potencial de reposo
17. La PTH promueve todo lo siguiente excepto:
a aumento de la absorción intestinal de calcio
b- aumento de la absorción tubulár de fosfato..
c. aumento de la resorción ósea de fosfato
d- aumento de la resorción osea de calcio
e, disminución de la reabsorción de fosfato

18. Un varon de 57 esta con hemodiálisis de mantenimiento para insuficiencia renal


cronica, ¿Cuál de las sig anomalías metabolicas podría ser anticipáda?
a. hipernatremia
b. hiponatremia
c.
d. exceso de vitamina D
e. hipoparatiroidismo

19. Un px de 25años fue ingresado a UCI por lesiones graves en cabeza y fractura
de la base del Graneo, aprox X horas después de la lesion manifestaba poliuria, la
OSMU 150, y la de suero 350. Los líquidos IV fueron detenidos y 3 hors después la
producción de orina y la osm urinaria permanecieron sin cambio. Se administraron
5 unidades de vasopresina IV, la osm de la orina se
incremento a 300. ¿Cuál es el dx mas probable?
a- Diabetes insípida central
b. diabetes insípida nefrogenica
c. intoxicación acuosa
d. sobrecarga de solutos
e. S IADH

55. indica falla renal aguda


a- U..N.. = 4.1
b- UN = 1
c. U..N.. = 1/4i/

152
d. Gravedad especifica 1.026
e. Ninguna

56. deshidratación por perdida de agua y densidad urinaria 1.003, creatinina 6


a. falla renal aguda parenquimatosa
b- falla prerenal
c. falla renal obstructiva
d. falla renal cronica
e. no hay falla renal

57. uno de los indices de falla renales


a. reabsorción de Na/oferta tubular de Na x 100
b. excresion urinaria deNa/oferta tubular de x 100
c.- oferta tubular de Na/excrecion urinaria de Na x 100 ~.--~
d- Una/Pna x 100
e. Ninguna

58. el Sdr hepatorenal es causa de falla:


a- prerrenal
b. renal
c. póstrenal
d. falla cardiaca
e. daño tubular

59. causa mas común de falla renal crónica:


A- DM

60- en la glomendonefritis aguda por infeccion, la causa, excepto


a- proteina mayor de 10 g
b- osmolaridad urinaria normal
c. hematuria
d. HT A
e. Edema

61. falla renal cronica, excepto


a. PTH aumentado
b- Hipofosfatemia
c- Acidemia
d. Hipocalcemia
e. 1.25 OH (D3) disminuida

62. EN la falla renal aguda siempre habra


a- Aumento de creatinina
b- fraccion de excrecion de Na aumentado
c- hiperkalemia
d- Una aumentado
e- Hipernatremia

63. causa de falla prerenal


a. Sdr nefrotico
b- obstrucción de ambos uréteres
c- falla cardiaca
d. Grlomerulonefritis aguda post-infeciosa
e. Calculo renal

64. en la nefritis tubular interticial se puede dar, excepto


a. h.ipernatremia
b. proteinuria de 1.5 g
c. normotension arterial
d. hipostenuria
e. hipocalcemia

153
65. Sugiere glomerulonefritis:
a. leucocituia
b-cilindros eritrocitarios
c- anuria
d. -

66 -causa de falla renal parenquimatosa


m. papilitis
n. calculo en pelvis derecha
o. estenosis unilateral de arteria renal
p. cistitis hemorragica

2--En el riñón la hormona paratiroides:


Aumenta la reabsorción tubular de calcio y disminuye la de fósforo

3- La hipercalcemia produce
Potencialización de los efectos de la hipocalemia

36- La hipomagnesemia produce.


Vasodilatación periférica

36-De los factores: nefrotoxicidad./ -La buena hidratación

37 - En la concentración sérica para calcular Ca2+:: 8,8

38- la velocidad de administración de potasio: 10-20 meq

39-hipocalcemia produce : fosfaturia

40-La causa de hipertensión con hipocalemia, excepto: Síndrome de Bartter

41-En paciente con vomito severo hipotensión arterial o ir a la sar con mas de 40
meq
Insuficiencia suprarrenal;

42-La ,administración de sol isotónica aumento de vol extracelulares

43-Glomerulonefritis post estreptococcica Altera el cociente de ultrafiltración

44-En-el túbulo` intersticial se puede dar, excepto: La proteinuria de 4 g al día.

25. Cual de las siguientes opciones esta contraindicada en el tratamiento de la


hiperkalemia
a, gluconato de calcio
b. aldactone
c, kayaxelate
d. Bicarbonato de sodio e. Todas las anteriores

43. Rotando en sala de medicina interna le piden que evalue un paciente con HIV
el cual porpresentar una meningitis por criptococci recibe anfotericina B; dicho este
medicamento buscaría como complicación:
a. aparicion de diarrea
b. aumento de la presion arterial
c- hipokalemia
d . cefalea
e. dolor toráxico

81- donde se halla el calcio en el organismo:


a- 50% sangre
b- 20 % proteínas
c- 99% hueso

154
9- una solucion isotonica
a- aumenta el volumen extracell

10- la vel de administración de potasio en caso de riesgo para la vida


a—40 meq por hora

11- Px cronico con deshidratación severa puede presentar los siguiente


excepto:
a-osmolaridad urinaria de 800 mosmol/Kg

12- Px con vomito abundante, HTA, sodio urinario en 6, oligurico,


Osmolaridad de 800 presente,
a- insuficiencia suprarrenal -(FAAAAAAAALLLLLSSSSSOOOOO)

13- Px uremico cronico con vomito severo hipotension arterial orina al azar,
muestra sodio mas de 40, Osmolaridad de 800 puede tener :
a- Falla suprarrenal (CIERTO)

76. Cuándo disminuye el fosfato?


R. Aumento de PTH

77. Ejemplo de hipernatremia hipervolémica:


R. Aldosteronismo primario

78. La hipocalemia produce:


R. DISMINUCIÒN de la magnitud del potencial de reposo

79. La depleción de agua produce, excepto:


R. Salida de electrolitos del cerebro

80. Velocidad de administración de KCl en caso de urgencia:


R. 40 mEq/h

81. Cambios en EKG de la hipermagnesemia es igual:


R. Hipocalemia

82. Enfermo de 60 Kg con 60% de agua corporal, osmolaridad plasmática 260


mosmol/Kg, ¿cuántos gramos de NaCl aumenta el Na en 10 mEq en 24 horas?
R. a) 15
b) 21
c) 30
d) 19

83. Uso de Calcio en tratamiento de hipercalemia para:


R. Modificar el Potencial de Membrana

84. Hipermagnesemia severa produce:


R. Paro respiratorio

85. Venoclisis de 1L de D/A 5% conteniendo 10 mg de MgSO 4 y al 50%:


R. 20mL

86. La hipocalcemia:
R. Prolongo QT a expensas del segmento ST

87. Tratamiento de urgencia de hipercalcemia:


R. SSN 0.9%

88. Causa de hiperfosfatemia:


R. Insuficiencia renal

155
89. Adaptación cerebral, hipertonicidad, excepto:
R. Pérdida de iones

90. En la adaptación cerebral, la hipotensión da lo siguiente, excepto:


R. Ganancia de aminoácidos
91. En ausencia de síntomas de hipernatremia, el sodio debe ser disminuido a
razón de:
R. 0.5 mEq/L/h

92. Para corregir un sodio cerca de 185 mEq se administró en 86 horas, 66 letras
de D/SS 5%. ¿cuál es la velocidad de administración de la venoclisis?
R. 69 mL ¿?????

93. Manifestación clínica de hipercalcemia, excepto:


R. a) Hipotensión arterial
b) Bradicardia
c) Nauseas
d) Anorexia

94. Consecuencia de hipofosfatemia, excepto:


R. a) No afecta al SNC
b) Disfunción plaquetaria
c) Cardiomiopatía
d) Osteomalacia

95. Causa de hipofosfatemia, excepto:


R. a) Falla renal
b) Recuperación de cetoacidosis diabética
c) Alcalosis respiratoria
d) Quemaduras graves

96. Causa de HTA con hipocalemia, aldosterona-renina baja:


R. Síndrome de Cushing

97. Causas de HTA con hipocalemia y renina alta:


R. Hipertensión maligna

98. En una deshidratación severa hipertónica con hipotensión arterial severa, la


prioridad inicial es administrar:
R. SSN 0.9%

99. En la secreción inapropiada de ADH se da lo siguiente, excepto:


R. No hay expansión de volumen

100. Tejido con mayor contenido de K+:


R. a) Hígado
b) Músculo
c) Eritrocito
d) Hueso
101. La insuficiencia renal
funcional de la cirrosis hepática es debida a:
R. Vasoconstricción de las arterias renales

RENAL:

23. Px con IRC tine una taza de filtración glomerular de menor de 25 ml/min
;aunque hay yuna tabla de las etapas en esta enfemedad crónica.
Estas osn las TFG:
1. 90 en riesgo
2. 20 ( con TFG normal o incrementada)
3. 60 -89

156
4. 30 – 59
5. 15-29
Menor a 15 diálisis.

24. Factor que altera el balance hídrico en el periódo post- operatorio:


Disminución de la ADH (Excepto) →FALSOOOOO

25. Un ejemplo de hipokalemia con aldosterona alta y osmolaridad baja es:


Hiperaldosteronismo primario

26.Ejemplode hipernatremia hipervolémica: Hiperaldosteronismo primario

27.Hiperkalemioa son alteraciones del EKG no es necesario administrar gluconato


de calcio se colocará un antagonista de K asíe l Px no cae en paro y estoes si
tiene manifestación en el EKG.

28. La fracción de excreción de sodio


estará aumentada en falla cardíaca.

29.En la hiperkalemia: hay menos diferencia entre el potencial de reposo y el


umbral. (AL FIN UNA CIERTAAAA CON RESPECTO A ESTA PREGUNTA,
ESTABA HARTO DE CAMBIAR EL MENOS POR MAYOR JAJAJA)

Manifestaciones cardíacas. El problema médico más grave de la hiperkalemia es


la cardiotoxicidad. Los cambios en el ECG producidos por los niveles altos de
potasio son bastante constantes. A medida que aumentan los niveles se aprecian
los siguientes cambios:

Ondas T picudas (con intervalo QT normal o ligeramente reducido)


Prolongación del intervalo PR con depresión de ST
Desaparición progresiva de la onda P
Bloqueo cardíaco progresivo
Arritmias ventriculares
Paro cardíaco

Las ondas T picudas constituyen el dato en el ECG más constante en la


hiperkalemia.

Efectos neuromusculares. El primer signo neuromuscular de la hiperkalemia


suele ser la aparición de parestesias seguidas de debilidad progresiva de varios
grupos musculares. Si el cuadro se agrava se observa cuadriplejia fláccida. Las
funciones cerebrales y de los pares craneanos se conservan y la parálisis de la
musculatura respiratoria puede ocurrir, pero es excepcional.

30. La infusión de potasio sin riesgo para la vida del PX: es de 40 meq; porque

I lb de solución salina: debe llevr de 20 a40 mEq de KCl a una velocidad de 10 a


20 mEq por hora; si aumenta la velocidad puede provocar un bloqueo A-V. Se
recomienda una Velocidad mayor si la hipokalemia es muy severa.

Sis e usa venas priféricas es:

10 meq en 100 cc de sol.salina para pasar en una hora

20 meq en 200 cc de sol.salina para pasar en una hora

En vena central:

20 meql en 50 cc pasarla en una hora

157
40 meq en 100cc pasar la en una hora

31.Los indicios de falla renal aguda son:

(Na filtrado/Na excretado) x 100%

32.Na urinario menor de 10: falla cardíaca, presentará edema estará hinchado y
lo puede llevar a una I.C.C. ( Esto lo saque de Medline: Los valores normales
generalmente son de 15 a 250 mEq/L/día, dependiendo del estado de
deshidratación y la ingesta diaria de sodio en la dieta. Los rangos de los valores
normales pueden variar ligeramente entre diferentes laboratorios. Nota: mEq/L/día
= miliequivalentes por litro por día.)

33.Causa de enuresis: necrosis cortical difusa.

34.En SIDH la concentración urinaria es menor de 20. -----(MAYOR)

35. Deshidratación pura de agua excepto: hay salida de electrolitos del cerebro.

38. Factor que predispone a la hipernatremia son intoxicación hídrica.

39. Se presenta en acidosis tubular renal proximal excepto: NaHCO3 y lo alfa


antagonistas.(HE VISTO ESTA PREGUNTA VARIS VECES Y SIGO SIN
ENTENDER QUÈ TRATA DE DECIR)

1. En la Hiperkalemia:
R/. Hay una menor diferencia entre el potencial de reposo y de umbral

2. La hipokalemia en un cirrotico puede desencadenar:


R/. Coma Hepático

3. La hipermagnesemia puede producir:


R/. Paro respiratorio

7. Tratamiento de hiprecalcemia:
R/. SSN 0.9%

8. La hipofosfatemia son causas, excepto:


a. Aumento de la PTH
b. Aumento de la absorción intestinal de fosfato
c. Disminuye la fosfatemia
d. Aumento de la vitamina D

9. Una solución isotónica:


R/. Aumenta el volumen extracelular

13. La velocidad de administración de potasio en caso de gran riesgo para la vida


puede ser:
a. 10-20 mEq/L
b. 8mE/L
c. 110mE/L
d. 40 mEq/h

Paciente urémico crónico con vómitos severos, hipotensión arterial, orina al azar
muestra Na mas de 40, osmolaridad de 800 puede tener:
Falla suprarrenal

158
16. Todo lo siguiente es cierto acerca de Colitis Ulcerosa, EXCEPTO:
R/. No está en relación las alteraciones inmunológicas.

17. En quien predomina la enfermedad de Hígado Graso:


R/. Sexo masculino

18. Paciente con Insuficiencia renal crónica, con K sérico de 6, leve academia,
EKG normal, se trata con:
R/. Kayetzalate

22. La hopikalemia predispone, excepto:


R/. Amoniogénesis

25. Cual de las siguientes es causa de hiperfosfatemia:


R/. Insuficiencia renal

26. Paciente con deshidratación hipertónica e hipertensión:


R/. SSN 0.9%

29. En la Glomerulonefritis Aguda post-infecciosa disminuye la tasa de filtración:


R/. Por alteración en el coeficiente de ultrafiltración

30. Causa de Insuficiencia preerenal, excepto:


R/. Aneurisma Disecante de la Aorta(ESTA NO ES LA RESPUESTA
CORRECTA PUES SÌ ES PRERRENAL)

2) Cuál de las siguientes medicinas pueden dar hiperkalemia


f) inhibidor de la enzima convertidota
g) bloqueador alfa adrenérgico
h) bloqueador beta adrenérgico
i) ayb
j) ayc

2) Hormonas que disminuyen en la falla renal crónica, excepto


a) 1,25 dihidroxi vitatamina D
b) eritropoyetina
c) paratohormona
d) foliculo estimulante

3) MgSO4 (PM 120) 1 gramo contiene de magnesio en meq


a) 10
b) 7.9
c) 8.3
d) 6

4) Factores que conducen a la progresión de falla renal crónica


a) hiperlipidemia
b) toxinas urémicas
c) proteinuria
d) b y c
c) todas

5) Paciente de 20 años presenta abundantes vômitos y diarrea de 3 días de


evolución, examen físico depleción de volumen, suero muestra Na 155, K
3meq/l, Cl 117meq/l, HCO3 25 meq/l, tratamiento de elección:
a) D/A 5% + KCl
b) solución salina 3% + KCl
c) solución salina 23.4% + KCl
d) solución salina 0.45% + KCl

6) En la enfermedad ateroembólica renal se da lo siguiente excepto:

159
d) Velocidad de eritrosedimentación

7) Pseudohiperkalemia de 8 mEq/l que se ve en el EKG: (ESTA ES LA


CORRECTA A Y B)
Onda T picuda
Aplanamiento de la onda P
Ay B
Ninguna

8)Criterios para iniciar diálisis excepto:


Alcalosiss Metabolica

9)Hipokalemia con normotension excepto


Síndrome de liddle

10) Plasama creatinina 8mg/dl N de urea 100mg/dl U na 40 Meq/l :


Insuficiência renal

11) Factores reversibles responsables de deterioro de la funcion renal excepto:


Fibrosis intersticial

12) Causa hipokalemia con renina y aldosterona disminuida:


Licorice

13. Sugiere trastorno túbulo intersticial:


.Proteinuria mayor a 3.5 g /24 horas
.Notrmotensión
.Edema
.Osmolarida urinaria normal
HPOSTENURIA

14. Glomérulonefritis rápidamente progresiva


.Engrosamiento de la membrana basal
Fusión de los pedículos de los podocitos
prolñiferacion intracapilar
proliferación extracapilar ( creo que es esa verificar)

15.Una de las siguientes causas no produce hematuria


.enfermedad de cambios mínimos

16.Plasma Na:115, Posm240 mOsm,Uosm 680,Una 60:meq/l:


.Deshidratación hipotónica
.secreción inapropiada de ADH
.Deshidratación isotónica
.Deshidratación primaria

17.en la falla renal crónca produce necrosis


calcifilaxia

18. No produce hipokalemia:


Pancreatitis aguda
rabdomiolisis
neoplasia
Def. Vit D

19. Paciente con glucosa 90mg/dl, Na+= 115mEq/L, Urea en 140mg/dl


a. Síndrome hipoosmolar
b. Síndrome hiperosmolar
c. Diarrea
d. Ninguna de las anteriores.

20. Causa de seudohiponatremia con osmolaridad elevada

160
a. Hiperproteinemia
b. Diabetes mellitus
c. Hiperlipidemidemia
d. Reserción transuretral prostática.

21. Manifestaciones de hipermagnesemia excepto


Normotensión

22. Piel en uremia crónica excepto


Paño blanco

23. No es causa de necrosis papilar aguda


a.Diabetes mellitas
b. Uropatía obstructiva
c. Pielonefritis
d. Trombosis de venas renal.

24. Puede presentarse hiperuricemia excepto


d. Adenocarcinoma gástrico

25. Compromete riñones y pulmones:


a. Enf de GoodPasture
b. Granulomatosis de Wegener
c. LES
d. Ay C
TODAAAAAS
26. Sugiere glomerulopatia:
-anasarca (NOOOOO)

27. TFG < 15ml/min corresponde al estadio de la enfermedad renal cronica:


* 4

28. Causa obstetrica de necrosis cortical bilateral


- desprendimiento prematuro de placenta

29. Promueve amoniogenesis em cirróticos


- hipokalemia

30. Criterio para diagnostico de síndrome nefrosico


a. Proteinuria en 24 hr de 3.5 gr para superficie corporal de 1. 73
b. Albumina serica < 3g/dl
c.Hiperlipidemia y edema
d. Todas las anteriores
e. ay b

90-94: Mencione, los diversos grupos de fármacos que se pueden usar para el
tratamiento de la EII, y de al menos un ejemplo para cada grupo de
medicamentos:

a. ___________________________________________________________.
b. ___________________________________________________________.
c. ___________________________________________________________.
d. ___________________________________________________________.
e. __________________________________________________________.

95. La EII, es temida, entre otras cosas, porque puede evolucionar de ser, una
enfermedad de tipo inflamatoria, a un CA de Colon, la entidad que mas se asocia
al Cáncer es:

a. CUCI.
b. CD.

161
c. Colitis Indeterminada.
d. Todas por igual.

96. Paciente que llega a su consultorio, diagnosticado con DM hace 2 años, tiene
60 años; llega aquejando debilidad y cansancio, a lo cual se suma una perdida de
peso de mas o menos 15 libras en los últimos dos meses. Al examen físico a
usted le llama la atención, que el paciente tiene ictericia progresiva, vesícula
palpable, pero no dolorosa, y un dolor epigastrico irradiado a la espalda. Mediante
una biopsia de páncreas, al paciente se le diagnostica CA de Páncreas, por sus
características clínicas, debe estar localizado en:

a. Cabeza del páncreas.


b. Cuello del páncreas.
c. Cola del páncreas.
d. Cuerpo del páncreas.

97. Uno de los siguientes, es el principal marcador tumoral del CA de páncreas,


cual es?

a. CA-19.9
b. P-53
c. hMSH2.
d. hMSH1.

98. Por lo general, el diagnostico del CA gastrico se hace en fases:

a. Tempranas.
b. Intermedias.
c. Tardias.
d. Todas las anteriores.

99. La cirugía, para extirpar un CA de páncreas, es la cirugía de:

a. Wipple.
b. Levin.
c. Little.
d. Swan-Ganz.

100. El principal estudio, del tracto digestivo superior es:

a. SEGD.
b. CPRE.
c. Colon por enema.
d. Endoscopia digestiva alta.

101. Paciente masculino de 33 años llega a su consulta por presentar fiebre, un


pequeño rash, signo de Jordano(JAJAJA ES GIORDANO) positivo; el medico de
turno le envía un urinalisis que revela proteinuria, hematuria y múltiples leucocitos;
dentro de su diagnostico diferencial, debe estar una de las siguientes patologías,
cual:

a. Nefritis Intersticial Crónica.


b. Nefritis Intersticial Aguda.
c. Síndrome Nefrosico.
d. Fracaso Renal Agudo.

102. Dentro de las Glomerulopatias, existen algunas, que se pueden presentar con
Síndrome Nefrosico, y otras con Síndrome Nefrítico, cual de las siguientes se
suele presentar con Síndrome Nefrítico:

a. Cambios Mínimos. → nada jjajajaajaja

162
b. Nefropatia IgA. → nefrìtico
c. Nefropatia Membranosa. →nefròtico
d. Diabetes. →nefròtico

103. En un paciente de 65 años, que desarrolla un Sd. Nefrítico Agudo, la etiología


mas común causando de la Glomerulopatia es:

a. Vasculitis.
b. GN rápidamente progresiva.
c. Nefropatia IgA.
d. LES.

104. Cual de las siguientes Glomerulopatia, siempre desarrolla Insufiencia Renal:

a. Cambios Mínimos.
b. Membranosa.
c. Rápidamente Progresiva.
d. Mesangiocapilar.

105. En un paciente con SIADH, al medirle la osmolaridad urinaria (si fuese


plasmática està disminuida por obvias razones), la podemos encontrar:

b. Elevada.
c. Disminuida.
d. Normal.

106. La hiponatermia, se hace clínicamente evidente cuando el sodio, ha


disminuido a:

a. 110 mEq/lt.
b. 115 mEq/lt.
c. 120 mEq/lt.
d. 125 mEq/lt.

107-111. Mencione 5 medicamentos que puedan causar hiperkalemia:

a. _gluconato de Ca2+ y SSN________________________________.


b. __kayexalate (sulfonato de poliestireno)______
______________________.
c. __dextrosa 5%+insulina_____________________________.
d. ______NaHCO3________________.
e. ____diàlisis_________________________.
112-117. Las características del Síndrome Nefrotico son:

a. hipoalbuminemia
b. proteinuria masiva (>3.5/24h)
c. edema
d. hiperlipidemia
e. hematuria leve
f. cilindros cereos

118-120. Los tres puntos en los que se basa la corrección de la hiperkalemia son:

a. _eliminaciòn de K
b. ____internalizacion de K (que no estè en sangre)
c. __inhibir absorciòn de K
121-123. Para cada uno de estos puntos, se suele usar: se le solicita al estudiante
ser lo mas explicito posible en su respuesta, en cuanto a dosis y tratamiento
exacto.

a. __eliminaciòn (gluconato de calcio 1-3 ampollas)


b. __internalizaciòn (dextrosa con insulina (g de dextrosa/5=Ude insulina))

163
c. ____ inhibir absorciòn de K (kayexalate con sorbitol o H2O)

124-126. Debemos sospechar, de alteraciones del Magnesio, en que pacientes:

a. _____IR_____________________.
b. _____hipercalcemia______________________________________.
c. _____alcoholismo_________________________________.

127. Paciente con hipokalemia + alcalosis metabólica + hipertensión arterial +


disminución de la renina plasmática, esta desarrollando un:

a. _______hiperaldosteronismo primario__________.

128. Única causa de diarrea con alcalosis metabólica es:

a. ___administraciòn excesiva de laxantes________.

129. Si un paciente, tiene un potasio de 3 mEq/lt, decimos que ha perdido ya:

a. _0,5 mEq/L de K+(hipokalemia leve)

130. Cual es la principal preocupación que se tiene, cuando un paciente tiene un


aumento en los niveles plasmáticos de Magnesio:

a. _______paro respiratorio

131-132. Causas de acidosis metabólica con hipokalemia:

a. ______diarrea
b. ______cetoacidosis diabètica tratada con insulina

133. Que examen le ayudaria para diferenciarlas: (QUÈ COSA)

a. ________________________________________________.

134. Que esperaria encontrar en una ATR tipo 2:

a. ________Acidosis metabólica por pèrdida

135-136. Principales causas de hipercalcemia:

a. Hiperparatiroidismo secundario
b. _hiperparatiroidismo primario

136-137. Principales causas de hipocalcemia:

a. ____IRC
b. ___hipoparatiroidismo__________.

138-139. Consecuencias de la hiperfosfatemia:

a. hipocalcemia________________.
b. _____calcificación ectópica de tejidos_________.

140-143. El Sd. De Barter, se caracteriza clínicamente por:

a. hipokalemia
b. ________________________________________________.
c. ________________________________________________.
d. ________________________________________________.

164
144. En un paciente con diagnostico de Glomerulonefritis post estreptococcica, el
diagnostico serologico se basa en el hallazgo de anticuerpos:

b. Antiestreptolisina O.
c. Antiestreptolisina A.
d. Antiestreptolisina P.
e. Antiestreptolisina K.

145. En la Nefiritis secundaria al LES, se suelen encontrar positivos los:

b. ANA.
c. ANCA c.
d. ANTI LU
e. ANCA p.

146. Si un paciente es diagnosticado con Falla Renal Crónica, el siguiente punto


es el estadiage del mismo, si tiene una TFG en 82 ml/minuto, esta en estadio:

a. I
b. II
c. III
d. IV
e. V

147-148. La Osteodistrofia Renal, se va a caracterizar clínicamente por la


presencia de:

a. ________________________________________________.
b. ________________________________________________.

149.Normalmente la Kps entre el Fósforo y el calcio es de:

a. ________________________________________________.

150-154. Las características de una muestra de orina con sedimento de tipo


nefrítico, son:

a. ________________________________________________.
b. ________________________________________________.
c. ________________________________________________.
d. ________________________________________________.
e. ________________________________________________.

155-158. Mencione algunos parámetros de laboratorio, para pensar en una IRC


Renal:

a. hiperfosfatemia
b. _____hipocalcemia_______________________________.
c. _____creatinina alta___________________________.
d. _____nitrògeno de urea alto________________________.
e. tfg dismunida

159. El Gasto cardiaco normal es de 5 – 6 litros por minuto; de los cuales, un 25%
le corresponde al riñón, en una relación de:

a. 1200 ml/min.
b. 1300 ml/min.
c. 1400 ml/min.
d. 1500 ml/min.

165
160. Si un paciente, diagnosticado con FRC, presenta ademas una acidosis
metabolica hipercloremica, usted sabe que este paciente debe tener una Brecha
Anionica:

a. Normal.
b. Aumentada.
c. Disminuida.
d. Ninguna de las anteriores.

161. Este mismo paciente, por tener estas características clínicas, debe
encontrarse en estadios de la FRC:

b. Avanzados.
c. Iniciales.
d. No es relevante.
e. Ninguna de las anteriores.

162. Para que por una obstrucción, aumente la creatinina en IRA, se deben haber
perdido:

a. ¼ partes de la masa renal.


b. 2/4 partes de la masa renal.
c. ¾ partes de la masa renal.
d. 4/4 partes de la masa renal.

163. La lesión histologica mas común de IRA es:

a. Necrosis cortical bilateral.


b. Necrosis tubular aguda.
c. Nefritis intersticial.
d. Papilitis necrotizante.

164. Y la mas peligrosa y temida es:

b. Necrosis cortical bilateral.


c. Necrosis tubular aguda.
d. Nefritis intersticial.
e. Papilitis necrotizante.

165. Cual de las siguientes, es la principal causa de IRA, en la maternidad.

b. Abortos infectados.
c. Fetos muertos retenidos.
d. Desprendimiento prematuro de membrana.
e. Eclampsia.
f. Se dan todas por igual.

166
Profeta nefrología I
1) Causa de hiperfosfatemia:
a) Falla 167ecesita
b) Falla renal 167ecesit
c) Falla cardiaca
d) Insuficiencia suprarrenal

2) Causas de hiperfosfatemia grave, excepto:


a) 167ecesitaría167 de cetoacidosis 167ecesitar.
b) Alcalosis respiratoria
c) Acidosis respiratoria.

3) Paciente con vómitos severos, oligurico con hipotensión arterial Una> 40…… 800 mOsm/Kg
puede tener:
a) Insuciencia renal aguda.
b) Insuficiencia suprarrenal
c) Falla pre-renal
d) Exceso de mineralocorticoides

4) Paciente urémico crónico con deshidratación severa puede presentar lo siguiente excepto:
a) Densidad urinaria 1010.
b) Osmolaridad urinaria de 800 mOsm/Kg
c) K sérico aumentado
d) Oliguria.

5) La hipokalemia produce:
a) Aumento de la magnitud del potencial de reposo

6) Primera*****Falla renal crónica:


a.HTA
b.Glomerulopatias
c.Diabetes Mellitus
d.Nefritis tubulo intersticial

7) Uno de los 167ecesit de falla renal aguda es:


a) Uosm/Tosm

8) Hormona producida en los riñones que disminuye en la falla renal 167ecesit:


a) ADH
b) Noradrenalina
c) Peptido natriuretico (según arjona)
d) 1,25 (OH)2 Vit D.

9) Las calcificaciones metastásicas se deben:??


a) Aumento del producto Ca, P
b) Aumento del producto Ca, Mg
c) Hipoparatiroidismo
d) Hiperparatiroidismo 1ario

10) Causa de anuria:


a) Pielonefritis unilateral
b) Necrosis tubular aguda
c) Enf cambios minimo
d) Necrosis cortical

11) la etapa de la insuficiencia renal 167ecesit con tasa de filtración glomerular de 60-89 ml/min es
la:??
a) 1
b) 2
c) 3
d) 4

12) La FENA es la relación entre:


a) Oferta tubular/Na reabsorbido x 100
b) depuración de Na/ depuración de creatinina x 100

167
c) oferta tubular/ Na excretado x 100.
d) Na excretado/ Na reabsorvido x 100.

13) En la nefritis tubular intersticial aguda se pierde la capacidad de poder concentrar la orina
debida a:??
a) Daño medular
b) Daño glomerular
c) Daño tubular proximal
d) Daño de la 168ecesit cortical del tubulo colector

14) La 168ecesita de NaCl al 23,4 % tiene:


a) 8000 mOsm/l

15) Se asocia con el SIAD, excepto:


a) lipoma

16) El efecto 168ecesitar mas importante de la calcitonina es, excepto:


a) Responder a la hipocalcemia
b) Aumenta la calciuria
c) Estimular la 168ecesitar osea osteoclastica
d) Utilizada en el tx de la hipercalcemia.

17) Favorece la entrada de K al espacio intracelular, excepto:


a) Agonista 168ecesitaría
b) Estimulacion 168ecesitaría
c) Insulina
d) Alcalosis metabolica

18) Factores que afectan el balance hídrico en el post-operatorio, excepto:


a) Fármacos
b) Calcitriol
c) Administración de liquidos 168ecesitarí
d) Aumento de la ADH

19) Las primeras manifestaciones clínicas de la hiponatremia son:??


a) neurológica
b) gastrointestinal
c) pulmonares
d) musculares

20) En la hipernatremia 168ecesitaría se da lo siguiente excepto:


a) Entrada de Na
b) Salida de aa
c) Ganancia de iones
d) Disminución del agua cerebral

21) Manifestaciones 168ecesita de la hipercalcemia, excepto:


a) Taquicardia
b) HTA
c) Constipación
d) Vomitos

22) En falla renal aguda por nefrotoxicidad:


a) Se conserva la membrana basal
b) Se destruye la membrana basal
c) Frecuente hay oliguria
d) La creatinina no aumenta

23) Causa de hipomagnesemia:


a) alcoholismo

24) Na serico de 168 mEq/l 70 kg (utilize como % de agua al 50% y Na normal 140 mEq/l
a) 7 litros

25) Indicaciones de diálisis en falla renal aguda, excepto:


a) Encefalopatia 168ecesit
b) Hiperkalemia refractaria
c) Ac. Metabolica refractaria

168
d) hipocalcemia

26) El KCL IV se puede asministrar asi, excepto:


a) 169ecesita por 169eces 169ecesitarí de 10 meq en 100 ml en 1hora
b) “ “ central de 20 meq em 50ml em 1 hora
c) “ “ central de 100 meq em 50ml em 1 hora
d) “ “ 169ecesitarí de 20 meq en 20ml en 2 horas

27) Aumenta la 169ecesitar urinaria de K, excepto:


a) Triamtereno
b) Bumetanida
c) Furosemida
d) Metolazona

28) Factores que regulan la 169ecesitar distal de K, excepto:


a) Flujo tubular distal y aporte distal de Na
b) Excrecion de aniones no reabsorbibles
c) Aldosterona
d) PTH

29) Enfermo de 60 Kg con 60% de agua corporal, Posm 260 mOsm/kg, ¿Cuántos gramos de
NaCl 169ecesitarí el Na en 10 mEq?
a) 21 gramos

30) Conteniendo 10 g de MgSO4 al %, cuantos ml de sal 169ecesitaría?


a) 20 ml

Profeta II

102. Cuándo disminuye el fosfato?


R. Aumento de PTH

103. Ejemplo de hipernatremia hipervolémica:


R. Aldosteronismo primario

104. La hipocalemia produce:


R. Aumento de la magnitud del potencial de reposo

105. La depleción de agua produce, excepto:


R. Salida de electrolitos del cerebro

106. Velocidad de administración de KCl en caso de urgencia:


R. 40 mEq/h

107. Cambios en EKG de la hipermagnesemia es igual:


R. Hipercalemia

108. Enfermo de 60 Kg con 60% de agua corporal, osmolaridad plasmática 260


mosmol/Kg, ¿cuántos gramos de NaCl aumenta el Na en 10 mEq en 24 horas?
R. a) 15
b) 21
c) 30
d) 19

109. Uso de Calcio en tratamiento de hipercalemia para:


R. Modificar el Potencial de Membrana

110. Hipermagnesemia severa produce:


R. Paro respiratorio

111. Venoclisis de 1L de D/A 5% conteniendo 10 mg de MgSO4 y al 50%:


R. 20mL

169
112. La hipocalcemia:
R. Prolongo QT a expensas del segmento ST

113. Tratamiento de urgencia de hipercalcemia:


R. SSN 0.9%

114. Causa de hiperfosfatemia:


R. Insuficiencia renal

115. Adaptación cerebral, hipertonicidad, excepto:


R. Pérdida de iones

116. En la adaptación cerebral, la hipotensión da lo siguiente, excepto:


R. Ganancia de aminoácidos
117. En ausencia de síntomas de hipernatremia, el sodio debe ser disminuido a razón de:
R. 0.5 mEq/L/h

118. Para corregir un sodio cerca de 185 mEq se administró en 86 horas, 66 letras de
D/SS 5%. ¿cuál es la velocidad de administración de la venoclisis?
R. 69 mL ¿?????

119. Manifestación clínica de hipercalcemia, excepto:


R. a) Hipotensión arterial
b) Bradicardia
c) Nauseas
d) Anorexia

120. Consecuencia de hipofosfatemia, excepto:


R. a) No afecta al SNC
b) Disfunción plaquetaria
c) Cardiomiopatía
d) Osteomalacia

121. Causa de hipofosfatemia, excepto:


R. a) Falla renal
b) Recuperación de cetoacidosis diabética
c) Alcalosis respiratoria
d) Quemaduras graves

122. Causa de HTA con hipocalemia, aldosterona-renina baja:


R. Síndrome de Cushing

123. Causas de HTA con hipocalemia y renina alta:


R. Hipertensión maligna

124. En una deshidratación severa hipertónica con hipotensión arterial severa, la


prioridad inicial es administrar:
R. SSN 0.9%

125. En la secreción inapropiada de ADH se da lo siguiente, excepto:


R. No hay expansión de volumen

126. Tejido con mayor contenido de K+:


R. a) Hígado
b) Músculo
c) Eritrocito
d) Hueso

profeta III

15) En IRA se mide todo menos:

170
a) Uosm/Posm x100
b) Na excretado/ Na filtrado x200
c) Na filtrado/ Na excretado x100
d) Na urinario >40
e) Ninguna de las anteriores

16) Paciente con hipertensión , hipokalemia, cretinina normal tiene:

a) Hipertensión arterial maligna


b) Cushing
c) Algo relacionado a licorice

31) En hiponatremia el daño principal es:

a) Neurológica
b) Respiratoria
c) Renal

32) Adaptación del cerebro ante solución hipotónica:

a) Pérdida de iones
b) Pérdida de aminoácidos
c) Edema cerebral
d) Ganancia de iones

53) Paciente con hipercalcemia puede ser por todo menos:

a) Aumento de secreción de TSH


b) Hiperparatioidismo
c) Toxicidad de Vitamina D
d) Medicamentosa
e) Leche de álcalis

62) La PTH promueve:

a) Reabsorción tubular de calcio y excreción de fosfato

1) Paciente que sufre golpe en la cabeza y tiene fractura de cráneo. Tiene


osmolaridad urinaria disminuída , pero al darle tratamiento con vasopresina ésta
aumenta. El paciente tiene:

a) Diabetes insípida central


b) Diabetes insípida nefrógena

Profeta III

1) Hiponatremia hipervolemica:
a) R/.Falla cardiaca

2) Las perdidas fecales excesivas de potasio evocan:


a) R/.tumor velloso

3) Paciente de 84 anos quien recibe diuretico tiazidico para la hipertensión


arterial es admitido por presentar diarrea y trastornos del sensorio. Tiene
disminución del turgor de la piel y la presion arterial es normal, Pna 174mEq/L,
Una 5mEq/L, Uosm 606mEq/kg. La hipertensión se debe a:
a) perdidas insensibles
b) polidipsia
c) uso de diuretico
d) diabetes insipida
e) secrecion inapropiada de hormona antidiuretica

171
4) Hipernatremia Hipervolemica
a) diuresis osmótica
b) diarrea
c) aldosteronismo primario
d) diabetes insipida
e) SIADH

5) La hipokalemia en un cirrotico puede precipitar COMA por:


a) Hipermagnesemia asociada
b) Aumento de la amoniogenesis
c) Hiperfosfatemia asociada
d) Rabdomiolisis
e) Induce vomitos

172
6) Causa de hiperkalemia:
a) Aumento del volumen arterial eficaz
b) Hipercatabolismo tisular
c) Hiperaldosteronismo
d) Alcalosis metabolica
e) Cirrosis hepatica

7) Causa de hipomagnesemia
a) cirrosis avanzada
b) falla renal cronica
c) falla cardiaca
d) constipacion
e) acidosis respiratoria

8) Causa renal de hipomagnesemia


a) falla renal cronica
b) esteatorrea
c) desnutricion severa
d) uso de diureticos de asa
e) infeccion urinaria

9) en caso de hipercalcemia pensar en:


a) neoplasia
b) hiperfosfatemia severa
c) nefrolitiasis
d) nefrocalcinosis
e) diureticos

10) En el síndrome nefrosico puede encontrarse cilindros:


a) cereos
b) hematicos
c) leucocitarios
d) epiteliales
e) mixtos leucocitario y hematicos

101. Paciente masculino de 33 años llega a su consulta por presentar fiebre, un pequeño rash,
signo de Jordano positivo; el medico de turno le envía un urinalisis que revela proteinuria,
hematuria y múltiples leucocitos; dentro de su diagnostico diferencial, debe estar una de las
siguientes patologías, cual:

e. Nefritis Intersticial Crónica.


f. Nefritis Intersticial Aguda.
g. Síndrome Nefrosico.
h. Fracaso Renal Agudo.

102. Dentro de las Glomerulopatias, existen algunas, que se pueden presentar con Síndrome
Nefrosico, y otras con Síndrome Nefrítico, cual de las siguientes se suele presentar con Síndrome
Nefrítico:

e. Cambios Mínimos.
f. Nefropatia IgA.
g. Nefropatia Membranosa.
h. Diabetes.

103. En un paciente de 65 años, que desarrolla un Sd. Nefrítico Agudo, la etiología mas común
causando de la Glomerulopatia es:

e. Vasculitis.
f. GN rápidamente progresiva.

173
g. Nefropatia IgA.
h. LES.
Creo..

104. Cual de las siguientes Glomerulopatia, siempre desarrolla Insufiencia Renal:

e. Cambios Mínimos.
f. Membranosa.
g. Rápidamente Progresiva.
h. Mesangiocapilar.
Creo…

105. En un paciente con SIADH, al medirle la osmolaridad urinaria, la podemos encontrar:

e. Elevada.
f. Disminuida.
g. Normal.

106. La hiponatermia, se hace clínicamente evidente cuando el sodio, ha disminuido a:

e. 110 mEq/lt.
f. 115 mEq/lt.
g. 120 mEq/lt.
h. 125 mEq/lt.

107-111. Mencione 5 medicamentos que puedan causar hiperkalemia:

f. _____________________espironolactona___________________________.
g. __________________________________trimetropin______________.
h. ___________________________________________pentamidina_____.
i. ________________________________________________.IECA
j. ________________________________________________.AINES
112-117. Las características del Síndrome Nefrotico son:

g. _________________________hipoalbuminemia mayor de 3g
/dl_______________________.
h. ____________________3.5 g de preoteina en orina en 24 h en 1.73m2 de superficie
corporal____________________________.
i. _______hiperlipidemia_________________________________________.
j. ___________________edema_____________________________.
k. _________________________hipertension_______________________.
l. ___________________________________cilindro cereo _____________.

118-120. Los tres puntos en los que se basa la corrección de la hiperkalemia son:

d. ____________________excrecion____________________________.
e. ____________________________absorcion____________________.
f. ____________________________________resorcion____________.

121-123. Para cada uno de estos puntos, se suele usar: se le solicita al estudiante ser lo mas
explicito posible en su respuesta, en cuanto a dosis y tratamiento exacto.

d. __________expandir volumen con ssn+kcl cuando no hay IC +


furosemida.______________________________________.
e. ___________________quelante HPO4 o EDTA(no se usa por daño
renal)_____________________________.
f. ____________________ bifosfonatos pamidronato 1 dosis por semana

124-126. Debemos sospechar, de alteraciones del Magnesio, en que pacientes:

d. __________________________trastornos de tubo digestivo x mal


absorcion______________________.
e. ________________________cirroticos________________________.
f. ________________________________embarazadas con THE________________.

174
127. Paciente con hipokalemia + alcalosis metabólica + hipertensión arterial + disminución de la
renina plasmática, esta desarrollando un:

b. ________________________aldosteronismo primarios
(creo)________________________.

128. Única causa de diarrea con alcalosis metabólica es:

b. ______________________________________________no se__.

129. Si un paciente, tiene un potasio de 3 mEq/lt, decimos que ha perdido ya:

b. __________________________creo q 200______________________. Ya que por cada


mol de K que se pierde hay menos 200 mmol de k corporal.

130. Cual es la principal preocupación que se tiene, cuando un paciente tiene un aumento en los
niveles plasmáticos de Magnesio:

b. ________________________bloqueo av completo deprsion


respirarotia________________________.

131-132. Causas de acidosis metabólica con hipokalemia:

c. ___________________________acidosis tubular renal_____________________.


d. ___________________________no se la otra_____________________.

133. Que examen le ayudaria para diferenciarlas:

b. ________________________no se________________________.

134. Que esperaria encontrar en una ATR tipo 2:

b. _____________________nose q es esto___________________________.

135-136. Principales causas de hipercalcemia:

c. ____________________rabdomiolisis____________________________.
d. _______________________________acidosis diabetica_________________.

136-137. Principales causas de hipocalcemia:

c. __________________________falla renal cronica______________________.


d. ______________poca ingestión (en el libro decía que por comer arcilla pero no creo q
sea principal causa.)__________________________________.

138-139. Consecuencias de la hiperfosfatemia:

c. ___________________hipocalcemia por disminución de


paratohomrona_____________________________.
d. _______________________________________________.

140-143. El Sd. De Barter, se caracteriza clínicamente por:

e. _________________hipocalemia_______________________________.
f. ____________________normotensoion_________________________.
g. _________________________________alcalosis metabolica
h. ________________________________________________.hiperaldosteronismo
hipereninemico

144. En un paciente con diagnostico de Glomerulonefritis post estreptococcica, el diagnostico


serologico se basa en el hallazgo de anticuerpos:

175
f. Antiestreptolisina O.
g. Antiestreptolisina A.
h. Antiestreptolisina P.
i. Antiestreptolisina K.

145. En la Nefiritis secundaria al LES, se suelen encontrar positivos los:

f. ANA.
g. ANCA c.
h. ANTI LU
i. ANCA p.

146. Si un paciente es diagnosticado con Falla Renal Crónica, el siguiente punto es el estadiage del
mismo, si tiene una TFG en 82 ml/minuto, esta en estadio:

f. I
g. II
h. III
i. IV
j. V

147-148. La Osteodistrofia Renal, se va a caracterizar clínicamente por la presencia de:

c. _____________________osteitis fibrosa quistica__.


d. _____________________osteomallacia u psteopatia adinamica

149.Normalmente la Kps entre el Fósforo y el calcio es de:

b. ________________________no c____________________.

150-154. Las características de una muestra de orina con sedimento de tipo nefrítico, son:
noc
f. _____________________________microhematuria_____.
g. ____________________cilindrono hematico______.
h. ________________________________________________.
i. ________________________________________________.
j. ________________________________________________.

155-158. Mencione algunos parámetros de laboratorio, para pensar en una IRC Renal:
________________________BUN_____________________.
f. __________________________creatinina____________.
g. ________hemoglobina
h. _________________me __ da pereza pensa___ y no c.__________________________.

159. El Gasto cardiaco normal es de 5 – 6 litros por minuto; de los cuales, un 25% le corresponde al
riñón, en una relación de:

e. 1200 ml/min.
f. 1300 ml/min.
g. 1400 ml/min.
h. 1500 ml/min.

160. Si un paciente, diagnosticado con FRC, presenta ademas una acidosis metabolica
hipercloremica, usted sabe que este paciente debe tener una Brecha Anionica:

e. Normal.
f. Aumentada.
g. Disminuida.
h. Ninguna de las anteriores.
Nunca entendi esto ni en fisio
161. Este mismo paciente, por tener estas características clínicas, debe encontrarse en estadios de
la FRC:

f. Avanzados.

176
g. Iniciales.
h. No es relevante.
i. Ninguna de las anteriores.

162. Para que por una obstrucción, aumente la creatinina en IRA, se deben haber perdido:

e. ¼ partes de la masa renal.


f. 2/4 partes de la masa renal.
g. ¾ partes de la masa renal.
h. 4/4 partes de la masa renal.

163. La lesión histologica mas común de IRA es:

e. Necrosis cortical bilateral.


f. Necrosis tubular aguda.
g. Nefritis intersticial.
h. Papilitis necrotizante.

164. Y la mas peligrosa y temida es:

f. Necrosis cortical bilateral.


g. Necrosis tubular aguda.
h. Nefritis intersticial.
i. Papilitis necrotizante.

165. Cual de las siguientes, es la principal causa de IRA, en la maternidad.

g. Abortos infectados.
h. Fetos muertos retenidos.
i. Desprendimiento prematuro de membrana.
j. Eclampsia.
k. Se dan todas por igual.
Todas pueden llevar a IRA pero creo q la principal es esta

1) Un paciente de 35 años con insuficiencia renal crónica secundaria a pielonefritis


crónica recibe un trasplante renal de cadáver con el que compartía dos identidades
en A y B y una en DR. Recibe tratamiento inmunosupresor con ciclosporina A y
corticoides a dosis estándar. En el postoperatorio inmediato se observa buena
diuresis y no es necesario el tratamiento sustitutivo con hemodiálisis. En el 5º día
de evolución, el paciente presenta fiebre de 38ª, TA de 180/110, oliguria y
disminución en la concentración urinaria de sodio. El diagnóstico más probable
sería:

a) Crisis hipertensiva.
b) Infección respiratoria.
c) Pielonefritis aguda del injerto renal.
d) Recidiva de su enfermedad renal.
e) Rechazo agudo del injerto renal.

2) ¿A cuál de los siguientes factores NO se asocia la Pielonefritis Aguda por


Pseudomonas Aeuruginosa?:

a) Embarazo.
b) Sonda urinaria.
c) Nefrolitiasis.
d) Manipulación urológica.
e) Estenosis de la vía urinaria.

3) Paciente diagnosticado de hiperplasia benigna de próstata que presenta como


sintomatología dificultad para el inicio de la micción, disminución de la fuerza y
volumen del chorro miccional, micción entrecortada y goteo terminal; la aparición
de polaquiuria, y urgencia miccional se debe generalmente a:

a) Presencia de infección urinaria.

177
b) Inestabilidad vesical.
c) Prostatitis (adenomitis).
d) Existencia de residuo vesical.
e) Presencia de divertículos vesicales.

4) Una paciente de 15 años de edad consulta por poliuria y nicturia, cansancio fácil y
astenia. Presenta dichos síntomas desde hace años, y tienden a intensificarse
durante los veranos muy calurosos, en los que se asocia hormigueo lingual y
peribucal. A la exploración física sólo destaca una tensión de 100/50 mmHg. El
ionograma muestra Na 135 mEq/l, K 2 mEq/l, Cl 105 mEq/l, pH 7.45, bicarbonato 30
mEq/l. La determinación de renina y aldosterona muestra cifras elevadas, tanto
basales como tras estímulo. De los procesos que siguen ¿cuál es compatible con el
cuadro clínico descrito?:

a) Hipoaldosteronismo primario.
b) Estenosis de la arteria renal.
c) Síndrome de Liddle.
d) Enfermedad de Addison.
e) Síndrome de Bartter. M. clínicas: hipokalemia, alcalosis metabólica, hiperaldosteronismo
por hiperplasia del aparato yuxtaglomerular y disminución del volumen circulatorio
efectivo →que puede causar hipotensión arterial o normotensión.

5) Un hombre, adicto a drogas por vía parenteral, está ingresado por endocarditis
infecciosa. Durante su enfermedad presenta un cuadro de glomerulonefritis aguda.
¿Cuál de las respuestas es INCORRECTA?:

a) Suele ser debida a inmunocomplejos.


b) No suele presentar piuria.
c) El complemento está descendido.
d) A veces produce síndrome nefrótico.
e) Suele evolucionar favorablemente al controlar la infección cardiaca.

6) En la poliquistosis renal del adulto, ¿cuál de las siguientes afirmaciones es FALSA?:

a) Es una enfermedad hereditaria, autosómica dominante.


b) Habitualmente se detecta en la primera infancia con ecografía.
c) Es causa de deterioro progresivo de la función renal.
d) Frecuentemente cursa con hipertensión arterial.
e) Se le asocia litiasis renal en un 15-20% de los casos.

7) Señalar la respuesta correcta en relación con la fisiopatología del calcio en la


insuficiencia renal crónica:

a) La hormona paratiroidea se eleva precozmente y de forma progresiva.


b) Mecanismos compensadores consiguen mantener a la hormona paratiroidea dentro de
límites normales hasta estadíos avanzados de la insuficiencia renal.
c) Disminuye la síntesis de 25-hidroxi-vitamina D.
d) El calcio sanguíneo aumenta paralelamente a la disminución del filtrado glomerular.
e) Aumenta la síntesis de 1,25-dihidroxi-vitamina D.

8) Una mujer de 68 años acude al Servicio de Urgencias por malestar general que ha
ido progresando en los últimos 15 días, a partir de un episodio gripal. Ha notado
disminución progresiva del volumen de diuresis, edemas maleolares y dificultad
respiratoria. Es hipertensa. En la analítica destaca una creatinina plamática de 5
mg/dl, urea 180 mg/dl, Na 138 mEq/l, K 4.9 mEq/l. Las cifras de complemento son
normales. Los anticuerpos anti-membrana basal son negativos. En la orina presenta
cilindros hemáticos, proteinuria de 1 g/l y microhematuria. Aporta una analítica de
un mes antes, sin alteraciones. ¿Cuál de los siguientes diagnósticos es más
probable?: al haber falta de opciones puse las posibles consideraciones que se
puedan escoger.

a) El cuadro clínico que presenta esta paciente es el SINDROME NEFRÍTICO AGUDO en el


cual habría que descartar todas las causas de este tipo.
b) Las cifras normales de complemento y las pruebas de autoinmunidad normales descartan un
nefrtis lupica u otro tipo de enfermedad auto inmunitarias.
c) Al haber padecido un cuadro clínico tipo gripal se puede pensar en una infección viral.(

178
citomegalovirus, influenza, sarampión, parvovirus, virus parotiditis, Epstein-Barr, dengue
hemorrágico, virus coxackie).

9) Un varón de 45 años llega comatoso a Urgencias. Presenta PaCO2 basal 25 mmHg,


pH 7,15, hipocalcemia moderada con Gap aniónico y osmolar elevados, leucocitosis
y cristaluria. Con más probabilidad este paciente tiene una intoxicación por:

a) Barbitúricos.
b) Monóxido de carbono.
c) Etilenglicol.
d) Benzodiacepinas.
e) Salicilatos.

10) Cuando un paciente bajo tratamiento diurético con tiazidas o furosemida incumple
la dieta y come más sal de la prescrita, el resultado análitico esperable es:

a) Mayor hipernatremia.
b) Mayor hiponatremia.
c) Mayor hiperpotasemia.
d) Mayor hipopotasemia.
e) Mayor acidosis.

11) ¿En cuál de las siguientes glomerulonefritis hay activación del complemento por la
vía alternativa?:

a) Aguda secundaria a endocarditis.


b) Aguda postestreptocócica.
c) Difusa lúpica.
d) Por crioglobulinemia.
e) Membranosa.

12) En un enfermo con un síndrome de Goodpasture que le ha conducido a una


insuficiencia renal crónica:

a) No debe realizarse un trasplante renal, pues la enfermedad le dañará el riñón trasplantado.


b) El trasplante puede realizarse si la producción de anticuerpos anti-membrana basal ha
cesado. ( Pag. 1788 dice: el transplante es posible, pero ante el peligro de recidiva, la
experiencia sugiere que el enfermo debe esperar 6 meses y hasta que no se detecten en
suero los anticuerpos.)
c) Debe seguir siendo tratado indefinidamente con inmunosupresores para proteger su pulmón.
d) Debe seguir siendo tratado indefinidamente con esteroides para proteger su pulmón.
e) La función renal se recuperará cuando dejen de producirse autoanticuerpos anti-membrana
basal.

13) En relación a la hemoglobinuria paroxística nocturna, señale cuál de las siguientes


afirmaciones es FALSA:

a) Es un defecto adquirido.
b) Falta la molécula de anclaje glicosilfosfoinositol en el membrana.
c) Cursa con un aumento de la resistencia del hematíe a la lisis por el complemento.
d) Se acompaña de un aumento de frecuencia de trombosis venosas.
e) Presenta a menudo leucopenia y trombopenia.

14) Con respecto a la nefropatía diabética, señalar la respuesta FALSA:

a) La diabetes tipo 2 es la etiología más frecuente de insuficiencia renal terminal en el mundo


occidental.
b) Más del 90% de los diabéticos tipo 1 desarrollan nefropatía a los 30 años del diagnóstico de
diabetes.
c) La alteración renal más temprana es la hiperfiltración.
d) La existencia de microalbuminuria predice el desarrollo de nefropatía clínica.

179
e) La gran mayoría de los diabéticos tipo 1 con nefropatía tienen también retinopatía.

15) El diagnóstico de la anemia hemolítica se realiza gracias a cinco signos biológicos


característicos: elevación de los reticulocitos, hiperregeneración eritroblástica,
hiperbilirrubinemia no conjugada, incremento de la lácticodeshidrogenasa sérica
(LDH) y descenso de la haptoglobina. ¿Cuáles de estos signos biológicos pueden
observarse también en las pérdidas de sangre por hemorragia?:

a) Descenso de la haptoglobina e hiperregeneración eritroblástica.


b) Elevación de LDH y bilirrubina no conjugada.
c) Hiperregeneración eritroblástica y elevación de la cifra de reticulocitos.
d) Elevación de la bilirrubina no conjugada y descenso de la haptoglobina.
e) Descenso de la haptoglobina y elevación de la LDH.

16) En el tratamiento de la hiperpotasemia grave de un paciente urémico NO está


indicado administrar:

a) Resinas de intercambio catiónico (sodio, calcio).


b) Bicarbonato sódico i.v.
c) Glucosa con insulina i.v.
d) Gluconato cálcico o cloruro cálcico i.v.
e) Tiacidas por vía oral.

17) Paciente de 82 años de edad, que refiere síndrome constitucional de tres semanas
de evolución, con astenia, anorexia y pérdida de peso con oligoanuria progresiva en
las veinticuatro horas previas al ingreso hospitalario. No signos de
hiperhidratación. Creatinina plasmática 6 mg/dl. Proteinuria 1gr/24h. Sedimiento:
microhematuria. Determinación de ANCA positivo, patrón p-ANCA anti MPO. ECO
renal que muestra riñón derecho pequeño y riñón izquierdo de tamaño normal.
¿Cuál cree que es el procedimiento más adecuado y prioritario?:

a) Iniciar tratamiento sustitutivo con diálisis.


b) Proceder a practicar biopsia renal.
c) Iniciar tratamiento con pulsos intravenosos de metilprednisolona y Ciclofosfamida oral.
d) Plantear plasmaféresis.
e) Iniciar tratamiento con Prednisona oral.

180
18)

181
19) Un paciente diabético conocido, de 2 años de evolución y 64 años de edad, consulta por anemia,
proteinuria de 3 gramos/24 horas junto con hematuria, hipertensión arterial y discreta insuficiencia
renal con creatinina plasmática de 2.3 mg/dl. El diagnóstico MENOS probable es:
a) Nefropatía diabética.
b) Angeítis necrotizante del tipo poliangeítis microscópica.
c) Glomerulonefritis rápidamente progresiva.
d) Granulomatosis de Wegener.
e) Síndrome úremico-hemolítico.

20) Si un paciente con una pérdida neuronal progresiva, la presencia de hiperpotasemia indica que ha
perdido al menos:
a) Un 25% del filtrado glomerular.
b) Un 35% del filtrado glomerular.
c) Un 50% del filtrado glomerular.
d) Un 75% del filtrado glomerular.
e) Un 100% del filtrado glomerular.

21) Un paciente de 35 años con insuficiencia renal crónica secundaria a pielonefritis crónica recibe un
trasplante renal de cadáver con el que compartía dos identidades en Ay B y una en DR. Recibe
tratamiento inmunosupresor con ciclosporina A y corticoides a dosis estándar. En el postoperatorio
inmediato se observa buena diuresis y no es necesario el tratamiento sustitutivo con hemodiálisis.
En el 5º día de evolución, el paciente presenta fiebre de 38º. TA de 180/ 110, oliguria y disminución
en la concentración urinaria de sodio. El diagnóstico más probable sería:
a) Crisis hipertensiva.
b) Infección respiratoria.
c) Pielonefritis aguda del injerto renal.
d) Recidiva de su enfermedad renal.
e) Rechazo agudo del injerto renal.

22) ¿Cuál de las siguientes medidas terapéuticas NO está indicada en el tratamiento de la


Hiperpotasemia?:
a) Glucosa e insulina.
b) Bicarbonato sódico.
c) Gluconato cálcico.
d) Resinas de intercambio iónico.
e) Glucocorticoides.

23) ¿Cuál de las siguientes nefropatías glomerulares no se relaciona con hipocomplementemia?:


a) Glomerulonefritis aguda postinfecciosa.
b) Glomerulonefritis membrano-proliferativa.
c) Nefropatía lúpica.
d) Nefropatía diabética.
e) Nefropatía de la crioglobulinemia.

24) 185. Un paciente de 28 años presenta hematuria macroscópica al día siguiente de una infección
faríngea y edemas maleolares. Dos años antes había presentado un cuadro similar que desapareció
con rapidez, por lo que no había consultado previamente. En la exploración física se objetiva una
T.A. de 180/110, una Creatinina Plasmática de 2 mgrs/dl e Hipocomplementemia. ¿Cuál de los
siguientes es el diagnóstico más probable?:

182
a) Glomerulonefritis membrano-proliferativa.
b) Glomerulonefritis post-estreptocócica aguda.
c) Glomerulonefritis rápidamente progresiva.
d) Glomerulonefritis por depósitos mesangiales de IgA (Enfermedad de Berger).
e) Hialinosis Segmentaria y Focal.

25) Niña de 3 años, que presenta afectación brusca del estado general dentro del contexto de un cuadro
febril con diarrea mucosanguinolenta. A la exploración aparece pálida y soñolienta, la auscultación
cardiopulmonar es normal, el abdomen doloroso y el resto del examen físico sólo revela la presencia
del petequias puntiformes diseminadas. La orina es hematúrica y se constata hipertensión arterial.
El hemograma muestra HB 7 g/dl, 17.000 leucocitos con neutrofilia, 37.000 plaquetas con
normalidad de las pruebas de coagulación. ¿Cuál es el diagnóstico más probable?:
a) Sepsis por Salmonella.
b) Púrpura de Schönlein-Henoch.
c) Glomerulonefritis aguda post-infecciosa.
d) Síndrome hemolítico-urémico.
e) Coagulación intravascular diseminada.

26) Se trata de un niño de 7 años, con retraso estaturo-ponderal, que presenta signos radiológicos de
raquitismo y ecográficos de nefrocalcinosis. Los análisis demuestran acidosis metabólica con PH
inferior a 7.32 y Bicarbonato plasmático inferior a 17 mEq/ litro. El PH de la orina en 3
determinaciones es siempre superior a 5.5. Otras alteraciones metabólicas consisten en
hipercalciuria, hipocitruria y discreta disminución de la resorción del fósforo. No se detecta
glucosuria ni aminoaciduria. De las siguientes posibilidades, señale el diagnóstico correcto:
a) Síndrome de Bartter.
b) Acidosis tubular distal (Tipo I).
c) Acidosis tubular proximal (Tipo II).
d) Síndrome completo de Fanconi*. Puede ser que sea un síndrome de fanconi por el raquitismo;
pero no es completo en su totalidad por falta de aminoaciduria no selectiva y glucosuria.
e) Enfermedad de Harnup.

27) 174. Paciente de 50 años de edad, que consulta por dolor en la fosa renal, polaquiuria, disuria y
hematuria. En el análisis de la orina se observa piuria y pH ácido con cultivos repetidamente
negativos. ¿Cuál sería la primera posibilidad diagnóstica, de entre las siguientes?:
a) Pielonefritis aguda.
b) Síndrome nefrítico.
c) Tuberculosis genitourinaria.
d) Prostatitis aguda.
e) Carcinoma renal de células claras.

183
28) La insuficiencia renal funcional de la cirrosis hepática es debida a:
a) lesión glomerular
b) lesión tubular
c) hipovolemia
d) compresión de la ascitis sobre los grandes vasos renales
e) vasoconstricción de las arterias renales
f) Todas las anteriores

29) Urémico crónico con potasio sérico de 6.5mEq/L con leve acidemia, sin trastorno
muscular y EKG normal, puede ser tratado con:
a) kayaxelate
b) gluconato de Calcio al 10%
c) NaHCO3
d) Diálisis

30) La insuficiencia renal crónica ocasiona hiperfosfatemia permanentemente cuando la tasa


de filtración glomerular esta por debajo de:
a) 60ml/min
b) 50 ml/min
c) 25 ml/min
d) 40 ml/min

31) La hipofosfatemia es causa de excepto


a) inotropismo positivo
b) disminución de 2,3 DPG
c) hemólisis
d) curva de HbO2 desplazada hacia la izquierda

32) La solución de NaCl 23,4% tiene de osmolaridad:


a) 8000 mosm/l
b) 800 mosm/l
c) 80 mosm/l
d) 0,8 mosm/l

33) En la hiperkalemia:
a) aumenta la sístole eléctrica
b) hay menor diferencia entre el potencial de reposo y el umbral
c) el potencial de reposo transmembrana se hace mas negativo
d) el potencial umbral se hace menos negativo

34) La velocidad de administración de potasio en caso de gran riesgo para la vida puede ser
de:
a) 10-20 mEq/L
b) 8 mEq/L
c) 110 mEq/L
d) 40 mEq/L

35) Un ejemplo de hiponatremia hipotónica normovolemica es:


a) síndrome nefrósico
b) SIADH
c) Diarrea
d) Cirrosis

36) A una embarazada con eclampsia se le ordena 1 litro de D/A 5% con 10g de MgSO4.
¿Cuántos ml de MgSO4 al 50% se han utilizado?
a. 20
b. 5
c. 12,5
d. 30

37) Causa hipomagnesemia:


a) estreñimiento
b) alcoholismo
c) deshidratación hipertónica
d) insuficiencia renal aguda

38) Paciente con diuresis de 300ml/día, K sérico 7mEq/L, Posm y Uosm 300mosm/L, el
diagnóstico más probable es:

184
a) insuficiencia suprarrenal
b) insuficiencia renal
c) SIADH
d) Deshidratación primaria

39) La mielinosis pontina se caracteriza por:


a) pares craneales intactos
b) cuadriplejía flácida
c) comportamiento normal
d) hipertonía de los miembros inferiores

40) Calcule el déficit de agua en una paciente con sodio sérico 168mEq/L y que pesa 70Kg
(utilice como porcentaje de agua 50% y sodio normal mEq/L):
a) 5L
b) 3L
[𝑁𝑎+]𝑝𝑙𝑎𝑠𝑚𝑎−140
c) 10L 𝑑𝑒𝑓𝑖𝑐𝑖𝑡 𝑑𝑒 𝑎𝑔𝑢𝑎 = 140
× 𝐴𝐶𝑇
d) 7L

41) En la SIADH la concentración de sodio urinario es de:


a) menos de 10mEq/L
b) 20mEq/L
c) 15mEq/L
d) Mas de 40mEq/L

42) En paciente con vómitos abundantes y prolongados con hipokalemia severa debe
administrarse:
a) NaCl 0.9% con KHCO3
b) NaCl 0.9% con KCl
c) NaCl 0.9% con gluconato de potasio
d) NaCl 0.9% con NaHCO3

43) La hipokalemia en un paciente con cirrosis puede contribuir:


a) coma
b) alcalosis respiratoria
c) acidosis metabolica
d) aumento de la síntesis de urea

44) El plasma de un paciente contiene Na 125mEq/L, glucosa 108mg/dl, urea 300mg/dl,


podrá presentar:
a) cirrosis
b) síndrome nefrósico
c) síntomas de hipertonicidad
d) síntomas de hipotonicidad

45) La hiponatremia aguda sintomática es debida a, excepto:


a) exceso de glucocorticoides
b) SIADH
c) post-operatorio
d) intoxicación hídrica

46) La administración de solución salina 0.9% produce:


a) disminución del volumen intracelular y aumento del extracelular
b) aumento del volumen intra y extracelular
c) aumento del volumen extracelular
d) aumento del volumen intracelular

47) Alcalosis metabólica con hipertensión arterial, hipokalemia, aldosterona normal y renina
normal:
a) Hipertensión renovascular
b) Hipertensión maligna
c) Sindroma de Cushing
d) Administración de licor

48) La hipokalemia puede predisponer, excepto:


a) intoxicación digitálica
b) disminución de la amoniogénesis
c) rabdomiolisis

185
d) ondas U en el EKG

49) Cuál de las siguientes medicinas pueden dar hiperkalemia


a) inhibidor de la enzima convertidota
b) bloqueador alfa adrenérgico
c) bloqueador beta adrenérgico
d) a y b
e) a y c

50) Hormonas que disminuyen en la falla renal crónica, excepto


a) 1,25 dihidroxi vitatamina D
b) eritropoyetina
c) paratohormona
d) foliculo estimulante

51) Factores que conducen a la progresión de falla renal crónica


a) hiperlipidemia
b) toxinas urémicas
c) proteinuria
d) b y c
e) todas

52) Paciente de 20 años presenta abundantes vômitos y diarrea de 3 días de evolución,


examen físico depleción de volumen, suero muestra Na 155, K 3meq/l, Cl 117meq/l, HCO3
25 meq/l, tratamiento de elección:
a) D/A 5% + KCl
b) solución salina 3% + KCl
c) solución salina 23.4% + KCl
d) solución salina 0.45% + KCl

53) En la enfermedad ateroembólica renal se da lo siguiente excepto:


a) Velocidad de eritrosedimentación

54) Pseudohiperkalemia de 8 mEq/l que se ve en el EKG:


a) Onda T picuda
b) Aplanamiento de la onda P
c) Ay B
d) Ninguna

55) 8)Criterios para iniciar diálisis excepto:


a) Alcalosiss Metabolica

56) 9)Hipokalemia con normotension excepto


a) Síndrome de liddle

57) Plasama creatinina 8mg/dl N de urea 100mg/dl U na 40 Meq/l :


a) Insuficiência renal

58) Factores reversibles responsables de deterioro de la funcion renal excepto:


a) Fibrosis intersticial

59) Sugiere trastorno túbulo intersticial:


a) Proteinuria mayor a 3.5 g /24 horas
b) Notrmotensión
c) Edema
d) Osmolaridad urinaria normal

60) Glomérulonefritis rápidamente progresiva


a) Engrosamiento de la membrana basal
b) Fusión de los pedículos de los podocitos
c) prolñiferacion intracapilar
d) proliferación extracapilar

61) 16.Plasma Na:115, Posm240 mOsm,Uosm 680,Una 60:meq/l:


a) .Deshidratación hipotónica

186
b) .secreción inapropiada de ADH
c) .Deshidratación isotónica
d) .Deshidratación primaria

62) No produce hipokalemia:


a) Pancreatitis aguda
b) rabdomiolisis
c) neoplasia
d) Def. Vit D

63) Paciente con glucosa 90mg/dl, Na+= 115mEq/L, Urea en 140mg/dl


a) Síndrome hipoosmolar
b) Síndrome hiperosmolar
c) Diarrea
d) Ninguna de las anteriores.

64) Causa de seudohiponatremia con osmolaridad elevada


a) Hiperproteinemia
b) Diabetes mellitus
c) Hiperlipidemidemia
d) Reserción transuretral prostática.

65) 21. Manifestaciones de hipermagnesemia excepto


a) Normotensión

66) 22. Piel en uremia crónica excepto


a) Paño blanco

67) 23. No es causa de necrosis papilar aguda


a) .Diabetes mellitus
b) . Uropatía obstructiva
c) . Pielonefritis
d) Trombosis de venas renal.

68) 25. Compromete riñones y pulmones:


a) Enf de GoodPasture
b) Granulomatosis de Wegener
c) LES
d) Ay C

69) 26. Sugiere glomerulopatia:


a) -anasarca

70) 27. TFG < 15ml/min corresponde al estadio de la enfermedad renal crónica:
i) 5

71) 28. Causa obstetrica de necrosis cortical bilateral


a) - desprendimiento prematuro de placenta

72) 29. Promueve amoniogenesis en cirróticos


a) - hipokalemia

73) Criterio para diagnostico de síndrome nefrosico


a) Proteinuria en 24 hr de 3.5 gr para superficie corporal de 1. 73
b) Albumina serica < 3g/dl
c) Hiperlipidemia y edema
d) Todas las anteriores
e) ay b

74) La hipermagnesemia puede producir:


i) R/. Paro respiratorio

75) Tratamiento de hiprecalcemia:


i) R/. SSN 0.9%

187
76) La hipofosfatemia son causas, excepto:
a) Aumento de la PTH
b) Aumento de la absorción intestinal de fosfato
c) Disminuye la fosfatemia
d) Aumento de la vitamina D

77) 14. Paciente crónico con deshidratación severa puede presentar lo siguiente excepto:
a) Densidad urinaria 1010
b) Osmolaridad urinaria de 800 mOsm/kg
c) K sérico aumentado
d) Oliguria

78) Paciente con vómito abundante, hipertensión arterial, sodio urinario de 6, oligúrico,
osmolaridad de 800 presenta:
a) R/.Insuficiencia suprarrenal

79) Paciente urémico crónico con vómitos severos, hipotensión arterial, orina al azar muestra
Na mas de 40, osmolaridad de 800 puede tener:
a) Falla suprarrenal

80) Cual de las siguientes es causa de hiperfosfatemia:


a) R/. Insuficiencia renal

81) Paciente con deshidratación hipertónica e hipertensión:


a) R/. SSN 0.9%

82) En la Glomerulonefritis Aguda post-infecciosa disminuye la tasa de filtración:


a) R/. Por alteración en el coeficiente de ultrafiltración

83) Un ejemplo de hipokalemia con aldosterona alta y osmolaridad baja es:


a) Hiperaldosteronismo primario

84) Manifestaciones cardíacas. El problema médico más grave de la hiperkalemia es la


cardiotoxicidad. Los cambios en el ECG producidos por los niveles altos de potasio son
bastante constantes. A medida que aumentan los niveles se aprecian los siguientes cambios:
a) Ondas T picudas (con intervalo QT normal o ligeramente reducido)
b) Prolongación del intervalo PR con depresión de ST
c) Desaparición progresiva de la onda P
d) Bloqueo cardíaco progresivo
e) Arritmias ventriculares
f) Paro cardíaco .

85) La hipokalemia produce:


a) mayor negatividad del potencial de reposo.

86) Exceso del kayaxelate en el tratamiento de hiperkalemia puede producir:


a) constipación.

87) Hipofosfatemia:
a) aumenta el calcitriol.

88) Consecuencia metabólica de hipofosfatemia severa:


a) miocardiopatía

89) Causa de hiperfosfatemia:


a) insuficiencia renal

90) El Síndrome Nefrósicose caracteriza por:


a) proteinuria mayor o igual de 3g y albúmina sérica menor o igual de 3g/dl.

91) En el Síndrome Nefrósico pueden encontrarse:

188
a) cilindros céreos

92) Urémico agudo, oligúrico, acidótico, deshidratado puede presentar:


a) osmolaridad urinaria de 250mosm/kg

93) La falla renal crónica avanzada cursa con:


a) brecha aniónica elevada

94) Uno de los indices de falla renal aguda es:


a) Uosm/Tosm

95) Hormona producida en los riñones que disminuye en la falla renal cronica:
a) ADH
b) Noradrenalina
c) Peptido natriuretico (segun arjona)
d) 1,25 (OH)2 Vit D.

96) Las calcificaciones metastásicas se deben:??


a) Aumento del producto Ca, P
b) Aumento del producto Ca, Mg
c) Hipoparatiroidismo
d) Hiperparatiroidismo 1ario

97) Causa de anuria:


i) Pielonefritis unilateral
ii) Necrosis tubular aguda
iii) Enf cambios minimo
iv) Necrosis cortical

98) la etapa de la insuficiencia renal cronica con tasa de filtración glomerular de 60-89 ml/min es
la:??
i) 1
ii) 2
iii) 3
iv) 4

99) La FENA es la relación entre:


i) Oferta tubular/Na reabsorbido x 100
ii) depuración de Na/ depuración de creatinina x 100
iii) oferta tubular/ Na excretado x 100.
iv) Na excretado/ Na reabsorvido x 100.

100) En la nefritis tubular intersticial aguda se pierde la capacidad de poder concentrar la orina
debida a:??
i) Daño medular
ii) Daño glomerular
iii) Daño tubular proximal
iv) Daño de la porcion cortical del tubulo colector

101) El efecto biologico mas importante de la calcitonina es, excepto:


i) Responder a la hipocalcemia
ii) Aumenta la calciuria
iii) Estimular la resorcion osea osteoclastica
iv) Utilizada en el tx de la hipercalcemia.

102) Favorece la entrada de K al espacio intracelular, excepto:


i) Agonista adrenergico
ii) Estimulacion adrenergico
iii) Insulina
iv) Alcalosis metabolica

103) Factores que afectan el balance hídrico en el post-operatorio, excepto:


i) Fármacos
ii) Calcitriol
iii) Administración de liquidos hipotonico
iv) Aumento de la ADH

189
104) Las primeras manifestaciones clínicas de la hiponatremia son:??
i) neurológica
ii) gastrointestinal
iii) pulmonares
iv) musculares

105) En la hipernatremia hipertonica se da lo siguiente excepto:


i) Entrada de Na
ii) Salida de aa
iii) Ganancia de iones
iv) Disminución del agua cerebral

106) Manifestaciones clinicas de la hipercalcemia, excepto:


i) Taquicardia
ii) HTA
iii) Constipación
iv) Vomitos

107) En falla renal aguda por nefrotoxicidad:


i) Se conserva la membrana basal
ii) Se destruye la membrana basal
iii) Frecuente hay oliguria
iv) La creatinina no aumenta

108) Causa de hipomagnesemia:


i) alcoholismo

109) Na serico de 168 mEq/l 70 kg (utilize como % de agua al 50% y Na normal 140 mEq/l
i) 7 litros

110) Indicaciones de diálisis en falla renal aguda, excepto:


i) Encefalopatia uremica
ii) Hiperkalemia refractaria
iii) Ac. Metabolica refractaria
1. d. hipocalcemia

111) El KCL IV se puede asministrar asi, excepto:


a) a.infusion por linea periferica de 10 meq en 100 ml en 1hora
b) b. “ “ central de 20 meq em 50ml en 1 hora
c) c. “ “ central de 100 meq em 50ml en 1 hora
d) d. “ “ periferica de 20 meq en 20ml en 2 horas

112) Aumenta la excrecion urinaria de K, excepto:


a) Triamtereno
b) Bumetanida
c) Furosemida
d) Metolazona

113) Factores que regulan la secrecion distal de K, excepto:


a) Flujo tubular distal y aporte distal de Na
b) Excrecion de aniones no reabsorbibles
c) Aldosterona
d) PTH

114) Enfermo de 60 Kg con 60% de agua corporal, Posm 260 mOsm/kg, ¿Cuántos gramos de
NaCl aumentaria el Na en 10 mEq?
a. 21 gramos

b) 50. Conteniendo 10 g de MgSO4 al %, cuantos ml de sal necesitaria?


i) 20 ml

115) Causa de hiperfosfatemia:


116) Falla hepatica
117) Falla renal cronica
118) Falla cardiaca
119) Insuficiencia suprarrenal

190
120) Causas de hiperfosfatemia grave, excepto:
121) recuperacion de cetoacidosis diabetica.
122) Alcalosis respiratoria
123) Acidosis respiratoria.

124) Paciente con vómitos severos, oligurico con hipotensión arterial Una> 40…… 800
mOsm/Kg puede tener:
125) Insuciencia renal aguda.
126) Insuficiencia suprarrenal
127) Falla pre-renal
128) Exceso de mineralocorticoides

129) Paciente urémico crónico con deshidratación severa puede presentar lo siguiente excepto:
a) Densidad urinaria 1010.
b) Osmolaridad urinaria de 800 mOsm/Kg
c) K sérico aumentado
d) Oliguria.

130) La hipokalemia produce:


a. Aumento de la magnitud del potencial de reposo
131) 17. La PTH promueve todo lo siguiente excepto:
132) a aumento de la absorción intestinal de calcio
133) b- aumento de la absorción tubulár de fosfato..
134) c. aumento de la resorción ósea de fosfato
135) d- aumento de la resorción osea de calcio
136) e, disminución de la reabsorción de fosfato

137) 18. Un varon de 57 esta con hemodiálisis de mantenimiento para insuficiencia renal
cronica, ¿Cuál de las sig anomalías metabolicas podría ser anticipáda?
a. hipernatremia
b. hiponatremia
138) c.
139) d. exceso de vitamina D
140) e. hipoparatiroidismo

141) 19. Un px de 25años fue ingresado a UCI por lesiones graves en cabeza y fractura de la
base del Graneo, aprox X horas después de la lesion manifestaba poliuria, la OSMU 150, y la
de suero 350. Los líquidos IV fueron detenidos y 3 hors después la producción de orina y la
osm urinaria permanecieron sin cambio. Se administraron 5 unidades de vasopresina IV, la
osm de la orina se
142) incremento a 300. ¿Cuál es el dx mas probable?
(1) Diabetes insípida central
143) b. diabetes insípida nefrogenica
144) c. intoxicación acuosa
145) d. sobrecarga de solutos
146) e. S IADH

147) 55. indica falla renal aguda


i) U..N.. = 4.1
ii) UN = 1
148) c. U..N.. = 1/4i/
149) d. Gravedad especifica 1.026
150) e. Ninguna

151) 56. deshidratación por perdida de agua y densidad urinaria 1.003, creatinina 6 /
a. falla renal aguda parenquimatosa
ii) falla prerenal
a. falla renal obstructiva
b. falla renal cronica
c. no hay falla renal

152) 57. uno de los indices de falla renales


a. reabsorción de Na/oferta tubular de Na x 100
b. excresion urinaria deNa/oferta tubular de x 100
153) c.- oferta tubular de Na/excrecion urinaria de Na x 100 ~.--~
i) Una/Pna x 100

191
154) e. Ninguna

155) 58. el Sdr hepatorenal es causa de falla:


i) prerrenal
156) b. renal
157) c. póstrenal
158) d. falla cardiaca
159) e. daño tubular

160) 59. causa mas común de falla renal crónica:


(1) DM

161) 60- en la glomendonefritis aguda por infeccion, la causa, excepto


a) proteina mayor de 10 g
b) osmolaridad urinaria normal
162) c. hematuria
163 ) d. H T A
164 ) e. Edema

165) 61. falla renal cronica, excepto


a. PTH aumentado
b) Hipofosfatemia
c) Acidemia
166) d. Hipocalcemia
167) e. 1.25 OH (D3) disminuida

168) 62. EN la falla renal aguda siempre habra


a) Aumento de creatinina
b) fraccion de excrecion de Na aumentado
c) hiperkalemia
d) Una aumentado
e) Hipernatremia

169) 63. causa de falla prerenal


a. Sdr nefrotico
ii) obstrucción de ambos uréteres
iii) falla cardiaca
170) d. Grlomerulonefritis aguda post-infeciosa
171) e. Calculo renal

172) 64. en la nefritis tubular interticial se puede dar, excepto


a. h.ipernatremia
b. proteinuria de 1.5 g
c. normotension arterial
d. hipostenuria
e. hipocalcemia

173) 65. Sugiere glomerulonefritis:


a. leucocituia
174) b-cilindros eritrocitarios
i) anuria
175) d. -

176) 66 -causa de falla renal parenquimatosa


177) papilitis
178) calculo en pelvis derecha
179) estenosis unilateral de arteria renal
180) cistitis hemorragica

181) 2--En el riñón la hormona paratiroides:


182) Aumenta la reabsorción tubular de calcio y disminuye la de fósforo

183) 3- La hipercalcemia produce


184) Potencialización de los efectos de la hipercalemia

185) 36- La hipomagnesemia produce.

192
a) Vasodilatación periférica

186) 36-De los factores: nefrotoxicidad./ -La buena hidratación

187) 37 - En la concentración sérica para calcular:: 8,8

188) 38- la velocidad de administración de potasio: 40 meq

189) 39-hipocalcemia produce : fosfaturia

190) 40-La causa de hipertensión con hipocalemia: Síndrome de Bartter=:,

191) 41-En paciente con vomito severo hipotensión arterial o ir a la sar con mas de 40 meq
192) Insuficiencia suprarrenal;

193) 42-La ,administración de sol isotónica aumento de vol extracelulares

194) 43-Glomerulonefritis post estreptococcica Altera el cociente de ultrafiltración

195) 44-En-el túbulo` intersticial se puede dar: La proteinuria de 4 g al día.

196) 25. Cual de las siguientes opciones esta contraindicada en el tratamiento de la


hiperkalcmia
197) a, gluconato de calcio
198) b. aldactone
199) c, kayaxelate
200) d. Bicarbonato de sodio e. Todas las anteriores

201) 43. Rotando en sala de medicina interna le piden que evalue un paciente con HIV el cual
porpresentar una meningitis por criptococci recibe anfotericina B; dicho este
medicamento buscaría como complicación:
a. aparicion de diarrea
b. aumento de la presion arterial
(2) hipokalemia
202) d . cefalea
203) e. dolor toráxico

204) 81- donde se halla el calcio en el organismo:


a) 50% sangre
b) 20 % proteínas
c) 99% hueso

205) aumento de PTH


(1) aumento de la absorción intestinal de fosfato
(2) disminuye fosfatemia
(3) aumento de vitamina D

206) 9- una solucion isotonica


a) aumenta el volumen extracell

207) 10- la vel de administración de potasio en caso de riesgo para la vida


208) a—40 meq por hora

209) 11- Px cronico con deshidratación severa puede presentar los siguiente
excepto:
210) a-osmolaridad urinaria de 800 mosmol/Kg

211) 12- Px con vomito abundante, HTA, sodio urinario en 6, oligurico, Osmolaridad
de 800 presente,
a) insuficiencia suprarrenal

212) 13- Px uremico cronico con vomito severo hipotension arterial orina al azar,
muestra sodio mas de 40, Osmolaridad de 800 puede tener :
a) Falla suprarrenal

213) Cuándo disminuye el fosfato?


214) R. Aumento de PTH

193
215) Ejemplo de hipernatremia hipervolémica:
216) R. Aldosteronismo primario

217) La hipocalemia produce:


218) R. Aumento de la magnitud del potencial de reposo

219) La depleción de agua produce, excepto:


220) R. Salida de electrolitos del cerebro

221) Velocidad de administración de KCl en caso de urgencia:


222) R. 40 mEq/h

223) Cambios en EKG de la hipermagnesemia es igual:


224) R. Hipercalemia

225) Enfermo de 60 Kg con 60% de agua corporal, osmolaridad plasmática 260 mosmol/Kg,
¿cuántos gramos de NaCl aumenta el Na en 10 mEq en 24 horas?
226) R. a) 15
a) 21
b) 30
c) 19

227) Uso de Calcio en tratamiento de hipercalemia para:


228) R. Modificar el Potencial de Membrana

229) Hipermagnesemia severa produce:


230) R. Paro respiratorio

231) Venoclisis de 1L de D/A 5% conteniendo 10 mg de MgSO4 y al 50%:


232) R. 20mL

233) La hipocalcemia:
234) R. Prolongo QT a expensas del segmento ST

235) Tratamiento de urgencia de hipercalcemia:


236) R. SSN 0.9%

237) Causa de hiperfosfatemia:


238) R. Insuficiencia renal

239) Adaptación cerebral, hipertonicidad, excepto:


240) R. Pérdida de iones

241) En la adaptación cerebral, la hipotensión da lo siguiente, excepto:


242) R. Ganancia de aminoácidos
243) En ausencia de síntomas de hipernatremia, el sodio debe ser disminuido a razón de:
244) R. 0.5 mEq/L/h

245) Para corregir un sodio cerca de 185 mEq se administró en 86 horas, 66 letras de D/SS 5%.
¿cuál es la velocidad de administración de la venoclisis?
246) R. 69 mL ¿?????

247) Manifestación clínica de hipercalcemia, excepto:


248) R. a) Hipotensión arterial
a) b) Bradicardia
b) Nauseas
c) Anorexia

249) Consecuencia de hipofosfatemia, excepto:


250) R. a) No afecta al SNC
a) b) Disfunción plaquetaria
b) Cardiomiopatía
c) Osteomalacia

251) Causa de hipofosfatemia, excepto:


252) R. a) Falla renal

194
a) b) Recuperación de cetoacidosis diabética
b) Alcalosis respiratoria
c) Quemaduras graves

253) Causa de HTA con hipocalemia, aldosterona-renina baja:


254) R. Síndrome de Cushing

255) Causas de HTA con hipocalemia y renina alta:


256) R. Hipertensión maligna

257) En una deshidratación severa hipertónica con hipotensión arterial severa, la prioridad inicial
es administrar:
258) R. SSN 0.9%

259) En la secreción inapropiada de ADH se da lo siguiente, excepto:


260) R. No hay expansión de volumen

261) Tejido con mayor contenido de K+:


262) R. a) Hígado
a) b) Músculo
b) Eritrocito
c) Hueso
263) La insuficiencia renal
funcional de la cirrosis hepática es debida a:
264) R. Vasoconstricción de
las arterias renales

265) 23. Px con IRC tine una taza de filtración glomerular de menor de 25 ml/min ;aunque
hay yuna tabla de las etapas en esta enfemedad crónica.
266) Estas osn las TFG:
1. 90 en riesgo
2. 20 ( con TFG normal o incrementada)
3. 60 -89
4. 30 – 59
5. 15-29
b) Menor a 15 diálisis.

267) 24. Factor que altera el balance hídrico en el periódo post- operatorio:
268) Disminución de la ADH (Excepto)

269) 25. Un ejemplo de hipokalemia con aldosterona alta y osmolaridad baja es:
270) Hiperaldosteronismo primario

271) 26.Ejemplode hipernatremia hipervolémica: Hiperaldosteronismo primario

272) 27.Hiperkalemioa son alteraciones del EKG no es necesario administrar gluconato de


calcio se colocará un antagonista de K asíe l Px no cae en paro y estoes si tiene
manifestación en el EKG.

273) 28. La fracción de excreción de sodio estará aumentada en falla cardíaca.

274) 29.En la hiperkalemia: hay menos diferencia entre el potencial de reposo y el umbral.
275) Manifestaciones cardíacas. El problema médico más grave de la hiperkalemia es la
cardiotoxicidad. Los cambios en el ECG producidos por los niveles altos de potasio son
bastante constantes. A medida que aumentan los niveles se aprecian los siguientes cambios:
276) Ondas T picudas (con intervalo QT normal o ligeramente reducido)
277) Prolongación del intervalo PR con depresión de ST
278) Desaparición progresiva de la onda P
279) Bloqueo cardíaco progresivo
280) Arritmias ventriculares
281) Paro cardíaco
282) Las ondas T picudas constituyen el dato en el ECG más constante en la hiperkalemia.
283) Efectos neuromusculares. El primer signo neuromuscular de la hiperkalemia suele ser la
aparición de parestesias seguidas de debilidad progresiva de varios grupos musculares. Si el
cuadro se agrava se observa cuadriplejia fláccida. Las funciones cerebrales y de los pares

195
craneanos se conservan y la parálisis de la musculatura respiratoria puede ocurrir, pero es
excepcional.
284) 30. La infusión de potasio sin riesgo para la vida del PX: es de 40 meq; porque
285) I lb de solución salina: debe llevr de 20 a40 mEq de KCl a una velocidad de 10 a 20 mEq
por hora; si aumenta la velocidad puede provocar un bloqueo A-V. Se recomienda una
Velocidad mayor si la hipokalemia es muy severa.
286) Sis e usa venas priféricas es:
287) 10 meq en 100 cc de sol.salina para pasar en una hora
288) 20 meq en 200 cc de sol.salina para pasar en una hora
289) En vena central:
290) 20 meql en 50 cc pasarla en una hora
291) 40 meq en 100cc pasar la en una hora
292) 31.Los indicios de falla renal aguda son:
293) (Na filtrado/Na excretado) x 100%
294) 32.Na urinario menor de 10: falla cardíaca, presentará edema estará hinchado y lo puede
llevar a una I.C.C. ( Esto lo saque de Medline: Los valores normales generalmente son de 15 a
250 mEq/L/día, dependiendo del estado de deshidratación y la ingesta diaria de sodio en la
dieta. Los rangos de los valores normales pueden variar ligeramente entre diferentes
laboratorios. Nota: mEq/L/día = miliequivalentes por litro por día.)
295) 33.Causa de enuresis: necrosis cortical difusa.
296) 34.En SIDH la concentración urinaria es menor de 20.
297) 35. Deshidratación pura de agua excepto: hay salida de electrolitos del cerebro.
298) 36.Urémico crónico con deshidratación hipertónica severa y acidótico presentará lo
siguiento excepto: osmolaridad urinaria de 8mm mosmol/kg.
299) 37. Px con vómitos abundante, hipertensión arterial; Na urinario=6, oliguria, osmolaridad
de 800 presenta: insuficiencia suprarrenal.
300) 38. Factor que predispone a la hipernatremia son intoxicación hídrica.
301) 39. Se presenta en acidosis tubular renal proximal excepto: NaHCO3 y lo alfa antagonistas.

1. En la Hiperkalemia:
ii) R/. Hay una menor diferencia entre el potencial de reposo y de umbral

302) 2. La hipokalemia en un cirrotico puede desencadenar:


i) R/. Coma Hepático

303) 3. La hipermagnesemia puede producir:


i) R/. Paro respiratorio

304) 7. Tratamiento de hiprecalcemia:


i) R/. SSN 0.9%

305) 8. La hipofosfatemia son causas, excepto:


a. Aumento de la PTH
b. Aumento de la absorción intestinal de fosfato
c. Disminuye la fosfatemia
d. Aumento de la vitamina D

306) 9. Una solución isotónica:


i) R/. Aumenta el volumen extracelular

307) Paciente crónico con deshidratación severa puede presentar lo siguiente excepto:
a) Densidad urinaria 1010
b) Osmolaridad urinaria de 800 mOsm/kg
c) K sérico aumentado
d) Oliguria

308) 15. Paciente con vómito abundante, hipertensión arterial, sodio urinario de 6,
oligúrico, osmolaridad de 800 presenta:
i) R/.Insuficiencia suprarrenal
309) Paciente urémico crónico con vómitos severos, hipotensión arterial, orina al azar
muestra Na mas de 40, osmolaridad de 800 puede tener:
310) Falla suprarrenal

311) 16. Todo lo siguiente es cierto acerca de Colitis Ulcerosa, EXCEPTO:

196
i) R/. No está en relación las alteraciones inmunológicas.

312) 17. En quien predomina la enfermedad de Hígado Graso:


i) R/. Sexo masculino

313) 18. Paciente con Insuficiencia renal crónica, con K sérico de 6, leve academia, EKG
normal, se trata con:
i) R/. Kayetzalate

314) 22. La hopikalemia predispone, excepto:


i) R/. Amoniogénesis

315) 25. Cual de las siguientes es causa de hiperfosfatemia:


i) R/. Insuficiencia renal

316) 26. Paciente con deshidratación hipertónica e hipertensión:


i) R/. SSN 0.9%

317) 29. En la Glomerulonefritis Aguda post-infecciosa disminuye la tasa de filtración:


i) R/. Por alteración en el coeficiente de ultrafiltración

318) 30. Causa de Insuficiencia preerenal, excepto:


i) R/. Aneurisma Disecante de la Aorta

(a) El magnesio está presente o


aumentado en:
ii) R/. Eritrocito

319) Causa de hipomagnesemia:


320) estreñimiento
321) alcoholismo
322) falla pre-renal
323) IRA (insuficiencia renal aguda)

324) La hipocalcemia en la falla renal aguda puede ser asintomática por la:
325) acidemia
326) hipoalbuminemia
327) Deficiencia de PTH
328) Alcalemia

329) La hipofosfatemia causa, excepto:


330) Aumento de vit. D
331) Aumento de PTH
332) Disminución de la excresión de fosfato
333) Aumento de la absorción de fosfato a nivel intestinal

334) Un pte con diuresis de 250 ml al día con osmolaridad plasmática (presión osmótia) y
osmolaridad urinaria de 300 mosm/l, el dx probable e:
335) Insuficiencia suprarrenal
336) Insuficiencia renal
337) Nefritis túbulo intersticial crónica
338) Deshidratación hipertónica

339) Pte con oliguria, hiperkalemia, hiponatremia, creatinina 0.8 mg/dl y CPK normal: Esto
puede corresponder a:
340) IRA
341) Insuficiencia suprarrenal
342) Síndrome hepatorrenal
343) Rabdomiolisis

344) No está indicado en caso de hiperkalemia:


345) Aldactone

346) Mujer con hiperemesis grávida:


347) Alcalosis hipoclórica

348) En causa de hipomagnesemia:

197
349) Alcoholismo.

350) En el metabolismo del calcio está implicado, menos:


351) Insulina

352) Hiponatremia, hipotónico, normovolémico:


353) SIADH

354) Fórmula del clearence de creatinina:


355) clearence = 140 – edad x peso en Kg
a. 72 x creatinina

356) Su infección produce litiasis renal:


357) proteus sp.

358) Fórmula de irritabilidad muscular:


359) IR = Na + K + OH
(a) ( ) + Mg + H

360) Con respecto al síndrome diarreico, nombre 2 tipos osmóticos:


361) Laxantes osmóticos (Mg2+, PO4, SO4)
362)
Carbohidratos no absorbibles (sorbitol, lactulosa).

363) Nombre dos síndromes diarréicos tipo secretora:


364) Consumo crónico de etanol.
365)
Obstrucción intestinal incompleta.

366) Nombre 2 sindromes diarréicos motores con aumento de motilidad:


367) Hipertiroidismo
368)
Prostaglandinas.

369) Indica falla renal aguda:


370) U.N. = 4.1
371)
U.N. = 1
372)
U.N. = ¼
373)
Gravedad específica: 1.026
374)
Ninguna.

375) Deshidratación por pérdida de agua y densidad urinaria 1.003, creatinina 6:


376) falla renal aguda parenquimatosa
377)
Falla pre-renal
378)
Falla renal obstructiva
379)
Falla renal crónica
380)
No hay falla renal.

381) Uno de los índices de falla renal es:


382) Reabsorción de Na/ oferta tubular de Na x 100
383)
Excreción urinaria de Na/ oferta tubular de Na x 100
384)
Oferta tubular de Na/ excresión urinaria de Na x 100 **

385)
Una / pna x 100
386)
Ninguna

387) El síndrome heatorrenal es causa de falla:


388) pre-renal
389)
renal
390)
posrenal
391)
falla cardiaca
392)
daño tubular

393) Causa más comun de falla renal cronica:


394) Diabetes melitus
395)
HTA
396)
Glomerulonefritis

198
397)
Enfermedad poliquística renal
398)
Vasculitis

a) 36. En la glomerulnefritis aguda por infección, la causa, excepto:


a. proteínas mayor de 10g
b) b. osmolaridad urinaria normal
c) c. hematuria
d) d. HTA
e) e. Edema

1. Paciente que pesa 50Kg, con sodio sérico de 160 mEq/dL , con ACT =40%
, su déficit de agua es: ( 50 x 0.4 = 20/5 = 4, creo que es asi pero esta no
fue la formula q el nos dio)
a) 5L b) 3L c) 4L d) 6L
2. Para un calcio total sérico de 7mg/dl y albúmina de 2,2 g/dl ¿Cuál será el
calcio total corregido? Tome el valor de la albúmina normal de 4 g/dl
a) 7,44 mg/dl b) 8,44 mg/dl c)9,44 mg/dl d) 8,90 mg/dl
3. Velocidad de administración de KCl en condiciones normales, en
mEq/hora
a) 8 b) 120 c) 20 d) 40
4. Causa falla prerrenal
a)Obstrucción de ambos uréteres b)Falla cardiaca c) Glomérulonefritis
aguda d) Síndrome nefrósico
5. Manifestación en el EKG de un paciente con Hipokalemia:
a) Onda T picuda b) Supradesnivel del segmento ST
c) Intervalo QT deprimido d) Onda S profunda
e) Prolongación del intervalo PR f) N/A
6. Cuál es la concentración sérica de fósforo por encima de la cual se
considera que una persona tiene Hiperfosfatemia (en mg/dL): (2.7 a
4.5)
a) 3.0 b) 4.0 c) 5.0 d) 6.0 e) 7.0
7.. Diurético de uso común en el tratamiento del edema en los
síndromes nefrítico y nefrótico:
a) Indapamida b) Espironolactona c) Furosemida d)
Acetazolamida
e) Hidroclorotiazida f) N/A
8. Cuál es el principal tratamiento para pacientes con nefritis
tubulointersticial alérgica: en realidad el principal tx es suspender el fármaco
causante asi que puede ser la e… pero se usa prednisona también, y diálisis en
severidad
a) Prednisona b) Enalapril c) Ibuprofeno d) Diálisis e) N/A
9. Cuál de las siguientes enfermedades puede producir un Síndrome
Nefrítico mediado por inmunocomplejos
a) Granulomatosis de Wegener b) Síndrome de Goodpasture
c) Púrpura Trombocitopénica Trombótica d) Amiloidosis e)
N/A
10. Ejemplo de hiponatremia hipotónica normovolémica
a)Diarrea B)Cirrosis hepática c) Síndrome nefrósico d) SIADH
11. Mencione cuál de las siguientes condiciones puede ser causa de
Hiperkalemia:
a) Uso de IECA b) Hiperaldosteronismo primario c)
Hiperinsulinemia d) Alcalosis Metabólica e) N/A

199
12. Mencione cuál de las siguientes condiciones cursa con el desarrollo
de Enfermedad Crónica de los Riñones:
a) Nefrolitiasis unilateral b) Enfermedad de Cambios Mínimos
c) Nefropatía Diabética d) Carcinoma de Riñón e) N/A
13. Oliguria implica una diuresis menor de (mL/día):
a) 50 b) 400 c) 800 d) 1000 e) N/A
14. Señale la aseveración Incorrecta acerca de la Hormona
Antidiurética:
a) Polipéptido sintetizado en el hipotálamo si
b) También se le conoce como Arginina Vasopresina si
c) El efecto neto de su acción a nivel de la nefrona es la
reabsorción
pasiva de agua y sodio (creo xq es reabs de solo H20)
d) El mayor estímulo para su secreción es la hipertonicidad si
15. Generalmente la proporción BUN / creatinina plasmática en
hiperaazoemia
prerrenal es de: NO SE
a) <10 b) > 20 c) 100% d) d) 1%
16. El sodio urinario en hiperazoemia renal intrínseca es de:
a) <10 b) >20 c) > 30 d) >1
17. La causa principal del síndrome nefrítico agudo es :
a) Hipertensión arterial b) GN postestreptocóccica c) Síndrome
deGoodpasture
d) Nefritis lúpica
18. Cuál es el componente esencial en el síndrome nefrótico?.
a) hipoalbuminemia b) hiperlipidemia c) proteinuria d) edema
19. Cuadro clínico clásico del síndrome nefrítico incluye lo siguiente
excepto:
a) lipiduria b)hipertensión c) Oliguria d) Hematuria
20. El tx en cuarto de urgencias de hipercalcemia intensa es:
a) Bifosfonatos b) Furosemida y SSisotónica c) Furosemida y
Bifosfonatos
d) SSisotónica +Calcitriol.

Calcule el déficit de agua en una paciente con sodio sérico 168mEq/L y que pesa 70Kg
(utilice como porcentaje de agua 50% y sodio normal mEq/L):
q. 5L
r. 3L
s. 10L
t. 7L

En la SIADH la concentración de sodio urinario es de:


q. menos de 10mEq/L
r. 20mEq/L
s. 15mEq/L
t. Mas de 40mEq/L

En paciente con vómitos abundantes y prolongados con hipokalemia severa debe


administrarse:
q. NaCl 0.9% con KHCO3
r. NaCl 0.9% con KCl
s. NaCl 0.9% con gluconato de potasio
t. NaCl 0.9% con NaHCO3

La hipokalemia en un paciente con cirrosis puede contribuir:


q. coma

200
r. alcalosis respiratoria
s. acidosis metabolica
t. aumento de la síntesis de urea

El plasma de un paciente contiene Na 125mEq/L, glucosa 108mg/dl, urea 300mg/dl, podrá


presentar:
q. cirrosis
r. síndrome nefrósico
s. síntomas de hipertonicidad
t. síntomas de hipotonicidad

La hiponatremia aguda sintomática es debida a, excepto:


q. exceso de glucocorticoides
r. SIADH
s. post-operatorio
t. intoxicación hídrica

La administración de solución salina 0.9% produce:


q. disminución del volumen intracelular y aumento del extracelular
r. aumento del volumen intra y extracelular
s. aumento del volumen extracelular
t. aumento del volumen intracelular

Alcalosis metabólica con hipertensión arterial, hipokalemia, aldosterona normal y renina


normal:
m. Hipertensión renovascular
n. Hipertensión maligna
o. Sindroma de Cushing
p. Administración de licor

La hipokalemia puede predisponer, excepto:


q. intoxicación digitálica
r. disminución de la amoniogénesis
s. rabdomiolisis
t. ondas U en el EKG

17. La PTH promueve todo lo siguiente excepto:


a aumento de la absorción intestinal de calcio
b- aumento de la absorción tubulár de fosfato..
c. aumento de la resorción ósea de fosfato
d- aumento de la resorción osea de calcio
e, disminución de la reabsorción de fosfato

18. Un varon de 57 esta con hemodiálisis de mantenimiento para insuficiencia renal


cronica, ¿Cuál de las sig anomalías metabolicas podría ser anticipáda?
a. hipernatremia
b. hiponatremia
c.
d. exceso de vitamina D
e. hipoparatiroidismo

19. Un px de 25años fue ingresado a UCI por lesiones graves en cabeza y fractura
de la base del Graneo, aprox X horas después de la lesion manifestaba poliuria, la
OSMU 150, y la de suero 350. Los líquidos IV fueron detenidos y 3 hors después la
producción de orina y la osm urinaria permanecieron sin cambio. Se administraron
5 unidades de vasopresina IV, la osm de la orina se
incremento a 300. ¿Cuál es el dx mas probable?
a- Diabetes insípida central
b. diabetes insípida nefrogenica
c. intoxicación acuosa
d. sobrecarga de solutos
e. S IADH

201
55. indica falla renal aguda
a- U..N.. = 4.1
b- UN = 1
c. U..N.. = 1/4i/
d. Gravedad especifica 1.026
e. Ninguna

56. deshidratación por perdida de agua y densidad urinaria 1.003, creatinina 6 /


a. falla renal aguda parenquimatosa
b- falla prerenal
c. falla renal obstructiva
d. falla renal cronica
e. no hay falla renal

57. uno de los indices de falla renales


a. reabsorción de Na/oferta tubular de Na x 100
b. excresion urinaria deNa/oferta tubular de x 100
c.- oferta tubular de Na/excrecion urinaria de Na x 100 ~.--~
d- Una/Pna x 100
e. Ninguna

58. el Sdr hepatorenal es causa de falla:


a- prerrenal
b. renal
c. póstrenal
d. falla cardiaca
e. daño tubular

59. causa mas común de falla renal crónica:


A- DM

60- en la glomendonefritis aguda por infeccion, la causa, excepto


a- proteina mayor de 10 g
b- osmolaridad urinaria normal
c. hematuria
d. HT A
e. Edema

61. falla renal cronica, excepto


a. PTH aumentado
b- Hipofosfatemia
c- Acidemia
d. Hipocalcemia
e. 1.25 OH (D3) disminuida

62. EN la falla renal aguda siempre habra


a- Aumento de creatinina
b- fraccion de excrecion de Na aumentado
c- hiperkalemia
d- Una aumentado
e- Hipernatremia

63. causa de falla prerenal


a. Sdr nefrotico
b- obstrucción de ambos uréteres
c- falla cardiaca
d. Grlomerulonefritis aguda post-infeciosa
e. Calculo renal

64. en la nefritis tubular interticial se puede dar, excepto


a. h.ipernatremia

202
b. proteinuria de 1.5 g
c. normotension arterial
d. hipostenuria
e. hipocalcemia

65. Sugiere glomerulonefritis:


a. leucocituia
b-cilindros eritrocitarios
c- anuria
d. -

66 -causa de falla renal parenquimatosa


q. papilitis
r. calculo en pelvis derecha
s. estenosis unilateral de arteria renal
t. cistitis hemorragica

98 – Hep C periodo de incubación:


a- 50 dias

60- Causa de hipercalemia:


a Aumento del volumen arterial eficaz
b. Hipercatabolismo tisular
c Hiperaldosteronismo
d. Alcalosis Inetabolica
e. Cirrosis hepatica

61. Causa de hipomagnesemia


a. cirrosis avanzada
b. falla renal cronica
c. falla cardiaca
d. constipacion
e. acidosis respiratoria

62. Causa renal, de hipomagnesemia


a. falla renal crônica
b. esteatorreaa
c desnutricion severa
d. usó de diureticos de asa
e. infeccion urinaria

63. en caso de hipercalcemia pensar en:


a. neoplasia
b: hiperfosfatemia severa
c. nefrolitiasis
d. nefrocalcinosis
e. diuréticos

64. En el síndrome nefrosico puede encontrarse cilindros


a. cereos
b. hematicos
c. leucocitarios
d. epiteliales
e. mixtos leucocitario y hematicos,

2--En el riñón la hormona paratiroides:


Aumenta la reabsorción tubular de calcio y disminuye la de fósforo

3- La hipercalcemia produce
Potencialización de los efectos de la hipercalemia

203
36- La hipomagnesemia produce.
Vasodilatación periférica

36-De los factores: nefrotoxicidad./ -La buena hidratación

37 - En la concentración sérica para calcular:: 8,8

38- la velocidad de administración de potasio: 40 meq

39-hipocalcemia produce : fosfaturia

40-La causa de hipertensión con hipocalemia: Síndrome de Bartter=:,

41-En paciente con vomito severo hipotensión arterial o ir a la sar con mas de 40
meq
Insuficiencia suprarrenal;

42-La ,administración de sol isotónica aumento de vol extracelulares

43-Glomerulonefritis post estreptococcica Altera el cociente de ultrafiltración

44-En-el túbulo` intersticial se puede dar: La proteinuria de 4 g al día.

18- La hipofosfatemia cansa excepto: aumento de vitamina D


20. La renal aguda, oligurica todo es cierto excepto: Osmolaridad urinaria
aumentada
21 Paciente can alcalosis metabolica e hipokalemia debe ser tratada con KCL
3) Cuál de las siguientes medicinas pueden dar hiperkalemia
k) inhibidor de la enzima convertidota
l) bloqueador alfa adrenérgico
m) bloqueador beta adrenérgico
n) a y b
o) a y c

2) Hormonas que disminuyen en la falla renal crónica, excepto


a) 1,25 dihidroxi vitatamina D
b) eritropoyetina
c) paratohormona
d) foliculo estimulante

3) MgSO4 (PM 120) 1 gramo contiene de magnesio en meq


a) 10
b) 7.9
c) 8.3
d) 6

4) Factores que conducen a la progresión de falla renal crónica


a) hiperlipidemia
b) toxinas urémicas
c) proteinuria
d) b y c
c) todas

5) Paciente de 20 años presenta abundantes vômitos y diarrea de 3 días de evolución,


examen físico depleción de volumen, suero muestra Na 155, K 3meq/l, Cl 117meq/l,
HCO3 25 meq/l, tratamiento de elección:
a) D/A 5% + KCl
b) solución salina 3% + KCl
c) solución salina 23.4% + KCl
d) solución salina 0.45% + KCl

6) En la enfermedad ateroembólica renal se da lo siguiente excepto:

204
d) Velocidad de eritrosedimentación

7) Pseudohiperkalemia de 8 mEq/l que se ve en el EKG:


Onda T picuda
Aplanamiento de la onda P
Ay B
Ninguna

8)Criterios para iniciar diálisis excepto:


Alcalosiss Metabolica

9)Hipokalemia con normotension excepto


Síndrome de liddle

10) Plasama creatinina 8mg/dl N de urea 100mg/dl U na 40 Meq/l :


Insuficiência renal

11) Factores reversibles responsables de deterioro de la funcion renal excepto:


Fibrosis intersticial

12) Causa hipokalemia con renina y aldosterona disminuida:


Licorice

13. Sugiere trastorno túbulo intersticial:


.Proteinuria mayor a 3.5 g /24 horas
.Notrmotensión
.* Edema
.Osmolarida urinaria normal

14. Glomérulonefritis rápidamente progresiva


.Engrosamiento de la membrana basal
Fusión de los pedículos de los podocitos
prolñiferacion intracapilar
proliferación extracapilar ( creo que es esa verificar)

15.Una de las siguientes causas no produce hematuria


.enfermedad de cambios mínimos

16.Plasma Na:115, Posm240 mOsm,Uosm 680,Una 60:meq/l:


.Deshidratación hipotónica
.* secreción inapropiada de ADH
.Deshidratación isotónica
.Deshidratación primaria

17.en la falla renal crónca produce necrosis


calcifilaxia

18. No produce hipokalemia:


Pancreatitis aguda
rabdomiolisis
neoplasia
Def. Vit D

19. Paciente con glucosa 90mg/dl, Na+= 115mEq/L, Urea en 140mg/dl


a. Síndrome hipoosmolar
b. * Síndrome hiperosmolar
c. Diarrea
d. Ninguna de las anteriores.

20. Causa de seudohiponatremia con osmolaridad elevada


a. Hiperproteinemia
b. * Diabetes mellitus

205
c. Hiperlipidemidemia
d. Reserción transuretral prostática.

21. Manifestaciones de hipermagnesemia excepto


Normotensión

22. Piel en uremia crónica excepto


Paño blanco

23. No es causa de necrosis papilar aguda


a.Diabetes mellitas
b. Uropatía obstructiva
c. Pielonefritis
d. Trombosis de venas renal.

24. Puede presentarse hiperuricemia excepto


d. Adenocarcinoma gástrico

25. Compromete riñones y pulmones:


a. Enf de GoodPasture
b. Granulomatosis de Wegener
c. LES
d. Ay C
TODAS

26. Sugiere glomerulopatia:


-anasarca

27. TFG < 15ml/min corresponde al estadio de la enfermedad renal cronica:


* 4

28. Causa obstetrica de necrosis cortical bilateral


- desprendimiento prematuro de placenta

29. Promueve amoniogenesis en cirróticos


- hipokalemia

30. Criterio para diagnostico de síndrome nefrosico


a. Proteinuria en 24 hr de 3.5 gr para superficie corporal de 1. 73
b. Albumina serica < 3g/dl
c.Hiperlipidemia y edema
d. Todas las anteriores
e. ay b
62. Causa de hipomagnesemia:
q. estreñimiento
r. alcoholismo
s. falla pre-renal
t. IRA (insuficiencia renal aguda)

63. La hipocalcemia en la falla renal aguda puede ser asintomática por la:
q. acidemia
r. hipoalbuminemia
s. Deficiencia de PTH
t. Alcalemia

64. La hipofosfatemia causa, excepto:


q. Aumento de vit. D
r. Aumento de PTH
s. Disminución de la excresión de fosfato
t. Aumento de la absorción de fosfato a nivel intestinal

206
65. Un pte con diuresis de 250 ml al día con osmolaridad plasmática (presión osmótia) y
osmolaridad urinaria de 300 mosm/l, el dx probable e:
q. Insuficiencia suprarrenal
r. Insuficiencia renal
s. Nefritis túbulo intersticial crónica
t. Deshidratación hipertónica

66. Pte con oliguria, hiperkalemia, hiponatremia, creatinina 0.8 mg/dl y CPK normal:
Esto puede corresponder a:
q. IRA
r. Insuficiencia suprarrenal
s. Síndrome hepatorrenal
t. Rabdomiolisis

67. No está indicado en caso de hiperkalemia:


t. Aldactone

68. Mujer con hiperemesis grávida:


t. Alcalosis hipoclórica

69. En causa de hipomagnesemia:


t. Alcoholismo.

70. En el metabolismo del calcio está implicado, menos:


t. Insulina

71. Manifestación de hipertensión portal, excepto:


r. Ictericia

72. Terapia optima para hepatitis C:


r. Interferon alfa, PG-IFN y ribevirina

73. Causa de hepatitis fulminante en embarazada:


r. Hepatitis E

74. Hiponatremia, hipotónico, normovolémico:


t. SIADH

75. Complicación de cirrosis hepática, excepto:


r. Perforación del ílio

76. Es cierto sobre el H. pylori, excepto:


r. La mayoría tienen úlceras

77. Fórmula del clearence de creatinina:


t. clearence = 140 – edad x peso en Kg
72 x creatinina

78. Su infección produce litiasis renal:


s. proteus sp.

79. Fórmula de irritabilidad muscular:


t. IR = Na + K + OH
( ) + Mg + H

80. Con respecto al síndrome diarreico, nombre 2 tipos osmóticos:


e. Laxantes osmóticos (Mg2+, PO4, SO4)
f.
Carbohidratos no absorbibles (sorbitol, lactulosa).

81. Nombre dos síndromes diarréicos tipo secretora:


e. Consumo crónico de etanol.

207
f.
Obstrucción intestinal incompleta.

82. Nombre 2 sindromes diarréicos motores con aumento de motilidad:


i. Hipertiroidismo
j.
Prostaglandinas.

83. Indica falla renal aguda:


u. U.N. = 4.1
v.
U.N. = 1
w.
U.N. = ¼
x.
Gravedad específica: 1.026
y.
Ninguna.

84. Deshidratación por pérdida de agua y densidad urinaria 1.003, creatinina 6:


u. falla renal aguda parenquimatosa
v.
Falla pre-renal
w.
Falla renal obstructiva
x.
Falla renal crónica
y.
No hay falla renal.

85. Uno de los índices de falla renal es:


u. Reabsorción de Na/ oferta tubular de Na x 100
v.
Excreción urinaria de Na/ oferta tubular de Na x 100
w.
Oferta tubular de Na/ excresión urinaria de Na x 100 **

x.
Una / pna x 100
y.
Ninguna

86. El síndrome heatorrenal es causa de falla:


u. pre-renal
v.
renal
w.
posrenal
x.
falla cardiaca
y.
daño tubular

87. Causa más comun de falla renal cronica:


u. Diabetes melitus
v.
HTA
w.
Glomerulonefritis
x.
Enfermedad poliquística renal
y.
Vasculitis

36. En la glomerulnefritis aguda por infección, la causa, excepto:


a. proteínas mayor de 10g
b. osmolaridad urinaria normal
c. hematuria
d. HTA
e. Edema

• A pesar de la diversidad de enfermedades indicadas abajo, ellas tienen muchos


síntomas en común. Con frecuencia, los síntomas y signos observados incluyen:
disminución en el volumen de orina, proteína en la orina (proteinuria), sangre en la
orina microscópica o macrscópica (hematuria), hinchazón (edema), presión
sanguínea alta (HTA) y una disminución en la capacidad del riñón de extraer
eficazmente los residuos.

88. Falla renal crónica, excepto:


u. PTH aumentada
v. Hipofosfatemia
w. Acidemia
x. Hipocalcemia

208
y. 1.25 OH (D3) disminuída.

89. En la falla renal aguda siempre habrá:


u. aumento de creatinina
v. fracción de excresión de Na aumentado
w. Hiperkalemia
x. Una aumentado
y. Hipernatremia

90. Causa de falla pre-renal:


u. Sd. Nefrótico
v. Obstrucción de ambos uréteres
w. Falla cardiaca
x. Glomerulonefritis aguda pos- infecciosa
y. Calculo renal

91. En la nefritis tubular intersticial se puede dar, excepto:


u. hipernatremia
v. proteinuria de 1.5 g
w. normotension arterial
x. hipostenuria
y. hipocalcemia

92. Sugiere glomerulonefritis


n. leucocituria
o. cilindros eritrocitarios
p. anuria
q. ……

93. Causas de falla renal parenquimatosa:


j. papilitis
k. calculo en pelvis derecha
l. estenosis unilateral de arteria renal
m. cistitis hemorragica
n. ………

94. Causa de anuria:


u. Enfermedad de cambios mínimos
v. Necrosis cortical
w. Pielonefritis aguda leve bilateral
x. Necrosis tubular aguda
y. Calculo coloriforme bilateral

95. La hipofosfatemia causa todo, excepto:


s. Aumento de vit. D

96. Hepatitis C periodo de incubación:


s. 15-150 media de 5º (7 semanas)

97. En pte con falla renal crónica que ha sido transplantado esperamos encontrar:
t. osteomalacia.

Profeta nefrología I

31) Causa de hiperfosfatemia:


a) Falla 209ecesita
b) Falla renal 209ecesit
c) Falla cardiaca
d) Insuficiencia suprarrenal

32) Causas de hiperfosfatemia grave, excepto:


a) 209ecesitaría209 de cetoacidosis 209ecesitar.
b) Alcalosis respiratoria

209
c) Acidosis respiratoria.

33) Paciente con vómitos severos, oligurico con hipotensión arterial Una> 40…… 800 mOsm/Kg
puede tener:
a) Insuciencia renal aguda.
b) Insuficiencia suprarrenal
c) Falla pre-renal
d) Exceso de mineralocorticoides

34) Paciente urémico crónico con deshidratación severa puede presentar lo siguiente excepto:
a) Densidad urinaria 1010.
b) Osmolaridad urinaria de 800 mOsm/Kg
c) K sérico aumentado
d) Oliguria.

35) La hipokalemia produce:


a) Aumento de la magnitud del potencial de reposo

36) Primera*****Falla renal crónica:


a.HTA
b.Glomerulopatias
c.Diabetes Mellitus
d.Nefritis tubulo intersticial

37) Uno de los 210ecesit de falla renal aguda es:


a) Uosm/Tosm

38) Hormona producida en los riñones que disminuye en la falla renal 210ecesit:
a) ADH
b) Noradrenalina
c) Peptido natriuretico (según arjona)
d) 1,25 (OH)2 Vit D.

39) Las calcificaciones metastásicas se deben:??


a) Aumento del producto Ca, P
b) Aumento del producto Ca, Mg
c) Hipoparatiroidismo
d) Hiperparatiroidismo 1ario

40) Causa de anuria:


a) Pielonefritis unilateral
b) Necrosis tubular aguda
c) Enf cambios minimo
d) Necrosis cortical

41) la etapa de la insuficiencia renal 210ecesit con tasa de filtración glomerular de 60-89 ml/min es
la:??
a) 1
b) 2
c) 3
d) 4

42) La FENA es la relación entre:


a) Oferta tubular/Na reabsorbido x 100
b) depuración de Na/ depuración de creatinina x 100
c) oferta tubular/ Na excretado x 100.
d) Na excretado/ Na reabsorvido x 100.

43) En la nefritis tubular intersticial aguda se pierde la capacidad de poder concentrar la orina
debida a:??
a) Daño medular
b) Daño glomerular
c) Daño tubular proximal
d) Daño de la 210ecesit cortical del tubulo colector

44) La 210ecesita de NaCl al 23,4 % tiene:


a) 8000 mOsm/l

210
45) Se asocia con el SIAD, excepto:
a) lipoma

46) El efecto 211ecesitar mas importante de la calcitonina es, excepto:


a) Responder a la hipocalcemia
b) Aumenta la calciuria
c) Estimular la 211ecesitar osea osteoclastica
d) Utilizada en el tx de la hipercalcemia.

47) Favorece la entrada de K al espacio intracelular, excepto:


a) Agonista 211ecesitaría
b) Estimulacion 211ecesitaría
c) Insulina
d) Alcalosis metabolica

48) Factores que afectan el balance hídrico en el post-operatorio, excepto:


a) Fármacos
b) Calcitriol
c) Administración de liquidos 211ecesitarí
d) Aumento de la ADH

49) Las primeras manifestaciones clínicas de la hiponatremia son:??


a) neurológica
b) gastrointestinal
c) pulmonares
d) musculares

50) En la hipernatremia 211ecesitaría se da lo siguiente excepto:


a) Entrada de Na
b) Salida de aa
c) Ganancia de iones
d) Disminución del agua cerebral

51) Manifestaciones 211ecesita de la hipercalcemia, excepto:


a) Taquicardia
b) HTA
c) Constipación
d) Vomitos

52) En falla renal aguda por nefrotoxicidad:


a) Se conserva la membrana basal
b) Se destruye la membrana basal
c) Frecuente hay oliguria
d) La creatinina no aumenta

53) Causa de hipomagnesemia:


a) alcoholismo

54) Na serico de 168 mEq/l 70 kg (utilize como % de agua al 50% y Na normal 140 mEq/l
a) 7 litros

55) Indicaciones de diálisis en falla renal aguda, excepto:


a) Encefalopatia 211ecesit
b) Hiperkalemia refractaria
c) Ac. Metabolica refractaria
d) hipocalcemia

56) El KCL IV se puede asministrar asi, excepto:


a) 211ecesita por 211eces 211ecesitarí de 10 meq en 100 ml en 1hora
b) “ “ central de 20 meq em 50ml em 1 hora
c) “ “ central de 100 meq em 50ml em 1 hora
d) “ “ 211ecesitarí de 20 meq en 20ml en 2 horas

57) Aumenta la 211ecesitar urinaria de K, excepto:


a) Triamtereno
b) Bumetanida
c) Furosemida

211
d) Metolazona

58) Factores que regulan la 212ecesitar distal de K, excepto:


a) Flujo tubular distal y aporte distal de Na
b) Excrecion de aniones no reabsorbibles
c) Aldosterona
d) PTH

59) Enfermo de 60 Kg con 60% de agua corporal, Posm 260 mOsm/kg, ¿Cuántos gramos de
NaCl 212ecesitarí el Na en 10 mEq?
a) 21 gramos

60) Conteniendo 10 g de MgSO4 al %, cuantos ml de sal 212ecesitaría?


a) 20 ml

Profeta II

127. Cuándo disminuye el fosfato?


R. Aumento de PTH

128. Ejemplo de hipernatremia hipervolémica:


R. Aldosteronismo primario

129. La hipocalemia produce:


R. Aumento de la magnitud del potencial de reposo

130. La depleción de agua produce, excepto:


R. Salida de electrolitos del cerebro

131. Velocidad de administración de KCl en caso de urgencia:


R. 40 mEq/h

132. Cambios en EKG de la hipermagnesemia es igual:


R. Hipercalemia

133. Enfermo de 60 Kg con 60% de agua corporal, osmolaridad plasmática 260


mosmol/Kg, ¿cuántos gramos de NaCl aumenta el Na en 10 mEq en 24 horas?
R. a) 15
b) 21
c) 30
d) 19

134. Uso de Calcio en tratamiento de hipercalemia para:


R. Modificar el Potencial de Membrana

135. Hipermagnesemia severa produce:


R. Paro respiratorio

136. Venoclisis de 1L de D/A 5% conteniendo 10 mg de MgSO4 y al 50%:


R. 20mL

137. La hipocalcemia:
R. Prolongo QT a expensas del segmento ST

138. Tratamiento de urgencia de hipercalcemia:


R. SSN 0.9%

139. Causa de hiperfosfatemia:


R. Insuficiencia renal

140. Adaptación cerebral, hipertonicidad, excepto:


R. Pérdida de iones

212
141. En la adaptación cerebral, la hipotensión da lo siguiente, excepto:
R. Ganancia de aminoácidos
142. En ausencia de síntomas de hipernatremia, el sodio debe ser disminuido a razón de:
R. 0.5 mEq/L/h

143. Para corregir un sodio cerca de 185 mEq se administró en 86 horas, 66 letras de
D/SS 5%. ¿cuál es la velocidad de administración de la venoclisis?
R. 69 mL ¿?????

144. Manifestación clínica de hipercalcemia, excepto:


R. a) Hipotensión arterial
b) Bradicardia
c) Nauseas
d) Anorexia

145. Consecuencia de hipofosfatemia, excepto:


R. a) No afecta al SNC
b) Disfunción plaquetaria
c) Cardiomiopatía
d) Osteomalacia

146. Causa de hipofosfatemia, excepto:


R. a) Falla renal
b) Recuperación de cetoacidosis diabética
c) Alcalosis respiratoria
d) Quemaduras graves

147. Causa de HTA con hipocalemia, aldosterona-renina baja:


R. Síndrome de Cushing

148. Causas de HTA con hipocalemia y renina alta:


R. Hipertensión maligna

149. En una deshidratación severa hipertónica con hipotensión arterial severa, la


prioridad inicial es administrar:
R. SSN 0.9%

150. En la secreción inapropiada de ADH se da lo siguiente, excepto:


R. No hay expansión de volumen

151. Tejido con mayor contenido de K+:


R. a) Hígado
b) Músculo
c) Eritrocito
d) Hueso

profeta III

15) En IRA se mide todo menos:

a) Uosm/Posm x100
b) Na excretado/ Na filtrado x200
c) Na filtrado/ Na excretado x100
d) Na urinario >40
e) Ninguna de las anteriores

16) Paciente con hipertensión , hipokalemia, cretinina normal tiene:

a) Hipertensión arterial maligna


b) Cushing
c) Algo relacionado a licorice

213
31) En hiponatremia el daño principal es:

a) Neurológica
b) Respiratoria
c) Renal

32) Adaptación del cerebro ante solución hipotónica:

a) Pérdida de iones
b) Pérdida de aminoácidos
c) Edema cerebral
d) Ganancia de iones

53) Paciente con hipercalcemia puede ser por todo menos:

a) Aumento de secreción de TSH


b) Hiperparatioidismo
c) Toxicidad de Vitamina D
d) Medicamentosa
e) Leche de álcalis

62) La PTH promueve:

a) Reabsorción tubular de calcio y excreción de fosfato

2) Paciente que sufre golpe en la cabeza y tiene fractura de cráneo. Tiene


osmolaridad urinaria disminuída , pero al darle tratamiento con vasopresina ésta
aumenta. El paciente tiene:

c) Diabetes insípida central


d) Diabetes insípida nefrógena

Profeta III

11)Hiponatremia hipervolemica:
a) R/.Falla cardiaca

12)Las perdidas fecales excesivas de potasio evocan:


a) R/.tumor velloso

13) Paciente de 84 anos quien recibe diuretico tiazidico para la hipertensión


arterial es admitido por presentar diarrea y trastornos del sensorio. Tiene
disminución del turgor de la piel y la presion arterial es normal, Pna 174mEq/L,
Una 5mEq/L, Uosm 606mEq/kg. La hipertensión se debe a:
a) perdidas insensibles
b) polidipsia
c) uso de diuretico
d) diabetes insipida
e) secrecion inapropiada de hormona antidiuretica

14) Hipernatremia Hipervolemica


a) diuresis osmótica
b) diarrea
c) aldosteronismo primario
d) diabetes insipida
e) SIADH

15) La hipokalemia en un cirrotico puede precipitar COMA por:


a) Hipermagnesemia asociada

214
b) Aumento de la amoniogenesis
c) Hiperfosfatemia asociada
d) Rabdomiolisis
e) Induce vomitos

215
16) Causa de hiperkalemia:
a) Aumento del volumen arterial eficaz
b) Hipercatabolismo tisular
c) Hiperaldosteronismo
d) Alcalosis metabolica
e) Cirrosis hepatica

17) Causa de hipomagnesemia


a) cirrosis avanzada
b) falla renal cronica
c) falla cardiaca
d) constipacion
e) acidosis respiratoria

18) Causa renal de hipomagnesemia


a) falla renal cronica
b) esteatorrea
c) desnutricion severa
d) uso de diureticos de asa
e) infeccion urinaria

19) en caso de hipercalcemia pensar en:


a) neoplasia
b) hiperfosfatemia severa
c) nefrolitiasis
d) nefrocalcinosis
e) diureticos

20) En el síndrome nefrosico puede encontrarse cilindros:


a) cereos
b) hematicos
c) leucocitarios
d) epiteliales
e) mixtos leucocitario y hematicos

101. Paciente masculino de 33 años llega a su consulta por presentar fiebre, un pequeño rash,
signo de Jordano positivo; el medico de turno le envía un urinalisis que revela proteinuria,
hematuria y múltiples leucocitos; dentro de su diagnostico diferencial, debe estar una de las
siguientes patologías, cual:

i. Nefritis Intersticial Crónica.


j. Nefritis Intersticial Aguda.
k. Síndrome Nefrosico.
l. Fracaso Renal Agudo.

102. Dentro de las Glomerulopatias, existen algunas, que se pueden presentar con Síndrome
Nefrosico, y otras con Síndrome Nefrítico, cual de las siguientes se suele presentar con Síndrome
Nefrítico:

i. Cambios Mínimos.
j. Nefropatia IgA.
k. Nefropatia Membranosa.
l. Diabetes.

103. En un paciente de 65 años, que desarrolla un Sd. Nefrítico Agudo, la etiología mas común
causando de la Glomerulopatia es:

i. Vasculitis.
j. GN rápidamente progresiva.

216
k. Nefropatia IgA.
l. LES.
Creo..

104. Cual de las siguientes Glomerulopatia, siempre desarrolla Insufiencia Renal:

i. Cambios Mínimos.
j. Membranosa.
k. Rápidamente Progresiva.
l. Mesangiocapilar.
Creo…

105. En un paciente con SIADH, al medirle la osmolaridad urinaria, la podemos encontrar:

h. Elevada.
i. Disminuida.
j. Normal.

106. La hiponatermia, se hace clínicamente evidente cuando el sodio, ha disminuido a:

i. 110 mEq/lt.
j. 115 mEq/lt.
k. 120 mEq/lt.
l. 125 mEq/lt.

107-111. Mencione 5 medicamentos que puedan causar hiperkalemia:

k. _____________________espironolactona___________________________.
l. __________________________________trimetropin______________.
m. ___________________________________________pentamidina_____.
n. ________________________________________________.IECA
o. ________________________________________________.AINES
112-117. Las características del Síndrome Nefrotico son:

m. _________________________hipoalbuminemia mayor de 3g
/dl_______________________.
n. ____________________3.5 g de preoteina en orina en 24 h en 1.73m2 de superficie
corporal____________________________.
o. _______hiperlipidemia_________________________________________.
p. ___________________edema_____________________________.
q. _________________________hipertension_______________________.
r. ___________________________________cilindro cereo _____________.

118-120. Los tres puntos en los que se basa la corrección de la hiperkalemia son:

g. ____________________excrecion____________________________.
h. ____________________________absorcion____________________.
i. ____________________________________resorcion____________.

121-123. Para cada uno de estos puntos, se suele usar: se le solicita al estudiante ser lo mas
explicito posible en su respuesta, en cuanto a dosis y tratamiento exacto.

g. __________expandir volumen con ssn+kcl cuando no hay IC +


furosemida.______________________________________.
h. ___________________quelante HPO4 o EDTA(no se usa por daño
renal)_____________________________.
i. ____________________ bifosfonatos pamidronato 1 dosis por semana

124-126. Debemos sospechar, de alteraciones del Magnesio, en que pacientes:

g. __________________________trastornos de tubo digestivo x mal


absorcion______________________.
h. ________________________cirroticos________________________.
i. ________________________________embarazadas con THE________________.

217
127. Paciente con hipokalemia + alcalosis metabólica + hipertensión arterial + disminución de la
renina plasmática, esta desarrollando un:

c. ________________________aldosteronismo primarios
(creo)________________________.

128. Única causa de diarrea con alcalosis metabólica es:

c. ______________________________________________no se__.

129. Si un paciente, tiene un potasio de 3 mEq/lt, decimos que ha perdido ya:

c. __________________________creo q 200______________________. Ya que por cada


mol de K que se pierde hay menos 200 mmol de k corporal.

130. Cual es la principal preocupación que se tiene, cuando un paciente tiene un aumento en los
niveles plasmáticos de Magnesio:

c. ________________________bloqueo av completo deprsion


respirarotia________________________.

131-132. Causas de acidosis metabólica con hipokalemia:

e. ___________________________acidosis tubular renal_____________________.


f. ___________________________no se la otra_____________________.

133. Que examen le ayudaria para diferenciarlas:

c. ________________________no se________________________.

134. Que esperaria encontrar en una ATR tipo 2:

c. _____________________nose q es esto___________________________.

135-136. Principales causas de hipercalcemia:

e. ____________________rabdomiolisis____________________________.
f. _______________________________acidosis diabetica_________________.

136-137. Principales causas de hipocalcemia:

e. __________________________falla renal cronica______________________.


f. ______________poca ingestión (en el libro decía que por comer arcilla pero no creo q
sea principal causa.)__________________________________.

138-139. Consecuencias de la hiperfosfatemia:

e. ___________________hipocalcemia por disminución de


paratohomrona_____________________________.
f. _______________________________________________.

140-143. El Sd. De Barter, se caracteriza clínicamente por:

i. _________________hipocalemia_______________________________.
j. ____________________normotensoion_________________________.
k. _________________________________alcalosis metabolica
l. ________________________________________________.hiperaldosteronismo
hipereninemico

144. En un paciente con diagnostico de Glomerulonefritis post estreptococcica, el diagnostico


serologico se basa en el hallazgo de anticuerpos:

218
j. Antiestreptolisina O.
k. Antiestreptolisina A.
l. Antiestreptolisina P.
m. Antiestreptolisina K.

145. En la Nefiritis secundaria al LES, se suelen encontrar positivos los:

j. ANA.
k. ANCA c.
l. ANTI LU
m. ANCA p.

146. Si un paciente es diagnosticado con Falla Renal Crónica, el siguiente punto es el estadiage del
mismo, si tiene una TFG en 82 ml/minuto, esta en estadio:

k. I
l. II
m. III
n. IV
o. V

147-148. La Osteodistrofia Renal, se va a caracterizar clínicamente por la presencia de:

e. _____________________osteitis fibrosa quistica__.


f. _____________________osteomallacia u psteopatia adinamica

149.Normalmente la Kps entre el Fósforo y el calcio es de:

c. ________________________no c____________________.

150-154. Las características de una muestra de orina con sedimento de tipo nefrítico, son:
noc
k. _____________________________microhematuria_____.
l. ____________________cilindrono hematico______.
m. ________________________________________________.
n. ________________________________________________.
o. ________________________________________________.

155-158. Mencione algunos parámetros de laboratorio, para pensar en una IRC Renal:
________________________BUN_____________________.
i. __________________________creatinina____________.
j. ________hemoglobina
k. _________________me __ da pereza pensa___ y no c.__________________________.

159. El Gasto cardiaco normal es de 5 – 6 litros por minuto; de los cuales, un 25% le corresponde al
riñón, en una relación de:

i. 1200 ml/min.
j. 1300 ml/min.
k. 1400 ml/min.
l. 1500 ml/min.

160. Si un paciente, diagnosticado con FRC, presenta ademas una acidosis metabolica
hipercloremica, usted sabe que este paciente debe tener una Brecha Anionica:

i. Normal.
j. Aumentada.
k. Disminuida.
l. Ninguna de las anteriores.
Nunca entendi esto ni en fisio
161. Este mismo paciente, por tener estas características clínicas, debe encontrarse en estadios de
la FRC:

j. Avanzados.

219
k. Iniciales.
l. No es relevante.
m. Ninguna de las anteriores.

162. Para que por una obstrucción, aumente la creatinina en IRA, se deben haber perdido:

i. ¼ partes de la masa renal.


j. 2/4 partes de la masa renal.
k. ¾ partes de la masa renal.
l. 4/4 partes de la masa renal.

163. La lesión histologica mas común de IRA es:

i. Necrosis cortical bilateral.


j. Necrosis tubular aguda.
k. Nefritis intersticial.
l. Papilitis necrotizante.

164. Y la mas peligrosa y temida es:

j. Necrosis cortical bilateral.


k. Necrosis tubular aguda.
l. Nefritis intersticial.
m. Papilitis necrotizante.

165. Cual de las siguientes, es la principal causa de IRA, en la maternidad.

l. Abortos infectados.
m. Fetos muertos retenidos.
n. Desprendimiento prematuro de membrana.
o. Eclampsia.
p. Se dan todas por igual.
Todas pueden llevar a IRA pero creo q la principal es esta

399) Un paciente de 35 años con insuficiencia renal crónica secundaria a pielonefritis


crónica recibe un trasplante renal de cadáver con el que compartía dos identidades
en A y B y una en DR. Recibe tratamiento inmunosupresor con ciclosporina A y
corticoides a dosis estándar. En el postoperatorio inmediato se observa buena
diuresis y no es necesario el tratamiento sustitutivo con hemodiálisis. En el 5º día
de evolución, el paciente presenta fiebre de 38ª, TA de 180/110, oliguria y
disminución en la concentración urinaria de sodio. El diagnóstico más probable
sería:

a) Crisis hipertensiva.
b) Infección respiratoria.
c) Pielonefritis aguda del injerto renal.
d) Recidiva de su enfermedad renal.
e) Rechazo agudo del injerto renal.

400) ¿A cuál de los siguientes factores NO se asocia la Pielonefritis Aguda por


Pseudomonas Aeuruginosa?:

a) Embarazo.
b) Sonda urinaria.
c) Nefrolitiasis.
d) Manipulación urológica.
e) Estenosis de la vía urinaria.

401) Paciente diagnosticado de hiperplasia benigna de próstata que presenta como


sintomatología dificultad para el inicio de la micción, disminución de la fuerza y
volumen del chorro miccional, micción entrecortada y goteo terminal; la aparición
de polaquiuria, y urgencia miccional se debe generalmente a:

a) Presencia de infección urinaria.

220
b) Inestabilidad vesical.
c) Prostatitis (adenomitis).
d) Existencia de residuo vesical.
e) Presencia de divertículos vesicales.

402) Una paciente de 15 años de edad consulta por poliuria y nicturia, cansancio fácil
y astenia. Presenta dichos síntomas desde hace años, y tienden a intensificarse
durante los veranos muy calurosos, en los que se asocia hormigueo lingual y
peribucal. A la exploración física sólo destaca una tensión de 100/50 mmHg. El
ionograma muestra Na 135 mEq/l, K 2 mEq/l, Cl 105 mEq/l, pH 7.45, bicarbonato 30
mEq/l. La determinación de renina y aldosterona muestra cifras elevadas, tanto
basales como tras estímulo. De los procesos que siguen ¿cuál es compatible con el
cuadro clínico descrito?:

a) Hipoaldosteronismo primario.
b) Estenosis de la arteria renal.
c) Síndrome de Liddle.
d) Enfermedad de Addison.
e) Síndrome de Bartter. M. clínicas: hipokalemia, alcalosis metabólica, hiperaldosteronismo
por hiperplasia del aparato yuxtaglomerular y disminución del volumen circulatorio
efectivo →que puede causar hipotensión arterial o normotensión.

403) Un hombre, adicto a drogas por vía parenteral, está ingresado por endocarditis
infecciosa. Durante su enfermedad presenta un cuadro de glomerulonefritis aguda.
¿Cuál de las respuestas es INCORRECTA?:

a) Suele ser debida a inmunocomplejos.


b) No suele presentar piuria.
c) El complemento está descendido.
d) A veces produce síndrome nefrótico.
e) Suele evolucionar favorablemente al controlar la infección cardiaca.

404) En la poliquistosis renal del adulto, ¿cuál de las siguientes afirmaciones es


FALSA?:

a) Es una enfermedad hereditaria, autosómica dominante.


b) Habitualmente se detecta en la primera infancia con ecografía.
c) Es causa de deterioro progresivo de la función renal.
d) Frecuentemente cursa con hipertensión arterial.
e) Se le asocia litiasis renal en un 15-20% de los casos.

405) Señalar la respuesta correcta en relación con la fisiopatología del calcio en la


insuficiencia renal crónica:

a) La hormona paratiroidea se eleva precozmente y de forma progresiva.


b) Mecanismos compensadores consiguen mantener a la hormona paratiroidea dentro de
límites normales hasta estadíos avanzados de la insuficiencia renal.
c) Disminuye la síntesis de 25-hidroxi-vitamina D.
d) El calcio sanguíneo aumenta paralelamente a la disminución del filtrado glomerular.
e) Aumenta la síntesis de 1,25-dihidroxi-vitamina D.

406) Una mujer de 68 años acude al Servicio de Urgencias por malestar general que
ha ido progresando en los últimos 15 días, a partir de un episodio gripal. Ha notado
disminución progresiva del volumen de diuresis, edemas maleolares y dificultad
respiratoria. Es hipertensa. En la analítica destaca una creatinina plamática de 5
mg/dl, urea 180 mg/dl, Na 138 mEq/l, K 4.9 mEq/l. Las cifras de complemento son
normales. Los anticuerpos anti-membrana basal son negativos. En la orina presenta
cilindros hemáticos, proteinuria de 1 g/l y microhematuria. Aporta una analítica de
un mes antes, sin alteraciones. ¿Cuál de los siguientes diagnósticos es más
probable?: al haber falta de opciones puse las posibles consideraciones que se
puedan escoger.

a) El cuadro clínico que presenta esta paciente es el SINDROME NEFRÍTICO AGUDO en el


cual habría que descartar todas las causas de este tipo.
b) Las cifras normales de complemento y las pruebas de autoinmunidad normales descartan un
nefrtis lupica u otro tipo de enfermedad auto inmunitarias.

221
c) Al haber padecido un cuadro clínico tipo gripal se puede pensar en una infección viral.(
citomegalovirus, influenza, sarampión, parvovirus, virus parotiditis, Epstein-Barr, dengue
hemorrágico, virus coxackie).

407) Un varón de 45 años llega comatoso a Urgencias. Presenta PaCO2 basal 25


mmHg, pH 7,15, hipocalcemia moderada con Gap aniónico y osmolar elevados,
leucocitosis y cristaluria. Con más probabilidad este paciente tiene una intoxicación
por:

a) Barbitúricos.
b) Monóxido de carbono.
c) Etilenglicol.
d) Benzodiacepinas.
e) Salicilatos.

408) Cuando un paciente bajo tratamiento diurético con tiazidas o furosemida


incumple la dieta y come más sal de la prescrita, el resultado análitico esperable es:

a) Mayor hipernatremia.
b) Mayor hiponatremia.
c) Mayor hiperpotasemia.
d) Mayor hipopotasemia.
e) Mayor acidosis.

409) ¿En cuál de las siguientes glomerulonefritis hay activación del complemento por
la vía alternativa?:

a) Aguda secundaria a endocarditis.


b) Aguda postestreptocócica.
c) Difusa lúpica.
d) Por crioglobulinemia.
e) Membranosa.

410) En un enfermo con un síndrome de Goodpasture que le ha conducido a una


insuficiencia renal crónica:

a) No debe realizarse un trasplante renal, pues la enfermedad le dañará el riñón trasplantado.


b) El trasplante puede realizarse si la producción de anticuerpos anti-membrana basal ha
cesado. ( Pag. 1788 dice: el transplante es posible, pero ante el peligro de recidiva, la
experiencia sugiere que el enfermo debe esperar 6 meses y hasta que no se detecten en
suero los anticuerpos.)
c) Debe seguir siendo tratado indefinidamente con inmunosupresores para proteger su pulmón.
d) Debe seguir siendo tratado indefinidamente con esteroides para proteger su pulmón.
e) La función renal se recuperará cuando dejen de producirse autoanticuerpos anti-membrana
basal.

411) En relación a la hemoglobinuria paroxística nocturna, señale cuál de las


siguientes afirmaciones es FALSA:

a) Es un defecto adquirido.
b) Falta la molécula de anclaje glicosilfosfoinositol en el membrana.
c) Cursa con un aumento de la resistencia del hematíe a la lisis por el complemento.
d) Se acompaña de un aumento de frecuencia de trombosis venosas.
e) Presenta a menudo leucopenia y trombopenia.

412) Con respecto a la nefropatía diabética, señalar la respuesta FALSA:

a) La diabetes tipo 2 es la etiología más frecuente de insuficiencia renal terminal en el mundo


occidental.
b) Más del 90% de los diabéticos tipo 1 desarrollan nefropatía a los 30 años del diagnóstico de
diabetes.

222
c) La alteración renal más temprana es la hiperfiltración.
d) La existencia de microalbuminuria predice el desarrollo de nefropatía clínica.
e) La gran mayoría de los diabéticos tipo 1 con nefropatía tienen también retinopatía.

413) El diagnóstico de la anemia hemolítica se realiza gracias a cinco signos


biológicos característicos: elevación de los reticulocitos, hiperregeneración
eritroblástica, hiperbilirrubinemia no conjugada, incremento de la
lácticodeshidrogenasa sérica (LDH) y descenso de la haptoglobina. ¿Cuáles de estos
signos biológicos pueden observarse también en las pérdidas de sangre por
hemorragia?:

a) Descenso de la haptoglobina e hiperregeneración eritroblástica.


b) Elevación de LDH y bilirrubina no conjugada.
c) Hiperregeneración eritroblástica y elevación de la cifra de reticulocitos.
d) Elevación de la bilirrubina no conjugada y descenso de la haptoglobina.
e) Descenso de la haptoglobina y elevación de la LDH.

414) En el tratamiento de la hiperpotasemia grave de un paciente urémico NO está


indicado administrar:

a) Resinas de intercambio catiónico (sodio, calcio).


b) Bicarbonato sódico i.v.
c) Glucosa con insulina i.v.
d) Gluconato cálcico o cloruro cálcico i.v.
e) Tiacidas por vía oral.

415) Paciente de 82 años de edad, que refiere síndrome constitucional de tres


semanas de evolución, con astenia, anorexia y pérdida de peso con oligoanuria
progresiva en las veinticuatro horas previas al ingreso hospitalario. No signos de
hiperhidratación. Creatinina plasmática 6 mg/dl. Proteinuria 1gr/24h. Sedimiento:
microhematuria. Determinación de ANCA positivo, patrón p-ANCA anti MPO. ECO
renal que muestra riñón derecho pequeño y riñón izquierdo de tamaño normal.
¿Cuál cree que es el procedimiento más adecuado y prioritario?:

a) Iniciar tratamiento sustitutivo con diálisis.


b) Proceder a practicar biopsia renal.
c) Iniciar tratamiento con pulsos intravenosos de metilprednisolona y Ciclofosfamida oral.
d) Plantear plasmaféresis.
e) Iniciar tratamiento con Prednisona oral.

223
416)

224
417) Un paciente diabético conocido, de 2 años de evolución y 64 años de edad, consulta por anemia,
proteinuria de 3 gramos/24 horas junto con hematuria, hipertensión arterial y discreta insuficiencia
renal con creatinina plasmática de 2.3 mg/dl. El diagnóstico MENOS probable es:
a) Nefropatía diabética.
b) Angeítis necrotizante del tipo poliangeítis microscópica.
c) Glomerulonefritis rápidamente progresiva.
d) Granulomatosis de Wegener.
e) Síndrome úremico-hemolítico.

418) Si un paciente con una pérdida neuronal progresiva, la presencia de hiperpotasemia indica que
ha perdido al menos:
a) Un 25% del filtrado glomerular.
b) Un 35% del filtrado glomerular.
c) Un 50% del filtrado glomerular.
d) Un 75% del filtrado glomerular.
e) Un 100% del filtrado glomerular.

419) Un paciente de 35 años con insuficiencia renal crónica secundaria a pielonefritis crónica recibe
un trasplante renal de cadáver con el que compartía dos identidades en Ay B y una en DR. Recibe
tratamiento inmunosupresor con ciclosporina A y corticoides a dosis estándar. En el postoperatorio
inmediato se observa buena diuresis y no es necesario el tratamiento sustitutivo con hemodiálisis.
En el 5º día de evolución, el paciente presenta fiebre de 38º. TA de 180/ 110, oliguria y disminución
en la concentración urinaria de sodio. El diagnóstico más probable sería:
a) Crisis hipertensiva.
b) Infección respiratoria.
c) Pielonefritis aguda del injerto renal.
d) Recidiva de su enfermedad renal.
e) Rechazo agudo del injerto renal.

420) ¿Cuál de las siguientes medidas terapéuticas NO está indicada en el tratamiento de la


Hiperpotasemia?:
a) Glucosa e insulina.
b) Bicarbonato sódico.
c) Gluconato cálcico.
d) Resinas de intercambio iónico.
e) Glucocorticoides.

421) ¿Cuál de las siguientes nefropatías glomerulares no se relaciona con hipocomplementemia?:


a) Glomerulonefritis aguda postinfecciosa.
b) Glomerulonefritis membrano-proliferativa.
c) Nefropatía lúpica.
d) Nefropatía diabética.
e) Nefropatía de la crioglobulinemia.

422) 185. Un paciente de 28 años presenta hematuria macroscópica al día siguiente de una infección
faríngea y edemas maleolares. Dos años antes había presentado un cuadro similar que desapareció
con rapidez, por lo que no había consultado previamente. En la exploración física se objetiva una
T.A. de 180/110, una Creatinina Plasmática de 2 mgrs/dl e Hipocomplementemia. ¿Cuál de los
siguientes es el diagnóstico más probable?:

225
a) Glomerulonefritis membrano-proliferativa.
b) Glomerulonefritis post-estreptocócica aguda.
c) Glomerulonefritis rápidamente progresiva.
d) Glomerulonefritis por depósitos mesangiales de IgA (Enfermedad de Berger).
e) Hialinosis Segmentaria y Focal.

423) Niña de 3 años, que presenta afectación brusca del estado general dentro del contexto de un
cuadro febril con diarrea mucosanguinolenta. A la exploración aparece pálida y soñolienta, la
auscultación cardiopulmonar es normal, el abdomen doloroso y el resto del examen físico sólo
revela la presencia del petequias puntiformes diseminadas. La orina es hematúrica y se constata
hipertensión arterial. El hemograma muestra HB 7 g/dl, 17.000 leucocitos con neutrofilia, 37.000
plaquetas con normalidad de las pruebas de coagulación. ¿Cuál es el diagnóstico más probable?:
a) Sepsis por Salmonella.
b) Púrpura de Schönlein-Henoch.
c) Glomerulonefritis aguda post-infecciosa.
d) Síndrome hemolítico-urémico.
e) Coagulación intravascular diseminada.

424) Se trata de un niño de 7 años, con retraso estaturo-ponderal, que presenta signos radiológicos de
raquitismo y ecográficos de nefrocalcinosis. Los análisis demuestran acidosis metabólica con PH
inferior a 7.32 y Bicarbonato plasmático inferior a 17 mEq/ litro. El PH de la orina en 3
determinaciones es siempre superior a 5.5. Otras alteraciones metabólicas consisten en
hipercalciuria, hipocitruria y discreta disminución de la resorción del fósforo. No se detecta
glucosuria ni aminoaciduria. De las siguientes posibilidades, señale el diagnóstico correcto:
a) Síndrome de Bartter.
b) Acidosis tubular distal (Tipo I).
c) Acidosis tubular proximal (Tipo II).
d) Síndrome completo de Fanconi*. Puede ser que sea un síndrome de fanconi por el raquitismo;
pero no es completo en su totalidad por falta de aminoaciduria no selectiva y glucosuria.
e) Enfermedad de Harnup.

425) 174. Paciente de 50 años de edad, que consulta por dolor en la fosa renal, polaquiuria, disuria y
hematuria. En el análisis de la orina se observa piuria y pH ácido con cultivos repetidamente
negativos. ¿Cuál sería la primera posibilidad diagnóstica, de entre las siguientes?:
a) Pielonefritis aguda.
b) Síndrome nefrítico.
c) Tuberculosis genitourinaria.
d) Prostatitis aguda.
e) Carcinoma renal de células claras.

226
426) La insuficiencia renal funcional de la cirrosis hepática es debida a:
a) lesión glomerular
b) lesión tubular
c) hipovolemia
d) compresión de la ascitis sobre los grandes vasos renales
e) vasoconstricción de las arterias renales
f) Todas las anteriores

427) Urémico crónico con potasio sérico de 6.5mEq/L con leve acidemia, sin trastorno
muscular y EKG normal, puede ser tratado con:
a) kayaxelate
b) gluconato de Calcio al 10%
c) NaHCO3
d) Diálisis

428) La insuficiencia renal crónica ocasiona hiperfosfatemia permanentemente cuando la


tasa de filtración glomerular esta por debajo de:
a) 60ml/min
b) 50 ml/min
c) 25 ml/min
d) 40 ml/min

429) La hipofosfatemia es causa de excepto


a) inotropismo positivo
b) disminución de 2,3 DPG
c) hemólisis
d) curva de HbO2 desplazada hacia la izquierda

430) La solución de NaCl 23,4% tiene de osmolaridad:


a) 8000 mosm/l
b) 800 mosm/l
c) 80 mosm/l
d) 0,8 mosm/l

431) En la hiperkalemia:
a) aumenta la sístole eléctrica
b) hay menor diferencia entre el potencial de reposo y el umbral
c) el potencial de reposo transmembrana se hace mas negativo
d) el potencial umbral se hace menos negativo

432) La velocidad de administración de potasio en caso de gran riesgo para la vida puede
ser de:
a) 10-20 mEq/L
b) 8 mEq/L
c) 110 mEq/L
d) 40 mEq/L

433) Un ejemplo de hiponatremia hipotónica normovolemica es:


a) síndrome nefrósico
b) SIADH
c) Diarrea
d) Cirrosis

434) A una embarazada con eclampsia se le ordena 1 litro de D/A 5% con 10g de MgSO4.
¿Cuántos ml de MgSO4 al 50% se han utilizado?
a. 20
b. 5
c. 12,5
d. 30

435) Causa hipomagnesemia:


a) estreñimiento
b) alcoholismo
c) deshidratación hipertónica
d) insuficiencia renal aguda

436) Paciente con diuresis de 300ml/día, K sérico 7mEq/L, Posm y Uosm 300mosm/L, el
diagnóstico más probable es:

227
a) insuficiencia suprarrenal
b) insuficiencia renal
c) SIADH
d) Deshidratación primaria

437) La mielinosis pontina se caracteriza por:


a) pares craneales intactos
b) cuadriplejía flácida
c) comportamiento normal
d) hipertonía de los miembros inferiores

438) Calcule el déficit de agua en una paciente con sodio sérico 168mEq/L y que pesa 70Kg
(utilice como porcentaje de agua 50% y sodio normal mEq/L):
a) 5L
b) 3L
[𝑁𝑎+]𝑝𝑙𝑎𝑠𝑚𝑎−140
c) 10L 𝑑𝑒𝑓𝑖𝑐𝑖𝑡 𝑑𝑒 𝑎𝑔𝑢𝑎 = 140
× 𝐴𝐶𝑇
d) 7L

439) En la SIADH la concentración de sodio urinario es de:


a) menos de 10mEq/L
b) 20mEq/L
c) 15mEq/L
d) Mas de 40mEq/L

440) En paciente con vómitos abundantes y prolongados con hipokalemia severa debe
administrarse:
a) NaCl 0.9% con KHCO3
b) NaCl 0.9% con KCl
c) NaCl 0.9% con gluconato de potasio
d) NaCl 0.9% con NaHCO3

441) La hipokalemia en un paciente con cirrosis puede contribuir:


a) coma
b) alcalosis respiratoria
c) acidosis metabolica
d) aumento de la síntesis de urea

442) El plasma de un paciente contiene Na 125mEq/L, glucosa 108mg/dl, urea 300mg/dl,


podrá presentar:
a) cirrosis
b) síndrome nefrósico
c) síntomas de hipertonicidad
d) síntomas de hipotonicidad

443) La hiponatremia aguda sintomática es debida a, excepto:


a) exceso de glucocorticoides
b) SIADH
c) post-operatorio
d) intoxicación hídrica

444) La administración de solución salina 0.9% produce:


a) disminución del volumen intracelular y aumento del extracelular
b) aumento del volumen intra y extracelular
c) aumento del volumen extracelular
d) aumento del volumen intracelular

445) Alcalosis metabólica con hipertensión arterial, hipokalemia, aldosterona normal y


renina normal:
a) Hipertensión renovascular
b) Hipertensión maligna
c) Sindroma de Cushing
d) Administración de licor

446) La hipokalemia puede predisponer, excepto:


a) intoxicación digitálica
b) disminución de la amoniogénesis
c) rabdomiolisis

228
d) ondas U en el EKG

447) Cuál de las siguientes medicinas pueden dar hiperkalemia


a) inhibidor de la enzima convertidota
b) bloqueador alfa adrenérgico
c) bloqueador beta adrenérgico
d) ayb
e) ayc

448) Hormonas que disminuyen en la falla renal crónica, excepto


a) 1,25 dihidroxi vitatamina D
b) eritropoyetina
c) paratohormona
d) foliculo estimulante

449) Factores que conducen a la progresión de falla renal crónica


a) hiperlipidemia
b) toxinas urémicas
c) proteinuria
d) byc
e) todas

450) Paciente de 20 años presenta abundantes vômitos y diarrea de 3 días de evolución,


examen físico depleción de volumen, suero muestra Na 155, K 3meq/l, Cl 117meq/l, HCO3
25 meq/l, tratamiento de elección:
a) D/A 5% + KCl
b) solución salina 3% + KCl
c) solución salina 23.4% + KCl
d) solución salina 0.45% + KCl

451) En la enfermedad ateroembólica renal se da lo siguiente excepto:


a) Velocidad de eritrosedimentación

452) Pseudohiperkalemia de 8 mEq/l que se ve en el EKG:


a) Onda T picuda
b) Aplanamiento de la onda P
c) Ay B
d) Ninguna

453) 8)Criterios para iniciar diálisis excepto:


a) Alcalosiss Metabolica

454) 9)Hipokalemia con normotension excepto


a) Síndrome de liddle

455) Plasama creatinina 8mg/dl N de urea 100mg/dl U na 40 Meq/l :


a) Insuficiência renal

456) Factores reversibles responsables de deterioro de la funcion renal excepto:


a) Fibrosis intersticial

457) Sugiere trastorno túbulo intersticial:


e) Proteinuria mayor a 3.5 g /24 horas
f) Notrmotensión
g) Edema
h) Osmolaridad urinaria normal

458) Glomérulonefritis rápidamente progresiva


a) Engrosamiento de la membrana basal
b) Fusión de los pedículos de los podocitos
c) prolñiferacion intracapilar
d) proliferación extracapilar

459) 16.Plasma Na:115, Posm240 mOsm,Uosm 680,Una 60:meq/l:


e) .Deshidratación hipotónica

229
f) .secreción inapropiada de ADH
g) .Deshidratación isotónica
h) .Deshidratación primaria

460) No produce hipokalemia:


a) Pancreatitis aguda
b) rabdomiolisis
c) neoplasia
d) Def. Vit D

461) Paciente con glucosa 90mg/dl, Na+= 115mEq/L, Urea en 140mg/dl


a) Síndrome hipoosmolar
b) Síndrome hiperosmolar
c) Diarrea
d) Ninguna de las anteriores.

462) Causa de seudohiponatremia con osmolaridad elevada


a) Hiperproteinemia
b) Diabetes mellitus
c) Hiperlipidemidemia
d) Reserción transuretral prostática.

463) 21. Manifestaciones de hipermagnesemia excepto


a) Normotensión

464) 22. Piel en uremia crónica excepto


a) Paño blanco

465) 23. No es causa de necrosis papilar aguda


a) .Diabetes mellitus
b) . Uropatía obstructiva
c) . Pielonefritis
d) Trombosis de venas renal.

466) 25. Compromete riñones y pulmones:


a) Enf de GoodPasture
b) Granulomatosis de Wegener
c) LES
d) Ay C

467) 26. Sugiere glomerulopatia:


a) -anasarca

468) 27. TFG < 15ml/min corresponde al estadio de la enfermedad renal crónica:
i) 5

469) 28. Causa obstetrica de necrosis cortical bilateral


a) - desprendimiento prematuro de placenta

470) 29. Promueve amoniogenesis en cirróticos


a) - hipokalemia

471) Criterio para diagnostico de síndrome nefrosico


a) Proteinuria en 24 hr de 3.5 gr para superficie corporal de 1. 73
b) Albumina serica < 3g/dl
c) Hiperlipidemia y edema
d) Todas las anteriores
e) ay b

472) La hipermagnesemia puede producir:


i) R/. Paro respiratorio

473) Tratamiento de hiprecalcemia:


i) R/. SSN 0.9%

230
474) La hipofosfatemia son causas, excepto:
a) Aumento de la PTH
b) Aumento de la absorción intestinal de fosfato
c) Disminuye la fosfatemia
d) Aumento de la vitamina D

475) 14. Paciente crónico con deshidratación severa puede presentar lo siguiente excepto:
a) Densidad urinaria 1010
b) Osmolaridad urinaria de 800 mOsm/kg
c) K sérico aumentado
d) Oliguria

476) Paciente con vómito abundante, hipertensión arterial, sodio urinario de 6, oligúrico,
osmolaridad de 800 presenta:
a) R/.Insuficiencia suprarrenal

477) Paciente urémico crónico con vómitos severos, hipotensión arterial, orina al azar
muestra Na mas de 40, osmolaridad de 800 puede tener:
a) Falla suprarrenal

478) Cual de las siguientes es causa de hiperfosfatemia:


a) R/. Insuficiencia renal

479) Paciente con deshidratación hipertónica e hipertensión:


a) R/. SSN 0.9%

480) En la Glomerulonefritis Aguda post-infecciosa disminuye la tasa de filtración:


a) R/. Por alteración en el coeficiente de ultrafiltración

481) Un ejemplo de hipokalemia con aldosterona alta y osmolaridad baja es:


a) Hiperaldosteronismo primario

482) Manifestaciones cardíacas. El problema médico más grave de la hiperkalemia es la


cardiotoxicidad. Los cambios en el ECG producidos por los niveles altos de potasio son
bastante constantes. A medida que aumentan los niveles se aprecian los siguientes cambios:
a) Ondas T picudas (con intervalo QT normal o ligeramente reducido)
b) Prolongación del intervalo PR con depresión de ST
c) Desaparición progresiva de la onda P
d) Bloqueo cardíaco progresivo
e) Arritmias ventriculares
f) Paro cardíaco .

483) La hipokalemia produce:


a) mayor negatividad del potencial de reposo.

484) Exceso del kayaxelate en el tratamiento de hiperkalemia puede producir:


a) constipación.

485) Hipofosfatemia:
a) aumenta el calcitriol.

486) Consecuencia metabólica de hipofosfatemia severa:


a) miocardiopatía

487) Causa de hiperfosfatemia:


a) insuficiencia renal

488) El Síndrome Nefrósicose caracteriza por:


a) proteinuria mayor o igual de 3g y albúmina sérica menor o igual de 3g/dl.

489) En el Síndrome Nefrósico pueden encontrarse:

231
a) cilindros céreos

490) Urémico agudo, oligúrico, acidótico, deshidratado puede presentar:


a) osmolaridad urinaria de 250mosm/kg

491) La falla renal crónica avanzada cursa con:


a) brecha aniónica elevada

492) Uno de los indices de falla renal aguda es:


a) Uosm/Tosm

493) Hormona producida en los riñones que disminuye en la falla renal cronica:
a) ADH
b) Noradrenalina
c) Peptido natriuretico (segun arjona)
d) 1,25 (OH)2 Vit D.

494) Las calcificaciones metastásicas se deben:??


a) Aumento del producto Ca, P
b) Aumento del producto Ca, Mg
c) Hipoparatiroidismo
d) Hiperparatiroidismo 1ario

495) Causa de anuria:


i) Pielonefritis unilateral
ii) Necrosis tubular aguda
iii) Enf cambios minimo
iv) Necrosis cortical

496) la etapa de la insuficiencia renal cronica con tasa de filtración glomerular de 60-89 ml/min
es la:??
i) 1
ii) 2
iii) 3
iv) 4

497) La FENA es la relación entre:


i) Oferta tubular/Na reabsorbido x 100
ii) depuración de Na/ depuración de creatinina x 100
iii) oferta tubular/ Na excretado x 100.
iv) Na excretado/ Na reabsorvido x 100.

498) En la nefritis tubular intersticial aguda se pierde la capacidad de poder concentrar la orina
debida a:??
i) Daño medular
ii) Daño glomerular
iii) Daño tubular proximal
iv) Daño de la porcion cortical del tubulo colector

499) El efecto biologico mas importante de la calcitonina es, excepto:


i) Responder a la hipocalcemia
ii) Aumenta la calciuria
iii) Estimular la resorcion osea osteoclastica
iv) Utilizada en el tx de la hipercalcemia.

500) Favorece la entrada de K al espacio intracelular, excepto:


i) Agonista adrenergico
ii) Estimulacion adrenergico
iii) Insulina
iv) Alcalosis metabolica

501) Factores que afectan el balance hídrico en el post-operatorio, excepto:


i) Fármacos
ii) Calcitriol
iii) Administración de liquidos hipotonico
iv) Aumento de la ADH

232
502) Las primeras manifestaciones clínicas de la hiponatremia son:??
i) neurológica
ii) gastrointestinal
iii) pulmonares
iv) musculares

503) En la hipernatremia hipertonica se da lo siguiente excepto:


i) Entrada de Na
ii) Salida de aa
iii) Ganancia de iones
iv) Disminución del agua cerebral

504) Manifestaciones clinicas de la hipercalcemia, excepto:


i) Taquicardia
ii) HTA
iii) Constipación
iv) Vomitos

505) En falla renal aguda por nefrotoxicidad:


i) Se conserva la membrana basal
ii) Se destruye la membrana basal
iii) Frecuente hay oliguria
iv) La creatinina no aumenta

506) Causa de hipomagnesemia:


i) alcoholismo

507) Na serico de 168 mEq/l 70 kg (utilize como % de agua al 50% y Na normal 140 mEq/l
i) 7 litros

508) Indicaciones de diálisis en falla renal aguda, excepto:


i) Encefalopatia uremica
ii) Hiperkalemia refractaria
iii) Ac. Metabolica refractaria
1. d. hipocalcemia

509) El KCL IV se puede asministrar asi, excepto:


a) a.infusion por linea periferica de 10 meq en 100 ml en 1hora
b) b. “ “ central de 20 meq em 50ml en 1 hora
c) c. “ “ central de 100 meq em 50ml en 1 hora
d) d. “ “ periferica de 20 meq en 20ml en 2 horas

510) Aumenta la excrecion urinaria de K, excepto:


a) Triamtereno
b) Bumetanida
c) Furosemida
d) Metolazona

511) Factores que regulan la secrecion distal de K, excepto:


a) Flujo tubular distal y aporte distal de Na
b) Excrecion de aniones no reabsorbibles
c) Aldosterona
d) PTH

512) Enfermo de 60 Kg con 60% de agua corporal, Posm 260 mOsm/kg, ¿Cuántos gramos de
NaCl aumentaria el Na en 10 mEq?
a. 21 gramos

b) 50. Conteniendo 10 g de MgSO4 al %, cuantos ml de sal necesitaria?


i) 20 ml

513) Causa de hiperfosfatemia:


514) Falla hepatica
515) Falla renal cronica
516) Falla cardiaca
517) Insuficiencia suprarrenal

233
518) Causas de hiperfosfatemia grave, excepto:
519) recuperacion de cetoacidosis diabetica.
520) Alcalosis respiratoria
521) Acidosis respiratoria.

522) Paciente con vómitos severos, oligurico con hipotensión arterial Una> 40…… 800
mOsm/Kg puede tener:
523) Insuciencia renal aguda.
524) Insuficiencia suprarrenal
525) Falla pre-renal
526) Exceso de mineralocorticoides

527) Paciente urémico crónico con deshidratación severa puede presentar lo siguiente excepto:
a) Densidad urinaria 1010.
b) Osmolaridad urinaria de 800 mOsm/Kg
c) K sérico aumentado
d) Oliguria.

528) La hipokalemia produce:


a. Aumento de la magnitud del potencial de reposo
529) 17. La PTH promueve todo lo siguiente excepto:
530) a aumento de la absorción intestinal de calcio
531) b- aumento de la absorción tubulár de fosfato..
532) c. aumento de la resorción ósea de fosfato
533) d- aumento de la resorción osea de calcio
534) e, disminución de la reabsorción de fosfato

535) 18. Un varon de 57 esta con hemodiálisis de mantenimiento para insuficiencia renal
cronica, ¿Cuál de las sig anomalías metabolicas podría ser anticipáda?
a. hipernatremia
b. hiponatremia
536) c.
537) d. exceso de vitamina D
538) e. hipoparatiroidismo

539) 19. Un px de 25años fue ingresado a UCI por lesiones graves en cabeza y fractura de la
base del Graneo, aprox X horas después de la lesion manifestaba poliuria, la OSMU 150, y la
de suero 350. Los líquidos IV fueron detenidos y 3 hors después la producción de orina y la
osm urinaria permanecieron sin cambio. Se administraron 5 unidades de vasopresina IV, la
osm de la orina se
540) incremento a 300. ¿Cuál es el dx mas probable?
(1) Diabetes insípida central
541) b. diabetes insípida nefrogenica
542) c. intoxicación acuosa
543) d. sobrecarga de solutos
544) e. S IADH

545) 55. indica falla renal aguda


i) U..N.. = 4.1
ii) UN = 1
546) c. U..N.. = 1/4i/
547) d. Gravedad especifica 1.026
548) e. Ninguna

549) 56. deshidratación por perdida de agua y densidad urinaria 1.003, creatinina 6 /
a. falla renal aguda parenquimatosa
ii) falla prerenal
a. falla renal obstructiva
b. falla renal cronica
c. no hay falla renal

550) 57. uno de los indices de falla renales


a. reabsorción de Na/oferta tubular de Na x 100
b. excresion urinaria deNa/oferta tubular de x 100
551) c.- oferta tubular de Na/excrecion urinaria de Na x 100 ~.--~
i) Una/Pna x 100

234
552) e. Ninguna

553) 58. el Sdr hepatorenal es causa de falla:


i) prerrenal
554) b. renal
555) c. póstrenal
556) d. falla cardiaca
557) e. daño tubular

558) 59. causa mas común de falla renal crónica:


(1) DM

559) 60- en la glomendonefritis aguda por infeccion, la causa, excepto


a) proteina mayor de 10 g
b) osmolaridad urinaria normal
560) c. hematuria
561 ) d. H T A
562 ) e. Edema

563) 61. falla renal cronica, excepto


a. PTH aumentado
b) Hipofosfatemia
c) Acidemia
564) d. Hipocalcemia
565) e. 1.25 OH (D3) disminuida

566) 62. EN la falla renal aguda siempre habra


a) Aumento de creatinina
b) fraccion de excrecion de Na aumentado
c) hiperkalemia
d) Una aumentado
e) Hipernatremia

567) 63. causa de falla prerenal


a. Sdr nefrotico
ii) obstrucción de ambos uréteres
iii) falla cardiaca
568) d. Grlomerulonefritis aguda post-infeciosa
569) e. Calculo renal

570) 64. en la nefritis tubular interticial se puede dar, excepto


a. h.ipernatremia
b. proteinuria de 1.5 g
c. normotension arterial
d. hipostenuria
e. hipocalcemia

571) 65. Sugiere glomerulonefritis:


a. leucocituia
572) b-cilindros eritrocitarios
i) anuria
573) d. -

574) 66 -causa de falla renal parenquimatosa


575) papilitis
576) calculo en pelvis derecha
577) estenosis unilateral de arteria renal
578) cistitis hemorragica

579) 2--En el riñón la hormona paratiroides:


580) Aumenta la reabsorción tubular de calcio y disminuye la de fósforo

581) 3- La hipercalcemia produce


582) Potencialización de los efectos de la hipercalemia

583) 36- La hipomagnesemia produce.

235
a) Vasodilatación periférica

584) 36-De los factores: nefrotoxicidad./ -La buena hidratación

585) 37 - En la concentración sérica para calcular:: 8,8

586) 38- la velocidad de administración de potasio: 40 meq

587) 39-hipocalcemia produce : fosfaturia

588) 40-La causa de hipertensión con hipocalemia: Síndrome de Bartter=:,

589) 41-En paciente con vomito severo hipotensión arterial o ir a la sar con mas de 40 meq
590) Insuficiencia suprarrenal;

591) 42-La ,administración de sol isotónica aumento de vol extracelulares

592) 43-Glomerulonefritis post estreptococcica Altera el cociente de ultrafiltración

593) 44-En-el túbulo` intersticial se puede dar: La proteinuria de 4 g al día.

594) 25. Cual de las siguientes opciones esta contraindicada en el tratamiento de la


hiperkalcmia
595) a, gluconato de calcio
596) b. aldactone
597) c, kayaxelate
598) d. Bicarbonato de sodio e. Todas las anteriores

599) 43. Rotando en sala de medicina interna le piden que evalue un paciente con HIV el cual
porpresentar una meningitis por criptococci recibe anfotericina B; dicho este
medicamento buscaría como complicación:
a. aparicion de diarrea
b. aumento de la presion arterial
(2) hipokalemia
600) d . cefalea
601) e. dolor toráxico

602) 81- donde se halla el calcio en el organismo:


a) 50% sangre
b) 20 % proteínas
c) 99% hueso

603) aumento de PTH


(1) aumento de la absorción intestinal de fosfato
(2) disminuye fosfatemia
(3) aumento de vitamina D

604) 9- una solucion isotonica


a) aumenta el volumen extracell

605) 10- la vel de administración de potasio en caso de riesgo para la vida


606) a—40 meq por hora

607) 11- Px cronico con deshidratación severa puede presentar los sig uiente
excepto:
608) a-osmolaridad urinaria de 800 mosmol/Kg

609) 12- Px con vomito abundante, HTA, sodio urinario en 6, oligurico, Osmolaridad
de 800 presente,
a) insuficiencia suprarrenal

610) 13- Px uremico cronico con vomito severo hipotension arterial orina al az ar,
muestra sodio mas de 40, Osmolaridad de 800 puede tener :
a) Falla suprarrenal

611) Cuándo disminuye el fosfato?


612) R. Aumento de PTH

236
613) Ejemplo de hipernatremia hipervolémica:
614) R. Aldosteronismo primario

615) La hipocalemia produce:


616) R. Aumento de la magnitud del potencial de reposo

617) La depleción de agua produce, excepto:


618) R. Salida de electrolitos del cerebro

619) Velocidad de administración de KCl en caso de urgencia:


620) R. 40 mEq/h

621) Cambios en EKG de la hipermagnesemia es igual:


622) R. Hipercalemia

623) Enfermo de 60 Kg con 60% de agua corporal, osmolaridad plasmática 260 mosmol/Kg,
¿cuántos gramos de NaCl aumenta el Na en 10 mEq en 24 horas?
624) R. a) 15
a) 21
b) 30
c) 19

625) Uso de Calcio en tratamiento de hipercalemia para:


626) R. Modificar el Potencial de Membrana

627) Hipermagnesemia severa produce:


628) R. Paro respiratorio

629) Venoclisis de 1L de D/A 5% conteniendo 10 mg de MgSO4 y al 50%:


630) R. 20mL

631) La hipocalcemia:
632) R. Prolongo QT a expensas del segmento ST

633) Tratamiento de urgencia de hipercalcemia:


634) R. SSN 0.9%

635) Causa de hiperfosfatemia:


636) R. Insuficiencia renal

637) Adaptación cerebral, hipertonicidad, excepto:


638) R. Pérdida de iones

639) En la adaptación cerebral, la hipotensión da lo siguiente, excepto:


640) R. Ganancia de aminoácidos
641) En ausencia de síntomas de hipernatremia, el sodio debe ser disminuido a razón de:
642) R. 0.5 mEq/L/h

643) Para corregir un sodio cerca de 185 mEq se administró en 86 horas, 66 letras de D/SS 5%.
¿cuál es la velocidad de administración de la venoclisis?
644) R. 69 mL ¿?????

645) Manifestación clínica de hipercalcemia, excepto:


646) R. a) Hipotensión arterial
a) b) Bradicardia
b) Nauseas
c) Anorexia

647) Consecuencia de hipofosfatemia, excepto:


648) R. a) No afecta al SNC
a) b) Disfunción plaquetaria
b) Cardiomiopatía
c) Osteomalacia

649) Causa de hipofosfatemia, excepto:


650) R. a) Falla renal

237
a) b) Recuperación de cetoacidosis diabética
b) Alcalosis respiratoria
c) Quemaduras graves

651) Causa de HTA con hipocalemia, aldosterona-renina baja:


652) R. Síndrome de Cushing

653) Causas de HTA con hipocalemia y renina alta:


654) R. Hipertensión maligna

655) En una deshidratación severa hipertónica con hipotensión arterial severa, la prioridad inicial
es administrar:
656) R. SSN 0.9%

657) En la secreción inapropiada de ADH se da lo siguiente, excepto:


658) R. No hay expansión de volumen

659) Tejido con mayor contenido de K+:


660) R. a) Hígado
a) b) Músculo
b) Eritrocito
c) Hueso
661) La insuficiencia renal
funcional de la cirrosis hepática es debida a:
662) R. Vasoconstricción de
las arterias renales

663) 23. Px con IRC tine una taza de filtración glomerular de menor de 25 ml/min ;aunque
hay yuna tabla de las etapas en esta enfemedad crónica.
664) Estas osn las TFG:
1. 90 en riesgo
2. 20 ( con TFG normal o incrementada)
3. 60 -89
4. 30 – 59
5. 15-29
b) Menor a 15 diálisis.

665) 24. Factor que altera el balance hídrico en el periódo post- operatorio:
666) Disminución de la ADH (Excepto)

667) 25. Un ejemplo de hipokalemia con aldosterona alta y osmolaridad baja es:
668) Hiperaldosteronismo primario

669) 26.Ejemplode hipernatremia hipervolémica: Hiperaldosteronismo primario

670) 27.Hiperkalemioa son alteraciones del EKG no es necesario administrar gluconato de


calcio se colocará un antagonista de K asíe l Px no cae en paro y estoes si tiene
manifestación en el EKG.

671) 28. La fracción de excreción de sodio estará aumentada en falla cardíaca.

672) 29.En la hiperkalemia: hay menos diferencia entre el potencial de reposo y el umbral.
673) Manifestaciones cardíacas. El problema médico más grave de la hiperkalemia es la
cardiotoxicidad. Los cambios en el ECG producidos por los niveles altos de potasio son
bastante constantes. A medida que aumentan los niveles se aprecian los siguientes cambios:
674) Ondas T picudas (con intervalo QT normal o ligeramente reducido)
675) Prolongación del intervalo PR con depresión de ST
676) Desaparición progresiva de la onda P
677) Bloqueo cardíaco progresivo
678) Arritmias ventriculares
679) Paro cardíaco
680) Las ondas T picudas constituyen el dato en el ECG más constante en la hiperkalemia.
681) Efectos neuromusculares. El primer signo neuromuscular de la hiperkalemia suele ser la
aparición de parestesias seguidas de debilidad progresiva de varios grupos musculares. Si el
cuadro se agrava se observa cuadriplejia fláccida. Las funciones cerebrales y de los pares

238
craneanos se conservan y la parálisis de la musculatura respiratoria puede ocurrir, pero es
excepcional.
682) 30. La infusión de potasio sin riesgo para la vida del PX: es de 40 meq; porque
683) I lb de solución salina: debe llevr de 20 a40 mEq de KCl a una velocidad de 10 a 20 mEq
por hora; si aumenta la velocidad puede provocar un bloqueo A-V. Se recomienda una
Velocidad mayor si la hipokalemia es muy severa.
684) Sis e usa venas priféricas es:
685) 10 meq en 100 cc de sol.salina para pasar en una hora
686) 20 meq en 200 cc de sol.salina para pasar en una hora
687) En vena central:
688) 20 meql en 50 cc pasarla en una hora
689) 40 meq en 100cc pasar la en una hora
690) 31.Los indicios de falla renal aguda son:
691) (Na filtrado/Na excretado) x 100%
692) 32.Na urinario menor de 10: falla cardíaca, presentará edema estará hinchado y lo puede
llevar a una I.C.C. ( Esto lo saque de Medline: Los valores normales generalmente son de 15 a
250 mEq/L/día, dependiendo del estado de deshidratación y la ingesta diaria de sodio en la
dieta. Los rangos de los valores normales pueden variar ligeramente entre diferentes
laboratorios. Nota: mEq/L/día = miliequivalentes por litro por día.)
693) 33.Causa de enuresis: necrosis cortical difusa.
694) 34.En SIDH la concentración urinaria es menor de 20.
695) 35. Deshidratación pura de agua excepto: hay salida de electrolitos del cerebro.
696) 36.Urémico crónico con deshidratación hipertónica severa y acidótico presentará lo
siguiento excepto: osmolaridad urinaria de 8mm mosmol/kg.
697) 37. Px con vómitos abundante, hipertensión arterial; Na urinario=6, oliguria, osmolaridad
de 800 presenta: insuficiencia suprarrenal.
698) 38. Factor que predispone a la hipernatremia son intoxicación hídrica.
699) 39. Se presenta en acidosis tubular renal proximal excepto: NaHCO3 y lo alfa antagonistas.

1. En la Hiperkalemia:
ii) R/. Hay una menor diferencia entre el potencial de reposo y de umbral

700) 2. La hipokalemia en un cirrotico puede desencadenar:


i) R/. Coma Hepático

701) 3. La hipermagnesemia puede producir:


i) R/. Paro respiratorio

702) 7. Tratamiento de hiprecalcemia:


i) R/. SSN 0.9%

703) 8. La hipofosfatemia son causas, excepto:


a. Aumento de la PTH
b. Aumento de la absorción intestinal de fosfato
c. Disminuye la fosfatemia
d. Aumento de la vitamina D

704) 9. Una solución isotónica:


i) R/. Aumenta el volumen extracelular

705) Paciente crónico con deshidratación severa puede presentar lo siguiente excepto:
a) Densidad urinaria 1010
b) Osmolaridad urinaria de 800 mOsm/kg
c) K sérico aumentado
d) Oliguria

706) 15. Paciente con vómito abundante, hipertensión arterial, sodio urinario de 6,
oligúrico, osmolaridad de 800 presenta:
i) R/.Insuficiencia suprarrenal
707) Paciente urémico crónico con vómitos severos, hipotensión arterial, orina al azar
muestra Na mas de 40, osmolaridad de 800 puede tener:
708) Falla suprarrenal

709) 16. Todo lo siguiente es cierto acerca de Colitis Ulcerosa, EXCEPTO:

239
i) R/. No está en relación las alteraciones inmunológicas.

710) 17. En quien predomina la enfermedad de Hígado Graso:


i) R/. Sexo masculino

711) 18. Paciente con Insuficiencia renal crónica, con K sérico de 6, leve academia, EKG
normal, se trata con:
i) R/. Kayetzalate

712) 22. La hopikalemia predispone, excepto:


i) R/. Amoniogénesis

713) 25. Cual de las siguientes es causa de hiperfosfatemia:


i) R/. Insuficiencia renal

714) 26. Paciente con deshidratación hipertónica e hipertensión:


i) R/. SSN 0.9%

715) 29. En la Glomerulonefritis Aguda post-infecciosa disminuye la tasa de filtración:


i) R/. Por alteración en el coeficiente de ultrafiltración

716) 30. Causa de Insuficiencia preerenal, excepto:


i) R/. Aneurisma Disecante de la Aorta

(a) El magnesio está presente o


aumentado en:
ii) R/. Eritrocito

717) Causa de hipomagnesemia:


718) estreñimiento
719) alcoholismo
720) falla pre-renal
721) IRA (insuficiencia renal aguda)

722) La hipocalcemia en la falla renal aguda puede ser asintomática por la:
723) acidemia
724) hipoalbuminemia
725) Deficiencia de PTH
726) Alcalemia

727) La hipofosfatemia causa, excepto:


728) Aumento de vit. D
729) Aumento de PTH
730) Disminución de la excresión de fosfato
731) Aumento de la absorción de fosfato a nivel intestinal

732) Un pte con diuresis de 250 ml al día con osmolaridad plasmática (presión osmótia) y
osmolaridad urinaria de 300 mosm/l, el dx probable e:
733) Insuficiencia suprarrenal
734) Insuficiencia renal
735) Nefritis túbulo intersticial crónica
736) Deshidratación hipertónica

737) Pte con oliguria, hiperkalemia, hiponatremia, creatinina 0.8 mg/dl y CPK normal: Esto
puede corresponder a:
738) IRA
739) Insuficiencia suprarrenal
740) Síndrome hepatorrenal
741) Rabdomiolisis

742) No está indicado en caso de hiperkalemia:


743) Aldactone

744) Mujer con hiperemesis grávida:


745) Alcalosis hipoclórica

746) En causa de hipomagnesemia:

240
747) Alcoholismo.

748) En el metabolismo del calcio está implicado, menos:


749) Insulina

750) Hiponatremia, hipotónico, normovolémico:


751) SIADH

752) Fórmula del clearence de creatinina:


753) clearence = 140 – edad x peso en Kg
a. 72 x creatinina

754) Su infección produce litiasis renal:


755) proteus sp.

756) Fórmula de irritabilidad muscular:


757) IR = Na + K + OH
(a) ( ) + Mg + H

758) Con respecto al síndrome diarreico, nombre 2 tipos osmóticos:


759) Laxantes osmóticos (Mg2+, PO4, SO4)
760)
Carbohidratos no absorbibles (sorbitol, lactulosa).

761) Nombre dos síndromes diarréicos tipo secretora:


762) Consumo crónico de etanol.
763)
Obstrucción intestinal incompleta.

764) Nombre 2 sindromes diarréicos motores con aumento de motilidad:


765) Hipertiroidismo
766)
Prostaglandinas.

767) Indica falla renal aguda:


768) U.N. = 4.1
769)
U.N. = 1
770)
U.N. = ¼
771)
Gravedad específica: 1.026
772)
Ninguna.

773) Deshidratación por pérdida de agua y densidad urinaria 1.003, creatinina 6:


774) falla renal aguda parenquimatosa
775)
Falla pre-renal
776)
Falla renal obstructiva
777)
Falla renal crónica
778)
No hay falla renal.

779) Uno de los índices de falla renal es:


780) Reabsorción de Na/ oferta tubular de Na x 100
781)
Excreción urinaria de Na/ oferta tubular de Na x 100
782)
Oferta tubular de Na/ excresión urinaria de Na x 100 **

783)
Una / pna x 100
784)
Ninguna

785) El síndrome heatorrenal es causa de falla:


786) pre-renal
787)
renal
788)
posrenal
789)
falla cardiaca
790)
daño tubular

791) Causa más comun de falla renal cronica:


792) Diabetes melitus
793)
HTA
794)
Glomerulonefritis

241
795)
Enfermedad poliquística renal
796)
Vasculitis

a) 36. En la glomerulnefritis aguda por infección, la causa, excepto:


a. proteínas mayor de 10g
b) b. osmolaridad urinaria normal
c) c. hematuria
d) d. HTA
e) e. Edema

Renal

1. Paciente urémico crónico con deshidratación severa puede presentar lo


siguiente excepto:
f) Densidad urinaria 1010.
g) Osmolaridad urinaria de 800 mOsm/Kg
h) K sérico aumentado
i) Oliguria.

2. Se puede usar en la hiperfosfatemia de la IRC;


a. carbonato de Ca2+ b. Acetato de Ca2+ c. Resticción de fósforo d. Paricalcitol e.
Todas las anteriores

3. Causa de falla renal durante el embarazo, excepto:


a. Aborto séptico b. Desprendimiento prematuro de la placenta c. Eclampsia d.
Hígado graso e. Obstrucción uretral derecha

4. La anemia de la falla renal crónica se debe principalmente:


a. deficiencia de hierro b. Deficiencia de folato c. deficiencia de cianocobalamia
d. Deficiencia de eritropoyetina e. Todas las anteriores

5. Una de las alteraciones frecuentes de la falla renal crónica se deben principalmente:


a. prurito b. Xerodermia c. Calcifilaxis d. Hiperqueratosis folicular e. N/A.

6. Factores de riesgo para desarrollar IRC por el uso de medio contraste excepto:
a. edad avanzada b. ICC severa c. Uso de agentes potencialmente nefrotóxicos
d. NO SE VE, tiene algo que ver con la cantidad de medio de contraste que usan el
volumen máximo debe ser: 5mL x peso en Kg/ creatinina sérica.

7. Puede haber pérdida de la capacidad de concentrar la orina, excepto:


a. diabetes insípida nefrogénica b. Diabetes insípida central c. Anemia falciforme
d. Pielonefritis aguda severa e. GMN aguda

8. El término histopatológico GMN proliferativa intracapilar y exudativa corresponde


a:
a. Sdme. Nefrósico b. Sdme. De Goodpasture c. Granulomatosis de
Wegener d. GMN post-infecciosa e. Sdme. Nefrosico de cambios mínimos

9. Sugiere trastorno del túbulo intersticial:


a. pérdida de la capacidad de concentrar la orina b. HTA c. Edema d. Proteinuria >2g e.
Macrohematuria

10. Causa de falla prerrenal:


a. Necrosis tubular aguda b. Necrosis cortical c. Papilitis necrotizante d. Insuficiencia
cardíaca severa e. N/A

11. sedimento urinario en el síndrome nefrítico:


a. cilindros anchos b. Eritrocitos dismórficos c. Cilindros de eritrocitos d. B y C
e. Todas las anteriores

242
12. El síndrome nefrósico de cambios mínimos se caracteriza por:
a. Proliferación intracapilar
b. Engrosamiento de la membrana basal
c. Hiperplasia mensangial
d. Proliferación endotelial
e. Fusión de los pedicelos de los podocitos

13. En una nefritis tubulointersticial aguda, se puede encontrar en la orina:


a. Abundantes eritrocitos
b. cilindros anchos
c. Eosinófilos
d. Gravedad específica de 1.030
e. Cuerpos óvalos

14. En el tx no dialitico de un paciente con falla renal cronica, con oliguria.hinchado


hipertenso y con sodio de 120mM/L
RESTRICCION DE LIQUIDOS Y RESTICCION DE SAL. DIURETICOS DE ASA

15. En la falla renal cronica con oliguria se puede presentar excepto:


a. Exceso de PTH b. Normokalemia c. Hiperfosfatemia d. Hipocalcemia
e. Anemia normocítica normocrómica.

16. Indicaciòn para dializar.


a. Hipocalcemia severa
b. Náuseas
c. Edema de MI
d. pericarditis
ademas tambien: sobrecarga de volumen que no responde a
diureticos,hiperpotasemia.acidosis metabolica que no responde,encefalopatia,).
e. Pérdida de la capacidad de concentrar la orina

17. Pensar en obstruccion de vias urinarias en caso de :


a. Poliuria b. Oliguria c. Anuria d. Polaquiuria e. Mayor aumento de creatinina
sérica que de urea.

18. Disfuncion renal debida a hipercalcemia excepto:


a. poliuria
b. acidosis tubular renal tipo IV
c. nefrolitiasis
d. nefrocalcinosis
e. diabetes insipida nefrogenica.

61) Primera causa de Falla renal crónica:


a.HTA
b.Glomerulopatias
c. Diabetes Mellitus
d.Nefritis tubulo intersticial

62) Uno de los 243 necesita de falla renal aguda es:


a) Uosm/Tosm

63) Causa de anuria:


a) Pielonefritis unilateral
b) Necrosis tubular aguda
c) Enf cambios minimo
d) Necrosis cortical

64) En la nefritis tubular intersticial aguda se pierde la capacidad de poder


concentrar la orina debida a:
a) Daño medular
b) Daño glomerular

243
c) Daño tubular proximal
d) Daño de la 244ecesit cortical del tubulo colector

65) En falla renal aguda por nefrotoxicidad:


a) Se conserva la membrana basal
b) Se destruye la membrana basal
c) Frecuente hay oliguria
d) La creatinina no aumenta

66) Indicaciones de diálisis en falla renal aguda, excepto:


a) Encefalopatia 244ecesit
b) Hiperkalemia refractaria
c) Ac. Metabolica refractaria
d) Hipocalcemia

67) En IRA se mide todo menos:


a) Uosm/Posm x100
b) Na excretado/ Na filtrado x200
c) Na filtrado/ Na excretado x100
d) Na urinario >40
e) Ninguna de las anteriores

68) En el síndrome nefrosico puede encontrarse


cilindros:
a) cereos
b) hematicos
c) leucocitarios
d) epiteliales
e) mixtos leucocitario y hematicos

Paciente masculino de 33 años llega a su consulta por presentar fiebre, un pequeño


rash, signo de Jordano positivo; el medico de turno le envía un urinalisis que revela
proteinuria, hematuria y múltiples leucocitos; dentro de su diagnostico diferencial,
debe estar una de las siguientes patologías, cual:
m. Nefritis Intersticial Crónica.
n. Nefritis Intersticial Aguda.
o. Síndrome Nefrosico.
p. Fracaso Renal Agudo.

Dentro de las Glomerulopatias, existen algunas, que se pueden presentar con


Síndrome Nefrosico, y otras con Síndrome Nefrítico, cual de las siguientes se suele
presentar con Síndrome Nefrítico:
m. Cambios Mínimos.
n. Nefropatia IgA.
o. Nefropatia Membranosa.
p. Diabetes.

En un paciente de 65 años, que desarrolla un Sd. Nefrítico Agudo, la etiología mas


común causando de la Glomerulopatia es:
m. Vasculitis.
n. GN rápidamente progresiva.
o. Nefropatia IgA.
p. LES.

Cual de las siguientes Glomerulopatia, siempre desarrolla Insufiencia Renal:


m. Cambios Mínimos.
n. Membranosa.
o. Rápidamente Progresiva.
p. Mesangiocapilar.
Creo…

Las características del Síndrome Nefrotico son:

244
s. _hipoalbuminemia mayor de 3g /dl.
t. _3.5 g de preoteina en orina en 24 h en 1.73m2 de superficie
corporal____________________________.
u. hiperlipidemia.
v. ___________________edema
w. _hipertension
x. cilindro cereo

En un paciente con diagnostico de Glomerulonefritis post estreptococcica, el


diagnostico serologico se basa en el hallazgo de anticuerpos:
n. Antiestreptolisina O.
o. Antiestreptolisina A.
p. Antiestreptolisina P.
q. Antiestreptolisina K.

En la Nefiritis secundaria al LES, se suelen encontrar positivos los:


n. ANA.
o. ANCA c.
p. ANTI LU
q. ANCA p.

Si un paciente es diagnosticado con Falla Renal Crónica, el siguiente punto es el


estadiage del mismo, si tiene una TFG en 82 ml/minuto, esta en estadio:
p. I
q. II
r. III
s. IV
t. V

Las características de una muestra de orina con sedimento de tipo nefrítico, son:
p. microhematuria
q. cilindro no hematico

Mencione algunos parámetros de laboratorio, para pensar en una IRC Renal:


l. creatinina
m. hemoglobina
n. BUN
o. Aumento del calcio y del fósforo

El Gasto cardiaco normal es de 5 – 6 litros por minuto; de los cuales, un 25% le


corresponde al riñón, en una relación de:
m. 1200 ml/min.
n. 1300 ml/min.
o. 1400 ml/min.
p. 1500 ml/min.

Si un paciente, diagnosticado con IRC, presenta ademas una acidosis metabolica


hipercloremica, usted sabe que este paciente debe tener una Brecha Anionica:
m. Normal.
n. Aumentada.
o. Disminuida.
p. Ninguna de las anteriores.

Este mismo paciente, por tener estas características clínicas, debe encontrarse en
estadios de la IRC:
n. Avanzados.
o. Iniciales.
p. No es relevante.
q. Ninguna de las anteriores.

Para que por una obstrucción, aumente la creatinina en IRA, se deben haber perdido:
m. ¼ partes de la masa renal.

245
n. 2/4 partes de la masa renal.
o. ¾ partes de la masa renal.
p. 4/4 partes de la masa renal.

La lesión histologica mas común de IRA es:


m. Necrosis cortical bilateral.
n. Necrosis tubular aguda.
o. Nefritis intersticial.
p. Papilitis necrotizante.

la mas peligrosa y temida es:


n. Necrosis cortical bilateral.
o. Necrosis tubular aguda.
p. Nefritis intersticial.
q. Papilitis necrotizante.

Cual de las siguientes, es la principal causa de IRA, en la maternidad.


q. Abortos infectados.
r. Fetos muertos retenidos.
s. Desprendimiento prematuro de membrana.
t. Eclampsia.

Un paciente de 35 años con insuficiencia renal crónica secundaria a pielonefritis


crónica recibe un trasplante renal de cadáver con el que compartía dos identidades en
A y B y una en DR. Recibe tratamiento inmunosupresor con ciclosporina A y
corticoides a dosis estándar. En el postoperatorio inmediato se observa buena diuresis
y no es necesario el tratamiento sustitutivo con hemodiálisis. En el 5º día de evolución,
el paciente presenta fiebre de 38ª, TA de 180/110, oliguria y disminución en la
concentración urinaria de sodio. El diagnóstico más probable sería:

a. Crisis hipertensiva.
b. Infección respiratoria.
c. Pielonefritis aguda del injerto renal.
d. Recidiva de su enfermedad renal.
e. Rechazo agudo del injerto renal.

Causa más comun de falla renal cronica:


u. Diabetes melitus
v.
HTA
w.
Glomerulonefritis
x.
Enfermedad poliquística renal
y.
Vasculitis

Deshidratación por perdida de agua y densidad urinaria 1.003, creatinina 6 /


a. falla renal aguda parenquimatosa
b- falla prerenal
c. falla renal obstructiva
d. falla renal cronica
e. no hay falla renal

Urémico crónico con K 6.5 con leve academia, sin trastorno muscular y EKG normal,
tratamiento:
o Kayexalate.

La IRC ocaciona hiperfosfatemia permanente cuando la TFG esta por debajo de:
o 25 ml/min

La fracción de excreción de Na esta aumentada en:


o Falla cardiaca. También esta aumentada en FRC, de causas renales.

Un varon de 57 esta con hemodiálisis de mantenimiento para insuficiencia renal


cronica, ¿Cuál de las sig anomalías metabolicas podría ser anticipáda?

246
a. hipernatremia
b. hiponatremia
c. Osteomalacia
d. exceso de vitamina D
e. hipoparatiroidismo

uno de los indices de falla renales


a. reabsorción de Na/oferta tubular de Na x 100
b. excresion urinaria de Na/oferta tubular de x 100
c.- oferta tubular de Na/excrecion urinaria de Na x 100 ~.--~
d- Una/Pna x 100
e. Ninguna

falla renal cronica, excepto


a. PTH aumentado
b- Hipofosfatemia
c- Acidemia
d. Hipocalcemia
e. 1.25 OH (D3) disminuida

TFG < 15ml/min corresponde al estadio de la enfermedad renal cronica:


5

Hormona producida en los riñones que disminuye en la falla renal cronica:


j) ADH
k) Noradrenalina
l) Peptido natriuretico (segun arjona)
m) 1,25 (OH)2 Vit D. (disminuye a partir del estadío 3)

la etapa de la insuficiencia renal cronica con tasa de filtración glomerular de 60-89


ml/min es la:
i) 1
ii) 2
iii) 3
iv) 4

Causa de hipomagnesemia
n) cirrosis avanzada
o) falla renal cronica
p) falla cardiaca
q) constipacion
r) acidosis respiratoria

Causa renal de hipomagnesemia


s) falla renal cronica
t) esteatorrea
u) desnutricion severa
v) uso de diureticos de asa
w) infeccion urinaria

Si un paciente es diagnosticado con Falla Renal Crónica, el siguiente punto es el


estadiage del mismo, si tiene una TFG en 50 ml/minuto, esta en estadio:
a. I
b. II
c. III
d. IV
e. V

En pte con falla renal crónica que ha sido transplantado esperamos encontrar:
f. osteomalacia.

Enfermedad Crónica de los Riñones:

247
a) Nefrolitiasis unilateral b) Enfermedad de Cambios Mínimos
c) Nefropatía Diabética d) Carcinoma de Riñón e) N/A

Hormonas que disminuyen en la falla renal crónica, excepto


a) 1,25 dihidroxi vitatamina D
b) eritropoyetina
c) paratohormona
d) foliculo estimulante

Factores que conducen a la progresión de falla renal crónica


a) hiperlipidemia
b) toxinas urémicas
c) proteinuria
d) b y c
c) todas

De las siguientes hay una pregunta que no es diagnostico del S. de Sjogren:


o Ecografia reumatica
o Biopsia parotidea
o gammagrafia parotidea
o determinación de lizosima salival
o hialografía parotidea

el numero de celulas del liquido sinovial es bajo en


o artrosis

señalar las enzimas que no son de utilidad en el dx de poliomiositis


o transaminasa glutamicopirubica

que articulación es la que presenta artrosis con menos frecuencia


o tobillo

vasculitis de celulas gigantes


o takayasu
o temporal

se ha visto muy util en el dx de poliarteritis nodosa


o arteriografía

en la crisis de gota se debe


o a la precipitación de urato

la síntesis de calcitonina aumenta cuando


o esta aumentado el calcio serico

la vasculitis asociado a hepatitis B

248
o panarteritis nodosa

indicacion mas util del analisis del liquido sinovial


o artritis septica y por microcristales

hombre de 22 años se queja de dolor bajo de espalda y rigidez que empeora al levantarse y
mejora con el ejercicio, en la exploracion se le encuentra limitada en las articulaciones
sacroileacas de la columna lumbar, HLAB27 positiva, cual es la manifestación
extraarticular
o iritis aguda

un paciente la cual se encuentra en estudio por artralgia dentro de los laboratorios le sale
positivo el anticuerpo antihistona
o lupus medicamentoso

los marcadores bioquimicos por la osteoporosis estan indicados solamente


o seguimiento de la efectividad del tx de osteoporosis en los pacientes

citar que afección no es una espondiloartritis seronegativa


o osteitis condensante del iliaco
citar que enfermedad no presenta fenómeno de Raynoud
o artritis psoriasica

dx con artritis reumatoide, leucopenia y esplenomegalia


o síndrome de felty

el FR no se hay en
o hipotiroidismo

en el lupus medicamentoso son muy poco frecuentes


o erupcion

NEFRO
95. Señale la respuesta correcta referida a las alteraciones del examen de la orina:
1. La presencia de cilindros hialinos es siempre patológica.
2. Los cilindros granulosos contienen albúmina e Inmnoglobulinas.
3. Los cilindros leucocitarios son típicos de Glomerulonefritis postestreptocócica.
4. Los cilindros hemáticos se presentan en cualquier discrasia sanguínea.
5. Un resultado negativo de presencia de nitritos en tira reactiva, excluye la existencia de baterias.
96. Paciente de 68 años diabética, que consulta por malestar general. En la gasometría venosa
destaca pH 7.25, Bicarbonato 15 mmol/ l (normal 24-28 mmol/l). Hiato anionico
(anión GAP): 11 mmol/l (normal 10-12 mmol/l). ¿Cúal de las siguientes entidades
NO descartaría como diagnóstico?:
1. Cetoacidosis diabética.
2. Insuficiencia renal crónica.
3. Acidosis tubular renal.
4. Ingesta de salicilatos.
5. Acidosis láctica.
97. Cuando un paciente bajo tratamiento diurético con tiazidas o furosemida incumple la
dieta y come más sal de la prescrita, el resultado análitico esperable es:
1. Mayor hipernatremia.
2. Mayor hiponatremia.
3. Mayor hiperpotasemia.
4. Mayor hipopotasemia.
5. Mayor acidosis.
98. En la uremia pre-renal:
1. El sodio en orina es superior a 60 mEq/l, la osmolaridad urinaria es superior a 500 mOsm/
kg H2O y la relación urea en orina/urea en plasma es superior a 8.
2. El sodio en la orina es inferior a 20, la osmolaridad urinaria es inferior a 200 mOsm/ kg H2O y la
relación entre urea en orina / urea es plama es inferior a 2.

249
3. El sodio en orina es inferior a 20 mEq/l, la osmolaridad en orina es inferior a 200 mOsm/ Kg H 2O y la
relación urea en orina / urea en plama es superior a 8.
4. El sodio en orina es inferior a 20 mEq/l, la osmolaridad urinaria es superior a 500 mOsm/ Kg H 2O, y
la relación entre la urea en orina y la urea en plasma es superior a 8.
5. El sodio en orina es superior a 60, la osmolaridad urinaria es superior a 500 mOsm/Kg H 2O, y la
relación urea en orina/urea en plasma es superior a 8.
99. Un hombre, adicto a drogas por vía parenteral, está ingresado por endocarditis infecciosa.
Durante su enfermedad presenta un cuadro de glomerulonefritis aguda. ¿Cuál de las respuestas es
INCORRECTA?:
1. Suele ser debida a inmunocomplejos.
2. No suele presentar piuria.
3. El complemento está descendido.
4. A veces produce síndrome nefrótico.
5. Suele evolucionar favorablemente al controlar la infección cardiaca.
100. Una mujer de 68 años acude al Servicio de Urgencias por malestar general que ha ido
progresando en los últimos 15 días, a partir de un episodio gripal. Ha notado disminución
progresiva del volumen de diuresis, edemas maleolares y dificultad respiratoria. Es hipertensa.
En la analítica destaca una creatinina plamática de 5 mg/dl, urea 180 mg/dl, Na 138 mEq/l, K 4.9
mEq/l. Las cifras de complemento son normales. Los anticuerpos anti-membrana basal son
negativos. En la orina presenta cilindros hemáticos, proteinuria de 1 g/l y microhematuria. Aporta
una analítica de un mes antes, sin alteraciones. ¿Cuál de los siguientes diagnósticos es más
probable?:
1. PAN microscópica.
2. Brote lúpico.
3. Glomerulonefritis aguda postinfecciosa.
4. Crioglubulinemia.
5. Enfermedad de Goodpasture.
101. Con respecto a la nefropatía diabética, señalar la respuesta FALSA:
1. La diabetes tipo 2 es la etiología más frecuente de insuficiencia renal terminal en el mundo occidental.
2. Más del 90% de los diabéticos tipo 1 desarrollan nefropatía a los 30 años del diagnóstico de diabetes.
3. La alteración renal más temprana es la hiperfiltración.
4. La existencia de microalbuminuria predice el desarrollo de nefropatía clínica.
5. La gran mayoría de los diabéticos tipo 1 con nefropatía tienen también retinopatía.

102. Un paciente de 75 años de edad, con historia antigua de hipertensión arterial, de hábito
asténico, consulta en la Urgencia del Hospital por un cuadro de dolor en fosa lumbar derecha con
hematuria de 6 horas de evolución. La tensión arterial es del 120/ 80 mmHg, y el paciente está
agitado, nauseoso y sudoroso, con el pulso irregular a 80 latidos por minuto. Analíticamente tiene
una urea de 50 mg/dl, creatinina 1 mg/dl, GOT 120 mU/ml, GPT 35 mU/ml, amilasa 120 mU/ml,
LDH 1100 mU/ml. En el sedimento urinario hay microhematuria. La radiografía de torax muestra
una discreta cardiomegalia a expensas de cavidades izquierdas y el electrocardiograma muestra
una frecuencia auricular de 250 latidos minuto, con una respuesta ventricular irregular a 75
latidos por minuto. ¿Cuál es, entre las siguientes, la decisión más terminante?:
1. Calmar ante todo el dolor y la ansiedad.
2. Hacer una ecografía abdominal.
3. Hacer un TAC espiral con contraste.
4. Acidificar la orina.
5. Alcalinizar la orina.

250
PARCIAL 2 2006 1

TOXICO
NEFRO
GASTRO

Una joven de 33 años de edad presenta fiebre de 39.4ºC, cefalea y rigidez de cuello.
Usted realiza el diagnostico de meningitis bacteriana y comienza con antimicrobianos.
Con la meningitis bacteriana ¿Cuál de los siguientes es un hallazgo probable en líquido
cefalorraquideo (LCR)?
a) Leucocitos entre 100 y 500 ml
b) Presión del LCR entre 100 y 120 mm H2O
c) Tinción de gram negativa
d) Glucosa >120 mg/dl
e) Valores de proteinas >45 mg/dl

En este joven adulto por lo demás sano ¿Cuál es el microorganismo más probable?
a) Streptococcus del grupo B
b) Stafilococcus aureus
c) Haemophilus influenzae
d) Streptococcus pneumoniae
e) Listeria monocytogenes

Un cuadro febril cuyas oscilaciones diarias de la temperatura son superiores a 1ºC sin
alcanzar nunca los valores normales, corresponde a la definición de:
a) Fiebre continua
b) Fiebre recurrente
c) Fiebre remitente
d) Fiebre intermitente
e) Fiebre héctica

¿De que enfermedad es característico la pupilas de Argyll-Robertson?


a) Tuberculosis
b) Lepra
c) Sífilis
d) Toxoplasmosis
e) Tetáno

** ¿Cuál es la triada semiológica clínica del síndrome meningitis?


a) Fiebre, cefalea y rigidez de nucal (según libro)
b) Fiebre, cefalea y vómitos
c) Fiebre, rigidez de nucal y signos encefálicos (según Arjona)
d) Cefalea, rigidez de nuca y signos encefálicos
e) Cefalea, vómitos y rigidez de nuca
Entre los múltiples procesos infecciosos que producen adenopatías cervicales no suelen
incluirse: TODAS
a) Mononucleosis infecciosa
b) Rubéola
c) Enfermedad por arañazo de gato
d) Tuberculosis
e) Faringitis estreptocócica

¿Cuál de los siguientes antibioticos es el que tiene menor difusión al LCR?


a) Clindamicina
b) Metronidazol
c) Cotromoxazol
d) Penicilina
e) Cefotaxima

Cuando un paciente ingresa por sepsis por Stafilococcus aureus adquirida en la


comunidad y sin foco aparente. ¿Qué tipo de proceso patologico debe se descartado?
a) Un absceso cutaneo oculto
b) Una neumonía -
c) Una osteomielitis vertebral -
d) Una endocarditis -
e) Un absceso renal

Con respecto a la salmonella Typhi. ¿Cuál de las siguientes afirmaciones es correcta?


a) Es la unica especie de salmonella que es patógena tanto para el hombre como para
los animales
b) La vía de contagio de salmonella para el hombre es orofecal
c) La infección por S. Typhi ocurre prácticamente solo en verano
d) El inoculo mínimo necesario de S. Typha para desencadenar una infección es muy
bajo de solo unas 1,000 colonias de bacterias viables. (no)
e) Más del 50% de los pacientes con fiebre tifoidea presentan manifestaciones
neuropsiquiatritas.

** ¿Cuál de los siguientes antibióticos es el menos eficaz para el tratamiento de la


uretritis gonocócica?
a) Ceftriaxona 500mg IM (125mg)
b) Cefixima 400mg oral en una sola dosis
c) Azitromicina 1 g una dosis
d) Ciprofloxacina 1 g en una dosis (500mg)
e) Cefuroxina 750 mg en una sola dosis

¿Cuántos días debe prolongarse el tratamiento antibiótico eficaz de una meningitis


meningococcica?
a) 1-3
b) 4-7
c) 8-10
d) 12-15
e) 18-21

¿En cual de las situaciones puede observarse serología falsamente positiva para Brucella?
Yersinia, francisella, vibrio, inmunizaciones
a) Sífilis
b) Nunca
c) Fiebre tifoidea
d) Personas vacunas contra cólera
e) Tuberculosis

Con respecto al cólera. ¿Cuál de las siguientes afirmaciones es correcta?


a) El cólera se trasmite tanto por la ingestión de agua contaminada como por el
contacto con animales infectados.
b) Existen aproximadamente unas 70 especies diferentes de vibriones, todas ellas
potenciales causas de cólera grave.
c) De forma característica, el periodo de incubación del cólera es siempre superior a
5 días. (potencialmente autolimitada 5 dias)
d) El cuadro clínico se presenta como un proceso diarreico grave, que alcanza su
acmé a las 24 horas del inicio y que se acompaña de vómitos y fiebre elevada.
(no)
e) El cólera tiene un tratamiento fácil, basada en la reposición adecuada de líquidos
y electrolitos. Aunque no es imprescindible, la administración de tetraciclina en
monodosis puede disminuir la duración y el volumen de perdida de líquidos.

Que concentración de bacilos tuberculosis en esputo es necesario para que la baciloscopia


(tinción de Ziehl) sea positiva.
a) 100 bacilos /ml
b) 1000 bacilos /ml
c) 10000 bacilos /ml
d) 100000 bacilos /ml
e) 1000000 bacilos /ml

El síntoma más común de la TBC pulmonar es:


a) Tos
b) Hemoptisis (en ocasiones)
c) Fiebre
d) Perdida de peso
e) Anorexia

Cual de las siguientes enfermedades puede confundirse con tuberculosis pulmonar (Son
tres las entidades que con mayor frecuencia deben diferenciarse de la tuberculosis
pleural: neumonía bacteriana, tromboembolismo pulmonar y neoplasia pleural).

a) Sarcoidosis
b) Enfermedad por legionella
c) Histoplasmosis pulmonar (no)
d) Enfermedad metastásica pulmonar
e) Ninguna de las anteriores

La siguiente droga requiere de co administración de piridoxina el tratar la TBC


a) Isoniacida
b) Rifampicina
c) Etambutol
d) Rifampicina
e) Estreptomicina

Nombre dos condiciones de tuberculos extrapulmonar en donde usaria corticoides


TBC cerebral (meningea)
TBC pericardica

Unas de las siguientes drogas para el tratamiento de la TBC se metabolizan por


acetilación
a) Isoniacida
b) Rifampicina
c) Etambutol
d) Rifampicina
e) Estreptomicina

La profilaxis de plasmodium falciparum resistente a la cloroquina se realiza con:


a) Cloroquina
b) Primaquina
c) Mefloquina
d) Quinina
e) Pirimetamina

De las siguientes aseveraciones una es cierta respecto a la amebiasis


a) La transmisión por artropodos es frecuente
b) Transmisión de persona a persona es importante
c) La infeccion pulmonar es la mas frecuente extraintestinal
d) La forma quistica es la infectante
e) Ninguna de las anteriores

La diarrea en la infección por ameba ocurre dentro de:


a) 2 dias
b) 4 dias (NO)
c) 8 dias
d) 10 dias
e) 15 dias

En el tratamiento con metronidazol del absceso amebiano hepatico usted considera que
puede tratarse de un absceso bacteriano sino hay respuesta al tratamiento a
a) Las 24 horas
b) 48 horas
c) 72 horas
d) 5 dias
e) 7 dias

El absceso hepático puede drenar principalmente a los siguientes tres espacios:


Retroperitoneal
Intratoracico
Pericárdico

En caso de agua contaminada con ameba. Para descontaminarla se debe de usar:


a) Cloro
b) Yodo
c) Fluor
d) Cualquiera de las anteriores
e) Ninguna de las anteriores

Para considerar fiebre de origen por determinar, además del tiempo de la fiebre esta debe
ser por lo menos de:
a) 37. ºC
b) 37.5
c) 37.8
d) 38.3
e) 39

*** En la fiebre de origen por determinar además de la causa mas frecuente ser la
infecciosa y considerar las neoplasias, la ficticia enumere otras tres causas
Medicamentos
Inflamatoria
___________

Los siguientes pacientes con fiebre mediana requieren tratamiento excepto:


a) Embarazada
b) Niños con convulsiones
c) Falla cardiaca (no)
d) Falla pulmonar
e) Fiebre > 40ºC

Cual de las siguientes entidades se puede confundir con una intoxicación por
organofosforados
a) Constipación
b) Hipotiroidismo
c) Hipertermia maligna
d) Intoxicación por hongos
e) Boca reseca
La hemodiálisis como tratamiento de intoxicación por barbitúricos esta indicado en
a) Coma en estadio III
b) Cuando el paciente esta en estupor
c) Al llegar al cuarto de urgencia para prevenir el coma
d) Edema agudo del pulmón
e) Nunca se realiza diálisis

Paciente masculino de 47 años de edad que acude al cuarto de urgencia por


sintomatología de dolor abdominal, nauseas, vómitos, diarrea, sudoración profuso con
mucha ansiedad, 3 horas después de haber ingerido una sopa en su restaurante favorito.
Tiene como antecedentes que es hipertenso y toma atenolol. Al examen fisico tiene una
PA 110/70 FC: 60 x min. Ojos pupilas con la tendencia a la miosis, corazon con
bradicardia, pulmones con rudeza respiratoria. Usted sospecha que puede tratarse de cual
de las siguientes entidades. Usted sospecha que tiene una intoxicación por
organofosforado. Los síntomas mencionados son la mayoria:
a) Muscarínicos
b) Nicotínicos
c) Del SNC
d) Intoxicación…
e) Ninguna de las anteriores

Con respecto a la intoxicación anterior, cual de los siguientes es un síntoma del SNC
a) Agitación
b) Fasciculaciones (nicotínico)
c) Miosis (muscarinico)
d) Dolor abdominal (muscarinico)
e) Tremor

Si usted pudiera medir los niveles de paraquat en sangre por espectrofotometria, … nivel
que indica alta probabilidad de muerte es
a) 2 mg/L a las 4 horas
b) 2 mg/L a las 2 horas
c) 1.5 mg/L a las 4 horas
d) 0.9 mg/L a las 6 horas
e) 0.5 mg/L a las 6 horas

Es característico de la intoxicación por diquat


a) Convulsiones
b) Infarto agudo del miocardio
c) Insuficiencia hepatica
d) Edema pulmonar
e) Ninguna de la anteriores

Cuando usted administra pralidoxine en la intoxicación por organofosforado… actuando


sobre:
a) Síntomas pulmonares
b) Síntomas muscarinicos
c) Reactivacion enzimatica
d) Síntomas del SNC (y síntomas nicotinicos)
e) Ninguna de las anteriores

Condición que pueda afectar la actividad de colinesterasa cual de la siguiente es:


(enfermedad hepatica avanzada, desnutrición, alcoholismo cronico, diabetes, factor
genetico y parasitosis intestinal)
a) Sífilis
b) Parasitosis intestinal
c) Vasculitis
d) Hipertiroidismo
e) Sarcoidosis

El tratamiento específico del granuloma inguinal es:


a) Ceftriaxone IM
b) Azitromicina una sola dosis
c) Eritromicina
d) Tetraciclina
e) Cefepime

Secuela de infección por herpes tipo II


a) Impotencia
b) Infertilidad
c) Infeccion de vias urinarias crónicas (NO)
d) Radiculopatia sacro lumbar
e) Síndrome diarreico

Tratamiento de la sífilis secundaria en una paciente embarazada es:


a) Tetraciclina 250mg VO qid
b) Azitromicina 1 gramo cada dia por 3 dias
c) Penicilina desensibilizacon previa
d) Doxiciclina 100mg VO bid por 14 dias
e) Claritromicina 500 mg dos veces al dia

El tratamiento actual mas seguro para salmonella es:


a) Cloranfenicol (no)
b) Aminoglicosidos
c) Penicilina
d) Ciprofloxacina
e) Tetraciclinas

La vacunación con BCG utiliza


a) Bacilos vivos
b) Bacilos atenuados (cepa atenuada de M. Boris “Bacilo de Calmette Guerin)
c) Bacilos muertos
d) Productos del bacilo
e) Todas las anteriores

En un paciente que llegue con ictericia de tipo colestasica y esta recibiendo tratamiento
para TBC usted piensa que la droga que le produce esto es:
a) Isoniacida
b) Rifampicina
c) Etambutol
d) Pirazinamida
e) Estreptomicina

Es la dosis letal media de veneno para un adulto de 70 kg es, excepto


a) 24 mg de veneno crotálico y 150 mg de veneno laquésico
b) 5 mg de veneno de coral
c) 70 mg de veneno bothropico
d) Todas las anteriores
e) Ninguna de las anteriores

La dosis inicial calculada de suero antiofidico polivalente seria


a) 1-4 ampollas
b) 5-9 ampollas
c) 10-15 ampollas
d) 15-20 ampollas
e) 25 ampollas

Gastroenterología
1. Las siguientes son posibles causas de “atoramiento” en pacientes con
historia de Enfermedad por Reflujo, EXCEPTO:
a) Adenocarinoma esofágico
b) Estrechez péptica
c) Gastritis
d) Cambios en la motilidad esofágica
2. Entre las funciones del Hígado fundamentales para la vida estan las
siguientes excepto una:
a) Producción de bilis y metabolismo de los lípidos
b) Gluconeogénesis y glucogenolisis
c) Degradación y producción de insulina
d) Roptura del eritrocitos y creación de Biliverdina
3. La etiología más frecuente de la cirrosis hepática es:
a) Hepatitis C
b) Enfermedad colestásica crónica
c) Hepatitis autoinmune
d) Alcohol.
4. Cuál es la primera manifestación clínica de una cirrosis hepática?
a) Dispepsia, astenia o hiperpirexia.
b) Ictericia
c) Edema de Miembros Inferiores
d) Ascitis
5. El anillo de Kayser-Fleischer consiste en lo siguiente:
a) Anillo verdoso a nivel de la mácula
b) Anillo de color verdoso a nivel nervio óptico
c) Anillo de coloración pardo-verdosa en el borde límbico de la
córnea
d) Lesión pardo verdosa en la conjuntiva palpebral en forma de
anillo
6. La clínica de cirrosis hepática por alteraciones morfológica incluye
todo lo expuesto, excepto?
a) Ictericias.
b) Edema,
c) Coagulopatías
d) Hipertensión portal
7. Las manifestaciones clínicas de la cirrosis hepática casi siempre son
asintomática en un porcentaje de.
a) 5 a 30%
b) 10-40%
c) 50%
d) 60%
8. La ascitis que aparece en la descompensación de la cirrosis hepática
es un trasudado porque presenta:
a) < 3g/dl de proteínas
b) 3g/L de proteína
c) DHL mayor de 200 mg/dl
d) Glucosa alta
9. Los niveles de conciencia en la encefalopatía hepática disminuye por
la presencia de:
a) NH4
b) GABA
c) Urea
d) Octapamina

10. Favorece la penetración de amoniaco al cerebro situaciones de:


a) Acidosis metabólica
b) Alcalosis con Hipopotasemia
c) Acidosis con Hiperpotasemia
d) Alcalosis con hiperpotasemia
11. La enfermedad úlcera péptica por AINES se explica por :
a. Hipergastrinemia b. Aumento del moco gástrico c. Disminución de la PG d.
Histamina
12. Mecanismo por el cual se postula que H. pylori produce úlcera
duodenal:
a. Aumento de Somatostatina b. Metaplasia gástrica c. aumento de
HCO3 en duodeno.
13. El tabaquismo puede producir úlcera duodenal por mecanismo
propuesto como
excepto:.
a. generación de radicales de O2 b. vaciamiento gástrico alterado
c. disminución de infección x H. pylori d. bicarbonato
disminuido en duodeno
14. Colitis Ulcerativa
a. La enfermedad abarca hasta la Muscular
b. Afecta principalmente el Intestino Delgado
c. Cuadros obstructivos son frecuentes
d. Ulceraciones en placas en toda la mucosa
e. Ninguna de las anteriores
15. Evolución de la infección por Virus de Hepatitis B
a. Curación 90%- Cronicidad 5%- Cirrosis 2%
b. Curación 90%- Cronicidad 10%- Cirrosis 1%
c. Curación 90%-Cronicidad 1%- Cirrosis 5%
d. Curación 90%-Cronicidad 10% Cirrosis 5%
e. Ninguna de las anteriores
16 Son características de la Colitis Ulcerativa ( CUCI ) excepto
a. Mucosa friable granular
b. Puede perderse gran parte de la mucosa por ulceraciones
c. Las fistulizaciones son frecuentes
d. Se encuentran múltiples pseudopólipos
e. Ninguna de las anteriores
17. Pancreatitis Aguda Clasificación Tomográfica Balthazar C
a. Agrandamiento focal pancreático
b. Agrandamiento más inflamación peripancreática
c. Agrandamiento más colección líquida
d. Agrandamiento más presencia de gas adyacente
e. Ninguna de las anteriores
18. Son factores etiológicos del cáncer gástrico todo lo siguiente
excepto:
a. Nitratos b. Infección por H. pylori c. enf de Ménétrier d.
úlcera duodenal
19. . Factor de riesgo para cáncer de Colon.
a. Dieta pobre en fibra b. Dieta pobre en grasa c. Resistencia a insulina d.
Colitis
pseudomembranosa
20. Prueba dg más sensible para Cáncer colorectal es:
a. Sangre oculta en heces b. CT abdominal c. Colonoscopía d. colon x
enema
21. Son párametros de Child Pugh todos lo siguiente excepto:
a. bilirrubina b. Creatinina c. protrombina d. ascitis e. Albúmina
Nefrología
1. Paciente que pesa 50Kg, con sodio sérico de 160 mEq/dL , con ACT =40%
, su déficit de agua es:
a) 5L b) 3L c) 4L d) 6L
2. Para un calcio total sérico de 7mg/dl y albúmina de 2,2 g/dl ¿Cuál será
el calciototal
corregido? Tome el valor de la albúmina normal de 4 g/dl
a) 7,44 mg/dl b) 8,44 mg/dl c)9,44 mg/dl d) 8,90 mg/dl
3. Velocidad de administración de KCl en condiciones normales, en
mEq/hora
a) 8 b) 120 c) 20 d) 40
4. Causa falla prerrenal
a)Obstrucción de ambos uréteres b)Falla cardiaca c)
Glomérulonefritis aguda d) Síndrome nefrósico
5. Manifestación en el EKG de un paciente con Hipokalemia:
a) Onda T picuda b) Supradesnivel del segmento ST
c) Intervalo QT deprimido d) Onda S profunda
e) Prolongación del intervalo PR f) N/A
6. Cuál es la concentración sérica de fósforo por encima de la cual se
considera que una persona tiene Hiperfosfatemia (en mg/dL):
a) 3.0 b) 4.0 c) 5.0 d) 6.0 e) 7.0
7.. Diurético de uso común en el tratamiento del edema en los
síndromes
nefrítico y nefrótico:
a) Indapamida b) Espironolactona c) Furosemida d)
Acetazolamida
e) Hidroclorotiazida f) N/A
8. Cuál es el principal tratamiento para pacientes con nefritis
tubulointersticial
alérgica:
a) Prednisona b) Enalapril c) Ibuprofeno d) Diálisis e) N/A
9. Cuál de las siguientes enfermedades puede producir un Síndrome
Nefrítico
mediado por inmunocomplejos
a) Granulomatosis de Wegener b) Síndrome de Goodpasture
c) Púrpura Trombocitopénica Trombótica d) Amiloidosis e)
N/A
10. Ejemplo de hiponatremia hipotónica normovolémica
a)Diarrea B)Cirrosis hepática c) Síndrome nefrósico d) SIADH
11. Mencione cuál de las siguientes condiciones puede ser causa de
Hiperkalemia:
a) Uso de IECA b) Hiperaldosteronismo primario c)
Hiperinsulinemia d) Alcalosis Metabólica e) N/A
12. Mencione cuál de las siguientes condiciones cursa con el
desarrollo de
Enfermedad Crónica de los Riñones:
a) Nefrolitiasis unilateral b) Enfermedad de Cambios Mínimos
c) Nefropatía Diabética d) Carcinoma de Riñón e) N/A
13. Oliguria implica una diuresis menor de (mL/día):
a) 50 b) 400 c) 800 d) 1000 e) N/A
14. Señale la aseveración Incorrecta acerca de la Hormona
Antidiurética:
a) Polipéptido sintetizado en el hipotálamo
b) También se le conoce como Arginina Vasopresina
c) El efecto neto de su acción a nivel de la nefrona es la
reabsorción
pasiva de agua y sodio
d) El mayor estímulo para su secreción es la hipertonicidad
15. Generalmente la proporción BUN / creatinina plasmática en
hiperaazoemia
prerrenal es de:
a) <10 b) > 20 c) 100% d) d) 1%
16. El sodio urinario en hiperazoemia renal intrínseca es de:
a) <10 b) >20 c) > 30 d) >1
17. La causa principal del síndrome nefrítico agudo es :
a) Hipertensión arterial b) GN postestreptocóccica c) Síndrome
deGoodpasture
d) Nefritis lúpica
18. Cuál es el componente esencial en el síndrome nefrótico?.
a) hipoalbuminemia b) hiperlipidemia c) proteinuria d)
edema
19. Cuadro clínico clásico del síndrome nefrítico incluye lo siguiente
excepto:
a) lipiduria b)hipertensión c) Oliguria d) Hematuria
20. El tx en cuarto de urgencias de hipercalcemia intensa es:
a) Bifosfonatos b) Furosemida y SSisotónica c) Furosemida y
Bifosfonatos
d) SSisotónica +Calcitriol.

Toxicología
1. Paciente intoxicado con medicamento para los ojos presenta: rubor,
mucosas secas, Intranquilo y retención urinaria su tx incluiría:
a)Fisostigmina b) Atropina c) Propranolol d) Flumazenil

2. Paciente rociado con un tóxico llamado Malatión podrá presentar los


siguientes síntomas muscarínicos excepto:
a) lagrimeo b) sialorrea c) broncoespasmo d) rubicundez
b)

3. Paciente en coma porque se tomó 30 pastillas de alprazolam, enojado con su


novia. Usted utilizaría el siguiente antídoto:
a) Atropina b) Flumazenil c) Protamina d) Fisostigmina

4. Cuál de las siguientes no es una medida para evitar absorción del tóxico:
a) Descontaminación oral b) D. digestiva c)Diuresis forzada d) D.
Cutánea

5. Pte mordido por una culebra , no presenta signos al inicio, 24 horas después
presenta ptosis Palpebral. La intoxicación ofídica la clasificaría como:
a) Síndrome Botrops b) S. Elapídico c) S. Clotalico d) S. Botulínico
6. Pte mordido por una culebra en el pie , con aparición de equimosis , edema
hasta 1/3 medio de la pierna , con TPT y TP alterados ligeramente la
intoxicación ofídica se clasifica como:
a) Grado 0 b) leve c) moderada d) Severa

7. El tx con suero antiofídico del paciente anterior sería así:


a) < 5frascos b) 5-9 frascos c) 15 frascos d) > 15 frascos.

8. El tx de las arritmias ventriculares por intoxicación con cocaína incluye:


a) Digoxina b) Amiodarona c) Nifedipina d) Propranolol

9. Pte de 18 años, llega con sialorrea, miosis pupilar, broncoespasmo, dice


que tomó “Baygon”, usted lo trataría con:
a) Fisotigmina b) Atropina c) Flumazenil d) N- acetilcisteína

10. Tx sugerido para el intoxicado por Paraquat:


a) O2 b) Tierra fuller c) Diazepam d) Atropina

11. Intoxicaciones que cursan con excitación todo lo siguiente excepto:


a) Atropina b)Opiaceos c) Abstinencia d) Teofilina

12. Antídoto para Heparina es:


a) Protamina b) Coumadin c) Vitamina K d) Plasma fresco

13. Intoxicación por Malatión usted usaría como Antídoto


a)Fisostigmina b) Atropina c) Propranolol d) Flumazenil

14. Prioridad del manejo general en un intoxicado:


a) Canalizar vena b) Oxigenoterapia c) Glasgow d) Permeabilidad de
vías aéreas

1. Causa de hiperfosfatemia:
a. Falla hepatica
b. Falla renal cronica
c. Falla cardiaca
d. Insuficiencia suprarrenal

2. Causas de hiperfosfatemia grave, excepto:


a. recuperacion de cetoacidosis diabetica. SI
b. Alcalosis respiratoria
c. Acidosis respiratoria. SI

3. Paciente con vómitos severos, oligurico con hipotensión arterial Una> 40……
800 mOsm/Kg puede tener:
a. Insuciencia renal aguda.
b. Insuficiencia suprarrenal
c. Falla pre-renal
d. Exceso de mineralocorticoides

4. Paciente urémico crónico con deshidratación severa puede presentar lo siguiente


excepto:
a. Densidad urinaria 1010.
b. Osmolaridad urinaria de 800 mOsm/Kg
c. K sérico aumentado
d. Oliguria.

5. La hipokalemia produce:
a. Aumento de la magnitud del potencial de reposo

6. Uno de los indices de falla renal aguda es:


Uosm/Tosm

7. Hormona producida en los riñones que disminuye en la falla renal cronica:


a. ADH
b. Noradrenalina
c. Peptido natriuretico (segun arjona)
d. 1,25 (OH)2 Vit D.

8. Las calcificaciones metastásicas se deben:??


a. Aumento del producto Ca, P
b. Aumento del producto Ca, Mg
c. Hipoparatiroidismo
d. Hiperparatiroidismo 1ario

9. Causa de anuria:
a. Pielonefritis unilateral
b. Necrosis tubular aguda
c. Enf cambios minimo
d. Necrosis cortical

10. la etapa de la insuficiencia renal cronica con tasa de filtración glomerular de 60-
89 ml/min es la:??
a. 1
b. 2
c. 3
d. 4

11. La FENA es la relación entre:


a. Oferta tubular/Na reabsorbido x 100
b. depuración de Na/ depuración de creatinina x 100
c. oferta tubular/ Na excretado x 100.
d. Na excretado/ Na reabsorvido x 100.

12. En la nefritis tubular intersticial aguda se pierde la capacidad de poder concentrar


la orina debida a:??
a. Daño medular
b. Daño glomerular
c. Daño tubular proximal
d. Daño de la porcion cortical del tubulo colector

13. La solucion de NaCl al 23,4 % tiene:


8000 mOsm/l

14. Se asocia con el SIAD, excepto:


lipoma

15. El efecto biologico mas importante de la calcitonina es, excepto:


a. Responder a la hipocalcemia
b. Aumenta la calciuria
c. Estimular la resorcion osea osteoclastica
d. Utilizada en el tx de la hipercalcemia.

16. Favorece la entrada de K al espacio intracelular, excepto:


a. Agonista adrenergico
b. Estimulacion adrenergico
c. Insulina
d. Alcalosis metabolica

17. Factores que afectan el balance hídrico en el post-operatorio, excepto:


a. Fármacos
b. Calcitriol
c. Administración de liquidos hipotonico
d. Aumento de la ADH

18. Las primeras manifestaciones clínicas de la hiponatremia son:??


a. neurológica
b. gastrointestinal
c. pulmonares
d. musculares

19. En la hipernatremia hipertonica se da lo siguiente excepto:


a. Entrada de Na
b. Salida de aa
c. Ganancia de iones
d. Disminución del agua cerebral

20. Manifestaciones clinicas de la hipercalcemia, excepto:


a. Taquicardia
b. HTA
c. Constipación
d. Vomitos

21. En falla renal aguda por nefrotoxicidad:


a. Se conserva la membrana basal
b. Se destruye la membrana basal
c. Frecuente hay oliguria
d. La creatinina no aumenta

22. Causa de hipomagnesemia:


alcoholismo

23. Na serico de 168 mEq/l 70 kg (utilize como % de agua al 50% y Na normal 140
mEq/l
7 litros

24. Indicaciones de diálisis en falla renal aguda, excepto:


a. Encefalopatia uremica
b. Hiperkalemia refractaria
c. Ac. Metabolica refractaria
d. hipocalcemia

25. El KCL IV se puede asministrar asi, excepto:


a.infusion por linea periferica de 10 meq en 100 ml en 1hora
b. “ “ central de 20 meq em 50ml en 1 hora
c. “ “ central de 100 meq em 50ml en 1 hora
d. “ “ periferica de 20 meq en 20ml en 2 horas

26. Aumenta la excrecion urinaria de K, excepto:


a. Triamtereno
b. Bumetanida
c. Furosemida
d. Metolazona
27. Factores que regulan la secrecion distal de K, excepto:
a. Flujo tubular distal y aporte distal de Na
b. Excrecion de aniones no reabsorbibles
c. Aldosterona
d. PTH

28. Enfermo de 60 Kg con 60% de agua corporal, Posm 260 mOsm/kg, ¿Cuántos
gramos de NaCl aumentaria el Na en 10 mEq?
a. 21 gramos

50. Conteniendo 10 g de MgSO4 al %, cuantos ml de sal necesitaria?


20 ml

1. Cuándo disminuye el fosfato?


R. Aumento de PTH

2. Ejemplo de hipernatremia hipervolémica:


R. Aldosteronismo primario

3. La hipocalemia produce:
R. Aumento de la magnitud del potencial de reposo

4. La depleción de agua produce, excepto:


R. Salida de electrolitos del cerebro

5. Velocidad de administración de KCl en caso de urgencia:


R. 40 mEq/h

6. Cambios en EKG de la hipermagnesemia es igual:


R. Hipercalemia

7. Enfermo de 60 Kg con 60% de agua corporal, osmolaridad plasmática 260


mosmol/Kg, ¿cuántos gramos de NaCl aumenta el Na en 10 mEq en 24 horas?
R. a) 15
b) 21
c) 30
d) 19

8. Uso de Calcio en tratamiento de hipercalemia para:


R. Modificar el Potencial de Membrana

9. Hipermagnesemia severa produce:


R. Paro respiratorio
10. Venoclisis de 1L de D/A 5% conteniendo 10 mg de MgSO4 y al 50%:
R. 20mL

11. La hipocalcemia:
R. Prolongo QT a expensas del segmento ST

12. Tratamiento de urgencia de hipercalcemia:


R. SSN 0.9%

13. Causa de hiperfosfatemia:


R. Insuficiencia renal

14. Adaptación cerebral, hipertonicidad, excepto:


R. Pérdida de iones

15. En la adaptación cerebral, la hipotensión da lo siguiente, excepto:


R. Ganancia de aminoácidos
16. En ausencia de síntomas de hipernatremia, el sodio debe ser disminuido a razón de:
R. 0.5 mEq/L/h

17. Para corregir un sodio cerca de 185 mEq se administró en 86 horas, 66 letras de D/SS
5%. ¿cuál es la velocidad de administración de la venoclisis?
R. 69 mL ¿?????

18. Manifestación clínica de hipercalcemia, excepto:


R. a) Hipotensión arterial
b) Bradicardia
c) Nauseas
d) Anorexia

19. Consecuencia de hipofosfatemia, excepto:


R. a) No afecta al SNC
b) Disfunción plaquetaria
c) Cardiomiopatía
d) Osteomalacia

20. Causa de hipofosfatemia, excepto:


R. a) Falla renal
b) Recuperación de cetoacidosis diabética
c) Alcalosis respiratoria
d) Quemaduras graves

21. Causa de HTA con hipocalemia, aldosterona-renina baja:


R. Síndrome de Cushing

22. Causas de HTA con hipocalemia y renina alta:


R. Hipertensión maligna

23. En una deshidratación severa hipertónica con hipotensión arterial severa, la prioridad
inicial es administrar:
R. SSN 0.9%

24. En la secreción inapropiada de ADH se da lo siguiente, excepto:


R. No hay expansión de volumen

25. Tejido con mayor contenido de K+:


R. a) Hígado
b) Músculo
c) Eritrocito
d) Hueso

14) Paciente con Oliguria,……muchos síntomas :

a) Insuficiencia suprarrenal ( creo que era la respuesta)

15) En IRA se mide todo menos:

a) Uosm/Posm x100
b) Na excretado/ Na filtrado x200
c) Na filtrado/ Na excretado x100
d) Na urinario >40
e) Ninguna de las anteriores

16) Paciente con hipertensión , hipokalemia, cretinina normal tiene:

a) Hipertensión arterial maligna


b) Cushing
c) Algo relacionado a licorice

31) En hiponatremia el daño principal es:

a) Neurológica
b) Respiratoria
c) Renal

32) Adaptación del cerebro ante solución hipotónica:

a) Pérdida de iones
b) Pérdida de aminoácidos
c) Edema cerebral
d) Ganancia de iones

53) Paciente con hipercalcemia puede ser por todo menos:

a) Aumento de secreción de TSH


b) Hiperparatioidismo
c) Toxicidad de Vitamina D
d) Medicamentosa
e) Leche de álcalis

62) La PTH promueve:

a) Reabsorción tubular de calcio y excreción de fosfato

66) Paciente que sufre golpe en la cabeza y tiene fractura de cráneo. Tiene
osmolaridad urinaria disminuída , pero al darle tratamiento con vasopresina ésta
aumenta. El paciente tiene:

a) Diabetes insípida central


b) Diabetes insípida nefrógena

24. Usted tiene una paciente con hiperemesis gravidica. El trastorno metabolico que
espera encontrar es:
a. acidosis hipocloremica
b. acidosis hipercloremica
c. alcalosis hipocloremica
d. alcalosis hipercloremica
e. no hay alteración acido base

25. Cual de las siguientes opciones esta contraindicada en el tratamiento de la


hiperkalemia
a. gluconato de calcio
b. aldactone
c. kayaxelate
d. Bicarbonato de sodio
e. Todas las anteriores

40. La accion toxica mas importante del ganciclovir es:


a. Nauseas y vomitos
b. Depresion de la medula osea
c. Diarrea
d. Insuficiencia Renal
e. Insuficiencia Respiratoria
41. Rotando en sala de medicina interna le piden que evalue un paciente con HIV el
cual por presentar una meningitis por criptococci recibe anfotericina B; debido a
este medicamento buscaria como complicación:
a. aparicion de diarrea
b. aumento de la presion arterial
c. hipokalemia
d. cefalea
e. dolor toráxico

42. Paciente de 84 anos quien recibe diuretico tiazidico para la hipertensión arterial
es admitido por presentar diarrea y trastornos del sensorio. Tiene disminución del
turgor de la piel y la presion arterial es normal, Pna 174mEq/L, Una 5mEq/L,
Uosm 606mEq/kg. La hipertensión se debe a:
a. perdidas insensibles
b. polidipsia
c. uso de diuretico
d. diabetes insipida
e. secrecion inapropiada de hormona antidiuretica

43. Hipernatremia Hipervolemica


a. diuresis osmótica
b. diarrea
c. aldosteronismo primario
d. diabetes insipida
e. SIADH

44. La hipokalemia en un cirrotico puede precipitar COMA por:


a. Hipermagnesemia asociada
b. Aumento de la amoniogenesis
c. Hiperfosfatemia asociada
d. Rabdomiolisis
e. Induce vomitos

Causa de hiperkalemia:
a. Aumento del volumen arterial eficaz
b. Hipercatabolismo tisular
c. Hiperaldosteronismo
d. Alcalosis metabolica
e. Cirrosis hepatica

45. Causa de hipomagnesemia


a. cirrosis avanzada
b. falla renal cronica
c. falla cardiaca
d. constipacion
e. acidosis respiratoria

46. Causa renal de hipomagnesemia


a. falla renal cronica
b. esteatorrea
c. desnutricion severa
d. uso de diureticos de asa
e. infeccion urinaria

47. en caso de hipercalcemia pensar en:


a. neoplasia
b. hiperfosfatemia severa
c. nefrolitiasis
d. nefrocalcinosis
e. diureticos

48. En el síndrome nefrosico puede encontrarse cilindros:


a. cereos
b. hematicos
c. leucocitarios
d. epiteliales
e. mixtos leucocitario y hematicos

1. Causa de hipomagnesemia:
a. estreñimiento
b. alcoholismo
c. falla pre-renal
d. IRA (insuficiencia renal aguda)

2. La hipocalcemia en la falla renal aguda puede ser asintomática por la:


a. acidemia
b. hipoalbuminemia
c. Deficiencia de PTH
d. Alcalemia

3. La hipofosfatemia causa, excepto:


a. Aumento de vit. D
b. Aumento de PTH
c. Disminución de la excresión de fosfato
d. Aumento de la absorción de fosfato a nivel intestinal

4. Un pte con diuresis de 250 ml al día con osmolaridad plasmática (presión


osmótia) y osmolaridad urinaria de 300 mosm/l, el dx probable e:
a. Insuficiencia suprarrenal
b. Insuficiencia renal
c. Nefritis túbulo intersticial crónica
d. Deshidratación hipertónica

5. Pte con oliguria, hiperkalemia, hiponatremia, creatinina 0.8 mg/dl y CPK normal:
Esto puede corresponder a:
a. IRA <de 30 mosm es ira
b. Insuficiencia suprarrenal<--
c. Síndrome hepatorrenal
d. Rabdomiolisis

6. No está indicado en caso de hiperkalemia:


r. Aldactone

7. Mujer con hiperemesis grávida:


r. Alcalosis hipoclórica

8. En causa de hipomagnesemia:
r. Alcoholismo.

9. En el metabolismo del calcio está implicado, menos:


r. Insulina

10. Hiponatremia, hipotónico, normovolémico:


r. SIADH

11. Fórmula del clearence de creatinina:


r. clearence = 140 – edad x peso en Kg
72 x creatinina

12. Su infección produce litiasis renal:


r. proteus sp.

13. Fórmula de irritabilidad muscular:


r. IR = Na + K + OH
(Ca) + Mg + H

14. Con respecto al síndrome diarreico, nombre 2 tipos osmóticos:


a. Laxantes osmóticos (Mg2+, PO4, SO4)
b.
Carbohidratos no absorbibles (sorbitol, lactulosa).

15. Nombre dos síndromes diarréicos tipo secretora:


a. Consumo crónico de etanol.
b.
Obstrucción intestinal incompleta.
16. Nombre 2 sindromes diarréicos motores con aumento de motilidad:
a. Hipertiroidismo
b.
Prostaglandinas.

17. Indica falla renal aguda:


a. U.N. = 4.1
b.
U.N. = 1
c.
U.N. = ¼
d.
Gravedad específica: 1.026
e.
Ninguna.

18. Deshidratación por pérdida de agua y densidad urinaria 1.003, creatinina 6:


a. falla renal aguda parenquimatosa
b.
Falla pre-renal
c.
Falla renal obstructiva
d.
Falla renal crónica
e.
No hay falla renal.

19. Uno de los índices de falla renal es:


a. Reabsorción de Na/ oferta tubular de Na x 100
b.
Excreción urinaria de Na/ oferta tubular de Na x 100
c.
Oferta tubular de Na/ excresión urinaria de Na x 100 **

d.
Una / pna x 100
e.
Ninguna

20. El síndrome heatorrenal es causa de falla:


a. pre-renal
b.
renal
c.
posrenal
d.
falla cardiaca
e.
daño tubular

21. Causa más comun de falla renal cronica:


a. Diabetes melitus
b.
HTA
c.
Glomerulonefritis
d.
Enfermedad poliquística renal
e.
Vasculitis

36. En la glomerulnefritis aguda por infección, la causa, excepto:


a. proteínas mayor de 10g
b. osmolaridad urinaria normal
c. hematuria
d. HTA
e. Edema

• A pesar de la diversidad de enfermedades indicadas abajo, ellas tienen muchos


síntomas en común. Con frecuencia, los síntomas y signos observados incluyen:
disminución en el volumen de orina, proteína en la orina (proteinuria), sangre en
la orina microscópica o macrscópica (hematuria), hinchazón (edema), presión
sanguínea alta (HTA) y una disminución en la capacidad del riñón de extraer
eficazmente los residuos.

22. Falla renal crónica, excepto:


a. PTH aumentada
b. Hipofosfatemia
c. Acidemia
d. Hipocalcemia
e. 1.25 OH (D3) disminuída.

23. En la falla renal aguda siempre habrá:


a. aumento de creatinina
b. fracción de excresión de Na aumentado NO
c. Hiperkalemia
d. Una aumentado
e. Hipernatremia NO XQ HAY HIPONATREMIA

24. Causa de falla pre-renal:


a. Sd. Nefrótico
b. Obstrucción de ambos uréteres
c. Falla cardiaca
d. Glomerulonefritis aguda pos- infecciosa
e. Calculo renal

25. En la nefritis tubular intersticial se puede dar, excepto:


a. hipernatremia
b. proteinuria de 1.5 g SI
c. normotension arterial
d. hipostenuria
e. hipocalcemia

26. Sugiere glomerulonefritis


a. leucocituria
b. cilindros eritrocitarios
c. anuria
d. ……

27. Causas de falla renal parenquimatosa:


a. papilitis
b. calculo en pelvis derecha
c. estenosis unilateral de arteria renal
d. cistitis hemorragica
e. ………

28. Causa de anuria:


a. Enfermedad de cambios mínimos
b. Necrosis cortical
c. Pielonefritis aguda leve bilateral
d. Necrosis tubular aguda
e. Calculo coloriforme bilateral

29. La hipofosfatemia causa todo, excepto:


r- aumento de PTH

30. Hepatitis C periodo de incubación:


r. 15-150 media de 5º (7 semanas)

31. En pte con falla renal crónica que ha sido transplantado esperamos encontrar:
r. osteomalacia.

Urémico crónico con potasio sérico de 6.5mEq/L con leve academia, sin trastorno
muscular y EKG normal, puede ser tratado con:
a. kayaxelate
b. gluconato de Calcio al 10%
c. NaHCO3
d. Diálisis

La insuficiencia renal crónica ocasiona hiperfosfatemia permanentemente cuando la tasa


de filtración glomerular esta por debajo de:
a. 60ml/min
b. 50 ml/min
c. 25 ml/min
d. 40 ml/min

La hipofosfatemia es causa de excepto


a. inotropismo positivo
b. disminución de 2,3 DPG
c. hemólisis
d. curva de HbO2 desplazada hacia la izquierda

La solución de NaCl 23,4% tiene de osmolaridad:


a. 8000 mosm/l
b. 800 mosm/l
c. 80 mosm/l
d. 0,8 mosm/l

En la hiperkalemia:
a. aumenta la sístole eléctrica
b. hay menor diferencia entre el potencial de reposo y el umbral
c. el potencial de reposo transmembrana se hace mas negativo
d. el potencial umbral se hace menos negativo

La velocidad de administración de potasio en caso de gran riesgo para la vida puede ser
de:
a. 10-20 mEq/L
b. 8 mEq/L
c. 110 mEq/L
d. 40 mEq/L

Un ejemplo de hiponatremia hipotónica normovolemica es:


a. síndrome nefrósico
b. SIADH
c. Diarrea
d. Cirrosis

A una embarazada con eclampsia se le ordena 1 litro de D/A 5% con 10g de MgSO4.
¿Cuántos ml de MgSO4 al 50% se han utilizado?
a. 20
b. 5
c. 12,5
d. 30

Causa hipomagnesemia:
a. estreñimiento
b. alcoholismo
c. deshidratación hipertónica
d. insuficiencia renal aguda

Paciente con diuresis de 300ml/día, K sérico 7mEq/L, Posm y Uosm 300mosm/L, el


diagnóstico más probable es:
a. insuficiencia suprarrenal
b. insuficiencia renal <350
c. SIADH
d. Deshidratación primaria

La mielinosis pontina se caracteriza por:


a. pares craneales intactos
b. cuadriplejía flácida
c. comportamiento normal
d. hipertonía de los miembros inferiores
Calcule el déficit de agua en una paciente con sodio sérico 168mEq/L y que pesa 70Kg
(utilice como porcentaje de agua 50% y sodio normal mEq/L):
a. 5L
b. 3L
c. 10L
d. 7L

En la SIADH la concentración de sodio urinario es de:


a. menos de 10mEq/L
b. 20mEq/L
c. 15mEq/L
d. Mas de 40mEq/L

En paciente con vómitos abundantes y prolongados con hipokalemia severa debe


administrarse:
a. NaCl 0.9% con KHCO3
b. NaCl 0.9% con KCl
c. NaCl 0.9% con gluconato de potasio
d. NaCl 0.9% con NaHCO3

La hipokalemia en un paciente con cirrosis puede contribuir:


a. coma
b. alcalosis respiratoria
c. acidosis metabolica
d. aumento de la síntesis de urea

El plasma de un paciente contiene Na 125mEq/L, glucosa 108mg/dl, urea 300mg/dl,


podrá presentar:
a. cirrosis
b. síndrome nefrósico
c. síntomas de hipertonicidad
d. síntomas de hipotonicidad

La hiponatremia aguda sintomática es debida a, excepto:


a. exceso de glucocorticoides
b. SIADH
c. post-operatorio
d. intoxicación hídrica

La administración de solución salina 0.9% produce:


a. disminución del volumen intracelular y aumento del extracelular
b. aumento del volumen intra y extracelular
c. aumento del volumen extracelular
d. aumento del volumen intracelular
Alcalosis metabólica con hipertensión arterial, hipokalemia, aldosterona normal y renina
normal:
a. Hipertensión renovascular
b. Hipertensión maligna
c. Sindroma de Cushing
d. Administración de licor

La hipokalemia puede predisponer, excepto:


a. intoxicación digitálica
b. disminución de la amoniogénesis
c. rabdomiolisis
d. ondas U en el EKG

Compendio renal

1) Cuál de las siguientes medicinas pueden dar hiperkalemia


a) inhibidor de la enzima convertidota
b) bloqueador alfa adrenérgico
c) bloqueador beta adrenérgico
d) ayb
e) ayc

2) Hormonas que disminuyen en la falla renal crónica, excepto


a) 1,25 dihidroxi vitatamina D
b) eritropoyetina
c) paratohormona
d) foliculo estimulante

3) MgSO4 (PM 120) 1 gramo contiene de magnesio en meq


a) 10
b) 7.9
c) 8.3
d) 6

4) Factores que conducen a la progresión de falla renal crónica


a) hiperlipidemia
b) toxinas urémicas
c) proteinuria
d) b y c
c) todas

5) Paciente de 20 años presenta abundantes vômitos y diarrea de 3 días de evolución,


examen físico depleción de volumen, suero muestra Na 155, K 3meq/l, Cl 117meq/l,
HCO3 25 meq/l, tratamiento de elección:
a) D/A 5% + KCl
b) solución salina 3% + KCl
c) solución salina 23.4% + KCl
d) solución salina 0.45% + KCl

6) En la enfermedad ateroembólica renal se da lo siguiente excepto:


d) Velocidad de eritrosedimentación

7) Pseudohiperkalemia de 8 mEq/l que se ve en el EKG:


Onda T picuda
Aplanamiento de la onda P
Ay B
Ninguna

8)Criterios para iniciar diálisis excepto:


Alcalosiss Metabolica

9)Hipokalemia con normotension excepto


Síndrome de liddle

10) Plasama creatinina 8mg/dl N de urea 100mg/dl U na 40 Meq/l :


Insuficiência renal

11) Factores reversibles responsables de deterioro de la funcion renal excepto:


Fibrosis intersticial

12) Causa hipokalemia con renina y aldosterona disminuida:


Licorice

13. Sugiere trastorno túbulo intersticial:


.Proteinuria mayor a 3.5 g /24 horas
.Notrmotensión
.Edema
.Osmolarida urinaria normal

14. Glomérulonefritis rápidamente progresiva


.Engrosamiento de la membrana basal
Fusión de los pedículos de los podocitos
prolñiferacion intracapilar
proliferación extracapilar ( creo que es esa verificar)

15.Una de las siguientes causas no produce hematuria


.enfermedad de cambios mínimos

16.Plasma Na:115, Posm240 mOsm,Uosm 680,Una 60:meq/l:


.Deshidratación hipotónica
.secreción inapropiada de ADH
.Deshidratación isotónica
.Deshidratación primaria

17.en la falla renal crónca produce necrosis


calcifilaxia

18. No produce hipokalemia:


Pancreatitis aguda
rabdomiolisis
neoplasia
Def. Vit D

19. Paciente con glucosa 90mg/dl, Na+= 115mEq/L, Urea en 140mg/dl


a. Síndrome hipoosmolar
b. Síndrome hiperosmolar
c. Diarrea
d. Ninguna de las anteriores.

20. Causa de seudohiponatremia con osmolaridad elevada


a. Hiperproteinemia
b. Diabetes mellitus
c. Hiperlipidemidemia
d. Reserción transuretral prostática.

21. Manifestaciones de hipermagnesemia excepto


Normotensión

22. Piel en uremia crónica excepto


Paño blanco

23. No es causa de necrosis papilar aguda


a.Diabetes mellitas ESTA NO ES CAUSA AGUDA
b. Uropatía obstructiva SI
c. Pielonefritis SI
d. Trombosis de venas renal. SI
E- AINE SI

24. Puede presentarse hiperuricemia excepto


d. Adenocarcinoma gástrico

25. Compromete riñones y pulmones:


a. Enf de GoodPasture
b. Granulomatosis de Wegener
c. LES
d. Ay C
26. Sugiere glomerulopatia:
-anasarca

27. TFG < 15ml/min corresponde al estadio de la enfermedad renal cronica:


* 4

28. Causa obstetrica de necrosis cortical bilateral


- desprendimiento prematuro de placenta

29. Promueve amoniogenesis em cirróticos


- hipokalemia

30. Criterio para diagnostico de síndrome nefrosico


a. Proteinuria en 24 hr de 3.5 gr para superficie corporal de 1. 73
b. Albumina serica < 3g/dl
c.Hiperlipidemia y edema
d. Todas las anteriores
e. ay b

1. En la Hiperkalemia:
R/. Hay una menor diferencia entre el potencial de reposo y de umbral

2. La hipokalemia en un cirrotico puede desencadenar:


R/. Coma Hepático

3. La hipermagnesemia puede producir:


R/. Paro respiratorio

7. Tratamiento de hiprecalcemia:
R/. SSN 0.9%

8. La hipofosfatemia son causas, excepto:


a. Aumento de la PTH
b. Aumento de la absorción intestinal de fosfato
c. Disminuye la fosfatemia
d. Aumento de la vitamina D

9. Una solución isotónica:


R/. Aumenta el volumen extracelular

13. La velocidad de administración de potasio en caso de gran riesgo para la vida puede
ser:
a. 10-20 mEq/L
b. 8mE/L
c. 110mE/L
d. 40 mEq/h

14. Paciente crónico con deshidratación severa puede presentar lo siguiente excepto:
a. Densidad urinaria 1010
b. Osmolaridad urinaria de 800 mOsm/kg
c. K sérico aumentado
d. Oliguria

15. Paciente con vómito abundante, hipertensión arterial, sodio urinario de 6, oligúrico,
osmolaridad de 800 presenta:
R/.Insuficiencia suprarrenal
Paciente urémico crónico con vómitos severos, hipotensión arterial, orina al azar muestra
Na mas de 40, osmolaridad de 800 puede tener:
Falla suprarrenal

18. Paciente con Insuficiencia renal crónica, con K sérico de 6, leve academia, EKG
normal, se trata con:
R/. Kayetzalate

22. La hopikalemia predispone, excepto:


R/. Amoniogénesis

25. Cual de las siguientes es causa de hiperfosfatemia:


R/. Insuficiencia renal

26. Paciente con deshidratación hipertónica e hipertensión:


R/. SSN 0.9%

29. En la Glomerulonefritis Aguda post-infecciosa disminuye la tasa de filtración:


R/. Por alteración en el coeficiente de ultrafiltración

23. Px con IRC tine una taza de filtración glomerular de menor de 25 ml/min ;aunque
hay yuna tabla de las etapas en esta enfemedad crónica.
Estas osn las TFG:
1. 90 en riesgo)
2. 60 -89
3. 30 – 59
4. 15-29
5. Menor a 15 diálisis.

24. Factor que altera el balance hídrico en el periódo post- operatorio:


Disminución de la ADH (Excepto creo que es calcitriol)
25. Un ejemplo de hipokalemia con aldosterona alta y osmolaridad baja es:
Hiperaldosteronismo primario

26.Ejemplode hipernatremia hipervolémica: Hiperaldosteronismo primario

27.Hiperkalemioa son alteraciones del EKG no es necesario administrar gluconato de


calcio se colocará un antagonista de K asíe l Px no cae en paro y estoes si tiene
manifestación en el EKG.

28. La fracción de excreción de sodio estará aumentada en falla cardíaca.

29.En la hiperkalemia: hay menos diferencia entre el potencial de reposo y el umbral.

Manifestaciones cardíacas. El problema médico más grave de la hiperkalemia es la


cardiotoxicidad. Los cambios en el ECG producidos por los niveles altos de potasio son
bastante constantes. A medida que aumentan los niveles se aprecian los siguientes
cambios:

Ondas T picudas (con intervalo QT normal o ligeramente reducido)


Prolongación del intervalo PR con depresión de ST
Desaparición progresiva de la onda P
Bloqueo cardíaco progresivo
Arritmias ventriculares
Paro cardíaco

Las ondas T picudas constituyen el dato en el ECG más constante en la hiperkalemia.

Efectos neuromusculares. El primer signo neuromuscular de la hiperkalemia suele ser la


aparición de parestesias seguidas de debilidad progresiva de varios grupos musculares. Si
el cuadro se agrava se observa cuadriplejia fláccida. Las funciones cerebrales y de los
pares craneanos se conservan y la parálisis de la musculatura respiratoria puede ocurrir,
pero es excepcional.

30. La infusión de potasio sin riesgo para la vida del PX: es de 40 meq; porque

I lb de solución salina: debe llevr de 20 a40 mEq de KCl a una velocidad de 10 a 20


mEq por hora; si aumenta la velocidad puede provocar un bloqueo A-V. Se recomienda
una Velocidad mayor si la hipokalemia es muy severa.

Sis e usa venas priféricas es:

10 meq en 100 cc de sol.salina para pasar en una hora

20 meq en 200 cc de sol.salina para pasar en una hora


En vena central:

20 meql en 50 cc pasarla en una hora

40 meq en 100cc pasar la en una hora

31.Los indicios de falla renal aguda son:

(Na filtrado/Na excretado) x 100%

32.Na urinario menor de 10: falla cardíaca, presentará edema estará hinchado y lo
puede llevar a una I.C.C. ( Esto lo saque de Medline: Los valores normales generalmente
son de 15 a 250 mEq/L/día, dependiendo del estado de deshidratación y la ingesta diaria
de sodio en la dieta. Los rangos de los valores normales pueden variar ligeramente entre
diferentes laboratorios. Nota: mEq/L/día = miliequivalentes por litro por día.)

33.Causa de enuresis: necrosis cortical difusa.

34.En SIDH la concentración urinaria es menor de 20.

35. Deshidratación pura de agua excepto: hay salida de electrolitos del cerebro.

36.Urémico crónico con deshidratación hipertónica severa y acidótico presentará lo


siguiento excepto: osmolaridad urinaria de 8mm mosmol/kg.

37. Px con vómitos abundante, hipertensión arterial; Na urinario=6, oliguria, osmolaridad


de 800 presenta: insuficiencia suprarrenal.

38. Factor que predispone a la hipernatremia son intoxicación hídrica.

39. Se presenta en acidosis tubular renal proximal excepto: NaHCO3 y lo alfa


antagonistas.

La hipokalemia produce:
- mayor negatividad del potencial de reposo.

La hipokalemia en un cirrótico puede precipitar COMA por:


- aumento de la amniogénesis

Exceso del kayaxelate en el tratamiento de hiperkalemia puede producir:


- constipación.

Peligro de la hipermagnesemia severa:


- paro respiratorio.

Hipofosfatemia:
- aumenta el calcitriol.

Consecuencia metabólica de hipofosfatemia severa:


- miocardiopatía

Causa de hiperfosfatemia:
- insuficiencia renal

El Síndrome Nefrósicose caracteriza por:


- proteinuria mayor o igual de 3g y albúmina sérica menor o igual de 3g/dl.

En el Síndrome Nefrósico pueden encontrarse:


- cilindros céreos

Urémico agudo, oligúrico, acidótico, deshidratado puede presentar:


- osmolaridad urinaria de 250mosm/kg

La falla renal crónica avanzada cursa con:


- brecha aniónica elevada

Primera*****Falla renal crónica:


a.HTA
b.Glomerulopatias
c.Diabetes Mellitus
d.Nefritis tubulo intersticial

- Uno de los indices de falla renal aguda es:


Uosm/Tosm

- Hormona producida en los riñones que disminuye en la falla renal cronica:


▪ ADH
▪ Noradrenalina
▪ Peptido natriuretico (segun arjona)
▪ 1,25 (OH)2 Vit D.

- Las calcificaciones metastásicas se deben:??


▪ Aumento del producto Ca, P
▪ Aumento del producto Ca, Mg
▪ Hipoparatiroidismo
▪ Hiperparatiroidismo 1ario

- Causa de anuria:
▪ Pielonefritis unilateral
▪ Necrosis tubular aguda
▪ Enf cambios minimo
▪ Necrosis cortical

- la etapa de la insuficiencia renal cronica con tasa de filtración glomerular de 60-


89 ml/min es la:??
▪ 1
▪ 2
▪ 3
▪ 4

- La FENA es la relación entre:


▪ Oferta tubular/Na reabsorbido x 100
▪ depuración de Na/ depuración de creatinina x 100
▪ oferta tubular/ Na excretado x 100.
▪ Na excretado/ Na reabsorvido x 100.

- En la nefritis tubular intersticial aguda se pierde la capacidad de poder concentrar


la orina debida a:??
▪ Daño medular
▪ Daño glomerular
▪ Daño tubular proximal
▪ Daño de la porcion cortical del tubulo colector

- La solucion de NaCl al 23,4 % tiene:


8000 mOsm/l

- Se asocia con el SIAD, excepto:


lipoma

- El efecto biologico mas importante de la calcitonina es, excepto:


▪ Responder a la hipocalcemia
▪ Aumenta la calciuria
▪ Estimular la resorcion osea osteoclastica
▪ Utilizada en el tx de la hipercalcemia.

- Favorece la entrada de K al espacio intracelular, excepto:


▪ Agonista adrenergico
▪ Estimulacion adrenergico
▪ Insulina
▪ Alcalosis metabolica

- Factores que afectan el balance hídrico en el post-operatorio, excepto:


▪ Fármacos
▪ Calcitriol
▪ Administración de liquidos hipotonico
▪ Aumento de la ADH

- Las primeras manifestaciones clínicas de la hiponatremia son:??


▪ neurológica
▪ gastrointestinal
▪ pulmonares
▪ musculares

- En la hipernatremia hipertonica se da lo siguiente excepto:


▪ Entrada de Na
▪ Salida de aa
▪ Ganancia de iones
▪ Disminución del agua cerebral

- Manifestaciones clinicas de la hipercalcemia, excepto:


▪ Taquicardia
▪ HTA
▪ Constipación
▪ Vomitos

- En falla renal aguda por nefrotoxicidad:


▪ Se conserva la membrana basal
▪ Se destruye la membrana basal
▪ Frecuente hay oliguria
▪ La creatinina no aumenta

- Causa de hipomagnesemia:
alcoholismo

- Na serico de 168 mEq/l 70 kg (utilize como % de agua al 50% y Na normal 140


mEq/l
7 litros

- Indicaciones de diálisis en falla renal aguda, excepto:


▪ Encefalopatia uremica
▪ Hiperkalemia refractaria
▪ Ac. Metabolica refractaria
d. hipocalcemia

- El KCL IV se puede asministrar asi, excepto:


a.infusion por linea periferica de 10 meq en 100 ml en 1hora
b. “ “ central de 20 meq em 50ml en 1 hora
c. “ “ central de 100 meq em 50ml en 1 hora
d. “ “ periferica de 20 meq en 20ml en 2 horas

- Aumenta la excrecion urinaria de K, excepto:


o Triamtereno
o Bumetanida
o Furosemida
o Metolazona

- Factores que regulan la secrecion distal de K, excepto:


o Flujo tubular distal y aporte distal de Na
o Excrecion de aniones no reabsorbibles
o Aldosterona
o PTH

- Enfermo de 60 Kg con 60% de agua corporal, Posm 260 mOsm/kg, ¿Cuántos


gramos de NaCl aumentaria el Na en 10 mEq?
a. 21 gramos

50. Conteniendo 10 g de MgSO4 al %, cuantos ml de sal necesitaria?


20 ml

29. Causa de hiperfosfatemia:


e. Falla hepatica
f. Falla renal cronica
g. Falla cardiaca
h. Insuficiencia suprarrenal

30. Causas de hiperfosfatemia grave, excepto:


d. recuperacion de cetoacidosis diabetica.
e. Alcalosis respiratoria
f. Acidosis respiratoria.

31. Paciente con vómitos severos, oligurico con hipotensión arterial Una> 40……
800 mOsm/Kg puede tener:
e. Insuciencia renal aguda.
f. Insuficiencia suprarrenal
g. Falla pre-renal
h. Exceso de mineralocorticoides

32. Paciente urémico crónico con deshidratación severa puede presentar lo siguiente
excepto:
a. Densidad urinaria 1010.
b. Osmolaridad urinaria de 800 mOsm/Kg
c. K sérico aumentado
d. Oliguria.
33. La hipokalemia produce:
a. Aumento de la magnitud del potencial de reposo
17. La PTH promueve todo lo siguiente excepto:
a aumento de la absorción intestinal de calcio
b- aumento de la absorción tubulár de fosfato..
c. aumento de la resorción ósea de fosfato
d- aumento de la resorción osea de calcio
e, disminución de la reabsorción de fosfato

18. Un varon de 57 esta con hemodiálisis de mantenimiento para insuficiencia


renal cronica, ¿Cuál de las sig anomalías metabolicas podría ser anticipáda?
a. hipernatremia
b. hiponatremia
c.
d. exceso de vitamina D
e. hipoparatiroidismo

19. Un px de 25años fue ingresado a UCI por lesiones graves en cabeza y fractura
de la base del Graneo, aprox X horas después de la lesion manifestaba poliuria, la
OSMU 150, y la de suero 350. Los líquidos IV fueron detenidos y 3 hors después
la producción de orina y la osm urinaria permanecieron sin cambio. Se
administraron 5 unidades de vasopresina IV, la osm de la orina se
incremento a 300. ¿Cuál es el dx mas probable?
a- Diabetes insípida central
b. diabetes insípida nefrogenica
c. intoxicación acuosa
d. sobrecarga de solutos
e. S IADH

55. indica falla renal aguda


a- U..N.. = 4.1
b- UN = 1
c. U..N.. = 1/4i/
d. Gravedad especifica 1.026
e. Ninguna

56. deshidratación por perdida de agua y densidad urinaria 1.003, creatinina 6 /


a. falla renal aguda parenquimatosa
b- falla prerenal
c. falla renal obstructiva
d. falla renal cronica
e. no hay falla renal

57. uno de los indices de falla renales


a. reabsorción de Na/oferta tubular de Na x 100
b. excresion urinaria deNa/oferta tubular de x 100
c.- oferta tubular de Na/excrecion urinaria de Na x 100 ~.--~
d- Una/Pna x 100
e. Ninguna

58. el Sdr hepatorenal es causa de falla:


a- prerrenal
b. renal
c. póstrenal
d. falla cardiaca
e. daño tubular

59. causa mas común de falla renal crónica:


A- DM

60- en la glomendonefritis aguda por infeccion, la causa, excepto


a- proteina mayor de 10 g
b- osmolaridad urinaria normal
c. hematuria
d. HT A
e. Edema

61. falla renal cronica, excepto


a. PTH aumentado
b- Hipofosfatemia
c- Acidemia
d. Hipocalcemia
e. 1.25 OH (D3) disminuida

62. EN la falla renal aguda siempre habra


a- Aumento de creatinina
b- fraccion de excrecion de Na aumentado
c- hiperkalemia
d- Una aumentado
e- Hipernatremia

63. causa de falla prerenal


a. Sdr nefrotico
b- obstrucción de ambos uréteres
c- falla cardiaca
d. Grlomerulonefritis aguda post-infeciosa
e. Calculo renal

64. en la nefritis tubular interticial se puede dar, excepto


a. h.ipernatremia
b. proteinuria de 1.5 g
c. normotension arterial
d. hipostenuria
e. hipocalcemia

65. Sugiere glomerulonefritis:


a. leucocituia
b-cilindros eritrocitarios
c- anuria
d. -

66 -causa de falla renal parenquimatosa


a) papilitis
b) calculo en pelvis derecha
c) estenosis unilateral de arteria renal
d) cistitis hemorragica

2--En el riñón la hormona paratiroides:


Aumenta la reabsorción tubular de calcio y disminuye la de fósforo

3- La hipercalcemia produce
Potencialización de los efectos de la hipercalemia

36- La hipomagnesemia produce.


Vasodilatación periférica

36-De los factores: nefrotoxicidad./ -La buena hidratación

37 - En la concentración sérica para calcular:: 8,8

38- la velocidad de administración de potasio: 40 meq

39-hipocalcemia produce : fosfaturia

40-La causa de hipertensión con hipocalemia: Síndrome de Bartter=:,

41-En paciente con vomito severo hipotensión arterial o ir a la sar con mas de 40
meq
Insuficiencia suprarrenal;

42-La ,administración de sol isotónica aumento de vol extracelulares

43-Glomerulonefritis post estreptococcica Altera el cociente de ultrafiltración

44-En-el túbulo` intersticial se puede dar: La proteinuria de 4 g al día.


25. Cual de las siguientes opciones esta contraindicada en el tratamiento de la
hiperkalcmia
a, gluconato de calcio
b. aldactone
c, kayaxelate
d. Bicarbonato de sodio e. Todas las anteriores

43. Rotando en sala de medicina interna le piden que evalue un paciente con HIV
el cual porpresentar una meningitis por criptococci recibe anfotericina B; dicho
este medicamento buscaría como complicación:
a. aparicion de diarrea
b. aumento de la presion arterial
c- hipokalemia
d . cefalea
e. dolor toráxico

81- donde se halla el calcio en el organismo:


a- 50% sangre
b- 20 % proteínas
c- 99% hueso

aumento de PTH
b- aumento de la absorción intestinal de fosfato
c- disminuye fosfatemia
d- aumento de vitamina D

9- una solucion isotonica


a- aumenta el volumen extracell

10- la vel de administración de potasio en caso de riesgo para la vida


a—40 meq por hora

11- Px cronico con deshidratación severa puede presentar los siguiente


excepto:
a-osmolaridad urinaria de 800 mosmol/Kg

12- Px con vomito abundante, HTA, sodio urinario en 6, oligurico,


Osmolaridad de 800 presente,
a- insuficiencia suprarrenal

13- Px uremico cronico con vomito severo hipotension arterial orina al


azar, muestra sodio mas de 40, Osmolaridad de 800 puede tener :
a- Falla suprarrenal

26. Cuándo disminuye el fosfato?


R. Aumento de PTH
27. Ejemplo de hipernatremia hipervolémica:
R. Aldosteronismo primario

28. La hipocalemia produce:


R. Aumento de la magnitud del potencial de reposo

29. La depleción de agua produce, excepto:


R. Salida de electrolitos del cerebro

30. Velocidad de administración de KCl en caso de urgencia:


R. 40 mEq/h

31. Cambios en EKG de la hipermagnesemia es igual:


R. Hipercalemia

32. Enfermo de 60 Kg con 60% de agua corporal, osmolaridad plasmática 260


mosmol/Kg, ¿cuántos gramos de NaCl aumenta el Na en 10 mEq en 24 horas?
R. a) 15
b) 21
c) 30
d) 19

33. Uso de Calcio en tratamiento de hipercalemia para:


R. Modificar el Potencial de Membrana

34. Hipermagnesemia severa produce:


R. Paro respiratorio

35. Venoclisis de 1L de D/A 5% conteniendo 10 mg de MgSO4 y al 50%:


R. 20mL

36. La hipocalcemia:
R. Prolongo QT a expensas del segmento ST

37. Tratamiento de urgencia de hipercalcemia:


R. SSN 0.9%

38. Causa de hiperfosfatemia:


R. Insuficiencia renal

39. Adaptación cerebral, hipertonicidad, excepto:


R. Pérdida de iones

40. En la adaptación cerebral, la hipotensión da lo siguiente, excepto:


R. Ganancia de aminoácidos
41. En ausencia de síntomas de hipernatremia, el sodio debe ser disminuido a razón de:
R. 0.5 mEq/L/h

42. Para corregir un sodio cerca de 185 mEq se administró en 86 horas, 66 letras de D/SS
5%. ¿cuál es la velocidad de administración de la venoclisis?
R. 69 mL ¿?????

43. Manifestación clínica de hipercalcemia, excepto:


R. a) Hipotensión arterial
b) Bradicardia
c) Nauseas
d) Anorexia

44. Consecuencia de hipofosfatemia, excepto:


R. a) No afecta al SNC
b) Disfunción plaquetaria
c) Cardiomiopatía
d) Osteomalacia

45. Causa de hipofosfatemia, excepto:


R. a) Falla renal
b) Recuperación de cetoacidosis diabética
c) Alcalosis respiratoria
d) Quemaduras graves

46. Causa de HTA con hipocalemia, aldosterona-renina baja:


R. Síndrome de Cushing

47. Causas de HTA con hipocalemia y renina alta:


R. Hipertensión maligna

48. En una deshidratación severa hipertónica con hipotensión arterial severa, la prioridad
inicial es administrar:
R. SSN 0.9%

49. En la secreción inapropiada de ADH se da lo siguiente, excepto:


R. No hay expansión de volumen

50. Tejido con mayor contenido de K+:


R. a) Hígado
b) Músculo
c) Eritrocito
d) Hueso
51. La insuficiencia renal funcional de la cirrosis hepática es debida a:
R. Vasoconstricción de las arterias renales
RENAL:

23. Px con IRC tine una taza de filtración glomerular de menor de 25 ml/min ;aunque
hay yuna tabla de las etapas en esta enfemedad crónica.
Estas osn las TFG:
1. 90 en riesgo
2. 20 ( con TFG normal o incrementada)
3. 60 -89
4. 30 – 59
5. 15-29
Menor a 15 diálisis.

24. Factor que altera el balance hídrico en el periódo post- operatorio:


Disminución de la ADH (Excepto)

25. Un ejemplo de hipokalemia con aldosterona alta y osmolaridad baja es:


Hiperaldosteronismo primario

26.Ejemplode hipernatremia hipervolémica: Hiperaldosteronismo primario

27.Hiperkalemioa son alteraciones del EKG no es necesario administrar gluconato de


calcio se colocará un antagonista de K asíe l Px no cae en paro y estoes si tiene
manifestación en el EKG.

28. La fracción de excreción de sodio estará aumentada en falla cardíaca.

29.En la hiperkalemia: hay menos diferencia entre el potencial de reposo y el umbral.

Manifestaciones cardíacas. El problema médico más grave de la hiperkalemia es la


cardiotoxicidad. Los cambios en el ECG producidos por los niveles altos de potasio son
bastante constantes. A medida que aumentan los niveles se aprecian los siguientes
cambios:

Ondas T picudas (con intervalo QT normal o ligeramente reducido)


Prolongación del intervalo PR con depresión de ST
Desaparición progresiva de la onda P
Bloqueo cardíaco progresivo
Arritmias ventriculares
Paro cardíaco

Las ondas T picudas constituyen el dato en el ECG más constante en la hiperkalemia.


Efectos neuromusculares. El primer signo neuromuscular de la hiperkalemia suele ser la
aparición de parestesias seguidas de debilidad progresiva de varios grupos musculares. Si
el cuadro se agrava se observa cuadriplejia fláccida. Las funciones cerebrales y de los
pares craneanos se conservan y la parálisis de la musculatura respiratoria puede ocurrir,
pero es excepcional.

30. La infusión de potasio sin riesgo para la vida del PX: es de 40 meq; porque

I lb de solución salina: debe llevr de 20 a40 mEq de KCl a una velocidad de 10 a 20


mEq por hora; si aumenta la velocidad puede provocar un bloqueo A-V. Se recomienda
una Velocidad mayor si la hipokalemia es muy severa.

Sis e usa venas priféricas es:

10 meq en 100 cc de sol.salina para pasar en una hora

20 meq en 200 cc de sol.salina para pasar en una hora

En vena central:

20 meql en 50 cc pasarla en una hora

40 meq en 100cc pasar la en una hora

31.Los indicios de falla renal aguda son:

(Na filtrado/Na excretado) x 100%

32.Na urinario menor de 10: falla cardíaca, presentará edema estará hinchado y lo
puede llevar a una I.C.C. ( Esto lo saque de Medline: Los valores normales generalmente
son de 15 a 250 mEq/L/día, dependiendo del estado de deshidratación y la ingesta diaria
de sodio en la dieta. Los rangos de los valores normales pueden variar ligeramente entre
diferentes laboratorios. Nota: mEq/L/día = miliequivalentes por litro por día.)

33.Causa de enuresis: necrosis cortical difusa.

34.En SIDH la concentración urinaria es menor de 20.

35. Deshidratación pura de agua excepto: hay salida de electrolitos del cerebro.

36.Urémico crónico con deshidratación hipertónica severa y acidótico presentará lo


siguiento excepto: osmolaridad urinaria de 8mm mosmol/kg.

37. Px con vómitos abundante, hipertensión arterial; Na urinario=6, oliguria, osmolaridad


de 800 presenta: insuficiencia suprarrenal.
38. Factor que predispone a la hipernatremia son intoxicación hídrica.

39. Se presenta en acidosis tubular renal proximal excepto: NaHCO3 y lo alfa


antagonistas.

1. En la Hiperkalemia:
R/. Hay una menor diferencia entre el potencial de reposo y de umbral

2. La hipokalemia en un cirrotico puede desencadenar:


R/. Coma Hepático

3. La hipermagnesemia puede producir:


R/. Paro respiratorio

7. Tratamiento de hiprecalcemia:
R/. SSN 0.9%

8. La hipofosfatemia son causas, excepto:


a. Aumento de la PTH
b. Aumento de la absorción intestinal de fosfato
c. Disminuye la fosfatemia
d. Aumento de la vitamina D

9. Una solución isotónica:


R/. Aumenta el volumen extracelular

13. La velocidad de administración de potasio en caso de gran riesgo para la vida puede
ser:
a. 10-20 mEq/L
b. 8mE/L
c. 110mE/L
d. 40 mEq/h

14. Paciente crónico con deshidratación severa puede presentar lo siguiente excepto:
a. Densidad urinaria 1010
b. Osmolaridad urinaria de 800 mOsm/kg
c. K sérico aumentado
d. Oliguria

15. Paciente con vómito abundante, hipertensión arterial, sodio urinario de 6, oligúrico,
osmolaridad de 800 presenta:
R/.Insuficiencia suprarrenal
Paciente urémico crónico con vómitos severos, hipotensión arterial, orina al azar muestra
Na mas de 40, osmolaridad de 800 puede tener:
Falla suprarrenal

16. Todo lo siguiente es cierto acerca de Colitis Ulcerosa, EXCEPTO:


R/. No está en relación las alteraciones inmunológicas.

17. En quien predomina la enfermedad de Hígado Graso:


R/. Sexo masculino

18. Paciente con Insuficiencia renal crónica, con K sérico de 6, leve academia, EKG
normal, se trata con:
R/. Kayetzalate

22. La hopikalemia predispone, excepto:


R/. Amoniogénesis

25. Cual de las siguientes es causa de hiperfosfatemia:


R/. Insuficiencia renal

26. Paciente con deshidratación hipertónica e hipertensión:


R/. SSN 0.9%

29. En la Glomerulonefritis Aguda post-infecciosa disminuye la tasa de filtración:


R/. Por alteración en el coeficiente de ultrafiltración

30. Causa de Insuficiencia preerenal, excepto:


R/. Aneurisma Disecante de la Aorta

2) Cuál de las siguientes medicinas pueden dar hiperkalemia


f) inhibidor de la enzima convertidota
g) bloqueador alfa adrenérgico
h) bloqueador beta adrenérgico
i) ayb
j) ayc

2) Hormonas que disminuyen en la falla renal crónica, excepto


a) 1,25 dihidroxi vitatamina D
b) eritropoyetina
c) paratohormona
d) foliculo estimulante

3) MgSO4 (PM 120) 1 gramo contiene de magnesio en meq


a) 10
b) 7.9
c) 8.3
d) 6

4) Factores que conducen a la progresión de falla renal crónica


a) hiperlipidemia
b) toxinas urémicas
c) proteinuria
d) b y c
c) todas

5) Paciente de 20 años presenta abundantes vômitos y diarrea de 3 días de evolución,


examen físico depleción de volumen, suero muestra Na 155, K 3meq/l, Cl 117meq/l,
HCO3 25 meq/l, tratamiento de elección:
a) D/A 5% + KCl
b) solución salina 3% + KCl
c) solución salina 23.4% + KCl
d) solución salina 0.45% + KCl

6) En la enfermedad ateroembólica renal se da lo siguiente excepto:


d) Velocidad de eritrosedimentación

7) Pseudohiperkalemia de 8 mEq/l que se ve en el EKG:


Onda T picuda
Aplanamiento de la onda P
Ay B
Ninguna

8)Criterios para iniciar diálisis excepto:


Alcalosiss Metabolica

9)Hipokalemia con normotension excepto


Síndrome de liddle

10) Plasama creatinina 8mg/dl N de urea 100mg/dl U na 40 Meq/l :


Insuficiência renal

11) Factores reversibles responsables de deterioro de la funcion renal excepto:


Fibrosis intersticial

12) Causa hipokalemia con renina y aldosterona disminuida:


Licorice

13. Sugiere trastorno túbulo intersticial:


.Proteinuria mayor a 3.5 g /24 horas
.Notrmotensión
.Edema
.Osmolarida urinaria normal

14. Glomérulonefritis rápidamente progresiva


.Engrosamiento de la membrana basal
Fusión de los pedículos de los podocitos
prolñiferacion intracapilar
proliferación extracapilar ( creo que es esa verificar)

15.Una de las siguientes causas no produce hematuria


.enfermedad de cambios mínimos

16.Plasma Na:115, Posm240 mOsm,Uosm 680,Una 60:meq/l:


.Deshidratación hipotónica
.secreción inapropiada de ADH
.Deshidratación isotónica
.Deshidratación primaria

17.en la falla renal crónca produce necrosis


calcifilaxia

18. No produce hipokalemia:


Pancreatitis aguda
rabdomiolisis
neoplasia
Def. Vit D

19. Paciente con glucosa 90mg/dl, Na+= 115mEq/L, Urea en 140mg/dl


a. Síndrome hipoosmolar
b. Síndrome hiperosmolar
c. Diarrea
d. Ninguna de las anteriores.

20. Causa de seudohiponatremia con osmolaridad elevada


a. Hiperproteinemia
b. Diabetes mellitus
c. Hiperlipidemidemia
d. Reserción transuretral prostática.

21. Manifestaciones de hipermagnesemia excepto


Normotensión

22. Piel en uremia crónica excepto


Paño blanco

23. No es causa de necrosis papilar aguda


a.Diabetes mellitas
b. Uropatía obstructiva
c. Pielonefritis
d. Trombosis de venas renal.
TODAS SON

24. Puede presentarse hiperuricemia excepto


d. Adenocarcinoma gástrico

25. Compromete riñones y pulmones:


a. Enf de GoodPasture
b. Granulomatosis de Wegener
c. LES
d. Ay C

26. Sugiere glomerulopatia:


-anasarca

27. TFG < 15ml/min corresponde al estadio de la enfermedad renal cronica:


* 4

28. Causa obstetrica de necrosis cortical bilateral


- desprendimiento prematuro de placenta

29. Promueve amoniogenesis em cirróticos


- hipokalemia

30. Criterio para diagnostico de síndrome nefrosico


a. Proteinuria en 24 hr de 3.5 gr para superficie corporal de 1. 73
b. Albumina serica < 3g/dl
c.Hiperlipidemia y edema
d. Todas las anteriores
e. ay b

101. Paciente masculino de 33 años llega a su consulta por presentar fiebre, un pequeño
rash, signo de Jordano positivo; el medico de turno le envía un urinalisis que revela
proteinuria, hematuria y múltiples leucocitos; dentro de su diagnostico diferencial, debe
estar una de las siguientes patologías, cual:

a. Nefritis Intersticial Crónica.


b. Nefritis Intersticial Aguda. Rash, leucositosis puede ser un LES
c. Síndrome Nefrosico.
d. Fracaso Renal Agudo.
102. Dentro de las Glomerulopatias, existen algunas, que se pueden presentar con
Síndrome Nefrosico, y otras con Síndrome Nefrítico, cual de las siguientes se suele
presentar con Síndrome Nefrítico:

a. Cambios Mínimos.
b. Nefropatia IgA.
c. Nefropatia Membranosa.
d. Diabetes.

103. En un paciente de 65 años, que desarrolla un Sd. Nefrítico Agudo, la etiología mas
común causando de la Glomerulopatia es:

a. Vasculitis.
b. GN rápidamente progresiva.
c. Nefropatia IgA.
d. LES.

104. Cual de las siguientes Glomerulopatia, siempre desarrolla Insufiencia Renal:

a. Cambios Mínimos.
b. Membranosa.
c. Rápidamente Progresiva.
d. Mesangiocapilar.

105. En un paciente con SIADH, al medirle la osmolaridad urinaria, la podemos


encontrar:

a. Elevada.
b. Disminuida.
c. Normal.

106. La hiponatermia, se hace clínicamente evidente cuando el sodio, ha disminuido a:

a. 110 mEq/lt.
b. 115 mEq/lt.
c. 120 mEq/lt.
d. 125 mEq/lt.

107-111. Mencione 5 medicamentos que puedan causar hiperkalemia:

a. ________________aldactone________________________________.
b. ________________captopril________________________________.
c. ______________losartan__________________________________.
d. ______________carvedilol__________________________________.
e. ______________________lisinopril__________________________.
112-117. Las características del Síndrome Nefrotico son:
a. _________________edema_______________________________.
b. ______________________hipoalbuminemia__________
c. ______________hipercoagulabilidad_______________________.
d. ___________proteinuria >3.5_____________________________________.
e. _________________hiperlipidemia________________________.
f. __________________cilindros lipidicos en orina__.

118-120. Los tres puntos en los que se basa la corrección de la hiperkalemia son:

a. ___________aum de excresion renal___________.


b. _________dism de trasvase celular de K (salida de celulas
c. _______disminucion d eposicion de K________________.

121-123. Para cada uno de estos puntos, se suele usar: se le solicita al estudiante ser lo
mas explicito posible en su respuesta, en cuanto a dosis y tratamiento exacto.

a. ____SSN 0.9, gluconato cálcico y aument de Na renal_______.


b. Insulina,dism relajantes musculares________________.
c. Dieta sin K platano, tomate, naranja, espinaca. Quitar fármaco que aumente K

124-126. Debemos sospechar, de alteraciones del Magnesio, en que pacientes:

a. _____ALCOHOLICOS,QUE USEN DIURETICOS DE ASA____.


b. ______CIRROTICOS, SD DE MALA ABSORCION _________.
c. _USEN LAXANTES, TX DE PREECLAMSIA, _________.

127. Paciente con hipokalemia + alcalosis metabólica + hipertensión arterial +


disminución de la renina plasmática, esta desarrollando un:

a. ______________________HIPERTENSION MALIGNA CREO!!_.

128. Única causa de diarrea con alcalosis metabólica es:

a. __HIPOPOTASEMIA →SD DE GITELMAN O BARTTER_____________.

129. Si un paciente, tiene un potasio de 3 mEq/lt, decimos que ha perdido ya:

a. _________.

130. Cual es la principal preocupación que se tiene, cuando un paciente tiene un aumento
en los niveles plasmáticos de Magnesio:

a. PARO RESP_________________.
131-132. Causas de acidosis metabólica con hipokalemia:

a. ________diarrea con perdida de K______.


b. INTOXICACION POR TOLUENO,
c. EXCRESION DE ANION NO REABSSORBIBLE_____CAD____.

133. Que examen le ayudaria para diferenciarlas:

a. POTASIO EN ORINA, ESTADO AC BASE,

134. Que esperaria encontrar en una ATR tipo 2:

a. ________HIPOCALEMIA Y AC METABOLICA____.

135-136. Principales causas de hipercalcemia:

a. ____HIPERPARATIROIDISMO 1______.
b. __________NEOPLASIA MALIGNA POR TUMOR SECRETOR DE PTH

136-137. Principales causas de hipocalcemia:

a. ______HIPOPARATIROIDISMO ___(SD DI GEORGE, CONGEITO,


QUIRURGICO_______.
b. ______DEFICIENCIA DE VIT D (_____SD NEFROTICO IRC___.

138-139. Consecuencias de la hiperfosfatemia:

a. _________HIPERPARATIROIDISMO_______________.
b. ____________________CALCIFICACION ECTOPICA____
140-143. El Sd. De Barter, se caracteriza clínicamente por:

a. __________________HIPOPOTASEMIA_________
b. ___ALCALOCIS MET________________________________.
c. ________HIPERCALCIURIA___________________________.
d. _______hipocloremia, nefrocalcinosis_________________.

144. En un paciente con diagnostico de Glomerulonefritis post estreptococcica, el


diagnostico serologico se basa en el hallazgo de anticuerpos:

a. Antiestreptolisina O.
b. Antiestreptolisina A.
c. Antiestreptolisina P.
d. Antiestreptolisina K.

145. En la Nefiritis secundaria al LES, se suelen encontrar positivos los:


a. ANA.
b. ANCA c.
c. ANTI LU
d. ANCA p.

146. Si un paciente es diagnosticado con Falla Renal Crónica, el siguiente punto es el


estadiage del mismo, si tiene una TFG en 82 ml/minuto, esta en estadio:

a. I
b. II
c. III
d. IV
e. V

147-148. La Osteodistrofia Renal, se va a caracterizar clínicamente por la presencia de:

a. ______DOOR OSEO, FRACTURAS, HIPERPTH 2__________.


b. _____
149.Normalmente la Kps entre el Fósforo y el calcio es de:

a. ________________________________________________.

150-154. Las características de una muestra de orina con sedimento de tipo nefrítico, son:

a. ___________HEMATURIA_>5__
b. ______PROTEINURIA__<3.5______________.
c. ________LEUCOS_________________.
d. CILINDROS_ERITROCITARIOS___________________.
e. ________________________________________________.

155-158. Mencione algunos parámetros de laboratorio, para pensar en una IRC Renal:

a. ___________ALBUMINURIA
b. _________hipercloremia___________________________.
c. __________hiperfosfatemia_______BUN
AUMENTADO_____________________.
d. ______proteinuria, hematuria_____
e. Cilindros hialinos con sedimento normal, TFG DISMINUIDA, CRET
AUMENTADA____________________.

159. El Gasto cardiaco normal es de 5 – 6 litros por minuto; de los cuales, un 25% le
corresponde al riñón, en una relación de:

a. 1200 ml/min.
b. 1300 ml/min.
c. 1400 ml/min.
d. 1500 ml/min.

160. Si un paciente, diagnosticado con FRC, presenta ademas una acidosis metabolica
hipercloremica, usted sabe que este paciente debe tener una Brecha Anionica:

a. Normal.
b. Aumentada.
c. Disminuida.
d. Ninguna de las anteriores.

161. Este mismo paciente, por tener estas características clínicas, debe encontrarse en
estadios de la FRC:

a. Avanzados.
b. Iniciales.
c. No es relevante.
d. Ninguna de las anteriores.

162. Para que por una obstrucción, aumente la creatinina en IRA, se deben haber perdido:

a. ¼ partes de la masa renal.


b. 2/4 partes de la masa renal.
c. ¾ partes de la masa renal.
d. 4/4 partes de la masa renal.

163. La lesión histologica mas común de IRA es:

a. Necrosis cortical bilateral.


b. Necrosis tubular aguda.!!!
c. Nefritis intersticial.
d. Papilitis necrotizante.

164. Y la mas peligrosa y temida es:

a. Necrosis cortical bilateral.


b. Necrosis tubular aguda.
c. Nefritis intersticial.
d. Papilitis necrotizante.

165. Cual de las siguientes, es la principal causa de IRA, en la maternidad.

a. Abortos infectados.
b. Fetos muertos retenidos.
c. Desprendimiento prematuro de membrana.
d. Eclampsia.
e. Se dan todas por igual.
13. Favorece la penetración de amoniaco al cerebro situaciones de:
a) Acidosis metabólica
b) Alcalosis con Hipopotasemia
c) Acidosis con Hiperpotasemia
d) Alcalosis con hiperpotasemia
3. Paciente que pesa 50Kg, con sodio sérico de 160 mEq/dL , con ACT =40%
, su déficit de agua es:
a) 5L b) 3L c) 4L d) 6L
2. Para un calcio total sérico de 7mg/dl y albúmina de 2,2 g/dl ¿Cuál será
el calciototal
corregido? Tome el valor de la albúmina normal de 4 g/dl
a) 7,44 mg/dl b) 8,44 mg/dl c)9,44 mg/dl d) 8,90 mg/dl
3. Velocidad de administración de KCl en condiciones normales, en
mEq/hora
a) 8 b) 120 c) 20 d) 40
4. Causa falla prerrenal
a)Obstrucción de ambos uréteres b)Falla cardiaca c)
Glomérulonefritis aguda d) Síndrome nefrósico
5. Manifestación en el EKG de un paciente con Hipokalemia:
a) Onda T picuda b) Supradesnivel del segmento ST
c) Intervalo QT deprimido d) Onda S profunda
e) Prolongación del intervalo PR f) N/A
6. Cuál es la concentración sérica de fósforo por encima de la cual se
considera que una persona tiene Hiperfosfatemia (en mg/dL):
a) 3.0 b) 4.0 c) 5.0 d) 6.0 e) 7.0
7.. Diurético de uso común en el tratamiento del edema en los
síndromes
nefrítico y nefrótico:
a) Indapamida b) Espironolactona c) Furosemida d)
Acetazolamida
e) Hidroclorotiazida f) N/A
8. Cuál es el principal tratamiento para pacientes con nefritis
tubulointersticial
alérgica:
a) Prednisona b) Enalapril c) Ibuprofeno d) Diálisis e) N/A
9. Cuál de las siguientes enfermedades puede producir un Síndrome
Nefrítico
mediado por inmunocomplejos
a) Granulomatosis de Wegener b) Síndrome de Goodpasture
c) Púrpura Trombocitopénica Trombótica d) Amiloidosis e)
N/A
10. Ejemplo de hiponatremia hipotónica normovolémica
a)Diarrea B)Cirrosis hepática c) Síndrome nefrósico d) SIADH
11. Mencione cuál de las siguientes condiciones puede ser causa de
Hiperkalemia:
a) Uso de IECA b) Hiperaldosteronismo primario c)
Hiperinsulinemia d) Alcalosis Metabólica e) N/A
12. Mencione cuál de las siguientes condiciones cursa con el
desarrollo de
Enfermedad Crónica de los Riñones:
a) Nefrolitiasis unilateral b) Enfermedad de Cambios Mínimos
c) Nefropatía Diabética d) Carcinoma de Riñón e) N/A
13. Oliguria implica una diuresis menor de (mL/día):
a) 50 b) 400 c) 800 d) 1000 e) N/A
14. Señale la aseveración Incorrecta acerca de la Hormona
Antidiurética:
a) Polipéptido sintetizado en el hipotálamo
b) También se le conoce como Arginina Vasopresina
c) El efecto neto de su acción a nivel de la nefrona es la
reabsorción
pasiva de agua y sodio
d) El mayor estímulo para su secreción es la hipertonicidad
15. Generalmente la proporción BUN / creatinina plasmática en
hiperaazoemia
prerrenal es de:
a) <10 b) > 20 c) 100% d) d) 1%
16. El sodio urinario en hiperazoemia renal intrínseca es de:
a) <10 b) >20 c) > 30 d) >1
18. La causa principal del síndrome nefrítico agudo es :
a) Hipertensión arterial b) GN postestreptocóccica c) Síndrome
deGoodpasture
d) Nefritis lúpica
18. Cuál es el componente esencial en el síndrome nefrótico?.
a) hipoalbuminemia b) hiperlipidemia c) proteinuria d)
edema
19. Cuadro clínico clásico del síndrome nefrítico incluye lo siguiente
excepto:
a) lipiduria b)hipertensión c) Oliguria d) Hematuria
20. El tx en cuarto de urgencias de hipercalcemia intensa es:
a) Bifosfonatos b) Furosemida y SSisotónica c) Furosemida y
Bifosfonatos
d) SSisotónica +Calcitriol.

Toxicología
1. Paciente intoxicado con medicamento para los ojos presenta: rubor,
mucosas secas, Intranquilo y retención urinaria su tx incluiría:
a)Fisostigmina b) Atropina c) Propranolol d) Flumazenil

4. Paciente rociado con un tóxico llamado Malatión podrá presentar los


siguientes síntomas muscarínicos excepto:
c) lagrimeo b) sialorrea c) broncoespasmo d) rubicundez
d)

3. Paciente en coma porque se tomó 30 pastillas de alprazolam, enojado con su


novia. Usted utilizaría el siguiente antídoto:
a) Atropina b) Flumazenil c) Protamina d) Fisostigmina

4. Cuál de las siguientes no es una medida para evitar absorción del tóxico:
a) Descontaminación oral b) D. digestiva c)Diuresis forzada d) D.
Cutánea

5. Pte mordido por una culebra , no presenta signos al inicio, 24 horas después
presenta ptosis Palpebral. La intoxicación ofídica la clasificaría como:
a) Síndrome Botrops b) S. Elapídico c) S. Clotalico d) S. Botulínico

6. Pte mordido por una culebra en el pie , con aparición de equimosis , edema
hasta 1/3 medio de la pierna , con TPT y TP alterados ligeramente la
intoxicación ofídica se clasifica como:
a) Grado 0 b) leve c) moderada d) Severa

7. El tx con suero antiofídico del paciente anterior sería así:


a) < 5frascos b) 5-9 frascos c) 15 frascos d) > 15 frascos.

8. El tx de las arritmias ventriculares por intoxicación con cocaína incluye:


a) Digoxina b) Amiodarona c) Nifedipina d) Propranolol
9. Pte de 18 años, llega con sialorrea, miosis pupilar, broncoespasmo, dice
que tomó “Baygon”, usted lo trataría con:
a) Fisotigmina b) Atropina c) Flumazenil d) N- acetilcisteína

10. Tx sugerido para el intoxicado por Paraquat:


a) O2 b) Tierra fuller c) Diazepam d) Atropina

11. Intoxicaciones que cursan con excitación todo lo siguiente excepto:


a) Atropina b)Opiaceos c) Abstinencia d) Teofilina

12. Antídoto para Heparina es:


a) Protamina b) Coumadin c) Vitamina K d) Plasma fresco

13. Intoxicación por Malatión usted usaría como Antídoto


a)Fisostigmina b) Atropina c) Propranolol d) Flumazenil

14. Prioridad del manejo general en un intoxicado:


a) Canalizar vena b) Oxigenoterapia c) Glasgow d) Permeabilidad de
vías aéreas

Renal

1. Paciente urémico crónico con deshidratación severa puede presentar lo siguiente


excepto:
a) Densidad urinaria 1010.
b) Osmolaridad urinaria de 800 mOsm/Kg
c) K sérico aumentado
d) Oliguria.

2. Se puede usar en la hiperfosfatemia de la IRC;


a. carbonato de Ca2+ b. Acetato de Ca2+ c. Resticción de fósforo d. Paricalcitol
e. Todas las anteriores

3. Causa de falla renal durante el embarazo, excepto:


a. Aborto séptico b. Desprendimiento prematuro de la placentac. Eclampsia d.
Hígado graso e. Obstrucción uretral derecha

4. La anemia de la falla renal crónica se debe principalmente:


a. deficiencia de hierro b. Deficiencia de folato c. deficiencia de
cianocobalamia d. Deficiencia de eritropoyetina e. Todas las anteriores
5. Una de las alteraciones frecuentes de la falla renal crónica se deben
principalmente:
a. prurito b. Xerodermia c. Calcifilaxis d. Hiperqueratosis folicular e. N/A.

6. Factores de riesgo para desarrollar IRC por el uso de medio contraste excepto:
a. edad avanzada b. ICC severa c. Uso de agentes potencialmente
nefrotóxicos d. NO SE VE, tiene algo que ver con la cantidad de medio de contraste
que usan el volumen máximo debe ser: 5mL x peso en Kg/ creatinina sérica.

7. Puede haber pérdida de la capacidad de concentrar la orina, excepto:


a. diabetes insípida nefrogénica b. Diabetes insípida central c. Anemia falciforme
d. Pielonefritis aguda severa e. GMN aguda

8. El término histopatológico GMN proliferativa intracapilar y exudativa


corresponde a:
a. Sdme. Nefrósico b. Sdme. De Goodpasture c. Granulomatosis de
Wegener d. GMN post-infecciosa e. Sdme. Nefrosico de cambios mínimos

9. Sugiere trastorno del túbulo intersticial:


a. pérdida de la capacidad de concentrar la orina b. HTA c. Edema d. Proteinuria >2g e.
Macrohematuria

10. Causa de falla prerrenal:


a. Necrosis tubular aguda b. Necrosis cortical c. Papilitis necrotizante d. Insuficiencia
cardíaca severa e. N/A

11. sedimento urinario en el síndrome nefrítico:


a. cilindros anchos b. Eritrocitos dismórficos c. Cilindros de eritrocitos d. B y
C e. Todas las anteriores

12. El síndrome nefrósico de cambios mínimos se caracteriza por:


a. Proliferación intracapilar
b. Engrosamiento de la membrana basal
c. Hiperplasia mensangial
d. Proliferación endotelial
e. Fusión de los pedicelos de los podocitos

13. En una nefritis tubulointersticial aguda, se puede encontrar en la orina:


a. Abundantes eritrocitos
b. cilindros anchos
c. Eosinófilos
d. Gravedad específica de 1.030
e. Cuerpos óvalos
14. En el tx no dialitico de un paciente con falla renal cronica, con oliguria.hinchado
hipertenso y con sodio de 120mM/L
RESTRICCION DE LIQUIDOS Y RESTICCION DE SAL. DIURETICOS DE
ASA

15. En la falla renal cronica con oliguria se puede presentar excepto:


a. Exceso de PTH b. Normokalemia c. Hiperfosfatemia d. Hipocalcemia
e. Anemia normocítica normocrómica.

16. Indicaciòn para dializar.


a. Hipocalcemia severa
b. Náuseas
c. Edema de MI
d. pericarditis
ademas tambien: sobrecarga de volumen que no responde a
diureticos,hiperpotasemia.acidosis metabolica que no responde,encefalopatia,).
e. Pérdida de la capacidad de concentrar la orina

17. Pensar en obstruccion de vias urinarias en caso de :


a. Poliuria b. Oliguria c. Anuria d. Polaquiuria e. Mayor aumento de
creatinina sérica que de urea.

18. Disfuncion renal debida a hipercalcemia excepto:


a. poliuria
b. acidosis tubular renal tipo IV
c. nefrolitiasis
d. nefrocalcinosis
e. diabetes insipida nefrogenica.

1) Primera cauda de Falla renal crónica:


a.HTA
b.Glomerulopatias
c. Diabetes Mellitus
d.Nefritis tubulo intersticial

2) Uno de los 64 necesita de falla renal aguda es:


a) Uosm/Posm

3) Causa de anuria:
a) Pielonefritis unilateral
b) Necrosis tubular aguda
c) Enf cambios minimo
d) Necrosis cortical

4) En la nefritis tubular intersticial aguda se pierde la capacidad de poder


concentrar la orina debida a:
a) Daño medular
b) Daño glomerular
c) Daño tubular proximal
d) Daño de la 65ecesit cortical del tubulo colector

5) En falla renal aguda por nefrotoxicidad:


a) Se conserva la membrana basal
b) Se destruye la membrana basal
c) Frecuente hay oliguria
d) La creatinina no aumenta

6) Indicaciones de diálisis en falla renal aguda, excepto:


a) Encefalopatia 65ecesit
b) Hiperkalemia refractaria
c) Ac. Metabolica refractaria
d) Hipocalcemia

7) En IRA se mide todo menos:


a) Uosm/Posm x100
b) Na excretado/ Na filtrado x200
c) Na filtrado/ Na excretado x100
d) Na urinario >40
e) Ninguna de las anteriores

8) En el síndrome nefrosico puede encontrarse cilindros:


a) cereos
b) hematicos
c) leucocitarios
d) epiteliales
e) mixtos leucocitario y hematicos

Paciente masculino de 33 años llega a su consulta por presentar fiebre, un pequeño


rash, signo de Jordano positivo; el medico de turno le envía un urinalisis que revela
proteinuria, hematuria y múltiples leucocitos; dentro de su diagnostico diferencial,
debe estar una de las siguientes patologías, cual:
e. Nefritis Intersticial Crónica.
f. Nefritis Intersticial Aguda.
g. Síndrome Nefrosico.
h. Fracaso Renal Agudo.

Dentro de las Glomerulopatias, existen algunas, que se pueden presentar con


Síndrome Nefrosico, y otras con Síndrome Nefrítico, cual de las siguientes se suele
presentar con Síndrome Nefrítico:
e. Cambios Mínimos.
f. Nefropatia IgA.
g. Nefropatia Membranosa.
h. Diabetes.

En un paciente de 65 años, que desarrolla un Sd. Nefrítico Agudo, la etiología mas


común causando de la Glomerulopatia es:
e. Vasculitis.
f. GN rápidamente progresiva.
g. Nefropatia IgA.
h. LES.

Cual de las siguientes Glomerulopatia, siempre desarrolla Insufiencia Renal:


e. Cambios Mínimos.
f. Membranosa.
g. Rápidamente Progresiva.
h. Mesangiocapilar.
Creo…

Las características del Síndrome Nefrotico son:


g. _hipoalbuminemia mayor de 3g /dl.
h. _3.5 g de preoteina en orina en 24 h en 1.73m2 de superficie
corporal____________________________.
i. hiperlipidemia.
j. ___________________edema
k. _hipertension
l. cilindro cereo

En un paciente con diagnostico de Glomerulonefritis post estreptococcica, el


diagnostico serologico se basa en el hallazgo de anticuerpos:
e. Antiestreptolisina O.
f. Antiestreptolisina A.
g. Antiestreptolisina P.
h. Antiestreptolisina K.

En la Nefiritis secundaria al LES, se suelen encontrar positivos los:


e. ANA.
f. ANCA c.
g. ANTI LU
h. ANCA p.

Si un paciente es diagnosticado con Falla Renal Crónica, el siguiente punto es el


estadiage del mismo, si tiene una TFG en 82 ml/minuto, esta en estadio:
f. I
g. II
h. III
i. IV
j. V
Las características de una muestra de orina con sedimento de tipo nefrítico, son:
f. microhematuria
g. cilindro hematico

Mencione algunos parámetros de laboratorio, para pensar en una IRC Renal:


f. creatinina
g. hemoglobina
h. BUN
i. Aumento del calcio y del fósforo

El Gasto cardiaco normal es de 5 – 6 litros por minuto; de los cuales, un 25% le


corresponde al riñón, en una relación de:
e. 1200 ml/min.
f. 1300 ml/min.
g. 1400 ml/min.
h. 1500 ml/min.

Si un paciente, diagnosticado con IRC, presenta ademas una acidosis metabolica


hipercloremica, usted sabe que este paciente debe tener una Brecha Anionica:
e. Normal.
f. Aumentada.
g. Disminuida.
h. Ninguna de las anteriores.

Este mismo paciente, por tener estas características clínicas, debe encontrarse en
estadios de la IRC:
e. Avanzados.
f. Iniciales.
g. No es relevante.
h. Ninguna de las anteriores.

Para que por una obstrucción, aumente la creatinina en IRA, se deben haber
perdido:
e. ¼ partes de la masa renal.
f. 2/4 partes de la masa renal.
g. ¾ partes de la masa renal.
h. 4/4 partes de la masa renal.

La lesión histologica mas común de IRA es:


e. Necrosis cortical bilateral.
f. Necrosis tubular aguda.
g. Nefritis intersticial.
h. Papilitis necrotizante.

la mas peligrosa y temida es:


e. Necrosis cortical bilateral.
f. Necrosis tubular aguda.
g. Nefritis intersticial.
h. Papilitis necrotizante.

Cual de las siguientes, es la principal causa de IRA, en la maternidad.


f. Abortos infectados.
g. Fetos muertos retenidos.
h. Desprendimiento prematuro de la placenta.
i. Eclampsia.

Un paciente de 35 años con insuficiencia renal crónica secundaria a pielonefritis


crónica recibe un trasplante renal de cadáver con el que compartía dos identidades
en A y B y una en DR. Recibe tratamiento inmunosupresor con ciclosporina A y
corticoides a dosis estándar. En el postoperatorio inmediato se observa buena
diuresis y no es necesario el tratamiento sustitutivo con hemodiálisis. En el 5º día de
evolución, el paciente presenta fiebre de 38ª, TA de 180/110, oliguria y disminución
en la concentración urinaria de sodio. El diagnóstico más probable sería:

a. Crisis hipertensiva.
b. Infección respiratoria.
c. Pielonefritis aguda del injerto renal.
d. Recidiva de su enfermedad renal.
e. Rechazo agudo del injerto renal.

Causa más comun de falla renal cronica:


j. Diabetes melitus
k.
HTA
l.
Glomerulonefritis
m.
Enfermedad poliquística renal
n.
Vasculitis

Deshidratación por perdida de agua y densidad urinaria 1.003, creatinina 6 /


a. falla renal aguda parenquimatosa
b- falla prerenal
c. falla renal obstructiva
d. falla renal cronica
e. no hay falla renal

Urémico crónico con K 6.5 con leve acidemia, sin trastorno muscular y EKG
normal, tratamiento:
o Kayexalate.

La IRC ocaciona hiperfosfatemia permanente cuando la TFG esta por debajo de:
o 25 ml/min
La fracción de excreción de Na esta aumentada en:
o Falla renal aguda

Un varon de 57 esta con hemodiálisis de mantenimiento para insuficiencia renal


cronica, ¿Cuál de las sig anomalías metabolicas podría ser anticipáda?
a. hipernatremia
b. hiponatremia
c. Osteomalacia
d. exceso de vitamina D
e. hipoparatiroidismo

uno de los indices de falla renales


a. reabsorción de Na/oferta tubular de Na x 100
b. excresion urinaria de Na/oferta tubular de x 100
c.- oferta tubular de Na/excrecion urinaria de Na x 100 ~.--~
d- Una/Pna x 100
e. Ninguna

falla renal cronica, excepto


a. PTH aumentado
b- Hipofosfatemia
c- Acidemia
d. Hipocalcemia
e. 1.25 OH (D3) disminuida

TFG < 15ml/min corresponde al estadio de la enfermedad renal cronica:


5

Hormona producida en los riñones que disminuye en la falla renal cronica:


e) ADH
f) Noradrenalina
g) Peptido natriuretico (segun arjona)
h) 1,25 (OH)2 Vit D. (disminuye a partir del estadío 3)

la etapa de la insuficiencia renal cronica con tasa de filtración glomerular de 60-89


ml/min es la:
i) 1
ii) 2
iii) 3
iv) 4

Causa de hipomagnesemia
i) cirrosis avanzada
j) falla renal cronica
k) falla cardiaca
l) constipacion
m) acidosis respiratoria

Causa renal de hipomagnesemia


n) falla renal cronica
o) esteatorrea
p) desnutricion severa
q) uso de diureticos de asa
r) infeccion urinaria

Si un paciente es diagnosticado con Falla Renal Crónica, el siguiente punto es el


estadiage del mismo, si tiene una TFG en 50 ml/minuto, esta en estadio:
a. I
b. II
c. III
d. IV
e. V

En pte con falla renal crónica que ha sido transplantado esperamos encontrar:
f. osteomalacia.

Enfermedad Crónica de los Riñones:


a) Nefrolitiasis unilateral b) Enfermedad de Cambios Mínimos
c) Nefropatía Diabética d) Carcinoma de Riñón e) N/A

Hormonas que disminuyen en la falla renal crónica, excepto


a) 1,25 dihidroxi vitatamina D
b) eritropoyetina
c) paratohormona
d) foliculo estimulante

Factores que conducen a la progresión de falla renal crónica


a) hiperlipidemia
b) toxinas urémicas
c) proteinuria
d) b y c
c) todas
34. Causa de hiperfosfatemia:
i. Falla hepatica
j. Falla renal cronica
k. Falla cardiaca
l. Insuficiencia suprarrenal

35. Causas de hiperfosfatemia grave, excepto:


g. recuperacion de cetoacidosis diabetica.
h. Alcalosis respiratoria
i. Acidosis respiratoria.

36. Paciente con vómitos severos, oligurico con hipotensión arterial Una> 40……
800 mOsm/Kg puede tener:
i. Insuciencia renal aguda.
j. Insuficiencia suprarrenal
k. Falla pre-renal
l. Exceso de mineralocorticoides

37. Paciente urémico crónico con deshidratación severa puede presentar lo siguiente
excepto:
a. Densidad urinaria 1010.
b. Osmolaridad urinaria de 800 mOsm/Kg
c. K sérico aumentado
d. Oliguria.

38. La hipokalemia produce:


a. Aumento de la magnitud del potencial de reposo
b. Hace más negativo el potencial de transmembrana

39. Son drogas hepatotoxicas excepto:


a. Ticlopidina
b. Metil dopa
c. Fluoxetina
d. Pravastatina

40. Primera*****Falla renal crónica:


a.HTA
b.Glomerulopatias
c. Diabetes Mellitus
d.Nefritis tubulo intersticial

41. Uno de los indices de falla renal aguda es:


Uosm/Tosm
Uosm/Posm

42. Hormona producida en los riñones que disminuye en la falla renal cronica:
a. ADH
b. Noradrenalina
c. Peptido natriuretico (segun arjona)
d. 1,25 (OH)2 Vit D.

43. Las calcificaciones metastásicas se deben:??


a. Aumento del producto Ca, P
b. Aumento del producto Ca, Mg
c. Hipoparatiroidismo
d. Hiperparatiroidismo 1ario

44. Causa de anuria:


a. Pielonefritis unilateral
b. Necrosis tubular aguda
c. Enf cambios minimo
d. Necrosis cortical

45. la etapa de la insuficiencia renal cronica con tasa de filtración glomerular de 60-
89 ml/min es la:??
a. 1
b. 2
c. 3
d. 4

46. La FENA es la relación entre:


a. Oferta tubular/Na reabsorbido x 100
b. depuración de Na/ depuración de creatinina x 100
c. oferta tubular/ Na excretado x 100.
d. Na excretado/ Na reabsorvido x 100.
47. En la nefritis tubular intersticial aguda se pierde la capacidad de poder concentrar
la orina debida a:??
a. Daño medular
b. Daño glomerular
c. Daño tubular proximal
d. Daño de la porcion cortical del tubulo colector

48. La solucion de NaCl al 23,4 % tiene:


8000 mOsm/l

49. Se asocia con el SIAD, excepto:


lipoma

50. El efecto biologico mas importante de la calcitonina es, excepto:


a. Responder a la hipocalcemia
b. Aumenta la calciuria
c. Estimular la resorcion osea osteoclastica
d. Utilizada en el tx de la hipercalcemia.

51. Favorece la entrada de K al espacio intracelular, excepto:


a. Agonista adrenergico
b. Estimulacion adrenergico
c. Insulina
d. Alcalosis metabolica

52. Factores que afectan el balance hídrico en el post-operatorio, excepto:


a. Fármacos
b. Calcitriol
c. Administración de liquidos hipotonico
d. Aumento de la ADH

53. Las primeras manifestaciones clínicas de la hiponatremia son:??


a. neurológica
b. gastrointestinal
c. pulmonares
d. musculares

54. En la hipernatremia hipertonica se da lo siguiente excepto:


a. Entrada de Na
b. Salida de aa
c. Ganancia de iones
d. Disminución del agua cerebral

55. Manifestaciones clinicas de la hipercalcemia, excepto:


a. Taquicardia
b. HTA
c. Constipación
d. Vomitos

56. En falla renal aguda por nefrotoxicidad:


a. Se conserva la membrana basal
b. Se destruye la membrana basal
c. Frecuente hay oliguria
d. La creatinina no aumenta

57. Causa de hipomagnesemia:


alcoholismo

58. Na serico de 168 mEq/l 70 kg (utilize como % de agua al 50% y Na normal 140
mEq/l
7 litros

59. Indicaciones de diálisis e


60. n falla renal aguda, excepto:
a. Encefalopatia uremica
b. Hiperkalemia refractaria
c. Ac. Metabolica refractaria
d. hipocalcemia

61. El KCL IV se puede asministrar asi, excepto:


a.infusion por linea periferica de 10 meq en 100 ml en 1hora
b. “ “ central de 20 meq em 50ml en 1 hora
c. “ “ central de 100 meq em 50ml en 1 hora
d. “ “ periferica de 20 meq en 20ml en 2 horas

62. Aumenta la excrecion urinaria de K, excepto:


a. Triamtereno
b. Bumetanida
c. Furosemida
d. Metolazona

63. Factores que regulan la secrecion distal de K, excepto:


a. Flujo tubular distal y aporte distal de Na
b. Excrecion de aniones no reabsorbibles
c. Aldosterona
d. PTH

64. Enfermo de 60 Kg con 60% de agua corporal, Posm 260 mOsm/kg, ¿Cuántos
gramos de NaCl aumentaria el Na en 10 mEq?
a. 21 gramos
50. Conteniendo 10 g de MgSO4 al %, cuantos ml de sal necesitaria?
20 ml

52. Cuándo disminuye el fosfato?


R. Aumento de PTH

53. Ejemplo de hipernatremia hipervolémica:


R. Aldosteronismo primario

54. La hipocalemia produce:


R. Aumento de la magnitud del potencial de reposo

55. La depleción de agua produce, excepto:


R. Salida de electrolitos del cerebro

56. Velocidad de administración de KCl en caso de urgencia:


R. 40 mEq/h

57. Cambios en EKG de la hipermagnesemia es igual:


R. Hipercalemia

58. Enfermo de 60 Kg con 60% de agua corporal, osmolaridad plasmática 260


mosmol/Kg, ¿cuántos gramos de NaCl aumenta el Na en 10 mEq en 24 horas?
R. a) 15
b) 21
c) 30
d) 19

59. Uso de Calcio en tratamiento de hipercalemia para:


R. Modificar el Potencial de Membrana

60. Hipermagnesemia severa produce:


R. Paro respiratorio

61. Venoclisis de 1L de D/A 5% conteniendo 10 mg de MgSO4 y al 50%:


R. 20mL

62. La hipocalcemia:
R. Prolongo QT a expensas del segmento ST

63. Tratamiento de urgencia de hipercalcemia:


R. SSN 0.9%

64. Causa de hiperfosfatemia:


R. Insuficiencia renal
65. Adaptación cerebral, hipertonicidad, excepto:
R. Pérdida de iones

66. En la adaptación cerebral, la hipotensión da lo siguiente, excepto:


R. Ganancia de aminoácidos
67. En ausencia de síntomas de hipernatremia, el sodio debe ser disminuido a razón de:
R. 0.5 mEq/L/h

68. Para corregir un sodio cerca de 185 mEq se administró en 86 horas, 66 letras de D/SS
5%. ¿cuál es la velocidad de administración de la venoclisis?
R. 69 mL ¿?????

69. Manifestación clínica de hipercalcemia, excepto:


R. a) Hipotensión arterial
b) Bradicardia
c) Nauseas
d) Anorexia

70. Consecuencia de hipofosfatemia, excepto:


R. a) No afecta al SNC
b) Disfunción plaquetaria
c) Cardiomiopatía
d) Osteomalacia

71. Causa de hipofosfatemia, excepto:


R. a) Falla renal
b) Recuperación de cetoacidosis diabética
c) Alcalosis respiratoria
d) Quemaduras graves

72. Causa de HTA con hipocalemia, aldosterona-renina baja:


R. Síndrome de Cushing

73. Causas de HTA con hipocalemia y renina alta:


R. Hipertensión maligna

74. En una deshidratación severa hipertónica con hipotensión arterial severa, la prioridad
inicial es administrar:
R. SSN 0.9%

75. En la secreción inapropiada de ADH se da lo siguiente, excepto:


R. No hay expansión de volumen

76. Tejido con mayor contenido de K+:


R. a) Hígado
b) Músculo
c) Eritrocito
d) Hueso

11.habia una de renal de un man q estaba en UCI con fractura de base de craneo:
Diabetes insidia central

14) Paciente con Oliguria,……muchos síntomas :

a) Insuficiencia suprarrenal ( creo que era la respuesta)

15) En IRA se mide todo menos:

a) Uosm/Posm x100
b) Na excretado/ Na filtrado x200
c) Na filtrado/ Na excretado x100
d) Na urinario >40
e) Ninguna de las anteriores

16) Paciente con hipertensión , hipokalemia, cretinina normal tiene:

a) Hipertensión arterial maligna


b) Cushing
c) Algo relacionado a licorice

31) En hiponatremia el daño principal es:

a) Neurológica
b) Respiratoria
c) Renal

32) Adaptación del cerebro ante solución hipotónica:

a) Pérdida de iones
b) Pérdida de aminoácidos
c) Edema cerebral
d) Ganancia de iones

62) La PTH promueve:

a) Reabsorción tubular de calcio y excreción de fosfato

9.Hiponatremia hipervolemica:
R/.Falla cardiaca
3. En un paciente con Insuficiencia Renal moderada, antecedentes de convulsiones y una
infección nosocomial por gérmenes multiresistentes ¿Cuál de los siguientes antibióticos
usted evitaria para disminuir el riesgo teorico de convulsiones?
a. Piperacilina con Tazobactam
b. Ertapenem
c. Imipenem
d. Meropenem
e. Gentamicina

25. Cual de las siguientes opciones esta contraindicada en el tratamiento de la


hiperkalemia
f. gluconato de calcio
g. aldactone
h. kayaxelate
i. Bicarbonato de sodio
j. Todas las anteriores

La accion toxica mas importante del ganciclovir es:


f. Nauseas y vomitos
g. Depresion de la medula osea
h. Diarrea
i. Insuficiencia Renal
j. Insuficiencia Respiratoria

49. Rotando en sala de medicina interna le piden que evalue un paciente con HIV el
cual por presentar una meningitis por criptococci recibe anfotericina B; debido a
este medicamento buscaria como complicación:
f. aparicion de diarrea
g. aumento de la presion arterial
h. hipokalemia
i. cefalea
j. dolor toráxico

50. Paciente de 84 anos quien recibe diuretico tiazidico para la hipertensión arterial
es admitido por presentar diarrea y trastornos del sensorio. Tiene disminución del
turgor de la piel y la presion arterial es normal, Pna 174mEq/L, Una 5mEq/L,
Uosm 606mEq/kg. La hipertensión se debe a:
f. perdidas insensibles
g. polidipsia
h. uso de diuretico
i. diabetes insipida
j. secrecion inapropiada de hormona antidiuretica

51. Hipernatremia Hipervolemica


f. diuresis osmótica
g. diarrea
h. aldosteronismo primario
i. diabetes insipida
j. SIADH

52. Causa de hiperkalemia:


f. Aumento del volumen arterial eficaz
g. Hipercatabolismo tisular
h. Hiperaldosteronismo
i. Alcalosis metabolica
j. Cirrosis hepatica

53. Causa de hipomagnesemia


f. cirrosis avanzada
g. falla renal cronica
h. falla cardiaca
i. constipacion
j. acidosis respiratoria

54. Causa renal de hipomagnesemia


f. falla renal cronica
g. esteatorrea
h. desnutricion severa
i. uso de diureticos de asa
j. infeccion urinaria

55. en caso de hipercalcemia pensar en:


f. neoplasia
g. hiperfosfatemia severa
h. nefrolitiasis
i. nefrocalcinosis
j. diureticos

56. En el síndrome nefrosico puede encontrarse cilindros:


f. cereos
g. hematicos
h. leucocitarios
i. epiteliales
j. mixtos leucocitario y hematicos

57. El ciclo de la urea tiene como finalidad fundamental:


a. sintetizar protrombina
b. transformar el amoniaco
c. producir sales biliares
d. sintetizar colesterol
e. producir acido folico
58. Donde se halla el calcio del organismo
a. el 50% en la sangre
b. el 20% en las proteinas
c. el 99% en el hueso
d. el 50% en el hueso
e. el 50% en el hueso y el 50% en la sangre

7 La PTH promueve todo lo siguiente excepto:


Aumento de la absorción tubular de fosfato

8 Varón de 57a esta con hemodiálisis de mantenimiento para insuficiencia renal


crónica. Cual de las siguientes anormalidades metabólicas podría ser anticipada?
Osteomalacia

12 Indica falla renal aguda:


UN = 1

13 Síndrome hepatorenal es causa de falla:


Prerenal

14 Síndromes Hipercoagulabes, excepto:


Sepsis

17 Cristal de birrefringencia negativa en forma de aguja a la luz polarizada:


Pirofosfato de calcio
Cristales de urato/ Monourato Sodico

35 Hombre con fractura de cráneo, osm 800:


Diabetes insípida central

40 Marcados de LES:
Anti-DNA y anti-smith

42 ml de agua que se da:


o 3 litros

50 S Nefrosico:
Cilindros cereos

51 Falla renal crónica avanzada:


Brecha aniónica elevada

• La PTH promueve todo lo siguiente excepto:


o EL aumento de la reabsorción tubular de fosfato.
La PTH a nivel renal es eminentemente fosfatúrica, promueve la excreción de fosfato y la
reabsorción de Calcio.
• Síndrome nefrósico de cambios mínimos se caracteriza por:
o Fusión de los pedicelos de los podocitos.
La fusión de los pedicelos no se puede ver en microscopia de luz, hay que usar
microscopia electrónica, pero se puede hacer el dx con la proteinuria masiva
característica del síndrome.
• En un paciente con eclampsia se le ordeno un litro de solución de dextrosa al 5%,
conteniendo 20 ml de MgSO4 al 50% (PM= 100 g), ¿cuantos ml de sal se necesita
para preparar la solución?
o 10 ml.
• El porcentaje de solución es:
o 1%
• ¿Cuantos mEq de Mg tiene la venoclisis?
o 83.3
• La hipocalcemia de la falla renal crónica puede cursar:
o Asintomática.
• La velocidad de administración de KCL en un caso de alto riesgo puede ser:
o 40 mEq/L
En los pacientes con una hipokalemia muy severa se usa 40, pero se debe pasar a una
velocidad de 20, porque si se pasa muy rápido puede ocasionar un bloqueo AV completo.
• Paciente urémico, con K sérico de 6, Academia, EKG normal se trata con:
o Kayexalate
Ya que la hiperkalemia no es tan severa se puede tratar con el Kayexalate, que es una
resina de intercambio catiónico, la misma remueve el potasio del cuerpo.
• Paciente femenina de 50 Kg presenta diarrea, presión arterial un poco baja, sin signos
neurológicos, Na=120, K=2, Cl=100, HCO3=10, asuma 60% de agua, cuantos
gramos de Sal para elevar el Na a 130
o 17.5
Calculo: (60*0.5)*10=300 se necesita dos soluciones salinas (154*2=308), cada solución
salina tiene 58.5 g NaCl y cada gramo de solución tiene 17.5 g de Na.
• ¿Qué solución salina usted escogería?
o 0.9 %
• La hipofosfatemia produce:
o Fosfaturia
La disminución del calcio, produce un aumento de la PTH y esta a su vez un aumento de
la eliminación del fósforo por la orina.
• La hipokalemia puede predisponer, excepto:
o Disminución de la amoniogenesis.
La hipokalemia por lo general cursa con acidosis metabólica, en donde va a estar
aumentada la formación de amonio.
• Un inhibidor de la anhidrasa carbónica es causa de:
o Hipokalemia
Ellos producen un aumento de la excreción de bicarbonato al ser este un anión, arrastra el
potasio que es un catión.
• Un hombre llega al hospital comatoso, con fractura de cráneo, orina 175, suero de
Na=178, K=4, Cl=130, HCO3=25 Osm=350 en la orina 800 (no c ve bien).
o Diabetes insípida central.
Recordar que una de las causas de hipernatremia es la diabetes insípida.
• Confirmaría su dx así:
o Prueba de restricción y administración de vasopresina.
• Causa de HTA con hipokalemia excepto:
o Síndrome de Bartter.
El síndrome de Bartter no cursa con hipertensión arterial (HTA maligna,
hiperaldosteronismo primario y síndrome de hiperprostaglandismo entre otros).
• Se relaciona con hipofosfatemia, excepto:
o Disminución de la afinidad de la hemoglobina por el O2.
La hipofosfatemia produce una disminución del 2,3-DPG, y hay una desviación a la
izquierda, lo que produce una aumento de la afinidad por el oxigeno.
• Paciente con vómitos severos, hipotensión arterial, la orina al azar muestra Na mas de
40, osmolaridad de 800, puede tener:
o Falla suprarrenal.
El paciente al tener una hipovolemia, se activa el SRAA, y aumenta la reabsorción de Na
y disminuye su cantidad en orina.
• Un hombre de 30 Kg padece de Ca pulmonar, tiene 2 semanas de estar letargico y
obnubilado, suero de Na 105, Cl 72, K 4, HCO3 21, Osm 122 y en la orina Na 78,
Osm 604, el dx mas probable:
o SIADH.
Hay una hiponatremia, hipotónica y euvolémica.
• Tratamiento:
o Solución salina al 3%
El tratamiento de SIADH, es restricción de agua y suplir el déficit de sodio.
• Causas de enfermedad tubular intersticial crónica excepto:
o Nefritis alérgica.
En su mayoría medicamentos y medio de contraste.
• La administración de solución salina isotónica produce:
o Aumento del volumen extracelular.
• Sospecha de obstrucción de las vías urinarias en caso de:
o Anuria.
• En la insuficiencia renal aguda siempre da, excepto:
o Retensión nitrogenada
Es un parámetro a tomar en cuenta para dializar al paciente cuando es mayor de 100.
• Siempre habrá insuficiencia renal en caso de:
o Glomérulo nefritis rápidamente progresiva
La proliferación extracapilar comprime el glomérulo comprimiéndolo y dañándolo
severamente.
• En caso de glomérulo nefritis post estreptocócica la filtración glomerular puede
disminuir por:
o Alteración del coeficiente de ultrafiltración.
• En la nefritis túbulo intersticial puede dar lo siguiente, excepto:
o Proteinuria de 4 gr/día
Ese dato es de síndrome nefrósico.
• Es la causa de falla prerrenal.
o Pancreatitis aguda.
• En la insuficiencia renal aguda siempre habrá:
o Aumento de la creatinina.
• Causa de hipmagnesemia:
o Alcoholismo.
Otra causa importante son las pérdidas gastrointestinales.
• La hipofosfatemia causa, excepto:
o Aumento de PTH.
• Paciente con diuresis de 250 ml al día con Osm plasmática y osmolaridad urinaria de
300, el Dx probable:
o FRA, de causa renal.
• Paciente con oliguria, hiperkalemia, hiponatremia, creat 0.8 mg/dl y cpk normal, esto
puede corresponer a:
o Falla renal aguda, de causa prerrenal.
• En falla renal crónica:
o Hipofosfatemia.
• Urémico crónico con K 6.5 con leve academia, sin trastorno muscular y EKG normal,
tratamiento:
o Kayexalate.
• Urémico agudo, acidótico, oligúrico, deshidratado, todo es cierto, excepto:
o Densidad urinaria 1010.
Esto indica isostenuria, el riñón no tiene la capacidad de concentrar y es indicativo de
riñones muy maltratados.
• Falla renal aguda, oliguria, todo es cierto, excepto:
o Osmolaridad urinaria elevada.
• Síndrome nefrósico de cambios mínimos se caracteriza:
o Fusión de los pedicelos de los podocitos.
• En el shock hipovolémico, el uso de dextrosa en agua puede causar:
o Hiponatremia sintomática.
Primero siempre corregir la volemia con SSN 0.9.
• En la deshidratación por perdida fundamental de solutos, se da la siguiente, excepto:
o Entrada de electrolitos al cerebro.

• En el SIADH se da lo siguiente excepto:


o Edema de los miembros inferiores.
• Paciente con gastroenteritis, hipovolémico, cuyo suero contiene Na 155, K 2, Cl 117:
o SSN 0.45 + 40 mEq KCl.
• Una medida general en el tx de un hipercalcémico:
o SSN 0.9%
Para expandir el volumen.
• La velocidad de adminis tracion de K en caso de riesgo para la vida puede ser:
o 40
• Un ejemplo de hipernatremia hipervolemica es:
o Hiperaldosteronismo primario.
• Una dieta de 3gr de NaCl corresponde a:
o 17 gr de Sodio.
• La IRC ocaciona hiperfosfatemia permanente cuando la TFG esta por debajo de:
o 25 ml/min
El valor es por debajo de 30.
• Paciente con alcalosis metabólica e hipokalemia severa debe ser tratado con:
o KCl
• No esta indicada en la hipercalcemia:
o Aldactone
Es un diurético ahorrador de K, ósea produce hipercalcemia, se utiliza en el
hiperaldosteronismo primaria y edema.
• Metabolismo del calcio están implicados menos:
o Insulina.
• Hiponatremia hipotónica normovolémica:
o SIADH.
• Causa de enuresis
o Necrosis cortical difusa.
• La situación clínica de hiperfosfatemia con hipocalcemia con cirrosis hepática es:
o Cirrosis alcohólica.
• Factor que altera el balance hídrico en el periodo post operatorio excepto:
o Disminución de ADH.
• Ejemplo de hipernatremia hipervolemica:
o Hiperaldosteronismo primario.
• Ejemplo de hipokalemia con aldosterona alta y osm baja:
o Hiperaldosteronismo primario.
También con renina baja.
• Hiperkalemia sin alteración en el EKG no es necesario administrar:
o Gluconato de calcio.
• Urémico crónico con deshidratación hipertónica severa y acidótico presentara lo
siguiente excepto:
o Osmolaridad urinaria de 800.
• La fracción de excreción de Na esta aumentada en:
o Falla cardiaca.
También esta aumentada en FRC, de causas renales.
• Paciente con vomito abundante, HTA, Una 6, oligúrico y osmolaridad de 800,
presentara:
o Insuficiencia suprarrenal.
• En cirrótico con ascitis se recomienda, excepto:
o Dieta con 8.5 mEq de Na.
60 to 90 mEq per day, equivalent to approximately 1500 to 2000 mg of salt per day
(NEJM).
• Factor que predispone a la hipernatremia son, excepto:
o Intoxicación hídrica.
• En la hiperkalemia:
o Hay menos diferencia entre el potencial de reposo y el umbral.
• Los indicios de falla renal aguda son:
Pueden ser cualquiera de las siguientes:
o Alteraciones en la osmolaridad urinaria.
o Alteraciones sen la relación Uosm/Posm.
o Alteraciones en el Una.
o Alteraciones en el FENA.
• Se presenta en acidosis tubular renal proximal, excepto:
o NaHCO3 y los alpha agonista.
• Na urinario menor de 10:
o Falla cardiaca.

Urémico crónico con potasio sérico de 6.5mEq/L con leve academia, sin trastorno
muscular y EKG normal, puede ser tratado con:
e. kayaxelate
f. gluconato de Calcio al 10%
g. NaHCO3
h. Diálisis

La insuficiencia renal crónica ocasiona hiperfosfatemia permanentemente cuando la tasa


de filtración glomerular esta por debajo de:
e. 60ml/min
f. 50 ml/min
g. 25 ml/min
h. 40 ml/min

La hipofosfatemia es causa de excepto


e. inotropismo positivo
f. disminución de 2,3 DPG
g. hemólisis
h. curva de HbO2 desplazada hacia la izquierda

La solución de NaCl 23,4% tiene de osmolaridad:


e. 8000 mosm/l
f. 800 mosm/l
g. 80 mosm/l
h. 0,8 mosm/l

En la hiperkalemia:
e. aumenta la sístole eléctrica
f. hay menor diferencia entre el potencial de reposo y el umbral
g. el potencial de reposo transmembrana se hace mas negativo
h. el potencial umbral se hace menos negativo

La velocidad de administración de potasio en caso de gran riesgo para la vida puede ser
de:
e. 10-20 mEq/L
f. 8 mEq/L
g. 110 mEq/L
h. 40 mEq/L

Un ejemplo de hiponatremia hipotónica normovolemica es:


e. síndrome nefrósico
f. SIADH
g. Diarrea
h. Cirrosis

A una embarazada con eclampsia se le ordena 1 litro de D/A 5% con 10g de MgSO4.
¿Cuántos ml de MgSO4 al 50% se han utilizado?
a. 20
b. 5
c. 12,5
d. 30

Causa hipomagnesemia:
e. estreñimiento
f. alcoholismo
g. deshidratación hipertónica
h. insuficiencia renal aguda

Paciente con diuresis de 300ml/día, K sérico 7mEq/L, Posm y Uosm 300mosm/L, el


diagnóstico más probable es:
e. insuficiencia suprarrenal
f. insuficiencia renal
g. SIADH
h. Deshidratación primaria

La mielinosis pontina se caracteriza por:


e. pares craneales intactos
f. cuadriplejía flácida
g. comportamiento normal
h. hipertonía de los miembros inferiores

Calcule el déficit de agua en una paciente con sodio sérico 168mEq/L y que pesa 70Kg
(utilice como porcentaje de agua 50% y sodio normal mEq/L):
e. 5L
f. 3L
g. 10L
h. 7L

En la SIADH la concentración de sodio urinario es de:


e. menos de 10mEq/L
f. 20mEq/L
g. 15mEq/L
h. Mas de 40mEq/L

En paciente con vómitos abundantes y prolongados con hipokalemia severa debe


administrarse:
e. NaCl 0.9% con KHCO3
f. NaCl 0.9% con KCl
g. NaCl 0.9% con gluconato de potasio
h. NaCl 0.9% con NaHCO3

La hipokalemia en un paciente con cirrosis puede contribuir:


e. coma
f. alcalosis respiratoria
g. acidosis metabolica
h. aumento de la síntesis de urea

El plasma de un paciente contiene Na 125mEq/L, glucosa 108mg/dl, urea 300mg/dl,


podrá presentar:
e. cirrosis
f. síndrome nefrósico
g. síntomas de hipertonicidad
h. síntomas de hipotonicidad

La hiponatremia aguda sintomática es debida a, excepto:


e. exceso de glucocorticoides
f. SIADH
g. post-operatorio
h. intoxicación hídrica

La administración de solución salina 0.9% produce:


e. disminución del volumen intracelular y aumento del extracelular
f. aumento del volumen intra y extracelular
g. aumento del volumen extracelular
h. aumento del volumen intracelular

Alcalosis metabólica con hipertensión arterial, hipokalemia, aldosterona normal y renina


normal:
e. Hipertensión renovascular
f. Hipertensión maligna
g. Sindroma de Cushing
h. Administración de licor

La hipokalemia puede predisponer, excepto:


e. intoxicación digitálica
f. disminución de la amoniogénesis
g. rabdomiolisis
h. ondas U en el EKG

77. Cuándo disminuye el fosfato?


R. Aumento de PTH

78. Ejemplo de hipernatremia hipervolémica:


R. Aldosteronismo primario

79. La hipocalemia produce:


R. Aumento de la magnitud del potencial de reposo

80. La depleción de agua produce, excepto:


R. Salida de electrolitos del cerebro

81. Velocidad de administración de KCl en caso de urgencia:


R. 40 mEq/h

82. Cambios en EKG de la hipermagnesemia es igual:


R. Hipercalemia

83. Enfermo de 60 Kg con 60% de agua corporal, osmolaridad plasmática 260


mosmol/Kg, ¿cuántos gramos de NaCl aumenta el Na en 10 mEq en 24 horas?
R. a) 15
b) 21
c) 30
d) 19

84. Uso de Calcio en tratamiento de hipercalemia para:


R. Modificar el Potencial de Membrana

85. Hipermagnesemia severa produce:


R. Paro respiratorio

86. Venoclisis de 1L de D/A 5% conteniendo 10 mg de MgSO4 y al 50%:


R. 20mL

87. La hipocalcemia:
R. Prolongo QT a expensas del segmento ST

88. Tratamiento de urgencia de hipercalcemia:


R. SSN 0.9%

89. Causa de hiperfosfatemia:


R. Insuficiencia renal
90. Adaptación cerebral, hipertonicidad, excepto:
R. Pérdida de iones

91. En la adaptación cerebral, la hipotensión da lo siguiente, excepto:


R. Ganancia de aminoácidos
92. En ausencia de síntomas de hipernatremia, el sodio debe ser disminuido a razón de:
R. 0.5 mEq/L/h

93. Para corregir un sodio cerca de 185 mEq se administró en 86 horas, 66 letras de D/SS
5%. ¿cuál es la velocidad de administración de la venoclisis?
R. 69 mL ¿?????

94. Manifestación clínica de hipercalcemia, excepto:


R. a) Hipotensión arterial
b) Bradicardia
c) Nauseas
d) Anorexia

95. Consecuencia de hipofosfatemia, excepto:


R. a) No afecta al SNC
b) Disfunción plaquetaria
c) Cardiomiopatía
d) Osteomalacia

96. Causa de hipofosfatemia, excepto:


R. a) Falla renal
b) Recuperación de cetoacidosis diabética
c) Alcalosis respiratoria
d) Quemaduras graves

97. Causa de HTA con hipocalemia, aldosterona-renina baja:


R. Síndrome de Cushing

98. Causas de HTA con hipocalemia y renina alta:


R. Hipertensión maligna

99. En una deshidratación severa hipertónica con hipotensión arterial severa, la prioridad
inicial es administrar:
R. SSN 0.9%

100. En la secreción inapropiada de ADH se da lo siguiente, excepto:


R. No hay expansión de volumen

101. Tejido con mayor contenido de K+:


R. a) Hígado
b) Músculo
c) Eritrocito
d) Hueso

1. Plasma: creatinina 1mg/dl,140 mEq/L. Orina: creatinina 150 mg/dl, Na 40 mEq/L


la FE Na es:
b) 0.19%

2. Factor determinante en tratamiento de hipokalemia real es:


a) función cardiovascular

3. Tratamiento de inicio de SIADH es:


a) restricción hidrica

4. Paciente con alcalosis metabolica e hipokalemia debe ser tratada con:


a) KCl

5. Causa de hiperkalemia
a) aumento de volumen arterial eficaz
b) hipercatabolismo tisular
c) hiperaldosteronismo
d) alcalosis metabolica
e) cirrosis hepatica

6. Causa de hipomagnesemia
a) cirrosis avanzada
b) falla renal cronica
c) falla cardiaca
d) constipación
e) acidosis respiratoria

7. Causa renal de hipomagnesemia


a) falla renal cronica
b) esteatorrea
c) desnutrición severa
d) uso de diureticos
e) infeccion urinaria

8. En caso de hipercalcemia pensar en:


a) neoplasia
b) hiperfosfatemia severa
c) nefrolitiasis
d) nefrocalcinosis
e) diureticos

9. El síndrome nefrosico puede encontrarse cilindros:


a) cereos
b) hematicos
c) leucocitarios
d) epiteliales
e) mixtos

24. U Na 115, Uosm 240. P osm 680, P Na 62


SIADH

25. Paciente de 84 años que recibe diureticos para la hipertensión arterial. Presenta
diarrea y trastornos del sensorio y disminución del rubor de la piel. Na 174 U
Na 5 U osm 606. La hipernatremia se debe a:
Perdidas insensibles

26. hipernatremia hipervolemica


Aldosteronismo primario

27. causa de hipomagnesemia


Alcoholismo

28. hipocalcemia:
Provoca potencializacion de efectos de hiperkalemia

29. En hipocalcemia
Prolonga QT a expensas de segmento ST

30. Tratamiento de urgencia hipercalcemica


Solucion salina isotonica 0.9% 4 a 5 L

31. Síndrome nefrotico


Edema, proteinuria, hipertensión arterial, hematuria

17. La PTH promueve todo lo siguiente excepto:


a aumento de la absorción intestinal de calcio
b- aumento de la absorción tubulár de fosfato..
c. aumento de la resorción ósea de fosfato
d- aumento de la resorción osea de calcio
e, disminución de la reabsorción de fosfato

18. Un varon de 57 esta con hemodiálisis de mantenimiento para insuficiencia


renal cronica, ¿Cuál de las sig anomalías metabolicas podría ser anticipáda?
a. hipernatremia
b. hiponatremia
c.
d. exceso de vitamina D
e. hipoparatiroidismo
19. Un px de 25años fue ingresado a UCI por lesiones graves en cabeza y fractura
de la base del Graneo, aprox X horas después de la lesion manifestaba poliuria, la
OSMU 150, y la de suero 350. Los líquidos IV fueron detenidos y 3 hors después
la producción de orina y la osm urinaria permanecieron sin cambio. Se
administraron 5 unidades de vasopresina IV, la osm de la orina se
incremento a 300. ¿Cuál es el dx mas probable?
a- Diabetes insípida central
b. diabetes insípida nefrogenica
c. intoxicación acuosa
d. sobrecarga de solutos
e. S IADH

55. indica falla renal aguda


a- U..N.. = 4.1
b- UN = 1
c. U..N.. = 1/4i/
d. Gravedad especifica 1.026
e. Ninguna

56. deshidratación por perdida de agua y densidad urinaria 1.003, creatinina 6 /


a. falla renal aguda parenquimatosa
b- falla prerenal
c. falla renal obstructiva
d. falla renal cronica
e. no hay falla renal

57. uno de los indices de falla renales


a. reabsorción de Na/oferta tubular de Na x 100
b. excresion urinaria deNa/oferta tubular de x 100
c.- oferta tubular de Na/excrecion urinaria de Na x 100 ~.--~
d- Una/Pna x 100
e. Ninguna

58. el Sdr hepatorenal es causa de falla:


a- prerrenal
b. renal
c. póstrenal
d. falla cardiaca
e. daño tubular

59. causa mas común de falla renal crónica:


A- DM

60- en la glomendonefritis aguda por infeccion, la causa, excepto


a- proteina mayor de 10 g
b- osmolaridad urinaria normal
c. hematuria
d. HT A
e. Edema

61. falla renal cronica, excepto


a. PTH aumentado
b- Hipofosfatemia
c- Acidemia
d. Hipocalcemia
e. 1.25 OH (D3) disminuida

62. EN la falla renal aguda siempre habra


a- Aumento de creatinina
b- fraccion de excrecion de Na aumentado
c- hiperkalemia
d- Una aumentado
e- Hipernatremia

63. causa de falla prerenal


a. Sdr nefrotico
b- obstrucción de ambos uréteres
c- falla cardiaca
d. Grlomerulonefritis aguda post-infeciosa
e. Calculo renal

64. en la nefritis tubular interticial se puede dar, excepto


a. h.ipernatremia
b. proteinuria de 1.5 g
c. normotension arterial
d. hipostenuria
e. hipocalcemia

65. Sugiere glomerulonefritis:


a. leucocituia
b-cilindros eritrocitarios
c- anuria
d. -

66 -causa de falla renal parenquimatosa


e) papilitis
f) calculo en pelvis derecha
g) estenosis unilateral de arteria renal
h) cistitis hemorragica

60- Causa de hipercalemia:


a Aumento del volumen arterial eficaz
b. Hipercatabolismo tisular
c Hiperaldosteronismo
d. Alcalosis Inetabolica
e. Cirrosis hepatica

61. Causa de hipomagnesemia


a. cirrosis avanzada
b. falla renal cronica
c. falla cardiaca
d. constipacion
e. acidosis respiratoria

62. Causa renal, de hipomagnesemia


a. falla renal crônica
b. esteatorreaa
c desnutricion severa
d. usó de diureticos de asa
e. infeccion urinaria

63. en caso de hipercalcemia pensar en:


a. neoplasia
b: hiperfosfatemia severa
c. nefrolitiasis
d. nefrocalcinosis
e. diuréticos

64. En el síndrome nefrosico puede encontrarse cilindros


a. cereos
b. hematicos
c. leucocitarios
d. epiteliales
e. mixtos leucocitario y hematicos,

2--En el riñón la hormona paratiroides:


Aumenta la reabsorción tubular de calcio y disminuye la de fósforo

3- La hipercalcemia produce
Potencialización de los efectos de la hipercalemia

36- La hipomagnesemia produce.


Vasodilatación periférica

36-De los factores: nefrotoxicidad./ -La buena hidratación

37 - En la concentración sérica para calcular:: 8,8


38- la velocidad de administración de potasio: 40 meq

39-hipocalcemia produce : fosfaturia

40-La causa de hipertensión con hipocalemia: Síndrome de Bartter=:,

41-En paciente con vomito severo hipotensión arterial o ir a la sar con mas de 40
meq
Insuficiencia suprarrenal;

42-La ,administración de sol isotónica aumento de vol extracelulares

43-Glomerulonefritis post estreptococcica Altera el cociente de ultrafiltración

44-En-el túbulo` intersticial se puede dar: La proteinuria de 4 g al día.

16: Es una señal de haber sufrido de pancreatitis: las opciones eran


hiperamilasemia, aumento de dolor, hipercalemia, aumento de tamaño.

18- La hipofosfatemia cansa excepto: aumento de vitamina D


20. La renal aguda, oligurica todo es cierto excepto: Osmolaridad urinaria
aumentada
21 Paciente can alcalosis metabolica e hipokalemia debe ser tratada con KCL

24. Usted tiene una paciente con hiperhemesis gravidica. El trastorno metabolico
que espera encontrar es:
A - acidosis hipocloremica
b, acidosis hipercloremica'
c- alcalosis hipocloremica
d. alcalosis hipercloremica
e. no hay alteración acido base

25. Cual de las siguientes opciones esta contraindicada en el tratamiento de la


hiperkalcmia
a, gluconato de calcio
b. aldactone
c, kayaxelate
i. Bicarbonato de sodio e. Todas las anteriores

43. Rotando en sala de medicina interna le piden que evalue un paciente con HIV
el cual porpresentar una meningitis por criptococci recibe anfotericina B; dicho
este medicamento buscaría como complicación:
a. aparicion de diarrea
b. aumento de la presion arterial
c- hipokalemia
d . cefalea
e. dolor toráxico

81- donde se halla el calcio en el organismo:


a- 50% sangre
b- 20 % proteínas
c- 99% hueso

4- en la hipercalemia
a- hay una menor diferencia entre el potencial de reposo y el umbral

5—la hipercalemia en un cirrótico puede desencadenar


a- coma hepatico

6- la hipermagnesemia puede producir


a- paro respiratorio

7- tx de hipercalcemia
a- SSN 0.9 %

8- la hipofosfatemia son causas excepto:


a- aumento de PTH
b- aumento de la absorción intestinal de fosfato
c- disminuye fosfatemia
d- aumento de vitamina D

9- una solucion isotonica


a- aumenta el volumen extracell

10- la vel de administración de potasio en caso de riesgo para la vida


a—40 meq por hora

11- Px cronico con deshidratación severa puede presentar los siguiente


excepto:
a-osmolaridad urinaria de 800 mosmol/Kg

12- Px con vomito abundante, HTA, sodio urinario en 6, oligurico,


Osmolaridad de 800 presente,
a- insuficiencia suprarrenal

13- Px uremico cronico con vomito severo hipotension arterial orina al


azar, muestra sodio mas de 40, Osmolaridad de 800 puede tener :
a- Falla suprarrenal

15- La hipocalemia predispone excepto:


a- amoniogenesis
16- Cual de las siguientes es causa de hiperfosfatemia
a- insuficiencia renal

17- en la glomerulonefritis aguda post infecciosa disminuye la tasa de


filtración:
a- por alteración en el coeficiente de ultrafiltracion

18- causa de insuficiencia prerenal excpto:.


a- Aneurisma disecante de la aorta

19- el magnesio esta aumentado


a- eritrocito

65. Causa de hiperfosfatemia:


m. Falla hepatica
n. Falla renal cronica
o. Falla cardiaca
p. Insuficiencia suprarrenal

66. Causas de hiperfosfatemia grave, excepto:


j. recuperacion de cetoacidosis diabetica.
k. Alcalosis respiratoria
l. Acidosis respiratoria.

67. Paciente con vómitos severos, oligurico con hipotensión arterial Una> 40……
800 mOsm/Kg puede tener:
m. Insuciencia renal aguda.
n. Insuficiencia suprarrenal
o. Falla pre-renal
p. Exceso de mineralocorticoides

68. Paciente urémico crónico con deshidratación severa puede presentar lo siguiente
excepto:
a. Densidad urinaria 1010.
b. Osmolaridad urinaria de 800 mOsm/Kg
c. K sérico aumentado
d. Oliguria.

69. La hipokalemia produce:


a. Aumento de la magnitud del potencial de reposo
70. Primera*****Falla renal crónica:
a.HTA
b.Glomerulopatias
c.Diabetes Mellitus
d.Nefritis tubulo intersticial
71. Uno de los indices de falla renal aguda es:
Uosm/Tosm

72. Hormona producida en los riñones que disminuye en la falla renal cronica:
a. ADH
b. Noradrenalina
c. Peptido natriuretico (segun arjona)
d. 1,25 (OH)2 Vit D.

73. Las calcificaciones metastásicas se deben:??


a. Aumento del producto Ca, P
b. Aumento del producto Ca, Mg
c. Hipoparatiroidismo
d. Hiperparatiroidismo 1ario

74. Causa de anuria:


a. Pielonefritis unilateral
b. Necrosis tubular aguda
c. Enf cambios minimo
d. Necrosis cortical

75. la etapa de la insuficiencia renal cronica con tasa de filtración glomerular de 60-
89 ml/min es la:??
a. 1
b. 2
c. 3
d. 4

76. La FENA es la relación entre:


a. Oferta tubular/Na reabsorbido x 100
b. depuración de Na/ depuración de creatinina x 100
c. oferta tubular/ Na excretado x 100.
d. Na excretado/ Na reabsorvido x 100.

77. En la nefritis tubular intersticial aguda se pierde la capacidad de poder concentrar


la orina debida a:??
a. Daño medular
b. Daño glomerular
c. Daño tubular proximal
d. Daño de la porcion cortical del tubulo colector

78. La solucion de NaCl al 23,4 % tiene:


8000 mOsm/l

79. Se asocia con el SIAD, excepto:


lipoma
80. El efecto biologico mas importante de la calcitonina es, excepto:
a. Responder a la hipocalcemia
b. Aumenta la calciuria
c. Estimular la resorcion osea osteoclastica
d. Utilizada en el tx de la hipercalcemia.

81. Favorece la entrada de K al espacio intracelular, excepto:


a. Agonista adrenergico
b. Estimulacion adrenergico
c. Insulina
d. Alcalosis metabolica

82. Factores que afectan el balance hídrico en el post-operatorio, excepto:


a. Fármacos
b. Calcitriol
c. Administración de liquidos hipotonico
d. Aumento de la ADH

83. Las primeras manifestaciones clínicas de la hiponatremia son:??


a. neurológica
b. gastrointestinal
c. pulmonares
d. musculares

84. En la hipernatremia hipertonica se da lo siguiente excepto:


a. Entrada de Na
b. Salida de aa
c. Ganancia de iones
d. Disminución del agua cerebral

85. Manifestaciones clinicas de la hipercalcemia, excepto:


a. Taquicardia
b. HTA
c. Constipación
d. Vomitos

86. En falla renal aguda por nefrotoxicidad:


a. Se conserva la membrana basal
b. Se destruye la membrana basal
c. Frecuente hay oliguria
d. La creatinina no aumenta

87. Causa de hipomagnesemia:


alcoholismo
88. Na serico de 168 mEq/l 70 kg (utilize como % de agua al 50% y Na normal 140
mEq/l
7 litros

89. Indicaciones de diálisis en falla renal aguda, excepto:


a. Encefalopatia uremica
b. Hiperkalemia refractaria
c. Ac. Metabolica refractaria
d. hipocalcemia

90. El KCL IV se puede asministrar asi, excepto:


a.infusion por linea periferica de 10 meq en 100 ml en 1hora
b. “ “ central de 20 meq em 50ml en 1 hora
c. “ “ central de 100 meq em 50ml en 1 hora
d. “ “ periferica de 20 meq en 20ml en 2 horas

91. Aumenta la excrecion urinaria de K, excepto:


a. Triamtereno
b. Bumetanida
c. Furosemida
d. Metolazona

92. Factores que regulan la secrecion distal de K, excepto:


a. Flujo tubular distal y aporte distal de Na
b. Excrecion de aniones no reabsorbibles
c. Aldosterona
d. PTH

93. Enfermo de 60 Kg con 60% de agua corporal, Posm 260 mOsm/kg, ¿Cuántos
gramos de NaCl aumentaria el Na en 10 mEq?
a. 21 gramos

50. Conteniendo 10 g de MgSO4 al %, cuantos ml de sal necesitaria?


20 ml

La hipokalemia produce:
- mayor negatividad del potencial de reposo.

La hipokalemia en un cirrótico puede precipitar COMA por:


- aumento de la amniogénesis

Exceso del kayaxelate en el tratamiento de hiperkalemia puede producir:


- constipación.

Peligro de la hipermagnesemia severa:


- paro respiratorio.

Hipofosfatemia:
- aumenta el calcitriol.

Consecuencia metabólica de hipofosfatemia severa:


- miocardiopatía

Causa de hiperfosfatemia:
- insuficiencia renal

El Síndrome Nefrósicose caracteriza por:


- proteinuria mayor o igual de 3g y albúmina sérica menor o igual de 3g/dl.

En el Síndrome Nefrósico pueden encontrarse:


- cilindros céreos

Urémico agudo, oligúrico, acidótico, deshidratado puede presentar:


- osmolaridad urinaria de 250mosm/kg

La falla renal crónica avanzada cursa con:


- brecha aniónica elevada

Cuál de los siguientes antibióticos produce nefritis intersticial:


- meticilina

El factor más determinante en el tratamiento de la hipokalemia renal es:


- la función cardiovascular.

El tratamiento de inicio de secreción inapropiada de ADH es:


- restricción hídrica

23. Px con IRC tine una taza de filtración glomerular de menor de 25 ml/min ;aunque
hay yuna tabla de las etapas en esta enfemedad crónica.
Estas osn las TFG:
1. 90 en riesgo
2. 20 ( con TFG normal o incrementada)
3. 60 -89
4. 30 – 59
5. 15-29
Menor a 15 diálisis.

24. Factor que altera el balance hídrico en el periódo post- operatorio:


Disminución de la ADH (Excepto)

25. Un ejemplo de hipokalemia con aldosterona alta y osmolaridad baja es:


Hiperaldosteronismo primario

26. Ejemplode hipernatremia hipervolémica: Hiperaldosteronismo primario

27. Hiperkalemioa son alteraciones del EKG no es necesario administrar gluconato de


calcio se colocará un antagonista de K asíe l Px no cae en paro y estoes si tiene
manifestación en el EKG.

28. La fracción de excreción de sodio estará aumentada en falla cardíaca.

29. En la hiperkalemia: hay menos diferencia entre el potencial de reposo y el umbral.

Manifestaciones cardíacas. El problema médico más grave de la hiperkalemia es la


cardiotoxicidad. Los cambios en el ECG producidos por los niveles altos de potasio son
bastante constantes. A medida que aumentan los niveles se aprecian los siguientes
cambios:

Ondas T picudas (con intervalo QT normal o ligeramente reducido)


Prolongación del intervalo PR con depresión de ST
Desaparición progresiva de la onda P
Bloqueo cardíaco progresivo
Arritmias ventriculares
Paro cardíaco

Las ondas T picudas constituyen el dato en el ECG más constante en la hiperkalemia.

Efectos neuromusculares. El primer signo neuromuscular de la hiperkalemia suele ser la


aparición de parestesias seguidas de debilidad progresiva de varios grupos musculares. Si
el cuadro se agrava se observa cuadriplejia fláccida. Las funciones cerebrales y de los
pares craneanos se conservan y la parálisis de la musculatura respiratoria puede ocurrir,
pero es excepcional.

30. La infusión de potasio sin riesgo para la vida del PX: es de 40 meq; porque

I lb de solución salina: debe llevr de 20 a40 mEq de KCl a una velocidad de 10 a 20


mEq por hora; si aumenta la velocidad puede provocar un bloqueo A-V. Se recomienda
una Velocidad mayor si la hipokalemia es muy severa.

Sis e usa venas priféricas es:

10 meq en 100 cc de sol.salina para pasar en una hora

20 meq en 200 cc de sol.salina para pasar en una hora

En vena central:
20 meql en 50 cc pasarla en una hora

40 meq en 100cc pasar la en una hora

31.Los indicios de falla renal aguda son:

(Na filtrado/Na excretado) x 100%

32.Na urinario menor de 10: falla cardíaca, presentará edema estará hinchado y lo
puede llevar a una I.C.C. ( Esto lo saque de Medline: Los valores normales generalmente
son de 15 a 250 mEq/L/día, dependiendo del estado de deshidratación y la ingesta diaria
de sodio en la dieta. Los rangos de los valores normales pueden variar ligeramente entre
diferentes laboratorios. Nota: mEq/L/día = miliequivalentes por litro por día.)

33.Causa de enuresis: necrosis cortical difusa.

34.En SIDH la concentración urinaria es menor de 20.

35. Deshidratación pura de agua excepto: hay salida de electrolitos del cerebro.

36.Urémico crónico con deshidratación hipertónica severa y acidótico presentará lo


siguiento excepto: osmolaridad urinaria de 8mm mosmol/kg.

37. Px con vómitos abundante, hipertensión arterial; Na urinario=6, oliguria, osmolaridad


de 800 presenta: insuficiencia suprarrenal.

38. Factor que predispone a la hipernatremia son intoxicación hídrica.

39. Se presenta en acidosis tubular renal proximal excepto: NaHCO3 y lo alfa


antagonistas.

Aldosterona aumentada, Renina dism. → Hiperaldosteronismo I


Limites de Hipo e Hiperkalemia

96. Paciente de 68 años diabética, que consulta por malestar general. En la gasometría venosa
destaca pH 7.25, Bicarbonato 15 mmol/l (normal 24-28 mmol/l). Hiato anionico (anión GAP): 11
mmol/l (normal 10-12 mmol/l). ¿Cúal de las siguientes entidades NO descartaría como
diagnóstico?:
1. Cetoacidosis diabética.
2. Insuficiencia renal crónica.
3. Acidosis tubular renal.
4. Ingesta de salicilatos.
5. Acidosis láctica.
97. Cuando un paciente bajo tratamiento diurético con tiazidas o furosemida incumple la dieta y
come más sal de la prescrita, el resultado análitico esperable es:
1. Mayor hipernatremia.
2. Mayor hiponatremia.
3. Mayor hiperpotasemia.
4. Mayor hipopotasemia.
5. Mayor acidosis.
98. En la uremia pre-renal:
1. El sodio en orina es superior a 60 mEq/l, la
osmolaridad urinaria es superior a 500 mOsm/
kg H2O y la relación urea en orina/urea en
plasma es superior a 8.
2. El sodio en la orina es inferior a 20, la
osmolaridad urinaria es inferior a 200 mOsm/
kg H2O y la relación entre urea en orina /
urea es plama es inferior a 2.
3. El sodio en orina es inferior a 20 mEq/l, la
osmolaridad en orina es inferior a 200 mOsm/
Kg H2O y la relación urea en orina / urea
en plama es superior a 8.
4. El sodio en orina es inferior a 20 mEq/l, la
osmolaridad urinaria es superior a 500 mOsm/
Kg H2O, y la relación entre la urea en orina
y la urea en plasma es superior a 8.
5. El sodio en orina es superior a 60, la osmolaridad
urinaria es superior a 500 mOsm/Kg
H2O, y la relación urea en orina/urea en
plasma es superior a 8.
99. Un hombre, adicto a drogas por vía parenteral, está ingresado por endocarditis infecciosa.
Durante su enfermedad presenta un cuadro de glomerulonefritis aguda. ¿Cuál de las respuestas
es INCORRECTA?:
1. Suele ser debida a inmunocomplejos.
2. No suele presentar piuria.
3. El complemento está descendido.
4. A veces produce síndrome nefrótico.
5. Suele evolucionar favorablemente al controlar
la infección cardiaca.
100. Una mujer de 68 años acude al Servicio de Urgencias por malestar general que ha ido
progresando en los últimos 15 días, a partir de un episodio gripal. Ha notado disminución
progresiva del volumen de diuresis, edemas maleolares y dificultad respiratoria. Es hipertensa.
En la analítica destaca una creatinina plamática de 5 mg/dl, urea 180 mg/dl, Na 138 mEq/l, K
4.9 mEq/l. Las cifras de complemento son normales. Los anticuerpos anti-membrana basal son
negativos. En la orina presenta cilindros hemáticos, proteinuria de 1 g/l y microhematuria.
Aporta una analítica de un mes antes, sin alteraciones. ¿Cuál de los siguientes diagnósticos es
más probable?:
1. PAN microscópica.
2. Brote lúpico.
3. Glomerulonefritis aguda postinfecciosa.
4. Crioglubulinemia.
5. Enfermedad de Goodpasture.
101. Con respecto a la nefropatía diabética, señalar la respuesta FALSA:
1. La diabetes tipo 2 es la etiología más frecuente
de insuficiencia renal terminal en el
mundo occidental.
2. Más del 90% de los diabéticos tipo 1 desarrollan
nefropatía a los 30 años del diagnóstico
de diabetes.
3. La alteración renal más temprana es la hiperfiltración.
4. La existencia de microalbuminuria predice el
desarrollo de nefropatía clínica.
5. La gran mayoría de los diabéticos tipo 1 con
nefropatía tienen también retinopatía.
102. Un paciente de 75 años de edad, con historia antigua de hipertensión arterial, de hábito
asténico, consulta en la Urgencia del Hospital por un cuadro de dolor en fosa lumbar derecha
con hematuria de 6 horas de evolución. La tensión arterial es del 120/80 mmHg, y el paciente
está agitado, nauseoso y sudoroso, con el pulso irregular a 80 latidos por minuto. Analíticamente
tiene una urea de 50 mg/dl, creatinina 1 mg/dl, GOT 120 mU/ml, GPT 35 mU/ml, amilasa 120
mU/ml, LDH 1100 mU/ml. En el sedimento urinario hay microhematuria. La radiografía de
torax muestra una discreta cardiomegalia
a expensas de cavidades izquierdas y el electrocardiograma muestra una frecuencia auricular de
250 latidos minuto, con una respuesta ventricular irregular a 75 latidos por minuto. ¿Cuál es,
entre las siguientes, la decisión más terminante?:
1. Calmar ante todo el dolor y la ansiedad.
2. Hacer una ecografía abdominal.
3. Hacer un TAC espiral con contraste.
4. Acidificar la orina.
5. Alcalinizar la orina.

79. Un paciente asintomático, hipertenso de 65 años en tratamiento farmacológico, acude a su


médico que le encuentra en la analítica los siguientes parámetros: Hb 14,1 g/dl, VCM 88 fl, Urea
75 mg/dl, Creatinina 1,4 mg/dl. Sodio sérico 128 mEq/l. Potasio sérico 2,8 mEq/l. Cloro 89
mEq/l. Lo más probable sería:
1. Que tenga una anomalía en la absorción de
cloro, tipo Bartter.
2. Que la hipertensión sea secundaria a una
tubulopatía perdedora de potasio.
3. Que en el tratamiento que reciba exista un
diurético de asa.
4. Que en el tratamiento se incluya un inhibidor
de la enzima convertidora de la angiotensina.
5. Que no tome fruta en la dieta.

85. Un paciente que presenta en el sedimento de orina microhematuria, proteinuria y cilindros


hemáticos. ¿cuál de los siguientes cuadros patológicos padece?:
1. Lesión glomerular.
2. Lesión túbulo-intersticial.
3. Obstrucción de la vía urinaria.
4. Infección renal.
5. Neoplasia renal.
90. Paciente de 70 años que hace 2 semanas fue sometido a una coronariografía, acude al
hospital por aparición de lesiones purpúricas palpables en miembros inferiores, elevación de la
creatinina sérica de 3 mg/dl, proteinuria de 1g/24 h, hipocomplementemia y microhematuria y
leucocituria en el sedimento urinario. ¿Cuál es el diagnóstico más probable?:
1. Glomerulonefritis aguda rápidamente progresiva.
2. Glomerulonefritis aguda postestreptocócica.
3. Sindrome hemolítico-urémico.
4. Enfermedad atero-embólica.
5. Glomerulonefritis membrano-proliferativa.
167. Todo lo que sigue acerca del síndrome nefrótico en la infancia, es cierto EXCEPTO:
1. Colesterol sérico elevado.
2. El 85% experimenta cambios mínimos de la
enfermedad.
3. Reabsorción reducida de sodio por el riñón.
4. Triglicéridos séricos elevados.
5. La hipoalbuminemia es la causa de la hipoproteinemia.
173. Un niño de 4 años de edad muestra un importante retraso de crecimiento, lesiones de
raquitismo resistentes al tratamiento con dosis habituales de vitamina D y poliuria. ¿Cuál de las
siguientes asociaciones considera que permite el diagnóstico de síndrome de Fanconi?:
1. Glucosuria + hiperaminoaciduria + alcalosis
+ hiperfosforemia.
2. Glucosuria + hipoglucemia + acidosis metabólica
+ hipofosforemia.
3. Glucosuria + hiperaminoaciduria + acidosis
metabólica + hipofosforemia.
4. Glucosuria + hiperaminoaciduria + alcalosis
metabólica + hipofosforemia.
5. Hipoglucemia + hiperaminoaciduria + alcalosis
metabólica + hipofosforemia.
195. Paciente de 38 años, con fenómeno de Raynaud y esclerodactilia, presenta diarrea de heces
pastosas, en número de 2-3 deposiciones/día, pérdida de 7 kg de peso, anemia con volumen
corpuscular medio de 112 fl, vitamina B12 en sangre, 70 pg/ml (normal, 200-900 pg/ml), ácido
fólico sérico, 18 ng/ml (normal, 6-20 ng/ml), grasas en heces, 13 g/día. La prueba con mayor
sensibilidad, específica y sencilla para el diagnóstico del síndrome digestivo que padece este
enfermo es:
1. Anticuerpos antiendomisio tipo IgA.
2. Prueba del aliento con 14C-D-xilosa.
3. Determinación de la lactasa en la mucosa
intestinal.
4. Prueba del aclaramiento de la 1-antitripsina
en heces.
5. Tinción con PAS de la biopsia intestinal.
255. ¿Qué entenderemos por "riñón de mieloma"?:
1. Cualquier fracaso renal en un paciente portador
de un mieloma.
2. La infiltración renal por el mieloma.
3. La precipitación de cadenas ligeras en los
túbulos renales.
4. El depósito de proteína de Bence-Jones en
el glomérulo.
5. El fracaso renal causado por la hipercalcemia
del mieloma.

81. ¿Cuál de las siguientes permite diferenciar


la pielonefritis aguda de la cistitis aguda?:
1) Leucocituria.
2) Hematuria.
3) Bacteriuria.
4) Antecedentes de infección urinaria.
5) Fiebre de más de 38.5ºC.

118. La acromegalia puede formar parte de:


1) Síndrome de Nelson.
2) Síndrome de Turner.
3) Neoplasias endocrinas múltiples I (Men I).
4) Neoplasias endocrinas múltiples II (Men II).
5) Síndrome poliglandular autoinmune.

123. Una mujer de 62 años acude a urgencias letárgica y con una TA de 100/60. Su glucemia es
de 1250 mg/dl, la natremia de 130 mEq/l, la potasemia de 4,6 mEq/l, el cloro sérico de 100
mEq/l, el bicarbonato sérico de 18 mEq/l, la urea de 67 mg/dl y la creatinina de 2 mg/dl. El
tratamiento de esta enfermedad debe incluir:
1) Suero salino hipotónico.
2) Suero salino hipotónico e insulina.
3) Suero salino isotónico e insulina.
4) Suero salino hipotónico, bicarbonato e insulina.
5) Suero salino isotónico, bicarbonato e insulina.

125. ¿Con qué objetivo prescribiría una tiazolidinodiona a un paciente con diabetes mellitus?:
1) Para aumentar la secreción pancreática de
insulina.
2) Para mejorar la sensibilidad periférica de
insulina.
3) Como diurético coadyuvante a un IECA en
un diabético tipo 2.
4) Para reducir la glucogenólisis hepática.
5) Como vasodilatador, para mejorar el flujo
sanguíneo por los vasa nervorum en casos
de neuropatía severa.

126. Un hombre fumador, de 60 años, consulta por astenia, perdida de peso y deterioro general
progresivo. En la analítica se observa alcalosis e hipopotasemia de 2,8 meq/l. ¿Cuál es su
diagnóstico de sospecha?:
1) Hiperaldosteronismo primario.
2) Hipertiroidismo inmune.
3) Secreción ectópica de ACTH.
4) Enfermedad de Addison.
5) Secreción inadecuada de ADH.

157. En relación con el metabolismo y funciones


de la vitamina D, una de las siguientes
respuestas es FALSA:
1) La vitamina D ingerida por vía oral es vitamina
D2 ó D3. 2) El riñón transforma la 25 (OH) vitamina D
en el principio activo. 1.25 (OH) vitamina D.
3) La absorción de calcio en el intestino está
facilitada por la 24.25 (OH) vitamina D.
4) La malabsorción de la grasa puede provocar
déficit de vitamina D en áreas geográficas
de latitud norte durante el invierno.
5) La 1.25 (OH) vitamina D estimula la calcificación
ósea a través del aumento de producción
de osteocalcina y osteopondina.

174. Paciente de 50 años de edad, que consulta


por dolor en la fosa renal, polaquiuria, disuria
y hematuria. En el análisis de la orina
se observa piuria y pH ácido con cultivos
repetidamente negativos. ¿Cuál sería la primera
posibilidad diagnóstica, de entre las
siguientes?:
1) Pielonefritis aguda.
2) Síndrome nefrítico.
3) Tuberculosis genitourinaria.
4) Prostatitis aguda.
5) Carcinoma renal de células claras.
175. Un paciente diabético conocido, de 2 años de
evolución y 64 años de edad, consulta por
anemia, proteinuria de 3 gramos/24 horas
junto con hematuria, hipertensión arterial y
discreta insuficiencia renal con creatinina
plasmática de 2.3 mg/dl. El diagnóstico MENOS
probable es:
1) Nefropatía diabética.
2) Angeítis necrotizante del tipo poliangeítis microscópica.
3) Glomerulonefritis rápidamente progresiva.
4) Granulomatosis de Wegener.
5) Síndrome úremico-hemolítico.
176. Hombre de 56 años, con antecedentes de
ataque agudo de gota hace 2 años y ácido
úrico elevado (8,5 mg/dl), que refiere dolor
tipo cólico en fosa renal derecha ocasionalmente;
en la ecografía de aparato urinario
tiene una imagen con sombra acústica posterior
compatible con litiasis de 2x1 cm, en
pelvis renal derecha. En la urografía intravenosa
se aprecia una imagen radiotransparente
en pelvis renal derecha de aproximadamente
2 cm. de diámetro máximo. ¿Cuál
de las siguientes opciones constituiría el tratamiento
inicial más apropiado?:
1) Litotricia extracorpórea por ondas de choque.
2) Nefolitotomía percutánea.
3) Hidratación y ácido acetohidroxámico.
4) Ureteroscopia y extracción endoscópica del
cálculo.
5) Hidratación, alcalinización de la orina con
citrato potásico y alopurinol.
177. Si un paciente con una pérdida nefronal
progresiva, la presencia de hiperpotasemia
indica que ha perdido al menos:
1) Un 25% del filtrado glomerular.
2) Un 35% del filtrado glomerular.
3) Un 50% del filtrado glomerular.
4) Un 75% del filtrado glomerular.
5) Un 100% del filtrado glomerular.
178. Un paciente de 35 años con insuficiencia
renal crónica secundaria a pielonefritis crónica
recibe un trasplante renal de cadáver
con el que compartía dos identidades en Ay
B y una en DR. Recibe tratamiento inmunosupresor
con ciclosporina A y corticoides
a dosis estándar. En el postoperatorio inmediato
se observa buena diuresis y no es
necesario el tratamiento sustitutivo con hemodiálisis.
En el 5º día de evolución, el
paciente presenta fiebre de 38º. TA de 180/
110, oliguria y disminución en la concentración
urinaria de sodio. El diagnóstico más
probable sería:
1) Crisis hipertensiva.
2) Infección respiratoria.
3) Pielonefritis aguda del injerto renal.
4) Recidiva de su enfermedad renal.
5) Rechazo agudo del injerto renal.
179. Paciente de 63 años, fumador importante, que
refiere hematuria total intermitente y síndrome
miccional irritativo desde hace 1 mes.
La citología de orina es positiva. Se realiza
una evaluación vesical bajo anestesia con
biopsias vesicales múltiples, siendo diagnosticado
un carcinoma vesical "in situ". ¿Cuál
de las siguientes estrategias terapéuticas es
la más apropiada?:
1) Instilaciones endovesicales con Mitomicina C.
2) Instilaciones endovesicales con BCG.
3) Cistectomía radical.
4) Quimioterapia sistémica.
5) Radioterapia pelviana.
180. Un paciente de 24 años de edad sufre un
accidente de circulación con traumatismo
pélvico a importante hemorragia retroperitoneal.
Es intervenido en situación de shock
hemodinámico con TA de 60/30 mmhg. Tras
la reparación quirúrgica de ruptura de aorta
abdominal, permanece con aspiración nasogástrica
y evoluciona sin problemas durante
tres días con TA de 110/80. Al cuarto día la
diuresis disminuye y una analítica sanguínea
demuestra una urea de 169 mg% con creatinina
de 2.7 mg/dl. Los valores en orina
son: osmolaridd 650 mosm/l, sodio 10 meq/
l, relación urea orina/urea plasma: 18, relación
creatinina en orina/creatinina en plasma
70. La conducta más adecuada será:
1) Ecografía abdominal inmediata para descartar
uropatía obstructiva por hematoma retroperitoneal.
2) Aumentar el aporte líquido i.v. a 5 l/día,
incluyendo transfusión de hematíes concentrados
si fuera preciso.
3) Arteriografía renal para descartar obstrucción
arterial renal.
4) Pautar Manitol y fusosemida para que la
diuresis retorne a la normalidad.
5) Hasta que se aclare el diagnóstico, lo primero
es iniciar hemodiálisis.
181. Dentro de las anomalías congénitas ureterales
que se citan a continuación, la que presenta
una mayor incidencia es:
1) Uréter retrocavo.
2) Megauréter.
3) Ureterocele.
4) Uréter retroiliaco.
5) Duplicidad ureteral.
182. Una paciente de 15 años de edad consulta
por poliuria y nicturia, cansancio fácil y
astenia. Presenta dichos síntomas desde hace
años, y tienden a intensificarse durante los
veranos muy calurosos, en los que se asocia
hormigueo lingual y peribucal. A la exploración
física sólo destaca una tensión de 100/
50 mmHg. El ionograma muestra Na 135
mEq/l, K 2 mEq/l. CI 105 mEq/l, pH 7.45,
bicarbonato 30 mEq/l. La determinación de
renina y aldosterona muestra cifras elevadas,
tanto basales como tras estímulo. De
los procesos que siguen ¿cuál es compatible
con el cuadro clínico descrito?:
1) Hipoaldosteronismo primario.
2) Estenosis de la arterial renal.
3) Síndrome de Liddle.
4) Enfermedad de Addison.
5) Síndrome de Bartter.
183. ¿Cuál de las siguientes medidas terapéuticas
NO está indicada en el tratamiento de la
Hiperpotasemia?:
1) Glucosa e insulina.
2) Bicarbonato sódico.
3) Gluconato cálcico.
4) Resinas de intercambio iónico.
5) Glucocorticoides.
184. ¿Cuál de las siguientes nefropatías glomerulares
no se relaciona con hipocomplementemia?:
1) Glomerulonefritis aguda postinfecciosa.
2) Glomerulonefritis membrano-proliferativa.
3) Nefropatía lúpica.
4) Nefropatía diabética.
5) Nefropatía de la crioglobulinemia.
- 25 -
EXAMEN MIR 15-03-2003
185. Un paciente de 28 años presenta hematuria
macroscópica al día siguiente de una infección
faríngea y edemas maleolares. Dos años
antes había presentado un cuadro similar
que desapareció con rapidez, por lo que no
había consultado previamente. En la exploración
física se objetiva una T.A. de 180/110,
una Creatinina Plasmática de 2 mgrs/dl e
Hipocomplementemia. ¿Cuál de los siguientes
es el diagnóstico más probable?:
1) Glomerulonefritis membrano-proliferativa.
2) Glomerulonefritis post-estreptocócica aguda.
3) Glomerulonefritis rápidamente progresiva.
4) Glomerulonefritis por depósitos mesangiales
de IgA (Enfermedad de Berger).
5) Hialinosis Segmentaria y Focal.

187. ¿Cuál de los siguientes fármacos produce


alcalosis metabólica?:
1) Diuréticos ahorradores de potasio.
2) Diuréticos del asa.
3) Inhibidores de la enzima convertidora de la
angiotensina.
4) Antiinflamatorios no esteroideos.
5) Bloqueantes de los receptores de la Angiotensina
II.

193. Niña de 3 años, que presenta afectación brusca


del estado general dentro del contexto de
un cuadro febril con diarrea mucosanguinolenta.
A la exploración aparece pálida y
soñolienta, la auscultación cardiopulmonar es
normal, el abdomen doloroso y el resto del
examen físico sólo revela la presencia del
petequias puntiformes diseminadas. La orina
es hematúrica y se constata hipertensión
arterial. El hemograma muestra HB 7 g/dl,
17.000 leucocitos con neutrofilia, 37.000 plaquetas
con normalidad de las pruebas de
coagulación. ¿Cuál es el diagnóstico más
probable?:
1) Sepsis por Salmonella.
- 26 -
EXAMEN MIR 15-03-2003
2) Púrpura de Schönlein-Henoch.
3) Glomerulonefritis aguda post-infecciosa.
4) Síndrome hemolítico-urémico.
5) Coagulación intravascular diseminada.
194. Se trata de un niño de 7 años, con retraso
estaturo-ponderal, que presenta signos radiológicos
de raquitismo y ecográficos de nefrocalcinosis.
Los análisis demuestran acidosis
metabólica con PH inferior a 7.32 y
Bicarbonato plasmático inferior a 17 mEq/
litro. El PH de la orina en 3 determinaciones
es siempre superior a 5.5. Otras alteraciones
metabólicas consisten en hipercalciuria,
hipocitruria y discreta disminución de la
resorción del fósforo. No se detecta glucosuria
ni aminoaciduria. De las siguientes posibilidades,
señale el diagnóstico correcto:
1) Síndrome de Bartter.
2) Acidosis tubular distal (Tipo I).
3) Acidosis tubular proximal (Tipo II).
4) Síndrome completo de Fanconi.
5) Enfermedad de Harnup.

1. Una paciente mujer, de 35 años, es atendida por el medico interno de turno en el HST;
la paciente aqueja molestias relacionadas al sistema renal, pero al examen físico, llama la
atención, que el interno, no logra palpar los pulsos ni captarle la PA braquial; este medico
interno, habiendo prestado atención a sus clases, en la universidad, debe pensar que el
posible daño renal de esta paciente, es secundario a:

a. Granulomatosis de Wegener.
b. Poliangeitis Microscópica.
c. Crioglobulinemia Mixta Esencial.
d. Arteritis de Takayasu.

2. Cual de las siguientes, se considera una Glomerulopatia Aguda:

a. GN proliferativa endocapilar.
b. GN mesangial.
c. GN mesangiocapilar.
d. Nefropatia membranosa.

3. Un paciente masculino de 33 años llega a su consulta por presentar fiebre, un pequeño


rash, signo de Jordano positivo; el medico de turno le envía un urinalisis que revela
proteinuria, hematuria y múltiples leucocitos; dentro de su diagnostico diferencial, debe
estar una de las siguientes patologías, cual:

i. Nefritis Intersticial Crónica.


j. Nefritis Intersticial Aguda.
k. Síndrome Nefrosico.
l. Fracaso Renal Agudo.

4. Un paciente de 12 años, llega a la consulta de Nefrología; el Dr. Medina, le encuentra


que esta desarrollando un Sd. Nefrosico, de acuerdo con su edad, la Glomerulopatia que
puede tener este paciente de base, podría ser:

a. Cambios Mínimos.
b. Nefropatia Membranosa.
c. Diabetes.
d. Amiloidosis.
5. Dentro de las Glomerulopatias, existen algunas, que se pueden presentar con Síndrome
Nefrosico, y otras con Síndrome Nefrítico, cual de las siguientes se suele presentar con
Síndrome Nefrítico:
i. Cambios Mínimos.
j. Nefropatia IgA.
k. Nefropatia Membranosa.
l. Diabetes.

6. En un paciente de 65 años, que desarrolla un Sd. Nefrítico Agudo, la etiología mas


común causando de la Glomerulopatia es:

i. Vasculitis.
j. GN rápidamente progresiva.
k. Nefropatia IgA.
l. LES.

7. Cual de las siguientes Glomerulopatia, siempre desarrolla Insufiencia Renal:

i. Cambios Mínimos.
j. Membranosa.
k. Rápidamente Progresiva.
l. Mesangiocapilar.

8. Cual, raramente desarrolla HTA:

a. Cambios Mínimos.
b. Membranosa.
c. Rápidamente Progresiva.
d. Mesangiocapilar.

9. Un paciente joven, con antecedentes de una faringitis tratada y curada sin remisiones,
llega a la consulta externa del Hospital Nicolás Solano de la Chorrera, por estar
presentando alteraciones renales, se llega al diagnostico de Glomerulonefritis post
streptococcica, la base fisiopatologica de esta enfermedad es:

a. Daño renal directo por la bacteria.


b. Deposito de Complejos Circulantes.
c. Deposito de Inmunocomplejos.
d. Daño mediado por complemento.

10. Cual de las siguientes Glomerulopatias, se produce por deposito mesangial:

a. Nefropatia IgA.
b. GN membranosa.
c. GN esclerosante y focal.
d. GN rápidamente proliferativa.
11. Escriba la formula usada en clínica para el calculo de la Osmolaridad:
12. Paciente de 60 años, que al hacercele una química sanguínea, se le encuentra el Sodio
plasmático en 120 mEq/lt, y al calcular la osmolaridad, se da cuenta que tiene un liquido
hipertonico, en su diagnostico diferencial debe estar:

a. DM de novo.
b. DM descompensada.
c. DM tipo 1.
d. Diabetes Insípida.

13. La hiponatremia hipotónica, también se suele clasificar en base a:

a. Volemia.
b. Edad.
c. Sexo.
d. Virginidad.

14. Mencione 2 causas, para cada caso, de hiponatremia hipotónica:

a. Hipervolemica:
___________________________________________________
Insuficiencia cardíaca congestiva, Cirrótico con ascitis, Síndrome nefrósico y
Desnutrición avanzada
b. Normovolemica:
__________________________________________________
SIADH, hipotiroidismo, insuficiencia de glucocorticoides, psicosis
c. Hipovolemica:
____________________________________________________
Vómitos (pierde K+, HCl), diarrea (pierde K+, HCO3-), pancreatitis, quemaduras del
tercer espacio, trauma muscular.

15. En un paciente con SIADH, al medirle la osmolaridad urinaria, la podemos encontrar:

d. Elevada.
e. Disminuida.
f. Normal.

16. La hiponatermia, se hace clínicamente evidente cuando el sodio, ha disminuido a:

e. 110 mEq/lt.
f. 115 mEq/lt.
g. 120 mEq/lt.
h. 125 mEq/lt.
17. Al corregir el Sodio en una hiponatremia, las guias dicen que debe hacerce:
_________
________________________________________________________________________
_

18. Si el medico, no estudio su modulo de Nefrología, y lo corrige a mayor velocidad,


corre el riesgo de que su paciente desarrolle una:

a. Desmielinizacion osmótica.
b. Hiponatremia mas severa.
c. Desmielinizacion optica.
d. SIADH.

19. Escriba la formula de correcion de la hiponatremia en una mujer:

20. El Hiperaldosteronismo primario, suele causar una alteración del sodio, cual:

a. Hipernatremia hipervolemica.
b. Hipernatremia normovolemica.
c. Hipernatremia hipovolemica.

21. El Síndrome de Cushing, tiene como base fisiopatologica el aumento de:

a. Corticoides.
b. Andrógenos.
c. Esteroides.
d. Cuerpos Cetónicos.

22. La corrección el Sodio en una hipernatremia, puede causar daño cerebral,


produciendo un Síndrome conocido como:

a. Deshidratación Cerebral Primaria.


b. Deshidratación Cerebral Primaria Hipertonica.
c. Deshidratación Cerebral Primaria Hipertonica Intracelular.
d. Ninguna de las anteriores.

23. Escriba la formula de corrección de la hipernatremia, en un varón:

(Peso)(60%)(sodio del paciente/ sodio al que se quiere llevar -1)

24. Mencione 5 medicamentos que puedan causar hiperkalemia:

f. ________________________________________________.
g. ________________________________________________.
h. ________________________________________________.
i. ________________________________________________.
j. ________________________________________________.
citrato de K, gluconato de K, acetato de K, bicarbonato de K y Triamtereno

25. Menciones en orden, las manifestaciones en el EKG de una hipokalemia:

a. ________________________________________________.
b. ________________________________________________.
c. ________________________________________________.

la onda T se alarga, el segmento QT (indica la sístole eléctrica del corazón) se prolonga y onda u

26. Las características del Síndrome Nefrotico son:

m. ________________________________________________.
n. ________________________________________________.
o. ________________________________________________.
p. ________________________________________________.
q. ________________________________________________.
r. ________________________________________________.

28. Defina Falla Renal Aguda:

29. Los tres puntos en los que se basa la corrección de la hiperkalemia son:

d. ________________________________________________.
e. ________________________________________________.
f. ________________________________________________.
EKG y posteriormente gases arteriales (pH), creatinina.

30. Para cada uno de estos puntos, se suele usar: se le solicita al estudiante ser lo mas
explicito posible en su respuesta, en cuanto a dosis y tratamiento exacto.

d. ________________________________________________.
e. ________________________________________________.
f. ________________________________________________.

31. Dentro de las medidas de sostén, para evitar, que durante el tratamiento de una
hiperkalemia, el potasio, de siga aumentando, tenemos:

a. ________________________________________________.
b. ________________________________________________.
Gluconato de calcio
Insulina

32. Debemos sospechar, de alteraciones del Magnesio, en que pacientes:

d. ________________________________________________.
e. ________________________________________________.
f. ________________________________________________.
Alcoholicos
Embarazadas
Falla renal

33. Paciente con hipokalemia + alcalosis metabólica + hipertensión arterial +


disminución de la renina plasmática, esta desarrollando un:

b. ________________________________________________.
Síndrome de cushing

34. Única causa de diarrea con alcalosis metabólica es:

b. ________________________________________________.
Adenoma velloso Hipercloremica

35. Si un paciente, tiene un potasio de 2 mEq/lt, decimos que ha perdido ya:

b. ________________________________________________.

36. Tratamiento de la hipermagnesemia:

a. ________________________________________________.
b. ________________________________________________.
c. ________________________________________________.

SS isotónica
Gluconato de calcio
Diálisis

37. Clínicamente, las alteraciones en el magnesio se ven así:

a. Cuando aumenta hay: _________________________________________.


inhibición de la conducción neuromuscular, letargia , hipotesión, ensanchamiento del
PR(bloqueo AV de primer grado) o un ensanchamiento del QRS y paro respiratorio
b. Cuando disminuye hay: ________________________________________.
Sistema nervioso (chvostek y trousseau, convulsiones. tetania), cardiaco
(alargamiento del segmento QT, arritmias), respiratorio (broncoespasmos,
laringoespasmos, apnea), y la esfera mental (puede haber demencia, psicosis).
38. Cual es la principal preocupación que se tiene, cuando un paciente tiene un aumento
en los niveles plasmáticos de Magnesio:

b. ________________________________________________.
Paro respiratorio

39. Causas de acidosis metabólica con hipokalemia:

d. ________________________________________________.
e. ________________________________________________.
Acidosis tubular renal proximal y distal

40. Que examen le ayudaria para diferenciarlas:

b. ________________________________________________.

41. Que esperaria encontrar en una ATR tipo 2:

b. ________________________________________________.

42. Menciones los niveles plasmticos normales de:

a. Sodio: ________________________________________________.
135 – 145 mEq/L
b. Potasio: ________________________________________________.
3.9 – 5.1 mEq/L
c. Magnesio: ________________________________________________.
1.6 – 2.3 mg/dL
d. Calcio: ________________________________________________.
8.5 – 10 mg/dL
e. Fósforo: ________________________________________________.
2.5 – 4.5 mg/dL
f. Bocarbonato: ________________________________________________.
g. pH: ________________________________________________.
h. pCO2: ________________________________________________.
i. Hemoglobina: ________________________________________________.
j. Leucocitos: ________________________________________________.

43. Principales causas de hipercalcemia:

c. ________________________________________________.
d. ________________________________________________.
Hiperparatiroidismo primario
Neoplasia malignas
44. Principales causas de hipocalcemia:

c. ________________________________________________.
d. ________________________________________________.
Hipoparatiroidismo
IRC

45. Tratamiento de la hipercalcemia:

a. ________________________________________________.
Hidratación con solución salina

46. Consecuencias de la hiperfosfatemia:

c. ________________________________________________.
d. ________________________________________________.
e. ________________________________________________.
f. ________________________________________________.
g. ________________________________________________.

el potencial de reposo transmembranal se hace menos negativo


(onda T) la cual se hace más picuda
la onda P desaparece
ensanchamiento del complejo QRS
arritmias

47. El Sd. De Barter, se caracteriza clínicamente por:

e. ________________________________________________.
f. ________________________________________________.
g. ________________________________________________.
h. ________________________________________________.
Hipercalciuria
Hipokalemia
Alcalosis metabolica
Normotensión
Infacia

48. Los 4 sindromes básicos de la nefrología son:

a. ________________________________________________.
b. ________________________________________________.
c. ________________________________________________.
d. ________________________________________________.

49. Podemos clasificar la FRA de acuerdo a el volumen de orina en 24 horas, en:


a. ________________________________________________.
b. ________________________________________________.
c. ________________________________________________.

50. De acuerdo al sitio anatomico, la FRA puede ser:

a. ________________________________________________.
b. ________________________________________________.
c. ________________________________________________.
51. En un paciente con diagnostico de Glomerulonefritis post estreptococcica, el
diagnostico serologico se basa en el hallazgo de anticuerpos:

i. Antiestreptolisina O.
j. Antiestreptolisina A.
k. Antiestreptolisina P.
l. Antiestreptolisina K.

52. En la Nefiritis secundaria al LES, se suelen encontrar positivos los:

i. ANA.
j. ANCA c.
k. ANTI LU
l. ANCA p.

53. Si una mujer, aparte de tener problemas renales, tiene manifestaciones clínicas de
ulceraciones cutáneas, artralgias, mialgias y fenómeno de Reynoud, hay que pensar en:

a. Glomerulopatia por Crioglobulinemia.


b. Glomerulopatia por LES.
c. Glomerulopatia inducida por medicamentos.
d. Glomerulopatia por IgA.

54. A la Nefropatia por IgA, también se le conoce como Nefropatia de:

a. Henoch-Scholein.
b. Berger.
c. Takayasu.
d. Gerber.

55. Si un paciente, tiene una glomerulopatia, y serologicamente presenta anti-GBM +,


esto nos confirma el diagnostico de glomerulopatia secundaria a:

a. LES.
b. Púrpura Trombocitopenica Trombotica.
c. Enfermedad anti membrana basal.
d. Nefritis tubulo intersticial.
56. Si este mismo paciente, tiene manifestaciones de hemorragias pulmonares, agregadas
a le lesión renal, se diagnostica con:

a. Sd. De Bartter.
b. Sd. De Lorein – Bitman.
c. Sd. De Goodpasteure.
d. Sd. De Lorein – Stevenson.

57. El Síndrome de la pregunta anterior, es mas frecuente en:

a. Varones.
b. Mujeres.
c. Ambos Sexos.
d. Homosexuales.

58. Y se presenta en que grupo etario predilectamente:

a. jóvenes.
b. Ancianos.
c. Recién nacidos.
d. Todos por igual.

59. Si un paciente con glomerulopatia, tiene ANCA positivo, hay que pensar en:

a. Glomerulopatia Pauciinminutaria.
b. Glomerulopatia de Berger.
c. Síndrome de Ghitelman.
d. Síndrome de Guillian Barre.

60. Si un paciente es diagnosticado con Falla Renal Crónica, el siguiente punto es el


estadiage del mismo, si tiene una TFG en 50 ml/minuto, esta en estadio:

k. I
l. II
m. III
n. IV
o. V

61. Si un paciente, diagnosticado con FRC, presenta ademas una acidosis metabolica
hipercloremica, usted sabe que este paciente debe tener una Brecha Anionica:

i. Normal.
j. Aumentada.
k. Disminuida.
l. Ninguna de las anteriores.

62. Este mismo paciente, por tener estas características clínicas, debe encontrarse en
estadios de la FRC:

i. Avanzados.
j. Iniciales.
k. No es relevante.
l. Ninguna de las anteriores.

63. Menciones algunas características físicas, que nos ayudarian a pensar que un
paciente, de la calle, por nuestra experiencia en este modulo, tiene una FRC:

a. ________________________________________________.
b. ________________________________________________.
c. ________________________________________________.
d. ________________________________________________.

64. La Osteodistrofia Renal, se va a caracterizar clínicamente por la presencia de:

c. ________________________________________________.
d. ________________________________________________.

65.Normalmente la Kps entre el Fósforo y el calcio es de:

b. ________________________________________________.

66. En un paciente con FRC, como espera usted encontrar:

a. Sodio: ________________________________________________.
b. Calcio: ________________________________________________.
c. Fósforo: ________________________________________________.

La hipokalemia produce:
- mayor negatividad del potencial de reposo.

La hipokalemia en un cirrótico puede precipitar COMA por:


- aumento de la amniogénesis

Exceso del kayaxelate en el tratamiento de hiperkalemia puede producir:


- constipación.

Peligro de la hipermagnesemia severa:


- paro respiratorio.
Hipofosfatemia:
- aumenta el calcitriol.

Consecuencia metabólica de hipofosfatemia severa:


- miocardiopatía

Causa de hiperfosfatemia:
- insuficiencia renal

El Síndrome Nefrósicose caracteriza por:


- proteinuria mayor o igual de 3g y albúmina sérica menor o igual de 3g/dl.

En el Síndrome Nefrósico pueden encontrarse:


- cilindros céreos

Urémico agudo, oligúrico, acidótico, deshidratado puede presentar:


- osmolaridad urinaria de 250mosm/kg

La falla renal crónica avanzada cursa con:


- brecha aniónica elevada

Cuál de los siguientes antibióticos produce nefritis intersticial:


- meticilina

El factor más determinante en el tratamiento de la hipokalemia renal es:


- la función cardiovascular.

El tratamiento de inicio de secreción inapropiada de ADH es:


- restricción hídrica

95. Señale la respuesta correcta referida a las alteraciones del examen de la orina:
1. La presencia de cilindros hialinos es siempre patológica.
2. Los cilindros granulosos contienen albúmina e Inmnoglobulinas.
3. Los cilindros leucocitarios son típicos de Glomerulonefritis postestreptocócica.
4. Los cilindros hemáticos se presentan en cualquier discrasia sanguínea.
5. Un resultado negativo de presencia de nitritos en tira reactiva, excluye la existencia de baterias.
96. Paciente de 68 años diabética, que consulta por malestar general. En la gasometría venosa
destaca pH 7.25, Bicarbonato 15 mmol/ l (normal 24-28 mmol/l). Hiato anionico
(anión GAP): 11 mmol/l (normal 10-12 mmol/l). ¿Cúal de las siguientes entidades
NO descartaría como diagnóstico?:
1. Cetoacidosis diabética.
2. Insuficiencia renal crónica.
3. Acidosis tubular renal.
4. Ingesta de salicilatos.
5. Acidosis láctica.
97. Cuando un paciente bajo tratamiento diurético con tiazidas o furosemida incumple la
dieta y come más sal de la prescrita, el resultado análitico esperable es:
1. Mayor hipernatremia.
2. Mayor hiponatremia.
3. Mayor hiperpotasemia.
4. Mayor hipopotasemia.
5. Mayor acidosis.
98. En la uremia pre-renal:
1. El sodio en orina es superior a 60 mEq/l, la osmolaridad urinaria es superior a 500 mOsm/
kg H2O y la relación urea en orina/urea en plasma es superior a 8.
2. El sodio en la orina es inferior a 20, la osmolaridad urinaria es inferior a 200 mOsm/ kg H2O y la
relación entre urea en orina / urea es plama es inferior a 2.
3. El sodio en orina es inferior a 20 mEq/l, la osmolaridad en orina es inferior a 200 mOsm/ Kg H 2O y
la relación urea en orina / urea en plama es superior a 8.
4. El sodio en orina es inferior a 20 mEq/l, la osmolaridad urinaria es superior a 500 mOsm/ Kg H 2O,
y la relación entre la urea en orina y la urea en plasma es superior a 8.
5. El sodio en orina es superior a 60, la osmolaridad urinaria es superior a 500 mOsm/Kg H 2O, y la
relación urea en orina/urea en plasma es superior a 8.
99. Un hombre, adicto a drogas por vía parenteral, está ingresado por endocarditis infecciosa.
Durante su enfermedad presenta un cuadro de glomerulonefritis aguda. ¿Cuál de las respuestas
es INCORRECTA?:
1. Suele ser debida a inmunocomplejos.
2. No suele presentar piuria.
3. El complemento está descendido.
4. A veces produce síndrome nefrótico.
5. Suele evolucionar favorablemente al controlar la infección cardiaca.
100. Una mujer de 68 años acude al Servicio de Urgencias por malestar general que ha ido
progresando en los últimos 15 días, a partir de un episodio gripal. Ha notado disminución
progresiva del volumen de diuresis, edemas maleolares y dificultad respiratoria. Es hipertensa.
En la analítica destaca una creatinina plamática de 5 mg/dl, urea 180 mg/dl, Na 138 mEq/l, K
4.9 mEq/l. Las cifras de complemento son normales. Los anticuerpos anti-membrana basal son
negativos. En la orina presenta cilindros hemáticos, proteinuria de 1 g/l y microhematuria.
Aporta una analítica de un mes antes, sin alteraciones. ¿Cuál de los siguientes diagnósticos es
más probable?:
1. PAN microscópica.
2. Brote lúpico.
3. Glomerulonefritis aguda postinfecciosa.
4. Crioglubulinemia.
5. Enfermedad de Goodpasture.
101. Con respecto a la nefropatía diabética, señalar la respuesta FALSA:
1. La diabetes tipo 2 es la etiología más frecuente de insuficiencia renal terminal en el mundo
occidental.
2. Más del 90% de los diabéticos tipo 1 desarrollan nefropatía a los 30 años del diagnóstico de
diabetes.
3. La alteración renal más temprana es la hiperfiltración.
4. La existencia de microalbuminuria predice el desarrollo de nefropatía clínica.
5. La gran mayoría de los diabéticos tipo 1 con nefropatía tienen también retinopatía.

102. Un paciente de 75 años de edad, con historia antigua de hipertensión arterial, de hábito
asténico, consulta en la Urgencia del Hospital por un cuadro de dolor en fosa lumbar derecha
con hematuria de 6 horas de evolución. La tensión arterial es del 120/ 80 mmHg, y el paciente
está agitado, nauseoso y sudoroso, con el pulso irregular a 80 latidos por minuto.
Analíticamente tiene
una urea de 50 mg/dl, creatinina 1 mg/dl, GOT 120 mU/ml, GPT 35 mU/ml, amilasa 120 mU/ml,
LDH 1100 mU/ml. En el sedimento urinario hay microhematuria. La radiografía de torax
muestra una discreta cardiomegalia a expensas de cavidades izquierdas y el electrocardiograma
muestra una frecuencia auricular de 250 latidos minuto, con una respuesta ventricular irregular
a 75 latidos por minuto. ¿Cuál es, entre las siguientes, la decisión más terminante?:
1. Calmar ante todo el dolor y la ansiedad.
2. Hacer una ecografía abdominal.
3. Hacer un TAC espiral con contraste.
4. Acidificar la orina.
5. Alcalinizar la orina.

Modulo de Renal
• La PTH promueve todo lo siguiente excepto:
o EL aumento de la reabsorción tubular de fosfato.
La PTH a nivel renal es eminentemente fosfatúrica, promueve la excreción de fosfato y la
reabsorción de Calcio.
• Síndrome nefrósico de cambios mínimos se caracteriza por:
o Fusión de los pedicelos de los podocitos.
La fusión de los pedicelos no se puede ver en microscopia de luz, hay que usar
microscopia electrónica, pero se puede hacer el dx con la proteinuria masiva
característica del síndrome.
• En un paciente con eclampsia se le ordeno un litro de solución de dextrosa al 5%,
conteniendo 20 ml de MgSO4 al 50% (PM= 100 g), ¿cuantos ml de sal se necesita
para preparar la solución?
o 10 ml.
• El porcentaje de solución es:
o 1%
• ¿Cuantos mEq de Mg tiene la venoclisis?
o 83.3
• La hipocalcemia de la falla renal crónica puede cursar:
o Asintomática.
• La velocidad de administración de KCL en un caso de alto riesgo puede ser:
o 40 mEq/L
En los pacientes con una hipokalemia muy severa se usa 40, pero se debe pasar a una
velocidad de 20, porque si se pasa muy rápido puede ocasionar un bloqueo AV completo.
• Paciente urémico, con K sérico de 6, Academia, EKG normal se trata con:
o Kayexalate
Ya que la hiperkalemia no es tan severa se puede tratar con el Kayexalate, que es una
resina de intercambio catiónico, la misma remueve el potasio del cuerpo.
• Paciente femenina de 50 Kg presenta diarrea, presión arterial un poco baja, sin signos
neurológicos, Na=120, K=2, Cl=100, HCO3=10, asuma 60% de agua, cuantos
gramos de Sal para elevar el Na a 130
o 17.5
Calculo: (60*0.5)*10=300 se necesita dos soluciones salinas (154*2=308), cada solución
salina tiene 58.5 g NaCl y cada gramo de solución tiene 17.5 g de Na.
• ¿Qué solución salina usted escogería?
o 0.9 %
• La hipofosfatemia produce:
o Fosfaturia
La disminución del calcio, produce un aumento de la PTH y esta a su vez un aumento de
la eliminación del fósforo por la orina.
• La hipokalemia puede predisponer, excepto:
o Disminución de la amoniogenesis.
La hipokalemia por lo general cursa con acidosis metabólica, en donde va a estar
aumentada la formación de amonio.
• Un inhibidor de la anhidrasa carbónica es causa de:
o Hipokalemia
Ellos producen un aumento de la excreción de bicarbonato al ser este un anión, arrastra el
potasio que es un catión.
• Un hombre llega al hospital comatoso, con fractura de cráneo, orina 175, suero de
Na=178, K=4, Cl=130, HCO3=25 Osm=350 en la orina 800 (no c ve bien).
o Diabetes insípida central.
Recordar que una de las causas de hipernatremia es la diabetes insípida.
• Confirmaría su dx así:
o Prueba de restricción y administración de vasopresina.
• Causa de HTA con hipokalemia excepto:
o Síndrome de Bartter.
El síndrome de Bartter no cursa con hipertensión arterial (HTA maligna,
hiperaldosteronismo primario y síndrome de hiperprostaglandismo entre otros).
• Se relaciona con hipofosfatemia, excepto:
o Disminución de la afinidad de la hemoglobina por el O2.
La hipofosfatemia produce una disminución del 2,3-DPG, y hay una desviación a la
izquierda, lo que produce una aumento de la afinidad por el oxigeno.
• Paciente con vómitos severos, hipotensión arterial, la orina al azar muestra Na mas de
40, osmolaridad de 800, puede tener:
o Falla suprarrenal.
El paciente al tener una hipovolemia, se activa el SRAA, y aumenta la reabsorción de Na
y disminuye su cantidad en orina.
• Un hombre de 30 Kg padece de Ca pulmonar, tiene 2 semanas de estar letargico y
obnubilado, suero de Na 105, Cl 72, K 4, HCO3 21, Osm 122 y en la orina Na 78,
Osm 604, el dx mas probable:
o SIADH.
Hay una hiponatremia, hipotónica y euvolémica.
• Tratamiento:
o Solución salina al 3%
El tratamiento de SIADH, es restricción de agua y suplir el déficit de sodio.
• Causas de enfermedad tubular intersticial crónica excepto:
o Nefritis alérgica.
En su mayoría medicamentos y medio de contraste.
• La administración de solución salina isotónica produce:
o Aumento del volumen extracelular.
• Sospecha de obstrucción de las vías urinarias en caso de:
o Anuria.
• En la insuficiencia renal aguda siempre da, excepto:
o Retensión nitrogenada
Es un parámetro a tomar en cuenta para dializar al paciente cuando es mayor de 100.
• Siempre habrá insuficiencia renal en caso de:
o Glomérulo nefritis rápidamente progresiva
La proliferación extracapilar comprime el glomérulo comprimiéndolo y dañándolo
severamente.
• En caso de glomérulo nefritis post estreptocócica la filtración glomerular puede
disminuir por:
o Alteración del coeficiente de ultrafiltración.
• En la nefritis túbulo intersticial puede dar lo siguiente, excepto:
o Proteinuria de 4 gr/día
Ese dato es de síndrome nefrósico.
• Es la causa de falla prerrenal.
o Pancreatitis aguda.
• En la insuficiencia renal aguda siempre habrá:
o Aumento de la creatinina.
• Causa de hipmagnesemia:
o Alcoholismo.
Otra causa importante son las pérdidas gastrointestinales.
• La hipofosfatemia causa, excepto:
o Aumento de PTH.
• Paciente con diuresis de 250 ml al día con Osm plasmática y osmolaridad urinaria de
300, el Dx probable:
o FRA, de causa renal.
• Paciente con oliguria, hiperkalemia, hiponatremia, creat 0.8 mg/dl y cpk normal, esto
puede corresponer a:
o Falla renal aguda, de causa prerrenal.
• En falla renal crónica:
o Hipofosfatemia.
• Urémico crónico con K 6.5 con leve academia, sin trastorno muscular y EKG normal,
tratamiento:
o Kayexalate.
• Urémico agudo, acidótico, oligúrico, deshidratado, todo es cierto, excepto:
o Densidad urinaria 1010.
Esto indica isostenuria, el riñón no tiene la capacidad de concentrar y es indicativo de
riñones muy maltratados.
• Falla renal aguda, oliguria, todo es cierto, excepto:
o Osmolaridad urinaria elevada.
• Síndrome nefrósico de cambios mínimos se caracteriza:
o Fusión de los pedicelos de los podocitos.
• En el shock hipovolémico, el uso de dextrosa en agua puede causar:
o Hiponatremia sintomática.
Primero siempre corregir la volemia con SSN 0.9.
• En la deshidratación por perdida fundamental de solutos, se da la siguiente, excepto:
o Entrada de electrolitos al cerebro.
• En el SIADH se da lo siguiente excepto:
o Edema de los miembros inferiores.
• Paciente con gastroenteritis, hipovolémico, cuyo suero contiene Na 155, K 2, Cl 117:
o SSN 0.45 + 40 mEq KCl.
• Una medida general en el tx de un hipercalcémico:
o SSN 0.9%
Para expandir el volumen.
• La velocidad de adminis tracion de K en caso de riesgo para la vida puede ser:
o 40
• Un ejemplo de hipernatremia hipervolemica es:
o Hiperaldosteronismo primario.
• Una dieta de 3gr de NaCl corresponde a:
o 17 gr de Sodio.
• La IRC ocaciona hiperfosfatemia permanente cuando la TFG esta por debajo de:
o 25 ml/min
El valor es por debajo de 30.
• Paciente con alcalosis metabólica e hipokalemia severa debe ser tratado con:
o KCl
• No esta indicada en la hipercalcemia:
o Aldactone
Es un diurético ahorrador de K, ósea produce hipercalcemia, se utiliza en el
hiperaldosteronismo primaria y edema.
• Metabolismo del calcio están implicados menos:
o Insulina.
• Hiponatremia hipotónica normovolémica:
o SIADH.
• Causa de enuresis
o Necrosis cortical difusa.
• La situación clínica de hiperfosfatemia con hipocalcemia con cirrosis hepática es:
o Cirrosis alcohólica.
• Factor que altera el balance hídrico en el periodo post operatorio excepto:
o Disminución de ADH.
• Ejemplo de hipernatremia hipervolemica:
o Hiperaldosteronismo primario.
• Ejemplo de hipokalemia con aldosterona alta y osm baja:
o Hiperaldosteronismo primario.
También con renina baja.
• Hiperkalemia sin alteración en el EKG no es necesario administrar:
o Gluconato de calcio.
• Urémico crónico con deshidratación hipertónica severa y acidótico presentara lo
siguiente excepto:
o Osmolaridad urinaria de 800.
• La fracción de excreción de Na esta aumentada en:
o Falla cardiaca.
También esta aumentada en FRC, de causas renales.
• Paciente con vomito abundante, HTA, Una 6, oligúrico y osmolaridad de 800,
presentara:
o Insuficiencia suprarrenal.
• En cirrótico con ascitis se recomienda, excepto:
o Dieta con 8.5 mEq de Na.
60 to 90 mEq per day, equivalent to approximately 1500 to 2000 mg of salt per day
(NEJM).
• Factor que predispone a la hipernatremia son, excepto:
o Intoxicación hídrica.
• En la hiperkalemia:
o Hay menos diferencia entre el potencial de reposo y el umbral.
• Los indicios de falla renal aguda son:
Pueden ser cualquiera de las siguientes:
o Alteraciones en la osmolaridad urinaria.
o Alteraciones sen la relación Uosm/Posm.
o Alteraciones en el Una.
o Alteraciones en el FENA.
• Se presenta en acidosis tubular renal proximal, excepto:
o NaHCO3 y los alpha agonista.
• Na urinario menor de 10:
o Falla cardiaca.

1. La policia encuentra en la calle, incosciente e inmóvil, a altas horas de la madrugada a un


indigente

2. que presenta múltiples hematomas y fetor etílico. En el hospital se le detecta urea de 200
mg/dl, creatinina de 6 mg/dl, ácido úrico de 10 mg/dl y CPK de 1500 U/l. El diagnóstico
probable es:

Necrosis tubular aguda alcohólica.


Fracaso renal agudo por urato.
Necrosis tubular aguda por hemólisis.
Infarto agudo de miocardio en paciente con insuficiencia renal crónica.
Fracaso renal agudo por radbomiolisis.
76 respuestas: 68.4% correctas/ 31.6% incorrectas => Pregunta fácil
2. Ante el cuadro descrito en la pregunta anterior, la actitud más adecuada, de las siguientes, sería:

Restricción de líquidos a 800-1000 ml/día.


Tratamiento antibiótico empírico en espera de los cultivos.
Reposición de líquidos con suero salino isotónico ajustando el ritmo a la diuresis.
Iniciar hemoperfusión.
Reposición de líquidos con suero salino hiperosmolar.
68 respuestas: 22.1% correctas/ 77.9% incorrectas => Pregunta difícil

4. Ante una sospecha de enfermedad renal poliquística del adulto, el paso diagnóstico siguiente más
razonable, ente los que se señalan, es:

Tomografía.
Urografía intravenosa.
Ecografía.
Tomografía axial (TAC).
Angiografía.
66 respuestas: 60.6% correctas/ 39.4% incorrectas => Pregunta regular

5. ¿Qué entenderemos por "riñón de mieloma"?:

Cualquier fracaso renal en un paciente portador de un mieloma.


La infiltración renal por el mieloma.
La precipitación de cadenas ligeras en los túbulos renales.
El depósito de proteína de Bence-Jones en el glomérulo.
El fracaso renal causado por la hipercalcemia del mieloma.
68 respuestas: 38.2% correctas/ 61.8% incorrectas => Pregunta regular

6. Un paciente de 68 años presenta progresivo deterioro del nivel de conciencia en las horas siguientes
a haber sido operado de herniorrafia inguinal con anestesia general, de la que despertó
normalmente. Su situación hemodinámica es estable, no hay signos de insuficiencia cardíaca, no tiene
fiebre ni focalidad neurológica. El hemograma y el estudio de coagulación son normales. La
bioquímica sérica muestra: urea 36 mg/dl, Glu 120 mg/dl, Na+ 118 mEq/l, K+ 4,5 mEq/l, úrico 2,3
mg/dl, osmolaridad urinaria 500 mOsm/ kg, Na+ urinario 35 mEq/l. Señale, de las siguientes, la
causa más probable de este cuadro:

Insuficiencia hepática aguda por anestésicos.


Deshidratación.
Hiponatremia ADH-dependiente.
Tubulopatía pierde-sal.
Sepsis en fase inicial.
69 respuestas: 49.3% correctas/ 50.7% incorrectas => Pregunta regular

7. ¿Cuál de las siguientes afirmaciones es cierta con respecto a la enfermedad de Hodgkin?:

Es de comienzo extranganglionar con mayor frecuencia que los linfomas no Hodgkin.


La célula de Redd-Sternberg, aunque característica de la enfermedad es un acompañante no
tumoral.
A diferencia de los linfomas no Hodgkin, es característica su extensión a otros territorios linfoides
por contigüidad.
La variante de esclerosis nodular corresponde al 1-5% de todos los casos de enfermedad de Hodgkin.
Los hallazgos moleculares sugieren que se trata de una neoplasia de origen histiocítico.
67 respuestas: 46.3% correctas/ 53.7% incorrectas => Pregunta regular

8. En la uremia pre-renal:

El sodio en orina es superior a 60 mEq/l, la osmolaridad urinaria es superior a 500 mOsm/ kg H2O y la
relación urea en orina/urea en plasma es superior a 8.
El sodio en la orina es inferior a 20, la osmolaridad urinaria es inferior a 200 mOsm/ kg H2O y la
relación entre urea en orina / urea es plama es inferior a 2.
El sodio en orina es inferior a 20 mEq/l, la osmolaridad en orina es inferior a 200 mOsm/ Kg H2O
y la relación urea en orina / urea en plama es superior a 8.
El sodio en orina es inferior a 20 mEq/l, la osmolaridad urinaria es superior a 500 mOsm/ Kg
H2O, y la relación entre la urea en orina y la urea en plasma es superior a 8.
El sodio en orina es superior a 60, la osmolaridad urinaria es superior a 500 mOsm/Kg H2O, y la
relación urea en orina/urea en plasma es superior a 8.
67 respuestas: 43.3% correctas/ 56.7% incorrectas => Pregunta regular
9. Señale la respuesta correcta referida a las alteraciones del examen de la orina:
La presencia de cilindros hialinos es siempre patológica.
Los cilindros granulosos contienen albúmina e Inmnoglobulinas.
Los cilindros leucocitarios son típicos de Glomerulonefritis postestreptocócica.
Los cilindros hemáticos se presentan en cualquier discrasia sanguínea.
Un resultado negativo de presencia de nitritos en tira reactiva, excluye la existencia de baterias.
71 respuestas: 49.3% correctas/ 50.7% incorrectas => Pregunta regular

11. En un paciente con síndrome nefrótico, las medidas generales no específicas, para corregir la
proteinuria incluye uno de los siguientes procedimientos:

Dieta hiperproteica.
Diuréticos y/o ?-bloqueantes.
Inhibidores del enzima conversor de la Angiotensina (IECAs).
Calcioantagonistas no dihidropiridínicos.
Corticoides por vía sistémica.
79 respuestas: 53.2% correctas/ 46.8% incorrectas => Pregunta regular

12. Paciente de 68 años diabética, que consulta por malestar general. En la gasometría venosa
destaca pH 7.25, Bicarbonato 15 mmol/ l (normal 24-28 mmol/l). Hiato anionico (anión GAP): 11
mmol/l (normal 10-12 mmol/l). ¿Cúal de las siguientes entidades NO descartaría como diagnóstico?:

Cetoacidosis diabética.
Insuficiencia renal crónica.
Acidosis tubular renal.
Ingesta de salicilatos.
Acidosis láctica.
52 respuestas: 40.4% correctas/ 59.6% incorrectas => Pregunta regular

13. Un paciente de 48 años, con antecedentes de hepatitis C (anti VHC +), presenta edemas
maleolares, proteinuria 3,5 g/día, creatinina sérica 1,6 mg/dL y microhematuria en el sedimento.
¿Qué proceso glomerular de los siguientes se encontrará con más probabilidad en la biopsia renal?:

Cambios mínimos.
Glomeruloesclerosis focal y esclerosante.
Glomerulonefritis extracapilar.
Glomerulonefritis membranosa.
Glomerulonefritis membranoproliferativa.
86 respuestas: 34.9% correctas/ 65.1% incorrectas => Pregunta regular

14. Una mujer de 68 años acude al Servicio de Urgencias por malestar general que ha ido
progresando en los últimos 15 días, a partir de un episodio gripal. Ha notado disminución progresiva
del volumen de diuresis, edemas maleolares y dificultad respiratoria. Es hipertensa. En la analítica
destaca una creatinina plamática de 5 mg/dl, urea 180 mg/dl, Na 138 mEq/l, K 4.9 mEq/l. Las cifras
de complemento son normales. Los anticuerpos anti-membrana basal son negativos. En la orina
presenta cilindros hemáticos, proteinuria de 1 g/l y microhematuria. Aporta una analítica de un mes
antes, sin alteraciones. ¿Cuál de los siguientes diagnósticos es más probable?:

PAN microscópica.

9. Señale la respuesta correcta referida a las alteraciones del examen de la orina:


La presencia de cilindros hialinos es siempre patológica.
Los cilindros granulosos contienen albúmina e Inmnoglobulinas.
Los cilindros leucocitarios son típicos de Glomerulonefritis postestreptocócica.
Los cilindros hemáticos se presentan en cualquier discrasia sanguínea.
Un resultado negativo de presencia de nitritos en tira reactiva, excluye la existencia de baterias.
71 respuestas: 49.3% correctas/ 50.7% incorrectas => Pregunta regular

1. Un paciente asintomático, hipertenso de 65 años en tratamiento farmacológico, acude a su médico


que le encuentra en la analítica los siguientes parámetros: Hb 14,1 g/dl, VCM 88 fl, Urea 75 mg/dl,
Creatinina 1,4 mg/dl, Sodio sérico 128 mEq/l, Potasio sérico 2,8 mEq/l, Cloro 89 mEq/l. Lo más
probable sería:

Que tenga una anomalía en la absorción de cloro, tipo Bartter.


Que la hipertensión sea secundaria a una tubolopatía perdedora de potasio.
Que en el tratamiento que reciba exista un diurético de asa.
Que en el tratamiento se incluya un inhibidor de la enzima convertidora de la angiotensina.
Que no tome fruta en la dieta.

2. Un paciente de 35 años con insuficiencia renal crónica secundaria a pielonefritis crónica recibe un
trasplante renal de cadáver con el que compartía dos identidades en A y B y una en DR. Recibe
tratamiento inmunosupresor con ciclosporina A y corticoides a dosis estándar. En el postoperatorio
inmediato se observa buena diuresis y no es necesario el tratamiento sustitutivo con hemodiálisis. En
el 5º día de evolución, el paciente presenta fiebre de 38ª, TA de 180/110, oliguria y disminución en la
concentración urinaria de sodio. El diagnóstico más probable sería:

Crisis hipertensiva.
Infección respiratoria.
Pielonefritis aguda del injerto renal.
Recidiva de su enfermedad renal.
Rechazo agudo del injerto renal.
80 respuestas: 67.5% correctas/ 32.5% incorrectas => Pregunta fácil

3. ¿A cuál de los siguientes factores NO se asocia la Pielonefritis Aguda por Pseudomonas


Aeuruginosa?:

Embarazo.
Sonda urinaria.
Nefrolitiasis.
Manipulación urológica.
Estenosis de la vía urinaria.
76 respuestas: 75.0% correctas/ 25.0% incorrectas => Pregunta fácil

6. Una paciente de 15 años de edad consulta por poliuria y nicturia, cansancio fácil y astenia.
Presenta dichos síntomas desde hace años, y tienden a intensificarse durante los veranos muy
calurosos, en los que se asocia hormigueo lingual y peribucal. A la exploración física sólo destaca una
tensión de 100/50 mmHg. El ionograma muestra Na 135 mEq/l, K 2 mEq/l, Cl 105 mEq/l, pH 7.45,
bicarbonato 30 mEq/l. La determinación de renina y aldosterona muestra cifras elevadas, tanto
basales como tras estímulo. De los procesos que siguen ¿cuál es compatible con el cuadro clínico
descrito?:

Hipoaldosteronismo primario.
Estenosis de la arteria renal.
Síndrome de Liddle.
Enfermedad de Addison.
Síndrome de Bartter.
84 respuestas: 31.0% correctas/ 69.0% incorrectas => Pregunta difícil

8. Un hombre, adicto a drogas por vía parenteral, está ingresado por endocarditis infecciosa.
Durante su enfermedad presenta un cuadro de glomerulonefritis aguda. ¿Cuál de las respuestas es
INCORRECTA?:

Suele ser debida a inmunocomplejos.


No suele presentar piuria.
El complemento está descendido.
A veces produce síndrome nefrótico.
Suele evolucionar favorablemente al controlar la infección cardiaca.
69 respuestas: 42.0% correctas/ 58.0% incorrectas => Pregunta regular

9. En la poliquistosis renal del adulto, ¿cuál de las siguientes afirmaciones es FALSA?:

Es una enfermedad hereditaria, autosómica dominante.


Habitualmente se detecta en la primera infancia con ecografía.
Es causa de deterioro progresivo de la función renal.
Frecuentemente cursa con hipertensión arterial.
Se le asocia litiasis renal en un 15-20% de los casos.
75 respuestas: 62.7% correctas/ 37.3% incorrectas => Pregunta regular

10. Señalar la respuesta correcta en relación con la fisiopatología del calcio en la insuficiencia renal
crónica:

La hormona paratiroidea se eleva precozmente y de forma progresiva.


Mecanismos compensadores consiguen mantener a la hormona paratiroidea dentro de límites
normales hasta estadíos avanzados de la insuficiencia renal.
Disminuye la síntesis de 25-hidroxi-vitamina D.
El calcio sanguíneo aumenta paralelamente a la disminución del filtrado glomerular.
Aumenta la síntesis de 1,25-dihidroxi-vitamina D.
66 respuestas: 24.2% correctas/ 75.8% incorrectas => Pregunta difícil

11. La actitud más correcta en el caso planteado en la pregunta anterior, debe ser:

Retirar el enalapril, indicar esteroides y controlar la evolución de la función renal.


Sustituir el enalapril por otro hipotensor de distinto mecanismo de acción.
Iniciar tratamiento dialítico.
Sustituir el enalapril por ramipril.
Disminuir la dosis de enalapril.
71 respuestas: 47.9% correctas/ 52.1% incorrectas => Pregunta regular

12. Una mujer de 68 años acude al Servicio de Urgencias por malestar general que ha ido
progresando en los últimos 15 días, a partir de un episodio gripal. Ha notado disminución progresiva
del volumen de diuresis, edemas maleolares y dificultad respiratoria. Es hipertensa. En la analítica
destaca una creatinina plamática de 5 mg/dl, urea 180 mg/dl, Na 138 mEq/l, K 4.9 mEq/l. Las cifras
de complemento son normales. Los anticuerpos anti-membrana basal son negativos. En la orina
presenta cilindros hemáticos, proteinuria de 1 g/l y microhematuria. Aporta una analítica de un mes
antes, sin alteraciones. ¿Cuál de los siguientes diagnósticos es más probable?:
PAN microscópica.
Brote lúpico.

14. Un varón de 45 años llega comatoso a Urgencias. Presenta PaCO2 basal 25 mmHg, pH 7,15,
hipocalcemia moderada con Gap aniónico y osmolar elevados, leucocitosis y cristaluria. Con más
probabilidad este paciente tiene una intoxicación por:

Barbitúricos.
Monóxido de carbono.
Etilenglicol.
Benzodiacepinas.
Salicilatos.
71 respuestas: 47.9% correctas/ 52.1% incorrectas => Pregunta regular

. Cuando un paciente bajo tratamiento diurético con tiazidas o furosemida incumple la dieta y come
más sal de la prescrita, el resultado análitico esperable es:

Mayor hipernatremia.
Mayor hiponatremia.
Mayor hiperpotasemia.
Mayor hipopotasemia.
Mayor acidosis.
57 respuestas: 33.3% correctas/ 66.7% incorrectas => Pregunta regular
20. ¿En cuál de las siguientes glomerulonefritis hay activación del complemento por la vía
alternativa?:

Aguda secundaria a endocarditis.


Aguda postestreptocócica.
Difusa lúpica.
Por crioglobulinemia.
Membranosa.
56 respuestas: 51.8% correctas/ 48.2% incorrectas => Pregunta regular

21. En un enfermo con un síndrome de Goodpasture que le ha conducido a una insuficiencia renal
crónica:

No debe realizarse un trasplante renal, pues la enfermedad le dañará el riñón trasplantado.


El trasplante puede realizarse si la producción de anticuerpos anti-membrana basal ha cesado.
Debe seguir siendo tratado indefinidamente con inmunosupresores para proteger su pulmón.
Debe seguir siendo tratado indefinidamente con esteroides para proteger su pulmón.
La función renal se recuperará cuando dejen de producirse autoanticuerpos anti-membrana basal.
63 respuestas: 46.0% correctas/ 54.0% incorrectas => Pregunta regular

22. En relación a la hemoglobinuria paroxística nocturna, señale cuál de las siguientes afirmaciones
es FALSA:

Es un defecto adquirido.
Falta la molécula de anclaje glicosilfosfoinositol en el membrana.
Cursa con un aumento de la resistencia del hematíe a la lisis por el complemento.
Se acompaña de un aumento de frecuencia de trombosis venosas.
Presenta a menudo leucopenia y trombopenia.
71 respuestas: 53.5% correctas/ 46.5% incorrectas => Pregunta regular
23. Con respecto a la nefropatía diabética, señalar la respuesta FALSA:

La diabetes tipo 2 es la etiología más frecuente de insuficiencia renal terminal en el mundo occidental.
Más del 90% de los diabéticos tipo 1 desarrollan nefropatía a los 30 años del diagnóstico de
diabetes.
La alteración renal más temprana es la hiperfiltración.
La existencia de microalbuminuria predice el desarrollo de nefropatía clínica.
La gran mayoría de los diabéticos tipo 1 con nefropatía tienen también retinopatía.
66 respuestas: 34.8% correctas/ 65.2% incorrectas => Pregunta regular

26. En el tratamiento de la hiperpotasemia grave de un paciente urémico NO está indicado


administrar:

Resinas de intercambio catiónico (sodio, calcio).


Bicarbonato sódico i.v.
Glucosa con insulina i.v.
Gluconato cálcico o cloruro cálcico i.v.
Tiacidas por vía oral.
67 respuestas: 56.7% correctas/ 43.3% incorrectas => Pregunta regular

. Un niño de 4 años de edad muestra un importante retraso de crecimiento, lesiones de raquitismo


resistentes al tratamiento con dosis habituales de vitamina D y poliuria. ¿Cuál de las siguientes
asociaciones considera que permite el diagnóstico de síndrome de Fanconi?:

Glucosuria + hiperaminoaciduria + alcalosis + hiperfosforemia.


Glucosuria + hipoglucemia + acidosis metabólica + hipofosforemia.
Glucosuria + hiperaminoaciduria + acidosis metabólica + hipofosforemia.
Glucosuria + hiperaminoaciduria + alcalosis metabólica + hipofosforemia.
Hipoglucemia + hiperaminoaciduria + alcalosis metabólica + hipofosforemia.
56 respuestas: 53.6% correctas/ 46.4% incorrectas => Pregunta regular

32. En el caso de un paciente con hiperplasia prostática benigna, ¿cuál, entre las siguientes
circunstancias, NO establece por sí misma indicación de cirugía?:

Hematuria severa recurrente.


Retención urinaria que requiere sondaje vesical permanente.
Infecciones de orina de repetición.
Nicturia de dos veces.
Residuo postmiccional mayor de 200 cc.
76 respuestas: 76.3% correctas/ 23.7% incorrectas => Pregunta fácil
33. De las siguientes afirmaciones respecto a la enfermedad renal poliquística del adulto, ¿cuál es la
correcta?:

Es más prevalente en mujeres, por el uso de anovulatorios.


Se puede acompañar de quistes hepáticos.
Sólo desarrolla hipertensión arterial si reciben antiinflamatorios.
Rara vez evolucionan a insuficiencia renal crónica, excepto en presencia de hipertensión arterial.
La cirugía precoz sobre los quistes es fundamental.
69 respuestas: 44.9% correctas/ 55.1% incorrectas => Pregunta regular

35. Paciente de 70 años, que hace 2 semanas fue sometido a una coronariografía, acude al hospital
por aparición de lesiones purpúricas palpables en miembros inferiores, elevación de la creatinina
sérica a 3 mg/dl, proteinuria de 1g/24h, hipocomplementemia y microhematuria y leucocituria en el
sedimento urinario. ¿Cuál es el diagnóstico más probable?:

Glomerulonefritis aguda rápidamente progresiva.


Glomerulonefritis aguda postestreptocócica.
Síndrome hemolítico-urémico.
Enfermedad atero-embólica.
Glomerulonefritis membrano-proliferativa.
80 respuestas: 33.8% correctas/ 66.3% incorrectas => Pregunta regular

36. ¿De cuál de los siguientes cuadros es característica la aparición de cilindros hemáticos en el
sedimento urinario?:

Necrosis tubular aguda.


Lesión a cualquier nivel de las vías urinarias.
Daño glomerular severo.
Daño tubular.
Cualquier lesión de la nefrona.
57 respuestas: 54.4% correctas/ 45.6% incorrectas => Pregunta regular

37. Paciente de 63 años, fumador importante, que refiere hematuria total intermitente y síndrome
miccional irritativo desde hace 1 mes. La citología de orina es positiva. Se realiza una evaluación
vesical bajo anestesis con biopsias vesicales múltiples, siendo diagnosticado un carcinoma vesical “in
situ”. ¿Cuál de las siguientes estrategias terapéuticas es la más apropiada?:

Instilaciones endovesicales con Mitomicina C.


Instilaciones endovesicales con BCG.
Cistectomía radical.
Quimioterapia sistémica.
Radioterapia pelviana.
52 respuestas: 51.9% correctas/ 48.1% incorrectas => Pregunta regular

39. Ante una sospecha de enfermedad renal poliquística del adulto, el paso diagnóstico siguiente más
razonable, ente los que se señalan, es:

Tomografía.
Urografía intravenosa.
Ecografía.
Tomografía axial (TAC).
Angiografía.
67 respuestas: 61.2% correctas/ 38.8% incorrectas => Pregunta regular
41. ¿Cuál de las siguientes nefropatías glomerulares NO se relaciona con hipocomplemetemia?:

Glomerulonefritis aguda post-infecciosa.


Glomerulonefritis membrano-proliferativa.
Nefropatía lúpica.
Nefropatía diabética.
Nefropatía de la crioglobulinemia.
89 respuestas: 60.7% correctas/ 39.3% incorrectas => Pregunta regular

43. ¿Cuál de las siguientes medidas terapéuticas NO está indicada en el tratamiento de la


Hiperpotasemia?:

Glucosa e insulina.
Bicarbonato sódico.
Gluconato cálcico.
Resinas de intercambio iónico.
Glucocorticoides.
74 respuestas: 60.8% correctas/ 39.2% incorrectas =>

175. Un paciente diabético conocido, de 2 años de evolución y 64 años de edad, consulta por anemia,
proteinuria de 3 gramos/24 horas junto con hematuria, hipertensión arterial y discreta insuficiencia renal con
creatinina plasmática de 2.3 mg/dl. El diagnóstico MENOS probable es:
1) Nefropatía diabética.
2) Angeítis necrotizante del tipo poliangeítis microscópica.
3) Glomerulonefritis rápidamente progresiva.
4) Granulomatosis de Wegener.
5) Síndrome úremico-hemolítico.

177. Si un paciente con una pérdida neuronal progresiva, la presencia de hiperpotasemia indica que ha
perdido al menos:
1) Un 25% del filtrado glomerular.
2) Un 35% del filtrado glomerular.
3) Un 50% del filtrado glomerular.
4) Un 75% del filtrado glomerular.
5) Un 100% del filtrado glomerular.

178. Un paciente de 35 años con insuficiencia renal crónica secundaria a pielonefritis crónica recibe un
trasplante renal de cadáver con el que compartía dos identidades en Ay B y una en DR. Recibe tratamiento
inmunosupresor con ciclosporina A y corticoides a dosis estándar. En el postoperatorio inmediato se
observa buena diuresis y no es necesario el tratamiento sustitutivo con hemodiálisis. En el 5º día de
evolución, el paciente presenta fiebre de 38º. TA de 180/ 110, oliguria y disminución en la concentración
urinaria de sodio. El diagnóstico más probable sería:
1) Crisis hipertensiva.
2) Infección respiratoria.
3) Pielonefritis aguda del injerto renal.
4) Recidiva de su enfermedad renal.
5) Rechazo agudo del injerto renal.

182. Una paciente de 15 años de edad consulta por poliuria y nicturia, cansancio fácil y astenia. Presenta
dichos síntomas desde hace años, y tienden a intensificarse durante los veranos muy calurosos, en los que se
asocia hormigueo lingual y peribucal. A la exploración física sólo destaca una tensión de 100/ 50 mmHg. El
ionograma muestra Na 135 mEq/l, K 2 mEq/l. CI 105 mEq/l, pH 7.45, bicarbonato 30 mEq/l. La
determinación de renina y aldosterona muestra cifras elevadas, tanto basales como tras estímulo. De los
procesos que siguen ¿cuál es compatible con el cuadro clínico descrito?:
1) Hipoaldosteronismo primario.
2) Estenosis de la arterial renal.
3) Síndrome de Liddle.
4) Enfermedad de Addison.
5) Síndrome de Bartter.

183. ¿Cuál de las siguientes medidas terapéuticas NO está indicada en el tratamiento de la Hiperpotasemia?:
1) Glucosa e insulina.
2) Bicarbonato sódico.
3) Gluconato cálcico.
4) Resinas de intercambio iónico.
5) Glucocorticoides.
184. ¿Cuál de las siguientes nefropatías glomerulares no se relaciona con hipocomplementemia?:
1) Glomerulonefritis aguda postinfecciosa.
2) Glomerulonefritis membrano-proliferativa.
3) Nefropatía lúpica.
4) Nefropatía diabética.
5) Nefropatía de la crioglobulinemia.

185. Un paciente de 28 años presenta hematuria macroscópica al día siguiente de una infección faríngea y
edemas maleolares. Dos años antes había presentado un cuadro similar que desapareció con rapidez, por lo
que no había consultado previamente. En la exploración física se objetiva una T.A. de 180/110, una
Creatinina Plasmática de 2 mgrs/dl e Hipocomplementemia. ¿Cuál de los siguientes es el diagnóstico más
probable?:
1) Glomerulonefritis membrano-proliferativa.
2) Glomerulonefritis post-estreptocócica aguda.
3) Glomerulonefritis rápidamente progresiva.
4) Glomerulonefritis por depósitos mesangiales de IgA (Enfermedad de Berger).
5) Hialinosis Segmentaria y Focal.

193. Niña de 3 años, que presenta afectación brusca del estado general dentro del contexto de un cuadro
febril con diarrea mucosanguinolenta. A la exploración aparece pálida y soñolienta, la auscultación
cardiopulmonar es normal, el abdomen doloroso y el resto del examen físico sólo revela la presencia del
petequias puntiformes diseminadas. La orina es hematúrica y se constata hipertensión arterial. El
hemograma muestra HB 7 g/dl, 17.000 leucocitos con neutrofilia, 37.000 plaquetas con normalidad de las
pruebas de coagulación. ¿Cuál es el diagnóstico más probable?:
1) Sepsis por Salmonella.
2) Púrpura de Schönlein-Henoch.
3) Glomerulonefritis aguda post-infecciosa.
4) Síndrome hemolítico-urémico.
5) Coagulación intravascular diseminada.

194. Se trata de un niño de 7 años, con retraso estaturo-ponderal, que presenta signos radiológicos de
raquitismo y ecográficos de nefrocalcinosis. Los análisis demuestran acidosis metabólica con PH inferior a
7.32 y Bicarbonato plasmático inferior a 17 mEq/ litro. El PH de la orina en 3 determinaciones es siempre
superior a 5.5. Otras alteraciones metabólicas consisten en hipercalciuria, hipocitruria y discreta
disminución de la resorción del fósforo. No se detecta glucosuria ni aminoaciduria. De las siguientes
posibilidades, señale el diagnóstico correcto:
1) Síndrome de Bartter.
2) Acidosis tubular distal (Tipo I).
3) Acidosis tubular proximal (Tipo II).
4) Síndrome completo de Fanconi.
5) Enfermedad de Harnup.

174. Paciente de 50 años de edad, que consulta por dolor en la fosa renal, polaquiuria, disuria y hematuria.
En el análisis de la orina se observa piuria y pH ácido con cultivos repetidamente negativos. ¿Cuál sería la
primera posibilidad diagnóstica, de entre las siguientes?:
1) Pielonefritis aguda.
2) Síndrome nefrítico.
3) Tuberculosis genitourinaria.
4) Prostatitis aguda.
5) Carcinoma renal de células claras.

134. ¿En qué tipo de pacientes está indicado realizar detección y tratamiento de bacteriuria asintomática?:
1) Pacientes con sondaje vesical permanente.
2) Gestantes.
3) Pacientes mayores de 65 años.
4) Pacientes ingresados en el hospital.
5) Paciente prostatectomizados.
Pregunta 175 R: 1
Pregunta de nivel medio de neuropatía diabética.
Presenta el caso de un paciente con DM relativamente reciente
en IR con hematuria y proteinuria en rango nefrótico. En principio el
cuadro que presenta es poco específico y sería compatible con cualquiera
de las cinco opciones, ya que todas ellas pueden dar IR por
lesión glomerular. El dato determinante es que la DM sólo lleve dos
años de evolución. La nefropatía diabética se suele diagnosticar a los
10-15 años de la aparición de la DM cuando ya existen neuropatía
y retinopatía asociadas, por lo que es poco probable que se trate de
esta patología.
Pregunta 175. Diabetes Mellitus. Gloméruloesclerosis.
Bibliografía: Manual CTO de Nefrología, 4ªEdición, pág. 27-28.
Pregunta 177 R: 4
Pregunta difícil de IRC, que implica conocer un dato muy concreto,
aunque se basa en un concepto general a cerca de la reserva funcional
del riñón.
El riñón tiene una importante reserva funcional y y por ello las
enfermedades que cursan con IRC inicialmente se manifiestan a través
de otro síndrome (sd. nefrótico, sd. nefrítico, alteraciones del
sedimento, …) o por sus expresiones sistémicas (diabetes mellitus,
hipertensión crónica, …). Cuando el filtrado glomerular disminuye
al 35-50% aumenta la creatinina en plasma, pero en general el paciente
sigue asintomático. Al llegar al 25-35% aparecen la hiperazoemia
y las manifestaciones iniciales. Con un filtrado glomerular menor
al 25% se produce insuficiencia renal manifiesta con hiperpotasemia,
hiperfosforemia e hipocalcemia. La opción 4 es por tanto la
correcta.
Bibliografía: Manual CTO de Nefrología, 4ª Edición, pág. 9-10.
Pág. 6
COMENTARIOS MIR 15 Febrero 2003
Pregunta 178 R: 5
Pregunta de dificultad media sobre el transplante renal, un tema poco
preguntado en el MIR. Con la picardía de haber hecho muchos desgloses,
podemos darnos cuenta de que en el MIR cuando hay problemas
en un transplantado, suelen preguntar por una infección por citomegalovirus
o por un rechazo del injerto, con lo que acertaríamos la pregunta
(respuesta correcta la 5).
Si bien, pensando la pregunta, podemos descartar la respuesta 1
ya que la HTA que aparece es secundaria al fallo renal, y no causa de
éste. La respuesta 2 no parece guardar ninguna relación con la pregunta,
ya que se trata de una afectación renal y no respiratoria. La
respuesta 4 la descartamos ya que la enfermedad renal previa era
crónica, no de carácter tan agudo e intenso. Por tanto podríamos
dudar entre la 3 y la 5, y parece mucho mas lógico que sea un
rechazo del transplante, ya que está en oliguria brusca asociada a
déficit de concentración de sodio (afectación tubulointersticial en la
forma celular del rechazo agudo).
Bibliografía: Manual CTO de Urología, 4ª Edición, pág. 22.
Pregunta 182 R: 5
Es un caso clínico sobre uno de los temas más complejos en nefrología,
los trastornos tubulares hereditarios, que son bastante preguntados.
Se trata de una adolescente con un cuadro de años de evolución
con poliuria y nicturia que sugiere una afectación tubulointersticial.
Esto ya sitúa la pregunta en el tema de trastornos tubulointersticiales
del riñón, concretamente en los síndromes congénitos. Por tanto se
puede descartar la opción 2, que además suele cursar con hipertensión
arterial vasculorrenal. Las opciones 1 y 4 cursan con un déficit
de mineralocorticoides con hiperpotasemia y acidosis, mientras que
aquí la aldosterona está elevada y hay hipopotasemia (que empeora
con el calor por pérdidas extrarrenales añadidas) y alcalosis. Queda
la duda entre el síndrome de Liddle y el de Bartter. Ambos cursan con
alcalosis hipopotasémica, en el caso del primero por hiperfunción
del transportador distal de Na+ sensible a aldosterona e intercambio
del sodio retenido con K+ y H+, mientras que en el Bartter es por fallo
del transportador Na+/2Cl-/K+ de la rama ascendente del asa de Henle.
La retención de Na+ en el Liddle produce hipertensión arterial por
expansión de volumen y supresión del eje renina-angiotensina aldosterona.
Por tanto la opción correcta es la 5.
Pregunta 182. Trastornos tubulares hereditarios.
Bibliografía: Manual de Nefrología, 4ª Edición, pág. 32.
Pregunta 183 R: 5
Pregunta fácil y directa sobre hiperpotasemia.
La hiperpotasemia es un cuadro potencialmente mortal por su
efecto arritmogénico. Para su tratamiento en fase aguda, podemos
neutralizar el efecto cardíaco con cloruro o gluconato cálcico (respuesta
3), forzar la captación intracelular de potasio con agonistas b2
adrenérgicos, insulina (+ suero glucosado) o bicarbonato (respuestas
1 y 2), o estimular las pérdidas con diálisis o resinas de intercambio
Pág. 7
COMENTARIOS MIR 15 Febrero 2003
iónico (respuesta 4). Para el tratamiento crónico, hacemos restricción
dietética, damos resinas de intercambio, y usamos diuréticos
(de asa o tiazidas). Los glucocorticoides no tienen lugar en el tratamiento
de la hiperpotasemia (respuesta 5 falsa) sino en el de la hipercalcemia;
pueden ser útiles los mineralcorticoides en caso de hipoaldosteronismo.
El tratamiento de la hiperpotasemia aguda se escalona
en función de los niveles y de las alteraciones del ECG: con ECG
normal y <6,5, restricción y resinas; con T picudas o 6,5-7,5, además
insulina, bicarbonato o b2 adrenérgicos; con nivel >7,5 o alteraciones
ECG graves, además gluconato cálcico.
Bibliografía: Manual CTO de Fisiología, 4ª Edición, pág. 13.
Pregunta 184 R: 4
Es una pregunta muy sencilla que se repite casi todos los años y que
nadie debería fallar.
En el tema de glomerulonefritis existe una tabla que resume aquellas
enfermedades glomerulares en las que se activa la vía clásica o
alternativa del complemento por la presencia de inmunocomplejos
o antígenos extraños de la superficie de patógenos. La diabetes mellitus
no lesiona los glomérulos mediante mecanismos inmunológicos
sino por la hiperfiltración glomerular secundaria a la microangiopatía
de las arteriolas aferente y eferente, que causa glomeruloesclerosis.
Por eso la neuropatía diabética cursa sin hipocomplementemia.
Pregunta 184. Glomerulonefritis con descenso del complemento.
• GN aguda postinfecciosa (GN endocapilar difusa).
• GN membranoproliferativa (o mesangiocapilar) tipo I (baja C3 y C4) y
tipo II (baja C3) o de depósitos densos.
• GN extracapilar tipo II.
• GN del lupus.
• GN del shunt.
• GN de la sepsis.
• GN de la endocarditis infecciosa.
• GN de la crioglobulinemia.
Bibliografía: Manual de Nefrología de CTO, 4ª Edición, pág. 18.
Pregunta 185 R: 1
Caso clínico relativamente complejo sobre las glomerulonefritis.
Existen tres tipos de glomerulonefritis que pueden asociarse a infecciones
respiratorias altas y cursar con hematuria, serían las opciones 1,
2 y 4. La glomerulonefritis aguda postestreptocócica es más frecuente
en niños y clásicamente presenta un periodo de latencia de 1-3 semanas
desde el inicio de la infección hasta que aparece la clínica renal,
generalmente en forma de síndrome nefrítico y con microhematuria
que persiste hasta 6 meses. Esto hace que se pueda descartar la opción
2. La enfermedad de Berger no tiene periodo de latencia y es típica la
macrohematuria de corta duración, pero no presenta hipocomplementemia,
por lo que podemos descartarla también. La opción correcta
es la glomerulonefritis membranoproliferativa.
Pregunta 185. Glomerulonefritis y complemento
Bibliografía: Manual CTO de Nefrología, 4ª Edición, pág. 21.
Pregunta 193 R: 4
Se trata de una pregunta de dificultad alta sobre un tema que puede
aparecer tanto en pediatría como en nefrología o infecciosas.
El síndrome hemolítico urémico desde el punto de vista pediátrico
es la causa más frecuente de insuficiencia renal aguda en niños
menores de 4 años. Desde el punto de vista infeccioso destaca que
Pág. 9
COMENTARIOS MIR 15 Febrero 2003
casi siempre va precedido por una gastroenteritis enteroinvasiva por
E.coli (aunque también se ha relacionado con Shigella, Salmonella y
Campylobacter). Desde el punto de vista clínico la tríada típica: anemia
hemolítica con esquistocitos, trombopenia con pruebas de coagulación
normales e insuficiencia renal.
El principal diagnóstico diferencial se establece con la trombosis
de la vena renal que cursa con HTA+hematuria+trombopenia.
El tratamiento médico de las manifestaciones hematológicas, junto
con una diálisis peritoneal precoz, es el que mejores expectativas
de recuperación renal ofrece, de tal manera que la función renal se
recupera en el 90% de los casos y es una enfermedad que no
recidiva.
De esta manera las pistas que podemos encontrar en este caso
clínico son:
- Cuadro febril con diarrea mucosanguinolenta.
- Orina hematúrica
- Anemia
- Trombopenia con normalidad de las pruebas de coagulación.
Podemos descartar otras opciones como:
- Púrpura de Schönlein-Henoch: la simple presencia de trombopenia
descarta esta opción porque es una púrpura no trombopénica.
Su manifestación cutánea no son las petequias sino la púrpura
palpable. Además suele asociar glomerulonefritis, artralgias y
dolor abdominal.
- Glomerulonefritis aguda post-infecciosa: suele aparecer en relación
con infección faríngea o cutánea por Estreptococo del grupo
A tras un periodo de latencia, no coincidiendo con la infección.
Desde el punto de vista renal cursa en forma de síndrome
nefrítico (hematuria, HTA y oliguria.)
Bibliografía: Manual CTO de Pediatría, 4ª Edición, pág. 47.
Pregunta 194 R: 2
Un caso clínico sobre las acidosis tubulares renales que son un tema
complejo en nefrología, pero muy preguntable.
Pregunta 194. Tipos de acidosis tubular renal.
Presenta el caso de un niño con acidosis metabólica, nefrocalcinosis
y enfermedad ósea por raquitismo. Estos datos en el MIR siempre
orientan hacia una acidosis tubular renal tipo I o distal. Pero
además hay tres opciones que se descartan con facilidad, el síndrome
de Bartter porque al igual que el Gitelman y el Liddle cursa con
alcalosis, el síndrome de Fanconi porque tiene un fallo global del
túbulo renal que incluye glucosuria y aminoaciduria y la enfermedad
de Hartnup porque es una alteración del transporte renal e
intestinal de determinados aminoácidos y por tanto tiene aminoaciduria.
La duda es entre la acidosis tubular renal distal y proximal. La
diferencia clínica es que la proximal suele tener una hipercalciuria
moderada que no produce nefrocalcinosis ni raquitismo. Para hacer
el diagnóstico diferencial habría que solicitar una sobrecarga oral
con cloruro amónico que acidifica la orina en la acidosis tubular
proximal, pero no en la distal.
Bibliografía: Manual CTO de Nefrología, 4ª Edición, pág. 32-33.
Pregunta 174 R: 3
Esta pregunta es muy frecuente en el MIR y no podemos fallarla, ya que
ha caído muchos años el mismo concepto.
Una Piuria ácida y estéril en el MIR es una tuberculosis genitourinaria
como primera posibilidad y hasta que se demuestre lo contrario.
Bibliografía: Manual CTO de Urología, 4ª Edición, pág. 5.
Pregunta 134 R: 2
Pregunta clásica en el MIR, que no debemos fallar. Se trata de la actitud
ante una bacteriuria asintomática.
Las indicaciones de tratamiento de la bacteriuria asintomática
son: embarazadas (respuesta 2 correcta), inmunodeprimidos, bacteriuria
por Proteus y previa a manipulaciones urológicas como profilaxis.
Bibliografía: Manual CTO de Urología, 4ª Edición, pág. 4.

Cuál de los siguientes no es un efecto secundario de los antiácidos?


a. estreñimiento
b. diarrea
c. hipercalcemia
d. hiperfosfatemia
e. retención de sodio

Urémico crónico con potasio sérico de 6.5mEq/L con leve academia, sin trastorno
muscular y EKG normal, puede ser tratado con:
i. kayaxelate
j. gluconato de Calcio al 10%
k. NaHCO3
l. Diálisis

La insuficiencia renal crónica ocasiona hiperfosfatemia permanentemente cuando la tasa


de filtración glomerular esta por debajo de:
i. 60ml/min
j. 50 ml/min
k. 25 ml/min
l. 40 ml/min

La hipofosfatemia es causa de excepto


i. inotropismo positivo
j. disminución de 2,3 DPG
k. hemólisis
l. curva de HbO2 desplazada hacia la izquierda

La solución de NaCl 23,4% tiene de osmolaridad:


i. 8000 mosm/l
j. 800 mosm/l
k. 80 mosm/l
l. 0,8 mosm/l

En la hiperkalemia:
i. aumenta la sístole eléctrica
j. hay menor diferencia entre el potencial de reposo y el umbral
k. el potencial de reposo transmembrana se hace mas negativo
l. el potencial umbral se hace menos negativo

La velocidad de administración de potasio en caso de gran riesgo para la vida puede ser
de:
i. 10-20 mEq/L
j. 8 mEq/L
k. 110 mEq/L
l. 40 mEq/L

Un ejemplo de hiponatremia hipotónica normovolemica es:


i. síndrome nefrósico
j. SIADH
k. Diarrea
l. Cirrosis

A una embarazada con eclampsia se le ordena 1 litro de D/A 5% con 10g de MgSO4.
¿Cuántos ml de MgSO4 al 50% se han utilizado?
a. 20
b. 5
c. 12,5
d. 30

Causa hipomagnesemia:
i. estreñimiento
j. alcoholismo
k. deshidratación hipertónica
l. insuficiencia renal aguda

Paciente con diuresis de 300ml/día, K sérico 7mEq/L, Posm y Uosm 300mosm/L, el


diagnóstico más probable es:
i. insuficiencia suprarrenal
j. insuficiencia renal
k. SIADH
l. Deshidratación primaria

La mielinosis pontina se caracteriza por:


i. pares craneales intactos
j. cuadriplejía flácida
k. comportamiento normal
l. hipertonía de los miembros inferiores

Calcule el déficit de agua en una paciente con sodio sérico 168mEq/L y que pesa 70Kg
(utilice como porcentaje de agua 50% y sodio normal mEq/L):
i. 5L
j. 3L
k. 10L
l. 7L

En la SIADH la concentración de sodio urinario es de:


i. menos de 10mEq/L
j. 20mEq/L
k. 15mEq/L
l. Mas de 40mEq/L

En paciente con vómitos abundantes y prolongados con hipokalemia severa debe


administrarse:
i. NaCl 0.9% con KHCO3
j. NaCl 0.9% con KCl
k. NaCl 0.9% con gluconato de potasio
l. NaCl 0.9% con NaHCO3

La hipokalemia en un paciente con cirrosis puede contribuir:


i. coma
j. alcalosis respiratoria
k. acidosis metabolica
l. aumento de la síntesis de urea

El plasma de un paciente contiene Na 125mEq/L, glucosa 108mg/dl, urea 300mg/dl,


podrá presentar:
i. cirrosis
j. síndrome nefrósico
k. síntomas de hipertonicidad
l. síntomas de hipotonicidad

La hiponatremia aguda sintomática es debida a, excepto:


i. exceso de glucocorticoides
j. SIADH
k. post-operatorio
l. intoxicación hídrica

La administración de solución salina 0.9% produce:


i. disminución del volumen intracelular y aumento del extracelular
j. aumento del volumen intra y extracelular
k. aumento del volumen extracelular
l. aumento del volumen intracelular

Alcalosis metabólica con hipertensión arterial, hipokalemia, aldosterona normal y renina


normal:
i. Hipertensión renovascular
j. Hipertensión maligna
k. Sindroma de Cushing
l. Administración de licor

La hipokalemia puede predisponer, excepto:


j. intoxicación digitálica
k. disminución de la amoniogénesis
l. rabdomiolisis
m. ondas U en el EKG

Compendio renal

3) Cuál de las siguientes medicinas pueden dar hiperkalemia


k) inhibidor de la enzima convertidota
l) bloqueador alfa adrenérgico
m) bloqueador beta adrenérgico
n) ayb
o) ayc

2) Hormonas que disminuyen en la falla renal crónica, excepto


a) 1,25 dihidroxi vitatamina D
b) eritropoyetina
c) paratohormona
d) foliculo estimulante

3) MgSO4 (PM 120) 1 gramo contiene de magnesio en meq


a) 10
b) 7.9
c) 8.3
d) 6

4) Factores que conducen a la progresión de falla renal crónica


a) hiperlipidemia
b) toxinas urémicas
c) proteinuria
d) b y c
c) todas

5) Paciente de 20 años presenta abundantes vômitos y diarrea de 3 días de evolución,


examen físico depleción de volumen, suero muestra Na 155, K 3meq/l, Cl 117meq/l,
HCO3 25 meq/l, tratamiento de elección:
a) D/A 5% + KCl
b) solución salina 3% + KCl
c) solución salina 23.4% + KCl
d) solución salina 0.45% + KCl

6) En la enfermedad ateroembólica renal se da lo siguiente excepto:


d) Velocidad de eritrosedimentación

7) Pseudohiperkalemia de 8 mEq/l que se ve en el EKG:


Onda T picuda
Aplanamiento de la onda P
Ay B
Ninguna

8)Criterios para iniciar diálisis excepto:


Alcalosiss Metabolica

9)Hipokalemia con normotension excepto


Síndrome de liddle

10) Plasama creatinina 8mg/dl N de urea 100mg/dl U na 40 Meq/l :


Insuficiência renal

11) Factores reversibles responsables de deterioro de la funcion renal excepto:


Fibrosis intersticial

12) Causa hipokalemia con renina y aldosterona disminuida:


Licorice

13. Sugiere trastorno túbulo intersticial:


.Proteinuria mayor a 3.5 g /24 horas
.Notrmotensión
.Edema
.Osmolarida urinaria normal

14. Glomérulonefritis rápidamente progresiva


.Engrosamiento de la membrana basal
Fusión de los pedículos de los podocitos
prolñiferacion intracapilar
proliferación extracapilar ( creo que es esa verificar)

15.Una de las siguientes causas no produce hematuria


.enfermedad de cambios mínimos

16.Plasma Na:115, Posm240 mOsm,Uosm 680,Una 60:meq/l:


.Deshidratación hipotónica
.secreción inapropiada de ADH
.Deshidratación isotónica
.Deshidratación primaria

17.en la falla renal crónca produce necrosis


calcifilaxia

18. No produce hipokalemia:


Pancreatitis aguda
rabdomiolisis
neoplasia
Def. Vit D

19. Paciente con glucosa 90mg/dl, Na+= 115mEq/L, Urea en 140mg/dl


a. Síndrome hipoosmolar
b. Síndrome hiperosmolar
c. Diarrea
d. Ninguna de las anteriores.

20. Causa de seudohiponatremia con osmolaridad elevada


a. Hiperproteinemia
b. Diabetes mellitus
c. Hiperlipidemidemia
d. Reserción transuretral prostática.

21. Manifestaciones de hipermagnesemia excepto


Normotensión

22. Piel en uremia crónica excepto


Paño blanco

23. No es causa de necrosis papilar aguda


a.Diabetes mellitas
b. Uropatía obstructiva
c. Pielonefritis
d. Trombosis de venas renal.

24. Puede presentarse hiperuricemia excepto


d. Adenocarcinoma gástrico

25. Compromete riñones y pulmones:


a. Enf de GoodPasture
b. Granulomatosis de Wegener
c. LES
d. Ay C

26. Sugiere glomerulopatia:


-anasarca

27. TFG < 15ml/min corresponde al estadio de la enfermedad renal cronica:


* 4

28. Causa obstetrica de necrosis cortical bilateral


- desprendimiento prematuro de placenta

29. Promueve amoniogenesis em cirróticos


- hipokalemia

30. Criterio para diagnostico de síndrome nefrosico


a. Proteinuria en 24 hr de 3.5 gr para superficie corporal de 1. 73
b. Albumina serica < 3g/dl
c.Hiperlipidemia y edema
d. Todas las anteriores
e. ay b

1. En la Hiperkalemia:
R/. Hay una menor diferencia entre el potencial de reposo y de umbral

2. La hipokalemia en un cirrotico puede desencadenar:


R/. Coma Hepático

3. La hipermagnesemia puede producir:


R/. Paro respiratorio

7. Tratamiento de hiprecalcemia:
R/. SSN 0.9%

8. La hipofosfatemia son causas, excepto:


a. Aumento de la PTH
b. Aumento de la absorción intestinal de fosfato
c. Disminuye la fosfatemia
d. Aumento de la vitamina D

9. Una solución isotónica:


R/. Aumenta el volumen extracelular

13. La velocidad de administración de potasio en caso de gran riesgo para la vida puede
ser:
a. 10-20 mEq/L
b. 8mE/L
c. 110mE/L
d. 40 mEq/h
14. Paciente crónico con deshidratación severa puede presentar lo siguiente excepto:
a. Densidad urinaria 1010
b. Osmolaridad urinaria de 800 mOsm/kg
c. K sérico aumentado
d. Oliguria

15. Paciente con vómito abundante, hipertensión arterial, sodio urinario de 6, oligúrico,
osmolaridad de 800 presenta:
R/.Insuficiencia suprarrenal
Paciente urémico crónico con vómitos severos, hipotensión arterial, orina al azar muestra
Na mas de 40, osmolaridad de 800 puede tener:
Falla suprarrenal

16. Todo lo siguiente es cierto acerca de Colitis Ulcerosa, EXCEPTO:


R/. No está en relación las alteraciones inmunológicas.

17. En quien predomina la enfermedad de Hígado Graso:


R/. Sexo masculino

18. Paciente con Insuficiencia renal crónica, con K sérico de 6, leve academia, EKG
normal, se trata con:
R/. Kayetzalate

22. La hopikalemia predispone, excepto:


R/. Amoniogénesis

25. Cual de las siguientes es causa de hiperfosfatemia:


R/. Insuficiencia renal

26. Paciente con deshidratación hipertónica e hipertensión:


R/. SSN 0.9%

29. En la Glomerulonefritis Aguda post-infecciosa disminuye la tasa de filtración:


R/. Por alteración en el coeficiente de ultrafiltración

23. Px con IRC tine una taza de filtración glomerular de menor de 25 ml/min ;aunque
hay yuna tabla de las etapas en esta enfemedad crónica.
Estas osn las TFG:
1. 90 en riesgo
2. 20 ( con TFG normal o incrementada)
3. 60 -89
4. 30 – 59
5. 15-29
Menor a 15 diálisis.

24. Factor que altera el balance hídrico en el periódo post- operatorio:


Disminución de la ADH (Excepto)

25. Un ejemplo de hipokalemia con aldosterona alta y osmolaridad baja es:


Hiperaldosteronismo primario

26.Ejemplode hipernatremia hipervolémica: Hiperaldosteronismo primario

27.Hiperkalemioa son alteraciones del EKG no es necesario administrar gluconato de


calcio se colocará un antagonista de K asíe l Px no cae en paro y estoes si tiene
manifestación en el EKG.

28. La fracción de excreción de sodio estará aumentada en falla cardíaca.

29.En la hiperkalemia: hay menos diferencia entre el potencial de reposo y el umbral.

Manifestaciones cardíacas. El problema médico más grave de la hiperkalemia es la


cardiotoxicidad. Los cambios en el ECG producidos por los niveles altos de potasio son
bastante constantes. A medida que aumentan los niveles se aprecian los siguientes
cambios:

Ondas T picudas (con intervalo QT normal o ligeramente reducido)


Prolongación del intervalo PR con depresión de ST
Desaparición progresiva de la onda P
Bloqueo cardíaco progresivo
Arritmias ventriculares
Paro cardíaco

Las ondas T picudas constituyen el dato en el ECG más constante en la hiperkalemia.

Efectos neuromusculares. El primer signo neuromuscular de la hiperkalemia suele ser la


aparición de parestesias seguidas de debilidad progresiva de varios grupos musculares. Si
el cuadro se agrava se observa cuadriplejia fláccida. Las funciones cerebrales y de los
pares craneanos se conservan y la parálisis de la musculatura respiratoria puede ocurrir,
pero es excepcional.

30. La infusión de potasio sin riesgo para la vida del PX: es de 40 meq; porque

I lb de solución salina: debe llevr de 20 a40 mEq de KCl a una velocidad de 10 a 20


mEq por hora; si aumenta la velocidad puede provocar un bloqueo A-V. Se recomienda
una Velocidad mayor si la hipokalemia es muy severa.
Sis e usa venas priféricas es:

10 meq en 100 cc de sol.salina para pasar en una hora

20 meq en 200 cc de sol.salina para pasar en una hora

En vena central:

20 meql en 50 cc pasarla en una hora

40 meq en 100cc pasar la en una hora

31.Los indicios de falla renal aguda son:

(Na filtrado/Na excretado) x 100%

32.Na urinario menor de 10: falla cardíaca, presentará edema estará hinchado y lo
puede llevar a una I.C.C. ( Esto lo saque de Medline: Los valores normales generalmente
son de 15 a 250 mEq/L/día, dependiendo del estado de deshidratación y la ingesta diaria
de sodio en la dieta. Los rangos de los valores normales pueden variar ligeramente entre
diferentes laboratorios. Nota: mEq/L/día = miliequivalentes por litro por día.)

33.Causa de enuresis: necrosis cortical difusa.

34.En SIDH la concentración urinaria es menor de 20.

35. Deshidratación pura de agua excepto: hay salida de electrolitos del cerebro.

36.Urémico crónico con deshidratación hipertónica severa y acidótico presentará lo


siguiento excepto: osmolaridad urinaria de 8mm mosmol/kg.

37. Px con vómitos abundante, hipertensión arterial; Na urinario=6, oliguria, osmolaridad


de 800 presenta: insuficiencia suprarrenal.

38. Factor que predispone a la hipernatremia son intoxicación hídrica.

39. Se presenta en acidosis tubular renal proximal excepto: NaHCO3 y lo alfa


antagonistas.

La hipokalemia produce:
- mayor negatividad del potencial de reposo.

La hipokalemia en un cirrótico puede precipitar COMA por:


- aumento de la amniogénesis

Exceso del kayaxelate en el tratamiento de hiperkalemia puede producir:


- constipación.

Peligro de la hipermagnesemia severa:


- paro respiratorio.

Hipofosfatemia:
- aumenta el calcitriol.

Consecuencia metabólica de hipofosfatemia severa:


- miocardiopatía

Causa de hiperfosfatemia:
- insuficiencia renal

El Síndrome Nefrósicose caracteriza por:


- proteinuria mayor o igual de 3g y albúmina sérica menor o igual de 3g/dl.

En el Síndrome Nefrósico pueden encontrarse:


- cilindros céreos

Urémico agudo, oligúrico, acidótico, deshidratado puede presentar:


- osmolaridad urinaria de 250mosm/kg

La falla renal crónica avanzada cursa con:


- brecha aniónica elevada
- Primera*****Falla renal crónica:
a.HTA
b.Glomerulopatias
c.Diabetes Mellitus
d.Nefritis tubulo intersticial

- Uno de los indices de falla renal aguda es:


Uosm/Tosm

- Hormona producida en los riñones que disminuye en la falla renal cronica:


▪ ADH
▪ Noradrenalina
▪ Peptido natriuretico (segun arjona)
▪ 1,25 (OH)2 Vit D.

- Las calcificaciones metastásicas se deben:??


▪ Aumento del producto Ca, P
▪ Aumento del producto Ca, Mg
▪ Hipoparatiroidismo
▪ Hiperparatiroidismo 1ario

- Causa de anuria:
▪ Pielonefritis unilateral
▪ Necrosis tubular aguda
▪ Enf cambios minimo
▪ Necrosis cortical

- la etapa de la insuficiencia renal cronica con tasa de filtración glomerular de 60-


89 ml/min es la:??
▪ 1
▪ 2
▪ 3
▪ 4

- La FENA es la relación entre:


▪ Oferta tubular/Na reabsorbido x 100
▪ depuración de Na/ depuración de creatinina x 100
▪ oferta tubular/ Na excretado x 100.
▪ Na excretado/ Na reabsorvido x 100.

- En la nefritis tubular intersticial aguda se pierde la capacidad de poder concentrar


la orina debida a:??
▪ Daño medular
▪ Daño glomerular
▪ Daño tubular proximal
▪ Daño de la porcion cortical del tubulo colector

- La solucion de NaCl al 23,4 % tiene:


8000 mOsm/l

- Se asocia con el SIAD, excepto:


lipoma

- El efecto biologico mas importante de la calcitonina es, excepto:


▪ Responder a la hipocalcemia
▪ Aumenta la calciuria
▪ Estimular la resorcion osea osteoclastica
▪ Utilizada en el tx de la hipercalcemia.

- Favorece la entrada de K al espacio intracelular, excepto:


▪ Agonista adrenergico
▪ Estimulacion adrenergico
▪ Insulina
▪ Alcalosis metabolica

- Factores que afectan el balance hídrico en el post-operatorio, excepto:


▪ Fármacos
▪ Calcitriol
▪ Administración de liquidos hipotonico
▪ Aumento de la ADH

- Las primeras manifestaciones clínicas de la hiponatremia son:??


▪ neurológica
▪ gastrointestinal
▪ pulmonares
▪ musculares

- En la hipernatremia hipertonica se da lo siguiente excepto:


▪ Entrada de Na
▪ Salida de aa
▪ Ganancia de iones
▪ Disminución del agua cerebral

- Manifestaciones clinicas de la hipercalcemia, excepto:


▪ Taquicardia
▪ HTA
▪ Constipación
▪ Vomitos

- En falla renal aguda por nefrotoxicidad:


▪ Se conserva la membrana basal
▪ Se destruye la membrana basal
▪ Frecuente hay oliguria
▪ La creatinina no aumenta

- Causa de hipomagnesemia:
alcoholismo

- Na serico de 168 mEq/l 70 kg (utilize como % de agua al 50% y Na normal 140


mEq/l
7 litros

- Indicaciones de diálisis en falla renal aguda, excepto:


▪ Encefalopatia uremica
▪ Hiperkalemia refractaria
▪ Ac. Metabolica refractaria
d. hipocalcemia
- El KCL IV se puede asministrar asi, excepto:
a.infusion por linea periferica de 10 meq en 100 ml en 1hora
b. “ “ central de 20 meq em 50ml en 1 hora
c. “ “ central de 100 meq em 50ml en 1 hora
d. “ “ periferica de 20 meq en 20ml en 2 horas

- Aumenta la excrecion urinaria de K, excepto:


o Triamtereno
o Bumetanida
o Furosemida
o Metolazona

- Factores que regulan la secrecion distal de K, excepto:


o Flujo tubular distal y aporte distal de Na
o Excrecion de aniones no reabsorbibles
o Aldosterona
o PTH

- Enfermo de 60 Kg con 60% de agua corporal, Posm 260 mOsm/kg, ¿Cuántos


gramos de NaCl aumentaria el Na en 10 mEq?
a. 21 gramos

50. Conteniendo 10 g de MgSO4 al %, cuantos ml de sal necesitaria?


20 ml

94. Causa de hiperfosfatemia:


q. Falla hepatica
r. Falla renal cronica
s. Falla cardiaca
t. Insuficiencia suprarrenal

95. Causas de hiperfosfatemia grave, excepto:


m. recuperacion de cetoacidosis diabetica.
n. Alcalosis respiratoria
o. Acidosis respiratoria.

96. Paciente con vómitos severos, oligurico con hipotensión arterial Una> 40……
800 mOsm/Kg puede tener:
q. Insuciencia renal aguda.
r. Insuficiencia suprarrenal
s. Falla pre-renal
t. Exceso de mineralocorticoides
97. Paciente urémico crónico con deshidratación severa puede presentar lo siguiente
excepto:
a. Densidad urinaria 1010.
b. Osmolaridad urinaria de 800 mOsm/Kg
c. K sérico aumentado
d. Oliguria.

98. La hipokalemia produce:


a. Aumento de la magnitud del potencial de reposo
17. La PTH promueve todo lo siguiente excepto:
a aumento de la absorción intestinal de calcio
b- aumento de la absorción tubulár de fosfato..
c. aumento de la resorción ósea de fosfato
d- aumento de la resorción osea de calcio
e, disminución de la reabsorción de fosfato

18. Un varon de 57 esta con hemodiálisis de mantenimiento para insuficiencia


renal cronica, ¿Cuál de las sig anomalías metabolicas podría ser anticipáda?
a. hipernatremia
b. hiponatremia
c.
d. exceso de vitamina D
e. hipoparatiroidismo

19. Un px de 25años fue ingresado a UCI por lesiones graves en cabeza y fractura
de la base del Graneo, aprox X horas después de la lesion manifestaba poliuria, la
OSMU 150, y la de suero 350. Los líquidos IV fueron detenidos y 3 hors después
la producción de orina y la osm urinaria permanecieron sin cambio. Se
administraron 5 unidades de vasopresina IV, la osm de la orina se
incremento a 300. ¿Cuál es el dx mas probable?
a- Diabetes insípida central
b. diabetes insípida nefrogenica
c. intoxicación acuosa
d. sobrecarga de solutos
e. S IADH

55. indica falla renal aguda


a- U..N.. = 4.1
b- UN = 1
c. U..N.. = 1/4i/
d. Gravedad especifica 1.026
e. Ninguna

56. deshidratación por perdida de agua y densidad urinaria 1.003, creatinina 6 /


a. falla renal aguda parenquimatosa
b- falla prerenal
c. falla renal obstructiva
d. falla renal cronica
e. no hay falla renal

57. uno de los indices de falla renales


a. reabsorción de Na/oferta tubular de Na x 100
b. excresion urinaria deNa/oferta tubular de x 100
c.- oferta tubular de Na/excrecion urinaria de Na x 100 ~.--~
d- Una/Pna x 100
e. Ninguna

58. el Sdr hepatorenal es causa de falla:


a- prerrenal
b. renal
c. póstrenal
d. falla cardiaca
e. daño tubular

59. causa mas común de falla renal crónica:


A- DM

60- en la glomendonefritis aguda por infeccion, la causa, excepto


a- proteina mayor de 10 g
b- osmolaridad urinaria normal
c. hematuria
d. HT A
e. Edema

61. falla renal cronica, excepto


a. PTH aumentado
b- Hipofosfatemia
c- Acidemia
d. Hipocalcemia
e. 1.25 OH (D3) disminuida

62. EN la falla renal aguda siempre habra


a- Aumento de creatinina
b- fraccion de excrecion de Na aumentado
c- hiperkalemia
d- Una aumentado
e- Hipernatremia

63. causa de falla prerenal


a. Sdr nefrotico
b- obstrucción de ambos uréteres
c- falla cardiaca
d. Grlomerulonefritis aguda post-infeciosa
e. Calculo renal

64. en la nefritis tubular interticial se puede dar, excepto


a. h.ipernatremia
b. proteinuria de 1.5 g
c. normotension arterial
d. hipostenuria
e. hipocalcemia

65. Sugiere glomerulonefritis:


a. leucocituia
b-cilindros eritrocitarios
c- anuria
d. -

66 -causa de falla renal parenquimatosa


i) papilitis
j) calculo en pelvis derecha
k) estenosis unilateral de arteria renal
l) cistitis hemorragica

2--En el riñón la hormona paratiroides:


Aumenta la reabsorción tubular de calcio y disminuye la de fósforo

3- La hipercalcemia produce
Potencialización de los efectos de la hipercalemia

36- La hipomagnesemia produce.


Vasodilatación periférica

36-De los factores: nefrotoxicidad./ -La buena hidratación

37 - En la concentración sérica para calcular:: 8,8

38- la velocidad de administración de potasio: 40 meq

39-hipocalcemia produce : fosfaturia

40-La causa de hipertensión con hipocalemia: Síndrome de Bartter=:,

41-En paciente con vomito severo hipotensión arterial o ir a la sar con mas de 40
meq
Insuficiencia suprarrenal;
42-La ,administración de sol isotónica aumento de vol extracelulares

43-Glomerulonefritis post estreptococcica Altera el cociente de ultrafiltración

44-En-el túbulo` intersticial se puede dar: La proteinuria de 4 g al día.

25. Cual de las siguientes opciones esta contraindicada en el tratamiento de la


hiperkalcmia
a, gluconato de calcio
b. aldactone
c, kayaxelate
d. Bicarbonato de sodio e. Todas las anteriores

43. Rotando en sala de medicina interna le piden que evalue un paciente con HIV
el cual porpresentar una meningitis por criptococci recibe anfotericina B; dicho
este medicamento buscaría como complicación:
a. aparicion de diarrea
b. aumento de la presion arterial
c- hipokalemia
d . cefalea
e. dolor toráxico

81- donde se halla el calcio en el organismo:


a- 50% sangre
b- 20 % proteínas
c- 99% hueso

aumento de PTH
b- aumento de la absorción intestinal de fosfato
c- disminuye fosfatemia
d- aumento de vitamina D

9- una solucion isotonica


a- aumenta el volumen extracell

10- la vel de administración de potasio en caso de riesgo para la vida


a—40 meq por hora

11- Px cronico con deshidratación severa puede presentar los siguiente


excepto:
a-osmolaridad urinaria de 800 mosmol/Kg

12- Px con vomito abundante, HTA, sodio urinario en 6, oligurico,


Osmolaridad de 800 presente,
a- insuficiencia suprarrenal
13- Px uremico cronico con vomito severo hipotension arterial orina al
azar, muestra sodio mas de 40, Osmolaridad de 800 puede tener :
a- Falla suprarrenal

102. Cuándo disminuye el fosfato?


R. Aumento de PTH

103. Ejemplo de hipernatremia hipervolémica:


R. Aldosteronismo primario

104. La hipocalemia produce:


R. Aumento de la magnitud del potencial de reposo

105. La depleción de agua produce, excepto:


R. Salida de electrolitos del cerebro

106. Velocidad de administración de KCl en caso de urgencia:


R. 40 mEq/h

107. Cambios en EKG de la hipermagnesemia es igual:


R. Hipercalemia

108. Enfermo de 60 Kg con 60% de agua corporal, osmolaridad plasmática 260


mosmol/Kg, ¿cuántos gramos de NaCl aumenta el Na en 10 mEq en 24 horas?
R. a) 15
b) 21
c) 30
d) 19

109. Uso de Calcio en tratamiento de hipercalemia para:


R. Modificar el Potencial de Membrana

110. Hipermagnesemia severa produce:


R. Paro respiratorio

111. Venoclisis de 1L de D/A 5% conteniendo 10 mg de MgSO4 y al 50%:


R. 20mL

112. La hipocalcemia:
R. Prolongo QT a expensas del segmento ST

113. Tratamiento de urgencia de hipercalcemia:


R. SSN 0.9%

114. Causa de hiperfosfatemia:


R. Insuficiencia renal
115. Adaptación cerebral, hipertonicidad, excepto:
R. Pérdida de iones

116. En la adaptación cerebral, la hipotensión da lo siguiente, excepto:


R. Ganancia de aminoácidos
117. En ausencia de síntomas de hipernatremia, el sodio debe ser disminuido a razón
de:
R. 0.5 mEq/L/h

118. Para corregir un sodio cerca de 185 mEq se administró en 86 horas, 66 letras de
D/SS 5%. ¿cuál es la velocidad de administración de la venoclisis?
R. 69 mL ¿?????

119. Manifestación clínica de hipercalcemia, excepto:


R. a) Hipotensión arterial
b) Bradicardia
c) Nauseas
d) Anorexia

120. Consecuencia de hipofosfatemia, excepto:


R. a) No afecta al SNC
b) Disfunción plaquetaria
c) Cardiomiopatía
d) Osteomalacia

121. Causa de hipofosfatemia, excepto:


R. a) Falla renal
b) Recuperación de cetoacidosis diabética
c) Alcalosis respiratoria
d) Quemaduras graves

122. Causa de HTA con hipocalemia, aldosterona-renina baja:


R. Síndrome de Cushing

123. Causas de HTA con hipocalemia y renina alta:


R. Hipertensión maligna

124. En una deshidratación severa hipertónica con hipotensión arterial severa, la


prioridad inicial es administrar:
R. SSN 0.9%

125. En la secreción inapropiada de ADH se da lo siguiente, excepto:


R. No hay expansión de volumen

126. Tejido con mayor contenido de K+:


R. a) Hígado
b) Músculo
c) Eritrocito
d) Hueso
127. La insuficiencia renal
funcional de la cirrosis hepática es debida a:
R. Vasoconstricción de las arterias renales

23. Px con IRC tine una taza de filtración glomerular de menor de 25 ml/min ;aunque
hay yuna tabla de las etapas en esta enfemedad crónica.
Estas osn las TFG:
1. 90 en riesgo
2. 20 ( con TFG normal o incrementada)
3. 60 -89
4. 30 – 59
5. 15-29
Menor a 15 diálisis.

24. Factor que altera el balance hídrico en el periódo post- operatorio:


Disminución de la ADH (Excepto)

25. Un ejemplo de hipokalemia con aldosterona alta y osmolaridad baja es:


Hiperaldosteronismo primario

26.Ejemplode hipernatremia hipervolémica: Hiperaldosteronismo primario

27.Hiperkalemioa son alteraciones del EKG no es necesario administrar gluconato de


calcio se colocará un antagonista de K asíe l Px no cae en paro y estoes si tiene
manifestación en el EKG.

28. La fracción de excreción de sodio estará aumentada en falla cardíaca.

29.En la hiperkalemia: hay menos diferencia entre el potencial de reposo y el umbral.

Manifestaciones cardíacas. El problema médico más grave de la hiperkalemia es la


cardiotoxicidad. Los cambios en el ECG producidos por los niveles altos de potasio son
bastante constantes. A medida que aumentan los niveles se aprecian los siguientes
cambios:

Ondas T picudas (con intervalo QT normal o ligeramente reducido)


Prolongación del intervalo PR con depresión de ST
Desaparición progresiva de la onda P
Bloqueo cardíaco progresivo
Arritmias ventriculares
Paro cardíaco
Las ondas T picudas constituyen el dato en el ECG más constante en la hiperkalemia.

Efectos neuromusculares. El primer signo neuromuscular de la hiperkalemia suele ser la


aparición de parestesias seguidas de debilidad progresiva de varios grupos musculares. Si
el cuadro se agrava se observa cuadriplejia fláccida. Las funciones cerebrales y de los
pares craneanos se conservan y la parálisis de la musculatura respiratoria puede ocurrir,
pero es excepcional.

30. La infusión de potasio sin riesgo para la vida del PX: es de 40 meq; porque

I lb de solución salina: debe llevr de 20 a40 mEq de KCl a una velocidad de 10 a 20


mEq por hora; si aumenta la velocidad puede provocar un bloqueo A-V. Se recomienda
una Velocidad mayor si la hipokalemia es muy severa.

Sis e usa venas priféricas es:

10 meq en 100 cc de sol.salina para pasar en una hora

20 meq en 200 cc de sol.salina para pasar en una hora

En vena central:

20 meql en 50 cc pasarla en una hora

40 meq en 100cc pasar la en una hora

31.Los indicios de falla renal aguda son:

(Na filtrado/Na excretado) x 100%

32.Na urinario menor de 10: falla cardíaca, presentará edema estará hinchado y lo
puede llevar a una I.C.C. ( Esto lo saque de Medline: Los valores normales generalmente
son de 15 a 250 mEq/L/día, dependiendo del estado de deshidratación y la ingesta diaria
de sodio en la dieta. Los rangos de los valores normales pueden variar ligeramente entre
diferentes laboratorios. Nota: mEq/L/día = miliequivalentes por litro por día.)

33.Causa de enuresis: necrosis cortical difusa.

34.En SIDH la concentración urinaria es menor de 20.

35. Deshidratación pura de agua excepto: hay salida de electrolitos del cerebro.

36.Urémico crónico con deshidratación hipertónica severa y acidótico presentará lo


siguiento excepto: osmolaridad urinaria de 8mm mosmol/kg.
37. Px con vómitos abundante, hipertensión arterial; Na urinario=6, oliguria, osmolaridad
de 800 presenta: insuficiencia suprarrenal.

38. Factor que predispone a la hipernatremia son intoxicación hídrica.

39. Se presenta en acidosis tubular renal proximal excepto: NaHCO3 y lo alfa


antagonistas.

1. En la Hiperkalemia:
R/. Hay una menor diferencia entre el potencial de reposo y de umbral

2. La hipokalemia en un cirrotico puede desencadenar:


R/. Coma Hepático

3. La hipermagnesemia puede producir:


R/. Paro respiratorio

7. Tratamiento de hiprecalcemia:
R/. SSN 0.9%

8. La hipofosfatemia son causas, excepto:


a. Aumento de la PTH
b. Aumento de la absorción intestinal de fosfato
c. Disminuye la fosfatemia
d. Aumento de la vitamina D

9. Una solución isotónica:


R/. Aumenta el volumen extracelular

13. La velocidad de administración de potasio en caso de gran riesgo para la vida puede
ser:
a. 10-20 mEq/L
b. 8mE/L
c. 110mE/L
d. 40 mEq/h

14. Paciente crónico con deshidratación severa puede presentar lo siguiente excepto:
a. Densidad urinaria 1010
b. Osmolaridad urinaria de 800 mOsm/kg
c. K sérico aumentado
d. Oliguria
15. Paciente con vómito abundante, hipertensión arterial, sodio urinario de 6, oligúrico,
osmolaridad de 800 presenta:
R/.Insuficiencia suprarrenal
Paciente urémico crónico con vómitos severos, hipotensión arterial, orina al azar muestra
Na mas de 40, osmolaridad de 800 puede tener:
Falla suprarrenal

16. Todo lo siguiente es cierto acerca de Colitis Ulcerosa, EXCEPTO:


R/. No está en relación las alteraciones inmunológicas.

17. En quien predomina la enfermedad de Hígado Graso:


R/. Sexo masculino

18. Paciente con Insuficiencia renal crónica, con K sérico de 6, leve academia, EKG
normal, se trata con:
R/. Kayetzalate

22. La hopikalemia predispone, excepto:


R/. Amoniogénesis

25. Cual de las siguientes es causa de hiperfosfatemia:


R/. Insuficiencia renal

26. Paciente con deshidratación hipertónica e hipertensión:


R/. SSN 0.9%

29. En la Glomerulonefritis Aguda post-infecciosa disminuye la tasa de filtración:


R/. Por alteración en el coeficiente de ultrafiltración

30. Causa de Insuficiencia preerenal, excepto:


R/. Aneurisma Disecante de la Aorta

32. Causa de hipomagnesemia:


e. estreñimiento
f. alcoholismo
g. falla pre-renal
h. IRA (insuficiencia renal aguda)

33. La hipocalcemia en la falla renal aguda puede ser asintomática por la:
e. acidemia
f. hipoalbuminemia
g. Deficiencia de PTH
h. Alcalemia

34. La hipofosfatemia causa, excepto:


e. Aumento de vit. D
f. Aumento de PTH
g. Disminución de la excresión de fosfato
h. Aumento de la absorción de fosfato a nivel intestinal

35. Un pte con diuresis de 250 ml al día con osmolaridad plasmática (presión
osmótia) y osmolaridad urinaria de 300 mosm/l, el dx probable e:
e. Insuficiencia suprarrenal
f. Insuficiencia renal
g. Nefritis túbulo intersticial crónica
h. Deshidratación hipertónica

36. Pte con oliguria, hiperkalemia, hiponatremia, creatinina 0.8 mg/dl y CPK normal:
Esto puede corresponder a:
e. IRA
f. Insuficiencia suprarrenal
g. Síndrome hepatorrenal
h. Rabdomiolisis

37. No está indicado en caso de hiperkalemia:


s. Aldactone

38. Mujer con hiperemesis grávida:


s. Alcalosis hipoclórica

39. En causa de hipomagnesemia:


s. Alcoholismo.

40. En el metabolismo del calcio está implicado, menos:


s. Insulina
41. Fórmula del clearence de creatinina:
s. clearence = 140 – edad x peso en Kg
72 x creatinina

42. Su infección produce litiasis renal:


s. proteus sp.

43. Fórmula de irritabilidad muscular:


s. IR = Na + K + OH
( ) + Mg + H

44. Indica falla renal aguda:


f. U.N. = 4.1
g.
U.N. = 1
h.
U.N. = ¼
i.
Gravedad específica: 1.026
j.
Ninguna.

45. Deshidratación por pérdida de agua y densidad urinaria 1.003, creatinina 6:


f. falla renal aguda parenquimatosa
g.
Falla pre-renal
h.
Falla renal obstructiva
i.
Falla renal crónica
j.
No hay falla renal.

46. Uno de los índices de falla renal es:


f. Reabsorción de Na/ oferta tubular de Na x 100
g.
Excreción urinaria de Na/ oferta tubular de Na x 100
h.
Oferta tubular de Na/ excresión urinaria de Na x 100 **

i.
Una / pna x 100
j.
Ninguna

47. El síndrome heatorrenal es causa de falla:


f. pre-renal
g.
renal
h.
posrenal
i.
falla cardiaca
j.
daño tubular

48. Causa más comun de falla renal cronica:


f. Diabetes melitus
g.
HTA
h.
Glomerulonefritis
i.
Enfermedad poliquística renal
j.
Vasculitis

36. En la glomerulnefritis aguda por infección, la causa, excepto:


a. proteínas mayor de 10g
b. osmolaridad urinaria normal
c. hematuria
d. HTA
e. Edema

• A pesar de la diversidad de enfermedades indicadas abajo, ellas tienen muchos


síntomas en común. Con frecuencia, los síntomas y signos observados incluyen:
disminución en el volumen de orina, proteína en la orina (proteinuria), sangre en
la orina microscópica o macrscópica (hematuria), hinchazón (edema), presión
sanguínea alta (HTA) y una disminución en la capacidad del riñón de extraer
eficazmente los residuos.
49. Falla renal crónica, excepto:
f. PTH aumentada
g. Hipofosfatemia
h. Acidemia
i. Hipocalcemia
j. 1.25 OH (D3) disminuída.

50. En la falla renal aguda siempre habrá:


f. aumento de creatinina
g. fracción de excresión de Na aumentado
h. Hiperkalemia
i. Una aumentado
j. Hipernatremia

51. Causa de falla pre-renal:


f. Sd. Nefrótico
g. Obstrucción de ambos uréteres
h. Falla cardiaca
i. Glomerulonefritis aguda pos- infecciosa
j. Calculo renal

52. En la nefritis tubular intersticial se puede dar, excepto:


f. hipernatremia
g. proteinuria de 1.5 g
h. normotension arterial
i. hipostenuria
j. hipocalcemia

53. Sugiere glomerulonefritis


e. leucocituria
f. cilindros eritrocitarios
g. anuria
h. ……

54. Causas de falla renal parenquimatosa:


f. papilitis
g. calculo en pelvis derecha
h. estenosis unilateral de arteria renal
i. cistitis hemorragica
j. ………

55. Causa de anuria:


f. Enfermedad de cambios mínimos
g. Necrosis cortical
h. Pielonefritis aguda leve bilateral
i. Necrosis tubular aguda
j. Calculo coloriforme bilateral

56. La hipofosfatemia causa todo, excepto:


r. Aumento de vit. D

57. En pte con falla renal crónica que ha sido transplantado esperamos encontrar:
s. osteomalacia.

4) Cuál de las siguientes medicinas pueden dar hiperkalemia


p) inhibidor de la enzima convertidota
q) bloqueador alfa adrenérgico
r) bloqueador beta adrenérgico
s) ayb
t) ayc

2) Hormonas que disminuyen en la falla renal crónica, excepto


a) 1,25 dihidroxi vitatamina D
b) eritropoyetina
c) paratohormona
d) foliculo estimulante

3) MgSO4 (PM 120) 1 gramo contiene de magnesio en meq


a) 10
b) 7.9
c) 8.3
d) 6

4) Factores que conducen a la progresión de falla renal crónica


a) hiperlipidemia
b) toxinas urémicas
c) proteinuria
d) b y c
c) todas

5) Paciente de 20 años presenta abundantes vômitos y diarrea de 3 días de evolución,


examen físico depleción de volumen, suero muestra Na 155, K 3meq/l, Cl 117meq/l,
HCO3 25 meq/l, tratamiento de elección:
a) D/A 5% + KCl
b) solución salina 3% + KCl
c) solución salina 23.4% + KCl
d) solución salina 0.45% + KCl

6) En la enfermedad ateroembólica renal se da lo siguiente excepto:


d) Velocidad de eritrosedimentación
7) Pseudohiperkalemia de 8 mEq/l que se ve en el EKG:
Onda T picuda
Aplanamiento de la onda P
Ay B
Ninguna

8)Criterios para iniciar diálisis excepto:


Alcalosiss Metabolica

9)Hipokalemia con normotension excepto


Síndrome de liddle

10) Plasama creatinina 8mg/dl N de urea 100mg/dl U na 40 Meq/l :


Insuficiência renal

11) Factores reversibles responsables de deterioro de la funcion renal excepto:


Fibrosis intersticial

12) Causa hipokalemia con renina y aldosterona disminuida:


Licorice

13. Sugiere trastorno túbulo intersticial:


.Proteinuria mayor a 3.5 g /24 horas
.Notrmotensión
.Edema
.Osmolarida urinaria normal

14. Glomérulonefritis rápidamente progresiva


.Engrosamiento de la membrana basal
Fusión de los pedículos de los podocitos
prolñiferacion intracapilar
proliferación extracapilar ( creo que es esa verificar)

15.Una de las siguientes causas no produce hematuria


.enfermedad de cambios mínimos

16.Plasma Na:115, Posm240 mOsm,Uosm 680,Una 60:meq/l:


.Deshidratación hipotónica
.secreción inapropiada de ADH
.Deshidratación isotónica
.Deshidratación primaria

17.en la falla renal crónca produce necrosis


calcifilaxia

18. No produce hipokalemia:


Pancreatitis aguda
rabdomiolisis
neoplasia
Def. Vit D

19. Paciente con glucosa 90mg/dl, Na+= 115mEq/L, Urea en 140mg/dl


a. Síndrome hipoosmolar
b. Síndrome hiperosmolar
c. Diarrea
d. Ninguna de las anteriores.

20. Causa de seudohiponatremia con osmolaridad elevada


a. Hiperproteinemia
b. Diabetes mellitus
c. Hiperlipidemidemia
d. Reserción transuretral prostática.

21. Manifestaciones de hipermagnesemia excepto


Normotensión

22. Piel en uremia crónica excepto


Paño blanco

23. No es causa de necrosis papilar aguda


a.Diabetes mellitas
b. Uropatía obstructiva
c. Pielonefritis
d. Trombosis de venas renal.

24. Puede presentarse hiperuricemia excepto


d. Adenocarcinoma gástrico

25. Compromete riñones y pulmones:


a. Enf de GoodPasture
b. Granulomatosis de Wegener
c. LES
d. Ay C

26. Sugiere glomerulopatia:


-anasarca

27. TFG < 15ml/min corresponde al estadio de la enfermedad renal cronica:


* 4

28. Causa obstetrica de necrosis cortical bilateral


- desprendimiento prematuro de placenta
29. Promueve amoniogenesis em cirróticos
- hipokalemia

30. Criterio para diagnostico de síndrome nefrosico


a. Proteinuria en 24 hr de 3.5 gr para superficie corporal de 1. 73
b. Albumina serica < 3g/dl
c.Hiperlipidemia y edema
d. Todas las anteriores
e. ay b

3.4.5.6.7..Pareo
3..Nitrofurantoina → al final del embarazo produce hemolisis
4. Aminoglicosidos→ ototoxicidad y nefrotoxicidad
5.Sulfamidas→ icteria al final del embarazo
6.Trimetropin→ accion antifolica en el primer trimestre
7.Quinolonas→ alteración a nivel del cartílago oseo del feto.
101. Paciente masculino de 33 años llega a su consulta por presentar fiebre, un pequeño
rash, signo de Jordano positivo; el medico de turno le envía un urinalisis que revela
proteinuria, hematuria y múltiples leucocitos; dentro de su diagnostico diferencial, debe
estar una de las siguientes patologías, cual:

m. Nefritis Intersticial Crónica.


n. Nefritis Intersticial Aguda.
o. Síndrome Nefrosico.
p. Fracaso Renal Agudo.

102. Dentro de las Glomerulopatias, existen algunas, que se pueden presentar con
Síndrome Nefrosico, y otras con Síndrome Nefrítico, cual de las siguientes se suele
presentar con Síndrome Nefrítico:

m. Cambios Mínimos.
n. Nefropatia IgA.
o. Nefropatia Membranosa.
p. Diabetes.

103. En un paciente de 65 años, que desarrolla un Sd. Nefrítico Agudo, la etiología mas
común causando de la Glomerulopatia es:

m. Vasculitis.
n. GN rápidamente progresiva.
o. Nefropatia IgA.
p. LES.

104. Cual de las siguientes Glomerulopatia, siempre desarrolla Insufiencia Renal:

m. Cambios Mínimos.
n. Membranosa.
o. Rápidamente Progresiva.
p. Mesangiocapilar.

105. En un paciente con SIADH, al medirle la osmolaridad urinaria, la podemos


encontrar:

g. Elevada.
h. Disminuida.
i. Normal.

106. La hiponatermia, se hace clínicamente evidente cuando el sodio, ha disminuido a:

i. 110 mEq/lt.
j. 115 mEq/lt.
k. 120 mEq/lt.
l. 125 mEq/lt.

107-111. Mencione 5 medicamentos que puedan causar hiperkalemia:

k. ________________________________________________.
l. ________________________________________________.
m. ________________________________________________.
n. ________________________________________________.
o. ________________________________________________.
112-117. Las características del Síndrome Nefrotico son:

s. ________________________________________________.
t. ________________________________________________.
u. ________________________________________________.
v. ________________________________________________.
w. ________________________________________________.
x. ________________________________________________.

118-120. Los tres puntos en los que se basa la corrección de la hiperkalemia son:

g. ________________________________________________.
h. ________________________________________________.
i. ________________________________________________.

121-123. Para cada uno de estos puntos, se suele usar: se le solicita al estudiante ser lo
mas explicito posible en su respuesta, en cuanto a dosis y tratamiento exacto.

g. ________________________________________________.
h. ________________________________________________.
i. ________________________________________________.
124-126. Debemos sospechar, de alteraciones del Magnesio, en que pacientes:

g. ________________________________________________.
h. ________________________________________________.
i. ________________________________________________.

127. Paciente con hipokalemia + alcalosis metabólica + hipertensión arterial +


disminución de la renina plasmática, esta desarrollando un:

c. ________________________________________________.

128. Única causa de diarrea con alcalosis metabólica es:

c. ________________________________________________.

129. Si un paciente, tiene un potasio de 3 mEq/lt, decimos que ha perdido ya:

c. ________________________________________________.

130. Cual es la principal preocupación que se tiene, cuando un paciente tiene un aumento
en los niveles plasmáticos de Magnesio:

c. ________________________________________________.

131-132. Causas de acidosis metabólica con hipokalemia:

f. ________________________________________________.
g. ________________________________________________.

133. Que examen le ayudaria para diferenciarlas:

c. ________________________________________________.

134. Que esperaria encontrar en una ATR tipo 2:

c. ________________________________________________.

135-136. Principales causas de hipercalcemia:

e. ________________________________________________.
f. ________________________________________________.

136-137. Principales causas de hipocalcemia:


e. ________________________________________________.
f. ________________________________________________.

138-139. Consecuencias de la hiperfosfatemia:

h. ________________________________________________.
i. ________________________________________________.

140-143. El Sd. De Barter, se caracteriza clínicamente por:

i. ________________________________________________.
j. ________________________________________________.
k. ________________________________________________.
l. ________________________________________________.

144. En un paciente con diagnostico de Glomerulonefritis post estreptococcica, el


diagnostico serologico se basa en el hallazgo de anticuerpos:

m. Antiestreptolisina O.
n. Antiestreptolisina A.
o. Antiestreptolisina P.
p. Antiestreptolisina K.

145. En la Nefiritis secundaria al LES, se suelen encontrar positivos los:

m. ANA.
n. ANCA c.
o. ANTI LU
p. ANCA p.

146. Si un paciente es diagnosticado con Falla Renal Crónica, el siguiente punto es el


estadiage del mismo, si tiene una TFG en 82 ml/minuto, esta en estadio:

p. I
q. II
r. III
s. IV
t. V

147-148. La Osteodistrofia Renal, se va a caracterizar clínicamente por la presencia de:

e. ________________________________________________.
f. ________________________________________________.

149.Normalmente la Kps entre el Fósforo y el calcio es de:


c. ________________________________________________.

150-154. Las características de una muestra de orina con sedimento de tipo nefrítico, son:

h. ________________________________________________.
i. ________________________________________________.
j. ________________________________________________.
k. ________________________________________________.
l. ________________________________________________.

155-158. Mencione algunos parámetros de laboratorio, para pensar en una IRC Renal:

j. ________________________________________________.
k. ________________________________________________.
l. ________________________________________________.
m. ________________________________________________.

159. El Gasto cardiaco normal es de 5 – 6 litros por minuto; de los cuales, un 25% le
corresponde al riñón, en una relación de:

i. 1200 ml/min.
j. 1300 ml/min.
k. 1400 ml/min.
l. 1500 ml/min.

160. Si un paciente, diagnosticado con FRC, presenta ademas una acidosis metabolica
hipercloremica, usted sabe que este paciente debe tener una Brecha Anionica:

m. Normal.
n. Aumentada.
o. Disminuida.
p. Ninguna de las anteriores.

161. Este mismo paciente, por tener estas características clínicas, debe encontrarse en
estadios de la FRC:

m. Avanzados.
n. Iniciales.
o. No es relevante.
p. Ninguna de las anteriores.

162. Para que por una obstrucción, aumente la creatinina en IRA, se deben haber perdido:

i. ¼ partes de la masa renal.


j. 2/4 partes de la masa renal.
k. ¾ partes de la masa renal.
l. 4/4 partes de la masa renal.

163. La lesión histologica mas común de IRA es:

i. Necrosis cortical bilateral.


j. Necrosis tubular aguda.
k. Nefritis intersticial.
l. Papilitis necrotizante.

164. Y la mas peligrosa y temida es:

i. Necrosis cortical bilateral.


j. Necrosis tubular aguda.
k. Nefritis intersticial.
l. Papilitis necrotizante.

165. Cual de las siguientes, es la principal causa de IRA, en la maternidad.

o. Abortos infectados.
p. Fetos muertos retenidos.
q. Desprendimiento prematuro de membrana.
r. Eclampsia.
s. Se dan todas por igual.

10. Plasma: creatinina 1mg/dl,140 mEq/L. Orina: creatinina 150 mg/dl, Na 40 mEq/L
la FE Na es:
b) 0.19%

11. Factor determinante en tratamiento de hipokalemia real es:


a) función cardiovascular

12. Tratamiento de inicio de SIADH es:


a) restricción hidrica

13. Causa de hipomagnesemia


a) cirrosis avanzada
b) falla renal cronica
c) falla cardiaca
d) constipación
e) acidosis respiratoria

14. Causa renal de hipomagnesemia


a) falla renal cronica
b) esteatorrea
c) desnutrición severa
d) uso de diureticos
e) infeccion urinaria
15. En caso de hipercalcemia pensar en:
a) neoplasia
b) hiperfosfatemia severa
c) nefrolitiasis
d) nefrocalcinosis
e) diureticos

16. El síndrome nefrosico puede encontrarse cilindros:


a) cereos
b) hematicos
c) leucocitarios
d) epiteliales
e) mixtos

24. U Na 115, Uosm 240. P osm 680, P Na 62


SIADH

25. Paciente de 84 años que recibe diureticos para la hipertensión arterial. Presenta
diarrea y trastornos del sensorio y disminución del rubor de la piel. Na 174 U
Na 5 U osm 606. La hipernatremia se debe a:
Perdidas insensibles

26. hipernatremia hipervolemica


Aldosteronismo primario

27. causa de hipomagnesemia


Alcoholismo

28. hipocalcemia:
Provoca potencializacion de efectos de hiperkalemia

29. En hipocalcemia
Prolonga QT a expensas de segmento ST

30. Tratamiento de urgencia hipercalcemica


Solucion salina isotonica 0.9% 4 a 5 L

31. Síndrome nefrotico


Edema, proteinuria, hipertensión arterial, hematuria

La insuficiencia renal funcional de la cirrosis hepática es debida a:


a. lesión glomerular
b. lesión tubular
c. hipovolemia
d. compresión de la ascitis sobre los grandes vasos renales
e. vasoconstricción de las arterias renales
Urémico crónico con potasio sérico de 6.5mEq/L con leve academia, sin trastorno
muscular y EKG normal, puede ser tratado con:
m. kayaxelate
n. gluconato de Calcio al 10%
o. NaHCO3
p. Diálisis

La insuficiencia renal crónica ocasiona hiperfosfatemia permanentemente cuando la tasa


de filtración glomerular esta por debajo de:
m. 60ml/min
n. 50 ml/min
o. 25 ml/min
p. 40 ml/min

La hipofosfatemia es causa de excepto


m. inotropismo positivo
n. disminución de 2,3 DPG
o. hemólisis
p. curva de HbO2 desplazada hacia la izquierda

La solución de NaCl 23,4% tiene de osmolaridad:


m. 8000 mosm/l
n. 800 mosm/l
o. 80 mosm/l
p. 0,8 mosm/l

En la hiperkalemia:
m. aumenta la sístole eléctrica
n. hay menor diferencia entre el potencial de reposo y el umbral
o. el potencial de reposo transmembrana se hace mas negativo
p. el potencial umbral se hace menos negativo

La velocidad de administración de potasio en caso de gran riesgo para la vida puede ser
de:
m. 10-20 mEq/L
n. 8 mEq/L
o. 110 mEq/L
p. 40 mEq/L

Un ejemplo de hiponatremia hipotónica normovolemica es:


m. síndrome nefrósico
n. SIADH
o. Diarrea
p. Cirrosis
A una embarazada con eclampsia se le ordena 1 litro de D/A 5% con 10g de MgSO4.
¿Cuántos ml de MgSO4 al 50% se han utilizado?
a. 20
b. 5
c. 12,5
d. 30

Causa hipomagnesemia:
m. estreñimiento
n. alcoholismo
o. deshidratación hipertónica
p. insuficiencia renal aguda

Paciente con diuresis de 300ml/día, K sérico 7mEq/L, Posm y Uosm 300mosm/L, el


diagnóstico más probable es:
m. insuficiencia suprarrenal
n. insuficiencia renal
o. SIADH
p. Deshidratación primaria

La mielinosis pontina se caracteriza por:


m. pares craneales intactos
n. cuadriplejía flácida
o. comportamiento normal
p. hipertonía de los miembros inferiores

Calcule el déficit de agua en una paciente con sodio sérico 168mEq/L y que pesa 70Kg
(utilice como porcentaje de agua 50% y sodio normal mEq/L):
m. 5L
n. 3L
o. 10L
p. 7L

En la SIADH la concentración de sodio urinario es de:


m. menos de 10mEq/L
n. 20mEq/L
o. 15mEq/L
p. Mas de 40mEq/L

En paciente con vómitos abundantes y prolongados con hipokalemia severa debe


administrarse:
m. NaCl 0.9% con KHCO3
n. NaCl 0.9% con KCl
o. NaCl 0.9% con gluconato de potasio
p. NaCl 0.9% con NaHCO3
La hipokalemia en un paciente con cirrosis puede contribuir:
m. coma
n. alcalosis respiratoria
o. acidosis metabolica
p. aumento de la síntesis de urea

El plasma de un paciente contiene Na 125mEq/L, glucosa 108mg/dl, urea 300mg/dl,


podrá presentar:
m. cirrosis
n. síndrome nefrósico
o. síntomas de hipertonicidad
p. síntomas de hipotonicidad

La hiponatremia aguda sintomática es debida a, excepto:


m. exceso de glucocorticoides
n. SIADH
o. post-operatorio
p. intoxicación hídrica

La administración de solución salina 0.9% produce:


m. disminución del volumen intracelular y aumento del extracelular
n. aumento del volumen intra y extracelular
o. aumento del volumen extracelular
p. aumento del volumen intracelular

Alcalosis metabólica con hipertensión arterial, hipokalemia, aldosterona normal y renina


normal:
m. Hipertensión renovascular
n. Hipertensión maligna
o. Sindroma de Cushing
p. Administración de licor

La hipokalemia puede predisponer, excepto:


n. intoxicación digitálica
o. disminución de la amoniogénesis
p. rabdomiolisis
q. ondas U en el EKG

15. Las perdidas renales excesivas de potasio ....


a- Colitis ulcerativa
b- Enf de Cronh
c. colon irritable
d. ¿?
G. tumor velloso

17. La PTH promueve todo lo siguiente excepto:


a aumento de la absorción intestinal de calcio
b- aumento de la absorción tubulár de fosfato..
c. aumento de la resorción ósea de fosfato
d- aumento de la resorción osea de calcio
e, disminución de la reabsorción de fosfato

18. Un varon de 57 esta con hemodiálisis de mantenimiento para insuficiencia


renal cronica, ¿Cuál de las sig anomalías metabolicas podría ser anticipáda?
a. hipernatremia
b. hiponatremia
c.
d. exceso de vitamina D
e. hipoparatiroidismo

19. Un px de 25años fue ingresado a UCI por lesiones graves en cabeza y fractura
de la base del Graneo, aprox X horas después de la lesion manifestaba poliuria, la
OSMU 150, y la de suero 350. Los líquidos IV fueron detenidos y 3 hors después
la producción de orina y la osm urinaria permanecieron sin cambio. Se
administraron 5 unidades de vasopresina IV, la osm de la orina se
incremento a 300. ¿Cuál es el dx mas probable?
a- Diabetes insípida central
b. diabetes insípida nefrogenica
c. intoxicación acuosa
d. sobrecarga de solutos
e. S IADH

55. indica falla renal aguda


a- U..N.. = 4.1
b- UN = 1
c. U..N.. = 1/4i/
d. Gravedad especifica 1.026
e. Ninguna

56. deshidratación por perdida de agua y densidad urinaria 1.003, creatinina 6 /


a. falla renal aguda parenquimatosa
b- falla prerenal
c. falla renal obstructiva
d. falla renal cronica
e. no hay falla renal

57. uno de los indices de falla renales


a. reabsorción de Na/oferta tubular de Na x 100
b. excresion urinaria deNa/oferta tubular de x 100
c.- oferta tubular de Na/excrecion urinaria de Na x 100 ~.--~
d- Una/Pna x 100
e. Ninguna
58. el Sdr hepatorenal es causa de falla:
a- prerrenal
b. renal
c. póstrenal
d. falla cardiaca
e. daño tubular

59. causa mas común de falla renal crónica:


A- DM

60- en la glomendonefritis aguda por infeccion, la causa, excepto


a- proteina mayor de 10 g
b- osmolaridad urinaria normal
c. hematuria
d. HT A
e. Edema

61. falla renal cronica, excepto


a. PTH aumentado
b- Hipofosfatemia
c- Acidemia
d. Hipocalcemia
e. 1.25 OH (D3) disminuida

62. EN la falla renal aguda siempre habra


a- Aumento de creatinina
b- fraccion de excrecion de Na aumentado
c- hiperkalemia
d- Una aumentado
e- Hipernatremia

63. causa de falla prerenal


a. Sdr nefrotico
b- obstrucción de ambos uréteres
c- falla cardiaca
d. Grlomerulonefritis aguda post-infeciosa
e. Calculo renal

64. en la nefritis tubular interticial se puede dar, excepto


a. h.ipernatremia
b. proteinuria de 1.5 g
c. normotension arterial
d. hipostenuria
e. hipocalcemia

65. Sugiere glomerulonefritis:


a. leucocituia
b-cilindros eritrocitarios
c- anuria
d. -

66 -causa de falla renal parenquimatosa


a) papilitis
b) calculo en pelvis derecha
c) estenosis unilateral de arteria renal
d) cistitis hemorragica

60- Causa de hipercalemia:


a Aumento del volumen arterial eficaz
b. Hipercatabolismo tisular
c Hiperaldosteronismo
d. Alcalosis Inetabolica
e. Cirrosis hepatica

61. Causa de hipomagnesemia


a. cirrosis avanzada
b. falla renal cronica
c. falla cardiaca
d. constipacion
e. acidosis respiratoria

62. Causa renal, de hipomagnesemia


a. falla renal crônica
b. esteatorreaa
c desnutricion severa
d. usó de diureticos de asa
e. infeccion urinaria

63. en caso de hipercalcemia pensar en:


a. neoplasia
b: hiperfosfatemia severa
c. nefrolitiasis
d. nefrocalcinosis
e. diuréticos

64. En el síndrome nefrosico puede encontrarse cilindros


a. cereos
b. hematicos
c. leucocitarios
d. epiteliales
e. mixtos leucocitario y hematicos,

2--En el riñón la hormona paratiroides:


Aumenta la reabsorción tubular de calcio y disminuye la de fósforo

3- La hipercalcemia produce
Potencialización de los efectos de la hipercalemia

36- La hipomagnesemia produce.


Vasodilatación periférica

36-De los factores: nefrotoxicidad./ -La buena hidratación

37 - En la concentración sérica para calcular:: 8,8

38- la velocidad de administración de potasio: 40 meq

39-hipocalcemia produce : fosfaturia

40-La causa de hipertensión con hipocalemia: Síndrome de Bartter=:,

41-En paciente con vomito severo hipotensión arterial o ir a la sar con mas de 40
meq
Insuficiencia suprarrenal;

42-La ,administración de sol isotónica aumento de vol extracelulares

43-Glomerulonefritis post estreptococcica Altera el cociente de ultrafiltración

44-En-el túbulo` intersticial se puede dar: La proteinuria de 4 g al día.

18- La hipofosfatemia cansa excepto: aumento de vitamina D


20. La renal aguda, oligurica todo es cierto excepto: Osmolaridad urinaria
aumentada
21 Paciente can alcalosis metabolica e hipokalemia debe ser tratada con KCL

24. Usted tiene una paciente con hiperhemesis gravidica. El trastorno metabolico
que espera encontrar es:
A - acidosis hipocloremica
b, acidosis hipercloremica'
c- alcalosis hipocloremica
d. alcalosis hipercloremica
e. no hay alteración acido base

25. Cual de las siguientes opciones esta contraindicada en el tratamiento de la


hiperkalcmia
a, gluconato de calcio
b. aldactone
c, kayaxelate
d. Bicarbonato de sodio e. Todas las anteriores
43. Rotando en sala de medicina interna le piden que evalue un paciente con HIV
el cual porpresentar una meningitis por criptococci recibe anfotericina B; dicho
este medicamento buscaría como complicación:
a. aparicion de diarrea
b. aumento de la presion arterial
c- hipokalemia
d . cefalea
e. dolor toráxico

81- donde se halla el calcio en el organismo:


a- 50% sangre
b- 20 % proteínas
c- 99% hueso

4- en la hipercalemia
a- hay una menor diferencia entre el potencial de reposo y el umbral

5—la hipercalemia en un cirrótico puede desencadenar


a- coma hepatico

6- la hipermagnesemia puede producir


a- paro respiratorio

7- tx de hipercalcemia
a- SSN 0.9 %

8- la hipofosfatemia son causas excepto:


a- aumento de PTH
b- aumento de la absorción intestinal de fosfato
c- disminuye fosfatemia
d- aumento de vitamina D

9- una solucion isotonica


a- aumenta el volumen extracell

10- la vel de administración de potasio en caso de riesgo para la vida


a—40 meq por hora

11- Px cronico con deshidratación severa puede presentar los siguiente


excepto:
a-osmolaridad urinaria de 800 mosmol/Kg

12- Px con vomito abundante, HTA, sodio urinario en 6, oligurico,


Osmolaridad de 800 presente,
a- insuficiencia suprarrenal
13- Px uremico cronico con vomito severo hipotension arterial orina al
azar, muestra sodio mas de 40, Osmolaridad de 800 puede tener :
a- Falla suprarrenal

20- La hipocalemia predispone excepto:


b- amoniogenesis

21- Cual de las siguientes es causa de hiperfosfatemia


b- insuficiencia renal

22- en la glomerulonefritis aguda post infecciosa disminuye la tasa de


filtración:
b- por alteración en el coeficiente de ultrafiltracion

23- causa de insuficiencia prerenal excpto:.


b- Aneurisma disecante de la aorta

5) Cuál de las siguientes medicinas pueden dar hiperkalemia


u) inhibidor de la enzima convertidota
v) bloqueador alfa adrenérgico
w) bloqueador beta adrenérgico
x) ayb
y) ayc

2) Hormonas que disminuyen en la falla renal crónica, excepto


a) 1,25 dihidroxi vitatamina D
b) eritropoyetina
c) paratohormona
d) foliculo estimulante

3) MgSO4 (PM 120) 1 gramo contiene de magnesio en meq


a) 10
b) 7.9
c) 8.3
d) 6

4) Factores que conducen a la progresión de falla renal crónica


a) hiperlipidemia
b) toxinas urémicas
c) proteinuria
d) b y c
c) todas

5) Paciente de 20 años presenta abundantes vômitos y diarrea de 3 días de evolución,


examen físico depleción de volumen, suero muestra Na 155, K 3meq/l, Cl 117meq/l,
HCO3 25 meq/l, tratamiento de elección:
a) D/A 5% + KCl
b) solución salina 3% + KCl
c) solución salina 23.4% + KCl
d) solución salina 0.45% + KCl

6) En la enfermedad ateroembólica renal se da lo siguiente excepto:


d) Velocidad de eritrosedimentación

7) Pseudohiperkalemia de 8 mEq/l que se ve en el EKG:


Onda T picuda
Aplanamiento de la onda P
Ay B
Ninguna

8)Criterios para iniciar diálisis excepto:


Alcalosiss Metabolica

9)Hipokalemia con normotension excepto


Síndrome de liddle

10) Plasama creatinina 8mg/dl N de urea 100mg/dl U na 40 Meq/l :


Insuficiência renal

11) Factores reversibles responsables de deterioro de la funcion renal excepto:


Fibrosis intersticial

12) Causa hipokalemia con renina y aldosterona disminuida:


Licorice

13. Sugiere trastorno túbulo intersticial:


.Proteinuria mayor a 3.5 g /24 horas
.Notrmotensión
.* Edema
.Osmolarida urinaria normal

14. Glomérulonefritis rápidamente progresiva


.Engrosamiento de la membrana basal
Fusión de los pedículos de los podocitos
prolñiferacion intracapilar
proliferación extracapilar ( creo que es esa verificar)

15.Una de las siguientes causas no produce hematuria


.enfermedad de cambios mínimos

16.Plasma Na:115, Posm240 mOsm,Uosm 680,Una 60:meq/l:


.Deshidratación hipotónica
.* secreción inapropiada de ADH
.Deshidratación isotónica
.Deshidratación primaria

17.en la falla renal crónca produce necrosis


calcifilaxia

18. No produce hipokalemia:


Pancreatitis aguda
rabdomiolisis
neoplasia
Def. Vit D

19. Paciente con glucosa 90mg/dl, Na+= 115mEq/L, Urea en 140mg/dl


a. Síndrome hipoosmolar
b. * Síndrome hiperosmolar
c. Diarrea
d. Ninguna de las anteriores.

20. Causa de seudohiponatremia con osmolaridad elevada


a. Hiperproteinemia
b. * Diabetes mellitus
c. Hiperlipidemidemia
d. Reserción transuretral prostática.

21. Manifestaciones de hipermagnesemia excepto


Normotensión

22. Piel en uremia crónica excepto


Paño blanco

23. No es causa de necrosis papilar aguda


a.Diabetes mellitas
b. Uropatía obstructiva
c. Pielonefritis
d. Trombosis de venas renal.

24. Puede presentarse hiperuricemia excepto


d. Adenocarcinoma gástrico

25. Compromete riñones y pulmones:


a. Enf de GoodPasture
b. Granulomatosis de Wegener
c. LES
d. Ay C
TODAS

26. Sugiere glomerulopatia:


-anasarca

27. TFG < 15ml/min corresponde al estadio de la enfermedad renal cronica:


* 4

28. Causa obstetrica de necrosis cortical bilateral


- desprendimiento prematuro de placenta

29. Promueve amoniogenesis en cirróticos


- hipokalemia

30. Criterio para diagnostico de síndrome nefrosico


a. Proteinuria en 24 hr de 3.5 gr para superficie corporal de 1. 73
b. Albumina serica < 3g/dl
c.Hiperlipidemia y edema
d. Todas las anteriores
e. ay b

11. Medicamento que no se debe modificar en el tratamiento de la insuficiencia renal:


a. Penicilina
b. Amoxicilina
c. Doxicilina
d.Tetraciclina
e.Clindamicina
f. Meropenem

58. Causa de hipomagnesemia:


i. estreñimiento
j. alcoholismo
k. falla pre-renal
l. IRA (insuficiencia renal aguda)

59. La hipocalcemia en la falla renal aguda puede ser asintomática por la:
i. acidemia
j. hipoalbuminemia
k. Deficiencia de PTH
l. Alcalemia

60. La hipofosfatemia causa, excepto:


i. Aumento de vit. D
j. Aumento de PTH
k. Disminución de la excresión de fosfato
l. Aumento de la absorción de fosfato a nivel intestinal

61. Un pte con diuresis de 250 ml al día con osmolaridad plasmática (presión
osmótia) y osmolaridad urinaria de 300 mosm/l, el dx probable e:
i. Insuficiencia suprarrenal
j. Insuficiencia renal
k. Nefritis túbulo intersticial crónica
l. Deshidratación hipertónica

62. Pte con oliguria, hiperkalemia, hiponatremia, creatinina 0.8 mg/dl y CPK normal:
Esto puede corresponder a:
i. IRA
j. Insuficiencia suprarrenal
k. Síndrome hepatorrenal
l. Rabdomiolisis

63. No está indicado en caso de hiperkalemia:


t. Aldactone

64. Mujer con hiperemesis grávida:


t. Alcalosis hipoclórica

65. En causa de hipomagnesemia:


t. Alcoholismo.

66. En el metabolismo del calcio está implicado, menos:


t. Insulina

67. Hiponatremia, hipotónico, normovolémico:


s. SIADH

68. Fórmula del clearence de creatinina:


t. clearence = 140 – edad x peso en Kg
72 x creatinina

69. Su infección produce litiasis renal:


t. proteus sp.

70. Fórmula de irritabilidad muscular:


t. IR = Na + K + OH
( ) + Mg + H
71. Indica falla renal aguda:
k. U.N. = 4.1
l.
U.N. = 1
m.
U.N. = ¼
n.
Gravedad específica: 1.026
o.
Ninguna.

72. Deshidratación por pérdida de agua y densidad urinaria 1.003, creatinina 6:


k. falla renal aguda parenquimatosa
l.
Falla pre-renal
m.
Falla renal obstructiva
n.
Falla renal crónica
o.
No hay falla renal.

73. Uno de los índices de falla renal es:


k. Reabsorción de Na/ oferta tubular de Na x 100
l.
Excreción urinaria de Na/ oferta tubular de Na x 100
m.
Oferta tubular de Na/ excresión urinaria de Na x 100 **

n.
Una / pna x 100
o.
Ninguna

74. El síndrome heatorrenal es causa de falla:


k. pre-renal
l.
renal
m.
posrenal
n.
falla cardiaca
o.
daño tubular

75. Causa más comun de falla renal cronica:


k. Diabetes melitus
l.
HTA
m.
Glomerulonefritis
n.
Enfermedad poliquística renal
o.
Vasculitis

36. En la glomerulnefritis aguda por infección, la causa, excepto:


a. proteínas mayor de 10g
b. osmolaridad urinaria normal
c. hematuria
d. HTA
e. Edema

• A pesar de la diversidad de enfermedades indicadas abajo, ellas tienen muchos


síntomas en común. Con frecuencia, los síntomas y signos observados incluyen:
disminución en el volumen de orina, proteína en la orina (proteinuria), sangre en
la orina microscópica o macrscópica (hematuria), hinchazón (edema), presión
sanguínea alta (HTA) y una disminución en la capacidad del riñón de extraer
eficazmente los residuos.

76. Falla renal crónica, excepto:


k. PTH aumentada
l. Hipofosfatemia
m. Acidemia
n. Hipocalcemia
o. 1.25 OH (D3) disminuída.

77. En la falla renal aguda siempre habrá:


k. aumento de creatinina
l. fracción de excresión de Na aumentado
m. Hiperkalemia
n. Una aumentado
o. Hipernatremia

78. Causa de falla pre-renal:


k. Sd. Nefrótico
l. Obstrucción de ambos uréteres
m. Falla cardiaca
n. Glomerulonefritis aguda pos- infecciosa
o. Calculo renal

79. En la nefritis tubular intersticial se puede dar, excepto:


k. hipernatremia
l. proteinuria de 1.5 g
m. normotension arterial
n. hipostenuria
o. hipocalcemia

80. Sugiere glomerulonefritis


i. leucocituria
j. cilindros eritrocitarios
k. anuria
l. ……

81. Causas de falla renal parenquimatosa:


k. papilitis
l. calculo en pelvis derecha
m. estenosis unilateral de arteria renal
n. cistitis hemorragica
o. ………
82. Causa de anuria:
k. Enfermedad de cambios mínimos
l. Necrosis cortical
m. Pielonefritis aguda leve bilateral
n. Necrosis tubular aguda
o. Calculo coloriforme bilateral

83. La hipofosfatemia causa todo, excepto:


s. Aumento de vit. D

84. En pte con falla renal crónica que ha sido transplantado esperamos encontrar:
t. osteomalacia.

85. Causa de hipomagnesemia:


m. estreñimiento
n. alcoholismo
o. falla pre-renal
p. IRA (insuficiencia renal aguda)

86. La hipocalcemia en la falla renal aguda puede ser asintomática por la:
m. acidemia
n. hipoalbuminemia
o. Deficiencia de PTH
p. Alcalemia

87. La hipofosfatemia causa, excepto:


m. Aumento de vit. D
n. Aumento de PTH
o. Disminución de la excresión de fosfato
p. Aumento de la absorción de fosfato a nivel intestinal

88. Un pte con diuresis de 250 ml al día con osmolaridad plasmática (presión osmótia) y
osmolaridad urinaria de 300 mosm/l, el dx probable e:
m. Insuficiencia suprarrenal
n. Insuficiencia renal
o. Nefritis túbulo intersticial crónica
p. Deshidratación hipertónica

89. Pte con oliguria, hiperkalemia, hiponatremia, creatinina 0.8 mg/dl y CPK normal: Esto
puede corresponder a:
m. IRA
n. Insuficiencia suprarrenal
o. Síndrome hepatorrenal
p. Rabdomiolisis

90. No está indicado en caso de hiperkalemia:


u. Aldactone

91. Mujer con hiperemesis grávida:


u. Alcalosis hipoclórica

92. En causa de hipomagnesemia:


u. Alcoholismo.

93. En el metabolismo del calcio está implicado, menos:


u. Insulina
94. Hiponatremia, hipotónico, normovolémico:
t. SIADH
95. Fórmula del clearence de creatinina:
u. clearence = 140 – edad x peso en Kg
72 x creatinina

96. Fórmula de irritabilidad muscular:


u. IR = Na + K + OH
( ) + Mg + H
97. Indica falla renal aguda:
p. U.N. = 4.1
q.
U.N. = 1
r.
U.N. = ¼
s.
Gravedad específica: 1.026
t.
Ninguna.

98. Deshidratación por pérdida de agua y densidad urinaria 1.003, creatinina 6:


p. falla renal aguda parenquimatosa
q.
Falla pre-renal
r.
Falla renal obstructiva
s.
Falla renal crónica
t.
No hay falla renal.

99. Uno de los índices de falla renal es:


p. Reabsorción de Na/ oferta tubular de Na x 100
q.
Excreción urinaria de Na/ oferta tubular de Na x 100
r.
Oferta tubular de Na/ excresión urinaria de Na x 100 **

s.
Una / pna x 100
t.
Ninguna

100. El síndrome heatorrenal es causa de falla:


p. pre-renal
q.
renal
r.
posrenal
s.
falla cardiaca
t.
daño tubular

101. Causa más comun de falla renal cronica:


p. Diabetes melitus
q.
HTA
r.
Glomerulonefritis
s.
Enfermedad poliquística renal
t.
Vasculitis

18. En la glomerulnefritis aguda por infección, la causa, excepto:


a. proteínas mayor de 10g
b. osmolaridad urinaria normal
c. hematuria
d. HTA
e. Edema

102. A pesar de la diversidad de enfermedades indicadas abajo, ellas tienen muchos


síntomas en común. Con frecuencia, los síntomas y signos observados incluyen:
disminución en el volumen de orina, proteína en la orina (proteinuria), sangre en la
orina microscópica o macrscópica (hematuria), hinchazón (edema), presión sanguínea
alta (HTA) y una disminución en la capacidad del riñón de extraer eficazmente los
residuos.

103. Falla renal crónica, excepto:


p. PTH aumentada
q. Hipofosfatemia
r. Acidemia
s. Hipocalcemia
t. 1.25 OH (D3) disminuída.

104.En la falla renal aguda siempre habrá:


p. aumento de creatinina
q. fracción de excresión de Na aumentado
r. Hiperkalemia
s. Una aumentado
t. Hipernatremia

105. Causa de falla pre-renal:


p. Sd. Nefrótico
q. Obstrucción de ambos uréteres
r. Falla cardiaca
s. Glomerulonefritis aguda pos- infecciosa
t. Calculo renal

106.En la nefritis tubular intersticial se puede dar, excepto:


p. hipernatremia
q. proteinuria de 1.5 g
r. normotension arterial
s. hipostenuria
t. hipocalcemia

107. Sugiere glomerulonefritis


m. leucocituria
n. cilindros eritrocitarios
o. anuria
p. ……

108. Causas de falla renal parenquimatosa:


p. papilitis
q. calculo en pelvis derecha
r. estenosis unilateral de arteria renal
s. cistitis hemorragica
t. ………

109.Causa de anuria:
p. Enfermedad de cambios mínimos
q. Necrosis cortical
r. Pielonefritis aguda leve bilateral
s. Necrosis tubular aguda
t. Calculo coloriforme bilateral

110. La hipofosfatemia causa todo, excepto:


t. Aumento de vit. D

111. En pte con falla renal crónica que ha sido transplantado esperamos encontrar:
u. osteomalacia.
6) Cuál de las siguientes medicinas pueden dar hiperkalemia
z) inhibidor de la enzima convertidota
aa) bloqueador alfa adrenérgico
bb) bloqueador beta adrenérgico
cc) a y b
dd) ayc

28. Hormonas que disminuyen en la falla renal crónica, excepto


a) 1,25 dihidroxi vitatamina D
b) eritropoyetina
c) paratohormona
d) foliculo estimulante

29. MgSO4 (PM 120) 1 gramo contiene de magnesio en meq


a) 10
b) 7.9
c) 8.3
d) 6

30. Factores que conducen a la progresión de falla renal crónica


a) hiperlipidemia
b) toxinas urémicas
c) proteinuria
d) b y c
c) todas

31. Paciente de 20 años presenta abundantes vômitos y diarrea de 3 días de evolución,


examen físico depleción de volumen, suero muestra Na 155, K 3meq/l, Cl 117meq/l, HCO3 25
meq/l, tratamiento de elección:
a) D/A 5% + KCl
b) solución salina 3% + KCl
c) solución salina 23.4% + KCl
d) solución salina 0.45% + KCl

32. En la enfermedad ateroembólica renal se da lo siguiente excepto:


d) Velocidad de eritrosedimentación

33. Pseudohiperkalemia de 8 mEq/l que se ve en el EKG:


Onda T picuda
Aplanamiento de la onda P
Ay B
Ninguna
34. Criterios para iniciar diálisis excepto:
Alcalosiss Metabolica

35. Hipokalemia con normotension excepto


Síndrome de liddle

36. Plasama creatinina 8mg/dl N de urea 100mg/dl U na 40 Meq/l :


Insuficiência renal

37. Factores reversibles responsables de deterioro de la funcion renal excepto:


Fibrosis intersticial

38. Causa hipokalemia con renina y aldosterona disminuida:


Licorice

39. Sugiere trastorno túbulo intersticial:


.Proteinuria mayor a 3.5 g /24 horas
.Notrmotensión
.* Edema
.Osmolarida urinaria normal

40. Glomérulonefritis rápidamente progresiva


.Engrosamiento de la membrana basal
Fusión de los pedículos de los podocitos
prolñiferacion intracapilar
proliferación extracapilar ( creo que es esa verificar)

41. Una de las siguientes causas no produce hematuria


.enfermedad de cambios mínimos

42. Plasma Na:115, Posm240 mOsm,Uosm 680,Una 60:meq/l:


.Deshidratación hipotónica
.* secreción inapropiada de ADH
.Deshidratación isotónica
.Deshidratación primaria

43. .en la falla renal crónca produce necrosis


calcifilaxia

44. No produce hipokalemia:


Pancreatitis aguda
rabdomiolisis
neoplasia
Def. Vit D

45. Paciente con glucosa 90mg/dl, Na+= 115mEq/L, Urea en 140mg/dl


a. Síndrome hipoosmolar
b. * Síndrome hiperosmolar
c. Diarrea
d. Ninguna de las anteriores.

46. Causa de seudohiponatremia con osmolaridad elevada


a. Hiperproteinemia
b. * Diabetes mellitus
c. Hiperlipidemidemia
d. Reserción transuretral prostática.

47. Manifestaciones de hipermagnesemia excepto


Normotensión

48. Piel en uremia crónica excepto


Paño blanco

49. No es causa de necrosis papilar aguda


a.Diabetes mellitas
b. Uropatía obstructiva
c. Pielonefritis
d. Trombosis de venas renal.

50. Puede presentarse hiperuricemia excepto


d. Adenocarcinoma gástrico

51. Compromete riñones y pulmones:


a. Enf de GoodPasture
b. Granulomatosis de Wegener
c. LES
d. Ay C
TODAS

52. Sugiere glomerulopatia:


-anasarca

53. TFG < 15ml/min corresponde al estadio de la enfermedad renal cronica:


* 4

54. Causa obstetrica de necrosis cortical bilateral


- desprendimiento prematuro de placenta

55. Promueve amoniogenesis en cirróticos


- hipokalemia

56. Criterio para diagnostico de síndrome nefrosico


a. Proteinuria en 24 hr de 3.5 gr para superficie corporal de 1. 73
b. Albumina serica < 3g/dl
c.Hiperlipidemia y edema
d. Todas las anteriores
e. ay b
57. Urémico crónico con potasio sérico de 6.5mEq/L con leve academia, sin trastorno muscular
y EKG normal, puede ser tratado con:
q. kayaxelate
r. gluconato de Calcio al 10%
s. NaHCO3
t. Diálisis
58. La insuficiencia renal crónica ocasiona hiperfosfatemia permanentemente cuando la tasa
de filtración glomerular esta por debajo de:
q. 60ml/min
r. 50 ml/min
s. 25 ml/min
t. 40 ml/min

59. La hipofosfatemia es causa de excepto


q. inotropismo positivo
r. disminución de 2,3 DPG
s. hemólisis
t. curva de HbO2 desplazada hacia la izquierda

60.La solución de NaCl 23,4% tiene de osmolaridad:


q. 8000 mosm/l
r. 800 mosm/l
s. 80 mosm/l
t. 0,8 mosm/l

61. En la hiperkalemia:
q. aumenta la sístole eléctrica
r. hay menor diferencia entre el potencial de reposo y el umbral
s. el potencial de reposo transmembrana se hace mas negativo
t. el potencial umbral se hace menos negativo

62. La velocidad de administración de potasio en caso de gran riesgo para la vida puede ser
de:
q. 10-20 mEq/L
r. 8 mEq/L
s. 110 mEq/L
t. 40 mEq/L

63. Un ejemplo de hiponatremia hipotónica normovolemica es:


q. síndrome nefrósico
r. SIADH
s. Diarrea
t. Cirrosis
64. A una embarazada con eclampsia se le ordena 1 litro de D/A 5% con 10g de MgSO4.
¿Cuántos ml de MgSO4 al 50% se han utilizado?
a. 20
b. 5
c. 12,5
d. 30

65. Causa hipomagnesemia:


q. estreñimiento
r. alcoholismo
s. deshidratación hipertónica
t. insuficiencia renal aguda

66. Paciente con diuresis de 300ml/día, K sérico 7mEq/L, Posm y Uosm 300mosm/L, el
diagnóstico más probable es:
q. insuficiencia suprarrenal
r. insuficiencia renal
s. SIADH
t. Deshidratación primaria

67. La mielinosis pontina se caracteriza por:


q. pares craneales intactos
r. cuadriplejía flácida
s. comportamiento normal
t. hipertonía de los miembros inferiores

68. Calcule el déficit de agua en una paciente con sodio sérico 168mEq/L y que pesa 70Kg
(utilice como porcentaje de agua 50% y sodio normal mEq/L):
q. 5L
r. 3L
s. 10L
t. 7L

69. En la SIADH la concentración de sodio urinario es de:


q. menos de 10mEq/L
r. 20mEq/L
s. 15mEq/L
t. Mas de 40mEq/L

70. En paciente con vómitos abundantes y prolongados con hipokalemia severa debe
administrarse:
q. NaCl 0.9% con KHCO3
r. NaCl 0.9% con KCl
s. NaCl 0.9% con gluconato de potasio
t. NaCl 0.9% con NaHCO3

71. La hipokalemia en un paciente con cirrosis puede contribuir:


q. coma
r. alcalosis respiratoria
s. acidosis metabolica
t. aumento de la síntesis de urea

72. El plasma de un paciente contiene Na 125mEq/L, glucosa 108mg/dl, urea 300mg/dl, podrá
presentar:
q. cirrosis
r. síndrome nefrósico
s. síntomas de hipertonicidad
t. síntomas de hipotonicidad

73. La hiponatremia aguda sintomática es debida a, excepto:


q. exceso de glucocorticoides
r. SIADH
s. post-operatorio
t. intoxicación hídrica

74. La administración de solución salina 0.9% produce:


q. disminución del volumen intracelular y aumento del extracelular
r. aumento del volumen intra y extracelular
s. aumento del volumen extracelular
t. aumento del volumen intracelular

75. Alcalosis metabólica con hipertensión arterial, hipokalemia, aldosterona normal y renina
normal:
q. Hipertensión renovascular
r. Hipertensión maligna
s. Sindroma de Cushing
t. Administración de licor

76. La hipokalemia puede predisponer, excepto:


r. intoxicación digitálica
s. disminución de la amoniogénesis
t. rabdomiolisis
u. ondas U en el EKG

77. Aldosterona aumentada, Renina dism. → Hiperaldosteronismo I


78. Limites de Hipo e Hiperkalemia
79. Px con IRC tine una taza de filtración glomerular de menor de 25 ml/min ;aunque hay
yuna tabla de las etapas en esta enfemedad crónica.
Estas osn las TFG:
1. 90 en riesgo
2. 20 ( con TFG normal o incrementada)
3. 60 -89
4. 30 – 59
5. 15-29
Menor a 15 diálisis.

80. Factor que altera el balance hídrico en el periódo post- operatorio:


Disminución de la ADH (Excepto)

81. Un ejemplo de hipokalemia con aldosterona alta y osmolaridad baja es:


Hiperaldosteronismo primario

82.Ejemplode hipernatremia hipervolémica: Hiperaldosteronismo primario

83.Hiperkalemioa son alteraciones del EKG no es necesario administrar gluconato de calcio se


colocará un antagonista de K asíe l Px no cae en paro y estoes si tiene manifestación en el EKG.

84. La fracción de excreción de sodio estará aumentada en falla cardíaca.

85.En la hiperkalemia: hay menos diferencia entre el potencial de reposo y el umbral.

Manifestaciones cardíacas. El problema médico más grave de la hiperkalemia es la


cardiotoxicidad. Los cambios en el ECG producidos por los niveles altos de potasio son bastante
constantes. A medida que aumentan los niveles se aprecian los siguientes cambios:

Ondas T picudas (con intervalo QT normal o ligeramente reducido)


Prolongación del intervalo PR con depresión de ST
Desaparición progresiva de la onda P
Bloqueo cardíaco progresivo
Arritmias ventriculares
Paro cardíaco
Las ondas T picudas constituyen el dato en el ECG más constante en la hiperkalemia.

Efectos neuromusculares. El primer signo neuromuscular de la hiperkalemia suele ser la


aparición de parestesias seguidas de debilidad progresiva de varios grupos musculares. Si el
cuadro se agrava se observa cuadriplejia fláccida. Las funciones cerebrales y de los pares
craneanos se conservan y la parálisis de la musculatura respiratoria puede ocurrir, pero es
excepcional.

86. La infusión de potasio sin riesgo para la vida del PX: es de 40 meq; porque

I lb de solución salina: debe llevr de 20 a40 mEq de KCl a una velocidad de 10 a 20 mEq por
hora; si aumenta la velocidad puede provocar un bloqueo A-V. Se recomienda una Velocidad
mayor si la hipokalemia es muy severa.

Sis e usa venas priféricas es:

10 meq en 100 cc de sol.salina para pasar en una hora

20 meq en 200 cc de sol.salina para pasar en una hora

En vena central:

20 meql en 50 cc pasarla en una hora

40 meq en 100cc pasar la en una hora

31.Los indicios de falla renal aguda son:

(Na filtrado/Na excretado) x 100%

87.Na urinario menor de 10: falla cardíaca, presentará edema estará hinchado y lo puede
llevar a una I.C.C. ( Esto lo saque de Medline: Los valores normales generalmente son de 15 a
250 mEq/L/día, dependiendo del estado de deshidratación y la ingesta diaria de sodio en la
dieta. Los rangos de los valores normales pueden variar ligeramente entre diferentes
laboratorios. Nota: mEq/L/día = miliequivalentes por litro por día.)

88.Causa de enuresis: necrosis cortical difusa.

89.En SIDH la concentración urinaria es menor de 20.

90. Deshidratación pura de agua excepto: hay salida de electrolitos del cerebro.

91.Urémico crónico con deshidratación hipertónica severa y acidótico presentará lo siguiento


excepto: osmolaridad urinaria de 8mm mosmol/kg.

92. Px con vómitos abundante, hipertensión arterial; Na urinario=6, oliguria, osmolaridad de


800 presenta: insuficiencia suprarrenal.

93. Factor que predispone a la hipernatremia son intoxicación hídrica.


94. Se presenta en acidosis tubular renal proximal excepto: NaHCO3 y lo alfa antagonistas.

95. La hipokalemia produce:


- mayor negatividad del potencial de reposo.
96. La hipokalemia en un cirrótico puede precipitar COMA por:
- aumento de la amniogénesis
97. Exceso del kayaxelate en el tratamiento de hiperkalemia puede producir:
- constipación.

98. Peligro de la hipermagnesemia severa:


- paro respiratorio.
99. Hipofosfatemia:
- aumenta el calcitriol.
100. Consecuencia metabólica de hipofosfatemia severa:
- miocardiopatía
101. Causa de hiperfosfatemia:
- insuficiencia renal
102. El Síndrome Nefrósicose caracteriza por:
- proteinuria mayor o igual de 3g y albúmina sérica menor o igual de 3g/dl.
103. En el Síndrome Nefrósico pueden encontrarse:
- cilindros céreos
104. Urémico agudo, oligúrico, acidótico, deshidratado puede presentar:
- osmolaridad urinaria de 250mosm/kg
105. La falla renal crónica avanzada cursa con:
brecha aniónica elevada
106. Cuál de los siguientes antibióticos produce nefritis intersticial:
- meticilina
107. El factor más determinante en el tratamiento de la hipokalemia renal es:
- la función cardiovascular.
108. El tratamiento de inicio de secreción inapropiada de ADH es:
- restricción hídrica
109.Causa de hiperfosfatemia:
u. Falla hepatica
v. Falla renal cronica
w. Falla cardiaca
x. Insuficiencia suprarrenal
110. Causas de hiperfosfatemia grave, excepto:
p. recuperacion de cetoacidosis diabetica.
q. Alcalosis respiratoria
r. Acidosis respiratoria.

111.Paciente con vómitos severos, oligurico con hipotensión arterial Una> 40…… 800 mOsm/Kg
puede tener:
u. Insuciencia renal aguda.
v. Insuficiencia suprarrenal
w. Falla pre-renal
x. Exceso de mineralocorticoides
112. Paciente urémico crónico con deshidratación severa puede presentar lo siguiente excepto:
a. Densidad urinaria 1010.
b. Osmolaridad urinaria de 800 mOsm/Kg
c. K sérico aumentado
d. Oliguria.
113.La hipokalemia produce:
a. Aumento de la magnitud del potencial de reposo
114. Primera*****Falla renal crónica:
a.HTA
b.Glomerulopatias
c.Diabetes Mellitus
d.Nefritis tubulo intersticial

99. Uno de los indices de falla renal aguda es:


Uosm/Tosm

100. Hormona producida en los riñones que disminuye en la falla renal cronica:
a. ADH
b. Noradrenalina
c. Peptido natriuretico (segun arjona)
d. 1,25 (OH)2 Vit D.

101. Las calcificaciones metastásicas se deben:??


a. Aumento del producto Ca, P
b. Aumento del producto Ca, Mg
c. Hipoparatiroidismo
d. Hiperparatiroidismo 1ario

102. Causa de anuria:


a. Pielonefritis unilateral
b. Necrosis tubular aguda
c. Enf cambios minimo
d. Necrosis cortical

103. la etapa de la insuficiencia renal cronica con tasa de filtración glomerular de 60-89
ml/min es la:??
a. 1
b. 2
c. 3
d. 4

104.La FENA es la relación entre:


a. Oferta tubular/Na reabsorbido x 100
b. depuración de Na/ depuración de creatinina x 100
c. oferta tubular/ Na excretado x 100.
d. Na excretado/ Na reabsorvido x 100.

105. En la nefritis tubular intersticial aguda se pierde la capacidad de poder concentrar la


orina debida a:??
a. Daño medular
b. Daño glomerular
c. Daño tubular proximal
d. Daño de la porcion cortical del tubulo colector

106.La solucion de NaCl al 23,4 % tiene:


8000 mOsm/l
107. Se asocia con el SIAD, excepto:
lipoma
108. El efecto biologico mas importante de la calcitonina es, excepto:
a. Responder a la hipocalcemia
b. Aumenta la calciuria
c. Estimular la resorcion osea osteoclastica
d. Utilizada en el tx de la hipercalcemia.

109.Favorece la entrada de K al espacio intracelular, excepto:


a. Agonista adrenergico
b. Estimulacion adrenergico
c. Insulina
d. Alcalosis metabolica

110. Factores que afectan el balance hídrico en el post-operatorio, excepto:


a. Fármacos
b. Calcitriol
c. Administración de liquidos hipotonico
d. Aumento de la ADH

111. Las primeras manifestaciones clínicas de la hiponatremia son:??


a. neurológica
b. gastrointestinal
c. pulmonares
d. musculares

112. En la hipernatremia hipertonica se da lo siguiente excepto:


a. Entrada de Na
b. Salida de aa
c. Ganancia de iones
d. Disminución del agua cerebral

113. Manifestaciones clinicas de la hipercalcemia, excepto:


a. Taquicardia
b. HTA
c. Constipación
d. Vomitos

114. En falla renal aguda por nefrotoxicidad:


a. Se conserva la membrana basal
b. Se destruye la membrana basal
c. Frecuente hay oliguria
d. La creatinina no aumenta

115. Causa de hipomagnesemia:


alcoholismo

116. Na serico de 168 mEq/l 70 kg (utilize como % de agua al 50% y Na normal 140 mEq/l
7 litros

117. Indicaciones de diálisis en falla renal aguda, excepto:


a. Encefalopatia uremica
b. Hiperkalemia refractaria
c. Ac. Metabolica refractaria
d. hipocalcemia

118. El KCL IV se puede asministrar asi, excepto:


a.infusion por linea periferica de 10 meq en 100 ml en 1hora
b. “ “ central de 20 meq em 50ml en 1 hora
c. “ “ central de 100 meq em 50ml en 1 hora
d. “ “ periferica de 20 meq en 20ml en 2 horas

119. Aumenta la excrecion urinaria de K, excepto:


a. Triamtereno
b. Bumetanida
c. Furosemida
d. Metolazona

120. Factores que regulan la secrecion distal de K, excepto:


a. Flujo tubular distal y aporte distal de Na
b. Excrecion de aniones no reabsorbibles
c. Aldosterona
d. PTH

121. Enfermo de 60 Kg con 60% de agua corporal, Posm 260 mOsm/kg, ¿Cuántos gramos
de NaCl aumentaria el Na en 10 mEq?
a. 21 gramos
17. La PTH promueve todo lo siguiente excepto:
a aumento de la absorción intestinal de calcio
b- aumento de la absorción tubulár de fosfato..
c. aumento de la resorción ósea de fosfato
d- aumento de la resorción osea de calcio
e, disminución de la reabsorción de fosfato

18. Un varon de 57 esta con hemodiálisis de mantenimiento para insuficiencia renal cronica, ¿Cuál
de las sig anomalías metabolicas podría ser anticipáda?
a. hipernatremia
b. hiponatremia
c.
d. exceso de vitamina D
e. hipoparatiroidismo

19. Un px de 25años fue ingresado a UCI por lesiones graves en cabeza y fractura de la base del
Graneo, aprox X horas después de la lesion manifestaba poliuria, la OSMU 150, y la de suero 350.
Los líquidos IV fueron detenidos y 3 hors después la producción de orina y la osm urinaria
permanecieron sin cambio. Se administraron 5 unidades de vasopresina IV, la osm de la orina se
incremento a 300. ¿Cuál es el dx mas probable?
a- Diabetes insípida central
b. diabetes insípida nefrogenica
c. intoxicación acuosa
d. sobrecarga de solutos
e. S IADH

55. indica falla renal aguda


a- U..N.. = 4.1
b- UN = 1
c. U..N.. = 1/4i/
d. Gravedad especifica 1.026
e. Ninguna

56. deshidratación por perdida de agua y densidad urinaria 1.003, creatinina 6 /


a. falla renal aguda parenquimatosa
b- falla prerenal
c. falla renal obstructiva
d. falla renal cronica
e. no hay falla renal

57. uno de los indices de falla renales


a. reabsorción de Na/oferta tubular de Na x 100
b. excresion urinaria deNa/oferta tubular de x 100
c.- oferta tubular de Na/excrecion urinaria de Na x 100 ~.--~
d- Una/Pna x 100
e. Ninguna

58. el Sdr hepatorenal es causa de falla:


a- prerrenal
b. renal
c. póstrenal
d. falla cardiaca
e. daño tubular

59. causa mas común de falla renal crónica:


A- DM

60- en la glomendonefritis aguda por infeccion, la causa, excepto


a- proteina mayor de 10 g
b- osmolaridad urinaria normal
c. hematuria
d. HT A
e. Edema

61. falla renal cronica, excepto


a. PTH aumentado
b- Hipofosfatemia
c- Acidemia
d. Hipocalcemia
e. 1.25 OH (D3) disminuida

62. EN la falla renal aguda siempre habra


a- Aumento de creatinina
b- fraccion de excrecion de Na aumentado
c- hiperkalemia
d- Una aumentado
e- Hipernatremia

63. causa de falla prerenal


a. Sdr nefrotico
b- obstrucción de ambos uréteres
c- falla cardiaca
d. Grlomerulonefritis aguda post-infeciosa
e. Calculo renal

64. en la nefritis tubular interticial se puede dar, excepto


a. h.ipernatremia
b. proteinuria de 1.5 g
c. normotension arterial
d. hipostenuria
e. hipocalcemia

65. Sugiere glomerulonefritis:


a. leucocituia
b-cilindros eritrocitarios
c- anuria
d. -

66 -causa de falla renal parenquimatosa


m) papilitis
n) calculo en pelvis derecha
o) estenosis unilateral de arteria renal
p) cistitis hemorragica
60- Causa de hipercalemia:
a Aumento del volumen arterial eficaz
b. Hipercatabolismo tisular
c Hiperaldosteronismo
d. Alcalosis Inetabolica
e. Cirrosis hepatica

61. Causa de hipomagnesemia


a. cirrosis avanzada
b. falla renal cronica
c. falla cardiaca
d. constipacion
e. acidosis respiratoria

62. Causa renal, de hipomagnesemia


a. falla renal crônica
b. esteatorreaa
c desnutricion severa
d. usó de diureticos de asa
e. infeccion urinaria

63. en caso de hipercalcemia pensar en:


a. neoplasia
b: hiperfosfatemia severa
c. nefrolitiasis
d. nefrocalcinosis
e. diuréticos

64. En el síndrome nefrosico puede encontrarse cilindros


a. cereos
b. hematicos
c. leucocitarios
d. epiteliales
e. mixtos leucocitario y hematicos,
2--En el riñón la hormona paratiroides:
Aumenta la reabsorción tubular de calcio y disminuye la de fósforo

3- La hipercalcemia produce
Potencialización de los efectos de la hipercalemia

36- La hipomagnesemia produce.


Vasodilatación periférica

36-De los factores: nefrotoxicidad./ -La buena hidratación

37 - En la concentración sérica para calcular:: 8,8

38- la velocidad de administración de potasio: 40 meq

39-hipocalcemia produce : fosfaturia


18- La hipofosfatemia cansa excepto: aumento de vitamina D
20. La renal aguda, oligurica todo es cierto excepto: Osmolaridad urinaria aumentada
21 Paciente can alcalosis metabolica e hipokalemia debe ser tratada con KCL

24. Usted tiene una paciente con hiperhemesis gravidica. El trastorno metabolico que espera
encontrar es:
A - acidosis hipocloremica
b, acidosis hipercloremica'
c- alcalosis hipocloremica
d. alcalosis hipercloremica
e. no hay alteración acido base

25. Cual de las siguientes opciones esta contraindicada en el tratamiento de la hiperkalcmia


a, gluconato de calcio
b. aldactone
c, kayaxelate
d. Bicarbonato de sodio e. Todas las anteriores

43. Rotando en sala de medicina interna le piden que evalue un paciente con HIV el cual
porpresentar una meningitis por criptococci recibe anfotericina B; dicho este medicamento
buscaría como complicación:
a. aparicion de diarrea
b. aumento de la presion arterial
c- hipokalemia
d . cefalea
e. dolor toráxico

81- donde se halla el calcio en el organismo:


a- 50% sangre
b- 20 % proteínas
c- 99% hueso
2- cual de los siguientes datos refleja vieja gravedad
a- distension abdominal
b- hipocalcemia
c- hiperamilasemia
d- aumento de la intensidad de dolor
4- en la hipercalemia
a- hay una menor diferencia entre el potencial de reposo y el umbral

5—la hipercalemia en un cirrótico puede desencadenar


a- coma hepatico

6- la hipermagnesemia puede producir


a- paro respiratorio

7- tx de hipercalcemia
a- SSN 0.9 %

8- la hipofosfatemia son causas excepto:


a- aumento de PTH
b- aumento de la absorción intestinal de fosfato
c- disminuye fosfatemia
d- aumento de vitamina D

9- una solucion isotonica


a- aumenta el volumen extracell

10- la vel de administración de potasio en caso de riesgo para la vida


a—40 meq por hora

11- Px cronico con deshidratación severa puede presentar los siguiente excepto:
a-osmolaridad urinaria de 800 mosmol/Kg

12- Px con vomito abundante, HTA, sodio urinario en 6, oligurico, Osmolaridad de 800
presente,
a- insuficiencia suprarrenal

13- Px uremico cronico con vomito severo hipotension arterial orina al azar, muestra
sodio mas de 40, Osmolaridad de 800 puede tener :
a- Falla suprarrenal

14- En quien predomina la enfermedad de higado graso:


A- Sexo masculino

24- La hipocalemia predispone excepto:


c- amoniogenesis

25- Cual de las siguientes es causa de hiperfosfatemia


c- insuficiencia renal

26- en la glomerulonefritis aguda post infecciosa disminuye la tasa de filtración:


c- por alteración en el coeficiente de ultrafiltracion

27- causa de insuficiencia prerenal excpto:.


c- Aneurisma disecante de la aorta
d- Cuándo disminuye el fosfato?
R. Aumento de PTH
e- Ejemplo de hipernatremia hipervolémica:
R. Aldosteronismo primario

f- La hipocalemia produce:
R. Aumento de la magnitud del potencial de reposo

g- La depleción de agua produce, excepto:


R. Salida de electrolitos del cerebro

h- Velocidad de administración de KCl en caso de urgencia:


R. 40 mEq/h

i- Cambios en EKG de la hipermagnesemia es igual:


R. Hipercalemia

j- Enfermo de 60 Kg con 60% de agua corporal, osmolaridad plasmática 260


mosmol/Kg, ¿cuántos gramos de NaCl aumenta el Na en 10 mEq en 24 horas?
R. a) 15
b) 21
c) 30
d) 19

k- Uso de Calcio en tratamiento de hipercalemia para:


R. Modificar el Potencial de Membrana

l- Hipermagnesemia severa produce:


R. Paro respiratorio

m- Venoclisis de 1L de D/A 5% conteniendo 10 mg de MgSO4 y al 50%:


R. 20mL

n- La hipocalcemia:
R. Prolongo QT a expensas del segmento ST

o- Tratamiento de urgencia de hipercalcemia:


R. SSN 0.9%

p- Causa de hiperfosfatemia:
R. Insuficiencia renal

q- Adaptación cerebral, hipertonicidad, excepto:


R. Pérdida de iones

r- En la adaptación cerebral, la hipotensión da lo siguiente, excepto:


R. Ganancia de aminoácidos
s- En ausencia de síntomas de hipernatremia, el sodio debe ser disminuido a razón de:
R. 0.5 mEq/L/h

t- Para corregir un sodio cerca de 185 mEq se administró en 86 horas, 66 letras de D/SS
5%. ¿cuál es la velocidad de administración de la venoclisis?
R. 69 mL ¿?????
u- Manifestación clínica de hipercalcemia, excepto:
R. a) Hipotensión arterial
b) Bradicardia
c) Nauseas
d) Anorexia

v- Consecuencia de hipofosfatemia, excepto:


R. a) No afecta al SNC
b) Disfunción plaquetaria
c) Cardiomiopatía
d) Osteomalacia

w- Causa de hipofosfatemia, excepto:


R. a) Falla renal
b) Recuperación de cetoacidosis diabética
c) Alcalosis respiratoria
d) Quemaduras graves

x- Causa de HTA con hipocalemia, aldosterona-renina baja:


R. Síndrome de Cushing

y- Causas de HTA con hipocalemia y renina alta:


R. Hipertensión maligna

z- En una deshidratación severa hipertónica con hipotensión arterial severa, la prioridad


inicial es administrar:
R. SSN 0.9%

aa- En la secreción inapropiada de ADH se da lo siguiente, excepto:


R. No hay expansión de volumen

bb- Tejido con mayor contenido de K+:


R. a) Hígado
b) Músculo
c) Eritrocito
d) Hueso
La PTH promueve todo lo siguiente excepto:
Aumento de la absorción tubular de fosfato

8 Varón de 57a esta con hemodiálisis de mantenimiento para insuficiencia renal


crónica. Cual de las siguientes anormalidades metabólicas podría ser anticipada?
Osteomalacia
50 S Nefrosico:
Cilindros cereos

51 Falla renal crónica avanzada:


Brecha aniónica elevada
24. Usted tiene una paciente con hiperemesis gravidica. El trastorno metabolico que espera
encontrar es:
f. acidosis hipocloremica
g. acidosis hipercloremica
h. alcalosis hipocloremica
i. alcalosis hipercloremica
j. no hay alteración acido base

25. Cual de las siguientes opciones esta contraindicada en el tratamiento de la hiperkalemia


k. gluconato de calcio
l. aldactone
m. kayaxelate
n. Bicarbonato de sodio
o. Todas las anteriores
59. Paciente de 84 anos quien recibe diuretico tiazidico para la hipertensión arterial es
admitido por presentar diarrea y trastornos del sensorio. Tiene disminución del turgor
de la piel y la presion arterial es normal, Pna 174mEq/L, Una 5mEq/L, Uosm
606mEq/kg. La hipertensión se debe a:
k. perdidas insensibles
l. polidipsia
m. uso de diuretico
n. diabetes insipida
o. secrecion inapropiada de hormona antidiuretica

60. Hipernatremia Hipervolemica


k. diuresis osmótica
l. diarrea
m. aldosteronismo primario
n. diabetes insipida
o. SIADH

61. La hipokalemia en un cirrotico puede precipitar COMA por:


f. Hipermagnesemia asociada
g. Aumento de la amoniogenesis
h. Hiperfosfatemia asociada
i. Rabdomiolisis
j. Induce vomitos
62. Causa de hiperkalemia:
k. Aumento del volumen arterial eficaz
l. Hipercatabolismo tisular
m. Hiperaldosteronismo
n. Alcalosis metabolica
o. Cirrosis hepatica

63. Causa de hipomagnesemia


k. cirrosis avanzada
l. falla renal cronica
m. falla cardiaca
n. constipacion
o. acidosis respiratoria

64. Causa renal de hipomagnesemia


k. falla renal cronica
l. esteatorrea
m. desnutricion severa
n. uso de diureticos de asa
o. infeccion urinaria

65. en caso de hipercalcemia pensar en:


k. neoplasia
l. hiperfosfatemia severa
m. nefrolitiasis
n. nefrocalcinosis
o. diureticos

66. En el síndrome nefrosico puede encontrarse cilindros:


k. cereos
l. hematicos
m. leucocitarios
n. epiteliales
mixtos leucocitario y hematic
31) En hiponatremia el daño principal es:

a) Neurológica
b) Respiratoria
c) Renal

32) Adaptación del cerebro ante solución hipotónica:

a) Pérdida de iones
b) Pérdida de aminoácidos
c) Edema cerebral
d) Ganancia de iones
11.habia una de renal de un man q estaba en UCI con fractura de base de craneo:
Diabetes insidia central
25.En el riñon la PTH
Aumenta la reabsorción tubular de Ca y disminuye la de fosforo

122. Causa de hiperfosfatemia:


y. Falla hepatica
z. Falla renal cronica
aa. Falla cardiaca
bb. Insuficiencia suprarrenal

123. Causas de hiperfosfatemia grave, excepto:


s. recuperacion de cetoacidosis diabetica.
t. Alcalosis respiratoria
u. Acidosis respiratoria.

124. Paciente con vómitos severos, oligurico con hipotensión arterial Una> 40…… 800
mOsm/Kg puede tener:
y. Insuciencia renal aguda.
z. Insuficiencia suprarrenal
aa. Falla pre-renal
bb. Exceso de mineralocorticoides

125. Paciente urémico crónico con deshidratación severa puede presentar lo siguiente
excepto:
a. Densidad urinaria 1010.
b. Osmolaridad urinaria de 800 mOsm/Kg
c. K sérico aumentado
d. Oliguria.

126. La hipokalemia produce:


a. Aumento de la magnitud del potencial de reposo
127. Uno de los indices de falla renal aguda es:
Uosm/Tosm

128. Hormona producida en los riñones que disminuye en la falla renal cronica:
a. ADH
b. Noradrenalina
c. Peptido natriuretico (segun arjona)
1,25 (OH)2 Vit D.
129. Causa de anuria:
a. Pielonefritis unilateral
b. Necrosis tubular aguda
c. Enf cambios minimo
d. Necrosis cortical

130. la etapa de la insuficiencia renal cronica con tasa de filtración glomerular de 60-89
ml/min es la:??
a. 1
b. 2
c. 3
d. 4

131. La FENA es la relación entre:


a. Oferta tubular/Na reabsorbido x 100
b. depuración de Na/ depuración de creatinina x 100
c. oferta tubular/ Na excretado x 100.
d. Na excretado/ Na reabsorvido x 100.
132. En la nefritis tubular intersticial aguda se pierde la capacidad de poder concentrar la
orina debida a:??
a. Daño medular
b. Daño glomerular
c. Daño tubular proximal
d. Daño de la porcion cortical del tubulo colector

133. La solucion de NaCl al 23,4 % tiene:


8000 mOsm/l

134. Se asocia con el SIAD, excepto:


lipoma

135. El efecto biologico mas importante de la calcitonina es, excepto:


a. Responder a la hipocalcemia
b. Aumenta la calciuria
c. Estimular la resorcion osea osteoclastica
d. Utilizada en el tx de la hipercalcemia.

136. Favorece la entrada de K al espacio intracelular, excepto:


a. Agonista adrenergico
b. Estimulacion adrenergico
c. Insulina
d. Alcalosis metabolica

137. Factores que afectan el balance hídrico en el post-operatorio, excepto:


a. Fármacos
b. Calcitriol
c. Administración de liquidos hipotonico
d. Aumento de la ADH

138. Las primeras manifestaciones clínicas de la hiponatremia son:??


a. neurológica
b. gastrointestinal
c. pulmonares
d. musculares

139. En la hipernatremia hipertonica se da lo siguiente excepto:


a. Entrada de Na
b. Salida de aa
c. Ganancia de iones
d. Disminución del agua cerebral

140.Manifestaciones clinicas de la hipercalcemia, excepto:


a. Taquicardia
b. HTA
c. Constipación
d. Vomitos

141. En falla renal aguda por nefrotoxicidad:


a. Se conserva la membrana basal
b. Se destruye la membrana basal
c. Frecuente hay oliguria
d. La creatinina no aumenta

142. Causa de hipomagnesemia:


alcoholismo

143. Na serico de 168 mEq/l 70 kg (utilize como % de agua al 50% y Na normal 140 mEq/l
7 litros

144. Indicaciones de diálisis en falla renal aguda, excepto:


a. Encefalopatia uremica
b. Hiperkalemia refractaria
c. Ac. Metabolica refractaria
d. hipocalcemia

145. El KCL IV se puede asministrar asi, excepto:


a.infusion por linea periferica de 10 meq en 100 ml en 1hora
b. “ “ central de 20 meq em 50ml en 1 hora
c. “ “ central de 100 meq em 50ml en 1 hora
d. “ “ periferica de 20 meq en 20ml en 2 horas

146. Aumenta la excrecion urinaria de K, excepto:


a. Triamtereno
b. Bumetanida
c. Furosemida
d. Metolazona

147. Factores que regulan la secrecion distal de K, excepto:


a. Flujo tubular distal y aporte distal de Na
b. Excrecion de aniones no reabsorbibles
c. Aldosterona
d. PTH

148. Enfermo de 60 Kg con 60% de agua corporal, Posm 260 mOsm/kg, ¿Cuántos gramos
de NaCl aumentaria el Na en 10 mEq?
a. 21 gramos

19. Un px de 25años fue ingresado a UCI por lesiones graves en cabeza y fractura de la base del
Graneo, aprox X horas después de la lesion manifestaba poliuria, la OSMU 150, y la de suero 350.
Los líquidos IV fueron detenidos y 3 hors después la producción de orina y la osm urinaria
permanecieron sin cambio. Se administraron 5 unidades de vasopresina IV, la osm de la orina se
incremento a 300. ¿Cuál es el dx mas probable?
a- Diabetes insípida central
b. diabetes insípida nefrogenica
c. intoxicación acuosa
d. sobrecarga de solutos
e. S IADH

55. indica falla renal aguda


a- U..N.. = 4.1
b- UN = 1
c. U..N.. = 1/4i/
d. Gravedad especifica 1.026
e. Ninguna

56. deshidratación por perdida de agua y densidad urinaria 1.003, creatinina 6 /


a. falla renal aguda parenquimatosa
b- falla prerenal
c. falla renal obstructiva
d. falla renal cronica
e. no hay falla renal

57. uno de los indices de falla renales


a. reabsorción de Na/oferta tubular de Na x 100
b. excresion urinaria deNa/oferta tubular de x 100
c.- oferta tubular de Na/excrecion urinaria de Na x 100 ~.--~
d- Una/Pna x 100
e. Ninguna

58. el Sdr hepatorenal es causa de falla:


a- prerrenal
b. renal
c. póstrenal
d. falla cardiaca
e. daño tubular

59. causa mas común de falla renal crónica:


A- DM

60- en la glomendonefritis aguda por infeccion, la causa, excepto


a- proteina mayor de 10 g
b- osmolaridad urinaria normal
c. hematuria
d. HT A
e. Edema

61. falla renal cronica, excepto


a. PTH aumentado
b- Hipofosfatemia
c- Acidemia
d. Hipocalcemia
e. 1.25 OH (D3) disminuida

62. EN la falla renal aguda siempre habra


a- Aumento de creatinina
b- fraccion de excrecion de Na aumentado
c- hiperkalemia
d- Una aumentado
e- Hipernatremia

63. causa de falla prerenal


a. Sdr nefrotico
b- obstrucción de ambos uréteres
c- falla cardiaca
d. Grlomerulonefritis aguda post-infeciosa
e. Calculo renal

64. en la nefritis tubular interticial se puede dar, excepto


a. h.ipernatremia
b. proteinuria de 1.5 g
c. normotension arterial
d. hipostenuria
e. hipocalcemia

65. Sugiere glomerulonefritis:


a. leucocituia
b-cilindros eritrocitarios
c- anuria
d. -

66 -causa de falla renal parenquimatosa


q) papilitis
r) calculo en pelvis derecha
s) estenosis unilateral de arteria renal
t) cistitis hemorragica

98 – Hep C periodo de incubación:


a- 50 dias

60- Causa de hipercalemia:


a Aumento del volumen arterial eficaz
b. Hipercatabolismo tisular
c Hiperaldosteronismo
d. Alcalosis Inetabolica
e. Cirrosis hepatica

61. Causa de hipomagnesemia


a. cirrosis avanzada
b. falla renal cronica
c. falla cardiaca
d. constipacion
e. acidosis respiratoria

62. Causa renal, de hipomagnesemia


a. falla renal crônica
b. esteatorreaa
c desnutricion severa
d. usó de diureticos de asa
e. infeccion urinaria

63. en caso de hipercalcemia pensar en:


a. neoplasia
b: hiperfosfatemia severa
c. nefrolitiasis
d. nefrocalcinosis
e. diuréticos
64. En el síndrome nefrosico puede encontrarse cilindros
a. cereos
b. hematicos
c. leucocitarios
d. epiteliales
e. mixtos leucocitario y hematicos,
11- Px cronico con deshidratación severa puede presentar los siguiente excepto:
a-osmolaridad urinaria de 800 mosmol/Kg

12- Px con vomito abundante, HTA, sodio urina rio en 6, oligurico, Osmolaridad de 800
presente,
a- insuficiencia suprarrenal

13- Px uremico cronico con vomito severo hipotension arterial orina al azar, muestra
sodio mas de 40, Osmolaridad de 800 puede tener :
a- Falla suprarrenal

14- En quien predomina la enfermedad de higado graso:


A- Sexo masculino

28- La hipocalemia predispone excepto:


d- amoniogenesis

29- Cual de las siguientes es causa de hiperfosfatemia


d- insuficiencia renal

30- en la glomerulonefritis aguda post infecciosa disminuye la tasa de filtración:


d- por alteración en el coeficiente de ultrafiltracion

31- causa de insuficiencia prerenal excpto:.


cc- Aneurisma disecante de la aorta
Urémico crónico con potasio sérico de 6.5mEq/L con leve academia, sin trastorno muscular y
EKG normal, puede ser tratado con:
u. kayaxelate
v. gluconato de Calcio al 10%
w. NaHCO3
x. Diálisis

La insuficiencia renal crónica ocasiona hiperfosfatemia permanentemente cuando la tasa de


filtración glomerular esta por debajo de:
u. 60ml/min
v. 50 ml/min
w. 25 ml/min
x. 40 ml/min

La hipofosfatemia es causa de excepto


u. inotropismo positivo
v. disminución de 2,3 DPG
w. hemólisis
x. curva de HbO2 desplazada hacia la izquierda

La solución de NaCl 23,4% tiene de osmolaridad:


u. 8000 mosm/l
v. 800 mosm/l
w. 80 mosm/l
x. 0,8 mosm/l

En la hiperkalemia:
u. aumenta la sístole eléctrica
v. hay menor diferencia entre el potencial de reposo y el umbral
w. el potencial de reposo transmembrana se hace mas negativo
x. el potencial umbral se hace menos negativo

La velocidad de administración de potasio en caso de gran riesgo para la vida puede ser de:
u. 10-20 mEq/L
v. 8 mEq/L
w. 110 mEq/L
x. 40 mEq/L

Un ejemplo de hiponatremia hipotónica normovolemica es:


u. síndrome nefrósico
v. SIADH
w. Diarrea
x. Cirrosis

A una embarazada con eclampsia se le ordena 1 litro de D/A 5% con 10g de MgSO4. ¿Cuántos
ml de MgSO4 al 50% se han utilizado?
a. 20
b. 5
c. 12,5
d. 30

Causa hipomagnesemia:
u. estreñimiento
v. alcoholismo
w. deshidratación hipertónica
x. insuficiencia renal aguda

Paciente con diuresis de 300ml/día, K sérico 7mEq/L, Posm y Uosm 300mosm/L, el diagnóstico
más probable es:
u. insuficiencia suprarrenal
v. insuficiencia renal
w. SIADH
x. Deshidratación primaria

La mielinosis pontina se caracteriza por:


u. pares craneales intactos
v. cuadriplejía flácida
w. comportamiento normal
x. hipertonía de los miembros inferiores

Calcule el déficit de agua en una paciente con sodio sérico 168mEq/L y que pesa 70Kg (utilice
como porcentaje de agua 50% y sodio normal mEq/L):
u. 5L
v. 3L
w. 10L
x. 7L

En la SIADH la concentración de sodio urinario es de:


u. menos de 10mEq/L
v. 20mEq/L
w. 15mEq/L
x. Mas de 40mEq/L

En paciente con vómitos abundantes y prolongados con hipokalemia severa debe


administrarse:
u. NaCl 0.9% con KHCO3
v. NaCl 0.9% con KCl
w. NaCl 0.9% con gluconato de potasio
x. NaCl 0.9% con NaHCO3

La hipokalemia en un paciente con cirrosis puede contribuir:


u. coma
v. alcalosis respiratoria
w. acidosis metabolica
x. aumento de la síntesis de urea

El plasma de un paciente contiene Na 125mEq/L, glucosa 108mg/dl, urea 300mg/dl, podrá


presentar:
u. cirrosis
v. síndrome nefrósico
w. síntomas de hipertonicidad
x. síntomas de hipotonicidad

La hiponatremia aguda sintomática es debida a, excepto:


u. exceso de glucocorticoides
v. SIADH
w. post-operatorio
x. intoxicación hídrica

La administración de solución salina 0.9% produce:


u. disminución del volumen intracelular y aumento del extracelular
v. aumento del volumen intra y extracelular
w. aumento del volumen extracelular
x. aumento del volumen intracelular

Alcalosis metabólica con hipertensión arterial, hipokalemia, aldosterona normal y renina


normal:
u. Hipertensión renovascular
v. Hipertensión maligna
w. Sindroma de Cushing
x. Administración de licor

La hipokalemia puede predisponer, excepto:


v. intoxicación digitálica
w. disminución de la amoniogénesis
x. rabdomiolisis
y. ondas U en el EKG
Plasma: creatinina 1mg/dl,140 mEq/L. Orina: creatinina 150 mg/dl, Na 40 mEq/L la FE Na es:
b) 0.19%

Tratamiento de inicio de SIADH es:


a) restricción hidrica

24. U Na 115, Uosm 240. P osm 680, P Na 62


SIADH

25. Paciente de 84 años que recibe diureticos para la hipertensión arterial. Presenta
diarrea y trastornos del sensorio y disminución del rubor de la piel. Na 174 U
Na 5 U osm 606. La hipernatremia se debe a:
Perdidas insensibles

26. hipernatremia hipervolemica


Aldosteronismo primario

27. causa de hipomagnesemia


Alcoholismo

28. hipocalcemia:
Provoca potencializacion de efectos de hiperkalemia

29. En hipocalcemia
Prolonga QT a expensas de segmento ST

30. Tratamiento de urgencia hipercalcemica


Solucion salina isotonica 0.9% 4 a 5 L

31. Síndrome nefrotico


Edema, proteinuria, hipertensión arterial, hematuria
7) Cuál de las siguientes medicinas pueden dar hiperkalemia
ee) inhibidor de la enzima convertidota
ff) bloqueador alfa adrenérgico
gg) bloqueador beta adrenérgico
hh) a y b
ii) a y c

2) Hormonas que disminuyen en la falla renal crónica, excepto


a) 1,25 dihidroxi vitatamina D
b) eritropoyetina
c) paratohormona
d) foliculo estimulante

3) MgSO4 (PM 120) 1 gramo contiene de magnesio en meq


a) 10
b) 7.9
c) 8.3
d) 6

4) Factores que conducen a la progresión de falla renal crónica


a) hiperlipidemia
b) toxinas urémicas
c) proteinuria
d) b y c
c) todas

5) Paciente de 20 años presenta abundantes vômitos y diarrea de 3 días de evolución,


examen físico depleción de volumen, suero muestra Na 155, K 3meq/l, Cl 117meq/l, HCO3 25
meq/l, tratamiento de elección:
a) D/A 5% + KCl
b) solución salina 3% + KCl
c) solución salina 23.4% + KCl
d) solución salina 0.45% + KCl

6) En la enfermedad ateroembólica renal se da lo siguiente excepto:


d) Velocidad de eritrosedimentación

7) Pseudohiperkalemia de 8 mEq/l que se ve en el EKG:


Onda T picuda
Aplanamiento de la onda P
Ay B
Ninguna

8)Criterios para iniciar diálisis excepto:


Alcalosiss Metabolica

9)Hipokalemia con normotension excepto


Síndrome de liddle

10) Plasama creatinina 8mg/dl N de urea 100mg/dl U na 40 Meq/l :


Insuficiência renal

11) Factores reversibles responsables de deterioro de la funcion renal excepto:


Fibrosis intersticial

12) Causa hipokalemia con renina y aldosterona disminuida:


Licorice

13. Sugiere trastorno túbulo intersticial:


.Proteinuria mayor a 3.5 g /24 horas
.Notrmotensión
.Edema
.Osmolarida urinaria normal

14. Glomérulonefritis rápidamente progresiva


.Engrosamiento de la membrana basal
Fusión de los pedículos de los podocitos
prolñiferacion intracapilar
proliferación extracapilar ( creo que es esa verificar)

15.Una de las siguientes causas no produce hematuria


.enfermedad de cambios mínimos

16.Plasma Na:115, Posm240 mOsm,Uosm 680,Una 60:meq/l:


.Deshidratación hipotónica
.secreción inapropiada de ADH
.Deshidratación isotónica
.Deshidratación primaria

17.en la falla renal crónca produce necrosis


calcifilaxia

18. No produce hipokalemia:


Pancreatitis aguda
rabdomiolisis
neoplasia
Def. Vit D

19. Paciente con glucosa 90mg/dl, Na+= 115mEq/L, Urea en 140mg/dl


a. Síndrome hipoosmolar
b. Síndrome hiperosmolar
c. Diarrea
d. Ninguna de las anteriores.

20. Causa de seudohiponatremia con osmolaridad elevada


a. Hiperproteinemia
b. Diabetes mellitus
c. Hiperlipidemidemia
d. Reserción transuretral prostática.

21. Manifestaciones de hipermagnesemia excepto


Normotensión

22. Piel en uremia crónica excepto


Paño blanco

23. No es causa de necrosis papilar aguda


a.Diabetes mellitas
b. Uropatía obstructiva
c. Pielonefritis
d. Trombosis de venas renal.

24. Puede presentarse hiperuricemia excepto


d. Adenocarcinoma gástrico

25. Compromete riñones y pulmones:


a. Enf de GoodPasture
b. Granulomatosis de Wegener
c. LES
d. Ay C

26. Sugiere glomerulopatia:


-anasarca

27. TFG < 15ml/min corresponde al estadio de la enfermedad renal cronica:


* 4

28. Causa obstetrica de necrosis cortical bilateral


- desprendimiento prematuro de placenta

29. Promueve amoniogenesis em cirróticos


- hipokalemia

30. Criterio para diagnostico de síndrome nefrosico


a. Proteinuria en 24 hr de 3.5 gr para superficie corporal de 1. 73
b. Albumina serica < 3g/dl
c.Hiperlipidemia y edema
d. Todas las anteriores
e. ay b
24. Factor que altera el balance hídrico en el periódo post- operatorio:
Disminución de la ADH (Excepto)

25. Un ejemplo de hipokalemia con aldosterona alta y osmolaridad baja es:


Hiperaldosteronismo primario

26.Ejemplode hipernatremia hipervolémica: Hiperaldosteronismo primario

27.Hiperkalemioa son alteraciones del EKG no es necesario administrar gluconato de calcio se


colocará un antagonista de K asíe l Px no cae en paro y estoes si tiene manifestación en el EKG.

28. La fracción de excreción de sodio estará aumentada en falla cardíaca.

29.En la hiperkalemia: hay menos diferencia entre el potencial de reposo y el umbral.

30. La infusión de potasio sin riesgo para la vida del PX: es de 40 meq; porque

I lb de solución salina: debe llevr de 20 a40 mEq de KCl a una velocidad de 10 a 20 mEq por
hora; si aumenta la velocidad puede provocar un bloqueo A-V. Se recomienda una Velocidad
mayor si la hipokalemia es muy severa.

Sis e usa venas priféricas es:

10 meq en 100 cc de sol.salina para pasar en una hora

20 meq en 200 cc de sol.salina para pasar en una hora

En vena central:

20 meql en 50 cc pasarla en una hora

40 meq en 100cc pasar la en una hora

31.Los indicios de falla renal aguda son:

(Na filtrado/Na excretado) x 100%

33.Causa de enuresis: necrosis cortical difusa.


34.En SIDH la concentración urinaria es menor de 20.

35. Deshidratación pura de agua excepto: hay salida de electrolitos del cerebro.

36.Urémico crónico con deshidratación hipertónica severa y acidótico presentará lo siguiento


excepto: osmolaridad urinaria de 8mm mosmol/kg.

37. Px con vómitos abundante, hipertensión arterial; Na urinario=6, oliguria, osmolaridad de


800 presenta: insuficiencia suprarrenal.

38. Factor que predispone a la hipernatremia son intoxicación hídrica.

39. Se presenta en acidosis tubular renal proximal excepto: NaHCO3 y lo alfa antagonistas.
3 Parcial de MEDICINA INTERNA
Renal y Gastro
ENERO 2006

Semestral de MI – Nov 2007

PREGUNTAS DE INFECTO

1. Mujer con rigidez nucal, signos de Brudzinski. Hace una semana se le realizo una endoscopía. En el
cultivo de LCR no se presentó presencia de bacterias. PA (presin de apertura) fue 27 cm, Leu 154,
Glcsa 46.

R/ Pensamos en una parotiditis respuesta C, pero al ver el semstral taba mala asique no era esa y tampoco la
de respuesta q hablaba de dar profilaxis a la familia .asique .dice lesli q era la D.

2. Neumonía de la comunidad,, presencia de anerobios, G+ y G-, NO es nosocomial.(tb se usa en


diverticulitis abdominal)

R/ ETARPENEM

3. Caso clinico de nocardiosis. El tratamiento indicado es:

R/ El tratamiento de elección es la sulfonamida ( o el TMP-SMX) y reciente mente los B-lactamicos.

4. Caso de TBC: mujer de 33 años de edad, se le hace lectura de mantoux. ELLa vive con una prima que
se le ha diagnosticado tuberculosis pulmonar , 50 bacilos por campo. La mujer de 33 años tienes una
PPD de 10 mm.

R/ El PPD no es significativo, ya que al tratarse de una persona inmigrante de una zona de alta prevalencia
de TBC deberia tener una induracion a mayor de 15mm.

5. Paciente con cefalea intensa, fiebre continua, vomitos y refiere q no puede movilizar el hemicuerpo
derecho . Esta hablando incoherencia, tiene rigidez nucal, tiene hemiparesia braquio crural derecha.
Antecedentes endoscopia por epigastralgia.Abordaje??

R/CAT SCAN CEREBRAL

6. Si usted cree q es una infeccion del SNC lo mas seguro es q fue adquirido en la sig forma

R/hematogena y distribución a nivel de la cerebral media.

7. Era un caso de dengue ,donde en cual de las siguintes maniefestaciones lo hospitalizas??

a.abdominal
b.mialgia severa
c.fiebre

8. La penicilina actua contra los siguientes microbios excepto:

a.neumococo (yo creo )


b.serratia
c. listeria. Solo actuan las penicilinas de espectro ampliado
1
d.clostridio

9. Trtamiento pa la legionella:

R/ MACROLIDOS

10. Policia del darien. Con fiebre y con palpacion del bazo

R/ bueno yo creo q el man tieen malaria asiq yo escogi frotis de eritrocitos en sangre .

11. Amigos se reunen y comen un asado

R/ yo creo q era salmonella porq ella ta en las carnes semicrudas y mas en el pollo. Tambien puede ser E.
coli

12. CD4 menor de 200 con candidiasis oral


a.histoplasma
b.toxoplasma
c.peneumocistis
d.N/A

13. menor de 50
R/ Micobacterium avium

14. menor de 300_Candidiasis oral (Muguet)

15.Un man d 16 a q viajo por Europa al mes de regreso tiene malestar general, odinofagia,fiebre.. Al ex
fisico se destaca hipertrofia de amigdala con exudado blanquecino, adenopatias occipitales, laterocervicales
dolorosas. En el hemograma se muestra algunos leucocitos atipicos. Ante la sospecha dx se raliza:
R/ esta esta en el MIR pero no me acuerdo de q año por eso no se cual respuesta es
a.Biposia ganglionar Ó
b.Serologia para virus Epstein Barr

Sem MI – Nov 2007

Contestarlas…

1) Paciente presenta cefalea, fiebre de 390C , Kernig, Brudsinsky, rigidez nucal. Se hace punción
lumbar, se encuentran proteínas elevadas , glucosa disminuida. En el frotis se ven leucocitos
aumentados, no se identifica microorganismo. (El prof. Dice que era una meningitis viral) Usted:

a) Le dice a los familiares que no se preocupen, le puede dar antibióticos logrando recuperación completa.
b) Le da profilaxis a toda la familia.

2) Paciente llega con artralgia, fiebre, leve inflamación de rodilla izquierda. Usted hace el
diagnóstico de lo que tiene con:

a) Punción sinovial
b) Frotis sanguíneo

3)Imipenem se da con cilastatina para:

2
a) La cilastatina tiene sinergismo con él
b) La cilastatina lo ayuda en su metabolismo hepático. Es para la difuncion renal
c) La cilastatina inhibe la B-lactamasa para que imipenem funcione mejor. Cilastina inhibe enzima
dihidropeptidasa en el riñon para la degradación del imipenem
d) N/A

4)Paciente presenta diarrea sanguinolenta, usted piensa en todo, menos en:

a) Colitis ulcerativa
b) Enfermedad de Crohn
c) Síndrome de colon irritable

5) Paciente con VIH , usted le da tratamiento si CD4 es menor de :

a) 500
b) 450
c) 300
d) 350
e) 200

6) Paciente con CD4 <200 y Mycobacterium avium tienen celularidad:

a)200
b)100
c)50

3
7) Considera que hay osteoporosis cuando hay pérdida de masa ósea de:

a)-0.5
b)-1.0
c)-1.5
d)-2.0
e)-2.5

8) Paciente en que describen un montón de síntomas , nódulos pulmonares y dice el caso que se trata de
Nocardiosis. Se trata con:

a) Penicilina
b) Tobramicina

9) Penicilina sirve contra todos menos:

a) Clostridium
b) Leptospira
c) Actinomyces
d) Creo que era el nombre de un hongo

67) Paciente con SIDA , CD4 disminuído, le doy tratamiento profiláctico contra:

a) Pneumocystis carinii
b) M. avium
c) Toxoplasmosis
d) Histoplasma
e) Ninguna de las anteriores

25.el virus de la gripe aviar actual que no ha mutado y que no se contagia actualamente en humanos es:
R H5N1

26.-29. Nombre 4 floxacina que conozca:


a..ofloxacina
b..ciprofloxacina
c..levofloxacina
d ..gatifloxacina

30-33.efecto secundario principal de los sig antibioticos:


a rifampicina : hepatotoxico
b.imipenenm: convulsión
c. anfotericina B: hipokalemia
d.ketoconazol: cefalea, mareo, fotofobia, dif hepatica,
e.fluconazol: alopecia, nausea vomito

34.antimicrobiano usado en cualquier trimestre del embarazo:


R/ betalactamico

35.Px con Dx de diarrea del viajero, formulo tratamiento por:


R/ 3 dias

36.No es una condicion de uso de antimicrobiano parenteral:


R/ pobre penetración al sist nerv central

37.profilaxis para contactos de px con meningococo, se realiza con rifampicina o :


4
R/ ceftriaxone

38.mujer de 25 años de vida sexual activa, síntomas de cistitis, orina muestra piuria sin bacteuria, pienso en:
R/infeccion por chlamidia

39-40. Bacterias q infectan a px esplenectomizados:


a)haemophilus influenzae
b)estreptococco pneumonia

41.px de 20 años con sospecha de IVU Ph de orina de 8, la bacteria seria:


R/ proteus

42.cual de las sig es incorrecta con el consumo de sustancias


R/ el concepto de tolerancia a una sust hace referencia al hecho de que el consumo continuado se precisan
cada vez dosis menores para producir el mismo efecto

50.mujer de 32 años embarazada, alérgica a la penicilina , acude en busca de resultados de analítica del
primer trimestre , se evidencia bacteriuria, px asintomatica que accion terapeutica aconseja:
R/nitrofurantoina 100mg durante 3 a 7 dias , cultivo urinario mensual hasta final de la gestacion

1. Un paciente masculino de 56 años es admitido con un cuadro de diverticulitis del colon, Se le encuentran una
colección de pus en la cavidad peritoneal de la cual se aislaron germenes gram negativos, gram positivos y
anaerobios. Tomando en cuenta que esta infección surgió con el paciente en la comunidad (y no es una
infección nosocomial), ¿Cuál de los siguientes antibioticos usados en monoterapia sería el mas indicado?
a. Gentamicina
b. Aztreonam
c. Cloramfenicol
d. Ertrapenem
e. Penicilina Sódica

2. Por las características farmacocinéticas los aminoglicosidos no deben emplearse por vía oral salvo en ¿cuál
de las siguientes situaciones?
a. Infección de Vías urinarias
b. Infección de vías biliares
c. Recontaminación del tracto gastrointestinal
d. Infecciones Dermatológicas
e. Buscando el sinergismo con Vancomicina

3. En un paciente con Insuficiencia Renal moderada, antecedentes de convulsiones y una infección nosocomial
por gérmenes multiresistentes ¿Cuál de los siguientes antibióticos usted evitaria para disminuir el riesgo teorico
de convulsiones?
a. Piperacilina con Tazobactam
b. Ertapenem
c. Imipenem
d. Meropenem
e. Gentamicina

4. En cual de las siguientes situaciones clínicas consideraría usted razonable el uso de cloramfenicol
especialmente ante la falla de un betalactamico.
a. Infeccion Urinaria por Gram negativos
b. Infeccion del Sistema Nervioso Central por un anaerobio
c. Endocarditis bacteriana por Staphylococus metilcilino resistentes
d. Meningitis por Cryptococus neoformans
e. En combinación con penicilina para buscar sinergismo
5
5. Un paciente procedente de Tonosí, es traído al cuarto de urgencias por presentar fiebre escalofríos,
decaimiento a lo que se ha sumado dificultad respiratoria. La radiografía de torax muestra un infiltrado
bilateral. Además de iniciar antibióticos y trasladarlo a cuidados intensivos ¿cuál de las siguiente etiologías
trataría usted de confirmar?
a. Dengue hemorrágico
b. SARS
c. Hantavirus
d. VIH agudo
e. Virus de Norwalk

6
6. A cuál de los grupos de virus listado a continuación, que pueden causar infecciones respiratorias como
bronquitis y resfriado común, pertenece al causante de la epidemia de Síndrome Respiratorio Agudo Severo
(SARS) que afectó principalmente a China y a otros paises orientales hace algunos años.
a. Rhinovirus
b. Virus Sincitial Respiratorio
c. Coronavirus
d. Rhabdovirus
e. Influenza (H5 N5)

7. Un paciente infectado con VIH asintomático que tienen un conteo de CD4 de 455 células y una carga viral de
1500 copias de RNA por ml de plasma, acude a su consultorio para inicia terapia antiretroviral altamente
efectiva ¿Cuál de las siguientes esquemas/conductas elegiría usted?
a. No iniciar tratamiento por ahora
b. Efavirenz más nucleósidos
c. Indinavir más dos nucleósidos
d. Tres nucleósidos
e. Biterapia y cuando sea necesario triple terapia

8. Si usted sospecha que un paciente con un cuadro de mononucleosis tiene realmente VIH agudo y Síndrome
de Seroconversion, ¿Cuál de las siguientes pruebas diagnosticas seria mas util?
a. HIV RNA PCR
b. HIV Elisa
c. HIV Western Blot
d. CD4
e. HIV IgM

9. ¿Cuál de los siguientes infecciones por Bacilos Gram positivos es mediada por invasión y diseminación
hematogena del germen causal y no por toxinas?
a. Enterocolitis Pseudomembranosa
b. Tetano
c. Botulismo
d. Difteria
e. Ántrax

10. Cual de los siguientes germenes se ha asociado a Colitis Hemorragica y al Síndrome Hemolitico Uremico?
a. Salmonella Typha
b. Escherichia Coli
c. Vibrio cholerae
d. Vibrio Vulnificus
e. Pseudomonas aeruginosa

7
11. Una paciente de 30 anos de edad acude con fiebre de urgencia tres dias de evolucion, que se acompana de
disuria. El examen de orina muestra algo de leucocituria. Hace dos semanas por síntomas urinario se le manda
cultivo de orina y es negativo. Se le manda otro cultivo de orina el cual es negativo. Con este cultivo de orina
Usted iniciaria tratamiento para:
a. micoplasma
b. chlamidia
c. candida
d. tuberculosis
e. infeccion por gram negativo

12. Constituye riesgo para padecer tuberculosis excepto:


a. Infeccion por SIDA
b. Silicosis
c. Terapia corticoide
d. Colitis ulcerativa autoinmune
e. Desnutrición

13. Para que se considere representativa una muestra de esputo, Usted espera encontrar
a. 10 PMN y 10 cel. Epiteliales
b. 15 PMN y 15 cel. Epiteliales
c. 20 PMN y 25 cel. Epiteliales
d. Mas de 25 PMN y menos de 10 cel. Epiteliales
e. Tener menos de 10 cel. epiteliales

14. Cual de los siguientes fármacos antiretrovirales es un inhibidor de proteasa?


a. Dinanonsisa
b. Nevirapina
c. Indinavir
d. Zidovudina
e. Lamivudine

15. Todo los siguiente es cierto, excepto:


a. La coral pertenece a la familia Elapidae
b. El efecto biologico del veneno crotalico es exclusivamente neurotoxico
c. La fraccion toxica del veneno de serpiente son proteinas y peptido de 6000-30,000 menes con receptores
específicos
d. El veneno de serpiente se produce en las glandulas homologas a las parotidas
e. El suero antiofidico se obtiene inmunización equina

16. En que condiciones clinica debemos considerar la clindamicina como parte del tratamiento del paciente
neurocronico
a. Frotis de esputo con cocos Gram negativo
b. Falta de respuesta a la cefalosporina
c. Historia de convulsiones
d. Frotis de esputo con cocos gram positivos
e. Si ademas de sospecha una meningitis

8
17. El aciclovir se elimina casi totalmente por
a. via hepatica
b. por heces
c. via biliar
d. via renal
e. por el sudor

21. Cual de las siguientes quinolonas tiene mayor cobertura contra el streptococos pneumoniae?
a. ciprofloaxina
b. levofloxacina
c. pefloxacina
d. norfloxacina
e. ninguna de las anteriores

22. La deteccion simultanea de anti HBC y anti HBs es signo de:


a. Infeccion en fase prodromica
b. Infeccion pasada
c. Infeccion activa
d. Infeccion reciente
e. Ningúna de las anteriores

9
23. De los siguientes antibioticos cual es seguro durante el embarazo?
a. Tetraciclina
b. Vancomicina
c. Amoxicilina
d. Garamicina

40. Un paciente de 30 anos de edad acude al centro de salud con mialgias, fiebre elevada, dolor retrocular,
malestar general, hace el diagnostico de dengue; el le pregunta que cuando considera fue picado por el
mosquito que le causo la enfermedad y se le dice:
a. ese mismo dia
b. de 1-3 dias
c. de 5-8 dias
d. de 8-15 dias
e. de 15-20 dias

41. El paciente VIH positivo sin tratamiento, cual de las siguientes manifestaciones obligaria a iniciar
terapia antriretroviral:
a. El paciente inicia relacion de pareja
b. El paciente lo operan de apendicitis y se le infecta la herida
c. El paciente desarrolla un cuadro de influenza
d. Presenta onicomicosis
e. El paciente desarrolla candidiasis oral

10
42. La accion toxica mas importante del ganciclovir es:
a. Nauseas y vomitos
b. Depresion de la medula osea
c. Diarrea
d. Insuficiencia Renal
e. Insuficiencia Respiratoria

43. Rotando en sala de medicina interna le piden que evalue un paciente con HIV el cual por presentar una
meningitis por criptococci recibe anfotericina B; debido a este medicamento buscaria como
complicación:
a. aparicion de diarrea
b. aumento de la presion arterial
c. hipokalemia
d. cefalea
e. dolor toráxico

44. Un paciente con herida de la piel contaminada espera encontrar como bacteria anaerobia:
a. Fusobacterium
b. Bacterioides
c. Clostridium
d. Peptoestreptococo
e. Ninguna de las Anteriores

45. En la enfermedad por legionella es muy difícil pensar en este diagnostico cuando se enferma un
paciente:
a. Fumador
b. Adulto
c. Anciano
d. Alcoholico
e. Paciente Acinado

46. En la enfermedad por legionella el tratamiento de eleccion es:


a. Macrolidos
b. Tetraciclinas
c. Cefalosporinas
d. Aminoglicosidos
e. Fosfomicinas

47. La fosfomicina es un antibiotico que ademas de las infecciones de piel tiene indicacion en infecciones:
a. intestinales
b. urinarias
c. vesiculares
d. respiratorias bajas
e. otitis por gram negativo

11
48. Los macrolidos son antibioticos:
a. Primera eleccion en faringoamigdalitis
b. Bacteriostaticos
c. Tienen cobertura antipseudomonal
d. Se usan en diverticulitis
e. Excelente como monoterapia en el helicobacter sp.

49. En el tratamiento de la meningitis por meningo coco en paciente alergico a la penicilina y cefalosporina
es:
a. clorafenicol
b. ceftriaxone
c. amikacina
d. fosfomicina
e. clindamicina

50. En la sala de medicina y de pediatria en un hospital de un area rural usted ha visto un paciente con
antecedente de fiebre reumatica, un paciente diabetico con celulitis de la pierna, dos ninos con impetigo
contagioso, una joven de 16 anos con faringitis exudativa, un nino con fiebre escarlatina. Cual de los
siguientes es el agente etiologico de todas estas enfermedades:
a. VIH
b. Streptococo pyogenes
c. Staphylococo aureus
d. Streptococo agalactiae
e. En producida por un virus

51. Unos de los siguientes enunciados es falso con respecto a la candidiasis:


a. es muy rara en los pacientes diabeticos
b. la vaginal aparece con mayor frecuencia en mujeres embarazadas
c. es la especia mas frecuente en la infeccion cutanea
d. las lesiones se producen mas frecuentes en los pliegues
e. de las primeras manifestaciones en los pacientes HIV

52. En caso de mordedura humana, cual de las siguientes indicaciones no tendrian relevancia:
a. Toxicoide tetanico
b. Amoxicilina con acido clavulanico
c. Metronidazol
d. Curacion de la herida
e. Penicilina sodica

53. Una mujer de 25 anos de edad con vida sexual activa tiene síntomas de cistitis pero la orina muestra
piuria sin bacteria. Usted piensa que esta tiene:
a. Una TBC urinaria
b. Una enf. Por gonococo
c. Una infeccion por chlamydia
d. Una infeccion bacteriana
e. No tienen infeccion

12
54. 83. Concepto falso sobre el metronidazol
a. Alternativa de tratamiento para la prostatitis
b. metabolismo hepatico
c. antianaerobico
d. es un tratamiento para las tricomonas
e. penetra la barrera hematoencefalica

55. Indicacion para el uso de Vancomicina


a. Prostatitis por estafilococo
b. Colitis asociada a antibioticos
c. Síndrome del hombre rojo
d. Primera eleccion en las celulitis
e. Neumonía

85. Diferencia de la Teicoplanina en relacion a la Vancomicina


a. Mayor actividad contra estafilococo
b. Mayor nefrotoxicidad
c. Uso intramuscular
d. Vida media similar
e. No existe diferencias

86. Para los inhibidores de beta lactamasa es cierto:


a. Excelente actividad antibacteriana
b. Todos cruzan la barrera hematoencefalica
c. Son efectivos contra todas las betalactamasas
d. No producen anafilaxia
e. Ninguna de las anteriores

87. De los siguientes antibioticos, cual tiene la mejor cobertura contra klebsiella:
a. Vancomicina
b. Streptograminas
c. Linezolid
d. Cefotaxima
e. Lincomicina

56. Las siguientes son cefalosporinas que actuan sobre las pseudomonas excepto:
a. ceftriaxona
b. cefepima
c. cefpirome
d. ceftazidima
e. ninguna de las anteriores

87. Cual de las siguientes es cierta con respecto a la TBC post-primaria?


f. generalmente se inicia en los lóbulos inferiores
g. la cavitacion pulmonar es el fenómeno caracteristico
h. es la unica enfermedad pulmonar capaz de ocasionar un patron destructivo bilateral
i. la presencia de adenopatia hiliar es un elemento basico para el diagnostico radiologico
j. la hemoptisis aparece en el momento del diagnositco en la mayoria de los casos.

57. La duracion minima del tratamiento de la TBC cuando no se asocian rifampicina o la isoniacida es de:
a. 6 meses
b. 9 meses
c. 12 meses
d. 18 meses
13
e. 24 meses

14
58.
9 Virus de dengue:
Arbovirus

10
15 Antibiótico contra Listeria Monocitogenes:
Ampicilina

33 Pte. con dengue dar:


Acetaminofen, reposo, nutrición parenteral y trato ambulatorio

34 Tratamiento de Pneumocystis Carinii:


TMP-SMX

36 Raro en dengue:
Epistaxis- tos

37 Penicilina sigue siendo efectiva para:


peumococos

39 el dengue hemorragico es una enfermedad grave con trombocitopenia y shock que se diferencia del
dengue clásico por no presentar:
Mialgia y dolores articulares

64 Cual de los siguientes antibioticos tiene mejor cobertura contra klebsiella


o cefotaxima

65 Periodo de incubacion de chlamidia:


o 7 a 21 días

66 No es subjetivo de hantavirus:
o VES normal
71 Cual de estos no es betalactamico
o Vancomicina

72 Arterias musculares de mediano calibre

74 Cual es la meta para el tratamiento de pacientes con VIH


o Suprimir la carga viral

75 Falso positivo en VDRL a los 6 meses


o Malaria

Dosis de los fármacos para la TBC


Izoniacida: 5 mg
Rifampicina: 10 mg
Etambutol: 15 a 25 mg
Pirazinamida: 25 a 30 mg

Pareo:
Pirazinamida: hiperurecemia y artralgia
15
Etambutol: neuritis óptica
Anfotericina B: hipokalemia
Izoniacida: hepatitis y colestasis
Rifampicina: compromiso hepático

Llena espacio
Factores predisponentes a candidemia:
Nutrición parenteral
Neutropenia
Inmunosuprimidos
Uso prolongado de antibioticos

PARCIAL 2 2006

Una joven de 33 años de edad presenta fiebre de 39.4ºC, cefalea y rigidez de cuello. Usted realiza el
diagnostico de meningitis bacteriana y comienza con antimicrobianos. Con la meningitis bacteriana ¿Cuál de
los siguientes es un hallazgo probable en liquido cefalorraquideo (LCR)?
a) Leucocitos entre 100 y 500 ml
b) Presión del LCR entre 100 y 120 mm H2O
c) Tinción de gram negativa
d) Glucosa >120 mg/dl
e) Valores de proteinas >45 mg/dl

En este joven adulto por lo demás sano ¿Cuál es el microorganismo más probable?
a) Streptococcus del grupo B
b) Stafilococcus aureus
c) Haemophilus influenzae
d) Streptococcus pneumoniae
e) Listeria monocytogenes

Un cuadro febril cuyas oscilaciones diarias de la temperatura son superiores a 1ºC sin alcanzar nunca los
valores normales, corresponde a la definición de:
a) Fiebre continua
b) Fiebre recurrente
c) Fiebre remitente
d) Fiebre intermitente
e) Fiebre héctica

¿De que enfermedad es característico la pupilas de Argyll-Robertson?


a) Tuberculosis
b) Lepra
c) Sífilis
d) Toxoplasmosis
e) Tetáno

¿Cuál es la triada semiológica clínica del síndrome meningitis?


a) Fiebre, cefalea y rigidez de nuca (según libro)
b) Fiebre, cefalea y vómitos
c) Fiebre, rigidez de nuca y signos encefálicos (según arjona)
d) Cefalea, rigidez de nuca y signos encefálicos
e) Cefalea, vómitos y rigidez de nuca

Entre los múltiples procesos infecciosos que producen adenopatías cervicales no suelen incluirse: todas
a) Mononucleosis infecciosa
b) Rubéola
16
c) Enfermedad por arañazo de gato
d) Tuberculosis
e) Faringitis estreptocócica

¿Cuál de los siguientes antibioticos es el que tiene menor difusión al LCR?


a) Clindamicina
b) Metronidazol (no)
c) Cotromoxazol
d) Penicilina
e) Cefotaxima

Cuando un paciente ingresa por sepsis por Stafilococcus aureus adquirida en la comunidad y sin foco aparente.
¿Qué tipo de proceso patologico debe se descartado?
a) Un absceso cutaneo oculto
b) Una neumonía -
c) Una osteomielitis vertebral -
d) Una endocarditis -
e) Un absceso renal

Con respecto a la salmonella Typhi. ¿Cuál de las siguientes afirmaciones es correcta?


a) Es la unica especie de salmonella que es patógena tanto para el hombre como para los animales
b) La vía de contagio de salmonella para el hombre es orofecal
c) La infección por S. Typhi ocurre prácticamente solo en verano
d) El inoculo mínimo necesario de S. Typha para desencadenar una infección es muy bajo de solo unas
1,000 colonias de bacterias viables. (no)
e) Más del 50% de los pacientes con fiebre tifoidea presentan manifestaciones neuropsiquiatritas.

** ¿Cuál de los siguientes antibióticos es el menos eficaz para el tratamiento de la uretritis gonocócica?
a) Ceftriaxona 500mg IM (125mg)
b) Cefixima 400mg oral en una sola dosis
c) Azitromicina 1 g una dosis
d) Ciprofloxacina 1 g en una dosis (500mg)
e) Cefuroxina 750 mg en una sola dosis

¿Cuántos días debe prolongarse el tratamiento antibiótico eficaz de una meningitis meningococcica?
a) 1-3
b) 4-7
c) 8-10
d) 12-15
e) 18-21

Que concentración de bacilos tuberculosis en esputo es necesario para que la baciloscopia (tinción de Ziehl) sea
positiva.
a) 100 bacilos /ml
b) 1000 bacilos /ml
c) 10000 bacilos /ml
d) 100000 bacilos /ml
e) 1000000 bacilos /ml

El síntoma más común de la TBC pulmonar es:


a) Tos
b) Hemoptisis (en ocasiones)
c) Fiebre
17
d) Perdida de peso
e) Anorexia

Cual de las siguientes enfermedades puede confundirse con tuberculosis pulmonar (Son tres las entidades que
con mayor frecuencia deben diferenciarse de la tuberculosis pleural: neumonía bacteriana, tromboembolismo
pulmonar y neoplasia pleural).

a) Sarcoidosis
b) Enfermedad por legionella
c) Histoplasmosis pulmonar (no)
d) Enfermedad metastásica pulmonar
e) Ninguna de las anteriores

La siguiente droga requiere de co administración de piridoxina el tratar la TBC


a) Isoniacida
b) Rifampicina
c) Etambutol
d) Rifampicina
e) Estreptomicina

Nombre dos condiciones de tuberculos extrapulmonar en donde usaria corticoides


TBC cerebral (meningea)
TBC pericardica
Unas de las siguientes drogas para el tratamiento de la TBC se metaboliza por acetilación
a) Isoniacida
b) Rifampicina
c) Etambutol
d) Rifampicina
e) Estreptomicina

La profilaxis de plasmodium falciparum resistente a la cloroquina se realiza con:


a) Cloroquina
b) Primaquina
c) Mefloquina
d) Quinina
e) Pirimetamina

Secuela de infección por herpes tipo II


a) Impotencia
b) Infertilidad
c) Infeccion de vias urinarias crónicas (NO)
d) Radiculopatia sacro lumbar
e) Síndrome diarreico

Tratamiento de la sífilis secundaria en una paciente embarazada es:


a) Tetraciclina 250mg VO qid
b) Azitromicina 1 gramo cada dia por 3 dias
c) Penicilina desensibilizacon previa
d) Doxiciclina 100mg VO bid por 14 dias
e) Claritromicina 500 mg dos veces al dia

La vacunación con BCG utiliza


a) Bacilos vivos
18
b) Bacilos atenuados (cepa atenuada de M. Boris “Bacilo de Calmette Guerin)
c) Bacilos muertos
d) Productos del bacilo
e) Todas las anteriores

En un paciente que llegue con ictericia de tipo colestasica y esta recibiendo tratamiento para TBC usted piensa
que la droga que le produce esto es:
a) Isoniacida
b) Rifampicina
c) Etambutol
d) Pirazinamida
e) Estreptomicina

Rehab 2007

1- Efecto de reversión enzimática de la pralidoxima


- tiempo:________
- dosis:_______

2- Efectos Agudos de los OF:

3- Tx de Acinetobacter

4- Tx:
Medicamento Dosis Vía
Moraxella catharralis
Clostridium difficile
Clostridium perfringes
Haemophilus ducreyi

5- Seudotipo de los neumococos:

6- Myc. avium aparece cuando la cantidad de CD4 es: ___________ (gráfica)

7- Seudotipo de Vibrio cholerae

6. Que concentración de bacilos tuberculosos en esputo es necesario para que la baciloscopia tincion de Zielh
Neelsen sea positiva
a. 100 bacilos
b. 1,000
c. 10,000
d. 100,000
e. 1,000,000

7. Una de las siguientes enfermedades puede confundirse con tuberculosis pulmonar


a. Sarcodiosis
b. Enfermedad por Legionella
19
c. Histoplasmosis Pulmonar
d. Enfermedad Metastasica Pulmonar
e. N/A

***8. Cual de los siguientes antibióticos es menos eficaz para el tratamiento de la uretritis gonococcica
a. Ceftriaxona 500 mg IM
b. Pixima 400mg oral de 1 sola dosis
c. Azitromicina 1gr 1 dosis
d. Ciprofloxacina 1gr 1 dosis
e. Cefuroxina 750 mg 1 dosis

9. Cuantos días debe prolongarse el tratamiento antibiótico eficaz de una meningitis meningoccica
a. 1 -3 dias
b. 4-5 dias
c. 8-10 dias
d. 12-15 dias
e. 18-21dias

10. En cual de las siguientes situaciones puede observarse una serologia falsamente positiva para brucella
a. Sífilis
b. Nunca
c. Fiebre tifoidea
d. Personas vacunadas contra cólera
e. Tuberculosis

11. Cual es la triada serologica clasica en síndrome meninge


a. Fiebre, Cefalea, rigidez del músculo
b. Fiebre cefalea, vomito
c. fiebre, rigidez de nuca, signos encefaliticos
d. cefalea, rigidez de nuca, signos encefaliticos
e. cefalea, vomito, rigidez de nuca

12. Entre los múltiples procesos infecciosos que producen adenopatias cervicales no puede incluirse
a. Mononucleosis infecciosa
b. Rubéola
c. Enfermedad por arañazo de gato
d. Tuberculosis
e. Faringitis estreptocócica

***13. Cual de los siguientes antibióticos es el que tiene menor difusión al LCR
a. clindamicina
b. metronidazol
c. tromboxazol
d. Penicilina
e. Cefoxazima

***14. Cuando un px ingresa …por estafilococos aureus adquirida en la comunidad y sin foco aparente que tipo
de proceso patológico debe ser descartado
a. Un absceso cutaneo oculto
b. Una neumonía
c. Osteomelitis vertebral
20
d. Una endocarditis …
e. Un absceso renal

**15. Un cuadro febril cuya…diarias de la temperatura son superirores a 1grado C` sin alcanzar nunca los
valores normales corresponde a la definición de
a. Fiebre continúa.
b. Fiebre recurrente
c. Fiebre remitente
d. Fiebre intermitente
e. Fiebre …etdica..

***16. Una joven de 33 años de edad presenta fiebre de 39. 4, cefalea y rigidez de cuello, usted realiza el
diagnostico de meningitis bacteriana si comienza de inmediato con antimicrobianos

**17. Con la meningitis bacteriana cual de los siguientes es un hallazgo probable de LCR
a. Valores de proteína mayores de 45 mg/dl

Cuál de los siguientes antibióticos produce nefritis intersticial:


- meticilina

El factor más determinante en el tratamiento de la hipokalemia renal es:


- la función cardiovascular.

El tratamiento de inicio de secreción inapropiada de ADH es:


- restricción hídrica

En la cirrosis hepática complicada con edema, el medicamento diurético inicial es:


- aldactone

Todas las siguientes con complicaciones de la cirrosis hepática, excepto:


- perforación del íleo

En la hipertensión portal por cirrosis hepática alcohólica la resistencia es a nivel:


- sinusoidal

7. Los agentes causales más frecuentes de meningitis viral son:


1. Arbovirus.
2. Herpesvirus.
3. Virus de la coriomeningitis linfocitaria.
4. Virus de la parotiditis epidémica.
5. Enterovirus.

9. Mujer de 33 años de edad ecuatoriana, acude a su médico de familia para la lectura de Mantoux, realizado en el
contexto de un estudio de contactos. Una prima suya que vive en su casa y duerme en la misma habitación (junto con
otras 7 personas), ha sido diagnosticada de una tuberculosis pulmonar bacilífera (más de 50 bacilos por campo). Su
médico aprecia una induración de 7 mm en la lectura del PPD. ¿Cuál de las siguientes afirmaciones es la correcta?:
1. Se trata de una infección tuberculosa y hay que iniciar tratamiento quimioprofiláctico de inmediato.
2. Se trata de una Tuberculosis y hay que comenzar con tratamiento antituberculoso.
3. Se trata de una infección tuberculosa y hay que descartar enfermedad tuberculosa, previo a comenzar el tratamiento
quimioprofiláctico.
21
4. El PPD no es significativo, ya que al tratarse de una persona inmigrante de una zona de alta prevalencia de TBC debería
tener una induración mayor de 10 mm.
5. El PPD no es significativo, ya que al tratarse de una persona inmigrante de una zona de alta prevalencia de TBC debería
tener una induración a mayor de 15 mm.

1. En la neumonía por Pneumococistis carinii en los pacientes con infección por VIH, ¿cuál de las siguientes
respuestas es la verdadera?:
1. El diagnóstico definitivo se realiza por cultivo del esputo en medios específicos.
2. El uso de glucocorticoides está contraindicado.
3. La Pentamidina intravenosa es el tratamiento alternativo de elección en las formas graves.
4. El riesgo de padecerla es independiente de la cifra de linfocitos CD4+.
5. Nunca está indicada la profilaxis primaria.

3. Señale, entre las siguientes, la conducta más adecuada ante un paciente leucopénico que comienza con fiebre:
1. Observarle para vigilar si aparecen síntomas respiratorios.
2. Tomar los cultivos adecuados y comenzar tratamiento antibiótico empírico cuanto antes.
3. Tomar los cultivos adecuados y esperar a los resultados antes de iniciar ningún tratamiento.
4. Tratarle exclusivamente con factor estimulante de colonias de granulocitos.
5. Tratarle con paracetamol, sin mayor preocupación, pues no es probable que padezca una infección.

4.

5. Un joven de 16 años realiza un viaje de fin de curso por Europa. Al mes de regreso comienza con malestar general,
odinofagia y fiebre; en la exploración destaca hipertrofia amigdalar con exudado blanquecino, adenopatías
occipitales, laterocervicales dolorosas; en el hemograma se observa leucocitosis de 15000/mm3 con un 70% de
linfocitos, alguno de ellos atípico. Ante la sospecha diagnóstica se debe realizar:
1. Biopsia ganglionar.
2. Biopsia de médula ósea.
3. Tratamiento con Penicilina.
4. Serología para virus de Epstein Barr.
5. Tratamiento con clindamicina.

7. Un paciente de 40 años con antecedentes de adicción a drogas por vía parenteral, con infección por el virus de la
inmunodeficiencia humana (VIH) conocida desde hace 8 años, con un ingreso en institución penitenciaria hace 2
años, en que presentaba un Mantoux de 7 mm, nunca ha recibido ningún tratamiento ni profilaxis. En la actualidad se
encuentra asintomático, su CD4 son de 100 cel/uL y el Mantoux negativo. Sería INCORRECTO pensar que:
1. Debe recibir profilaxis con Isoniacida 300 mg + piridoxina durante 9 meses.
2. Debe recibir profilaxis con Rifabutina y Piracinamida durante 2 meses.
3. Probablemente presente una infección tuberculosa.
4. No debe recibir profilaxis si se encuentra en tratamiento deshabituador con Metadona.
5. Debe recibir profilaxis con Isonicida 900 mg + piridoxina, 2 veces por semana durante 9 meses.

8. Es conocido el riesgo potencial de transmisión de VIH o hepatitis desde un paciente portador al equipo médico-
quirúrgico que le atiende. La política recomendada actualmente para evitar estos contagios consiste en aplicar una
serie de medidas especiales para evitar el contacto con sangre o líquidos orgánicos del paciente. Para ello se debe:
1. Llevar a cabo las pruebas serológicas sistemáticamente a todos los pacientes y tomar medidas en los que den algún resultado
positivo.
2. Solicitar consentimiento previo y llevar a cabo pruebas serológicas a los pacientes que lo hayan otorgado, tomando medidas
en los portadores.
3. Averiguar el nivel de riesgo individual mediante historia clínica y aplicar las medidas a los pacientes con elevada
probabilidad de ser portadores.
4. Tomar precauciones especiales sólo en aquellos pacientes en que, por otro motivo, haya sido diagnosticados previamente de
portadores de alguno de los virus mencionados.
5. Considerar potenciales portadores a todos los pacientes y tomar medidas en todos ellos.

22
9. ¿Qué es el dengue?:
1. Una enfermedad causada por un poxvirus.
2. Una enfermedad limitada a los países del centro de Africa.
3. Una enfermedad vírica que puede producir una fiebre hemorrágica.
4. Una zoonosis que afecta al hombre ocasionalmente.
5. Una enfermedad vírica que ocasiona un eritema que evoluciona a mácula y pápula afectando fundamentalmente a la
población infantil.

1. Acerca de la fiebre de origen desconocido, ¿cuál de las siguientes afirmaciones es FALSA?:


1. Clásicamente se define como fiebre mayor de 38,3ºC, en varias ocasiones, durante más de tres semanas, sin encontrarse un
diagnóstico tras una semana de investigaciones en el hospital.
2. Hoy en día se prefiere clasificarla como clásica, en neutropénico, nosocomial y asociada a infección por VIH.
3. Es fundamental hacer una historia de los viajes realizados.
4. En pacientes neutropénicos (<50 neutrófilos/ mL), es útil realizar un tratamiento antibiótico empírico.
5. En pacientes que la presentan durante más de seis meses, la causa más frecuente es infecciosa.

2. Un paciente ingresa en urgencias con disminución del nivel de conciencia, fiebre de 39ºC y TA de 70/40 mmHg.
¿Cuál, de las siguientes, considera la actitud inicial más correcta?:
1. Iniciar tratamiento antibiótico empírico de amplio espectro.
2. Realizar una punción lumbar.
3. Bajar la fiebre.
4. Tomar muestras para hemocultivos.
5. Aportar líquidos intravenosos.

3. Paciente trasplantado renal de 2 meses de evolución que acude al servicio de urgencias por síndrome febril de 3
días de evolución bien tolerado y acompañado de epigastralgias. En la analítica practicada destaca una moderada
leucopenia (2400/mm3) con una leve elevación en la cifra de transaminasas (ALT 75 UI/l; AST 89 Ul/l. ¿Cuál sería el
primer diagnóstico de sospecha?:
1. Tuberculosis pulmonar.
2. Infección por Helicobacter pilorii.
3. Infección por Pneumocistis carinii.
4. Infección por Citomegalovirus.
5. Hepatitis por VHC.

4. El tratamiento antirretroviral en un paciente VIH+ con carga viral de 575.000/mm3 y una cifra de linfocitos de CD4
de 450/mm3:
1. Es obligado.
2. Sólo estaría indicado si la carga viral fuera superior a 1 millón de copias/ml.
3. En ningún caso se prescribiría si los CD4+ son >200/ml.
4.
5. 000/mm3 y una cifra de linfocitos de CD4 de 450/mm3: 1. Es obligado. 2. Sólo estaría indicado si la carga viral fuera
superior a 1 millón de copias/ml. 3. En ningún caso se prescribiría si los CD4+ son >200/ml. 4. Está indicado si el paciente lo
desea. 5. En estas condiciones sólo estaría indicado en el contexto de un estudio clínico prospectivo.

5. Una paciente de 45 años presenta de forma progresiva en los últimos 3 días un cuadro de cefalea, deterioro del
nivel de conciencia y fiebre de 39,5ºC. entre sus antecedentes destaca una enfermedad de Crohn que ha requerido
tratamiento de forma irregular en los últimos 3 años. Desde hace 6 meses está tomando mesalazina y 15 mg de
prednisona/ día. La exploración clínica no presenta datos significativos, salvo discretos signos de irritación
meníngea. El hemograma muestra 15.600 leucocitos, con una fórmula normal. La bioquímica elemental es normal. La
Rx de tórax no presenta alteraciones significativas. Se realiza una punción lumbar, obteniéndose un LCR con
proteínas de 560 mg/ dl, glucosa 25 mg/dl y 325 células/ml (85% células mononucleares). La tinción de Gram y de
Ziehl-Neelsen en LCR son negativas. ¿Cuál es la actitud más correcta?:
1. Sospechar una meningoencefalitis autoinmune y subir la dosis de prednisona a 1 mg/ kg de peso, añadiendo quimioprofilaxis
con isoniacida.
23
2. Sospechar una meningitis por gérmenes de origen gastrointestinal. Iniciar tratamiento antibiótico empírico que cubra
gérmenes Gram (-) y anaerobios.
3. Sospechar una meningitis tuberculosa. Iniciar tratamiento inmediato con tres tuberculostáticos.
4.
5. 600 leucocitos, con una fórmula normal. La bioquímica elemental es normal. La Rx de tórax no presenta alteraciones
significativas. Se realiza una punción lumbar, obteniéndose un LCR con proteínas de 560 mg/ dl, glucosa 25 mg/dl y 325
células/ml (85% células mononucleares). La tinción de Gram y de Ziehl-Neelsen en LCR son negativas. ¿Cuál es la actitud
más correcta?: 1. Sospechar una meningoencefalitis autoinmune y subir la dosis de prednisona a 1 mg/ kg de peso, añadiendo
quimioprofilaxis con

6. Un trabajador sanitario tiene una prueba de la tuberculina que mide 0 mm. Al repetirla 10 días después, el
diámetro de la induración mide 12 mm. ¿Cuál de las siguientes interpretaciones es más adecuada para estos
resultados?:
1. Primoinfección tuberculosa entre las dos pruebas.
2. Infección tuberculosa latente.
3. Presencia de anergia.
4. Tuberculosis activa hace años.
5. Mala realización de alguna de las pruebas.

7. ¿Cuál de las siguientes pruebas es la más adecuada para evaluar la respuesta terapéutica en un paciente tratado de
sífilis precoz?:
1. Realización de fondo oscuro sobre las lesiones que vayan apareciendo.
2. Examen anual de líquido cefalorraquídeo (VDRL).
3. Test de inmovilización del Treponema pallidum (TPI).
4. Cualquiera de las pruebas treponémicas (TPI, FTA, etc).
5. Evaluación seriada del título del VDRL o RPR (pruebas no treponémicas).

10. ¿Cuál de las siguientes afirmaciones, en cuanto a la Neumonía por Pneumocistis Carinii en pacientes VIH (Virus de
la inmunodeficiencia humana), positivos, es FALSA?:
1. Se presentan clínicamente de forma subaguda.
2. Con elevada frecuencia se prescribe tratamiento sin confirmación diagnóstica bacteriológica.
3. El 95% de los pacientes tiene un recuento de CD4<200cel/uL.
4. El diagnóstico se obtiene habitualmente mediante cultivo de secreciones bronquiales obtenidas por la inducción del
esputo.
5. La profilaxis primaria se puede suspender en pacientes con tratamiento antirretroviral que presenta una carga viral suprimida
(<500 cop/ml) y CD4>200cel/uL.

11. En la interpretación de los resultados de los hemocultivos practicados a un paciente con fiebre, ¿cuál de los
siguientes datos nos haría pensar que no estamos ante un caso de contaminación?:
1. Aislamiento de bacterias que normalmente colonizan la piel.
2. Aislamiento de cocos gram positivos.
3. Aislamiento de bacterias difteroides.
4. Aislamiento del mismo microorganismo en hemocultivos con la misma sensibilidad.
5. Aislamiento de un estafilococo meticilin resistente en un solo hemocultivo.

12. ¿Cuál, entre los siguientes, es un virus persistente que puede permanecer en latencia y reactivarse?:
1. El virus de la hepatitis A.
2. El virus respiratorio sincitial.
3. El citomegalovirus.
4. El virus de la poliomielitis.
5. El rotavirus.

24
13. Señale, entre las siguientes, la conducta más adecuada ante un paciente leucopénico que comienza con fiebre:
1. Observarle para vigilar si aparecen síntomas respiratorios.
2. Tomar los cultivos adecuados y comenzar tratamiento antibiótico empírico cuanto antes.
3. Tomar los cultivos adecuados y esperar a los resultados antes de iniciar ningún tratamiento.
4. Tratarle exclusivamente con factor estimulante de colonias de granulocitos.
5. Tratarle con paracetamol, sin mayor preocupación, pues no es probable que padezca una infección.

14. Una joven de 18 años acude al hospital por fiebre y cefalea de varias horas de evolución. Los días previos había
notado dolor de garganta y tos. Se observó tendencia al sueño, rigidez de nuca y petequias en conjuntivitis y
extremidades. El LCR era turbio y contenía 36.000 leucocitos /mm3, 200 mg/dL de proteínas y 20 mg/dL de glucosa
(glucemia simultánea 120 mg/dL). El examen con Gram fue negativo. Señale cuál de las siguientes afirmaciones NO es
correcta:
1. El diagnóstico más probable es meningitis meningocócica.
2. Los meningococos son siempre sensibles a la penicilina y, por tanto, la penicilina G es el tratamiento de elección.
3. Como un significativo porcentaje de meningococos del grupo C son resistentes a la penicilina, la cefotaxima es un
tratamiento más seguro.
4. El empleo de dexametasona reduce el riesgo de secuelas neurosensoriales en niños con meningitis bacteriana, pero su
uso en un caso como éste es cuestionable.
5. Se debe administrar rifampicina a las personas que conviven estrechamente con la paciente.

17. Un hombre de 45 años con leucemia mieloblástica aguda recibió su tercer ciclo de quimioterapia. A los 7 días
desarrolló fiebre y un súbito empeoramiento del estado general. Presentaba una lesión nodular equimótica y
dolorosa, con centro ulcerado y rodeada de eritema y edema, en la pierna izquierda. Tenía menos de 100
leucocitos/mm3, hemoglobina 7 gr/dL y 30.000 plaquetas/mm3. ¿Qué germen es aislado con mayor frecuencia en un
caso como éste?:
1. Aeromonas hydrophila.
2. Klebs
3. ¿Qué germen es aislado con mayor frecuencia en un caso como éste?: 1. Aeromonas hydrophila. 2. Klebsiella pneumoniae.
3. Staphyloccocus aureus.
4. Pseudomonas aeruginosa.
5. Xantomonas maltophilia.

18. ¿En qué situación clínica el uso de la reacción en cadena de la polimerasa (PCR) se ha convertido en la técnica de
referencia para realizar su diagnóstico?:
1. Diagnóstico de SIDA.
2. Tuberculosis pulmonar no bacilífera.
3. Síndrome mononucleósido por CMV.
4. Encefalitis por virus Herpes Simplex.
5. Meningitis meningocócica.

20. Una paciente de 43 años, con una prótesis mitral implantada 3 semanas antes, acude a Urgencias por fiebre de 5
días de evolución. Dos días más tarde el laboratorio de Microbiología informa del crecimiento en 5 de los 6 frascos de
hemocultivo de cocos Gram positivos en racimo. ¿Cuál, de los siguientes, es el tratamiento de elección hasta conocer
los resultados definitivos?:
1. Vancomicina + gentamicina + rifampicina.
2. Cloxacilina + gentamicina + rifampicina.
3. Penicilina + gentamicina + rifampicina.
4. Ceftriaxona.
5. Imipenem.

21. El término diferenciación aplicado al tejido neoplásico define:


1. El grado de similitud de las células neoplásicas desde el punto de vista morfológico y funcional con las células
normales de las que derivan.
25
2. La aparición dentro de un tumor de elementos neoplásicos que no están presentes en el órgano o en el tejido en el cual se
originan.
3. La presencia de un grado extremo de anaplasia.
4. La presencia de áreas tumorales de morfología diferente en cambios adyacentes del tumor (por ejemplo adenocarcinoma y
carcinoma epidermoide).
5. La variación de la morfología tumoral en la recurrencia de la enfermedad.

24. Los agentes causales más frecuentes de meningitis viral son:


1. Arbovirus.
2. Herpesvirus.
3. Virus de la coriomeningitis linfocitaria.
4. Virus de la parotiditis epidémica.
5. Enterovirus.

30. ¿Cuál es el patógeno involucrado más frecuentemente en la “diarrea del viajero”?:


1. Salmonella.
2. Giardia Lamblia.
3. Campylobacter.
4. Entamoeba Hystolítica.
5. Escherichia Coli.

31. ¿En cuál de las siguientes situaciones es más probable que podamos encontrar una reacción negativa a la
tuberculina?:
1. Niño de 2 años, inmunocompetente, al que se vacunó con BCG al nacer.
2. Mujer de 25 años que consulta por esterilidad y a quien se diagnostica tuberculosis genital.
3. Hombre de 45 años con lesiones pulmonares cavitarias y baciloscopia y cultivo positivos para M. tuberculosis.
4. Hombre de 70 años, tuberculoso antiguo, muy grave, con un patrón miliar en Rx de tórax y con hemocultivo positivo
para M. tuberculosis.
5. Hombre con hematuria, fiebre, orquiepididimitis derecha, dolores cólicos ureterales y cultivo positivo para M. tuberculosis
en orina.

37. ¿En cuál de las siguientes localizaciones de la enfermedad tuberculosa está indicado el tratamiento coadyuvante
con glucocorticoides para mejorar la supervivencia?:
1. Pulmonar.
2. Meníngea.
3. Ganglionar.
4. Genitourinaria.
5. Ostearticular.

38. Señale cuál de las siguientes afirmaciones es FALSA acerca de la vía piramidal:
1. En el mesencéfalo y protuberancia, la vía piramidal desciende por el mismo lado en el que se ha originado.
2. En el extremo inferior del bulbo se decusan únicamente las fibras que inervan la musculatura del tronco.
3. El origen de la vía se sitúa en el gyrus precentral.
4. El fascículo piramidal cruzado desciende por el cordón lateral de la médula.
5. La corteza cerebral donde se origina el fascículo piramidal está irrigada por las arterias cerebrales anterior y media.

47. Una de las siguientes aseveraciones NO es correcta en la tuberculosis miliar. Señálela:


1. Se debe a la diseminación hematógena del bacilo.
2. La tinción de Ziehl del esputo suele ser negativa.
3. La prueba de la tuberculina es positiva en el 80% de los casos.
4. La biopsia de médula ósea puede proporcionar el diagnóstico.

26
5. Puede presentarse como manifestación de una infección tuberculosa primaria.

50. Ante un joven de 16 años que ha estado conviviendo con un enfermo con tuberculosis pulmonar activa y tiene una
prueba de la tuberculina negativa, la actitud más correcta, de las siguientes, es realizar:
1. Quimioprofilaxis durante 6 meses.
2. Vigilancia clínica y prueba de tuberculina a los 3 meses.
3. Quimioprofilaxis durante 2-3 meses y luego repetir la prueba de la tuberculina.
4. Tratamiento con rifampicina, isoniacida y piracinamida durante 6 meses.
5. Tratamiento con rifampicina, isoniacida y piracinamida y, a los 2 meses, repetir la prueba de la tuberculina.

2. Un enfermo con neumonía, perteneciente a un brote epidémico de varias personas que ocasionalmente conviven en un
edificio, presenta un cuadro confusional desproporcionado a la fiebre, diarrea, hiponatremia y ligero ascenso de las enzimas
hepáticas. Entre las siguientes opciones, ¿cuál se debe incluir en el tratamiento?:
Cefalosporinas y aminoglucósidos a las dosis adecuadas.
Vancomicina a las dosis adecuadas.
Doxiciclina 100 mg/12 h durante 14 días.
Eritromicina 2-4 g/día durante 14 días.
Penicilina G procaína 0,6 – 1,2x106 U/12 h durante 7 días.

5. Un

7. Un enfermero de Urgencias le consulta porque ha tenido un accidente en el que ha recibido un pinchazo profundo, sin
guantes, con una aguja gruesa visiblemente manchada de sangre, de un paciente adicto a drogas por vía parenteral. Tras
interrogar al paciente, declara que comparte habitualmente jeringuillas intravenosas y que nunca se ha realizado una
serología para el VIH. ¿Cuál de las siguientes es la actitud más correcta?:
Esperar al día siguiente a que esté el resultado de la serología de VIH.
Iniciar inmediatamente tratamiento con tres antiretrovirales.
Iniciar inmediatamente tratamiento con AZT.
Realizar serología de VIH, carga viral de VIH y test de resistencias genotípicas (en caso de carga viral detectable) y
revisar, cuando estén los resultados, la necesidad de tratamiento antiretroviral.
Tranquilizar al enfermo debido a bajo riesgo de transmisión del VIH y reevaluar en un mes.

8. Mujer de 33 años de edad ecuatoriana, acude a su médico de familia para la lectura de Mantoux, realizado en el contexto de
un estudio de contactos. Una prima suya que vive en su casa y duerme en la misma habitación (junto con otras 7 personas), ha
sido diagnosticada de una tuberculosis pulmonar bacilífera (más de 50 bacilos por campo). Su médico aprecia una induración
de 7 mm en la lectura del PPD. ¿Cuál de las siguientes afirmaciones es la correcta?:
Se trata de una infección tuberculosa y hay que iniciar tratamiento quimioprofiláctico de inmediato.
Se trata de una Tuberculosis y hay que comenzar con tratamiento antituberculoso.
Se trata de una infección tuberculosa y hay que descartar enfermedad tuberculosa, previo a comenzar el tratamiento
quimioprofiláctico.
El PPD no es significativo, ya que al tratarse de una persona inmigrante de una zona de alta prevalencia de TBC debería tener una
induración mayor de 10 mm.
El PPD no es significativo, ya que al tratarse de una persona inmigrante de una zona de alta prevalencia de TBC debería
tener una induración a mayor de 15 mm.

10. ¿Cuál de las siguientes afirmaciones relativas a la fiebre tifoidea NO es correcta?:


La enfermedad se suele contraer por la ingesta de alimentos, agua o leche contaminados.
La leucopenia es más frecuente que la leucocitosis en las personas con enfermedad aguda.
La roseola se suele presentar en el momento en el que comienza la fiebre.
El cloranfenicol no es eficaz para prevenir las recaídas.
Las fluoroquinolonas erradican el germen, incluso en presencia de cálculos biliares.

11. Paciente de 64 años, fumador, que acude a urgencias por un cuadro de 48 h. de evolución de fiebre y tos con expectoración
mucopurulenta. La radiografía de tórax muestra una condensación alveolar en lóbulo inferior derecho y un pequeño
infiltrado en el lóbulo inferior izquierdo. La gasometría arterial muestra un pH de 7,39, una pO2 de 54 mmHg y una pCO2 de
29 mmHg. ¿Cuál de las siguientes opciones terapéuticas le parece más adecuada?:
Claritromicina 500 mg IV/12h.
Iprofloxacina 200 mg IV/12h.
Amoxicilina-Ácido clavulánico 1g IV/8h.
Ciprofloxacino 200 mg EV/12h + Claritromicina 500 mg IV/12h.
Ceftriaxona 2g IV/24h + Claritromicina 500 mg IV/12h.

27
13. La malaria es una enfermedad parasitaria erradicada en nuestro país, pero en los últimos años estamos asitiendo a un
resurgir de casos debidos a la inmigración y a los viajes a países tropicales. En relación a la malaria, cuál de las siguientes
afirmaciones es FALSA:
La gravedad de la infección está en relación con el grado de parasitemia.
La infección por plasmodium falciparum es la más grave.
La malaria cerebral es una manifestación tipica de plasmodium vivax.
El dato analítico más frecuente es una anemia normocítica.
La infección se ha descrito en adictos a drogas por vía parenteral que comparten jeringuillas.

14. Un paciente, fumador, de 60 años, acude a Urgencias con un cuadro de fiebre, tos y expectoración amarillenta de 36 horas
de evolución. Se le objetiva una condensación con broncograma aéreo en la base pulmonar derecha en la Rx de tórax. ¿Cuál
de las siguientes pruebas complementarias es de mayor utilidad para decidir su hospitalización?:
Realización de tinción de Gram en una muestra de esputo.
TC torácica.
Hemocultivos seriados.
Gasometría arterial basal.
Aspirado transtraqueal.

17. ¿Cuál es la principal causa de epididimitis en varones heterosexuales activos menores de 35 años?:
Chlamydia trachomatis.
Neisseria gonorrhoeae.
Ureplasma urealyticum.
Escherichia coli.
Pseudomonas aeruginosa.

19. Un paciente de 32 años con infección VIH, ingresa con un cuadro de pancreatitis aguda. No es consumidor de alcohol. La
ecografía abdominal descarta litiasis biliar. Sigue tratamiento antirretroviral con zidovudina (AZT), didanosina (ddI) e
indinavir. ¿Qué se debe suprimir de su tratamiento?:
Didanosina (ddI).
Zidovudina (AZT).
Indinavir.
Los tres fármacos.
Nada.

21. Una de las siguientes propiedades referidas a los virus RNA es cierta:
La mayoría de ellos se multiplican en el núcleo.
La estructura del genoma no es determinante del mecanismo de transcripción y replicación.
No es necesario que codifiquen RNA polimerasas RNA dependientes.
No muestran tendencias a las mutaciones.
El genoma RNA (+) de los retrovirus se convierte en DNA, que es integrado en la cromatina del huésped y transcrito
como un gen celular.

22. Un niño de 14 años acude a su consulta por presentar herida por mordedura en antebrazo, con exudado purulento que ha
empeorado a pesar del tratamiento con clindamicina. ¿Cuál es la etiología más probable de la infección de la herida?:
Staphylococcus Aureus.
Streptococo beta-hemolítico grupo A.
Eikenella Corrodens.
Capnocytophaga gingivalis.
Mycobacterium Tuberculosis. ANULADA

23. La candidiasis hepatoesplénica ocurre habitualmente en:


Paciente con SIDA.
Diabéticos tipo I con neuropatía avanzada.
Receptores de trasplante hepático en el período postrasplante precoz.
Pacientes con leucemia aguda y leucopenia prolongada en fase de resolución.
Pacientes con cirugía pancreática mayor y con nutrición parenteral.

24. El antibiótico de elección en el tratamiento de una neumonía producida por Legionella pneumophila es:
Eritromicina.
Penicilina G.
Cotrimoxazol.
Cefotaxima.
Clindamicina.

28
29. En relación a la sífilis, señale la afirmación correcta:
La lúes secundaria cursa excepcionalmente con manifestaciones cutáneas.
El control del tratamiento se puede realizar valorando los títulos de positividad de las pruebas treponémicas.
Durante los períodos de latencia de la enfermedad se negativizan las pruebas no treponémicas.
La peculiar estructura del treponema hace que no sea sensible a los antibióticos betalactámicos.
El chancro sifilítico es indurado, no doloroso y muy rico en bacterias.

30. Es conocido el riesgo potencial de transmisión de VIH o hepatitis desde un paciente portador al equipo médico-quirúrgico
que le atiende. La política recomendada actualmente para evitar estos contagios consiste en aplicar una serie de medidas
especiales para evitar el contacto con sangre o líquidos orgánicos del paciente. Para ello se debe:
Llevar a cabo las pruebas serológicas sistemáticamente a todos los pacientes y tomar medidas en los que den algún resultado
positivo.
Solicitar consentimiento previo y llevar a cabo pruebas serológicas a los pacientes que lo hayan otorgado, tomando
medidas en los portadores.
Averiguar el nivel de riesgo individual mediante historia clínica y aplicar las medidas a los pacientes con elevada probabilidad de
ser portadores.
Tomar precauciones especiales sólo en aquellos pacientes en que, por otro motivo, haya sido diagnosticados previamente de
portadores de alguno de los virus mencionados.
Considerar potenciales portadores a todos los pacientes y tomar medidas en todos ellos.

48. Las manifestaciones clínicas del tétanos se producen por la:


Invasión tisular directa del sistema nervioso central.
Producción de una exotoxina en la herida infectada.
Producción de citoquinas.
Destrucción tisular marcada en la zona infectada con sobreinfección bacteriana polimicrobiana secundaria.
Destrucción bacteriana masiva al iniciar el tratamiento antibiótico.

50. ¿Cuál, de los siguientes, es con más frecuencia el germen causal de espondilodiscitis piógena en pacientes con SIDA,
adictos a drogas por vía parenteral?:
Salmonella.
Haemophilus influenzae.
Pseudomonas aeruginosa.
Streptococcus granulosus.
Escherichia coli.

INFECTO

34. Señale, entre las siguientes, la conducta más adecuada ante un paciente leucopénico que comienza con fiebre:
Observarle para vigilar si aparecen síntomas respiratorios.
Tomar los cultivos adecuados y comenzar tratamiento antibiótico empírico cuanto antes.
Tomar los cultivos adecuados y esperar a los resultados antes de iniciar ningún tratamiento.
Tratarle exclusivamente con factor estimulante de colonias de granulocitos.
Tratarle con paracetamol, sin mayor preocupación, pues no es probable que padezca una infección.

37. La llamada meningitis de Mollaret es un cuadro, poco frecuente, consistente en episodios recurrentes de meningitis en las
que suelen verse células sugerentes de esta entidad en el LCR. Hoy se cree que:
Está causada por Enterovirus.
Está causada por Adenovirus.
Se trata de meningitis bacterianas abortadas por tratamiento antibiótico.
Se ha vinculado a Virus del Herpes Simple.
Es una forma frustrada de enfermedad tuberculosa.

39. En la neumonía por Pneumocistis carinii en los pacientes con infección por VIH ¿cuál de las siguientes respuestas es la
verdadera?:
El diagnóstico definitivo se realiza por cultivo del esputo en medios específicos.
El uso de glucocorticoides está contraindicado.
La Pentamidina intravenosa es el tratamiento alternativo de elección en las formas graves.
El riesgo de padecerla es independiente de la cifra de linfocitos CD4+.
Nunca está indicada la profilaxis primaria.

44. Una paciente de 45 años presenta de forma progresiva en los últimos 3 días un cuadro de cefalea, deterioro del nivel de
conciencia y fiebre de 39,5ºC. entre sus antecedentes destaca una enfermedad de Crohn que ha requerido tratamiento de
forma irregular en los últimos 3 años. Desde hace 6 meses está tomando mesalazina y 15 mg de prednisona/ día. La
exploración clínica no presenta datos significativos, salvo discretos signos de irritación meníngea. El hemograma muestra

29
15.600 leucocitos, con una fórmula normal. La bioquímica elemental es normal. La Rx de tórax no presenta alteraciones
significativas. Se realiza una punción lumbar, obteniéndose un LCR con proteínas de 560 mg/ dl, glucosa 25 mg/dl y 325
células/ml (85% células mononucleares). La tinción de Gram y de Ziehl-Neelsen en LCR son negativas. ¿Cuál es la actitud
más correcta?:
Sospechar una meningoencefalitis autoinmune y subir la dosis de prednisona a 1 mg/ kg de peso, añadiendo
quimioprofilaxis con isoniacida.
Sospechar una meningitis por gérmenes de origen gastrointestinal. Iniciar tratamiento antibiótico empírico que cubra gérmenes
Gram (-) y anaerobios.
Sospechar una meningitis tuberculosa. Iniciar tratamiento inmediato con tres tuberculostáticos.

600 leucocitos, con una fórmula normal. La bioquímica elemental es normal. La Rx de tórax no presenta alteraciones
significativas. Se realiza una punción lumbar, obteniéndose un LCR con proteínas de 560 mg/ dl, glucosa 25 mg/dl y 325
células/ml (85% células mononucleares). La tinción de Gram y de Ziehl-Neelsen en LCR son negativas. ¿Cuál es la actitud
más correcta?: 1. Sospechar una meningoencefalitis autoinmune y subir la dosis de prednisona a 1 mg/ kg de peso, añadiendo
quimioprofilaxis con

46. Enfermo de 55 años diagnosticado de gammapatía monoclonal presenta lesiones papulosas rosadas y traslúcidas en
regiones periorbitarias y peribucales junto con macroglosia. El diagnóstico más probable es:
Linfoma de células B.
Mucinosis papulosa.
Amiloidosis AL.
Calcinosis cutáneo-mucosa.
Lupus eritematoso.

1 Un paciente de 30 años adicto a drogas por vía parenteral, se presenta con un cuadro de 3 días de evolución de
tiritona, fiebre, dolor torácico y tos con expectoración verdosa con "hilillos" de sangre; en la radiografía de tórax
presenta varios infiltrados pulmonares con cavitación central en alguno de ellos. El diagnóstico más probable
sería:
1. Neumonía neumocócica.
2. Tuberculosis pulmonar.
3. Neumonía por anaerobios.
4. Neumonía hematógena.
5. Neumonía por H.Influenzae

2. Un paciente de 35 años de edad acude al servicio de urgencias por presentar fiebre elevada de hasta 39ºC, junto
con confusión mental. En la exploración física destaca la existencia de lesiones cutáneas en pie izquierdo a nivel
distal, maculares, de milímetros de diámetro, de aspecto isquémico hemorrágico y la auscultación cardiopulmonar
es normal. A los pocos día se obtiene crecimiento de Staphylococo aureus meticilin sensible en tres hemocultivos de
tres obtenidos. ¿Cuál de las siguientes sería la actitud correcta a seguir en ese momento?:
1. Considerar el resultado de los hemocultivos como probable contaminación.
2. Pautar de inmediato tratamiento antibiótico con penicilina y gentamicina durante 10 días.
3. Comenzar tratamiento con cloxacilina y gentamicina y realizar estudio ecocardiográfico por la existencia
probable de endocarditis aguda.
4. Descartar la existencia de endocarditis por la ausencia de soplos en la auscultación cardiaca y buscar focos de posible
osteomielitis.
5. Realizar TAC abdominal urgente por probable absceso abdominal.

3. Señale la cierta, en relación a la encefalitis herpética:


1. Está producida casi siempre por el virus del herpes simple tipo 2.
2. La clínica característica es la presencia de fiebre con síntomas focales del lóbulo occipital.
3. La PCR en el líquido cefalorraquídeo no es útil para establecer el diagnóstico.
4. El tratamiento de elección es el zanamivir i.v.
5. Las secuelas neurológicas son frecuentes.

4. ¿Cuál de los siguientes cuadros clínicos no se asocia con toxoplasmosis?:


1. Abscesos cerebrales en personas con SIDA.
2. Miocarditis en transplantados cardiacos.
3. Coriorretinitis en adultos inmunocompetentes.
4. Colitis asociada a inmunodeficiencia humoral.
5. Síndrome mononucleósico en personas sanas.

5 Señale la afirmación INCORRECTA con respecto a Haemophilus influenzae:


30
1. Es un parásito obligado de las mucosas humanas.
2. Es un bacilo gramnegativo pequeño y pleomórfico.
3. La vacunación es de poca utilidad porque no cubre el serotipo B, que es el más prevalente en nuestro ambiente.
4. Las infecciones no sistémicas (otitis, bronquitis, conjuntivitis,...) están producidas generalmente por serotipos
diferentes al B.
5. Sólo las cepas capsuladas causan infecciones sistémicas.

6. Señale la cierta respecto a la Fiebre Q:


1. Se transmite por inhalación de partículas contaminadas.
2. Los hemocultivos son positivos en la fase inicial.
3. La mancha negra se observa en el 60% de los casos.
4. El exantema suele afectar palmas y plantas.
5. El tratamiento de elección es la gentamicina.

7. Varón de 47 años de edad, fumador de 20cigarrillos al día, que consulta por fiebre, expectoración purulenta y
dolor pleurítico de dos días de evolución. La Rx de tórax muestra una condensación lobar derecha y en el
hemocultivo se aisla un "Coco Gram positivo en cadena". Se inicia tratamiento con ceftriaxona 2g/24 g i.v.. A las
72 h. el paciente persiste febril a 39ºC y con afectación del estado general. ¿Cuál es la causa más probable de la
mala evolución?:
1. Dosis de ceftriaxona insuficiente.
2. Bacteria responsable (probablemente Streptococcus pneumoniae) resistente al tratamiento.
3. Proceso complicado por una abscesificación de la condensación.
4. Coinfección por Legionela spp.
5. Presencia de un empiema pleural.

8. Una paciente de 42 años consulta por presentar, tres horas después de la ingesta de un pastel de crema en un
restaurante, un cuadro de vómitos y deposiciones diarreicas sin productos patológicos ni fiebre. Mientras que la
paciente es visitada, su marido inicia un cuadro similar. ¿Cuál de los siguientes microorganismos es
probablemente el responsable del cuadro clínico?:
1. Salmonella enteriditis.
2. Shigella sonnei.
3. Staphylococcus aureus.
4. Campylobacter jejuni.
5. E. Coli.

9. Día: sábado. Lugar: urgencias de un hospital comarcal. Paciente con síndrome febril y sopor a la vuelta de Mali.
El laboratorio es incapaz de hacer con garantías una prueba para diagnóstico de paludismo. La actitud a seguir
más lógica sería:
1. Dar un tratamiento sintomátivco y esperar el lunes a que lo vea alguien más experto.
2. Tratar como si fuera una infección por P. Vivax.
3. Administrar una cefalosporina de 3ª generación y esperar la evolución.
4. Hacer una gota gruesa para enviar a centro especializado y tratar como si fuera una infección por P. falciparum.
5. Administrar una combinación de Metronidazol y Doxiciclina.

10. Un paciente infectado por el VIH recibe tratamiento antirretroviral con AZT + 3TC + Efavirenz, desde hace 14
meses. La última determinación de linfocitos CD4 y carga viral era de 350/mm3 y <200 copias/ml respectivamente.
Consulta por disnea y palidez y se constata una anemia microcítica (Hemoglobina: 7,8 gr/dl, VCH 68 fl). El
diagnóstico más probable es:
1. Crisis aplástica por Parvovirus B.
2. Leishmaniasis diseminada.
3. Infección diseminada por Mycobacterium avium-intracellulare.
4. Toxicidad farmacológica.
5. Hemorragia digestiva crónica.

11. Un paciente de 40 años diagnosticado de infección por VIH (virus de la inmunodeficiencia humana) hace 10
años que no sigue tratamiento antirretroviral presenta síntomas compatibles con candidiasis esofágica y además
refiere un cuadro de 10 días de evolución de cefalea, fiebre, vómitos y en las últimas 24 horas disminución del nivel
de conciencia; la exploración física muestra confusión y rigidez de nuca, el TAC de cráneo es normal y en la
punción lumbar existe una presión de apertura elevada, no se ven células y las proteinas son del 300 mg/ dl. El
cuadro es compatible con:
31
1. Hipertensión intracraneal benigna.
2. Hidrocefalia.
3. Miningitis tuberculosa.
4. Meningitis criptocócica.
5. Toxoplasmosis cerebral.

12. Señalar la FALSA respecto a la leucoencefalitis multifocal progresiva en paciente con infección por VIH:
1. Ocurre en un 4% de pacientes con SIDA.
2. El agente causal es el virus del papiloma humano.
3. No se conoce ningún tratamiento específico.
4. Los pacientes suelen presentar defectos focales múltiples sin alteraciones de la conciencia.
5. Se afecta la sustancia blanca subcortical.

13. Una mujer de 67 años con antecedentes de polineuropatía periférica ha sido recientemente diagnosticada de
Arteritis de células gitantes y puesta en tratamiento con 60 mg al día de prednisona. La radiografía de tórax es
normal y el test cutáneo con 5U de PPD es de 18 mm de induración. ¿Qué actitud tomaría con esta paciente?:
1. Hay que tratar con isoniacida durante 3 meses.
2. Esperar la finalización del tratamiento esteroideo y después realizar quimioprofilaxis.
3. No está indicado utilizar quimioprofilaxis.
4. Hay que tratar con isoniacida más estreptomicina.
5. Está indicado utilizar isoniacida más piridoxina entre 6 y 9 meses.

14. Acerca del tratamiento de la tuberculosis:


1. El régimen de elección es isoniacida +rifampicina+etambutol.
2. En la mayoría de los pacientes, son mejor 4 fármacos que 3 fármacos.
3. Todos los tratamientos que duren menos de 9 meses se asocian con una tasa muy elevada
de recidivas.
4. La asociación de pirazinamida durante los dos primeros meses, permite acortar la duración
del tratamiento a 6 meses.
5. La tuberculosis diseminada debe tratarse durante un mínimo de 12 meses.

15. Varón de 30 años de edad, sexualmente activo que presenta disuria y secreción uretral
purulenta. En la tinción de esta última se observan diplococos gramnegativos intracelulares.
¿Cuál cree que sería el tratamiento de elección?:
1. Penicilina benzatina.
2. Doxiciclina.
3. Azitromicina.
4. Ceftriaxona.
5. Ampicilina.

16. ¿Cuál de los siguientes tratamientos antibióticos no sería suficiente en monoterapia en


el manejo del paciente oncológico en tratamiento quimioterápico con neutropenia febril?:
1. Cefepima.
2. Ceftazidima.
3. Meropenem.
4. Imipenem.
5. Piperacilina-tazobactam.

17. Todos los siguientes virus tienen como célula diana la que se indica en cada caso, salvo
uno. Indíquela:
1. El virus de Epstein Barr los linfocitos B.
2. Los rinovirus las células epiteliales (receptor ICAM-1, proteína de adherencia de la superfamilia
de las inmunoglobulinas).
3. El virus de la rabia las neuronas (receptor de acetilcolina).
4. Los reovirus las células epiteliales (receptor sialil oligosacáridos).
5. El virus de la inmunodeficiencia humana los linfocitos T facilitadores (moléculas CD4).

18. Una de las siguientes afirmaciones referidas al lipopolisacárido (LPS) de la membrana externa de la pared
celular de las bacterias gram negativas es cierta:
1. Es una toxina termolábil que no resiste la esterilización en autoclave.
32
2. Tiene una actividad endotóxica que está asociada con el lípido A.
3. Contiene el antígeno O que es esencial para la viabilidad celular.
4. Su concentración en sangre no está directamente relacionada con la mortalidad por
shock irreversible y colapso cardiovascular.
5. No proporciona resistencia a la fagocitosis.

19. Algunas especies bacterianas son capaces de penetrar a través de los epitelios mucosos para así invadir y
diseminarse por los tejidos. Esta fagocitosis realizada por células epiteliales (no fagocíticas) está inducida por la
bacteria mediante sustancias denomiadas "internalinas" y se denomina al proceso "endocitosis dirigida por el
parásito". De todas las especies que se citan a continuación sólo una NO penetra mediante este proceso:
1. Chlamydia trachomatis.
2. Listeria monocytogenes.
3. Bordetella pertussis.
4. Haemophilus infuenzae.
5. Neisseria gonorroheae.

20. La diferencia entre Entamoeba histolytica y Entamoeba dispar (patógena y comensal respectivamente) se hace
en los laboratorios de microbiología clínica por:
1. Estudio de las diferencias morfológicas (microscópica) con ayuda de una tinción permanente.
2. Estudio de las diferencias metabólicas (Galeria metabólica).
3. Estudio de sus diferencias antigénicas mediante pruebas inmunológicas.
4. Estudio de las características diferenciales de la movilidad (pseudópodos).
5. Tan sólo puede establecerse mediante secuenciación del tRNA.

21. ¿Cuál de las siguientes entidades NO se acompaña de eosinofilia?:


1. Asma intrínseca.
2. Infectaciones por helmintos.
3. Enfermedad de Hodgkin.
4. Fiebre tifoidea.
5. Mastocitosis.

22. Respecto a las filariosis, señale la respuesta INCORRECTA:


1. Se transmiten por invasión directa de larvas parasitarias a la piel desde tierras húmedas
al andar descalzo.
2. Onchocerca volvulus produce nódulos subcutáneos, prurito y afectación ocular (queratitis,
retinitis).
3. Loa Loa produce edemas transitorios subcutáneos y conjuntivitis.
4. Wuchereria bancrofti produce varicocele y faringitis.
5. Ivermectina es el tratamiento de elección para la oncocercosis.

23. Joven de 23 años, que trabaja como pinchadiscos en un bar nocturno y tiene una historia de promiscuidad
bisexual e ingesta de drogas sintéticas, consulta por fiebre, malestar general y dolor en glande. La exploración física
muestra múltiples tatuajes, "piercings" y la presencia de tres lesiones vesiculosas en glande y adenopatías inguinales
bilaterales.¿Cuál es el diagnóstico más probable?:
1) Herpes genital.
2) Secundarismo luético.
3) Primoinfección VIH.
4) Infección por citomegalovirus.
5) Condilomas acuminados.

24. En cuanto a la neumonía por Neumococo en paciente infectados por el virus de la inmunodeficiencia humana
(VIH) es FALSO que:
1) Es una infección frecuentemente bacteriémica.
2) Tiene mayor incidencia que en la población general.
3) Junto a Haemophilus influenzae es la causa más común de neumonía en pacientes con SIDA.
4) Se recomienda vacuna neumocócica en aquellos pacientes con CD4 < 100 cel/ L.
5) Esta neumonía puede ser vista en pacientes con sistema inmune relativamente intacto.

25. La tuberculosis asociada a la infección por VIH se caracteriza por:


1) Presentación subclínica de la enfermedad.
33
2) Aparición característica en los estadios de inmunodepresión más severa (>50 CD4/mm3).
3) Elevada frecuencia de afectación extrapulmonar y diseminada.
4) Escaso rendimiento de los métodos microbiológicos de diagnóstico.
5) Mala respuesta al tratamiento antituberculoso.

26. NO es propio del shock tóxico estafilocócico:


1) Fiebre elevada.
2) Lesiones cutáneas.
3) Metástasis sépticas.
4) Fracaso renal.
5) Rabdomiólisis.

27. Hombre de 60 años, con antecedentes de neumoconiosis, que acude a nuestra consulta por tos y fiebre de 48 h de
duración. A la exploración se detectan estertores en la base izquierda, que se corresponde en la placa de tórax, con un
nuevo infiltrado en lóbulo inferior izquierdo; también se observan lesiones de neumoconiosis simple. La
pulsioximetría digital es normal y en los análisis: 14.000 leucocitos/ml. Se realiza un PPD, que es positivo (15 mm), y
en 3 baciloscopias de esputo no se observan bacilos ácido-alcohol resistentes. El paciente se trata con amoxicilina-
clavulánico y desaparece la fiebre y mejora la tos a las 24h. ¿Cuál es la actitud?:
1) Mantener dicho tratamiento 1 semana.
2) Mantener dicho tratamiento 2 semanas.
3) Administrar tratamiento tuberculostáticos convencional.
4) Mantener el tratamiento 2 semanas y administrar isoniacida durante 18 meses, si cultivo de Löwestein en esputo es
positivo.
5) Mantener el tratamiento 2 semanas y administrar isoniacida durante 12 meses, si cultivo de Löwestein en esputo es
negativo.

28. Una mujer de 54 años sufrió durante una transfusión de sangre un cuadro de fiebre y escalofríos que evolucionó
rápidamente a hipotensión refractaria, falleciendo unas horas después. A las 24 horas se recibió un hemocultivo de la
paciente con crecimiento de Pseudomonas fluorescens. Señale la respuesta FALSA:
1) Las especies de Pseudomonas no crecen a temperaturas bajas y son por tanto excepcionales contaminantes
bacterianos de productos hemáticos.
2) El síndrome de respuesta inflamatoria sistémico puede deberse a causas no infecciosas.
3) Se denomina shock séptico refractario al que dura más de una hora y no ha respondido a fluidos y vasopresores.
4) Los gérmenes Gram negativos son la causa más frecuente de septicemia grave.
5) En el síndrome de distres respiratorio agudo la presión capilar pulmonar es menor de 18 mm.

29. ¿Cuál de las siguientes permite diferenciar la pielonefritis aguda de la cistitis aguda?:
1) Leucocituria.
2) Hematuria.
3) Bacteriuria.
4) Antecedentes de infección urinaria.
5) Fiebre de más de 38.5ºC.

30. Un paciente de 22 años, que acude para evaluación de una lesión genital ulcerada, presenta un VDRL positivo a
título de 1:8 con un FTA-Abs negativo. La interpretación más adecuada de estos resultados es:
1) Falso positivo de las pruebas no treponémicas.
2) Falso negativo de las pruebas treponémicas.
3) Sífilis curada (pendiente la positivización del FTA-Abs).
4) Sífilis curada (pendiente la negativización del VDRL).
5) Sífilis de larga evolución.

31. Señale cuál de las siguientes asociaciones de helmintos y su clínica característica es INCORRECTA:
1) Ascaris lumbricoides - Síndrome de Löeffler.
2) Onchocerca volvulus - Ceguera de los ríos.
3) Ancylostoma duodenale - Anemia megaloblástica.
4) Strongyloides stercolaris - Síndrome de hiperinfestación en inmunodeprimidos.
5) Taenia solium - Convulsiones generalizadas.

34
32. Paciente de 30 años, seropositivo VIH conocido desde 5 años antes, con antecedentes de neumonía por P. Carinii,
que consulta por cefalea desde 10 días antes. La exploración física muestra como datos más relevantes mínima rigidez
de nuca y temperatura de 37,5ºC, fondo de ojo normal, TAC: ligera atrofia cortical. La punción lumbar da salida a
líquido claro con 40 células mononucleares, proteínas: 90 mgrs%, glucosa: 30 mg% (glucemia: 90 mg%). Señalar, de
entre las siguientes, la causa más probable:
1) Herpesvirus tipo 8.
2) Listeria.
3) Criptococo.
4) CMV.
5) VIH.

33. Paciente de 52 años, natural de Egipto, con 5 años de residencia en España, que ingresa en Urgencias por
hematemesis. A la exploración destaca esplenomegalia importante, la analítica hepática es normal, y en la ecografía
aparece fibrosis periportal. ¿En qué parasitosis pensaría?:
1) Clonorchis sinensis.
2) Schistosoma mansoni.
3) Fasciola hepatica.
4) Echinococcus.
5) Entamoeba histolytica.

34. Señale la cierta en relación a Haemophilus influenzae:


1) Haemophilus influenzae del tipo B es causa frecuente de otitis media en el adulto.
2) La mayoría de las cepas que producen infección bronquial son no capsuladas.
3) Más del 70% de cepas son resistentes a amplicilina por producción de beta-lactamasas.
4) La incidencia de neumonía por Haemophilus influenzae en adultos ha disminuido de forma drástica con el uso de la
vacuna conjugada.
5) La práctica totalidad de cepas son sensibles al Cotrimoxazol.

35. En relación con Streptococcus pyogenes y la faringoamigdalitis, ¿cuál de las siguientes afirmaciones NO es
correcta?:
1) En tratamiento de la faringoamigdalitis estreptocócica se efectúa con una sola inyección i.m. de 1,200,000 UU de
penicilina benzatina.
2) El tratamiento antibiótico de la faringoamigdalitis estreptocócica se efectúa con 250,000 UU/6h oral de penicilina V
durante 10 días.
3) El tratamiento antibiótico de la faringoamigdalitis estreptocócica se efectúa con una sola inyección i.m. de 1,200,000
UU de penicilina procaína.
4) El tratamiento antibiótico de la faringoamigdalitis estreptocócica se efectúa con amoxicilina oral 500 mg/8h durante 10
días.
5) El tratamiento antibiótico recomendado de la faringoamigdalitis estreptocócica en los casos de alergia a la penicilina es un
macrólido oral durante 10 días.

36. A lo largo de los últimos 10 años se ha producido un cambio muy importante en la etiología de la Endocarditis
Infecciosa del adulto. El microorganismo más frecuente en la actualidad, es:
1) Microorganismos del grupo HACEK.
2) Staphylococcus aureus.
3) Estafilococos coagulasa negativos.
4) Bacilos gram negativos.
5) Streptococcus viridans.

37. Una paciente de 44 años, sometida a vasectomía izquierda con vaciamiento ganglionar, consulta un año después de
la intervención por un cuadro febril con celulitis en brazo izquierdo. ¿Cuál es el agente etiológico más probable?:
1) Salmonella enteriditis.
2) Streptococcus agalactiae.
3) Escherichia coli.
4) Streptococcus pyogenes.
5) Pasteurella multocida.

38. El eritema migratorio es una lesión cutáneaanular característica de infección por:


1) Leptospira)
2) Salmonella typhi.
35
3) Rickettsia conorii.
4) Borrelia burgdorfferi.
5) Coxiella burnetti.

39. En lo referente a Clostridium difficile, señale cuál de las siguientes afirmaciones es INCORRECTA:
1) Se trata de un Bacilo Gram positivo esporulado.
2) Es responsable de algunas diarreas asociadas al uso de antimicrobianos.
3) Solo las cepas toxigénicas con patógenas.
4) Es responsable de la inmensa mayoría de las colitis pseudomembranosas.
5) Sólo causa enfermedad en pacientes previamente inmunodeprimidos.

40. ¿Cuál de las siguientes afirmaciones es FALSA, en relación a Listeria?:


1) Es un patógeno intracelular.
2) No está presente en la flora gastrointestinal normal en personas sanas.
3) Causa enfermedad, con más frecuencia , en personas con trastorno de la inmunidad celular.
4) La contaminación de alimentos es relativamente común.
5) La listeriosis asociada a la gestación puede causar muerte fetal intraútero.

41. Si un niño toma el contenido de un jarabe antitusígeno con sabor a limón, que sus padres guardaban en un
armario, cuyo principio activo es fosfato de codeína, es probable que los próximos días el niño muestre:
1) Retención urinaria.
2) Visión borrosa.
3) Candidiasis orofaríngea.
4) Estreñímiento.
5) Cloración rojiza de la orina.

42. En un niño de 8 años previamente sano, la causa más frecuente de meningitis bacteriana es:
1) H. influenzae.
2) S. pneumoniae.
3) N. meningitides.
4) S. aureus.
5) Estreptococos del grupo B.

43. En la meningitis meningocócica, es FALSO que:


1) Los brotes epidémicos suelen estar causados por los serotipos A y C.
2) Las cepas de meningococo serotipo Y se asocian con neumonía.
3) El déficit de los últimos componentes del complemento, de C5 a C8 y de properdina son factores predisponentes.
4) Los pacientes con un sistema de complemento normal tienen menor mortalidad.
5) La mortalidad de la meningitis aislada es menor que la de la sepsis meningocócica sin meningitis.

44. Un estudiante de derecho de 20 años, previamente sano, presenta un cuadro de febrícula, artromialgias, tos seca
persistente y astenia de dos semanas de evolución. En el último mes, sus dos hermanos de 9 y 17 años han presentado
consecutivamente un cuadros similar, que se ha autolimitado de forma progresiva. Tras practicársele una radiografía
de tórax, el médico le ha diagnosticado de neumonía atípica. ¿Cuál es el agente etiológico más probable en este caso?:
1) Coxiella burnetti (fiebre Q).
2) Virus sincitial respiratorio.
3) Haemophilus influenzae.
4) Mycoplasma pneumoniae.
5) Legionella pneumophila.

¿En cuál de las siguientes localizaciones de la enfermedad tuberculosa está indicado el tratamiento
coadyuvante con glucocorticoides para mejorar la supervivencia?:
1. Pulmonar.
2. Meníngea.
3. Ganglionar.
4. Genitourinaria.
5. Ostearticular.

36
2. Un hombre de 31 años que tenía contacto directo con perros, consultó por fiebre y cefalea de 4 días de
evolución. Estaba febril y tenía un exantema maculopapular en cara, tronco y extremidades incluyendo
palmas y plantas. Existía una lesión costrosa y negruzca entre las nalgas y adenopatías inguinales. ¿Cuál de
los siguientes gérmenes es el responsable?:
1. Borrelia burgdorferi.
2. Rickettsia typhi.
3. Treponema pallidum.
4. Rickettsia conorii.
5. Bartonella henselae.

3. La llamada meningitis de Mollaret es un cuadro, poco frecuente, consistente en episodios recurrentes de


meningitis en las que suelen verse células sugerentes de esta entidad en el LCR. Hoy se cree que:
1. Está causada por Enterovirus.
2. Está causada por Adenovirus.
3. Se trata de meningitis bacterianas abortadas por tratamiento antibiótico.
4. Se ha vinculado a Virus del Herpes Simple.
5. Es una forma frustrada de enfermedad tuberculosa.

4. Un hombre de 45 años acudió al área de Urgencias de un hospital por fiebre elevada y exantema máculo-
papuloso generalizado, incluyendo palmas y plantas. El paciente vive en el campo con perros
frecuentemente parasitados por garrapatas. Señale la enfermedad a la que se refiere, el germen causante y
el tratamiento adecuado:
1. Fiebre botonosa, Ricckettsia Conori: Doxiciclina.
2. Kala-azar, Leishmaniae Donovani: Antimoniales.
3. Dengue, Arenaviridae Aegypti: Tratamiento sintomático.
4. Fiebre Q, Coxiella Burnetti, Doxiciclina.
5. Fiebre de Malta, Brucella Mellitensis: Cotrimoxazol.

5. ¿Cuál de las siguientes relaciones anatómicas de la arteria renal izquierda NO es correcta?:


1. Situada por detrás y algo por encima de la vena renal izquierda.
2. Situada por detrás y debajo del cuerpo del páncreas.
3. Situada por detrás y debajo de la vena esplénica.
4. Se origina en la cara lateral izquierda de la aorta abdominal.
5. Situada por detrás de la vena cava inferior.

6. Un paciente de 40 años con antecedentes de adicción a drogas por vía parenteral, con infección por el
virus de la inmunodeficiencia humana (VIH) conocida desde hace 8 años, con un ingreso en institución
penitenciaria hace 2 años, en que presentaba un Mantoux de 7 mm, nunca ha recibido ningún tratamiento
ni profilaxis. En la actualidad se encuentra asintomático, su CD4 son de 100 cel/uL y el Mantoux negativo.
Sería INCORRECTO pensar que:
1. Debe recibir profilaxis con Isoniacida 300 mg + piridoxina durante 9 meses.
2. Debe recibir profilaxis con Rifabutina y Piracinamida durante 2 meses.
3. Probablemente presente una infección tuberculosa.
4. No debe recibir profilaxis si se encuentra en tratamiento deshabituador con Metadona.
5. Debe recibir profilaxis con Isonicida 900 mg + piridoxina, 2 veces por semana durante 9 meses.

7. Los agentes causales más frecuentes de meningitis viral son:


6. Arbovirus.
7. Herpesvirus.
8. Virus de la coriomeningitis linfocitaria.
9. Virus de la parotiditis epidémica.
10. Enterovirus.

37
8. ¿Cuál de los siguientes procesos linfoproliferativos corresponde a un linfoma de linfocitos T?:
1. Linfoma folicular.
2. Linfoma de células del manto.
3. Micosis fungoides.
4. Linfoma linfoplasmocitario.
5. Plasmocitoma.

9. Mujer de 33 años de edad ecuatoriana, acude a su médico de familia para la lectura de Mantoux,
realizado en el contexto de un estudio de contactos. Una prima suya que vive en su casa y duerme en la
misma habitación (junto con otras 7 personas), ha sido diagnosticada de una tuberculosis pulmonar
bacilífera (más de 50 bacilos por campo). Su médico aprecia una induración de 7 mm en la lectura del PPD.
¿Cuál de las siguientes afirmaciones es la correcta?:
6. Se trata de una infección tuberculosa y hay que iniciar tratamiento quimioprofiláctico de inmediato.
7. Se trata de una Tuberculosis y hay que comenzar con tratamiento antituberculoso.
8. Se trata de una infección tuberculosa y hay que descartar enfermedad tuberculosa, previo a comenzar el
tratamiento quimioprofiláctico.
9. El PPD no es significativo, ya que al tratarse de una persona inmigrante de una zona de alta prevalencia de TBC
debería tener una induración mayor de 10 mm.
10. El PPD no es significativo, ya que al tratarse de una persona inmigrante de una zona de alta prevalencia de TBC
debería tener una induración a mayor de 15 mm.

1. En la neumonía por Pneumococistis carinii en los pacientes con infección por VIH, ¿cuál de las siguientes
respuestas es la verdadera?:
6. El diagnóstico definitivo se realiza por cultivo del esputo en medios específicos.
7. El uso de glucocorticoides está contraindicado.
8. La Pentamidina intravenosa es el tratamiento alternativo de elección en las formas graves.
9. El riesgo de padecerla es independiente de la cifra de linfocitos CD4+.
10. Nunca está indicada la profilaxis primaria.

3. Señale, entre las siguientes, la conducta más adecuada ante un paciente leucopénico que comienza con
fiebre:
6. Observarle para vigilar si aparecen síntomas respiratorios.
7. Tomar los cultivos adecuados y comenzar tratamiento antibiótico empírico cuanto antes.
8. Tomar los cultivos adecuados y esperar a los resultados antes de iniciar ningún tratamiento.
9. Tratarle exclusivamente con factor estimulante de colonias de granulocitos.
10. Tratarle con paracetamol, sin mayor preocupación, pues no es probable que padezca una infección.

4. Todas las enfermedades siguientes, EXCEPTO una, se han relacionado con un agente infeccioso. Señálela:
1. Angiodisplasia de colon.
2. Sarcoma de Kaposi del inmunodeprimido.
3. Linfoma MALT gástrico.
4. Úlcera péptica.
5. Linfoma asociado a inmunodepresión.

5. Un joven de 16 años realiza un viaje de fin de curso por Europa. Al mes de regreso comienza con
malestar general, odinofagia y fiebre; en la exploración destaca hipertrofia amigdalar con exudado
blanquecino, adenopatías occipitales, laterocervicales dolorosas; en el hemograma se observa leucocitosis

38
de 15000/mm3 con un 70% de linfocitos, alguno de ellos atípico. Ante la sospecha diagnóstica se debe
realizar:
6. Biopsia ganglionar.
7. Biopsia de médula ósea.
8. Tratamiento con Penicilina.
9. Serología para virus de Epstein Barr.
10. Tratamiento con clindamicina.

7. Un paciente de 40 años con antecedentes de adicción a drogas por vía parenteral, con infección por el
virus de la inmunodeficiencia humana (VIH) conocida desde hace 8 años, con un ingreso en institución
penitenciaria hace 2 años, en que presentaba un Mantoux de 7 mm, nunca ha recibido ningún tratamiento
ni profilaxis. En la actualidad se encuentra asintomático, su CD4 son de 100 cel/uL y el Mantoux negativo.
Sería INCORRECTO pensar que:
6. Debe recibir profilaxis con Isoniacida 300 mg + piridoxina durante 9 meses.
7. Debe recibir profilaxis con Rifabutina y Piracinamida durante 2 meses.
8. Probablemente presente una infección tuberculosa.
9. No debe recibir profilaxis si se encuentra en tratamiento deshabituador con Metadona.
10. Debe recibir profilaxis con Isonicida 900 mg + piridoxina, 2 veces por semana durante 9 meses.

8. Es conocido el riesgo potencial de transmisión de VIH o hepatitis desde un paciente portador al equipo
médico-quirúrgico que le atiende. La política recomendada actualmente para evitar estos contagios
consiste en aplicar una serie de medidas especiales para evitar el contacto con sangre o líquidos orgánicos
del paciente. Para ello se debe:
6. Llevar a cabo las pruebas serológicas sistemáticamente a todos los pacientes y tomar medidas en los que den
algún resultado positivo.
7. Solicitar consentimiento previo y llevar a cabo pruebas serológicas a los pacientes que lo hayan otorgado,
tomando medidas en los portadores.
8. Averiguar el nivel de riesgo individual mediante historia clínica y aplicar las medidas a los pacientes con
elevada probabilidad de ser portadores.
9. Tomar precauciones especiales sólo en aquellos pacientes en que, por otro motivo, haya sido diagnosticados
previamente de portadores de alguno de los virus mencionados.
10. Considerar potenciales portadores a todos los pacientes y tomar medidas en todos ellos.

9. ¿Qué es el dengue?:
6. Una enfermedad causada por un poxvirus.
7. Una enfermedad limitada a los países del centro de Africa.
8. Una enfermedad vírica que puede producir una fiebre hemorrágica.
9. Una zoonosis que afecta al hombre ocasionalmente.
10. Una enfermedad vírica que ocasiona un eritema que evoluciona a mácula y pápula afectando fundamentalmente a
la población infantil.

1. Acerca de la fiebre de origen desconocido, ¿cuál de las siguientes afirmaciones es FALSA?:


6. Clásicamente se define como fiebre mayor de 38,3ºC, en varias ocasiones, durante más de tres semanas, sin
encontrarse un diagnóstico tras una semana de investigaciones en el hospital.
7. Hoy en día se prefiere clasificarla como clásica, en neutropénico, nosocomial y asociada a infección por VIH.
8. Es fundamental hacer una historia de los viajes realizados.
9. En pacientes neutropénicos (<50 neutrófilos/ mL), es útil realizar un tratamiento antibiótico empírico.
10. En pacientes que la presentan durante más de seis meses, la causa más frecuente es infecciosa.

39
2. Un paciente ingresa en urgencias con disminución del nivel de conciencia, fiebre de 39ºC y TA de 70/40
mmHg. ¿Cuál, de las siguientes, considera la actitud inicial más correcta?:
6. Iniciar tratamiento antibiótico empírico de amplio espectro.
7. Realizar una punción lumbar.
8. Bajar la fiebre.
9. Tomar muestras para hemocultivos.
10. Aportar líquidos intravenosos.

3. Paciente trasplantado renal de 2 meses de evolución que acude al servicio de urgencias por síndrome
febril de 3 días de evolución bien tolerado y acompañado de epigastralgias. En la analítica practicada
destaca una moderada leucopenia (2400/mm3) con una leve elevación en la cifra de transaminasas (ALT
75 UI/l; AST 89 Ul/l. ¿Cuál sería el primer diagnóstico de sospecha?:
6. Tuberculosis pulmonar.
7. Infección por Helicobacter pilorii.
8. Infección por Pneumocistis carinii.
9. Infección por Citomegalovirus.
10. Hepatitis por VHC.

4. El tratamiento antirretroviral en un paciente VIH+ con carga viral de 575.000/mm3 y una cifra de
linfocitos de CD4 de 450/mm3:
6. Es obligado.
7. Sólo estaría indicado si la carga viral fuera superior a 1 millón de copias/ml.
8. En ningún caso se prescribiría si los CD4+ son >200/ml.
9.
10. 000/mm3 y una cifra de linfocitos de CD4 de 450/mm3: 1. Es obligado. 2. Sólo estaría indicado si la carga viral
fuera superior a 1 millón de copias/ml. 3. En ningún caso se prescribiría si los CD4+ son >200/ml. 4. Está
indicado si el paciente lo desea. 5. En estas condiciones sólo estaría indicado en el contexto de un estudio clínico
prospectivo.

5. Una paciente de 45 años presenta de forma progresiva en los últimos 3 días un cuadro de cefalea,
deterioro del nivel de conciencia y fiebre de 39,5ºC. entre sus antecedentes destaca una enfermedad de
Crohn que ha requerido tratamiento de forma irregular en los últimos 3 años. Desde hace 6 meses está
tomando mesalazina y 15 mg de prednisona/ día. La exploración clínica no presenta datos significativos,
salvo discretos signos de irritación meníngea. El hemograma muestra 15.600 leucocitos, con una fórmula
normal. La bioquímica elemental es normal. La Rx de tórax no presenta alteraciones significativas. Se
realiza una punción lumbar, obteniéndose un LCR con proteínas de 560 mg/ dl, glucosa 25 mg/dl y 325
células/ml (85% células mononucleares). La tinción de Gram y de Ziehl-Neelsen en LCR son negativas.
¿Cuál es la actitud más correcta?:
6. Sospechar una meningoencefalitis autoinmune y subir la dosis de prednisona a 1 mg/ kg de peso, añadiendo
quimioprofilaxis con isoniacida.
7. Sospechar una meningitis por gérmenes de origen gastrointestinal. Iniciar tratamiento antibiótico empírico que
cubra gérmenes Gram (-) y anaerobios.
8. Sospechar una meningitis tuberculosa. Iniciar tratamiento inmediato con tres tuberculostáticos.
9.
10. 600 leucocitos, con una fórmula normal. La bioquímica elemental es normal. La Rx de tórax no presenta
alteraciones significativas. Se realiza una punción lumbar, obteniéndose un LCR con proteínas de 560 mg/ dl,
glucosa 25 mg/dl y 325 células/ml (85% células mononucleares). La tinción de Gram y de Ziehl-Neelsen en LCR
son negativas. ¿Cuál es la actitud más correcta?: 1. Sospechar una meningoencefalitis autoinmune y subir la dosis
de prednisona a 1 mg/ kg de peso, añadiendo quimioprofilaxis con

40
6. Un trabajador sanitario tiene una prueba de la tuberculina que mide 0 mm. Al repetirla 10 días después,
el diámetro de la induración mide 12 mm. ¿Cuál de las siguientes interpretaciones es más adecuada para
estos resultados?:
6. Primoinfección tuberculosa entre las dos pruebas.
7. Infección tuberculosa latente.
8. Presencia de anergia.
9. Tuberculosis activa hace años.
10. Mala realización de alguna de las pruebas.

7. ¿Cuál de las siguientes pruebas es la más adecuada para evaluar la respuesta terapéutica en un paciente
tratado de sífilis precoz?:
6. Realización de fondo oscuro sobre las lesiones que vayan apareciendo.
7. Examen anual de líquido cefalorraquídeo (VDRL).
8. Test de inmovilización del Treponema pallidum (TPI).
9. Cualquiera de las pruebas treponémicas (TPI, FTA, etc).
10. Evaluación seriada del título del VDRL o RPR (pruebas no treponémicas).

10. ¿Cuál de las siguientes afirmaciones, en cuanto a la Neumonía por Pneumocistis Carinii en pacientes
VIH (Virus de la inmunodeficiencia humana), positivos, es FALSA?:
6. Se presentan clínicamente de forma subaguda.
7. Con elevada frecuencia se prescribe tratamiento sin confirmación diagnóstica bacteriológica.
8. El 95% de los pacientes tiene un recuento de CD4<200cel/uL.
9. El diagnóstico se obtiene habitualmente mediante cultivo de secreciones bronquiales obtenidas por la
inducción del esputo.
10. La profilaxis primaria se puede suspender en pacientes con tratamiento antirretroviral que presenta una carga viral
suprimida (<500 cop/ml) y CD4>200cel/uL.

11. En la interpretación de los resultados de los hemocultivos practicados a un paciente con fiebre, ¿cuál
de los siguientes datos nos haría pensar que no estamos ante un caso de contaminación?:
6. Aislamiento de bacterias que normalmente colonizan la piel.
7. Aislamiento de cocos gram positivos.
8. Aislamiento de bacterias difteroides.
9. Aislamiento del mismo microorganismo en hemocultivos con la misma sensibilidad.
10. Aislamiento de un estafilococo meticilin resistente en un solo hemocultivo.

12. ¿Cuál, entre los siguientes, es un virus persistente que puede permanecer en latencia y reactivarse?:
6. El virus de la hepatitis A.
7. El virus respiratorio sincitial.
8. El citomegalovirus.
9. El virus de la poliomielitis.
10. El rotavirus.

13. Señale, entre las siguientes, la conducta más adecuada ante un paciente leucopénico que comienza con
fiebre:
6. Observarle para vigilar si aparecen síntomas respiratorios.
7. Tomar los cultivos adecuados y comenzar tratamiento antibiótico empírico cuanto antes.
8. Tomar los cultivos adecuados y esperar a los resultados antes de iniciar ningún tratamiento.
9. Tratarle exclusivamente con factor estimulante de colonias de granulocitos.
41
10. Tratarle con paracetamol, sin mayor preocupación, pues no es probable que padezca una infección.

14. Una joven de 18 años acude al hospital por fiebre y cefalea de varias horas de evolución. Los días
previos había notado dolor de garganta y tos. Se observó tendencia al sueño, rigidez de nuca y petequias
en conjuntivitis y extremidades. El LCR era turbio y contenía 36.000 leucocitos /mm3, 200 mg/dL de
proteínas y 20 mg/dL de glucosa (glucemia simultánea 120 mg/dL). El examen con Gram fue negativo.
Señale cuál de las siguientes afirmaciones NO es correcta:
6. El diagnóstico más probable es meningitis meningocócica.
7. Los meningococos son siempre sensibles a la penicilina y, por tanto, la penicilina G es el tratamiento de
elección.
8. Como un significativo porcentaje de meningococos del grupo C son resistentes a la penicilina, la cefotaxima es
un tratamiento más seguro.
9. El empleo de dexametasona reduce el riesgo de secuelas neurosensoriales en niños con meningitis
bacteriana, pero su uso en un caso como éste es cuestionable.
10. Se debe administrar rifampicina a las personas que conviven estrechamente con la paciente.

17. Un hombre de 45 años con leucemia mieloblástica aguda recibió su tercer ciclo de quimioterapia. A los
7 días desarrolló fiebre y un súbito empeoramiento del estado general. Presentaba una lesión nodular
equimótica y dolorosa, con centro ulcerado y rodeada de eritema y edema, en la pierna izquierda. Tenía
menos de 100 leucocitos/mm3, hemoglobina 7 gr/dL y 30.000 plaquetas/mm3. ¿Qué germen es aislado
con mayor frecuencia en un caso como éste?:
6. Aeromonas hydrophila.
7. Klebs
8. ¿Qué germen es aislado con mayor frecuencia en un caso como éste?: 1. Aeromonas hydrophila. 2. Klebsiella
pneumoniae. 3. Staphyloccocus aureus.
9. Pseudomonas aeruginosa.
10. Xantomonas maltophilia.

18. ¿En qué situación clínica el uso de la reacción en cadena de la polimerasa (PCR) se ha convertido en la
técnica de referencia para realizar su diagnóstico?:
6. Diagnóstico de SIDA.
7. Tuberculosis pulmonar no bacilífera.
8. Síndrome mononucleósido por CMV.
9. Encefalitis por virus Herpes Simplex.
10. Meningitis meningocócica.

20. Una paciente de 43 años, con una prótesis mitral implantada 3 semanas antes, acude a Urgencias por
fiebre de 5 días de evolución. Dos días más tarde el laboratorio de Microbiología informa del crecimiento
en 5 de los 6 frascos de hemocultivo de cocos Gram positivos en racimo. ¿Cuál, de los siguientes, es el
tratamiento de elección hasta conocer los resultados definitivos?:
6. Vancomicina + gentamicina + rifampicina.
7. Cloxacilina + gentamicina + rifampicina.
8. Penicilina + gentamicina + rifampicina.
9. Ceftriaxona.
10. Imipenem.

21. El término diferenciación aplicado al tejido neoplásico define:


6. El grado de similitud de las células neoplásicas desde el punto de vista morfológico y funcional con las
células normales de las que derivan.

42
7. La aparición dentro de un tumor de elementos neoplásicos que no están presentes en el órgano o en el tejido en el
cual se originan.
8. La presencia de un grado extremo de anaplasia.
9. La presencia de áreas tumorales de morfología diferente en cambios adyacentes del tumor (por ejemplo
adenocarcinoma y carcinoma epidermoide).
10. La variación de la morfología tumoral en la recurrencia de la enfermedad.

24. Los agentes causales más frecuentes de meningitis viral son:


6. Arbovirus.
7. Herpesvirus.
8. Virus de la coriomeningitis linfocitaria.
9. Virus de la parotiditis epidémica.
10. Enterovirus.

30. ¿Cuál es el patógeno involucrado más frecuentemente en la “diarrea del viajero”?:


6. Salmonella.
7. Giardia Lamblia.
8. Campylobacter.
9. Entamoeba Hystolítica.
10. Escherichia Coli.

31. ¿En cuál de las siguientes situaciones es más probable que podamos encontrar una reacción negativa a
la tuberculina?:
6. Niño de 2 años, inmunocompetente, al que se vacunó con BCG al nacer.
7. Mujer de 25 años que consulta por esterilidad y a quien se diagnostica tuberculosis genital.
8. Hombre de 45 años con lesiones pulmonares cavitarias y baciloscopia y cultivo positivos para M. tuberculosis.
9. Hombre de 70 años, tuberculoso antiguo, muy grave, con un patrón miliar en Rx de tórax y con
hemocultivo positivo para M. tuberculosis.
10. Hombre con hematuria, fiebre, orquiepididimitis derecha, dolores cólicos ureterales y cultivo positivo para M.
tuberculosis en orina.

37. ¿En cuál de las siguientes localizaciones de la enfermedad tuberculosa está indicado el tratamiento
coadyuvante con glucocorticoides para mejorar la supervivencia?:
6. Pulmonar.
7. Meníngea.
8. Ganglionar.
9. Genitourinaria.
10. Ostearticular.

38. Señale cuál de las siguientes afirmaciones es FALSA acerca de la vía piramidal:
6. En el mesencéfalo y protuberancia, la vía piramidal desciende por el mismo lado en el que se ha originado.
7. En el extremo inferior del bulbo se decusan únicamente las fibras que inervan la musculatura del tronco.
8. El origen de la vía se sitúa en el gyrus precentral.
9. El fascículo piramidal cruzado desciende por el cordón lateral de la médula.
10. La corteza cerebral donde se origina el fascículo piramidal está irrigada por las arterias cerebrales anterior y
media.

43
39. Adulto de 41 años que desde hace 1 año tiene brotes de lesiones habonosas diseminadas por todo el
cuerpo, de color rojo, que duran días y se acompañan de artralgias. Histológicamente hay un infiltrado
neutrofílico perivascular con leucocitoclasia, en dermis superior. Este cuadro corresponde a:
1. Una urticaria crónica idiopática.
2. Un edema angioneurótico.
3. Una urticaria física.
4. Una urticaria-vasculitis.
5. Una púrpura papulosa.

40. Señale lo correcto en relación con la coriorretinitis por citomegalovirus:


1. Ocurre en cualquier estudio de la infección.
2. Siempre cursa con viremia.
3. Siempre afecta a la cámara anterior del ojo.
4. Es progresiva y puede causar ceguera.
5. No precisa tratamiento de por vida.

41. Un paciente consulta por diarrea sanguinolenta y fiebre de 39º C de más de una semana de duración.
En el coprocultivo se aísla Campylobacter Jejuni. ¿Cuál de los siguientes antibióticos es el recomendado
como de primera elección?:
1. Amoxicilina.
2. Ciprofloxacino.
3. Cefotaxima.
4. Gentamicina.
5. Eritromicina.

42. Indique la verdadera entre las siguientes cuestiones referidas al rechazo de injertos:
1. La base celular de la aloreactividad es el reconocimiento por los macrófagos del receptor de las moléculas de
histocompatibilidad del donante que se convierten en esta situación particular en antígenos.
2. La enfermedad injerto contra huésped está asociada fundamentalmente con el trasplante renal y es una de las
principales causas del fracaso del mismo.
3. En la mayor parte de los trasplantes, si la selección del donante ha sido adecuada, no es necesario el uso de
fármacos inmunosupresores que complicarían la supervivencia del trasplante.
4. En el rechazo agudo, debido a la activación de las células CD4+ activadas, las diferencias entre las
moléculas de clase II inducen una respuesta alogénica más fuerte que la inducida por diferencias en las de
clase I.
5. El rechazo crónico es la pérdida de injertos a partir de tres meses. Su intensidad es más débil que en el agudo y
responde habitualmente a los inmunosupresores.

44. Enfermo de 55 años diagnosticado de gammapatía monoclonal presenta lesiones papulosas rosadas y
traslúcidas en regiones periorbitarias y peribucales junto con macroglosia. El diagnóstico más probable
es:
1. Linfoma de células B.
2. Mucinosis papulosa.
3. Amiloidosis AL.
4. Calcinosis cutáneo-mucosa.
5. Lupus eritematoso.

44
45. Indique cuál de las siguientes respuestas en relación a la inmunidad frente a los tumores es la correcta:
1. Los tumores desencadenan habitualmente una fuerte respuesta inmunogénica, pero ésta no es suficiente para
controlar su crecimiento.
2. Las células tumorales segregan citocinas que tienen una acción estimuladora sobre el sistema inmune creando un
sistema de retroalimentación positiva de su crecimiento.
3. Una causa habitual de propagación de un tumor es la demora inmune, que es la diferencia entre la cinética
del crecimiento tumoral y la formación de una respuesta inmune adaptativa eficaz.
4. Las células tumorales segregan citocinas que potencian la acción de los linfocitos CD4+ Th1 responsables
de la respuesta inflamatoria; este aumento de la respuesta inflamatoria crea una red alrededor del tumor
que le protege en su expansión.
5. La presencia de moléculas coestimuladoras, como CD80, en la superficie del tumor provoca una activación
anormal de los linfocitos T que evita una respuesta citolítica eficaz.

47. Una de las siguientes aseveraciones NO es correcta en la tuberculosis miliar. Señálela:


6. Se debe a la diseminación hematógena del bacilo.
7. La tinción de Ziehl del esputo suele ser negativa.
8. La prueba de la tuberculina es positiva en el 80% de los casos.
9. La biopsia de médula ósea puede proporcionar el diagnóstico.
10. Puede presentarse como manifestación de una infección tuberculosa primaria.

50. Ante un joven de 16 años que ha estado conviviendo con un enfermo con tuberculosis pulmonar activa
y tiene una prueba de la tuberculina negativa, la actitud más correcta, de las siguientes, es realizar:
6. Quimioprofilaxis durante 6 meses.
7. Vigilancia clínica y prueba de tuberculina a los 3 meses.
8. Quimioprofilaxis durante 2-3 meses y luego repetir la prueba de la tuberculina.
9. Tratamiento con rifampicina, isoniacida y piracinamida durante 6 meses.
10. Tratamiento con rifampicina, isoniacida y piracinamida y, a los 2 meses, repetir la prueba de la tuberculina.

. El germen causal más frecuente de artritis séptica entre pacientes entre 15 a 40 años es:
Staphylococcus aureus.
Haemophilus influenzae.
Neisseria gonorrhoeae.
Streptococcus pyogenes.
Streptococcus pneumoniae.

2. Un enfermo con neumonía, perteneciente a un brote epidémico de varias personas que ocasionalmente conviven
en un edificio, presenta un cuadro confusional desproporcionado a la fiebre, diarrea, hiponatremia y ligero
ascenso de las enzimas hepáticas. Entre las siguientes opciones, ¿cuál se debe incluir en el tratamiento?:
Cefalosporinas y aminoglucósidos a las dosis adecuadas.
Vancomicina a las dosis adecuadas.
Doxiciclina 100 mg/12 h durante 14 días.
Eritromicina 2-4 g/día durante 14 días.
Penicilina G procaína 0,6 – 1,2x106 U/12 h durante 7 días.

5. Un paciente con 57 años y diabetes mellitus mal controlada, comienza con fiebre, dolor profundo en seno
maxilar, congestión y secreción nasal serosanguinolenta. Se instaura tratamiento antibiótico sin objetivar mejoría.
En la evolución de la enfermedad aparece ptosis parpebral y deterioro del nivel de conciencia. En la TC se aprecia
opacificación de senos maxilares y frontales. Se extrae muestra del seno y en el laboratorio de microbiología
informan de la presencia de hifas no tabicadas. ¿Cuál es el diagnóstico más probable?:
Aspergillosis.
Mucormicosis.
Candidiasis invasora.
Actinomicosis.
45
Rinosporidiosis.

7. Un enfermero de Urgencias le consulta porque ha tenido un accidente en el que ha recibido un pinchazo


profundo, sin guantes, con una aguja gruesa visiblemente manchada de sangre, de un paciente adicto a drogas por
vía parenteral. Tras interrogar al paciente, declara que comparte habitualmente jeringuillas intravenosas y que
nunca se ha realizado una serología para el VIH. ¿Cuál de las siguientes es la actitud más correcta?:
Esperar al día siguiente a que esté el resultado de la serología de VIH.
Iniciar inmediatamente tratamiento con tres antiretrovirales.
Iniciar inmediatamente tratamiento con AZT.
Realizar serología de VIH, carga viral de VIH y test de resistencias genotípicas (en caso de carga viral
detectable) y revisar, cuando estén los resultados, la necesidad de tratamiento antiretroviral.
Tranquilizar al enfermo debido a bajo riesgo de transmisión del VIH y reevaluar en un mes.

8. Mujer de 33 años de edad ecuatoriana, acude a su médico de familia para la lectura de Mantoux, realizado en el
contexto de un estudio de contactos. Una prima suya que vive en su casa y duerme en la misma habitación (junto
con otras 7 personas), ha sido diagnosticada de una tuberculosis pulmonar bacilífera (más de 50 bacilos por
campo). Su médico aprecia una induración de 7 mm en la lectura del PPD. ¿Cuál de las siguientes afirmaciones es
la correcta?:
Se trata de una infección tuberculosa y hay que iniciar tratamiento quimioprofiláctico de inmediato.
Se trata de una Tuberculosis y hay que comenzar con tratamiento antituberculoso.
Se trata de una infección tuberculosa y hay que descartar enfermedad tuberculosa, previo a comenzar el
tratamiento quimioprofiláctico.
El PPD no es significativo, ya que al tratarse de una persona inmigrante de una zona de alta prevalencia de TBC
debería tener una induración mayor de 10 mm.
El PPD no es significativo, ya que al tratarse de una persona inmigrante de una zona de alta prevalencia de
TBC debería tener una induración a mayor de 15 mm.

10. ¿Cuál de las siguientes afirmaciones relativas a la fiebre tifoidea NO es correcta?:


La enfermedad se suele contraer por la ingesta de alimentos, agua o leche contaminados.
La leucopenia es más frecuente que la leucocitosis en las personas con enfermedad aguda.
La roseola se suele presentar en el momento en el que comienza la fiebre.
El cloranfenicol no es eficaz para prevenir las recaídas.
Las fluoroquinolonas erradican el germen, incluso en presencia de cálculos biliares.

11. Paciente de 64 años, fumador, que acude a urgencias por un cuadro de 48 h. de evolución de fiebre y tos con
expectoración mucopurulenta. La radiografía de tórax muestra una condensación alveolar en lóbulo inferior
derecho y un pequeño infiltrado en el lóbulo inferior izquierdo. La gasometría arterial muestra un pH de 7,39, una
pO2 de 54 mmHg y una pCO2 de 29 mmHg. ¿Cuál de las siguientes opciones terapéuticas le parece más
adecuada?:
Claritromicina 500 mg IV/12h.
Iprofloxacina 200 mg IV/12h.
Amoxicilina-Ácido clavulánico 1g IV/8h.
Ciprofloxacino 200 mg EV/12h + Claritromicina 500 mg IV/12h.
Ceftriaxona 2g IV/24h + Claritromicina 500 mg IV/12h.

13. La malaria es una enfermedad parasitaria erradicada en nuestro país, pero en los últimos años estamos
asitiendo a un resurgir de casos debidos a la inmigración y a los viajes a países tropicales. En relación a la malaria,
cuál de las siguientes afirmaciones es FALSA:
La gravedad de la infección está en relación con el grado de parasitemia.
La infección por plasmodium falciparum es la más grave.
La malaria cerebral es una manifestación tipica de plasmodium vivax.
El dato analítico más frecuente es una anemia normocítica.
La infección se ha descrito en adictos a drogas por vía parenteral que comparten jeringuillas.

14. Un paciente, fumador, de 60 años, acude a Urgencias con un cuadro de fiebre, tos y expectoración amarillenta
de 36 horas de evolución. Se le objetiva una condensación con broncograma aéreo en la base pulmonar derecha en
la Rx de tórax. ¿Cuál de las siguientes pruebas complementarias es de mayor utilidad para decidir su
hospitalización?:
Realización de tinción de Gram en una muestra de esputo.
TC torácica.
46
Hemocultivos seriados.
Gasometría arterial basal.
Aspirado transtraqueal.

17. ¿Cuál es la principal causa de epididimitis en varones heterosexuales activos menores de 35 años?:
Chlamydia trachomatis.
Neisseria gonorrhoeae.
Ureplasma urealyticum.
Escherichia coli.
Pseudomonas aeruginosa.

19. Un paciente de 32 años con infección VIH, ingresa con un cuadro de pancreatitis aguda. No es consumidor de
alcohol. La ecografía abdominal descarta litiasis biliar. Sigue tratamiento antirretroviral con zidovudina (AZT),
didanosina (ddI) e indinavir. ¿Qué se debe suprimir de su tratamiento?:
Didanosina (ddI).
Zidovudina (AZT).
Indinavir.
Los tres fármacos.
Nada.

21. Una de las siguientes propiedades referidas a los virus RNA es cierta:
La mayoría de ellos se multiplican en el núcleo.
La estructura del genoma no es determinante del mecanismo de transcripción y replicación.
No es necesario que codifiquen RNA polimerasas RNA dependientes.
No muestran tendencias a las mutaciones.
El genoma RNA (+) de los retrovirus se convierte en DNA, que es integrado en la cromatina del huésped y
transcrito como un gen celular.

22. Un niño de 14 años acude a su consulta por presentar herida por mordedura en antebrazo, con exudado
purulento que ha empeorado a pesar del tratamiento con clindamicina. ¿Cuál es la etiología más probable de la
infección de la herida?:
Staphylococcus Aureus.
Streptococo beta-hemolítico grupo A.
Eikenella Corrodens.
Capnocytophaga gingivalis.
Mycobacterium Tuberculosis. ANULADA

23. La candidiasis hepatoesplénica ocurre habitualmente en:


Paciente con SIDA.
Diabéticos tipo I con neuropatía avanzada.
Receptores de trasplante hepático en el período postrasplante precoz.
Pacientes con leucemia aguda y leucopenia prolongada en fase de resolución.
Pacientes con cirugía pancreática mayor y con nutrición parenteral.

24. El antibiótico de elección en el tratamiento de una neumonía producida por Legionella pneumophila es:
Eritromicina.
Penicilina G.
Cotrimoxazol.
Cefotaxima.
Clindamicina.

25. Un veterinario de 40 años consultó por fiebre de 3 semanas de duración con dolor de espalda en los últimos
días. Al final de la primera semana había tenido inflamación testicular que cedió con trimetoprim-sulfametoxazol.
La exploración era normal salvo por dolor a la percusión en la columna dorsolumbar. Tenía una hemoglobina de
12,2 g/dL, GTP 320 u/l (normal 6-45) y fosfatasa alcalina 230 u/l (normal 41-117). La resonancia nuclear
magnética mostraba epifisitis anterior de D12 y signos de espondilodiscitis D12-L
¿Cuál debe ser la prueba diagnóstica siguiente en este caso?: 1. Biopsia para cultivo de la médula ósea.
Punción aspiración vertebral.
Hemocultivos y aglutinaciones para Brucella.
Biopsia hepática.
47
Cultivo de orina y de secreción prostática.

28. Una mujer de 17 años ha sido diagnosticada recientemente de lupus eritematoso sistémico grave y puesta en
tratamiento con esteroides a dosis elevadas. La radiografía de tórax es normal y el Mantoux de 12x15 mm de
induración. ¿Qué actitud tomaría?:
Efectuar quimioprofilaxis con isoniacida durante 6 meses.
Esperar 2 años para realizar quimioprofilaxis.
No utilizar quimioprofilaxis.
Tratar con isoniacida más rifampicina.
Tratar con isoniacida más rifampicina más etambutol.

29. En relación a la sífilis, señale la afirmación correcta:


La lúes secundaria cursa excepcionalmente con manifestaciones cutáneas.
El control del tratamiento se puede realizar valorando los títulos de positividad de las pruebas treponémicas.
Durante los períodos de latencia de la enfermedad se negativizan las pruebas no treponémicas.
La peculiar estructura del treponema hace que no sea sensible a los antibióticos betalactámicos.
El chancro sifilítico es indurado, no doloroso y muy rico en bacterias.

30. Es conocido el riesgo potencial de transmisión de VIH o hepatitis desde un paciente portador al equipo médico-
quirúrgico que le atiende. La política recomendada actualmente para evitar estos contagios consiste en aplicar una
serie de medidas especiales para evitar el contacto con sangre o líquidos orgánicos del paciente. Para ello se debe:
Llevar a cabo las pruebas serológicas sistemáticamente a todos los pacientes y tomar medidas en los que den algún
resultado positivo.
Solicitar consentimiento previo y llevar a cabo pruebas serológicas a los pacientes que lo hayan otorgado,
tomando medidas en los portadores.
Averiguar el nivel de riesgo individual mediante historia clínica y aplicar las medidas a los pacientes con elevada
probabilidad de ser portadores.
Tomar precauciones especiales sólo en aquellos pacientes en que, por otro motivo, haya sido diagnosticados
previamente de portadores de alguno de los virus mencionados.
Considerar potenciales portadores a todos los pacientes y tomar medidas en todos ellos.

34. Señale, entre las siguientes, la conducta más adecuada ante un paciente leucopénico que comienza con fiebre:
Observarle para vigilar si aparecen síntomas respiratorios.
Tomar los cultivos adecuados y comenzar tratamiento antibiótico empírico cuanto antes.
Tomar los cultivos adecuados y esperar a los resultados antes de iniciar ningún tratamiento.
Tratarle exclusivamente con factor estimulante de colonias de granulocitos.
Tratarle con paracetamol, sin mayor preocupación, pues no es probable que padezca una infección.

37. La llamada meningitis de Mollaret es un cuadro, poco frecuente, consistente en episodios recurrentes de
meningitis en las que suelen verse células sugerentes de esta entidad en el LCR. Hoy se cree que:
Está causada por Enterovirus.
Está causada por Adenovirus.
Se trata de meningitis bacterianas abortadas por tratamiento antibiótico.
Se ha vinculado a Virus del Herpes Simple.
Es una forma frustrada de enfermedad tuberculosa.

39. En la neumonía por Pneumocistis carinii en los pacientes con infección por VIH ¿cuál de las siguientes
respuestas es la verdadera?:
El diagnóstico definitivo se realiza por cultivo del esputo en medios específicos.
El uso de glucocorticoides está contraindicado.
La Pentamidina intravenosa es el tratamiento alternativo de elección en las formas graves.
El riesgo de padecerla es independiente de la cifra de linfocitos CD4+.
Nunca está indicada la profilaxis primaria.

44. Una paciente de 45 años presenta de forma progresiva en los últimos 3 días un cuadro de cefalea, deterioro del
nivel de conciencia y fiebre de 39,5ºC. entre sus antecedentes destaca una enfermedad de Crohn que ha requerido
tratamiento de forma irregular en los últimos 3 años. Desde hace 6 meses está tomando mesalazina y 15 mg de
prednisona/ día. La exploración clínica no presenta datos significativos, salvo discretos signos de irritación
meníngea. El hemograma muestra 15.600 leucocitos, con una fórmula normal. La bioquímica elemental es normal.
48
La Rx de tórax no presenta alteraciones significativas. Se realiza una punción lumbar, obteniéndose un LCR con
proteínas de 560 mg/ dl, glucosa 25 mg/dl y 325 células/ml (85% células mononucleares). La tinción de Gram y de
Ziehl-Neelsen en LCR son negativas. ¿Cuál es la actitud más correcta?:
Sospechar una meningoencefalitis autoinmune y subir la dosis de prednisona a 1 mg/ kg de peso, añadiendo
quimioprofilaxis con isoniacida.
Sospechar una meningitis por gérmenes de origen gastrointestinal. Iniciar tratamiento antibiótico empírico que cubra
gérmenes Gram (-) y anaerobios.
Sospechar una meningitis tuberculosa. Iniciar tratamiento inmediato con tres tuberculostáticos.

600 leucocitos, con una fórmula normal. La bioquímica elemental es normal. La Rx de tórax no presenta alteraciones
significativas. Se realiza una punción lumbar, obteniéndose un LCR con proteínas de 560 mg/ dl, glucosa 25
mg/dl y 325 células/ml (85% células mononucleares). La tinción de Gram y de Ziehl-Neelsen en LCR son
negativas. ¿Cuál es la actitud más correcta?: 1. Sospechar una meningoencefalitis autoinmune y subir la dosis de
prednisona a 1 mg/ kg de peso, añadiendo quimioprofilaxis con

46. Enfermo de 55 años diagnosticado de gammapatía monoclonal presenta lesiones papulosas rosadas y
traslúcidas en regiones periorbitarias y peribucales junto con macroglosia. El diagnóstico más probable es:
Linfoma de células B.
Mucinosis papulosa.
Amiloidosis AL.
Calcinosis cutáneo-mucosa.
Lupus eritematoso.

48. Las manifestaciones clínicas del tétanos se producen por la:


Invasión tisular directa del sistema nervioso central.
Producción de una exotoxina en la herida infectada.
Producción de citoquinas.
Destrucción tisular marcada en la zona infectada con sobreinfección bacteriana polimicrobiana secundaria.
Destrucción bacteriana masiva al iniciar el tratamiento antibiótico.

50. ¿Cuál, de los siguientes, es con más frecuencia el germen causal de espondilodiscitis piógena en pacientes con
SIDA, adictos a drogas por vía parenteral?:
Salmonella.
Haemophilus influenzae.
Pseudomonas aeruginosa.
Streptococcus granulosus.
Escherichia coli.

1. Paciente de 64 años, fumador, que acude a urgencias por un cuadro de 48 h. de evolución de fiebre y tos
con expectoración mucopurulenta. La radiografía de tórax muestra una condensación alveolar en lóbulo
inferior derecho y un pequeño infiltrado en el lóbulo inferior izquierdo. La gasometría arterial muestra un
pH de 7,39, una pO2 de 54 mm Hg y una pCO2 de 29 mmHg. ¿cuál de las siguientes opciones terapéuticas le
parece más adecuada?:

1. Claritromicina 500 mg IV/12h.


2. Ciprofloxacina 200 mg IV/12h.
3. Amoxicilina-Ácido clavulánico 1g IV/8h.
4. Cirprofloxacino 200 mgIV/12h + Claritromicina 500 mgIV/12h.
5. Ceftriaxona 2g IV/24h + Claritromicina 500 mg IV/12h.

3. El germen causal más frecuente de artritis séptica entre pacientes entre 15 a 40 años es:

1. Staphylococcus aureus.
2. Haemophilus influenzae.
3. Neisseria gonorrhoeae.
4. Streptococcus pyogenes.
5. Streptococcus pneumoniae.

49
7. Acerca del tratamiento de la tuberculosis:

1. El régimen de elección es isoniacida +rifampicina+etambutol.


2. En la mayoría de los pacientes, son mejor 4 fármacos que 3 fármacos.
3. Todos los tratamientos que duren menos de 9 meses se asocian con una tasa muy elevada de recidivas.
4. La asociación de pirazinamida durante los dos primeros meses, permite acortar la duración del
tratamiento a 6 meses.
5. La tuberculosis diseminada debe tratarse durante un mínimo de 12 meses.

9. ¿Cuál de los siguientes tratamientos antibióticos no sería suficiente en monoterapia en el manejo del
paciente oncológico en tratamiento quimioterápico con neutropenia febril?:

1. Cefepima.
2. Ceftazidima.
3. Meropenem.
4. Imipenem.
5. Piperacilina-tazobactam.

11. Los pacientes que desarrollan rechazo crónico del injerto tras trasplante renal, hepático o cardiaco,
presentan como lesión común a todos ellos:

1. Infiltración inflamatoria intersticial de carácter mixto.


2. Necrosis fibrinoide de la pared vascular.
3. Proliferación fibrosa endointimal arterial esteno-sante.
4. Angiogénesis difusa del injerto.
5. Infecciones frecuentes de repetición del injerto por gérmenes piógenos.

13. En relación a la sífilis, señale la afirmación correcta:

1. La lúes secundaria cursa excepcionalmente con manifestaciones cutáneas.


2. El control del tratamiento se puede realizar valorando los títulos de positividad de las pruebas treponémicas.
3. Durante los períodos de latencia de la enfermedad se negativizan las pruebas no treponémicas.
4. La peculiar estructura del treponema hace que no sea sensible a los antibióticos betalactámicos.
5. El chancro sifilítico es indurado, no doloroso y muy rico en bacterias.

14. Un niño de 14 años acude a su consulta por presentar herida por mordedura en antebrazo, con
exudado purulento que ha empeorado a pesar del tratamiento con clindamicina. ¿Cuál es la etiología más
probable de la infección de la herida?:

1. Staphylococcus Aureus.
2. Streptococo beta-hemolítico grupo A.
3. Eikenella Corrodens.
4. Capnocytophaga gingivalis.

16. Una paciente de 43 años, con una prótesis mitral implantada 3 semanas antes, acude a Urgencias por
fiebre de 5 días de evolución. Dos días más tarde el laboratorio de Microbiología informa del crecimiento
en 5 de los 6 frascos de hemocultivo de cocos Gram positivos en racimo. ¿Cuál, de los siguientes, es el
tratamiento de elección hasta conocer los resultados definitivos?:

1. Vancomicina + gentamicina + rifampicina.


2. Cloxacilina + gentamicina + rifampicina.
3. Penicilina + gentamicina + rifampicina.
4. Ceftriaxona.

50
19. La malaria es una enfermedad parasitaria erradicada en nuestro país, pero en los últimos años
estamos asitiendo a un resurgir de casos debidos a la inmigración y a los viajes a países tropicales. En
relación a la malaria, cuál de las siguientes afirmaciones es FALSA:

1. La gravedad de la infección está en relación con el grado de parasitemia.


2. La infección por plasmodium falciparum es la más grave.
3. La malaria cerebral es una manifestación tipica de plasmodium vivax.
4. El dato analítico más frecuente es una anemia normocítica.
5. La infección se ha descrito en adictos a drogas por vía parenteral que comparten jeringuillas.

20. Señale lo correcto en relación con la coriorretinitis por citomegalovirus:

1. Ocurre en cualquier estudio de la infección.


2. Siempre cursa con viremia.
3. Siempre afecta a la cámara anterior del ojo.
4. Es progresiva y puede causar ceguera.

22. En relación a los mecanismos de tolerancia a lo propio por los linfocitos B, señale cuál de estas
afirmaciones es FALSA:

1. Los mecanismos de adquisición de tolerancia a nivel central garantizan la ausencia de linfocitos B de


carácter autorreactivo a nivel periférico.
2. A nivel de médula ósea cuando los linfocitos B reconocen mediante sus receptores específicos una molécula
presente en la superficie celular, son eliminados por apoptosis.
3. A nivel de médula ósea cuando dos linfocitos B reconocen mediante sus receptores específicos una molécula que
se encuentra de forma soluble, quedan en situación de anergia.
4. Tan sólo aquellos linfocitos B que no han reconocido ningún tipo de antígeno en la médula ósea migran hacia
otros tejidos linfoides.
5. A nivel periférico, los linfocitos B autorreactivos no reaccionan contra lo propio por falta de cooperación con los
linfocitos T.

24. Hombre de 60 años con antecedentes de neumoconiosis, que acude a nuestra consulta por tos y fiebre
de 48 h de duración. A la exploración se detectan estertores en la base izquierda, que se corresponde en la
placa de tórax, con un nuevo infiltrado en lóbulo inferior izquierdo; también se observan lesiones de
neumoconiosis simple. La pulsioximetría digital es normal y en los análisis: 14.000 leucocitos/ml. Se
realiza un PPD que es positivo (15 mm) y en 3 baciloscopias de esputo no se observan bacilos ácido-
alcohol resistentes. El paciente se trata con amoxicilina-clavulánico y desaparece la fiebre y mejora la tos a
las 24 h. ¿Cuál es la actitud?:

1. Mantener dicho tratamiento 1 semana.


2. Mantener dicho tratamiento 2 semanas.
3.
4. 000 leucocitos/ml. Se realiza un PPD que es positivo (15 mm) y en 3 baciloscopias de esputo no se observan
bacilos ácido-alcohol resistentes. El paciente se trata con amoxicilina-clavulánico y desaparece la fiebre y mejora
la tos a las 24 h. ¿Cuál es la actitud?: 1. Mantener dicho tratamiento 1 semana. 2. Mantener dicho tratamiento 2
semanas. 3. Administrar tratamiento tuberculostáticos convencional. 4. Mantener el tratamiento 2 semanas y
administrar isoniacida durante 18 meses, si cultivo de Löwestein en esputo es positivo.
5. Mantener el tratamiento 2 semanas y administrar isoniacida durante 12 meses, si cultivo de Löwestein en
esputo es negativo.

51
27. Un enfermero de Urgencias le consulta porque ha tenido un accidente en el que ha recibido un
pinchazo profundo, sin guantes, con una aguja gruesa visiblemente manchada de sangre, de un paciente
adicto a drogas por vía parenteral. Tras interrogar al paciente, declara que comparte habitualmente
jeringuillas intravenosas y que nunca se ha realizado una serología para el VIH. ¿Cuál de las siguientes es
la actitud más correcta?:

1. Esperar al día siguiente a que esté el resultado de la serología de VIH.


2. Iniciar inmediatamente tratamiento con tres antiretrovirales.
3. Iniciar inmediatamente tratamiento con AZT.
4. Realizar serología de VIH, carga viral de VIH y test de resistencias genotípicas (en caso de carga viral detectable)
y revisar, cuando estén los resultados, la necesidad de tratamiento antiretroviral.
5. Tranquilizar al enfermo debido a bajo riesgo de transmisión del VIH y reevaluar en un mes.

28. La candidiasis hepatoesplénica ocurre habitualmente en:

1. Paciente con SIDA.


2. Diabéticos tipo I con neuropatía avanzada.
3. Receptores de trasplante hepático en el período postrasplante precoz.
4. Pacientes con leucemia aguda y leucopenia prolongada en fase de resolución.
5. Pacientes con cirugía pancreática mayor y con nutrición parenteral.

29. Día: sábado. Lugar: urgencias de un hospital comarcal. Paciente con síndrome febril y sopor a la vuelta
de Mali. El laboratorio es incapaz de hacer con garantías una prueba para diagnóstico de paludismo. La
actitud a seguir más lógica sería:

1. Dar un tratamiento sintomátivco y esperar el lunes a que lo vea alguien más experto.
2. Tratar como si fuera una infección por P. Vivax.
3. Administrar una cefalosporina de 3ª generación y esperar la evolución.
4. Hacer una gota gruesa para enviar a centro especializado y tratar como si fuera una infección por P.
falciparum.

31. ¿Cuál de los siguientes antimicrobianos utilizaría en primer lugar en monoterapia, para tratar una
infección grave por Staphylococcus Aureus resistente a meticilina?:

1. Eritromicina.
2. Rifampicina.
3. Vancomicina.
4. Levofloxacino.
5. Linezolide.

32. Enferma de 40 años consulta por la aparición hace dos meses de pápulas poligonales de color violáceo
en cara anterior de muñecas con prurito intenso. Además presenta lesiones en mucosa oral de aspecto
reticulado blanquecino. El diagnóstico más probable es:

1. Psoriasis.
2. Liquen plano.
3. Pitiriasis rubra pilaris.
4. Dermatofitosis.
5. Eccema numular.

35. ¿Cuál de las siguientes entidades NO se acompaña de eosinofilia?:

1. Asma intrínseca.
2. Infectaciones por helmintos.
3. Enfermedad de Hodgkin.
4. Fiebre tifoidea.
5. Mastocitosis.

52
38. Un paciente ingresa en el hospital con fiebre de más de 15 días de evolución, observándose en la Rx de
tórax un infiltrado pulmonar con cavitación en el lóbulo superior derecho. ¿Cuál de las siguientes
afirmaciones es correcta?:

1. Un Mantoux negativo excluye el diagnóstico de tuberculosis.


2. El paciente tiene una tuberculosis miliar.
3. Se debe realizar broncoscopia al día siguiente.
4. Debemos recoger al menos tres muestras de esputo para búsqueda de bacilos ácido-alcohol resistentes.
5. El crecimiento de M. Tuberculosis en medio de Lowenstein requiere de 4 a 5 días.

39. Paciente de 20 años con dolor de garganta, fiebre, subictericia y adenopatías cervicales y polo de bazo
palpable. En la analítica presenta leucocitosis con linfocitosis, "células linfoides activadas" y elevación
discreta de transaminasa. ¿Cuál es el diagnóstico más probable?:

1. Infección por virus de Epstein-Barr.


2. Hepatitis A aguda.
3. Infección por citomegalovirus.
4. Hepatitis B aguda.
5. Infección por herpes virus.

40. En relación con el Streptococcus Pneumoniae, ¿cuál de las siguientes frases NO es correcta?:

1. El neumococo es el principal agente bacteriano en la etiología de la otitis media aguda.


2. En los pacientes con neumonía neumocócica la bacteriemia se detecta en el 25% de los casos.
3. La mortalidad de la neumonía no bacteriémica es inferior al 5%.
4. En España, la resistencia del neumococo a la penicilina está entre el 30% y el 40%.
5. A la espera del antibiograma, el tratamiento recomendado de la meningitis neumocócica es vancomicina 1
g i.v./12 horas, durante unos 10 días.

41. Un trabajador sanitario tiene una prueba de la tuberculina que mide 0 mm. Al repetirla 10 días
después, el diámetro de la induración mide 12 mm. ¿Cuál de las siguientes interpretaciones es más
adecuada para estos resultados?:

1. Primoinfección tuberculosa entre las dos pruebas.


2. Infección tuberculosa latente.
3. Presencia de anergia.
4. Tuberculosis activa hace años.
5. Mala realización de alguna de las pruebas.

42. Un enfermo con neumonía, perteneciente a un brote epidémico de varias personas que ocasionalmente
conviven en un edificio, presenta un cuadro confusional desproporcionado a la fiebre, diarrea,
hiponatremia y ligero ascenso de las enzimas hepáticas. Entre las siguientes opciones, ¿cuál se debe incluir
en el tratamiento?:

1. Cefalosporinas y aminoglucósidos a las dosis adecuadas.


2. Vancomicina a las dosis adecuadas.
3. Doxiciclina 100 mg/12 h durante 14 días.
4. Eritromicina 2-4 g/día durante 14 días.
5. Penicilina G procaína 0,6 – 1,2x106 U/12 h durante 7 días.

44. Paciente trasplantado renal de 2 meses de evolución que acude al servicio de urgencias por síndrome
febril de 3 días de evolución bien tolerado y acompañado de epigastralgias. En la analítica practicada
destaca una moderada leucopenia (2400/mm3) con una leve elevación en la cifra de transaminasas (ALT
75 UI/l; AST 89 Ul/l. ¿Cuál sería el primer diagnóstico de sospecha?:

1. Tuberculosis pulmonar.
2. Infección por Helicobacter pilorii.
3. Infección por Pneumocistis carinii.

53
4. Infección por Citomegalovirus.
5. Hepatitis por VHC.

46. Un veterinario de 40 años consultó por fiebre de 3 semanas de duración con dolor de espalda en los
últimos días. Al final de la primera semana había tenido inflamación testicular que cedió con trimetoprim-
sulfametoxazol. La exploración era normal salvo por dolor a la percusión en la columna dorsolumbar.
Tenía una hemoglobina de 12,2 g/dL, GTP 320 u/l (normal 6-45) y fosfatasa alcalina 230 u/l (normal 41-
117). La resonancia nuclear magnética mostraba epifisitis anterior de D12 y signos de espondilodiscitis
D12-L

1. ¿Cuál debe ser la prueba diagnóstica siguiente en este caso?: 1. Biopsia para cultivo de la médula ósea.
2. Punción aspiración vertebral.
3. Hemocultivos y aglutinaciones para Brucella.
4. Biopsia hepática.
5. Cultivo de orina y de secreción prostática.

47. Una mujer de 17 años ha sido diagnosticada recientemente de lupus eritematoso sistémico grave y
puesta en tratamiento con esteroides a dosis elevadas. La radiografía de tórax es normal y el Mantoux de
12x15 mm de induración. ¿Qué actitud tomaría?:

1. Efectuar quimioprofilaxis con isoniacida durante 6 meses.


2. Esperar 2 años para realizar quimioprofilaxis.
3. No utilizar quimioprofilaxis.
4. Tratar con isoniacida más rifampicina.
5. Tratar con isoniacida más rifampicina más etambutol.

48. En la endocarditis infecciosa, ¿cuál, entre los que se mencionan, es el germen responsable con más
frecuencia de los casos con hemocultivo negativo?:

1. Staphylococcus Aureus resistente a meticilina.


2. Enterococcus Facium.
3. Streptococcus Salivarius.
4. Coxiella Burnetii.
5. Bacteroides Fragilis.

INFECTO
1 Un paciente de 30 años adicto a drogas por vía parenteral, se presenta con un cuadro de 3 días de evolución de tiritona, fiebre, dolor
torácico y tos con expectoración verdosa con "hilillos" de sangre; en la radiografía de tórax presenta varios infiltrados pulmonares
con cavitación central en alguno de ellos. El diagnóstico más probable sería:
1. Neumonía neumocócica.
2. Tuberculosis pulmonar.
3. Neumonía por anaerobios.
4. Neumonía hematógena.
5. Neumonía por H.Influenzae

2. Un paciente de 35 años de edad acude al servicio de urgencias por presentar fiebre elevada de hasta 39ºC, junto con confusión
mental. En la exploración física destaca la existencia de lesiones cutáneas en pie izquierdo a nivel distal, maculares, de milímetros de
diámetro, de aspecto isquémico hemorrágico y la auscultación cardiopulmonar es normal. A los pocos día se obtiene crecimiento de
Staphylococo aureus meticilin sensible en tres hemocultivos de tres obtenidos. ¿Cuál de las siguientes sería la actitud correcta a seguir
en ese momento?:
1. Considerar el resultado de los hemocultivos como probable contaminación.
2. Pautar de inmediato tratamiento antibiótico con penicilina y gentamicina durante 10 días.
3. Comenzar tratamiento con cloxacilina y gentamicina y realizar estudio ecocardiográfico por la existencia probable de endocarditis
aguda.

54
4. Descartar la existencia de endocarditis por la ausencia de soplos en la auscultación cardiaca y buscar focos de posible osteomielitis.
5. Realizar TAC abdominal urgente por probable absceso abdominal.

3. Señale la cierta, en relación a la encefalitis herpética:


1. Está producida casi siempre por el virus del herpes simple tipo 2.
2. La clínica característica es la presencia de fiebre con síntomas focales del lóbulo occipital.
3. La PCR en el líquido cefalorraquídeo no es útil para establecer el diagnóstico.
4. El tratamiento de elección es el zanamivir i.v.
5. Las secuelas neurológicas son frecuentes.

4. ¿Cuál de los siguientes cuadros clínicos no se asocia con toxoplasmosis?:


1. Abscesos cerebrales en personas con SIDA.
2. Miocarditis en transplantados cardiacos.
3. Coriorretinitis en adultos inmunocompetentes.
4. Colitis asociada a inmunodeficiencia humoral.
5. Síndrome mononucleósico en personas sanas.

5 Señale la afirmación INCORRECTA con respecto a Haemophilus influenzae:


1. Es un parásito obligado de las mucosas humanas.
2. Es un bacilo gramnegativo pequeño y pleomórfico.
3. La vacunación es de poca utilidad porque no cubre el serotipo B, que es el más prevalente en nuestro ambiente.
4. Las infecciones no sistémicas (otitis, bronquitis, conjuntivitis,...) están producidas generalmente por serotipos diferentes al B.
5. Sólo las cepas capsuladas causan infecciones sistémicas.

6. Señale la cierta respecto a la Fiebre Q:


1. Se transmite por inhalación de partículas contaminadas.
2. Los hemocultivos son positivos en la fase inicial.
3. La mancha negra se observa en el 60% de los casos.
4. El exantema suele afectar palmas y plantas.
5. El tratamiento de elección es la gentamicina.

7. Varón de 47 años de edad, fumador de 20cigarrillos al día, que consulta por fiebre, expectoración purulenta y dolor pleurítico de
dos días de evolución. La Rx de tórax muestra una condensación lobar derecha y en el hemocultivo se aisla un "Coco Gram positivo
en cadena". Se inicia tratamiento con ceftriaxona 2g/24 g i.v.. A las 72 h. el paciente persiste febril a 39ºC y con afectación del estado
general. ¿Cuál es la causa más probable de la mala evolución?:
1. Dosis de ceftriaxona insuficiente.
2. Bacteria responsable (probablemente Streptococcus pneumoniae) resistente al tratamiento.
3. Proceso complicado por una abscesificación de la condensación.
4. Coinfección por Legionela spp.
5. Presencia de un empiema pleural.

8. Una paciente de 42 años consulta por presentar, tres horas después de la ingesta de un pastel de crema en un restaurante, un
cuadro de vómitos y deposiciones diarreicas sin productos patológicos ni fiebre. Mientras que la paciente es visitada, su marido inicia
un cuadro similar. ¿Cuál de los siguientes microorganismos es probablemente el responsable del cuadro clínico?:
1. Salmonella enteriditis.
2. Shigella sonnei.
3. Staphylococcus aureus.
4. Campylobacter jejuni.
5. E. Coli.

9. Día: sábado. Lugar: urgencias de un hospital comarcal. Paciente con síndrome febril y sopor a la vuelta de Mali. El laboratorio es
incapaz de hacer con garantías una prueba para diagnóstico de paludismo. La actitud a seguir más lógica sería:
1. Dar un tratamiento sintomátivco y esperar el lunes a que lo vea alguien más experto.
2. Tratar como si fuera una infección por P. Vivax.
3. Administrar una cefalosporina de 3ª generación y esperar la evolución.
4. Hacer una gota gruesa para enviar a centro especializado y tratar como si fuera una infección por P. falciparum.
5. Administrar una combinación de Metronidazol y Doxiciclina.

10. Un paciente infectado por el VIH recibe tratamiento antirretroviral con AZT + 3TC + Efavirenz, desde hace 14 meses. La última
determinación de linfocitos CD4 y carga viral era de 350/mm3 y <200 copias/ml respectivamente. Consulta por disnea y palidez y se
constata una anemia microcítica (Hemoglobina: 7,8 gr/dl, VCH 68 fl). El diagnóstico más probable es:
1. Crisis aplástica por Parvovirus B.

55
2. Leishmaniasis diseminada.
3. Infección diseminada por Mycobacterium avium-intracellulare.
4. Toxicidad farmacológica.
5. Hemorragia digestiva crónica.

11. Un paciente de 40 años diagnosticado de infección por VIH (virus de la inmunodeficiencia humana) hace 10 años que no sigue
tratamiento antirretroviral presenta síntomas compatibles con candidiasis esofágica y además refiere un cuadro de 10 días de
evolución de cefalea, fiebre, vómitos y en las últimas 24 horas disminución del nivel de conciencia; la exploración física muestra
confusión y rigidez de nuca, el TAC de cráneo es normal y en la punción lumbar existe una presión de apertura elevada, no se ven
células y las proteinas son del 300 mg/ dl. El cuadro es compatible con:
1. Hipertensión intracraneal benigna.
2. Hidrocefalia.
3. Miningitis tuberculosa.
4. Meningitis criptocócica.
5. Toxoplasmosis cerebral.

11. Señalar la FALSA respecto a la leucoencefalitis multifocal progresiva en paciente con infección por VIH:

1. Ocurre en un 4% de pacientes con SIDA.


2. El agente causal es el virus del papiloma humano.
3. No se conoce ningún tratamiento específico.
4. Los pacientes suelen presentar defectos focales múltiples sin alteraciones de la conciencia.
5. Se afecta la sustancia blanca subcortical.

13. Una mujer de 67 años con antecedentes de polineuropatía periférica ha sido recientemente diagnosticada de Arteritis de células
gitantes y puesta en tratamiento con 60 mg al día de prednisona. La radiografía de tórax es normal y el test cutáneo con 5U de PPD es
de 18 mm de induración. ¿Qué actitud tomaría con esta paciente?:
1. Hay que tratar con isoniacida durante 3 meses.
2. Esperar la finalización del tratamiento esteroideo y después realizar quimioprofilaxis.
3. No está indicado utilizar quimioprofilaxis.
4. Hay que tratar con isoniacida más estreptomicina.
5. Está indicado utilizar isoniacida más piridoxina entre 6 y 9 meses.

14. Acerca del tratamiento de la tuberculosis:


1. El régimen de elección es isoniacida +rifampicina+etambutol.
2. En la mayoría de los pacientes, son mejor 4 fármacos que 3 fármacos.
3. Todos los tratamientos que duren menos de 9 meses se asocian con una tasa muy elevada
de recidivas.
4. La asociación de pirazinamida durante los dos primeros meses, permite acortar la duración
del tratamiento a 6 meses.
5. La tuberculosis diseminada debe tratarse durante un mínimo de 12 meses.

15. Varón de 30 años de edad, sexualmente activo que presenta disuria y secreción uretral
purulenta. En la tinción de esta última se observan diplococos gramnegativos intracelulares.
¿Cuál cree que sería el tratamiento de elección?:
1. Penicilina benzatina.
2. Doxiciclina.
3. Azitromicina.
4. Ceftriaxona.
5. Ampicilina.

16. ¿Cuál de los siguientes tratamientos antibióticos no sería suficiente en monoterapia en


el manejo del paciente oncológico en tratamiento quimioterápico con neutropenia febril?:
1. Cefepima.
2. Ceftazidima.
3. Meropenem.
4. Imipenem.
5. Piperacilina-tazobactam.

17. Todos los siguientes virus tienen como célula diana la que se indica en cada caso, salvo
uno. Indíquela:
1. El virus de Epstein Barr los linfocitos B.

56
2. Los rinovirus las células epiteliales (receptor ICAM-1, proteína de adherencia de la superfamilia
de las inmunoglobulinas).
3. El virus de la rabia las neuronas (receptor de acetilcolina).
4. Los reovirus las células epiteliales (receptor sialil oligosacáridos).
5. El virus de la inmunodeficiencia humana los linfocitos T facilitadores (moléculas CD4).

18. Una de las siguientes afirmaciones referidas al lipopolisacárido (LPS) de la membrana externa de la pared celular de las bacterias
gram negativas es cierta:
1. Es una toxina termolábil que no resiste la esterilización en autoclave.
2. Tiene una actividad endotóxica que está asociada con el lípido A.
3. Contiene el antígeno O que es esencial para la viabilidad celular.
4. Su concentración en sangre no está directamente relacionada con la mortalidad por
shock irreversible y colapso cardiovascular.
5. No proporciona resistencia a la fagocitosis.

19. Algunas especies bacterianas son capaces de penetrar a través de los epitelios mucosos para así invadir y diseminarse por los
tejidos. Esta fagocitosis realizada por células epiteliales (no fagocíticas) está inducida por la bacteria mediante sustancias denomiadas
"internalinas" y se denomina al proceso "endocitosis dirigida por el parásito". De todas las especies que se citan a continuación sólo
una NO penetra mediante este proceso:
1. Chlamydia trachomatis.
2. Listeria monocytogenes.
3. Bordetella pertussis.
4. Haemophilus infuenzae.
5. Neisseria gonorroheae.

20. La diferencia entre Entamoeba histolytica y Entamoeba dispar (patógena y comensal respectivamente) se hace en los laboratorios
de microbiología clínica por:
1. Estudio de las diferencias morfológicas (microscópica) con ayuda de una tinción permanente.
2. Estudio de las diferencias metabólicas (Galeria metabólica).
3. Estudio de sus diferencias antigénicas mediante pruebas inmunológicas.
4. Estudio de las características diferenciales de la movilidad (pseudópodos).
5. Tan sólo puede establecerse mediante secuenciación del tRNA.

21. ¿Cuál de las siguientes entidades NO se acompaña de eosinofilia?:


1. Asma intrínseca.
2. Infectaciones por helmintos.
3. Enfermedad de Hodgkin.
4. Fiebre tifoidea.
5. Mastocitosis.

22. Respecto a las filariosis, señale la respuesta INCORRECTA:


1. Se transmiten por invasión directa de larvas parasitarias a la piel desde tierras húmedas
al andar descalzo.
2. Onchocerca volvulus produce nódulos subcutáneos, prurito y afectación ocular (queratitis,
retinitis).
3. Loa Loa produce edemas transitorios subcutáneos y conjuntivitis.
4. Wuchereria bancrofti produce varicocele y faringitis.
5. Ivermectina es el tratamiento de elección para la oncocercosis.

23. Joven de 23 años, que trabaja como pinchadiscos en un bar nocturno y tiene una historia de promiscuidad bisexual e ingesta de
drogas sintéticas, consulta por fiebre, malestar general y dolor en glande. La exploración física muestra múltiples tatuajes, "piercings"
la presencia de tres lesiones vesiculosas en glande y adenopatías inguinales bilaterales.¿Cuál es el diagnóstico más probable?:
1) Herpes genital.
2) Secundarismo luético.
3) Primoinfección VIH.
4) Infección por citomegalovirus.
5) Condilomas acuminados.

24. En cuanto a la neumonía por Neumococo en paciente infectados por el virus de la inmunodeficiencia humana (VIH) es FALSO que
1) Es una infección frecuentemente bacteriémica.
2) Tiene mayor incidencia que en la población general.
3) Junto a Haemophilus influenzae es la causa más común de neumonía en pacientes con SIDA.

57
4) Se recomienda vacuna neumocócica en aquellos pacientes con CD4 < 100 cel/ L.
5) Esta neumonía puede ser vista en pacientes con sistema inmune relativamente intacto.

25. La tuberculosis asociada a la infección por VIH se caracteriza por:


1) Presentación subclínica de la enfermedad.
2) Aparición característica en los estadios de inmunodepresión más severa (>50 CD4/mm3).
3) Elevada frecuencia de afectación extrapulmonar y diseminada.
4) Escaso rendimiento de los métodos microbiológicos de diagnóstico.
5) Mala respuesta al tratamiento antituberculoso.

26. NO es propio del shock tóxico estafilocócico:


1) Fiebre elevada.
2) Lesiones cutáneas.
3) Metástasis sépticas.
4) Fracaso renal.
5) Rabdomiólisis.

27. Hombre de 60 años, con antecedentes de neumoconiosis, que acude a nuestra consulta por tos y fiebre de 48 h de duración. A la
exploración se detectan estertores en la base izquierda, que se corresponde en la placa de tórax, con un nuevo infiltrado en lóbulo infer
izquierdo; también se observan lesiones de neumoconiosis simple. La pulsioximetría digital es normal y en los análisis: 14.000
leucocitos/ml. Se realiza un PPD, que es positivo (15 mm), y en 3 baciloscopias de esputo no se observan bacilos ácido-alcohol resistent
El paciente se trata con amoxicilina-clavulánico y desaparece la fiebre y mejora la tos a las 24h. ¿Cuál es la actitud?:
1) Mantener dicho tratamiento 1 semana.
2) Mantener dicho tratamiento 2 semanas.
3) Administrar tratamiento tuberculostáticos convencional.
4) Mantener el tratamiento 2 semanas y administrar isoniacida durante 18 meses, si cultivo de Löwestein en esputo es positivo.
5) Mantener el tratamiento 2 semanas y administrar isoniacida durante 12 meses, si cultivo de Löwestein en esputo es negativo.

28. Una mujer de 54 años sufrió durante una transfusión de sangre un cuadro de fiebre y escalofríos que evolucionó rápidamente a
hipotensión refractaria, falleciendo unas horas después. A las 24 horas se recibió un hemocultivo de la paciente con crecimiento de
Pseudomonas fluorescens. Señale la respuesta FALSA:
1) Las especies de Pseudomonas no crecen a temperaturas bajas y son por tanto excepcionales contaminantes bacterianos de productos
hemáticos.
2) El síndrome de respuesta inflamatoria sistémico puede deberse a causas no infecciosas.
3) Se denomina shock séptico refractario al que dura más de una hora y no ha respondido a fluidos y vasopresores.
4) Los gérmenes Gram negativos son la causa más frecuente de septicemia grave.
5) En el síndrome de distres respiratorio agudo la presión capilar pulmonar es menor de 18 mm.

29. ¿Cuál de las siguientes permite diferenciar la pielonefritis aguda de la cistitis aguda?:
1) Leucocituria.
2) Hematuria.
3) Bacteriuria.
4) Antecedentes de infección urinaria.
5) Fiebre de más de 38.5ºC.

30. Un paciente de 22 años, que acude para evaluación de una lesión genital ulcerada, presenta un VDRL positivo a título de 1:8 con u
FTA-Abs negativo. La interpretación más adecuada de estos resultados es:
1) Falso positivo de las pruebas no treponémicas.
2) Falso negativo de las pruebas treponémicas.
3) Sífilis curada (pendiente la positivización del FTA-Abs).
4) Sífilis curada (pendiente la negativización del VDRL).
5) Sífilis de larga evolución.

31. Señale cuál de las siguientes asociaciones de helmintos y su clínica característica es INCORRECTA:
1) Ascaris lumbricoides - Síndrome de Löeffler.
2) Onchocerca volvulus - Ceguera de los ríos.
3) Ancylostoma duodenale - Anemia megaloblástica.
4) Strongyloides stercolaris - Síndrome de hiperinfestación en inmunodeprimidos.
5) Taenia solium - Convulsiones generalizadas.

32. Paciente de 30 años, seropositivo VIH conocido desde 5 años antes, con antecedentes de neumonía por P. Carinii, que consulta por

58
cefalea desde 10 días antes. La exploración física muestra como datos más relevantes mínima rigidez de nuca y temperatura de 37,5ºC
fondo de ojo normal, TAC: ligera atrofia cortical. La punción lumbar da salida a líquido claro con 40 células mononucleares, proteína
90 mgrs%, glucosa: 30 mg% (glucemia: 90 mg%). Señalar, de entre las siguientes, la causa más probable:
1) Herpesvirus tipo 8.
2) Listeria.
3) Criptococo.
4) CMV.
5) VIH.

33. Paciente de 52 años, natural de Egipto, con 5 años de residencia en España, que ingresa en Urgencias por hematemesis. A la
exploración destaca esplenomegalia importante, la analítica hepática es normal, y en la ecografía aparece fibrosis periportal. ¿En qué
parasitosis pensaría?:
1) Clonorchis sinensis.
2) Schistosoma mansoni.
3) Fasciola hepatica.
4) Echinococcus.
5) Entamoeba histolytica.

34. Señale la cierta en relación a Haemophilus influenzae:


1) Haemophilus influenzae del tipo B es causa frecuente de otitis media en el adulto.
2) La mayoría de las cepas que producen infección bronquial son no capsuladas.
3) Más del 70% de cepas son resistentes a amplicilina por producción de beta-lactamasas.
4) La incidencia de neumonía por Haemophilus influenzae en adultos ha disminuido de forma drástica con el uso de la vacuna conjugada.
5) La práctica totalidad de cepas son sensibles al Cotrimoxazol.

35. En relación con Streptococcus pyogenes y la faringoamigdalitis, ¿cuál de las siguientes afirmaciones NO es correcta?:
1) En tratamiento de la faringoamigdalitis estreptocócica se efectúa con una sola inyección i.m. de 1,200,000 UU de penicilina benzatina.
2) El tratamiento antibiótico de la faringoamigdalitis estreptocócica se efectúa con 250,000 UU/6h oral de penicilina V durante 10 días.
3) El tratamiento antibiótico de la faringoamigdalitis estreptocócica se efectúa con una sola inyección i.m. de 1,200,000 UU de penicilin
procaína.
4) El tratamiento antibiótico de la faringoamigdalitis estreptocócica se efectúa con amoxicilina oral 500 mg/8h durante 10 días.
5) El tratamiento antibiótico recomendado de la faringoamigdalitis estreptocócica en los casos de alergia a la penicilina es un macrólido oral
durante 10 días.

36. A lo largo de los últimos 10 años se ha producido un cambio muy importante en la etiología de la Endocarditis Infecciosa del adult
El microorganismo más frecuente en la actualidad, es:
1) Microorganismos del grupo HACEK.
2) Staphylococcus aureus.
3) Estafilococos coagulasa negativos.
4) Bacilos gram negativos.
5) Streptococcus viridans.

37. Una paciente de 44 años, sometida a vasectomía izquierda con vaciamiento ganglionar, consulta un año después de la intervención
un cuadro febril con celulitis en brazo izquierdo. ¿Cuál es el agente etiológico más probable?:
1) Salmonella enteriditis.
2) Streptococcus agalactiae.
3) Escherichia coli.
4) Streptococcus pyogenes.
5) Pasteurella multocida.

38. El eritema migratorio es una lesión cutáneaanular característica de infección por:


1) Leptospira)
2) Salmonella typhi.
3) Rickettsia conorii.
4) Borrelia burgdorfferi.
5) Coxiella burnetti.

39. En lo referente a Clostridium difficile, señale cuál de las siguientes afirmaciones es INCORRECTA:
1) Se trata de un Bacilo Gram positivo esporulado.
2) Es responsable de algunas diarreas asociadas al uso de antimicrobianos.
3) Solo las cepas toxigénicas con patógenas.
4) Es responsable de la inmensa mayoría de las colitis pseudomembranosas.

59
5) Sólo causa enfermedad en pacientes previamente inmunodeprimidos.

40. ¿Cuál de las siguientes afirmaciones es FALSA, en relación a Listeria?:


1) Es un patógeno intracelular.
2) No está presente en la flora gastrointestinal normal en personas sanas.
3) Causa enfermedad, con más frecuencia , en personas con trastorno de la inmunidad celular.
4) La contaminación de alimentos es relativamente común.
5) La listeriosis asociada a la gestación puede causar muerte fetal intraútero.

41. Si un niño toma el contenido de un jarabe antitusígeno con sabor a limón, que sus padres guardaban en un armario, cuyo principio
activo es fosfato de codeína, es probable que los próximos días el niño muestre:
1) Retención urinaria.
2) Visión borrosa.
3) Candidiasis orofaríngea.
4) Estreñímiento.
5) Cloración rojiza de la orina.

42. En un niño de 8 años previamente sano, la causa más frecuente de meningitis bacteriana es:
1) H. influenzae.
2) S. pneumoniae.
3) N. meningitides.
4) S. aureus.
5) Estreptococos del grupo B.

43. En la meningitis meningocócica, es FALSO que:


1) Los brotes epidémicos suelen estar causados por los serotipos A y C.
2) Las cepas de meningococo serotipo Y se asocian con neumonía.
3) El déficit de los últimos componentes del complemento, de C5 a C8 y de properdina son factores predisponentes.
4) Los pacientes con un sistema de complemento normal tienen menor mortalidad.
5) La mortalidad de la meningitis aislada es menor que la de la sepsis meningocócica sin meningitis.

44. Un estudiante de derecho de 20 años, previamente sano, presenta un cuadro de febrícula, artromialgias, tos seca persistente y asten
de dos semanas de evolución. En el último mes, sus dos hermanos de 9 y 17 años han presentado consecutivamente un cuadros similar,
que se ha autolimitado de forma progresiva. Tras practicársele una radiografía de tórax, el médico le ha diagnosticado de neumonía
atípica. ¿Cuál es el agente etiológico más probable en este caso?:
1) Coxiella burnetti (fiebre Q).
2) Virus sincitial respiratorio.
3) Haemophilus influenzae.
4) Mycoplasma pneumoniae.
5) Legionella pneumophila.

3. ¿Cuál de los siguientes hechos es preciso para el diagnóstico definitivo de una infección por
Aspergillus?:

1. Elevación al cuádruple de los títulos de anticuerpos séricos en un segundo control.


2. Aislamiento del microorganismo en cultivos de esputo.
3. Demostración de invasión tisular.
4. Demostración de material genético en muestras tisulares mediante reacción en cadena de la polimerasa.
5. Visualización del hongo mediante la tinción de tinta china.

4. Anciano de 80 años de edad que bruscamente presenta por el cuerpo y extremidades grandes
ampollas sobre una base urticarial. Algunas son purpúricas. No hay afectación de las mucosas. Se
conserva el estado general. El prurito es discreto y las erosiones postampollosas cicatrizan dejando
máculas pigmentadas. Histopatológicamente se ven ampollas subepidérmicas con abundantes
eosinófilos. Por inmunofluorescencia directa se observa un depósito lineal de IgG y C3 a nivel de la
membrana basal. ¿Cuál es el diagnóstico?:

1. Pénfigo vulgar.
2. Pénfigo cicatricial.
60
3. Penfigoide ampolloso.
4. Dermatitis herpetiforme.
5. Dermatosis ampollosa IgA lineal.

5. ¿Cuál de los siguientes antimicrobianos utilizaría en primer lugar en monoterapia, para tratar una
infección grave por Staphylococcus Aureus resistente a meticilina?:

1. Eritromicina.
2. Rifampicina.
3. Vancomicina.
4. Levofloxacino.
5. Linezolide.

7. El agente etiológico de la fiebre por arañazo de gato es:

1. Rickettsia Conori.
2. Coxiella Burnetti.
3. Bartonella Henselae.
4. Pasteurella Multocida.
5. Aeromona Caviae.

9. Señale cuál de las siguientes respuestas es la verdadera en relación a la apoptosis (muerte celular
programada):

1. La apoptosis es un proceso singular del sistema inmune cuya finalidad es la eliminación de células superfluas
mediante un mecanismo similar a la necrosis celular.
2. La apoptosis puede ser inducida por una gran variedad de estímulos que disparan receptores que pueden
estar tanto en la superficie celular como en el citoplasma.
3. Bc1-2 es una proteína crucial en la cascada de la apoptosis, regulando positivamente el proceso y conduciendo a la
formación del ADN en escalera.
4. La mólecula de superficie fas/APO-1 es una de las principales inductoras de apoptosis y, a diferencia de los
miembros de la familia del receptor del factor de necrosis tumoral ?, su función se circunscribe a regular las células
del sistema inmunológico.
5. Los anticuerpos bloqueantes del factor de necrosis tumoral ?, ya comercializados para uso en humanos, bloquean el
factor de necrosis tumoral ? soluble pero no interfieren en el mecanismo de apoptosis porque no se unen al factor de
necrosis tumoral ? presente en la membrana celular.

61
3. Uno de los siguientes microorganismos NO tiene un hábitat natural en la boca ni en la nasofaringe.
Señálelo:

1. Estreptococo del grupo "Viridans".


2. Haemophilus Influenzae.
3. Moraxella Catarrhalis.
4. Bacteroides.
5. Escherichia Coli.

4. Un hombre de 45 años con leucemia mieloblástica aguda recibió su tercer ciclo de quimioterapia.
A los 7 días desarrolló fiebre y un súbito empeoramiento del estado general. Presentaba una lesión
nodular equimótica y dolorosa, con centro ulcerado y rodeada de eritema y edema, en la pierna
izquierda. Tenía menos de 100 leucocitos/mm3, hemoglobina 7 gr/dL y 30.000 plaquetas/mm3.
¿Qué germen es aislado con mayor frecuencia en un caso como éste?:

1. Aeromonas hydrophila.
2. Klebs
3. ¿Qué germen es aislado con mayor frecuencia en un caso como éste?: 1. Aeromonas hydrophila. 2. Klebsiella
pneumoniae. 3. Staphyloccocus aureus.
4. Pseudomonas aeruginosa.
5. Xantomonas maltophilia.

11. Actualmente la indicación más habitual y fundamentada de la ribavirina es:

1. Hepatitis crónica B.
2. Herpes genital recidivante resistente a aciclovir.
3. Infección por virus sincitial respiratorio en niños pequeños, administrada en aerosol.
4. Infección avanzada por VIH.
5. Encefalitis por citomegalovirus.

15. Indique, entre las siguientes, que prueba diagnóstica de la infección de Legionella Pneumophila,
sería la indicada para realizar en un Servicio de Urgencias:

1. Inmunofluorescencia directa de la muestra.


2. Detección de antígeno de Legionella Pneumophila en orina.
3. Cultivo de muestras respiratorias.
4. Detección de anticuerpos específicos con técnicas serológicas.
5. Cultivo de sangre.

¿En cuál de las siguientes localizaciones de la enfermedad tuberculosa está indicado el tratamiento coadyuvante con
glucocorticoides para mejorar la supervivencia?:

6. Pulmonar.
7. Meníngea.
8. Ganglionar.
9. Genitourinaria.
10. Ostearticular.

62
2. Un hombre de 31 años que tenía contacto directo con perros, consultó por fiebre y cefalea de 4 días de evolución.
Estaba febril y tenía un exantema maculopapular en cara, tronco y extremidades incluyendo palmas y plantas. Existía
una lesión costrosa y negruzca entre las nalgas y adenopatías inguinales. ¿Cuál de los siguientes gérmenes es el
responsable?:

6. Borrelia burgdorferi.
7. Rickettsia typhi.
8. Treponema pallidum.
9. Rickettsia conorii.
10. Bartonella henselae.

3. La llamada meningitis de Mollaret es un cuadro, poco frecuente, consistente en episodios recurrentes de


meningitis en las que suelen verse células sugerentes de esta entidad en el LCR. Hoy se cree que:

6. Está causada por Enterovirus.


7. Está causada por Adenovirus.
8. Se trata de meningitis bacterianas abortadas por tratamiento antibiótico.
9. Se ha vinculado a Virus del Herpes Simple.
10. Es una forma frustrada de enfermedad tuberculosa.

4. Un hombre de 45 años acudió al área de Urgencias de un hospital por fiebre elevada y exantema máculo-papuloso
generalizado, incluyendo palmas y plantas. El paciente vive en el campo con perros frecuentemente parasitados por
garrapatas. Señale la enfermedad a la que se refiere, el germen causante y el tratamiento adecuado:

6. Fiebre botonosa, Ricckettsia Conori: Doxiciclina.


7. Kala-azar, Leishmaniae Donovani: Antimoniales.
8. Dengue, Arenaviridae Aegypti: Tratamiento sintomático.
9. Fiebre Q, Coxiella Burnetti, Doxiciclina.
10. Fiebre de Malta, Brucella Mellitensis: Cotrimoxazol.

5. ¿Cuál de las siguientes relaciones anatómicas de la arteria renal izquierda NO es correcta?:

6. Situada por detrás y algo por encima de la vena renal izquierda.


7. Situada por detrás y debajo del cuerpo del páncreas.
8. Situada por detrás y debajo de la vena esplénica.
9. Se origina en la cara lateral izquierda de la aorta abdominal.
10. Situada por detrás de la vena cava inferior.

6. Un paciente de 40 años con antecedentes de adicción a drogas por vía parenteral, con infección por el virus de la
inmunodeficiencia humana (VIH) conocida desde hace 8 años, con un ingreso en institución penitenciaria hace 2
años, en que presentaba un Mantoux de 7 mm, nunca ha recibido ningún tratamiento ni profilaxis. En la actualidad se
encuentra asintomático, su CD4 son de 100 cel/uL y el Mantoux negativo. Sería INCORRECTO pensar que:

6. Debe recibir profilaxis con Isoniacida 300 mg + piridoxina durante 9 meses.


7. Debe recibir profilaxis con Rifabutina y Piracinamida durante 2 meses.
8. Probablemente presente una infección tuberculosa.
9. No debe recibir profilaxis si se encuentra en tratamiento deshabituador con Metadona.
10. Debe recibir profilaxis con Isonicida 900 mg + piridoxina, 2 veces por semana durante 9 meses.

7. Los agentes causales más frecuentes de meningitis viral son:

11. Arbovirus.
12. Herpesvirus.
13. Virus de la coriomeningitis linfocitaria.
14. Virus de la parotiditis epidémica.
15. Enterovirus.

8. ¿Cuál de los siguientes procesos linfoproliferativos corresponde a un linfoma de linfocitos T?:

6. Linfoma folicular.
7. Linfoma de células del manto.
8. Micosis fungoides.
9. Linfoma linfoplasmocitario.
63
10. Plasmocitoma.

9. Mujer de 33 años de edad ecuatoriana, acude a su médico de familia para la lectura de Mantoux, realizado en el
contexto de un estudio de contactos. Una prima suya que vive en su casa y duerme en la misma habitación (junto con
otras 7 personas), ha sido diagnosticada de una tuberculosis pulmonar bacilífera (más de 50 bacilos por campo). Su
médico aprecia una induración de 7 mm en la lectura del PPD. ¿Cuál de las siguientes afirmaciones es la correcta?:

11. Se trata de una infección tuberculosa y hay que iniciar tratamiento quimioprofiláctico de inmediato.
12. Se trata de una Tuberculosis y hay que comenzar con tratamiento antituberculoso.
13. Se trata de una infección tuberculosa y hay que descartar enfermedad tuberculosa, previo a comenzar el tratamiento
quimioprofiláctico.
14. El PPD no es significativo, ya que al tratarse de una persona inmigrante de una zona de alta prevalencia de TBC debería
tener una induración mayor de 10 mm.
15. El PPD no es significativo, ya que al tratarse de una persona inmigrante de una zona de alta prevalencia de TBC debería
tener una induración a mayor de 15 mm.

1. En la neumonía por Pneumococistis carinii en los pacientes con infección por VIH, ¿cuál
de las siguientes respuestas es la verdadera?:

11. El diagnóstico definitivo se realiza por cultivo del esputo en medios específicos.
12. El uso de glucocorticoides está contraindicado.
13. La Pentamidina intravenosa es el tratamiento alternativo de elección en las formas graves.
14. El riesgo de padecerla es independiente de la cifra de linfocitos CD4+.
15. Nunca está indicada la profilaxis primaria.

• 66 respuestas: 51.5% correctas/ 48.5% incorrectas => Pregunta regular

• Hay 1 comentarios

2. Los agentes causales más frecuentes de meningitis viral son:

1. Arbovirus.
2. Herpesvirus.
3. Virus de la coriomeningitis linfocitaria.
4. Virus de la parotiditis epidémica.
5. Enterovirus.

• 77 respuestas: 53.2% correctas/ 46.8% incorrectas => Pregunta regular

• Hay 5 comentarios

3. Señale, entre las siguientes, la conducta más adecuada ante un paciente leucopénico
que comienza con fiebre:

11. Observarle para vigilar si aparecen síntomas respiratorios.


12. Tomar los cultivos adecuados y comenzar tratamiento antibiótico empírico cuanto antes.
13. Tomar los cultivos adecuados y esperar a los resultados antes de iniciar ningún tratamiento.
14. Tratarle exclusivamente con factor estimulante de colonias de granulocitos.
15. Tratarle con paracetamol, sin mayor preocupación, pues no es probable que padezca una infección.

• 83 respuestas: 83.1% correctas/ 16.9% incorrectas => Pregunta fácil

• Hay 5 comentarios

64
4. Todas las enfermedades siguientes, EXCEPTO una, se han relacionado con un agente
infeccioso. Señálela:

6. Angiodisplasia de colon.
7. Sarcoma de Kaposi del inmunodeprimido.
8. Linfoma MALT gástrico.
9. Úlcera péptica.
10. Linfoma asociado a inmunodepresión.

• 76 respuestas: 63.2% correctas/ 36.8% incorrectas => Pregunta regular

• Hay 4 comentarios

5. Un joven de 16 años realiza un viaje de fin de curso por Europa. Al mes de regreso
comienza con malestar general, odinofagia y fiebre; en la exploración destaca hipertrofia
amigdalar con exudado blanquecino, adenopatías occipitales, laterocervicales dolorosas;
en el hemograma se observa leucocitosis de 15000/mm3 con un 70% de linfocitos, alguno
de ellos atípico. Ante la sospecha diagnóstica se debe realizar:

11. Biopsia ganglionar.


12. Biopsia de médula ósea.
13. Tratamiento con Penicilina.
14. Serología para virus de Epstein Barr.
15. Tratamiento con clindamicina.

• 74 respuestas: 83.8% correctas/ 16.2% incorrectas => Pregunta fácil

• Hay 4 comentarios

6. Una de las siguientes afirmaciones referidas al lipopolisacárido (LPS) de la membrana


externa de la pared celular de las bacterias gram negativas es cierta:

1. Es una toxina termolábil que no resiste la esterilización en autoclave.


2. Tiene una actividad endotóxica que está asociada con el lípido A.
3. Contiene el antígeno O que es esencial para la viabilidad celular.
4. Su concentración en sangre no está directamente relacionada con la mortalidad por shock irreversible y
colapso cardiovascular.
5. No proporciona resistencia a la fagocitosis.

• 71 respuestas: 49.3% correctas/ 50.7% incorrectas => Pregunta regular

• Hay 1 comentarios

7. Un paciente de 40 años con antecedentes de adicción a drogas por vía parenteral, con
infección por el virus de la inmunodeficiencia humana (VIH) conocida desde hace 8 años,
con un ingreso en institución penitenciaria hace 2 años, en que presentaba un Mantoux de
7 mm, nunca ha recibido ningún tratamiento ni profilaxis. En la actualidad se encuentra
asintomático, su CD4 son de 100 cel/uL y el Mantoux negativo. Sería INCORRECTO pensar
que:

11. Debe recibir profilaxis con Isoniacida 300 mg + piridoxina durante 9 meses.
65
12. Debe recibir profilaxis con Rifabutina y Piracinamida durante 2 meses.
13. Probablemente presente una infección tuberculosa.
14. No debe recibir profilaxis si se encuentra en tratamiento deshabituador con Metadona.
15. Debe recibir profilaxis con Isonicida 900 mg + piridoxina, 2 veces por semana durante 9 meses.

• 87 respuestas: 47.1% correctas/ 52.9% incorrectas => Pregunta regular

• Hay 2 comentarios

8. Es conocido el riesgo potencial de transmisión de VIH o hepatitis desde un paciente


portador al equipo médico-quirúrgico que le atiende. La política recomendada
actualmente para evitar estos contagios consiste en aplicar una serie de medidas
especiales para evitar el contacto con sangre o líquidos orgánicos del paciente. Para ello
se debe:

11. Llevar a cabo las pruebas serológicas sistemáticamente a todos los pacientes y tomar medidas en los que
den algún resultado positivo.
12. Solicitar consentimiento previo y llevar a cabo pruebas serológicas a los pacientes que lo hayan
otorgado, tomando medidas en los portadores.
13. Averiguar el nivel de riesgo individual mediante historia clínica y aplicar las medidas a los
pacientes con elevada probabilidad de ser portadores.
14. Tomar precauciones especiales sólo en aquellos pacientes en que, por otro motivo, haya sido
diagnosticados previamente de portadores de alguno de los virus mencionados.
15. Considerar potenciales portadores a todos los pacientes y tomar medidas en todos ellos.

• 85 respuestas: 68.2% correctas/ 31.8% incorrectas => Pregunta fácil

• Hay 4 comentarios

9. ¿Qué es el dengue?:

11. Una enfermedad causada por un poxvirus.


12. Una enfermedad limitada a los países del centro de Africa.
13. Una enfermedad vírica que puede producir una fiebre hemorrágica.
14. Una zoonosis que afecta al hombre ocasionalmente.
15. Una enfermedad vírica que ocasiona un eritema que evoluciona a mácula y pápula afectando
fundamentalmente a la población infantil.

• 69 respuestas: 82.6% correctas/ 17.4% incorrectas => Pregunta fácil

• Hay 2 comentarios

10. Un paciente de 40 años diagnosticado de infección por VIH (virus de la


inmunodeficiencia humana) hace 10 años que no sigue tratamiento antirretroviral
presenta síntomas compatibles con candidiasis esofágica y además refiere un cuadro de
10 días de evolución de cefalea, fiebre, vómitos y en las últimas 24 horas disminución del
nivel de conciencia; la exploración física muestra confusión y rigidez de nuca, el TAC de
cráneo es normal y en la punción lumbar existe una presión de apertura elevada, no se
ven células y las proteinas son del 300 mg/ dl. El cuadro es compatible con:

1. Hipertensión intracraneal benigna.


66
2. Hidrocefalia.
3. Miningitis tuberculosa.
4. Meningitis criptocócica.
5. Toxoplasmosis cerebral.

• 61 respuestas: 37.7% correctas/ 62.3% incorrectas => Pregunta regular

1. Acerca de la fiebre de origen desconocido, ¿cuál de las siguientes afirmaciones es


FALSA?:

11. Clásicamente se define como fiebre mayor de 38,3ºC, en varias ocasiones, durante más de tres semanas,
sin encontrarse un diagnóstico tras una semana de investigaciones en el hospital.
12. Hoy en día se prefiere clasificarla como clásica, en neutropénico, nosocomial y asociada a infección por
VIH.
13. Es fundamental hacer una historia de los viajes realizados.
14. En pacientes neutropénicos (<50 neutrófilos/ mL), es útil realizar un tratamiento antibiótico
empírico.
15. En pacientes que la presentan durante más de seis meses, la causa más frecuente es infecciosa.

• 54 respuestas: 64.8% correctas/ 35.2% incorrectas => Pregunta regular

• Hay 2 comentarios

2. Un paciente ingresa en urgencias con disminución del nivel de conciencia, fiebre de


39ºC y TA de 70/40 mmHg. ¿Cuál, de las siguientes, considera la actitud inicial más
correcta?:

11. Iniciar tratamiento antibiótico empírico de amplio espectro.


12. Realizar una punción lumbar.
13. Bajar la fiebre.
14. Tomar muestras para hemocultivos.
15. Aportar líquidos intravenosos.

• 66 respuestas: 63.6% correctas/ 36.4% incorrectas => Pregunta regular

• Hay 6 comentarios

3. Paciente trasplantado renal de 2 meses de evolución que acude al servicio de urgencias


por síndrome febril de 3 días de evolución bien tolerado y acompañado de epigastralgias.
En la analítica practicada destaca una moderada leucopenia (2400/mm3) con una leve
elevación en la cifra de transaminasas (ALT 75 UI/l; AST 89 Ul/l. ¿Cuál sería el primer
diagnóstico de sospecha?:

11. Tuberculosis pulmonar.


12. Infección por Helicobacter pilorii.
13. Infección por Pneumocistis carinii.
14. Infección por Citomegalovirus.
15. Hepatitis por VHC.

67
• 73 respuestas: 71.2% correctas/ 28.8% incorrectas => Pregunta fácil

• Hay 3 comentarios

4. El tratamiento antirretroviral en un paciente VIH+ con carga viral de 575.000/mm3 y


una cifra de linfocitos de CD4 de 450/mm3:

11. Es obligado.
12. Sólo estaría indicado si la carga viral fuera superior a 1 millón de copias/ml.
13. En ningún caso se prescribiría si los CD4+ son >200/ml.
14.
15. 000/mm3 y una cifra de linfocitos de CD4 de 450/mm3: 1. Es obligado. 2. Sólo estaría indicado si la
carga viral fuera superior a 1 millón de copias/ml. 3. En ningún caso se prescribiría si los CD4+ son
>200/ml. 4. Está indicado si el paciente lo desea. 5. En estas condiciones sólo estaría indicado en el
contexto de un estudio clínico prospectivo.

• 68 respuestas: 54.4% correctas/ 45.6% incorrectas => Pregunta regular

• Hay 3 comentarios

5. Una paciente de 45 años presenta de forma progresiva en los últimos 3 días un cuadro
de cefalea, deterioro del nivel de conciencia y fiebre de 39,5ºC. entre sus antecedentes
destaca una enfermedad de Crohn que ha requerido tratamiento de forma irregular en los
últimos 3 años. Desde hace 6 meses está tomando mesalazina y 15 mg de prednisona/ día.
La exploración clínica no presenta datos significativos, salvo discretos signos de irritación
meníngea. El hemograma muestra 15.600 leucocitos, con una fórmula normal. La
bioquímica elemental es normal. La Rx de tórax no presenta alteraciones significativas. Se
realiza una punción lumbar, obteniéndose un LCR con proteínas de 560 mg/ dl, glucosa 25
mg/dl y 325 células/ml (85% células mononucleares). La tinción de Gram y de Ziehl-
Neelsen en LCR son negativas. ¿Cuál es la actitud más correcta?:

11. Sospechar una meningoencefalitis autoinmune y subir la dosis de prednisona a 1 mg/ kg de peso,
añadiendo quimioprofilaxis con isoniacida.
12. Sospechar una meningitis por gérmenes de origen gastrointestinal. Iniciar tratamiento antibiótico
empírico que cubra gérmenes Gram (-) y anaerobios.
13. Sospechar una meningitis tuberculosa. Iniciar tratamiento inmediato con tres tuberculostáticos.
14.
15. 600 leucocitos, con una fórmula normal. La bioquímica elemental es normal. La Rx de tórax no presenta
alteraciones significativas. Se realiza una punción lumbar, obteniéndose un LCR con proteínas de 560
mg/ dl, glucosa 25 mg/dl y 325 células/ml (85% células mononucleares). La tinción de Gram y de Ziehl-
Neelsen en LCR son negativas. ¿Cuál es la actitud más correcta?: 1. Sospechar una meningoencefalitis
autoinmune y subir la dosis de prednisona a 1 mg/ kg de peso, añadiendo quimioprofilaxis con

• 67 respuestas: 34.3% correctas/ 65.7% incorrectas => Pregunta regular

• Hay 1 comentarios

68
6. Un trabajador sanitario tiene una prueba de la tuberculina que mide 0 mm. Al repetirla
10 días después, el diámetro de la induración mide 12 mm. ¿Cuál de las siguientes
interpretaciones es más adecuada para estos resultados?:

11. Primoinfección tuberculosa entre las dos pruebas.


12. Infección tuberculosa latente.
13. Presencia de anergia.
14. Tuberculosis activa hace años.
15. Mala realización de alguna de las pruebas.

• 81 respuestas: 27.2% correctas/ 72.8% incorrectas => Pregunta difícil

• Hay 6 comentarios

7. ¿Cuál de las siguientes pruebas es la más adecuada para evaluar la respuesta


terapéutica en un paciente tratado de sífilis precoz?:

11. Realización de fondo oscuro sobre las lesiones que vayan apareciendo.
12. Examen anual de líquido cefalorraquídeo (VDRL).
13. Test de inmovilización del Treponema pallidum (TPI).
14. Cualquiera de las pruebas treponémicas (TPI, FTA, etc).
15. Evaluación seriada del título del VDRL o RPR (pruebas no treponémicas).

• 74 respuestas: 62.2% correctas/ 37.8% incorrectas => Pregunta regular

• Hay 3 comentarios

8. ¿Cuál de los siguientes hechos es preciso para el diagnóstico definitivo de una infección
por Aspergillus?:

1. Elevación al cuádruple de los títulos de anticuerpos séricos en un segundo control.


2. Aislamiento del microorganismo en cultivos de esputo.
3. Demostración de invasión tisular.
4. Demostración de material genético en muestras tisulares mediante reacción en cadena de la polimerasa.
5. Visualización del hongo mediante la tinción de tinta china.

• 62 respuestas: 46.8% correctas/ 53.2% incorrectas => Pregunta regular

• Hay 3 comentarios

9. ¿Cuál de los siguientes tratamientos antibióticos no sería suficiente en monoterapia en


el manejo del paciente oncológico en tratamiento quimioterápico con neutropenia febril?:

1. Cefepima.
2. Ceftazidima.
3. Meropenem.
4. Imipenem.
5. Piperacilina-tazobactam.

• 62 respuestas: 43.5% correctas/ 56.5% incorrectas => Pregunta regular

69
• Hay 3 comentarios

10. ¿Cuál de las siguientes afirmaciones, en cuanto a la Neumonía por Pneumocistis Carinii
en pacientes VIH (Virus de la inmunodeficiencia humana), positivos, es FALSA?:

11. Se presentan clínicamente de forma subaguda.


12. Con elevada frecuencia se prescribe tratamiento sin confirmación diagnóstica bacteriológica.
13. El 95% de los pacientes tiene un recuento de CD4<200cel/uL.
14. El diagnóstico se obtiene habitualmente mediante cultivo de secreciones bronquiales obtenidas
por la inducción del esputo.
15. La profilaxis primaria se puede suspender en pacientes con tratamiento antirretroviral que presenta una
carga viral suprimida (<500 cop/ml) y CD4>200cel/uL.

• 69 respuestas: 46.4% correctas/ 53.6% incorrectas => Pregunta regular

• Hay 1 comentarios

11. En la interpretación de los resultados de los hemocultivos practicados a un paciente


con fiebre, ¿cuál de los siguientes datos nos haría pensar que no estamos ante un caso de
contaminación?:

11. Aislamiento de bacterias que normalmente colonizan la piel.


12. Aislamiento de cocos gram positivos.
13. Aislamiento de bacterias difteroides.
14. Aislamiento del mismo microorganismo en hemocultivos con la misma sensibilidad.
15. Aislamiento de un estafilococo meticilin resistente en un solo hemocultivo.

• 69 respuestas: 68.1% correctas/ 31.9% incorrectas => Pregunta fácil

• Hay 2 comentarios

12. ¿Cuál, entre los siguientes, es un virus persistente que puede permanecer en latencia y
reactivarse?:

11. El virus de la hepatitis A.


12. El virus respiratorio sincitial.
13. El citomegalovirus.
14. El virus de la poliomielitis.
15. El rotavirus.

• 64 respuestas: 65.6% correctas/ 34.4% incorrectas => Pregunta regular

• Hay 0 comentarios

13. Señale, entre las siguientes, la conducta más adecuada ante un paciente leucopénico
que comienza con fiebre:

11. Observarle para vigilar si aparecen síntomas respiratorios.


12. Tomar los cultivos adecuados y comenzar tratamiento antibiótico empírico cuanto antes.
13. Tomar los cultivos adecuados y esperar a los resultados antes de iniciar ningún tratamiento.

70
14. Tratarle exclusivamente con factor estimulante de colonias de granulocitos.
15. Tratarle con paracetamol, sin mayor preocupación, pues no es probable que padezca una infección.

• 84 respuestas: 83.3% correctas/ 16.7% incorrectas => Pregunta fácil

• Hay 6 comentarios

14. Una joven de 18 años acude al hospital por fiebre y cefalea de varias horas de
evolución. Los días previos había notado dolor de garganta y tos. Se observó tendencia al
sueño, rigidez de nuca y petequias en conjuntivitis y extremidades. El LCR era turbio y
contenía 36.000 leucocitos /mm3, 200 mg/dL de proteínas y 20 mg/dL de glucosa
(glucemia simultánea 120 mg/dL). El examen con Gram fue negativo. Señale cuál de las
siguientes afirmaciones NO es correcta:

11. El diagnóstico más probable es meningitis meningocócica.


12. Los meningococos son siempre sensibles a la penicilina y, por tanto, la penicilina G es el
tratamiento de elección.
13. Como un significativo porcentaje de meningococos del grupo C son resistentes a la penicilina, la
cefotaxima es un tratamiento más seguro.
14. El empleo de dexametasona reduce el riesgo de secuelas neurosensoriales en niños con meningitis
bacteriana, pero su uso en un caso como éste es cuestionable.
15. Se debe administrar rifampicina a las personas que conviven estrechamente con la paciente.

• 72 respuestas: 52.8% correctas/ 47.2% incorrectas => Pregunta regular

• Hay 4 comentarios

15. Un paciente infectado por el VIH recibe tratamiento antirretroviral con AZT + 3TC +
Efavirenz, desde hace 14 meses. La última determinación de linfocitos CD4 y carga viral
era de 350/mm3 y <200 copias/ml respectivamente. Consulta por disnea y palidez y se
constata una anemia microcítica (Hemoglobina: 7,8 gr/dl, VCH 68 fl). El diagnóstico más
probable es:

1. Crisis aplástica por Parvovirus B.


2. Leishmaniasis diseminada.
3. Infección diseminada por Mycobacterium avium-intracellulare.
4. Toxicidad farmacológica.
5. Hemorragia digestiva crónica.

• 54 respuestas: 25.9% correctas/ 74.1% incorrectas => Pregunta difícil

• Hay 0 comentarios

16. : Señale cuál de las siguientes asociaciones de helmintos y su clínica característica es


INCORRECTA:

1. Ascaris Lumbricoides – Síndrome de Löeffler.


2. Onchocerca Volvulus – Ceguera de los ríos.
3. Anchylostoma Duodenale – Anemia megaloblástica.
4. Strongyloides Stercolaris – Síndrome de hiperinfestación en inmunodeprimidos.
5. Taenia Solium – Convulsiones generalizadas.
71
• 76 respuestas: 32.9% correctas/ 67.1% incorrectas => Pregunta difícil

• Hay 1 comentarios

17. Un hombre de 45 años con leucemia mieloblástica aguda recibió su tercer ciclo de
quimioterapia. A los 7 días desarrolló fiebre y un súbito empeoramiento del estado
general. Presentaba una lesión nodular equimótica y dolorosa, con centro ulcerado y
rodeada de eritema y edema, en la pierna izquierda. Tenía menos de 100 leucocitos/mm3,
hemoglobina 7 gr/dL y 30.000 plaquetas/mm3. ¿Qué germen es aislado con mayor
frecuencia en un caso como éste?:

11. Aeromonas hydrophila.


12. Klebs
13. ¿Qué germen es aislado con mayor frecuencia en un caso como éste?: 1. Aeromonas hydrophila. 2.
Klebsiella pneumoniae. 3. Staphyloccocus aureus.
14. Pseudomonas aeruginosa.
15. Xantomonas maltophilia.

• 69 respuestas: 46.4% correctas/ 53.6% incorrectas => Pregunta regular

• Hay 4 comentarios

18. ¿En qué situación clínica el uso de la reacción en cadena de la polimerasa (PCR) se ha
convertido en la técnica de referencia para realizar su diagnóstico?:

11. Diagnóstico de SIDA.


12. Tuberculosis pulmonar no bacilífera.
13. Síndrome mononucleósido por CMV.
14. Encefalitis por virus Herpes Simplex.
15. Meningitis meningocócica.

• 76 respuestas: 53.9% correctas/ 46.1% incorrectas => Pregunta regular

• Hay 7 comentarios

19. Una mujer de 17 años ha sido diagnosticada recientemente de lupus eritematoso


sistémico grave y puesta en tratamiento con esteroides a dosis elevadas. La radiografía de
tórax es normal y el Mantoux de 12x15 mm de induración. ¿Qué actitud tomaría?:

1. Efectuar quimioprofilaxis con isoniacida durante 6 meses.


2. Esperar 2 años para realizar quimioprofilaxis.
3. No utilizar quimioprofilaxis.
4. Tratar con isoniacida más rifampicina.
5. Tratar con isoniacida más rifampicina más etambutol.

• 81 respuestas: 56.8% correctas/ 43.2% incorrectas => Pregunta regular

• Hay 4 comentarios

72
20. Una paciente de 43 años, con una prótesis mitral implantada 3 semanas antes, acude a
Urgencias por fiebre de 5 días de evolución. Dos días más tarde el laboratorio de
Microbiología informa del crecimiento en 5 de los 6 frascos de hemocultivo de cocos Gram
positivos en racimo. ¿Cuál, de los siguientes, es el tratamiento de elección hasta conocer
los resultados definitivos?:

11. Vancomicina + gentamicina + rifampicina.


12. Cloxacilina + gentamicina + rifampicina.
13. Penicilina + gentamicina + rifampicina.
14. Ceftriaxona.
15. Imipenem.

• 65 respuestas: 56.9% correctas/ 43.1% incorrectas => Pregunta regular

• Hay 3 comentarios

21. El término diferenciación aplicado al tejido neoplásico define:

11. El grado de similitud de las células neoplásicas desde el punto de vista morfológico y funcional con
las células normales de las que derivan.
12. La aparición dentro de un tumor de elementos neoplásicos que no están presentes en el órgano o en el
tejido en el cual se originan.
13. La presencia de un grado extremo de anaplasia.
14. La presencia de áreas tumorales de morfología diferente en cambios adyacentes del tumor (por ejemplo
adenocarcinoma y carcinoma epidermoide).
15. La variación de la morfología tumoral en la recurrencia de la enfermedad.

• 63 respuestas: 68.3% correctas/ 31.7% incorrectas => Pregunta fácil

• Hay 2 comentarios

22. Un paciente de 40 años con antecedentes de adicción a drogas por vía parenteral, con
infección por el virus de la inmunodeficiencia humana (VIH) conocida desde hace 8 años,
con un ingreso en institución penitenciaria hace 2 años, en que presentaba un Mantoux de
7 mm, nunca ha recibido ningún tratamiento ni profilaxis. En la actualidad se encuentra
asintomático, su CD4 son de 100 cel/uL y el Mantoux negativo. Sería INCORRECTO pensar
que:

1. Debe recibir profilaxis con Isoniacida 300 mg + piridoxina durante 9 meses.


2. Debe recibir profilaxis con Rifabutina y Piracinamida durante 2 meses.
3. Probablemente presente una infección tuberculosa.
4. No debe recibir profilaxis si se encuentra en tratamiento deshabituador con Metadona.
5. Debe recibir profilaxis con Isonicida 900 mg + piridoxina, 2 veces por semana durante 9 meses.

• 88 respuestas: 46.6% correctas/ 53.4% incorrectas => Pregunta regular

• Hay 2 comentarios

23. ¿Qué es el dengue?:

1. Una enfermedad causada por un poxvirus.


73
2. Una enfermedad limitada a los países del centro de Africa.
3. Una enfermedad vírica que puede producir una fiebre hemorrágica.
4. Una zoonosis que afecta al hombre ocasionalmente.
5. Una enfermedad vírica que ocasiona un eritema que evoluciona a mácula y pápula afectando
fundamentalmente a la población infantil.

• 70 respuestas: 82.9% correctas/ 17.1% incorrectas => Pregunta fácil

• Hay 2 comentarios

24. Los agentes causales más frecuentes de meningitis viral son:

11. Arbovirus.
12. Herpesvirus.
13. Virus de la coriomeningitis linfocitaria.
14. Virus de la parotiditis epidémica.
15. Enterovirus.

• 78 respuestas: 53.8% correctas/ 46.2% incorrectas => Pregunta regular

• Hay 5 comentarios

25. Un paciente de 35 años de edad acude al servicio de urgencias por presentar fiebre
elevada de hasta 39ºC, junto con confusión mental. En la exploración física destaca la
existencia de lesiones cutáneas en pie izquierdo a nivel distal, maculares, de milímetros
de diámetro, de aspecto isquémico hemorrágico y la auscultación cardiopulmonar es
normal. A los pocos día se obtiene crecimiento de Staphylococo aureus meticilin sensible
en tres hemocultivos de tres obtenidos. ¿Cuál de las siguientes sería la actitud correcta a
seguir en ese momento?:

1. Considerar el resultado de los hemocultivos como probable contaminación.


2. Pautar de inmediato tratamiento antibiótico con penicilina y gentamicina durante 10 días.
3. Comenzar tratamiento con cloxacilina y gentamicina y realizar estudio ecocardiográfico por la
existencia probable de endocarditis aguda.
4. Descartar la existencia de endocarditis por la ausencia de soplos en la auscultación cardiaca y buscar
focos de posible osteomielitis.
5. Realizar TAC abdominal urgente por probable absceso abdominal.

• 66 respuestas: 65.2% correctas/ 34.8% incorrectas => Pregunta regular

• Hay 4 comentarios

26. ¿Cuál de las siguientes permite diferenciar la pielonefritis. aguda de la cistitis aguda?

1. Leucocituria.
2. Hematuria.
3. Bacteriuria.
4. Antecedentes de infección urinaria.
5. Fiebre de más de 38,5ºC.

74
• 65 respuestas: 36.9% correctas/ 63.1% incorrectas => Pregunta regular

• Hay 8 comentarios

27. Respecto a las filariosis, señale la respuesta INCORRECTA:

1. Se transmiten por invasión directa de larvas parasitarias a la piel desde tierras húmedas al andar
descalzo.
2. Onchocerca volvulus produce nódulos subcutáneos, prurito y afectación ocular (queratitis, retinitis).
3. Loa Loa produce edemas transitorios subcutáneos y conjuntivitis.
4. Wuchereria bancrofti produce varicocele y faringitis.
5. Ivermectina es el tratamiento de elección para la oncocercosis.

• 49 respuestas: 26.5% correctas/ 73.5% incorrectas => Pregunta difícil

• Hay 0 comentarios

28. En la neumonía por Pneumocistis carinii en los pacientes con infección por VIH ¿cuál
de las siguientes respuestas es la verdadera?:

1. El diagnóstico definitivo se realiza por cultivo del esputo en medios específicos.


2. El uso de glucocorticoides está contraindicado.
3. La Pentamidina intravenosa es el tratamiento alternativo de elección en las formas graves.
4. El riesgo de padecerla es independiente de la cifra de linfocitos CD4+.
5. Nunca está indicada la profilaxis primaria.

• 94 respuestas: 60.6% correctas/ 39.4% incorrectas => Pregunta regular

• Hay 3 comentarios

29. Un hombre de 45 años acudió al área de Urgencias de un hospital por fiebre elevada y
exantema máculo-papuloso generalizado, incluyendo palmas y plantas. El paciente vive en
el campo con perros frecuentemente parasitados por garrapatas. Señale la enfermedad a
la que se refiere, el germen causante y el tratamiento adecuado:

1. Fiebre botonosa, Ricckettsia Conori: Doxiciclina.


2. Kala-azar, Leishmaniae Donovani: Antimoniales.
3. Dengue, Arenaviridae Aegypti: Tratamiento sintomático.
4. Fiebre Q, Coxiella Burnetti, Doxiciclina.
5. Fiebre de Malta, Brucella Mellitensis: Cotrimoxazol.

• 72 respuestas: 51.4% correctas/ 48.6% incorrectas => Pregunta regular

• Hay 2 comentarios

30. ¿Cuál es el patógeno involucrado más frecuentemente en la “diarrea del viajero”?:

11. Salmonella.
12. Giardia Lamblia.
13. Campylobacter.

75
14. Entamoeba Hystolítica.
15. Escherichia Coli.

• 83 respuestas: 73.5% correctas/ 26.5% incorrectas => Pregunta fácil

• Hay 4 comentarios

31. ¿En cuál de las siguientes situaciones es más probable que podamos encontrar una
reacción negativa a la tuberculina?:

11. Niño de 2 años, inmunocompetente, al que se vacunó con BCG al nacer.


12. Mujer de 25 años que consulta por esterilidad y a quien se diagnostica tuberculosis genital.
13. Hombre de 45 años con lesiones pulmonares cavitarias y baciloscopia y cultivo positivos para M.
tuberculosis.
14. Hombre de 70 años, tuberculoso antiguo, muy grave, con un patrón miliar en Rx de tórax y con
hemocultivo positivo para M. tuberculosis.
15. Hombre con hematuria, fiebre, orquiepididimitis derecha, dolores cólicos ureterales y cultivo positivo
para M. tuberculosis en orina.

• 62 respuestas: 51.6% correctas/ 48.4% incorrectas => Pregunta regular

• Hay 6 comentarios

32. Señale la cierta, en relación a la encefalitis herpética:

1. Está producida casi siempre por el virus del herpes simple tipo
2. 2. La clínica característica es la presencia de fiebre con síntomas focales del lóbulo occipital.
3. La PCR en el líquido cefalorraquídeo no es útil para establecer el diagnóstico.
4. El tratamiento de elección es el zanamivir i.v.
5. Las secuelas neurológicas son frecuentes.

• 57 respuestas: 43.9% correctas/ 56.1% incorrectas => Pregunta regular

• Hay 3 comentarios

33. Un paciente de 40 años diagnosticado de infección por VIH (virus de la


inmunodeficiencia humana) hace 10 años que no sigue tratamiento antirretroviral
presenta síntomas compatibles con candidiasis esofágica y además refiere un cuadro de
10 días de evolución de cefalea, fiebre, vómitos y en las últimas 24 horas disminución del
nivel de conciencia; la exploración física muestra confusión y rigidez de nuca, el TAC de
cráneo es normal y en la punción lumbar existe una presión de apertura elevada, no se
ven células y las proteinas son del 300 mg/ dl. El cuadro es compatible con:

1. Hipertensión intracraneal benigna.


2. Hidrocefalia.
3. Miningitis tuberculosa.
4. Meningitis criptocócica.
5. Toxoplasmosis cerebral.

• 64 respuestas: 37.5% correctas/ 62.5% incorrectas => Pregunta regular


76
• Hay 5 comentarios

34. Un hombre de 45 años acudió al área de Urgencias de un hospital por fiebre elevada y
exantema máculo-papuloso generalizado, incluyendo palmas y plantas. El paciente vive en
el campo con perros frecuentemente parasitados por garrapatas. Señale la enfermedad a
la que se refiere, el germen causante y el tratamiento adecuado:

1. Fiebre botonosa, Ricckettsia Conori: Doxiciclina.


2. Kala-azar, Leishmaniae Donovani: Antimoniales.
3. Dengue, Arenaviridae Aegypti: Tratamiento sintomáticico.
4. Fiebre Q, Coxiella Burnetti, Doxiciclina.
5. Fiebre de malta, Brucella mellitensis: Cotrimoxazol.

• 77 respuestas: 71.4% correctas/ 28.6% incorrectas => Pregunta fácil

• Hay 4 comentarios

35. El tratamiento antirretroviral en un paciente VIH+ con carga viral de 575.000/mm3 y


una cifra de linfocitos de CD4 de 450/mm3:

1. Es obligado.
2. Sólo estaría indicado si la carga viral fuera superior a 1 millón de copias/ml.
3. En ningún caso se prescribiría si los CD4+ son >200/ml.
4.
5. 000/mm3 y una cifra de linfocitos de CD4 de 450/mm3: 1. Es obligado. 2. Sólo estaría indicado si la
carga viral fuera superior a 1 millón de copias/ml. 3. En ningún caso se prescribiría si los CD4+ son
>200/ml. 4. Está indicado si el paciente lo desea. 5. En estas condiciones sólo estaría indicado en el
contexto de un estudio clínico prospectivo.

• 69 respuestas: 43.5% correctas/ 56.5% incorrectas => Pregunta regular

• Hay 3 comentarios

36. La tuberculosis asociada a la infección por VIH se caracteriza por:

1. Presentación subclínica de la enfermedad.


2. Aparición característica en los estadios de inmunodepresión más severa (>50 CD4/mm3).
3. Elevada frecuencia de afectación extrapulmonar y diseminada.
4. Escaso rendimiento de los métodos microbiológicos de diagnóstico.
5. Mala respuesta al tratamiento antituberculoso.

• 73 respuestas: 74.0% correctas/ 26.0% incorrectas => Pregunta fácil

• Hay 2 comentarios

37. ¿En cuál de las siguientes localizaciones de la enfermedad tuberculosa está indicado el
tratamiento coadyuvante con glucocorticoides para mejorar la supervivencia?:

11. Pulmonar.
12. Meníngea.

77
13. Ganglionar.
14. Genitourinaria.
15. Ostearticular.

• 69 respuestas: 81.2% correctas/ 18.8% incorrectas => Pregunta fácil

• Hay 3 comentarios

38. Señale cuál de las siguientes afirmaciones es FALSA acerca de la vía piramidal:

11. En el mesencéfalo y protuberancia, la vía piramidal desciende por el mismo lado en el que se ha
originado.
12. En el extremo inferior del bulbo se decusan únicamente las fibras que inervan la musculatura del
tronco.
13. El origen de la vía se sitúa en el gyrus precentral.
14. El fascículo piramidal cruzado desciende por el cordón lateral de la médula.
15. La corteza cerebral donde se origina el fascículo piramidal está irrigada por las arterias cerebrales
anterior y media.

• 72 respuestas: 45.8% correctas/ 54.2% incorrectas => Pregunta regular

• Hay 1 comentarios

39. Adulto de 41 años que desde hace 1 año tiene brotes de lesiones habonosas
diseminadas por todo el cuerpo, de color rojo, que duran días y se acompañan de
artralgias. Histológicamente hay un infiltrado neutrofílico perivascular con
leucocitoclasia, en dermis superior. Este cuadro corresponde a:

6. Una urticaria crónica idiopática.


7. Un edema angioneurótico.
8. Una urticaria física.
9. Una urticaria-vasculitis.
10. Una púrpura papulosa.

• 62 respuestas: 56.5% correctas/ 43.5% incorrectas => Pregunta regular

• Hay 4 comentarios

40. Señale lo correcto en relación con la coriorretinitis por citomegalovirus:

6. Ocurre en cualquier estudio de la infección.


7. Siempre cursa con viremia.
8. Siempre afecta a la cámara anterior del ojo.
9. Es progresiva y puede causar ceguera.
10. No precisa tratamiento de por vida.

• 55 respuestas: 78.2% correctas/ 21.8% incorrectas => Pregunta fácil

• Hay 3 comentarios

78
41. Un paciente consulta por diarrea sanguinolenta y fiebre de 39º C de más de una
semana de duración. En el coprocultivo se aísla Campylobacter Jejuni. ¿Cuál de los
siguientes antibióticos es el recomendado como de primera elección?:

6. Amoxicilina.
7. Ciprofloxacino.
8. Cefotaxima.
9. Gentamicina.
10. Eritromicina.

• 68 respuestas: 44.1% correctas/ 55.9% incorrectas => Pregunta regular

• Hay 4 comentarios

42. Indique la verdadera entre las siguientes cuestiones referidas al rechazo de injertos:

6. La base celular de la aloreactividad es el reconocimiento por los macrófagos del receptor de las
moléculas de histocompatibilidad del donante que se convierten en esta situación particular en
antígenos.
7. La enfermedad injerto contra huésped está asociada fundamentalmente con el trasplante renal y es una
de las principales causas del fracaso del mismo.
8. En la mayor parte de los trasplantes, si la selección del donante ha sido adecuada, no es necesario
el uso de fármacos inmunosupresores que complicarían la supervivencia del trasplante.
9. En el rechazo agudo, debido a la activación de las células CD4+ activadas, las diferencias entre las
moléculas de clase II inducen una respuesta alogénica más fuerte que la inducida por diferencias
en las de clase I.
10. El rechazo crónico es la pérdida de injertos a partir de tres meses. Su intensidad es más débil que en el
agudo y responde habitualmente a los inmunosupresores.

• 79 respuestas: 31.6% correctas/ 68.4% incorrectas => Pregunta difícil

• Hay 1 comentarios

43. Un paciente de 22 años, que acude para evaluación de una lesión genital ulcerada,
presenta un VDRL positivo a título de 1:8 con un FTA-Abs negativo. La interpretación más
adecuada de estos resultados es:

1. Falso positivo de las pruebas no treponémicas


2. Falso negativo de las pruebas treponémicas.
3. Sífilis muy reciente (pendiente la positivización del FTA – Abs).
4. Sífilis curada (pendiente la negativización del VDRL).
5. Sífilis de larga evolución.

• 68 respuestas: 22.1% correctas/ 77.9% incorrectas => Pregunta difícil

• Hay 0 comentarios

79
44. Enfermo de 55 años diagnosticado de gammapatía monoclonal presenta lesiones
papulosas rosadas y traslúcidas en regiones periorbitarias y peribucales junto con
macroglosia. El diagnóstico más probable es:

6. Linfoma de células B.
7. Mucinosis papulosa.
8. Amiloidosis AL.
9. Calcinosis cutáneo-mucosa.
10. Lupus eritematoso.

• 69 respuestas: 53.6% correctas/ 46.4% incorrectas => Pregunta regular

• Hay 2 comentarios

45. Indique cuál de las siguientes respuestas en relación a la inmunidad frente a los
tumores es la correcta:

6. Los tumores desencadenan habitualmente una fuerte respuesta inmunogénica, pero ésta no es suficiente
para controlar su crecimiento.
7. Las células tumorales segregan citocinas que tienen una acción estimuladora sobre el sistema inmune
creando un sistema de retroalimentación positiva de su crecimiento.
8. Una causa habitual de propagación de un tumor es la demora inmune, que es la diferencia entre
la cinética del crecimiento tumoral y la formación de una respuesta inmune adaptativa eficaz.
9. Las células tumorales segregan citocinas que potencian la acción de los linfocitos CD4+ Th1
responsables de la respuesta inflamatoria; este aumento de la respuesta inflamatoria crea una red
alrededor del tumor que le protege en su expansión.
10. La presencia de moléculas coestimuladoras, como CD80, en la superficie del tumor provoca una
activación anormal de los linfocitos T que evita una respuesta citolítica eficaz.

• 77 respuestas: 41.6% correctas/ 58.4% incorrectas => Pregunta regular

• Hay 1 comentarios

46. En la neumonía por Pneumococistis carinii en los pacientes con infección por VIH,
¿cuál de las siguientes respuestas es la verdadera?:

1. El diagnóstico definitivo se realiza por cultivo del esputo en medios específicos.


2. El uso de glucocorticoides está contraindicado.
3. La Pentamidina intravenosa es el tratamiento alternativo de elección en las formas graves.
4. El riesgo de padecerla es independiente de la cifra de linfocitos CD4+.
5. Nunca está indicada la profilaxis primaria.

• 67 respuestas: 50.7% correctas/ 49.3% incorrectas => Pregunta regular

• Hay 1 comentarios

47. Una de las siguientes aseveraciones NO es correcta en la tuberculosis miliar. Señálela:

11. Se debe a la diseminación hematógena del bacilo.


12. La tinción de Ziehl del esputo suele ser negativa.
13. La prueba de la tuberculina es positiva en el 80% de los casos.

80
14. La biopsia de médula ósea puede proporcionar el diagnóstico.
15. Puede presentarse como manifestación de una infección tuberculosa primaria.

• 56 respuestas: 62.5% correctas/ 37.5% incorrectas => Pregunta regular

• Hay 4 comentarios

48. Todos los siguientes virus tienen como célula diana la que se indica en cada caso, salvo
uno. Indíquela:

1. El virus de Epstein Barr los linfocitos B.


2. Los rinovirus las células epiteliales (receptor ICAM-1, proteína de adherencia de la superfamilia de las
inmunoglobulinas).
3. El virus de la rabia las neuronas (receptor de acetilcolina).
4. Los reovirus las células epiteliales (receptor sialil oligosacáridos).
5. El virus de la inmunodeficiencia humana los linfocitos T facilitadores (moléculas CD4).

• 64 respuestas: 70.3% correctas/ 29.7% incorrectas => Pregunta fácil

• Hay 0 comentarios

49. ¿Qué es el dengue?:

1. Una enfermedad causada por un proxvirus.


2. Una enfermedad limitada a los países del centro de África.
3. Una enfermedad vírica que puede producir una fiebre hemorrágica.
4. Una zoonosis que afecta al hombre ocasionalmente.
5. Una enfermedad vírica que ocasiona un eritema que evoluciona a mácula y pápula afectando
fundamentalmente a la población infantil.

• 80 respuestas: 80.0% correctas/ 20.0% incorrectas => Pregunta fácil

• Hay 12 comentarios

50. Ante un joven de 16 años que ha estado conviviendo con un enfermo con tuberculosis
pulmonar activa y tiene una prueba de la tuberculina negativa, la actitud más correcta, de
las siguientes, es realizar:

12. Quimioprofilaxis durante 6 meses.


13. Vigilancia clínica y prueba de tuberculina a los 3 meses.
14. Quimioprofilaxis durante 2-3 meses y luego repetir la prueba de la tuberculina.
15. Tratamiento con rifampicina, isoniacida y piracinamida durante 6 meses.
16. Tratamiento con rifampicina, isoniacida y piracinamida y, a los 2 meses, repetir la prueba de la
tuberculina.

• 67 respuestas: 44.8% correctas/ 55.2% incorrectas => Pregunta regular

Hay 1 comentarios
. El germen causal más frecuente de artritis séptica entre pacientes entre 15 a 40 años es:

81
Staphylococcus aureus.
Haemophilus influenzae.
Neisseria gonorrhoeae.
Streptococcus pyogenes.
Streptococcus pneumoniae.
50 respuestas: 42.0% correctas/ 58.0% incorrectas => Pregunta regular
Hay 1 comentarios

2. Un enfermo con neumonía, perteneciente a un brote epidémico de varias personas que ocasionalmente conviven en un

edificio, presenta un cuadro confusional desproporcionado a la fiebre, diarrea, hiponatremia y ligero ascenso de las enzimas

hepáticas. Entre las siguientes opciones, ¿cuál se debe incluir en el tratamiento?:

Cefalosporinas y aminoglucósidos a las dosis adecuadas.


Vancomicina a las dosis adecuadas.
Doxiciclina 100 mg/12 h durante 14 días.
Eritromicina 2-4 g/día durante 14 días.
Penicilina G procaína 0,6 – 1,2x106 U/12 h durante 7 días.
61 respuestas: 34.4% correctas/ 65.6% incorrectas => Pregunta regular
Hay 4 comentarios

3. Adulto de 41 años que desde hace 1 año tiene brotes de lesiones habonosas diseminadas por todo el cuerpo, de color rojo,

que duran días y se acompañan de artralgias. Histológicamente hay un infiltrado neutrofílico perivascular con leucocitoclasia,

en dermis superior. Este cuadro corresponde a:

Una urticaria crónica idiopática.


Un edema angioneurótico.
Una urticaria física.
Una urticaria-vasculitis.
Una púrpura papulosa.
61 respuestas: 57.4% correctas/ 42.6% incorrectas => Pregunta regular
Hay 3 comentarios

4. Un joven de 16 años realiza un viaje de fin de curso por Europa. Al mes de regreso comienza con malestar general,

odinofagia y fiebre; en la exploración destaca hipertrofia amigdalar con exudado blanquecino, adenopatías occipitales,

laterocervicales dolorosas, en el hemograma se observa leucocitos, algunos de ellos atípicos. Ante la sospecha diagnóstica se

debe realizar:

Biopsia ganglionar.

82
Biopsia de médula ósea.
Tratamiento con Penicilina.
Serología para virus de Epstein Barr.
Tratamiento con Clindamicina.
68 respuestas: 76.5% correctas/ 23.5% incorrectas => Pregunta fácil
Hay 7 comentarios

5. Un paciente con 57 años y diabetes mellitus mal controlada, comienza con fiebre, dolor profundo en seno maxilar,

congestión y secreción nasal serosanguinolenta. Se instaura tratamiento antibiótico sin objetivar mejoría. En la evolución de la

enfermedad aparece ptosis parpebral y deterioro del nivel de conciencia. En la TC se aprecia opacificación de senos maxilares

y frontales. Se extrae muestra del seno y en el laboratorio de microbiología informan de la presencia de hifas no tabicadas.

¿Cuál es el diagnóstico más probable?:

Aspergillosis.
Mucormicosis.
Candidiasis invasora.
Actinomicosis.
Rinosporidiosis.
61 respuestas: 60.7% correctas/ 39.3% incorrectas => Pregunta regular
Hay 3 comentarios

6. ¿Qué es el dengue?:

Una enfermedad causada por un poxvirus.


Una enfermedad limitada a los países del centro de Africa.
Una enfermedad vírica que puede producir una fiebre hemorrágica.
Una zoonosis que afecta al hombre ocasionalmente.
Una enfermedad vírica que ocasiona un eritema que evoluciona a mácula y pápula afectando fundamentalmente a la población
infantil.
68 respuestas: 82.4% correctas/ 17.6% incorrectas => Pregunta fácil
Hay 2 comentarios

7. Un enfermero de Urgencias le consulta porque ha tenido un accidente en el que ha recibido un pinchazo profundo, sin

guantes, con una aguja gruesa visiblemente manchada de sangre, de un paciente adicto a drogas por vía parenteral. Tras

interrogar al paciente, declara que comparte habitualmente jeringuillas intravenosas y que nunca se ha realizado una

serología para el VIH. ¿Cuál de las siguientes es la actitud más correcta?:

Esperar al día siguiente a que esté el resultado de la serología de VIH.

83
Iniciar inmediatamente tratamiento con tres antiretrovirales.
Iniciar inmediatamente tratamiento con AZT.
Realizar serología de VIH, carga viral de VIH y test de resistencias genotípicas (en caso de carga viral detectable) y
revisar, cuando estén los resultados, la necesidad de tratamiento antiretroviral.
Tranquilizar al enfermo debido a bajo riesgo de transmisión del VIH y reevaluar en un mes.
80 respuestas: 50.0% correctas/ 50.0% incorrectas => Pregunta regular
Hay 5 comentarios

8. Mujer de 33 años de edad ecuatoriana, acude a su médico de familia para la lectura de Mantoux, realizado en el contexto de

un estudio de contactos. Una prima suya que vive en su casa y duerme en la misma habitación (junto con otras 7 personas), ha

sido diagnosticada de una tuberculosis pulmonar bacilífera (más de 50 bacilos por campo). Su médico aprecia una induración

de 7 mm en la lectura del PPD. ¿Cuál de las siguientes afirmaciones es la correcta?:

Se trata de una infección tuberculosa y hay que iniciar tratamiento quimioprofiláctico de inmediato.
Se trata de una Tuberculosis y hay que comenzar con tratamiento antituberculoso.
Se trata de una infección tuberculosa y hay que descartar enfermedad tuberculosa, previo a comenzar el tratamiento
quimioprofiláctico.
El PPD no es significativo, ya que al tratarse de una persona inmigrante de una zona de alta prevalencia de TBC debería tener una
induración mayor de 10 mm.
El PPD no es significativo, ya que al tratarse de una persona inmigrante de una zona de alta prevalencia de TBC debería
tener una induración a mayor de 15 mm.
65 respuestas: 33.8% correctas/ 66.2% incorrectas => Pregunta regular
Hay 3 comentarios

9. Un paciente de 40 años con antecedentes de adicción a drogas por vía parenteral, con infección por el virus de la

inmunodeficiencia humana (VIH) conocida desde hace 8 años, con un ingreso en institución penitenciaria hace 2 años, en que

presentaba un Mantoux de 7 mm, nunca ha recibido ningún tratamiento ni profilaxis. En la actualidad se encuentra

asintomático, su CD4 son de 100 cel/uL y el Mantoux negativo. Sería INCORRECTO pensar que:

Debe recibir profilaxis con Isoniacida 300 mg + piridoxina durante 9 meses.


Debe recibir profilaxis con Rifabutina y Piracinamida durante 2 meses.
Probablemente presente una infección tuberculosa.
No debe recibir profilaxis si se encuentra en tratamiento deshabituador con Metadona.
Debe recibir profilaxis con Isonicida 900 mg + piridoxina, 2 veces por semana durante 9 meses.
86 respuestas: 46.5% correctas/ 53.5% incorrectas => Pregunta regular
Hay 2 comentarios

10. ¿Cuál de las siguientes afirmaciones relativas a la fiebre tifoidea NO es correcta?:

84
La enfermedad se suele contraer por la ingesta de alimentos, agua o leche contaminados.
La leucopenia es más frecuente que la leucocitosis en las personas con enfermedad aguda.
La roseola se suele presentar en el momento en el que comienza la fiebre.
El cloranfenicol no es eficaz para prevenir las recaídas.
Las fluoroquinolonas erradican el germen, incluso en presencia de cálculos biliares.
64 respuestas: 34.4% correctas/ 65.6% incorrectas => Pregunta regular
Hay 4 comentarios

11. Paciente de 64 años, fumador, que acude a urgencias por un cuadro de 48 h. de evolución de fiebre y tos con expectoración

mucopurulenta. La radiografía de tórax muestra una condensación alveolar en lóbulo inferior derecho y un pequeño

infiltrado en el lóbulo inferior izquierdo. La gasometría arterial muestra un pH de 7,39, una pO2 de 54 mmHg y una pCO2 de

29 mmHg. ¿Cuál de las siguientes opciones terapéuticas le parece más adecuada?:

Claritromicina 500 mg IV/12h.


Iprofloxacina 200 mg IV/12h.
Amoxicilina-Ácido clavulánico 1g IV/8h.
Ciprofloxacino 200 mg EV/12h + Claritromicina 500 mg IV/12h.
Ceftriaxona 2g IV/24h + Claritromicina 500 mg IV/12h.
65 respuestas: 55.4% correctas/ 44.6% incorrectas => Pregunta regular
Hay 3 comentarios

12. ¿Cuál de las siguientes permite diferenciar la pielonefritis. aguda de la cistitis aguda?

Leucocituria.
Hematuria.
Bacteriuria.
Antecedentes de infección urinaria.
Fiebre de más de 38,5ºC.
64 respuestas: 37.5% correctas/ 62.5% incorrectas => Pregunta regular
Hay 8 comentarios

13. La malaria es una enfermedad parasitaria erradicada en nuestro país, pero en los últimos años estamos asitiendo a un

resurgir de casos debidos a la inmigración y a los viajes a países tropicales. En relación a la malaria, cuál de las siguientes

afirmaciones es FALSA:

La gravedad de la infección está en relación con el grado de parasitemia.


La infección por plasmodium falciparum es la más grave.
La malaria cerebral es una manifestación tipica de plasmodium vivax.
El dato analítico más frecuente es una anemia normocítica.
85
La infección se ha descrito en adictos a drogas por vía parenteral que comparten jeringuillas.
66 respuestas: 37.9% correctas/ 62.1% incorrectas => Pregunta regular
Hay 3 comentarios

14. Un paciente, fumador, de 60 años, acude a Urgencias con un cuadro de fiebre, tos y expectoración amarillenta de 36 horas

de evolución. Se le objetiva una condensación con broncograma aéreo en la base pulmonar derecha en la Rx de tórax. ¿Cuál

de las siguientes pruebas complementarias es de mayor utilidad para decidir su hospitalización?:

Realización de tinción de Gram en una muestra de esputo.


TC torácica.
Hemocultivos seriados.
Gasometría arterial basal.
Aspirado transtraqueal.
77 respuestas: 75.3% correctas/ 24.7% incorrectas => Pregunta fácil
Hay 5 comentarios

15. Paciente trasplantado renal de 2 meses de evolución que acude al servicio de urgencias por síndrome febril de 3 días de

evolución bien tolerado y acompañado de epigastralgias. En la analítica practicada destaca una moderada leucopenia

(2400/mm3) con una leve elevación en la cifra de transaminasas (ALT 75 UI/l; AST 89 Ul/l. ¿Cuál sería el primer diagnóstico

de sospecha?:

Tuberculosis pulmonar.
Infección por Helicobacter pilorii.
Infección por Pneumocistis carinii.
Infección por Citomegalovirus.
Hepatitis por VHC.
72 respuestas: 72.2% correctas/ 27.8% incorrectas => Pregunta fácil
Hay 3 comentarios

16. ¿Qué es el dengue?:

Una enfermedad causada por un proxvirus.


Una enfermedad limitada a los países del centro de África.
Una enfermedad vírica que puede producir una fiebre hemorrágica.
Una zoonosis que afecta al hombre ocasionalmente.
Una enfermedad vírica que ocasiona un eritema que evoluciona a mácula y pápula afectando fundamentalmente a la población
infantil.
79 respuestas: 79.7% correctas/ 20.3% incorrectas => Pregunta fácil
Hay 11 comentarios
86
17. ¿Cuál es la principal causa de epididimitis en varones heterosexuales activos menores de 35 años?:

Chlamydia trachomatis.
Neisseria gonorrhoeae.
Ureplasma urealyticum.
Escherichia coli.
Pseudomonas aeruginosa.
71 respuestas: 47.9% correctas/ 52.1% incorrectas => Pregunta regular
Hay 0 comentarios

18. Un paciente consulta por diarrea sanguinolenta y fiebre de 39ºC de más de una semana de duración. En el coprocultivo se

aisla Campylobacter Jejuni. ¿Cuál de los siguientes antibióticos es el recomendado como de primera elección?:

Amoxicilina.
Ciprofloxacino.
Cefotaxima.
Gentamicina.
Eritromicina.
58 respuestas: 39.7% correctas/ 60.3% incorrectas => Pregunta regular
Hay 4 comentarios

19. Un paciente de 32 años con infección VIH, ingresa con un cuadro de pancreatitis aguda. No es consumidor de alcohol. La

ecografía abdominal descarta litiasis biliar. Sigue tratamiento antirretroviral con zidovudina (AZT), didanosina (ddI) e

indinavir. ¿Qué se debe suprimir de su tratamiento?:

Didanosina (ddI).
Zidovudina (AZT).
Indinavir.
Los tres fármacos.
Nada.
78 respuestas: 48.7% correctas/ 51.3% incorrectas => Pregunta regular
Hay 3 comentarios

20. Un enfermo de Urgencias le consulta porque ha tenido un accidente en el que ha recibido un pinchazo profundo, sin

guantes, con una aguja gruesa visiblemente manchada de sangre, de un paciente adicto a drogas por vías parenteral. Tras

interrogar al paciente, declara que comporte habitualmente jeringuillas intravenosas y que nunca se ha realizado una

serología para el VIH. ¿Cuál de las siguientes es la actitud más correcta?:


87
Esperar al día siguiente a que esté el resultado de la serología de VIH.
Iniciar inmediatamente tratamiento con tres antiretrovirales.
Iniciar inmediatamente tratamiento con AZT.
Realizar serología de VIH, carga viral de VIH y test de resistencias genotípicas (en caso de carga viral detectable) y revisar,
cuando estén los resultados, la necesidad de tratamiento antiretroviral.
Tranquiliar al enfermo debido al bajo riesgo de transmisión del VIH y reevaluar en un mes.
83 respuestas: 57.8% correctas/ 42.2% incorrectas => Pregunta regular
Hay 4 comentarios

21. Una de las siguientes propiedades referidas a los virus RNA es cierta:

La mayoría de ellos se multiplican en el núcleo.


La estructura del genoma no es determinante del mecanismo de transcripción y replicación.
No es necesario que codifiquen RNA polimerasas RNA dependientes.
No muestran tendencias a las mutaciones.
El genoma RNA (+) de los retrovirus se convierte en DNA, que es integrado en la cromatina del huésped y transcrito
como un gen celular.
64 respuestas: 65.6% correctas/ 34.4% incorrectas => Pregunta regular
Hay 2 comentarios

22. Un niño de 14 años acude a su consulta por presentar herida por mordedura en antebrazo, con exudado purulento que ha

empeorado a pesar del tratamiento con clindamicina. ¿Cuál es la etiología más probable de la infección de la herida?:

Staphylococcus Aureus.
Streptococo beta-hemolítico grupo A.
Eikenella Corrodens.
Capnocytophaga gingivalis.
Mycobacterium Tuberculosis. ANULADA
66 respuestas: 15.2% correctas/ 84.8% incorrectas => Pregunta difícil
Hay 3 comentarios

23. La candidiasis hepatoesplénica ocurre habitualmente en:

Paciente con SIDA.


Diabéticos tipo I con neuropatía avanzada.
Receptores de trasplante hepático en el período postrasplante precoz.
Pacientes con leucemia aguda y leucopenia prolongada en fase de resolución.
Pacientes con cirugía pancreática mayor y con nutrición parenteral.
58 respuestas: 31.0% correctas/ 69.0% incorrectas => Pregunta difícil
Hay 2 comentarios
88
24. El antibiótico de elección en el tratamiento de una neumonía producida por Legionella pneumophila es:

Eritromicina.
Penicilina G.
Cotrimoxazol.
Cefotaxima.
Clindamicina.
66 respuestas: 75.8% correctas/ 24.2% incorrectas => Pregunta fácil
Hay 4 comentarios

25. Un veterinario de 40 años consultó por fiebre de 3 semanas de duración con dolor de espalda en los últimos días. Al final

de la primera semana había tenido inflamación testicular que cedió con trimetoprim-sulfametoxazol. La exploración era

normal salvo por dolor a la percusión en la columna dorsolumbar. Tenía una hemoglobina de 12,2 g/dL, GTP 320 u/l (normal

6-45) y fosfatasa alcalina 230 u/l (normal 41-117). La resonancia nuclear magnética mostraba epifisitis anterior de D12 y

signos de espondilodiscitis D12-L

¿Cuál debe ser la prueba diagnóstica siguiente en este caso?: 1. Biopsia para cultivo de la médula ósea.
Punción aspiración vertebral.
Hemocultivos y aglutinaciones para Brucella.
Biopsia hepática.
Cultivo de orina y de secreción prostática.
70 respuestas: 57.1% correctas/ 42.9% incorrectas => Pregunta regular
Hay 1 comentarios

26. Un joven de 16 años realiza un viaje de fin de curso por Europa. Al mes de regreso comienza con malestar general,

odinofagia y fiebre; en la exploración destaca hipertrofia amigdalar con exudado blanquecino, adenopatías occipitales,

laterocervicales dolorosas; en el hemograma se observa leucocitosis de 15000/mm3 con un 70% de linfocitos, alguno de ellos

atípico. Ante la sospecha diagnóstica se debe realizar:

Biopsia ganglionar.
Biopsia de médula ósea.
Tratamiento con Penicilina.
Serología para virus de Epstein Barr.
Tratamiento con clindamicina.
73 respuestas: 83.6% correctas/ 16.4% incorrectas => Pregunta fácil
Hay 4 comentarios

89
27. ¿Cuál de los siguientes hechos es preciso para el diagnóstico definitivo de una infección por Aspergillus?:

Elevación al cuádruple de los títulos de anticuerpos séricos en un segundo control.


Aislamiento del microorganismo en cultivos de esputo.
Demostración de invasión tisular.
Demostración de material genético en muestras tisulares mediante reacción en cadena de la polimerasa.
Visualización del hongo mediante la tinción de tinta china.
61 respuestas: 47.5% correctas/ 52.5% incorrectas => Pregunta regular
Hay 3 comentarios

28. Una mujer de 17 años ha sido diagnosticada recientemente de lupus eritematoso sistémico grave y puesta en tratamiento

con esteroides a dosis elevadas. La radiografía de tórax es normal y el Mantoux de 12x15 mm de induración. ¿Qué actitud

tomaría?:

Efectuar quimioprofilaxis con isoniacida durante 6 meses.


Esperar 2 años para realizar quimioprofilaxis.
No utilizar quimioprofilaxis.
Tratar con isoniacida más rifampicina.
Tratar con isoniacida más rifampicina más etambutol.
80 respuestas: 57.5% correctas/ 42.5% incorrectas => Pregunta regular
Hay 4 comentarios

29. En relación a la sífilis, señale la afirmación correcta:

La lúes secundaria cursa excepcionalmente con manifestaciones cutáneas.


El control del tratamiento se puede realizar valorando los títulos de positividad de las pruebas treponémicas.
Durante los períodos de latencia de la enfermedad se negativizan las pruebas no treponémicas.
La peculiar estructura del treponema hace que no sea sensible a los antibióticos betalactámicos.
El chancro sifilítico es indurado, no doloroso y muy rico en bacterias.
61 respuestas: 54.1% correctas/ 45.9% incorrectas => Pregunta regular
Hay 1 comentarios

30. Es conocido el riesgo potencial de transmisión de VIH o hepatitis desde un paciente portador al equipo médico-quirúrgico

que le atiende. La política recomendada actualmente para evitar estos contagios consiste en aplicar una serie de medidas

especiales para evitar el contacto con sangre o líquidos orgánicos del paciente. Para ello se debe:

Llevar a cabo las pruebas serológicas sistemáticamente a todos los pacientes y tomar medidas en los que den algún resultado
positivo.
Solicitar consentimiento previo y llevar a cabo pruebas serológicas a los pacientes que lo hayan otorgado, tomando

90
medidas en los portadores.
Averiguar el nivel de riesgo individual mediante historia clínica y aplicar las medidas a los pacientes con elevada probabilidad de
ser portadores.
Tomar precauciones especiales sólo en aquellos pacientes en que, por otro motivo, haya sido diagnosticados previamente de
portadores de alguno de los virus mencionados.
Considerar potenciales portadores a todos los pacientes y tomar medidas en todos ellos.
84 respuestas: 69.0% correctas/ 31.0% incorrectas => Pregunta fácil
Hay 4 comentarios

31. ¿Cuál es el patógeno involucrado más frecuentemente en la “diarrea del viajero”?:

Salmonella.
Giardia Lamblia.
Campylobacter.
Entamoeba Hystolítica.
Escherichia Coli.
82 respuestas: 73.2% correctas/ 26.8% incorrectas => Pregunta fácil
Hay 4 comentarios

32. Un paciente consulta por diarrea sanguinolenta y fiebre de 39º C de más de una semana de duración. En el coprocultivo se

aísla Campylobacter Jejuni. ¿Cuál de los siguientes antibióticos es el recomendado como de primera elección?:

Amoxicilina.
Ciprofloxacino.
Cefotaxima.
Gentamicina.
Eritromicina.
67 respuestas: 44.8% correctas/ 55.2% incorrectas => Pregunta regular
Hay 4 comentarios

33. ¿En cuál de las siguientes localizaciones de la enfermedad tuberculosa está indicado el tratamiento coadyuvante con

glucocorticoides para mejorar la supervivencia?:

Pulmonar.
Meníngea.
Ganglionar.
Genitourinaria.
Ostearticular.
68 respuestas: 82.4% correctas/ 17.6% incorrectas => Pregunta fácil
Hay 3 comentarios
91
34. Señale, entre las siguientes, la conducta más adecuada ante un paciente leucopénico que comienza con fiebre:

Observarle para vigilar si aparecen síntomas respiratorios.


Tomar los cultivos adecuados y comenzar tratamiento antibiótico empírico cuanto antes.
Tomar los cultivos adecuados y esperar a los resultados antes de iniciar ningún tratamiento.
Tratarle exclusivamente con factor estimulante de colonias de granulocitos.
Tratarle con paracetamol, sin mayor preocupación, pues no es probable que padezca una infección.
82 respuestas: 84.1% correctas/ 15.9% incorrectas => Pregunta fácil
Hay 5 comentarios

35. Hombre de 60 años con antecedentes de neumoconiosis, que acude a nuestra consulta por tos y fiebre de 48 h de duración.

A la exploración se detectan estertores en la base izquierda, que se corresponde en la placa de tórax, con un nuevo infiltrado

en lóbulo inferior izquierdo; también se observan lesiones de neumoconiosis simple. La pulsioximetría digital es normal y en

los análisis: 14.000 leucocitos/ml. Se realiza un PPD que es positivo (15 mm) y en 3 baciloscopias de esputo no se observan

bacilos ácido-alcohol resistentes. El paciente se trata con amoxicilina-clavulánico y desaparece la fiebre y mejora la tos a las 24

h. ¿Cuál es la actitud?:

Mantener dicho tratamiento 1 semana.


Mantener dicho tratamiento 2 semanas.

000 leucocitos/ml. Se realiza un PPD que es positivo (15 mm) y en 3 baciloscopias de esputo no se observan bacilos ácido-
alcohol resistentes. El paciente se trata con amoxicilina-clavulánico y desaparece la fiebre y mejora la tos a las 24 h. ¿Cuál es
la actitud?: 1. Mantener dicho tratamiento 1 semana. 2. Mantener dicho tratamiento 2 semanas. 3. Administrar tratamiento
tuberculostáticos convencional. 4. Mantener el tratamiento 2 semanas y administrar isoniacida durante 18 meses, si cultivo
de Löwestein en esputo es positivo.
Mantener el tratamiento 2 semanas y administrar isoniacida durante 12 meses, si cultivo de Löwestein en esputo es
negativo.
67 respuestas: 34.3% correctas/ 65.7% incorrectas => Pregunta regular
Hay 2 comentarios

36. En relación a la sífilis, señale la afirmación correcta:

La lúes secundaria cursa excepcionalmente con manifestaciones cutáneas.


El control del tratamiento se puede realizar valorando los títulos de positividad de las pruebas treponémicas.
Durante los períodos de latencia de la enfermedad se negativizan las pruebas no treponémicas.
La peculiar estructura del treponema hace que no sea sensible a los antibióticos betalactámicos.
El chancro sifilítico es indurado, no doloroso y muy rico en bacterias.
71 respuestas: 60.6% correctas/ 39.4% incorrectas => Pregunta regular
92
Hay 2 comentarios

37. La llamada meningitis de Mollaret es un cuadro, poco frecuente, consistente en episodios recurrentes de meningitis en las

que suelen verse células sugerentes de esta entidad en el LCR. Hoy se cree que:

Está causada por Enterovirus.


Está causada por Adenovirus.
Se trata de meningitis bacterianas abortadas por tratamiento antibiótico.
Se ha vinculado a Virus del Herpes Simple.
Es una forma frustrada de enfermedad tuberculosa.
66 respuestas: 43.9% correctas/ 56.1% incorrectas => Pregunta regular
Hay 4 comentarios

38. Un paciente de 22 años, que acude para evaluación de una lesión genital ulcerada, presenta un VDRL positivo a título de

1:8 con un FTA-Abs negativo. La interpretación más adecuada de estos resultados es:

Falso positivo de las pruebas no treponémicas


Falso negativo de las pruebas treponémicas.
Sífilis muy reciente (pendiente la positivización del FTA – Abs).
Sífilis curada (pendiente la negativización del VDRL).
Sífilis de larga evolución.
67 respuestas: 22.4% correctas/ 77.6% incorrectas => Pregunta difícil
Hay 0 comentarios

39. En la neumonía por Pneumocistis carinii en los pacientes con infección por VIH ¿cuál de las siguientes respuestas es la

verdadera?:

El diagnóstico definitivo se realiza por cultivo del esputo en medios específicos.


El uso de glucocorticoides está contraindicado.
La Pentamidina intravenosa es el tratamiento alternativo de elección en las formas graves.
El riesgo de padecerla es independiente de la cifra de linfocitos CD4+.
Nunca está indicada la profilaxis primaria.
93 respuestas: 61.3% correctas/ 38.7% incorrectas => Pregunta regular
Hay 3 comentarios

40. Paciente de 52 años, natural de Egipto, con 5 años de residencia en España, que ingresa en Urgencias por hematemesis. A

la exploración destaca esplenomegalia importante, la analítica hepática es normal, y en la ecografía aparece fibrosis

periportal. ¿En qué parasitosis pensaría?:


93
Clonorchis Sinensis.
Schistosoma Mansoni.
Fasciola Hepática.
Echinococcus.
Estamoeba Histolytica.
75 respuestas: 42.7% correctas/ 57.3% incorrectas => Pregunta regular
Hay 1 comentarios

41. Todas las enfermedades siguientes, EXCEPTO una, se han relacionado con un agente infeccioso. Señálela:

Angiodisplasia de colon.
Sarcoma de Kaposi del inmunodeprimido.
Linfoma MALT gástrico.
Úlcera péptica.
Linfoma asociado a inmunodepresión.
75 respuestas: 64.0% correctas/ 36.0% incorrectas => Pregunta regular
Hay 4 comentarios

42. Una paciente de 43 años, con una prótesis mitral implantada 3 semanas antes, acude a Urgencias por fiebre de 5 días de

evolución. Dos días más tarde el laboratorio de Microbiología informa del crecimiento en 5 de los 6 frascos de hemocultivo de

cocos Gram positivos en racimo. ¿Cuál, de los siguientes, es el tratamiento de elección hasta conocer los resultados

definitivos?:

Vancomicina + gentamicina + rifampicina.


Cloxacilina + gentamicina + rifampicina.
Penicilina + gentamicina + rifampicina.
Ceftriaxona.
Imipenem.
64 respuestas: 57.8% correctas/ 42.2% incorrectas => Pregunta regular
Hay 3 comentarios

43. Un paciente ingresa en urgencias con disminución del nivel de conciencia, fiebre de 39ºC y TA de 70/40 mmHg. ¿Cuál, de

las siguientes, considera la actitud inicial más correcta?:

Iniciar tratamiento antibiótico empírico de amplio espectro.


Realizar una punción lumbar.
Bajar la fiebre.
Tomar muestras para hemocultivos.
Aportar líquidos intravenosos.

94
65 respuestas: 64.6% correctas/ 35.4% incorrectas => Pregunta regular
Hay 5 comentarios

44. Una paciente de 45 años presenta de forma progresiva en los últimos 3 días un cuadro de cefalea, deterioro del nivel de

conciencia y fiebre de 39,5ºC. entre sus antecedentes destaca una enfermedad de Crohn que ha requerido tratamiento de

forma irregular en los últimos 3 años. Desde hace 6 meses está tomando mesalazina y 15 mg de prednisona/ día. La

exploración clínica no presenta datos significativos, salvo discretos signos de irritación meníngea. El hemograma muestra

15.600 leucocitos, con una fórmula normal. La bioquímica elemental es normal. La Rx de tórax no presenta alteraciones

significativas. Se realiza una punción lumbar, obteniéndose un LCR con proteínas de 560 mg/ dl, glucosa 25 mg/dl y 325

células/ml (85% células mononucleares). La tinción de Gram y de Ziehl-Neelsen en LCR son negativas. ¿Cuál es la actitud

más correcta?:

Sospechar una meningoencefalitis autoinmune y subir la dosis de prednisona a 1 mg/ kg de peso, añadiendo
quimioprofilaxis con isoniacida.
Sospechar una meningitis por gérmenes de origen gastrointestinal. Iniciar tratamiento antibiótico empírico que cubra gérmenes
Gram (-) y anaerobios.
Sospechar una meningitis tuberculosa. Iniciar tratamiento inmediato con tres tuberculostáticos.

600 leucocitos, con una fórmula normal. La bioquímica elemental es normal. La Rx de tórax no presenta alteraciones
significativas. Se realiza una punción lumbar, obteniéndose un LCR con proteínas de 560 mg/ dl, glucosa 25 mg/dl y 325
células/ml (85% células mononucleares). La tinción de Gram y de Ziehl-Neelsen en LCR son negativas. ¿Cuál es la actitud
más correcta?: 1. Sospechar una meningoencefalitis autoinmune y subir la dosis de prednisona a 1 mg/ kg de peso, añadiendo
quimioprofilaxis con
65 respuestas: 35.4% correctas/ 64.6% incorrectas => Pregunta regular
Hay 1 comentarios

45. Si un paciente con acné, en tratamiento con clindamicina por vía oral, presenta un cuadro de colitis con moco, sangre y

pus, además de suspender la clindamicina, ¿qué tratamiento de los siguientes es, en principio, el más indicado?:

Dieta astringente.
Prednisona.
Eritromicina.
Vancomicina.
Loperamida.
73 respuestas: 54.8% correctas/ 45.2% incorrectas => Pregunta regular
Hay 10 comentarios

46. Enfermo de 55 años diagnosticado de gammapatía monoclonal presenta lesiones papulosas rosadas y traslúcidas en
95
regiones periorbitarias y peribucales junto con macroglosia. El diagnóstico más probable es:

Linfoma de células B.
Mucinosis papulosa.
Amiloidosis AL.
Calcinosis cutáneo-mucosa.
Lupus eritematoso.
68 respuestas: 52.9% correctas/ 47.1% incorrectas => Pregunta regular
Hay 2 comentarios

47. Paciente de 20 años con dolor de garganta, fiebre, subictericia y adenopatías cervicales y polo de bazo palpable. En la

analítica presenta leucocitosis con linfocitosis, "células linfoides activadas" y elevación discreta de transaminasa. ¿Cuál es el

diagnóstico más probable?:

Infección por virus de Epstein-Barr.


Hepatitis A aguda.
Infección por citomegalovirus.
Hepatitis B aguda.
Infección por herpes virus.
67 respuestas: 77.6% correctas/ 22.4% incorrectas => Pregunta fácil
Hay 3 comentarios

48. Las manifestaciones clínicas del tétanos se producen por la:

Invasión tisular directa del sistema nervioso central.


Producción de una exotoxina en la herida infectada.
Producción de citoquinas.
Destrucción tisular marcada en la zona infectada con sobreinfección bacteriana polimicrobiana secundaria.
Destrucción bacteriana masiva al iniciar el tratamiento antibiótico.
68 respuestas: 69.1% correctas/ 30.9% incorrectas => Pregunta fácil
Hay 5 comentarios

49. En la neumonía por Pneumococistis carinii en los pacientes con infección por VIH, ¿cuál de las siguientes respuestas es la

verdadera?:

El diagnóstico definitivo se realiza por cultivo del esputo en medios específicos.


El uso de glucocorticoides está contraindicado.
La Pentamidina intravenosa es el tratamiento alternativo de elección en las formas graves.
El riesgo de padecerla es independiente de la cifra de linfocitos CD4+.

96
Nunca está indicada la profilaxis primaria.
65 respuestas: 50.8% correctas/ 49.2% incorrectas => Pregunta regular
Hay 1 comentarios

50. ¿Cuál, de los siguientes, es con más frecuencia el germen causal de espondilodiscitis piógena en pacientes con SIDA,

adictos a drogas por vía parenteral?:

Salmonella.
Haemophilus influenzae.
Pseudomonas aeruginosa.
Streptococcus granulosus.
Escherichia coli.
66 respuestas: 47.0% correctas/ 53.0% incorrectas => Pregunta regular
Hay 2 comentarios

1. Epstein Barr se asocia a:


a. enfermedad de hodgkin
b. retinopatía
c. síndrome diarreico
d. calcificaciones del bazo
e. neumonía
10. La rifampicina:
a. Actúa sobre el DNA
b. Es bacteriostatico
c. No atraviesa la BHE
d. Dosis es de 5 mg/kg de peso
e. Se elimina principalmente por la bilis

11. La isoniacida
a. Actúa sobre el acido micolico de la membrana celular

12. Pirazinamida
a. Procede del acido pirrolico
b. Actúa mejor a un pH ácido
c. Hepatotoxico
d. Bacteriostatico
e. No atraviesa la BHE

13.Generalmente esperamos infecciones por la tuberculosis en Px con SIDA con niveles de CD4 menores de:
a. 25 cel/ul
b. 50 cel/ul
c. 75 cel/ul
d. 100 cel/ul
e. 200 cel/ul

14. Infección por tuberculosis avium en SIDA


b. 50 células

97
15. falta

16. Trasmisión de VIH sigue siendo más común en :


-Homosexuales

17. Sensibilidad de ELISA:


• 99%

18. Los valores de CD4 se valoran por


• Citometria de flujo

19. Considerar tratamiento viral cuando valores de cd son menores de:


• 350 cel

20.Considerar cambio de tratamiento retroviral cuando ay descenso de cd34 de


• 20%

21.Bacilos gram + como strepto. Nemuo, encapsulados son fagocitados por neutrofilos a traves de :
• Bloqueo de las toxinas

22.23.24.Candice dice que no las encuentra y que estas eran profetas

25.26.27. Yariela no las encuentra.

28- El siguiente es un anaerobio que produce neurotoxina:


a- Bacteriodes fragilis
b- C. botulinum
c- Clostridium perfringes
d- Lactobacilos
e- peptostreptococos.

29- Anaerobio con polisacarido capsular


a- Bacteriodes fragilis
b- C. botulinum
c- Clostridium perfringes
d- Lactobacilos
e- peptostreptococos.

30- Anaerobio con exotoxina potente que produce hemólisis:


a- Bacteriodes fragilis
b- C. botulinum
c- Clostridium perfringes
d- Lactobacilos
e- peptostreptococos.

31-32 - Nombre de anaerobios en lesión intracraneal.


a- Actinomyces
b- Fusobacterium

33- Bacteremia de la boca por anaerobios.

98
a- Bacteriodes fragilis
b- C. botulinum
c- Clostridium perfringes
d- Lactobacilos
e- peptostreptococos.

34-35-36 La actinomicosis es una condición relativamente rara con distribución mundial y sin predilección por
edad, raza, estación del año y ocupación. Los factores predisponentes incluyen: Nombre tres condiciones que se
asocian a esta enfermedad

a- Inmunosupresión
b- Mala higiene dental
c- Uso de dispositivos intrauterinos

37- Actinomicosis:
a- es Crónica
b- Se presenta fístula
c- se confunde con celulitis
d- Drena material azurofilo
e- Presenta masa supurativa.

38- Intoxicación alimentaria después del stafilococo aureus:


a- Bacteriodes fragilis
b- C. botulinum
c- Clostridium perfringes
d- Lactobacilos
e- peptostreptococos.

39- Tratamiento de elección del actinomices:


a- Metronidazol
b- Penicilina sodica
c- clindamicina
d- tetraciclina
e- Quinolona

40.41.42 Mencione 3 antibioticos que tengan metabolismo hepatico:


Tetraciclinas, Doxiciclina, Isoniasida, Nitrofurantoina, Ciprofloxacina, Eritromicina, Amoxicilina, Acido
Clavulanico, etc.

43.44.45 Mencione 3 antibioticos que se pueden usar en el embarazo:


Penicilina, Ampicilina, Amoxicilina, Cefuroxima, Ceftriaxona, Eritromicina, Azitromicina,
Claritromicina

46. Tratamiento de ricketzias


a. doxiciclina
b. amoxicilina
c. ampicilina
d. penicilina
e. vancomicina

47. Tratamiento de leptospira


a. doxiciclina
b. amoxicilina
c. ampicilina

99
d. penicilina
e. vancomicina

48. VIH se aisló


a. 1980
b. 1981
c. 1983
d. 1985
e. 1987

49. La carbenicilina no debe de usarse en infeccion por:


a. Pseudomona Aeruginos
b. Especies de Proteus, con excepcion de P. Mirabilis
c. Algunas cepas de enterobacter
d. Klebsiella spp. y algunas especies de Serratia
e. Anaerobios

50. Las carboxipenicilinas y las ureidopenicilina difiern de la ampicilina en su mayor eficacia contra muchos:
a. Gram –
b.Anaerobios
c.Gram+
d.Micoplasma
e.No difieren

100
16. La endocarditis agua generalmente es causada por una de las siguientes bacterias excepto:
a. S. aureus
b. Strept. Viridans
c. A. Hemolytic streptococci
d. pneumococci
e. gonococci

17. Los nodulos eritematosos y dolorosos que aparecen en los dedos a nivel digital se llaman :
a. Nodulos de Osler
b. De Roth
c. De Janeway
d. De Heberden
e. De Huges

18. Criterio mayor de Duke para endocarditis:


a. Serologia de coxiella Burnetti
b. fiebre
c. petequia conjuntival
d. uso de droga IV
e. Factor reumatoideo positivo

19.El tratamiento profilactico de la endocarditis en manipulaciones dentarias:


a. una hora antes del procedimiento
b. cuatro horas antes
c. seis horas antes
d. ocho horas antes
e. doce horas antes

20.21.22. En la neumonía tres signos de distres respiratorios


- disnea
- taquipnea
- taquicardia

23. Defensa frente a M. tuberculosis depende esencialmente de :


a. Anticuerpo de la clase IgG
b. Interferon alfa
c. leucocitos polimorfonucleares basofilos
d. macrofagos activador por interferon alfa
e. leucocitos polimorfonucleares eosinofilos

24. Paciente hombre de 15 años de edad que realiza viaje a Italia, al mes de regreso comienza con malestar
general, odinofagia, fiebre, exudado blanquecino, adenopatia occipital, …..Hemograma leucitos algunos….
a. Biopsia ganglionar
b. Biopsia de medula
c. tratamiento con penicilina
d. Serologia por Virus del Epstein Barr
e. tratamiento con clindamicina

25. Paciente con fiebre de 39° diarrea sanguinolenta mas de una semana, positivo para C. yeyuni, fármaco de
elección:
101
• azitromicina o amoxicilina

26. Qué es el dengue:


• es una enfermedad virica que puede producir fiebre hemorrágica

27. Indicación de aminoglicósidos por vía oral:


• tracto gastrointestinal

28. Cuando realizamos un hemocultivo a 1 paciente ¿Cuál de los siguientes datos nos hace pensar que no
estamos ante un caso de contaminación?
• aislamiento mismo microorganismo en hemocultivos con la misma sensibilidad.

29.En relación a la sífilis, señale la afirmación correcta:


• chancro sifilítico es indurado, no doloroso y muy rico en bacterias.

30. Malaria, afirmación falsa:


• malaria cerebral es una manifestación típica de plasmodium vivax

31. En un paciente diabetico con historia de convulsiones y con una infeccion nosocomial por germenes multi
resistentes, usted evitaria el uso de:
R. Imipenem.

32. Uso del cloranfenicol, especialmente ante la falla de beta lactamicos:


R. Infeccion del SNC por anaerobios.

33. Puede causar infeccion respiratoria, bronquitis, resfriado comun, y es el causante de la epidemia de
Sindrome Respiratorio Agudo Severo (SARS) que afecto a China y a otros paises del medio oriente:
R. Coronavirus.

34. Cual de las siguientes infecciones por Bacilos Gram Positivos es mediada por invasion y diseminacion
hematogena del germen causal, y no por toxinas:
R. Antrax.

35. Cual de los siguientes germenes se ha asociado a Colitis Hemorragica y al Sindrome Hemolitico Uremico:
R. Escherichia coli.

36. Una chica de 29 años es evaluada en el Servicio de Urgencias por fiebre, escalofrios, tos seca, dolor
pleuritico del lado derecho, cefalea universal. El unico antecedente de importancia es una Esplenectomia hace
un año por accidente de carro. Al examen fisico encontramos PA 90/60, Fc 96, Temperatura 38 C. Paciente
consciente, desorientada en lugar. Pulmones con craquidos en base derecha. Tiene rigidez nucal. Resto del
examen normal. Laboratorios: leucocitosis de 16000, con 80% de eosinofilos. Rx de toraz con infiltrado y
consolidacion del lado derecho. Cual es el siguiente paso en el abordaje de este paciente?
R. Realizar una Puncion Lumbar.

37- Continuando con el abordaje de la paciente, el agente etiológico que puede explicar este síndrome clínico es:
a) Listeria Monocytogenes xq es un pte inmunosuprimido x la esplenectomía
b) Haemophylus influenza B a pesar de haber recibido la vacuna
c) Streptococo penumoniae
d) Neisseria meningitidis xq posiblemente es promiscua
e) Bacterias atípicas

102
38- Si el paciente tiene una mejoría al 3r día de Tx, caracterizado x disminución de la T°, Fc y puede tolerar
la dieta, el siguiente paso a seguir es:
a) Iniciar antibiótico vía oral y omitir el parenteral
b) Seguir el Tratamiento parenteral por 7 a 10 días
c) Dar de alta este 3r día con los antibióticos orales respectivos
d) Seguir con ambos Tx vía oral y parenteral x 2 días más
e) N.A

39- Los aneurismas micóticos:


a) son causa de endocarditis x candida
b) de evolución aguda
c) las bifurcaciones de grandes vasos
d) todas las anteriores
e) NA

40- Alternativo de Tx para un cuadro de secreción transuretral en mujer embarazada:


a) Ofloxacina 400mg V.O. BID por 7 días
b) Doxiciclina 1OOmg V_O_ BID por 7 días
c) Azitromicina lg dosis única
d) NA

41- Fiebre por dengue Ud. Recomienda que regrese y consulte por:
a- fiebre
b- Diarrea
c- Hiperemesis
d- todas
e- Ninguna

42- En cuál de las siguientes entidades la velocidad de eritrosedimentación no es ayuda al dx. diferencial de
dengue:
a. Influenza
b. Sarampión
c. Mononucleosis infecciosa
d. Infección por arenavirus
e. Ninguna de las anteriores.

43.Final del Sndr pulmonar por hantavirus es cierto todo lo siguiente excepto
R:

44.Tx para salmonella


R: ciprofloxacina

45.Meningitis aseptica o abacteriana


R: Es rara en ancianos

46. En la meningitis de los ancianos siempre hay que tener presente :


a. Listeria Monocitogena

47. Si el LCR usted observa un bacilo gram (-) usted consideraria que se trata de :
a. Haemophilus Influenzae
b. Streptococo
c. Listeria Monocitogena
d. Neisseria Meningitidis
e. Ninguna de lãs Anteriores

103
48. Tratamiento de meningitis por streptococos pneumoniae debe darse:
a. 7 dias
b. 14 dias
c. 21 dias
d 30 dias
e. mas de 30 dias

49.50. Describa el tratamiento profiláctico para meningitis por meningococo, dosis y tiempo:
a. Rifampicina _________________________
b. Ciprofloxacina _______________________

PARCIAL 2 2006 1
Una joven de 33 años de edad presenta fiebre de 39.4ºC, cefalea y rigidez de cuello. Usted realiza el
diagnostico de meningitis bacteriana y comienza con antimicrobianos. Con la meningitis bacteriana ¿Cuál de
los siguientes es un hallazgo probable en liquido cefalorraquideo (LCR)?
f) Leucocitos entre 100 y 500 ml
g) Presión del LCR entre 100 y 120 mm H2O
h) Tinción de gram negativa
i) Glucosa >120 mg/dl
j) Valores de proteinas >45 mg/dl

En este joven adulto por lo demás sano ¿Cuál es el microorganismo más probable?
f) Streptococcus del grupo B
g) Stafilococcus aureus
h) Haemophilus influenzae
i) Streptococcus pneumoniae
j) Listeria monocytogenes

Un cuadro febril cuyas oscilaciones diarias de la temperatura son superiores a 1ºC sin alcanzar nunca los
valores normales, corresponde a la definición de:
f) Fiebre continua
g) Fiebre recurrente
h) Fiebre remitente
i) Fiebre intermitente
j) Fiebre héctica

¿De que enfermedad es característico la pupilas de Argyll-Robertson?


f) Tuberculosis
g) Lepra
h) Sífilis
i) Toxoplasmosis
j) Tetáno

** ¿Cuál es la triada semiológica clínica del síndrome meningitis?


f) Fiebre, cefalea y rigidez de nuca (según libro)
g) Fiebre, cefalea y vómitos
h) Fiebre, rigidez de nuca y signos encefálicos (según Arjona)
i) Cefalea, rigidez de nuca y signos encefálicos
j) Cefalea, vómitos y rigidez de nuca

Entre los múltiples procesos infecciosos que producen adenopatías cervicales no suelen incluirse: TODAS
f) Mononucleosis infecciosa
g) Rubéola
104
h) Enfermedad por arañazo de gato
i) Tuberculosis
j) Faringitis estreptocócica

¿Cuál de los siguientes antibioticos es el que tiene menor difusión al LCR?


f) Clindamicina
g) Metronidazol
h) Cotromoxazol
i) Penicilina
j) Cefotaxima

Cuando un paciente ingresa por sepsis por Stafilococcus aureus adquirida en la comunidad y sin foco aparente.
¿Qué tipo de proceso patologico debe se descartado?
f) Un absceso cutaneo oculto
g) Una neumonía -
h) Una osteomielitis vertebral -
i) Una endocarditis -
j) Un absceso renal

Con respecto a la salmonella Typhi. ¿Cuál de las siguientes afirmaciones es correcta?


f) Es la unica especie de salmonella que es patógena tanto para el hombre como para los animales
g) La vía de contagio de salmonella para el hombre es orofecal
h) La infección por S. Typhi ocurre prácticamente solo en verano
i) El inoculo mínimo necesario de S. Typha para desencadenar una infección es muy bajo de solo unas
1,000 colonias de bacterias viables. (no)
j) Más del 50% de los pacientes con fiebre tifoidea presentan manifestaciones neuropsiquiatritas.

** ¿Cuál de los siguientes antibióticos es el menos eficaz para el tratamiento de la uretritis gonocócica?
f) Ceftriaxona 500mg IM (125mg)
g) Cefixima 400mg oral en una sola dosis
h) Azitromicina 1 g una dosis
i) Ciprofloxacina 1 g en una dosis (500mg)
j) Cefuroxina 750 mg en una sola dosis

¿Cuántos días debe prolongarse el tratamiento antibiótico eficaz de una meningitis meningococcica?
f) 1-3
g) 4-7
h) 8-10
i) 12-15
j) 18-21

Que concentración de bacilos tuberculosis en esputo es necesario para que la baciloscopia (tinción de Ziehl) sea
positiva.
f) 100 bacilos /ml
g) 1000 bacilos /ml
h) 10000 bacilos /ml
i) 100000 bacilos /ml
j) 1000000 bacilos /ml

El síntoma más común de la TBC pulmonar es:


f) Tos
g) Hemoptisis (en ocasiones)
h) Fiebre
i) Perdida de peso
105
j) Anorexia

Cual de las siguientes enfermedades puede confundirse con tuberculosis pulmonar (Son tres las entidades que
con mayor frecuencia deben diferenciarse de la tuberculosis pleural: neumonía bacteriana, tromboembolismo
pulmonar y neoplasia pleural).

f) Sarcoidosis
g) Enfermedad por legionella
h) Histoplasmosis pulmonar (no)
i) Enfermedad metastásica pulmonar
j) Ninguna de las anteriores

La siguiente droga requiere de co administración de piridoxina el tratar la TBC


f) Isoniacida
g) Rifampicina
h) Etambutol
i) Rifampicina
j) Estreptomicina

Nombre dos condiciones de tuberculos extrapulmonar en donde usaria corticoides


TBC cerebral (meningea)
TBC pericardica

Unas de las siguientes drogas para el tratamiento de la TBC se metabolizan por acetilación
f) Isoniacida
g) Rifampicina
h) Etambutol
i) Rifampicina
j) Estreptomicina

La profilaxis de plasmodium falciparum resistente a la cloroquina se realiza con:


f) Cloroquina
g) Primaquina
h) Mefloquina
i) Quinina
j) Pirimetamina

De las siguientes aseveraciones una es cierta respecto a la amebiasis


a) La transmisión por artropodos es frecuente
b) Transmisión de persona a persona es importante
c) La infeccion pulmonar es la mas frecuente extraintestinal
d) La forma quistica es la infectante
e) Ninguna de las anteriores

10. La penicilina G sigue siendo efectiva contra unas de estas bacterias, menciónela:
a. Actinomises israeli
b. Neisseria gonorroheae
c. Treponema pallidum
d. Legionella
11. Medicamento que no se debe modificar en el tratamiento de la insuficiencia renal:
a. Penicilina

106
b. Amoxicilina
c. Doxicilina
d.Tetraciclina
e.Clindamicina
f. Meropenem
12. Medicamento que se debe modificar en un paciente con insuficiencia hepática:
a. Gentamicina
b. Vancomicina
c. Cefuroxima
d. Rifampicina
e. Ciprofloxacino
13. Neumonía adquirida en la comunidad:
● Cefalosporina de tercera generación
PAREO:
14. Colecistitis aguda: Imipenem
15. Neumonía de la Comunidad: Cefalosporina de 3ra generación
16. Infección urinaria no complicada: Fosfomicina
17. Uretritis no gonocócica: Doxiciclina

18. Cefalosporina de 1ra generación de uso parenteral:


a. Cefalotina
b. Cefradina
c. Cefaclor
d. Cefadroxina
19. La única cefalosporina que se administra por vía oral:
● Cefuroxima-acetil
20. Si un paciente es alérgico a la penicilina en que porcentaje sería alérgico a las cefalosporinas de 3ra
generación:
● 10 %
21. Antibiótico que actúa sobre el RNA polimerasa:
● Rifampicina
22. Antibiótico bacteriostático que trabaja junto con PABA:
● Sulfonamida
27. Paciente con comunicación interauricular debe recibir profilaxis con antibióticos durante:
● Extracción Dental
30. La TBC renal se da principalmente:
● Edad avanzada
107
36. De los siguientes hallazgos cual no es característico de HANTA:
a. Leucocitosis
b. Hipercapnia
c. VES aumentado
d. Hipoxia
e. Trombocitopenia
37. Paciente con leucocitos por debajo de 200, tratamiento inmediato:
● Aislar al paciente
40. En que se usa la tinción de Giemsa:
● En la sangre
41. Cual de los siguientes no produce meningitis:
● Enterococo
42. Una de las siguientes no es una condición de uso de un antimicrobiano para uso por vía parenteral:
● Pobre penetración al SNC
43. Cuando se administran aminoglucósidos observar por:
● Falla renal
44. Paciente con hematuria, proteinuria, rash cutáneo y nefritis intersticial. Cuál medicamento lo produce:
● Penicilina y sus derivados (Meticilina)
45. Hay algunos fármacos que requieren vigilancia con hemograma por inmunodepresión durante su
utilización. Este fármaco no requiere vigilancia con hemograma por inmunodepresión
● Ceftriaxona
46. De los siguientes fármacos el único no que se elimina por vía hepática es:
● Cefalosporina
47. Uno de los siguientes antimicrobianos se puede usar en cualquier trimestre del embarazo:
● Betalactámico
49. Tiempo que demoran en desaparecer las adenopatías en el Dengue:
● 72 horas
50. Tratamiento de Chlamidia Tracomatis:
● Doxiciclina
● Acitromicina

108
REUMA
79. Paciente mujer de 48 años, sin antecedentes médicos de interés que presenta cuadro de
aproximadamente 3 meses de evolución consistente en: poliartritis de manos, muñecas
y rodillas, con rigidez matutina de 2 horas y factor reumatoide elevado en la analítica que le
realizó su médico de cabecera. Respecto a su enfermedad, ¿cuál de las siguientes afirmaciones
resulta INCORRECTA?:
1. Por la clínica que presenta la paciente padece una artritis reumatoide (AR).
2. Parece adecuado comenzar tratamiento con AINEs y/o corticoides a bajas dosis para conseguir
alivio sintomático.
3. Es importante comenzar lo antes posible tratamiento con fármacos modificadores de la
enfermedad (FME), inlcuso en combinación.
4. Antes de empezar el tratamiento con FME se debe esperar otros 3 meses a comprobar la
respuesta al tratamiento con AINEs y/o corticoides.
5. La presencia de erosiones radiológicas tempranas sería un signo de mal pronóstico.

80. A un varón de 65 años asintomático, se le encuentra en un estudio de control general una


cifra de fosfatasa alcalina dos veces superior a lo normal, con pruebas hepáticas normales. La
gammagrafía ósea muestra captación en la mitad superior de la hemipelvis derecha, y una
radiograía realizada en esta zona pone de manifiesto un patrón de corticales aumentadas de
grosor y patrón trabecular grosero. ¿Qué tratamiento es el más apropiado?:
1. Alendronato, 40 mg diarios.
2. Naproxeno, 500 mg dos veces al día.
3. Calcio (400 mg.) y vitamina D (400 UI).
4. Calciatonina nsal, 200 UI/día.
5. No es preciso tratamiento.

81. Un joven de 21 años viene a la consulta porque tiene desde hace 4 ó 5 meses un dolor
constante en la región lumbosacra, que es peor en las primeras horas del día y mejora con la
actividad. Recuerda que 2 años antes tuvo un episodio de inflamación en rodilla que se resolvió
completamente. También se queja de dolor en la caja torácica con los movimientos
respiratorios. El examen físico demuestra la ausencia de la movilidad de la columna lumbar.
¿Cuál de los siguientes test nos ayudaría a confirmar el diagnóstico del paciente?:
1. El TAC de columna lumbar.
2. Las radiografías de las articulaciones sacroilíacas.
3. Los niveles de antiestreptolisina 0 (ASLO).
4. La radiografía de tórax.
5. Los niveles de la proteína C reactiva (PCR) y velocidad de sedimentación.

82. En el lupus cutáneo subagudo es característica de la presencia de anticuerpos:


1. Anti-Sm.
2. Anti-centrómero.
3. Anti-Ro.
4. Anti Jo1.
5. Anti-histona.

84. En una esclerosis sistémica progresiva (esclerodermia) el peor pronóstico se asocia con:
1. Una extensa calcinosis cutánea.
2. La presencia de disfagia.
3. El número de articulaciones inflamadas.
4. La presencia de insuficiencia renal.
5. Fenómeno de Raynaud severo con úlceras necróticas en dedos.

85. ¿Cuál de estas afirmaciones es FALSA con respecto a las artritis inducidas por
microcristales?:
1. Pueden ser producidas por cualquiera de los siguientes cristales: urato monosódico, pirofosfato
cálcico, hidroxiapatita cálcica y oxalato cálcico.
2. El depósito de cristales puede producir cuadros clínicos similares a la artritis reumatoide o la
espondilitis anquilosante.
3. Los cuadros clínicos producidos son específicos para cada uno de los tipos de cristales
depositados.
4. Para hacer el diagnóstico es imprescindible el estudio del líquido sinovial con microscopio
de luz polarizada para identificar el tipo de cristales.
5. El líquido sinovial suele ser de tipo inflamatorio, aunque en ocasiones existen cristales en
ausencia de inflamación.

86. ¿Cuál de las siguientes afirmaciones en relación con la enfermedad de Wegener es


FALSA?:
1. Es una vasculitis sistémica que afecta sobre todo a vasos de mediano calibre.
2. En ausencia de tratamiento cursa de manera progresiva y con frecuencia mortal.
3. Presenta con frecuencia afectación renal, que es histológicamente indistinguible de la
glomerulonefritis necrótica con semilunas.
4. En presencia de afectación pulmonar y/o renal, el uso de ciclofosfamida vía oral es casi siempre
imprescindible para obtener un buen control de la enfermedad.
5. Se asocia a la presencia de anticuerpos anticitoplasma de neutrófilo (ANCAS) con patrón de
inmunofluorescencia de tipo citoplásmico.

142. Enferma de 32 años que cuando acude a la consulta refiere que hace unos 20 días,
después de una exposición solar, le aparece en la zona externa, hombros, brazos y región
escapular, unas lesiones anulares, eritematoedematosas en su borde y con regresión
central, algunas confluentes de dos o tres centímetros de diámetro que apenas le
ocasionan molestias. El diagnóstico sería:
1. Eritema polimorfo.
2. Liquen plano.
3. Porfiria hepatocutánea.
4. Lupus eritematoso cutáneo subagudo.
5. Dermatomiositis.

148. Varón de 28 años que presenta en su ojo izquierdo cuadro de vasculitis retiniana severa
y edema macular con una agudeza visual 0.1. El paciente refiere historia de aftas bucales y
genitales recidivantes. En base a los hallazgos clínicos y a la historia del paciente ¿cuál
considera, de los siguientes, que es el diagnóstico más probable?:
1. Sarcoidosis.
2. Enfermedad de Behçet.
3. Esclerosis múltiple.
4. Tuberculosis.
5. Sífilis.

186. Chico de trece años que refiere episodios de dolor abdominal, no filiados, y artralgias
erráticas en codos, rodillas. codos y muñecas. En las últimas 24 horas le han aparecido
manchas rojizas en muslos. Lo más destacado de la exploración física es la existencia de
púrpura palpable en nalgas y muslos. No presenta anemia, las plaquetas son normales, la IgA
está elevada y el aclaración de creatinina es normal. Se objetiva proteinuria de 1 gr/24 horas y
50-70 hematíes por campo. en la biopsia renal se observa proliferación mesangial y depósitos
de IgA (+++) e IgG(+). El diagnóstico más probable es
1. Vasculitis tipo PAN microscópica.
2. Lupus eritematoso sistémico.
3. Enfermedad de Wegener.
4. Síndrome de Goodpasture.
5. Síndrome de Schönlein-Henoch.

LOS QUE SON EN REUMA


1) El número de células del líquido sinovial es bajo en:

a Artritis tuberculosa b. Artritis reumatoidea c. Artrosis d. Fiebre reumática e. Artritis post


disentería

2) . Manifestaciones extraintestinales de CUCI, excepto:


a. Esteatosis Hepática b. Manchas cutáneas perianales c. Colangitis Esclerosante
d. Pioderma gangrenosa e. Carcinoma Hepatocelular

3) Señalar las enzimas que no son utilidad en el diagnóstico de la polimiositis

a. Creatinfosfocinasa b. Transaminasa glutamicooxalacética c. Transaminasa glutamicopirúbica


d. Aldolasa e. Ninguna de las anteriores

4) Que articulación es la que presenta artrosis con menos frecuencia:

a. Cadera b. Trapeciometarcarpiana c. Rodilla d. Tobillo e. vetebrales

5) Se ha visto que es muy útil en el diagnóstico de la poliarteritis nudosa:

a. Factor reumatoide b. Biosia sinovial c. Anticuerpo anti-DNA


d. Arteriografía e. Tasa de gammaglobulina

6) La crisis de gota se debe:

a. A la precipitación de uratos b. Al depósito de uratos


c. A la no eliminación de ácido úrico por el riñón
d. A una mayor concentración de ácido úrico en la articulación
e. A un aumento del pool de uratos

7) La síntesis de calcitonina aumenta cuando:

a. Disminuye la calcemia b. Aumenta la fosforemia c. Aumenta la calcemia


d. Disminuye la magnesemia e. Aumenta la magnesemia

8) Indicación más precisa y útil del análisis del líquido sinovial:


a. Artritis reumatoide b. Artritis por cristales c. Artropatía de charcot.
d. Artritis traumática. e. Osteoarthritis

9) Un varón de 22 años de edad se queja de dolor bajo de espalda y rigidez, que empeoran al levantarse y mejoran
con el ejercicio. En la exploracion se le encuentra movilidad limitada de las articulaciones sacroiliacas y de la
columna lumbar. Una prueba en suero para antigeno de histocompatibilidad HLA-B27 es positiva. ¿cuál es la
manifestación extraesquelética más frecuente de esta enfermedad?
a. Catarata prematuras b. Esplenomegalea c. Iritis aguda
d. Insuficiencia aortica e. Fibrosis pulmonar

10) Una paciente la cual se encuentra en estudio por artralgia, dentro de los laboratoriso le sale positivo el anticuerpo
Anti-histona; su preferencia diagnóstica seria:
a. Artritis Reumatoidea b. Lupus eritematoso sistemémico c. Síndrome de Sjogren
d. Lupus inducido por drogas e. Vasculitis autoinmune

11) Son indicaciones de densitometría, excepto:


a. Menopausia b. Inmovilización prolongada c. Osteopenia radiológica
d. Tratamiento con corticoides e. Tratamiento prolongado con heparina

12) Los anticuerpos antifosfolípidos producen sobre las plaquetas:


a. Aumento en el número de plaquetas
b. Aumento de la actividad de plaquetas
c. Aumento en la producción de la médula ósea
d. Aumento en el secuestro por el bazo
e. No tiene ningún efecto sobre las plaquetas

13) Es cierto para los anticuerpos antifosfolípidos:


a. El VDRL es falso negativo
b. TPT esta alargado generalmente
c. Test de Coombs es positivo
d. En la perdida fetales solo hay uno de los dos anticuerpos
e. La causa mas frecuente estadísticamente es la isquemia vascular cerebral
14) Son síndrome hipercoagulables, excepto:
a. Embarazo b. Hiperhomocisteína c. Neoplasias d. Sindrome antifosfolipido e. Sepsis

15) Articulación menor afectada en la Artritis Reumatoide:


a. interfalángicas proximales b. interfalángicas distales. c. Sacroilíacas
d. Temporo-mandibulares. E. Codo

16) El HLA B-27 tiene utilidad en:


a. Artritis Reumatoide. B. Artritis Reumatoide Seronegativa. c. Espondilitis anquilosante. d. Osteoartritis
e. Síndrome de la enfermedad sin pulso

17) Señora de 90 años de edad acude a urgencias por hinchazón muy dolorosa de rodilla izquierda, toma diuréticos
por hipertensión arterial. Usted punciona la rodilla y extrae lìquido cetrino, poco viscoso lo analiza al microscopio y
observa cristale birrefringentes negativos en forma de agujas dentro de los PMN y cristales romos abundantes con
birefringencia positiva. La RX de la rodilla muestra condrocalcinosis. Diagnóstico:
a. Artritis Sèptica. B. Artritis por acido ùrico. c. Artritis por pirofosfato cálcico.
d. Gota . e. Sjogren

18) A esta paciente usted la trataria con

a. Antinflamatorios no esteroideo b. Colchicina c. Corticoides


d. Azatioprina e. no se le da tratamiento

19) La polimiositis es una enfermedad del músculo estriado que provoca:


a. pérdida de fuerza y atrofia b. dolor y atrofia c. perdida de la fuerza e hipertrofia
d. dolor y deformidad e. No afecta al músculo
20) Citar, que sustancia no interviene en el metabolismo óseo:
a. Albumina b. Somatomedina c. Tiroxina d. calcitonina e. Interleukinas

21) Los nódulos de Bouchard son la denominación de la artrosis de:


a. Tobillo b. Interfalángica proximales c. Interfalángicas distales
d. Trapeciometacarpiana c. En la espalda

22) La artritis en el lupus eritematosos sistémico se caracteriza por:


a. Ser siempre deformante b. No presentar erosiones c. Se idéntica a la reumatoidea
d. Ser parecida a la espondilitis anquilosante e. Siempre simétrica

23) Citar qué enfermedad no presenta fenómeno de Reynaud:


a. Artritis psoriatica b. Artritis reumatoidea c. Síndrome de Sjogren
d. Enfermedad mixta del tejido conectivo e. Ninguna de las anteriores

24) En el Lupus Eritematoso Sistémico:


a. El ANA es positivo en el 95% de los pacientes.
b. El anti-DNA nativo y el anti-Smith son frecuentes
c. El Factor Reumatoide siempre es negativo.
d. Es más frecuente en mujeres orientales.
e. Su edad de aparición es arriba de los 50 años

25) En la artritis reactiva:


a. El staphilococo es agente inductor
b. Es más frecuente el gonococo
c. El ANA es de utilidad
d. Suele estar asociada a bacterias urogenitales.
e. Ninguna de las anteriores es cierta

26) En la Artritis Reumatoide Juvenil:


a. Suele ser poliarticular
b. La variante de Still cursa con serositis.
c. Rara vez afecta el ojo.
d. El tratamiento de elección es la prednisona.
e. Afecta la articulación atlato axial

27) En la Espondilitis anquilosante:


a. El Test de Schöber es positivo y sacroiliitis bilateral.
b. Factor reumatoide y ANA positivo.
c. HLA-B27 negativo.
d. más frecuente en tercera edad.
e. Solo el factor reumatoide es postivo

28) El tratamiento de elección en la embarazada con síndrome antifosfolípo es


a. Aspirina y heparina de bajo peso molecular b. Aspirina sola c. Warfarina
d. Heparina sódica e. No tiene tratamiento

29) Paciente que presenta artritis reumatoidea, leucopenia y esplenomegalia, se trata de

a. Síndrome de Hughes b. Síndrome de Felty c. Síndrome de Sjogren`s


d. Síndrome de Reiters e. Síndrome de Barré

30) El factor reumatoideo no se halla en:

a. Hipotiroidismo b. Lupus eritemotoso sistémico c. Esclerodermia


d. Artritis reumatoidea e. Ninguna de las anteriores

31) En el síndrome de CREST sobra uno de los siguientes:

a. Calcicosis b. Telangiectasia c. Escleroderma d. Irido escleritis


e. Raynaud
32) Para monitorizar la actividad clínica del lupus eritematoso sistémico es util:

a. La VSG y la cifra de leucocitos b. Los anticuerpo antinucleares c. La proteína C


b. Las proteínas séricas
d. El C3
33) Las manifestaciones neurológica más frecuente del lupus eritematosos sistémico son:

a. Ataxia cerebelosa b. Depresión y psicosis c. Hemiplejia y paraplejia


d. Polineuritis y mononeuritis múltiple e. Convulsiones

34) Síndromes Reumatológicos más frecuentes en el adulto mayor :

a- Artritis Reumatoide y Síndrome de Sjögren.


b- Arteririts Temporal y Pseudogota.
c- Gota y Esclerodermia.
d- Wegener´s graulomatosis y Fibromialgia.

35) Citoquinas que son blanco de anticuerpos monoclonales en el manejo de la


Artritis Reumatoide.
a. Interferon Beta y TNF alfa. b. TNF alfa e IL-6 c. L-2 e IL-18
d. CD20 y CD3. e. Ninguna de las anteriores

36) Paciente femenina de 65 años, con dolor súbito en la espalda a nivel de la


vértebra T10. Utiliza corticoides en forma crónica por atopia y asma bronquial.
Sospecha Osteoporosis que haría:
a- Densitometría ósea de cadera y columna.
b- USG de calcáneo.
c- Radiografías de columna torácica AP y lateral.
d- Iniciaría tratamiento con calcio y Vitamina D.

37) Vasculitis Primarias c-anca´s (anticuerpo contra citoplasma de neutrófilos) asociadas :


a- Takayasu. b. Wegener c. Kawasaki d. Células gigantes.

38) Enfermedades reumatológicas con agente infeccioso claramente asociada:


b- Lyme y Hanta Virus,
c- Fiebre Reumática y Enfermedad De Still.
d- Enfermedad de Lyme y Fiebre Mediterránea familiar.
e- Fiebre Reumática y Enfermedad de Lyme

39) Varón de 35 años , abogado luego de reunión con sus colegas en Hotel todo
incluído, amanece con artritis severa del Primer metatarsiano pie izquierdo.
Es alérgico a los AINES que opción terapéutica escogería:
a- Corticoide sistémico IM
b- Morfina v.o oral 15 mg /dia.
c- Metotrexate Parenteral.
d- Pulsos de 1 g de metilprednisolona IV.

40) El perfil de pseudogota sería.


a- mujer de 85 años con artritis de codo.
b- Mujer de 87 años con artritis de rodilla.
c- Mujer de 56 años con poliartritis.
d- Varón de 45 años con artritis de primera metatarso- falángica.

41) La Fibromialgia es un cuadro característico de:


a- Varón de 55 años con mialgias
b- Mujer adulta joven con mialgias y trastornos del sueño y afectivos.
c- Mujer de 58 años con polimialgias y VSG aumentado.
d- ama de casa con dolor en los hombros .

42) Las fracturas más frecuentes después de las vertebrales y cadera en la osteoporosis son las de :
a. Hombro b. Colles c. Tobillos d. Rodillas e. Dedos de la mano

43) Para evitar en lo posible la osteoporosis por corticoides se recomienda administrar uno de los siguientes:
a. Calcio con vit D b. Estrógenos c. Andrógenos d. Bifosfonato e. Flúor

44) Las dos hormonas fundamentales en el metabolismo fosfocálcico son:


a. Calcitonina y ACTH b. Parathormona y vitamina D c. Vitamina D y calcitonina
d. Parathormona y calcitonia e. ACTH y Parathormona

45) Citar que sustancias no intervienen en el metabolismo óseo:


a. Somatomedina b. Estrógenos c. Prostaglandinas d. globulinas e. Insulina
46) En el riñon la parathormona:
a. Aumenta la reabsorción de calcio y disminuye la de fósforo
b. Aumenta la reabsorción de calcio y fósforo
c. Disminuye la reabsorción de calcio y fósforo
d. Disminuye la reabsorción tubular de calcio y aumenta la de fósforo
e. Solo aumenta la reabsorción de calcio

47) La forma biológicamente más activa de la vitamina D:


a. 1,25(OH)D2 b. 1,25(OH)2D3 c. 2,25(OH)2D3 d. Calciferol e. 1,24(OH)2D3

48) mencionar que célula no es específicamente ósea:


a. Osteoblasto b. Osteoclasto c. Megaosteoclasto d. Osteocito e. Osteocito de superficie

49) Cuando se decide darle tratamiento para osteoporosis a una paciente con raloxifeno, Usted sigue preocupado
debido a que tiene incremento en:
a. El cáncer de seno
b. Trombosis venosa profunda
c. Hiperplasia cervical
d. Fractura vertebrales
e. Riesgo de endermedad coronaria

50) Un paciente le entrega un resultado de una densitometria, se considera que tiene una osteoporosis si muestra
resultados de:
a. -0.5 b. -1.0 c. -1-5 d. -2.0 e. -2.5

76) La enfermedad en este paciente puede ser mas severa en pacientes con:
A Brucelosis.
B Anemia falciforme
C Diabetes mellitus
D Pacientes con SIDA.

77) Una complicación tardia que usted puede encontrar en este paciente es:
A-Sindroma de Guillain-Barré.
B-Osteomielitis.
C-Perforación intestinal
D-Endocarditis Bacteriana
79) En cuanto a los test no treponemico para sífilis puede dar falso positivo en fase temprana las siguientes entidades:
A Fiebre escarlatina.
B Mieloma
C Edad avanzada
D Transfusiones multiples.

80) En cuanto a los test treponemicos para Sífilis pueden dar falso positivo las siguientes unidades:
A Leptospirosis B Malaria C Pneumonía por Pneumoco D Psitacosis

93) . Para evitar en lo posible la osteoporosis por corticoides se recomienda administrar uno de los siguientes:

a. Calcio con vit D


b. Estrógenos
c. Andrógenos
d. Bifosfonato
e. Flúor
94) En la osteoporosis los dolores raquídeos se deben a:
a. Fracturas vertebrales
b. Lesiones discales
c. Espondilosis vertebral
d. Deformaciones raquídeas
e. Falta ce calcio en los huesos

1. Una mujer de 28 años con anticuerpos anticardiolipina y antecedentes de tres


abortos en el primer trimestre del embarazo es evaluada en la sexta semana de un
cuarto embarazo. El embarazo actual transcurre con normalidad. Nunca ha sido
tratada por la positividad de los anticuerpos anticardiolipina. ¿Cuál de las
siguientes es la conducta más apropiada?:

1. Observación estrecha.
2. Prednisona.
3. Aspirina.
4. Infusión intravenosa de inmunoglobulinas.
5. Heparina y aspirina.

2. El tipo de afección pulmonar más frecuente en los enfermos con esclerodermia


limitada es:

1. Fibrosis pulmonar.
2. Alveolitis.
3. Hipertensión arterial pulmonar.
4. Neumonía de repetición.
5. Enfisema de predominio basal.

3. En relación con los hallazgos de laboratorio en la panarteritis nodosa (PAN),


señale la afirmación correcta:

1. En más del 90% de los casos hay anticuerpos anti-ribonucleoproteínas.


2. En más del 90% de los casos hay antígeno de superficie del virus de la hepatitis
B.
3. El hallazgo de títulos elevados de anticuer- 12 - pos antihistonas es muy
específico.
4. La ausencia de eosinofilia descarta el diagnóstico.
5. No existe ninguna prueba que sea específica.

4. A un varón de 65 años asintomático, se le encuentra en un estudio de control


general una cifra de fosfatasa alcalina dos veces superior a lo normal, con pruebas
hepáticas normales. La gammagrafía ósea muestra captación en la mitad superior
de la hemipelvis derecha, y una radiograía realizada en esta zona pone de
manifiesto un patrón de corticales aumentadas de grosor y patrón trabecular
grosero. ¿Qué tratamiento es el más apropiado?:

1. Alendronato, 40 mg diarios.
2. Naproxeno, 500 mg dos veces al día.
3. Calcio (400 mg.) y vitamina D (400 UI).
4. Calciatonina nsal, 200 UI/día.
5. No es preciso tratamiento.

5. Respecto a la poliarteritis nodosa (PAN), señale cuál de las siguientes


afirmaciones es FALSA:

1. La lesión vascular es de distribución segmentaria y se localiza sobre todo en la


bifurcación de los vasos.
2. Riñón y corazón son los órganos más afectados.
3. La afectación hepática es más frecuente en los casos asociados al virus B o
C de la hepatitis.
4. Puede aparecer a cualquier edad, pero es infrecuente en la infancia.
5. El 98% de los casos presenta anticuerpos c- ANCA positivos.

7. Enferma de 32 años que cuando acude a la consulta refiere que hace unos 20
días, después de una exposición solar, le aparece en la zona externa, hombros,
brazos y región escapular, unas lesiones anulares, eritematoedematosas en su
borde y con regresión central, algunas confluentes de dos o tres centímetros de
diámetro que apenas le ocasionan molestias. El diagnóstico sería:

1. Eritema polimorfo.
2. Liquen plano.
3. Porfiria hepatocutánea.
4. Lupus eritematoso cutáneo subagudo.
5. Dermatomiositis.

9. ¿Cuál de las siguientes respuestas NO es cierta acerca de la sarcoidosis?:

1. Existe una forma asintomática.


2. El 60% de los casos tienen una radiografía de tórax normal.
3. Existe linfadenopatía en más del 50% de los casos.
4. Suele existir hipergammaglobulinemia durante las fases de actividad.
5. Es rara la afectación pleural.

1. Un hombre de 32 años, con antecedentes paternos de espondilitis anquilosante,


consulta por dolor lumbar de 6 meses de evolución, de presentación durante el
reposo nocturno, acompañado de rigidez matutina intensa que mejora con la
actividad hasta desaparecer 2 horas después de levantarse. Refiere haber notado
mejoría importante de los síntomas en tratamiento con diclofenaco. La exploración
física únicamente muestra limitación ligera de la movilidad lumbar. ¿Cuál de las
siguientes pruebas complementarias considera más adecuada, inicialmente, para
establecer un diagnóstico?:

1. Resonancia nuclear magnética lumbar para descartar un proceso compresivo


radicular infeccioso o neoplásico.
2. Determinación de la presencia del antígeno HLA-B27 para confirmar el
diagnóstico de espondilitis anquilosante.
3. TC de articulaciones sacroiliacas para determinar la presencia de erosiones
yuxtaarticulares, lo cual establecería el diagnóstico de espondilitis anquilosante.
4. Radiografía anteroposterior de pelvis para valorar la presencia de
sacroileitis bilateral, que confirmaría el diagnóstico de espondiloartropatía.
SALE AS
5. Gammagrafía ósea con tecnecio-99 para valorar hipercaptación ósea patológica
vertebral, lo cual establecería el diagnóstico de espondiloartropatía.

3. Si una enferma con lupus eritematoso diseminado presenta un infiltrado


pulmonar, lo más probable es que se trate de:

1. Neumonitis intersticial con fibrosis.


2. Neumonitis aguda lúpica.
3. Edema pulmonar.
4. Hemorragias intraalveolares.
5. Infección pulmonar.

4. La supervivencia en un paciente con diagnóstico de esclerodermia está


especialmente determinada por la:

1. Gravedad de las lesiones cardíacas, pulmonares y renales.


2. Severidad del fenómeno de Raynaud.
3. Edad al diagnóstico.
4. Afectación gastrointestinal.
5. Severidad de la afectación cutánea.

5. En el tratamiento del síndrome antifosfolipídico es cierto que:

1. La presencia de anticuerpos antifosfolípido en una embarazada sin antecedente


de trombosis o abortos es una indicación para iniciar el tratamiento.
2. La anticoagulación se realiza en la actualidad con heparina de bajo peso
molecular, ya que con este procedimiento no se requieren controles.
3. La anticuagulación manteniendo un INR alto (?3) es el tratamiento de
elección en pacientes que ya han tenido trombosis.
4. La anticoagulación no es efectiva si no va acompañada de tratamiento
inmunosupresor.
5. En episodios trombóticos se deben emplear los corticoides además de la
aspirina.

6. Chico de trece años que refiere episodios de dolor abdominal, no filiados, y


artralgias erráticas en codos, rodillas. codos y muñecas. En las últimas 24 horas le
han aparecido manchas rojizas en muslos. Lo más destacado de la exploración
física es la existencia de púrpura palpable en nalgas y muslos. No presenta anemia,
las plaquetas son normales, la IgA está elevada y el aclaramiento de creatinina es
normal. Se objetiva proteinuria de 1 gr/24 horas y 50-70 hematíes por campo. en la
biopsia renal se observa proliferación mesangial y depósitos de IgA (+++) e IgG(+).
El diagnóstico más probable es

1. Vasculitis tipo PAN microscópica.


2. Lupus eritematoso sistémico.
3. Enfermedad de Wegener.
4. Síndrome de Goodpasture.
5. Síndrome de Schönlein-Henoch.

7. Ante un paciente de 54 años con condrocalcinosis, diabetes mellitus, alteración


de la función hepática y una artritis simétrica afectando 2ª y 3ª articulaciones
metacarpofalángicas, el primer diagnóstico que debe plantearse es:

1. Artritis reumatoide.
2. Artrosis.
3. Hemocromatosis.
4. Artritis psoriásica.
5. Síndrome de Reiter.

8. En un paciente que consulta por xerostomía y xeroftalmia. ¿Cuál de los


siguientes resultados de las exploraciones complementarias NO es concordante con
un diagnóstico de Síndrome de Sjögren primario?:

1. Presencia de anticuerpos anti-Ro y anti –La positivos en suero.


2. Test de Schirmer patológico que demuestra la presencia de hiposecreción
lagrimal.
3. Ulceraciones orales con apariencia de aftas.
4. Tinción corneal con fluorescencia que muestra queratoconjuntivitis punctata.
5. Biopsia de glándula salival menor con presencia de acúmulos focales
linfoplasmocitarios.

9. Paciente mujer de 48 años, sin antecedentes médicos de interés que presenta


cuadro de aproximadamente 3 meses de evolución consistente en: poliartritis de
manos, muñecas y rodillas, con rigidez matutina de 2 horas y factor reumatoide
elevado en la analítica que le realizó su médico de cabecera. Respecto a su
enfermedad, ¿cuál de las siguientes afirmaciones resulta INCORRECTA?:

1. Por la clínica que presenta la paciente padece una artritis reumatoide (AR).
2. Parece adecuado comenzar tratamiento con AINEs y/o corticoides a bajas dosis
para conseguir alivio sintomático.
3. Es importante comenzar lo antes posible tratamiento con fármacos
modificadores de la enfermedad (FME), inlcuso en combinación.
4. Antes de empezar el tratamiento con FME se debe esperar otros 3 meses a
comprobar la respuesta al tratamiento con AINEs y/o corticoides.
5. La presencia de erosiones radiológicas tempranas sería un signo de mal
pronóstico.
10. Varón de 28 años que presenta en su ojo izquierdo cuadro de vasculitis
retiniana severa y edema macular con una agudeza visual 0. El paciente refiere
historia de aftas bucales y genitales recidivantes. En base a los hallazgos clínicos y
a la historia del paciente ¿cuál considera, de los siguientes, que es el diagnóstico
más probable?:

1. Sarcoidosis.
2. Enfermedad de Behçet.
3. Esclerosis múltiple.
4. Tuberculosis.
5. Sífilis.

11. Indique cuál de los siguientes trastornos NO es causa de artropatía


neuropática:

1. Diabetes mellitus.
2. Tabes dorsal.
3. Siringomielia.
4. Hipotiroidismo.
5. Lepra.

12. ¿Cuál de los siguientes confirma el diagnóstico de lupus eritematoso sistémico?

1. Unos anticuerpos antinucleares positivos.


2. Una biopsia renal demostrando una glomerulonefritis proliferativa.
3. Unos niveles bajos de complemento sérico y aftosis oral.
4. La presencia de artritis no deformante, fotosensibilidad, leucopenia y
pleuritis.
5. Alopecia reciente.

13. Hombre de 35 años que presenta desde hace 1 mes fiebre en agujas, artralgias y
artritis y exantema asalmonado vespertino. La exploración física puso de
manifiesto artritis en pequeñas y grandes articulaciones. Se palpaba una
hepatomegalia de 2 cm. lisa y un polo de bazo. La velocidad de sedimentación
estaba aumentada: 140 mm en la 1ª hora. En el hemograma se evidenció una
leucocitosis de 23.000/mm3, con 80% de neutrófilos. La ferritinemia plasmática
era de 10.000 ng/ml (N = 15-90). Los hemocultivos fueron negativos. El
ecocardiograma transtorácico fue normal. La radiografía de tórax fue normal.
¿Cuál es su diagnóstico?:

1. Endocarditis aguda.
2. Lupus eritematoso sistémico.
3. Enfermedad de Still del adulto.
4. Sepsis por Staphylococcus Aureus.
5. Brucelosis.

16. Hombre de 25 años, que 10 días después de acudir a una despedida de soltero,
comienza con inflamación de rodilla derecha y de ambos tobillos, conjuntivitis
bilateral, aftas orales y erosiones superficiales no dolorosas en el glande. ¿Cuál
sería el diagnóstico más probable?:
1. Enfermedad de Still.
2. Infección gonocócica.
3. Infección por Staphilococcus Aureus.
4. Enfermedad de Reiter.
5. Sífilis.

17. Relación con las vasculitis sistémicas, señale cuál de las siguientes afirmaciones
es FALSA:

1. La poliarteritis nodosa (PAN) clásica, cursa con frecuencia con


glomerulonefritis y capilaritis pulmonar.
2. La presencia de anticuerpos anti-citoplasma de los neutrófilos con patrón
perinuclear es mucho más frecuente en la poliarteritis microscópica que en la
PAN clásica.
3. El tratamiento más eficaz para la granulomatisis de Wegener consiste en la
administración conjunta de ciclofosfamida y glucocorticoides.
4. La presencia de asma bronquial grave y eosinofilia periférica son características
de la granulomatosis alérgica de Churg-Strauss.
5. Es frecuente la asociación de manifestaciones propias de varios síndromes de
vasculitis en un mismo paciente.

18. Una mujer de 39 años diagnosticada previamente de prolapso mitral es


remitida por presentar hiperlaxitud articular generalizada. En el examen físico
objetivamos la presencia de pápulas blanquecino-amarillentas en zonas de flexión
de las axilas y el cuello. Un examen oftalmológico muestra estrias angioides en la
retina. ¿Cuál de las siguientes enfermedades hereditarias presenta la paciente?:

1. Pseudoxantoma elástico.
2. Síndrome de Ehlers-Danlos tipo III.
3. Síndrome de Marfan.
4. Ocronosis.
5. Osteogénesis imperfecta.

20. Un paciente de 28 años consulta por dolor lumbar de 9 meses de duración, de


predominio nocturno, que le despierta durante el sueño. Actualmente se ha
asociado dolor en la parte inferior del talón derecho, notando en los últimos días
hinchazón y dolor en rodilla derecha. Indique cuál de las siguientes actuaciones le
puede proporcionar datos que apoyen el diagnóstico que debe sospechar en primer
lugar:

1. Rx anteroposterior de pelvis.
2. Exploración física de caderas.
3. Análisis del líquido sinovial de la rodilla.
4. Rx lateral de calcáneos.
5. Determinación del antígeno HLA-B27.

1. ¿Cuál de los siguientes datos NO es considerado como un criterio de artritis


reumatoide de la Asociación Americana de Reumatología?:
1. Rigidez matutina de > 1 hora de duración.
2. Factor reumatoide positivo.
3. Presencia de HLA-D
4. Nódulos reumatoides.
5. Artritis simétrica.

2. Indique cuál de las siguientes afirmaciones en relación con los ataques de gota es
FALSA:

1. En los niños suelen afectarse las rodillas.


2. Se asocian a tratamiento con diuréticos.
3. Pueden ocurrir con ácido úrico normal.
4. Ceden con la administración de antiinflamatorios.
5. En ocasiones son poliarticulares.

3. Una mujer de 72 años acude al hospital con una historia de tres meses de
evolución de febrícula y sudoración nocturna, añadiéndose un mes más tarde dolor
inflamatorio severo en ambos hombros y en muslos, que le causan dificultad para
peinarse, vestirse y salir a la calle. A la exploración presenta movilidad limitada de
los miembros superiores y dificultad para levantarse de una silla debido al dolor.
No se evidencia artritis ni ninguna otra alteración en la exploración general y
neurológica. Se detecta anemia normocítica, trombocitosis leve y VSG de 90 mm a
la primera hora. La radiología de tórax es normal. ¿Cuál es la actitud diagnóstica
y terapéutica más correcta?

1. Iniciar de manera inmediata terapia con prednisona a dosis de 1 mg/kg.


2. Solicitar biopsia de la arteria temporal y tratar sólo si fuera positiva.
3. Iniciar tratamiento con dosis de 15-20 mg de prednisona y seguir respuesta
clínica.
4. Iniciar tratamiento con 3 bolos de 500 mg de metilprednisolona iv, y continuar
después con prednisona a dosis de 1 mg/kg por vía oral.
5. Solicitar biopsia muscular y tratar según resultado de la misma.

4. Varón de 68 años de edad sin antecedentes personales de interés que acude a


urgencias por un episodio de amaurosis fugax. En la anamnesis refiere dolor a
nivel de cinturas escapular y pelviana de 2 meses de evolución que le dificulte
peinarse y levantarse de una silla. Desde hace 15 días presenta cefalea holocraneal
que cede sólo parcialmente con analgésicos tipo paracetamol. ¿Cuál de las
siguientes asociaciones es la más relacionada con el cuadro anterior?:

1. VSG 10 mm/h. Hemoglobina 12 g/dL. Fosfatasa alcalina elevada. Biopsia de


piel con infiltrado inflamatorio.
2. VSG 100 mm/h. Hemoglobina 10 g/dL. Fosfatasa alcalina normal. Sistemático
de orina normal. Biopsia de arteria temporal con infiltrado con eosinófilos.
3. VSG 10 mm/h. Hemoglobina 10 g/dL. Fosfatasa alcalina elevada. Sistemático
de orina normal. Biopsia de vena auricular con infiltrado con neutrófilos.
4. VSG 100 mm/h. Hemoglobina 10 g/dL. Fosfatasa alcalina elevada.
Sistemático de orina normal. Biopsia de arteria temporal con infiltrado de
células mononucleares y rotura de la elástica interna.
5. VSG 100 mm/h. Hemoglobina 10 g/dL. Fosfatasa alcalina elevada. Sistemático
de orina normal. Biopsia renal con glomerulonefritis membrano proliferativa.

8. Paciente de 70 años con antecedentes de episodios transitorios de dolor y


tumefacción en rodilla derecha que, en su segundo día de postoperatorio por
apendicitis aguda no complicada, desarrolla dolor y signos inflamatorios locales en
ambas rodillas y tobillo derecho con temperatura elevada. Entre los siguientes,
¿cuál es el diagnóstico más probable?:

1. Artritis reumatoide de comienzo.


2. Condrocalcinosis (pseudogota).
3. Artritis séptica.
4. Artritis de Lyme.
5. Agudización de artrosis.

10. Un joven de 24 años refiere dolor en el glúteo derecho, irradiado por la cara
posterior del muslo hasta la rodilla, de 10 días de evolución. Su médico le prescribe
reposo, calor local y analgésicos. A los 15 días vuelve porque se le irradia el dolor a
la otra nalga y, además, tiene dolor en un hombro y en la región dorsal baja
acompañado de cierta rigidez matutina. El hemograma es normal, la radiografía
de columna no revela alteraciones y en la de articulaciones sacroilíacas se aprecia
un ligero borramiento del borde en el lado derecho. ¿Cuál de los siguientes
diagnósticos es más probable?:

1. Espondilitis infecciosa.
2. Artritis reumatoide.
3. Hiperostosis esquelética idiopática difusa.
4. Espondiloartritis anquilopoyética.
5. Artritis reactiva.

2. ¿Cuál de las siguientes es la forma de artritis psoriásica más frecuente?:

1. La oligoarticular asimétrica.
2. La poliarticular simétrica.
3. La mutilante.
4. La axial.
5. La afectación de las articulaciones interfalángicas distales.

6. De las siguientes opciones, ¿cuál de las siguientes NO se asocia a la enfermedad


de Raynaud?:

1. Esclerodermia.
2. Tromboangeítis obliterante.
3. Siringomielia.
4. Betabloqueantes.
5. Polimialgia reumática.
8. Hombre de 42 años que, un año antes, presentó lesiones eritematosas,
descamativas y pruriginosas en superficie extensora de ambos codos. Seis meses
después notó hinchazón y dolor en interfalángica del primer dedo de mano
derecha y en interfalángicas (IF) de 2º, 3º y 5º dedos de pie derecho. Ausencia de
antecedentes de pleuritis, diarrea, lumbalgia, ciática, iritis y uretritis. El examen
físico muestra las lesiones cutáneas referidas en codos y en el lado derecho del
cuero cabelludo. Junto a los fenómenos inflamatorios articulares descritos se
comprueba deformidad en "salchicha" (dactilitis) en IF de 3º y 4º dedos de pie
izquierdo. Las uñas presentan lesiones punteadas, onicólisis y surcos horizontales
palpables. Datos de laboratorio: ANA, factor reumatoide y serología de lúes
negativos, hiperuricemia moderada y nivel elevado de IgA, PCR++, VSG 30 mm
en 1ª hora. Rx de manos: pequeñas erosiones periarticulares en IF distales de 2º y
3º dedos de mano derecha y sugerencia de osteólisis de falanges distales. Con más
probabilidad el paciente tendrá:

1. Artritis reumatoide.
2. Artritis psoriásica.
3. Síndrome amiloide.
4. Artropatía amiloide.
5. Artritis asociada a enfermedad inflamatoria intestinal.

9. Un hombre de 58 años diagnosticado de artritis gotosa tofácea y litiasis ureteral


por cálculos de ácido úrico no tratado previamente, consulta por monoartritis
aguda de rodilla. El análisis del líquido sinovial obtenido por artrocentesis muestra
abundantes microcristales de urato intraleucocitarios. El ácido úrico sérico es de
9,4 mg/dl (valor normal < 7 mg/dl) y la creatinina está en rango normal. ¿Cuál de
las siguientes estrategias terapéuticas le parece más adecuada?:

1. Antiinflamatorios no esteroideos o colchicina durante el episodio de artritis


aguda, profilaxis de nuevos episodios de artritis con colchicina 1 mg diario
oral, y dieta con restricción de purinas y alopurinol 300 mg diarios como
tratamiento hipouricemiante.
2. Colchicina intravenosa como tratamiento del episodio de artritis aguda, y dieta
con restricción de purinas y alopurinol 300 mg como tratamiento
hipouricemiante.
3. Colchicina oral 3 mg diarios hasta la resolución del episodio agudo de artritis y
dieta con restricción de purinas y fármacos uricosúricos como benzobromarona
100 mg diarios como tratamiento hipouricemiante.
4. Antiinflamatorios no esteroideos durante el episodio de artritis aguda, y
colchicina oral 1 mg diario de forma ininterrumpida como profilaxis de nuevos
episodios de artritis.
5. Metilprednisolona intraarticular como tratamiento de la artritis aguda, y
alopurinol 300 mg diarios como tratamiento hipouricemiante.

10. Una mujer de 45 años presenta desde hace 6 meses dolor de características
inflamatorias en articulaciones metacarpofalángicas proximales de ambas manos,
en muñecas y en una rodilla, sin otra sintomatología. La exploración general es
normal, excepto por discreto aumento del tamaño y dolor a la palpación de
muñecas, de tres articulaciones metacarpofalángicas y en cuatro interfalángicas
proximales. La hematología y la bioquímica son normales, excepto una velocidad
de sedimentación globular de 40 mm a la primera hora. El estudio inmunológico
muestra unos Anticuerpos Antinucleares negativos y un Factor Reumatoide
positivo. La radiología muestra una erosión en muñeca derecha. ¿Cuál es la
actitud más correcta?:

1. Iniciar terapia inmediata con analgésicos y seguir evolución al mes.


2. Iniciar terapia con antiinflamatorios no esteroideos y repetir estudio
inmunológico y radiológico al año, para decidir o no fármacos modificadores de
la enfermedad.
3. Realizar gammagrafía ósea con Tc, para confirmar sinovitis y tratar según
resultados.
4. Iniciar terapia con antiinflamatorios no esteroideos e indicar inicio de
Metotrexate por vía oral, en dosis única semanal.
5. Iniciar terapia con corticosteroides por vía oral, a dosis de 20-30 mg al día.

11. En el caso de una paciente de 45 años de edad con historia de 6 meses de


evolución, consistente en artritis simétrica que incluye muñecas, rigidez matutina
de 2 horas y una radiología simple con erosiones en carpo. ¿Cuál de los siguientes
hallazgos inmunológicos NO se correlaciona con el cuadro clínico descrito?:

1. Aumento en el compartimento sinovial de citoquinas de origen macrofágico.


2. Presencia en suero de inmunoglobulinas con efecto anti-inmunoglobulinas G.
3. Presencia en compartimento sinovial de auto-anticuerpos con reactividad contra
la inmunoglobulina G.
4. Aumento en el compartimento sinovial de linfocitos T con marcadores de
activación en la superficie.
5. Presencia en la superficie de sus linfocitos B de HLA-DR6.

12. El tratamiento de elección de la granulomatosis de Wegener es:

1. D-penicilamina.
2. Esteroides a dosis altas.
3. Ciclofosfamida.
4. Azatioprina.
5. Metotrexate.

13. Una mujer de 60 años acude por la aparición aguda de inflamación y dolor en
su rodilla derecha. No refería antecedente traumático o una historia previa de
artritis. La exploración física demostró la presencia de derrame articular y
aumento de temperatura en su rodilla derecha. El factor reumatoide fue negativo y
el ácido úrico sérico era de 3,2 mg/dl. El diagnóstico más probable se establecería
por:

1. La respuesta terapéutica a esteroides intaarticulares.


2. Niveles elevados de calcio sérico.
3. Un recuento de células blancas en líquido sinovial de 500/mm3 con baja
viscosidad.
4. La presencia de una fina línea de calcificación en la radiografía de la rodilla
afectada.
5. Una respuesta excelente a un curso corto de tratamiento con indometacina.
18. Durante un crucero por el Mediterráneo se produjo entre los pasajeros un
brote de diarrea aguda. Los coprocultivos demostraron la presencia de Shigella
Flexneri en la mayoría de los pacientes. El médico del barco instauró tratamiento
con medidas de soporte sin antibióticos con lo que todos los pacientes se
recuperaron en menos de una semana. Sin embargo, en los últimos días del viaje,
la mitad de los pasajeros afectados por la infección tuvieron que ser evacuados a su
lugar de origen porque desarrollaron diferentes formas de afectación articular
cuyo patrón más habitual fue oligoartritis de miembros inferiores. Respecto al
episodio descrito ¿Cuál de las siguientes afirmaciones es correcta?:

1. El cuadro articular descrito se corresponde con una artritis reactiva y sólo


pudo presentarse en los pacientes con antígeno HLA B27 positivo.
2. El diagnóstico más probable del cuadro descrito es el de artritis reactiva,
también denominado síndrome de Reiter, en especial cuando se acompaña de
manifestaciones mucocutáneas y oculares. En este proceso, el diagnóstico se
establece por la positividad del antígeno HLA B27.
3. El diagnóstico más probable es de artritis reactiva o síndrome de Reiter.
Los pacientes con antígeno HLA B27 positivo que la desarrollan tienen un
peor pronóstico que los que no lo tienen.
4. La utilización de antibióticos de amplio espectro en el brote de gastroenteritis
habría evitado el desarrollo de artritis reactiva en todos los casos.
5. La Shigella Flexneri no se encuentra entre los gérmenes causales de la artritis
reactiva por lo que el cuadro descrito parece corresponder a un episodio de
artritis de probable origen viral de nueva aparición.

20. En el tratamiento del síndrome antifosfolipídico es cierto que:

1. La presencia de anticuerpos antifosfolípido en una embarazada sin antecedente


de trombosis o abortos es una indicación para iniciar el tratamiento.
2. La anticoagulación se realiza en la actualidad con heparina de bajo peso
molecular, ya que con este procedimiento no se requieren controles.
3. La anticoagulación manteniendo un INR alto (-3) es el tratamiento de
elección en pacientes que ya han tenido trombosis.
4. La anticoagulación no es efectiva si no va acompañada de tratamiento
inmunosupresor.
5. En episodios trombóticos se deben emplear los corticoides además de la
aspirina.

22. En una esclerosis sistémica progresiva (esclerodermia) el peor pronóstico se


asocia con:

1. Una extensa calcinosis cutánea.


2. La presencia de disfagia.
3. El número de articulaciones inflamadas.
4. La presencia de insuficiencia renal.
5. Fenómeno de Raynaud severo con úlceras necróticas en dedos.

26. Paciente de 72 años, noruega, residente en la Costa del Sol, acude a Urgencias
por un episodio brusco y autolimitado de pérdida de visión 2 horas antes. Desde la
semana previa aquejaba cefalea. En el último mes, había perdido peso en el curso
de un proceso caracterizado por febrícula, artralgias, astenia, anorexia,
cervicobraquialgia bilateral y mialgias en región glútea y muslos. En el examen
físico destacaban una temperatura de 37,8ºC y palidez cutaneomucosa: el resto de
la exploración, incluyendo examen oftalmológico y neurológico, fue normal. Se
objetivaron: Hb 9,8 gr/dl, valor hematocrito 29%, VCM 87, leucos 9800/mm3,
plaquetas 470.000/mm3, y VSG 72mm a la 1ª hora. ¿Cuál de las siguientes
actitudes es la correcta?:

1. Iniciar tratamiento con 60mg/días de prednisona y proseguir estudio.


2. Transfundir dos unidades de concentrado de hematíes.
3. Iniciar tratamiento con bolos IV de ciclosfosftamida.
4. Ingresar, iniciar antibioterapia empírica, tras extracción de hemocultivos y
proseguir estudio.
5. Realizar una tomografía computerizada de cráneo urgente y tratar en caso de
hallazgos patológicos.

28. En el caso de una paciente de 45 años de edad con historia de 6 meses de


evolución, consistente en artritis simétrica que incluye muñecas, rigidez matutina
de 2 horas y una radiología simple con erosiones en carpo. ¿Cuál de los siguientes
hallazgos inmunológicos NO se correlaciona con el cuadro clínico descrito?:

1. Aumento en el compartimento sinovial de citoquinas de origen macrofágico.


2. Presencia en suero de inmonoglobulinas con efecto anti-inmunoglobulina G.
3. Presencia en compartimento sinovial de autoanticuerpos con reactividad contra
la inmunoglubulina G.
4. Aumento en el comportamiento sinovial de linfocitos T con marcadores de
activación en la superficie.
5. Presencia en la superficie de sus linfocitos B de HLA-DR6.

29. Hombre de 30 años, que presenta un cuadro clínico de 10 meses de evolución


consistente en dolor lumbar continuo, que le despierta por la noche, y que se
acompaña de rigidez matutina de tres horas de duración. ¿Cuál sería el
tratamiento de primera elección?:

1. Diazepan oral.
2. Dexametasona intramuscular.
3. Indometacina oral.
4. Metamizol intramuscular.
5. Sales de oro intramuscular.

31. En relación a la artritis reumatoide, ¿cuál de las siguientes afirmaciones es


FALSA?:

1. El haplotipo HLA DR4 es más frecuente en la artritis reumatoide que en la


población general.
2. La inflamación de la membrana sinovial está mediada por el depósito de
inmunocomplejos circulantes.
3. En el infiltrado inflamatorio de la membrana sinovial se encuentran
linfocitos, macrófagos y fibroblastos activados.
4. La interleuquina-1 (IL-1), interleuquina-6 (IL-6) y el factor de necrosis tumoral-
alfa (TNF?) son citoquinas proinflamatorias abundantes en el medio sinovial.
5. El factor reumatoide es una inmunoglobulina con reactividad para la porción Fc
de la inmunoglobulina G (IgG).

32. ¿Cuál de estas afirmaciones es FALSA con respecto a las artritis inducidas por
microcristales?:

1. Pueden ser producidas por cualquiera de los siguientes cristales: urato


monosódico, pirofosfato cálcico, hidroxiapatita cálcica y oxalato cálcico.
2. El depósito de cristales puede producir cuadros clínicos similares a la artritis
reumatoide o la espondilitis anquilosante.
3. Los cuadros clínicos producidos son específicos para cada uno de los tipos
de cristales depositados.
4. Para hacer el diagnóstico es imprescindible el estudio del líquido sinovial con
microscopio de luz polarizada para identificar el tipo de cristales.
5. El líquido sinovial suele ser de tipo inflamatorio, aunque en ocasiones existen
cristales en ausencia de inflamación.

34. ¿Cuál de los siguientes parámetros es de MENOR utilidad como monitor de la


actividad de la artritis reumatoide?:

1. Velocidad de sedimentación.
2. Factor reumatoide.
3. Proteína C reactiva.
4. Otros reactantes de fase aguda.
5. Hemoglobina.

37. ¿Cuál de las siguientes determinaciones de anticuerpos es, de resultar positiva,


más específica de lupus eritematoso sistémico?:

1. Antinucleares.
2. Anti-RNP.
3. Anti-SSA.
4. Anti-SSB.
5. Anti-Sm.

38. NO es una característica de la gota:

1. La artritis afecta predominantemente a las articulaciones de las extremidades


inferiores.
2. En el líquido articular se observa un aumento de los leucocitos
polimorfonucleares.
3. Entre las crisis de podagra el enfermo está completamente asintomático.
4. La manera más habitual de presentación es en forma de monoartritis.
5. Los cristales dan birrefringencia positiva cuando se examinan con
microscopio de luz polarizada.

40. Hombre de 34 años, sin antecedentes de interés. Presenta desde hace una
semana dolor en zona lumbar baja, que no le ha impedido realizar su actividad
laboral. En las últimas 24 horas el dolor ha aumentado hasta convertirse en severo
e incapacitante, dificultándole tareas como deambular o levantarse de la cama. El
paciente acude a Urgencias, donde se objetiva una exploración física general
estrictamente normal, una exploración neurológica dificultada por el dolor, sin
alteraciones en la sensibilidad, con Lassègue a 60º, Bragard negativo y con reflejos
osteotendinosos conservados y simétricos en las cuatro extremidades. ¿Qué actitud
es la más indicada en el estudio y tratamiento de este paciente?:

1. Se realizan Rx simples de columna lumbar, que son normales, se


diagnostica de lumbalgia aguda, se prescriben analgésicos nivel II,
relajantes musculares, reposo en cama durante dos semanas y se valora
evolución final del período de reposo.
2. Se diagnostica de lumbalgia aguda, no se realiza ningún estudio
complementario, se informa al paciente y su familia sobre el cuadro que
padece, se prescriben analgésicos nivel II, relajantes musculares,
movilización precoz y se valora evolución en una semana.
3. Se diagnostica de hernia discal y se ingresa para cirugía.
4. Se realiza TAC urgente de columna lumbar, que es normal, se ingresa al
paciente para completar el estudio con Resonancia Magnética Nuclear e
Isótopos, con sospecha de neoplasia o infección.
5. Se realizan TAC y Resonancia Magnética Nuclear urgentes, que son
estrictamente normales, se consulta a Psiquiatría para descartar componente
funcional.

7. ¿Cuál de las siguientes afirmaciones en relación con la enfermedad de Wegener


es FALSA?:

1. Es una vasculitis sistémica que afecta sobre todo a vasos de mediano


calibre.
2. En ausencia de tratamiento cursa de manera progresiva y con frecuencia mortal.
3. Presenta con frecuencia afectación renal, que es histológicamente indistinguible
de la glomerulonefritis necrótica con semilunas.
4. En presencia de afectación pulmonar y/o renal, el uso de ciclofosfamida vía oral
es casi siempre imprescindible para obtener un buen control de la enfermedad.
5. Se asocia a la presencia de anticuerpos anticitoplasma de neutrófilo (ANCAS)
con patrón de inmunofluorescencia de tipo citoplásmico.

10. Señale en cuál de las siguientes manifestaciones clínicas del lupus eritematoso
sistémico NO está indicado el tratamiento con corticosteroides:

1. Anemia hemolítica.
2. Glomerulonefritis.
3. Artritis aislada.
4. Miositis.
5. Trombopenia importante.

12. En relación al Lupus Eritematoso Sistémico, indique cuál de las siguientes


afirmaciones es FALSA:
1. Es una enfermedad de claro predominio femenino.
2. La afectación renal es frecuente y es uno de los indicadores de agresividad de la
enfermedad.
3. La biopsia renal es de particular utilidad dado que el tipo de afectación
renal no varía con la evolución de la enfermedad.
4. Los pacientes pueden presentar lesiones cutáneas en relación a la exposición
solar.
5. Los autoanticuerpos más característicos son los antinucleares y los anti-ADN.

20. A un varón de 65 años asintomático, se le encuentra en un estudio de control


general una cifra de fosfatasa alcalina dos veces superior a lo normal, con pruebas
hepáticas normales. La gammagrafía ósea muestra captación en la mitad superior
de la hemipelvis derecha, y una radiograía realizada en esta zona pone de
manifiesto un patrón de corticales aumentadas de grosor y patrón trabecular
grosero. ¿Qué tratamiento es el más apropiado?:

1. Alendronato, 40 mg diarios.
2. Naproxeno, 500 mg dos veces al día.
3. Calcio (400 mg.) y vitamina D (400 UI).
4. Calciatonina nsal, 200 UI/día.
5. No es preciso tratamiento.

30. Hombre de 30 años, que presenta un cuadro clínico de 10 meses de evolución


consistente en dolor lumbar continuo, que le despierta por la noche, y que se
acompaña de rigidez matutina de tres horas de duración. ¿Cuál sería el
tratamiento de primera elección?:

1. Diazepan oral.
2. Dexametasona intramuscular.
3. Indometacina oral.
4. Metamizol intramuscular.
5. Sales de oro intramuscular.

36. El tratamiento de elección de la granulomatosis de Wegener es:

1. D-penicilamina.
2. Esteroides a dosis altas.
3. Ciclofosfamida.
4. Azatioprina.
5. Metotrexate.

37. Respecto al tratamiento del Lupus eritematoso sistémico señale, entre las
siguientes, la respuesta INCORRECTA:

1. Se deben utilizar glucocorticoides a dosis altas y fármacos citotóxicos en los


casos de afectación orgánica severa potencialmente reversible.
2. Los fármacos antipalúdicos son eficaces para el control de formas leves
moderadas de la enfermedad, siendo recomendable vigilar periódicamente la
posible toxicidad retiniana.
3. En períodos de inactividad de la enfermedad es posible prescindir del
tratamiento, aunque las remisiones completas son raras.
4. En caso de insuficiencia renal terminal deben recibir tratamiento con
diálisis, estando contraindicado el transplante renal de cadáver.
5. Las complicaciones infecciosas son un riesgo importante en los pacientes que
reciben tratamiento inmunosupresor.

41. Una mujer de 60 años acude por la aparición aguda de inflamación y dolor en
su rodilla derecha. No refería antecedente traumático o una historia previa de
artritis. La exploración física demostró la presencia de derrame articular y
aumento de temperatura en su rodilla derecha. El factor reumatoide fue negativo y
el ácido úrico sérico era de 3,2 mg/dl. El diagnóstico más probable se establecería
por:

1. La respuesta terapéutica a esteroides intraarticulares.


2. Niveles elevados de calcio sérico.
3. Un recuento de células blancas en líquido sinovial de 500/mm3 con baja
viscosidad.
4. La presencia de una fina línea de calcificación en la radiografía de la rodilla
afectada.
5. Una respuesta excelente a un curso corto de tratamiento con indometacina.

47. Un joven de 21 años viene a la consulta porque tiene desde hace 4 ó 5 meses un
dolor constante en la región lumbosacra, que es peor en las primeras horas del día
y mejora con la actividad. Recuerda que 2 años antes tuvo un episodio de
inflamación en rodilla que se resolvió completamente. También se queja de dolor
en la caja torácica con los movimientos respiratorios. El examen físico demuestra
la ausencia de la movilidad de la columna lumbar. ¿Cuál de los siguientes test nos
ayudaría a confirmar el diagnóstico del paciente?:

1. El TAC de columna lumbar.


2. Las radiografías de las articulaciones sacroilíacas.
3. Los niveles de antiestreptolisina 0 (ASLO).
4. La radiografía de tórax.
5. Los niveles de la proteína C reactiva (PCR) y velocidad de sedimentación.

50. De las siguientes opciones, ¿cuál de las siguientes NO se asocia a la enfermedad


de Raynaud?:

1. Esclerodermia.
2. Tromboangeítis obliterante.
3. Siringomielia.
4. Betabloqueantes.
5. Polimialgia reumática.
31. En un paciente que consulta por xerostomía y xeroftalmia. ¿Cuál de los
siguientes resultados de las exploraciones complementarias NO es concordante con
un diagnóstico de Síndrome de Sjögren primario?:

1. Presencia de anticuerpos anti-Ro y anti –La positivos en suero.


2. Test de Schirmer patológico que demuestra la presencia de hiposecreción
lagrimal.
3. Ulceraciones orales con apariencia de aftas.
4. Tinción corneal con fluorescencia que muestra queratoconjuntivitis punctata.
5. Biopsia de glándula salival menor con presencia de acúmulos focales
linfoplasmocitarios.

27. ¿Cuál de los siguientes confirma el diagnóstico de lupus eritematoso sistémico?

1. Unos anticuerpos antinucleares positivos.


2. Una biopsia renal demostrando una glomerulonefritis proliferativa.
3. Unos niveles bajos de complemento sérico y aftosis oral.
4. La presencia de artritis no deformante, fotosensibilidad, leucopenia y
pleuritis.
5. Alopecia reciente.

22. En el lupus cutáneo subagudo es caracteríscica de la presencia de anticuerpos:

1. Anti-Sm.
2. Anti-centrómero.
3. Anti-Ro.
4. Anti Jo1.
5. Anti-histona.

14. Mujer de 53 años que padece, desde hace 20, un cuadro no diagnosticado
caracterizado por episodios de dolor en manos, con cambios de coloración y
aparición de lesiones ulceradas recidivantes en pulpejo de los dedos. Dos meses
antes su TA era de 110/ 80 mmHg. Consulta por oliguria, con orina de aspecto
normal en los días previos y TA de 190/130 mmHg. Niega ingesta previa de
fármacos y se objetiva, junto a secuelas de las lesiones referidas en dedos,
hinchazón de las manos y cierto endurecimiento cutáneo en brazos y antebrazos.
Con mayor probabilidad la paciente tendrá:

1. Crioglobulinemia mixta esencial.


2. Granulomatosis de Wegener.
3. Crisis renal de la esclerodermia.
4. Nefroesclerosis arterial maligna.
5. Nefropatía por hipersensibilidad.

1.Nombre las 2 vasculitis de cel gigantes: a. Arteritis temporal y b. A. de Takayasu

2.Señale las enzimas q no son de utilidad en el Dx de polimiositis: creatinofosfocinasa

3.La crisis de gota se debe: a un aumento del pool de uratos

4.Una pcte la cual se encuentra es estudio por artralgia, dentro de los lab le sale positivo
el anticuerpo Anti-histona; su preferencia Dx sería: lupus inducido por drogas

5.Un varón de 22 años se queja de dolor bajo de espalda y rigidez, q empeora al


levantarse y mejoran con el ejercicio. En la exploración se le encuentra movilidad
limitada de las articulaciones sacroiliadas y de columna lumbar. Una prueba en suero
para antígeno de histocompatibilidad HLA-B27 es positiva. Cuál es la manifestación
extraesquelética más frecuente de esta enfermedad?: Iritis aguda

6.Es sd hipercoagulable, excepto: cirrosis hepática

7.Los corticoides tienen una serie de acciones sobre el hueso, menos una: Estimulan la
actividad de los osteoclastos

8.Citar que sustancia no interviene en el metabolismo óseo: Albúmina

9.Citar que afección no es una espondiloartritis seronegativa: Osteitis condensante del


iliaco

10. En el Sd de CREST sobra una de las siguientes: Irido escleritis

11. modo de transmisión de TBC (5 rayas): orificio en orejas, circuncición, tatuaje,


respiración boca a boca, inyecciones

12. cuales son los campos q se obsevan en Baciloscopia...y son 4 rayas

13. efectos adversos de los medicamentos: Rifampicina, Eritromicina.....o un pareo de


efectos adversos

14. Uso del Ceftriazone: (3 rayas): Tx por gonococo, Ins Renal Aguda y barro biliar

15. Mencione 2 cefalosporina de 2° generación: cefaclor y cefoxitina


TOXICOLOGIA
Del veneno de coral todo lo siguiente es cierto excepto:
A. Es de acción exclusivamente neurotóxica.
B. Ataca el sistema nervioso en la placa mioneural
C. produce una parálisis muscular progresiva
D. la afección de los trayectos nerviosos es en sentido centrífugo.
E. Se fija a los tejidos en un período de 4 horas

En el manejo de los accidentes ofídicos lo más importante es:


A. Determinar que especie de serpiente está involucrada en el accidente
B. Trasladar a la víctima lo antes posible a un centro médico
C. Administrar suero antiofíodico específico
D. Determinar el grado de envenenamiento
E. Todas las anteriores

De las escorpionotoxinas solo es cierto


A. Actúan sobre los canales calcio de varios tejidos excitables.
B. Actúan sobre el sistema nervioso simpático produciendo liberación de acetilcolina lo que
determina el cuadro sistémico.
C. La acción de las toxinas escorpiónicas produce hipertensión e
D. hiperglicemia.

Los órganos blancos más afectados son:


A. corazón
B. páncreas y riñón.
C. Ninguna de las anteriores

Todo lo siguiente es cierto, excepto:


A. La coral pertenece a la familia Elapidae
B. El efecto biológico del veneno crotálico es exclusivamente neurotóxico
C. La fracción tóxica del veneno de serpiente son proteínas y péptido de 6000-30,000 moles con
receptores específicos
D. El veneno de serpiente se produce en las glándulas homologas a las parótidas
E. El suero antiofídico se obtiene inmunización equina

Las diferencias entre el sao (suero antiofidico) liofilizado y el líquido es


A. El sao liquido dura mas que el liofilizado
B. el sao liofilizado no requiere refrigeración
C. el sao liquido no requiere refrigeración,pero debe ser
D. guardado en lugares frescos
E. ninguna de las anteriores
F. todas las anteriores.

Un paciente de 30 años es mordido en el pie por una serpiente venenosa. Posteriormente, desarrolla
edema de la pierna hasta la mitad del muslo, con signos inflamatorios y sangrado por encías y hematuria.
Podemos decir según la clasificación clínica que nos encontramos ante un:
A. Síndrome Crotalico
B. Síndrome Elapidico
C. Síndrome Laquesico
D. Síndrome Bothropico

La ruta más importante de intoxicación por tóxicos es:


A. Piel
B. Respiratorio
C. Ojos
D. Digestivo
E. Ninguna de las anteriores
El producto que puede ayudar a la intoxicación por paraquat es el siguiente:
A. Ditionita
B. Oxigeno
C. N-Acetil-Cisteina
D. Antinflamatorios
E. Acetominofen

Cuál de los siguientes síntomas y signos forma parte del Síndrome muscarínico en intoxicación aguda de
órganos fosforados excepto
A. Dolor abdominal
B. Miosis
C. Pesadez, de piernas
D. Rinorrea
E. Diarrea
(SÍNTOMAS DE SÍNDROME MUSCARÍNICO:Miosis, visión borrosa, rinorrea, disnea, tos, diarrea,
vómitos, incontinencia, bradicardia, hipotención)

En el manejo terapéutico del paciente intoxicado por órganos fosforado es mejor guiarse por el siguiente
patrón de laboratorio o gabinete.
A. Niveles de Pseudocolinesterasas diario.
B. Rx de Tórax diario.
C. Hemograma diario
D. Electrolitos y Pruebas funcionales Hepáticas diarias.
E. Gases arteriales diarios

Paciente intoxicado con medicamento para los ojos presenta: rubor, mucosas secas, intranquilo y
retención urinaria su tx incluiría:
A. Fisostigmina
B. Atropina
C. Propranolol
D. Flumazenil

Paciente rociado con un tóxico llamado Malatión podrá presentar los siguientes síntomas muscarínicos
excepto:
A. lagrimeo
B. sialorrea
C. broncoespasmo
D. rubicundez

Paciente en coma porque se tomó 30 pastillas de alprazolam, enojado con su novia. Usted utilizaría el siguiente
antídoto:
A. Atropina
B. Flumazenil
C. Protamina
D. Fisostigmina
Cuál de las siguientes no es una medida para evitar absorción del tóxico:
A. Descontaminación oral
B. Digestiva
C. Diuresis forzada
D. Cutánea

La causa más frecuente de enfisema subcutáneo en el estadio II y III de la intoxicación por Paraquat es:
A. Perforación pulmonar por el daño directo del Paraquat al Pulmón.
B. Perforación de una bula Pulmonar.
C. La erosión directa en la piel que es severa.
D. Perforación Esofágica.

Quien dota material como tierra Fuller al cuarto de Urgencia para el manejo del Paraquat:
A. Ministerio de Salud
B. Ministerio de Desarrollo Agropecuario
C. La OMS.
D. La casa farmacéutica que produce el Paraquat
El antídoto principal para las manifestaciones Nicotinitas es:
A. Atropina.
B. Protopan.
C. Toxogonin.
D. Ninguna de las anteriores.

Tenemos un paciente en la sala de Medicina y lo llama la enfermera porque la infusión de Atropina se


terminó y el paciente intoxicado por Carbamatos está asintomático cual sería el paso a seguir:
A. No continuar con la atropina.
B. Continuarle a dosis bajas esperando los niveles de laboratorio de colinesterasas.
C. No continuar la atropina, pero administrarla en bolo de atropina si presenta manifestaciones de
Broncorrea.
D. Administrarle Protopan y suspender la Atropina.

El tratamiento de la neurolues es con:


A. Pen. Sódica
B. Cloramfenicol
C. Ciprofloxacina
D. Penicilina benzatínica

Cuál sería el antídoto para los siguientes tóxicos:


A. Oxígeno y N-acetil cisteina para el paraquat.
B. Oxígeno y Diazepan para el diquat.
C. Atropina y Oxima para intoxicación por órganos fosforados.
D. Tratamiento de sostén para órganos clorados.

La prueba de Ditionita tiene valor pronóstico en:


A. En las primeras 24 horas.
B. En las primeras 36 horas.
C. 36 horas en adelante
D. Después de las 48 horas.

Complicación tardía del Paraquat a nivel de los ojos es:


E. Desprendimiento de Retina
F. Conjuntivitis Sicca
G. Keratitis y Uveítis
H. Opacidad Cornea

Niveles de Paraquat en sangre que indican alta probabilidad de muerte:


A) 0.9mg/L a las 24 horas
B) 1.0mg/L a las 6 horas
C) 1.5mg/L a las 4 horas
D) 2.0mg/L a las 4 horas

Usted es llamado al cuarto de urgencia por un paciente que está intoxicado por paraquat, para que le
realice lavado gástrico. Usted lo realiza si el paciente:
A. Tiene 6 horas de haber ingerido la sustancia
B. Tiene 7 horas de haber ingerido la sustancia
C. Tiene 4 horas de haber ingerido la sustancia
D. No sabe el tiempo de haber ingerido, lo importante es salvarlo

En relación con el uso de atropina en la intoxicación por organofosforados, cuál de las siguientes
afirmaciones es falsa:
A. Debe administrarse en forma precoz
B. Contrarresta la hipersecreción bronquial y las bradiarritmias.
C. Actúa sobre los síntomas muscarínicos y nicotínicos
D. El delirio atropínico es consecuencia de su excesiva administración.

Todos estos síntomas son propios de la intoxicación con inhibidores de colinesterasa excepto:
A. Bradicardia
B. Fasciculaciones
C. Miosis
D. Sequedad de piel y mucosas

En el caso de la investigación de los 2 pacientes lo mandaron a casa porque se pensaba que era una
gastroenteritis sin saber que era una intoxicación por Carbamatos regresan por convulsiones y paro
cardiorrespiratorio. Usted pensaría que son manifestaciones típicas de:
A. Síndrome Nicotínico
B. Síndrome muscarínico a nivel pulmonar y cardiaco. c..Síndrome intermedio
C. Síndrome del Sistema Nervioso Central.
D. Síndrome de la medula espinal

Conociendo usted las propiedades bioquímicas del Paraquat. ¿Cuál será el objetivo de utilizar como
tratamiento inicial tierra o tierra fuller?
A. Bloquea la absorción intestinal y se elimina en orina
B. Elimina los metabolitos del mismo por orina.
C. Se inactiva en contacto con minerales arcillosos
D. Ayuda a eliminar el paraquat vía intestinal por heces al unirse a ciertas proteínas intestinales

Usted, aunque no lo crea está esperando los niveles de Paraquat realizado por cromatografía para
dializar. Cuál sería el nivel que se puede beneficiar este paciente con dicho procedimiento:
A) 2.5 mg/Lt a las 4 horas
B) mg/Lt a las 6 horas
C) 1.5 mg/Lt a las 6 horas
D) 1.83mg/Lt a las 4 horas

El manejo apropiado de un paciente con intoxicación por un organofosforado incluye las siguientes
medidas, Excepto:
A) Atropina
B) Nitrito de sodio
C) Cloruro de pralidoxima
D) Descontaminación.
(TRATAMIENTO: Atropina, Oxima, Pralidoxima y sulfato de magnesio)

Intervalo de tiempo desde la intoxicación con un organofosforado en el cual es más beneficiosa y efectiva
la administración de una oxima:
A. Antes de transcurridas las 24 a 48 horas
B. Luego de transcurridas las 48 horas.
C. Entre las 72 a 96 horas
D. Su administración es igual de efectiva en cualquier momento.

Mientras usted investiga han pasado 3 horas, ya que usted no solo está viendo estos pacientes, uno de
los pacientes está somnoliento y convulsiona, y el otro comienza a presentar síndrome confucional, ya
definitivamente comienza a sospechar en otro tipo de intoxicación:
A. Intoxicación por Diquat por las convulsiones y dolor abdominal.
B. Intoxicación por órganos fosforados porque al principio tenían síntomas nicotínicos.
C. Intoxicación por Paraquat por el dolor abdominal secundario a Pancreatitis.
D. Intoxicación por órganos fosforados porque al principio tenían síntomas muscarínicos.
Pte mordido por una culebra en el pie con aparición de equimosis , edema hasta 1/3 medio de la pierna , con TPT
y TP alterados ligeramente la intoxicación ofídica se clasifica como:
A. Grado 0
B. leve
C. moderada
D. d) Severa

El tx con suero antiofídico del paciente anterior sería así:


A. < 5frascos
B. 5-9 frascos
C. 15 frascos
D. 15 frascos.

Efectos simpaticomimeticos de cocaína: ES HIPERGLICEMIA

Con respecto a la adicción de las drogas uno de los siguientes enunciados es falso: La cafeína se
considera un estimulante mayor (es menor)

Con respecto a la neurobiología de la adicción es correcto: Las anfetaminas son dopaminergicas

Nicotina cierta: Tiene receptores específicos

En las caracterìsticas generales de los bipiridilos èstos se inactivan


A. se inactivan en presencia de la luz solar
B. se inactivan en presencia del agua
C. se inactivan con presencia de otro producto mezclado en el envase
D. se inactivan al tener contacto con los minerales arcillosos del suelo

El antídoto principal para las manifestaciones nicotìnicas es


A. atropina
B. protopam (pralidoxima)
C. terazocin
D. ninguna de las anteriores
E. todas las anteriores
El tratamiento de las arritmias ventriculares por intoxicación con cocaína incluye
A. Digoxina
B. amiodarona
C. nifedipina
D. propanolol

Indique el efecto neurológico tardío provocado por la intoxicación aguda con organofosforado:
A. Degeneración cerebelar
B. Convulsiones
C. Demencia
D. Polineuropatía motora

Paciente masculino de 52 años de edad que es llevado al Cuarto de Urgencia luego de 6 horas después
de tomar una sopa en la fonda, en donde siempre almorzaba con los compañeros del trabajo. El cuadro
se caracterizaba por nauseas, vómitos, dolor abdominal, y diarrea. Según el médico del cuarto de
urgencia los signos vitales eran estables y solo tenía algo de signos de deshidratación. Se trata al
paciente como una gastroenteritis y se inicia el manejo con hidratación parenteral y ciproxina I.V. A los 30
minutos es traído el compañero que siempre lo acompaña al almuerzo con el mismo cuadro, por lo que
los médicos del cuarto de Urgencia lo manejaron como una intoxicación alimentaria. A eso de las 2 horas
uno de los pacientes comienza a presentar broncorrea, sialorrea, cuando se ausculta tiene estertores
subcrepitantes y roncus el médico le suspende la venoclisis porque piensa que el paciente tiene un
edema de pulmón. Usted está entrando al turno y le presentan los 2 casos, ¿Cuál sería su abordaje?
A. Iniciar nuevamente la venoclisis pero en forma más lenta ya que el paciente mejoro levemente
con furosemida y aminofilina.
B. Omitir la ciproxina porque el medicamento desencadeno el cuadro de edema.
C. Abordar nuevamente a los 2 pacientes e investigar si hay más casos parecidos.
D. Iniciar mejor hidratación oral.

Pte mordido por una culebra, no presenta signos al inicio, 24 horas después presenta ptosis palpebral. La
intoxicación ofídica la clasificaría como:
A. Síndrome Botrops
B. S. Elapídico
C. S. Clotalico
D. S. Botulínico

En intoxicación por Paraquat tratamiento es:


A. Ditionita de sodio
B. N-acetil-cisteína
C. Antiinflamatorios+
D. AINES

En intoxicación por organofosforados los síntomas nicotínicos incluyen:


Agitación
Parálisis
Convulsiones

El tratamiento con atropina da todo menos:


A. Miosis
B. Sequedad de boca
C. Bradicardia

La hemodiálisis como tratamiento de intoxicación por barbitúricos está indicada en:


A. coma en estadio III
B. cuando el paciente está en estupor
C. al llegar al cuarto de urgencia para prevenir el coma
D. Edema agudo de pulmón
E. se el paciente tiene diarrea

Un paciente con herida de la piel contaminada espera encontrar como bacteria anaerobia:
A. Fusobacterium
B. Bacterioides
C. Clostridium
D. Peptoestreptococo
E. Ninguna de las Anteriores

En la enfermedad por legionella es muy difícil pensar en este diagnóstico cuando se enferma un paciente:
A. Fumador
B. Adulto
C. Anciano
D. Alcohólico
E. Paciente Acinado

En la enfermedad por legionella el tratamiento de eleccion es:


A. Macrolidos
B. Tetraciclinas
C. Cefalosporinas
D. Aminoglicosidos
E. Fosfomicinas

La fosfomicina es un antibiotico que ademas de las infecciones de piel tiene indicacion en infecciones:
A. intestinales
B. urinarias
C. vesiculares
D. respiratorias bajas
E. otitis por gram negativo

Es la dosis letal media de veneno para un adulto de 70 kg es, excepto


A. 24 mg de veneno crotálico y 150 mg de veneno laquésico
B. 5 mg de veneno de coral
C. 70 mg de veneno bothropico
D. Todas las anteriores
E. Ninguna de las anteriores

La dosis inicial calculada de suero antiofidico polivalente seria


A. 1-4 ampollas
B. 5-9 ampollas
C. 10-15 ampollas
D. 15-20 ampollas
E. 25 ampollas

Paciente masculino de 47 años de edad que acude al cuarto de urgencia por sintomatología de dolor
abdominal, náuseas, vómitos, diarrea, sudoración profusa con mucha ansiedad, 3 horas después de
haber ingerido una sopa en su restaurante favorito.
Tiene como antecedentes que es hipertenso y toma atenolol. Al examen fisico tiene una PA 110/70 FC:
60 x min. Ojos pupilas con la tendencia a la miosis, corazon con bradicardia, pulmones con rudeza
respiratoria. Usted sospecha que puede tratarse de cual de las siguientes entidades. Usted sospecha que
tiene una intoxicación por organofosforado. Los síntomas mencionados son la mayoria:
A. Muscarínicos
B. Nicotínicos
C. Del SNC
D. Intoxicación…
E. Ninguna de las anteriores

Con respecto a la intoxicación anterior, cuál de los siguientes es un síntoma del SNC
A. Agitación
B. Fasciculaciones (nicotínico)
C. Miosis (muscarinico)
D. Dolor abdominal (muscarinico)
E. Tremor

Si usted pudiera medir los niveles de paraquat en sangre por espectrofotometria, … nivel que indica alta
probabilidad de muerte es
A. 2 mg/L a las 4 horas
B. 2 mg/L a las 2 horas
C. 1.5 mg/L a las 4 horas
D. 0.9 mg/L a las 6 horas
E. 0.5 mg/L a las 6 horas

Es característico de la intoxicación por diquat


A. Convulsiones
B. Infarto agudo del miocardio
C. Insuficiencia hepatica
D. Edema pulmonar
E. Ninguna de la anteriores

Cuando usted administra pralidoxine en la intoxicación por organofosforado… actuando sobre:


A. Síntomas pulmonares
B. Síntomas muscarinicos
C. Reactivacion enzimatica
D. Síntomas del SNC (y síntomas nicotinicos)
E. Ninguna de las anteriores
Cual de las siguientes entidades se puede confundir con una intoxicación por organofosforados
A. Constipación
B. Hipotiroidismo
C. Hipertermia maligna
D. Intoxicación por hongos
E. Boca reseca

Para considerar fiebre de origen por determinar, además del tiempo de la fiebre esta debe ser por lo
menos de:
A. 37. ºC
B. 37.5
C. 37.8
D. 38.3
E. 39

*** En la fiebre de origen por determinar además de la causa más frecuente ser la infecciosa y considerar
las neoplasias, la ficticia enumere otras tres causas
Medicamentos
Inflamatoria
___________

Los siguientes pacientes con fiebre mediana requieren tratamiento excepto:


A. Embarazada
B. Niños con convulsiones
C. Falla cardiaca (no)
D. Falla pulmonar
E. Fiebre > 40ºC

Cuál sería su primer paso en este momento?


A. lavado gastrico
B. Dar jarabe de ipecacuana
C. Hemodiálisis
D. Cumplir el ABC de los primeros auxilios

Como explicaria el ultimo cuadro de los pacientes (convulsiones, somnolencia), ya que sus síntomas
estan relacionados con:
A. efecto del diquat a nivel del SNC
B. cuadro del SNC de organos fosforados
C. Cuadro del paraquat a nivel del SNC
D. Cuadro nicotínico por organos fosforados

Cual seria el antidoto para los siguientes toxicos:


A. Oxigeno y N-acetil cisterna para el paraquat
B. Oxigeno y Diazepam para el diaquat
C. Atropina y Oxima para intoxicación por organos fosforados
D. Tratamiento de soten para organos clorados

Desde el punto de vista toxicologico de la via de absorción mas importante es la intoxicación por paraquat
es:
A. digestivo
B. respiratorio
C. piel
D. conjuntival

Organos afecta el paraquat que puede tener dano irreversible cuando la dosis es de 20 a 40/kg de peso
corporal:
A. e. pulmón
B. f. hígado
C. g. riñón
D. h. corazón

Complicación tardia del paraquat a nivel de los ojos es:


A. Desprendimiento de retina
B. Conjuntiva de retina
C. Keratitis y Uveitis
D. Opacidad corneal

Toxic6. Las siguientes situaciones se presentan en caso de hipocalcemia con hipoalbuminemia, excepto
A. gastritis
B. enfermedades agudas
C. sepsis
D. síndrome de mala absorciòn
E. nefrosis

Paciente mordido por una culebra no presenta signos al inicio, 24 horas después presenta la intoxicación
crotálica la clasificaría como
A. síndrome bothtropico
B. s. e…
C. s. crotálico
D. s. botulínico

Paciente mordido por una culebra en el pie con aparición de edema un tercio de la pierna …
A. grado 0
B. leve
C. intermedio
D. severa

Todo los siguiente es cierto, excepto:


A. La coral pertenece a la familia Elapidae
B. El efecto biologico del veneno crotalico es exclusivamente neurotoxico
C. La fraccion toxica del veneno de serpiente son proteinas y peptido de 6000-30,000 menes con
receptores específicos
D. El veneno de serpiente se produce en las glandulas homologas a las parotidas
E. El suero antiofidico se obtiene inmunización equina

Efectos simpaticomimeticos de cocaína:


hipoglicemia

Con respecto a la adicción de las drogas uno de los siguientes enunciados es falso:
La cafeína se considera un estimulante mayor

Con respecto a la neurobiología de la adicción es correcto:


Las anfetaminas son dopaminergicas

Las siguientes manifestaciones indican atropinización, excepto:


A. Sed
B. Bradicardia
C. midriasis
D. Visión borrosa
E. salivación

Los bisfosfonatos
A. Inhiben la absorción de calcio intestinal
B. Aumentan la excreción urinaria de calcio
C. Inhiben la resorción ósea
D. Actúan sobre los fibroblastos
E. No tienen acción sobre el hueso

El diguat es menos toxico que el paraguat y actúan peroxidando la membrana celular


a. Primera parte cierta y segunda parte cierta
Nicotina cierta:
o Tiene receptores específicos

Pie mordido por una culebra, con aparición de equimosis, edema hasta 1/3 medio de la pierna con tpt Y
tp alterados ligeramente , la intoxicación ofídica se clasifica como
- leve

Cuando se requiere antibiótico esta indicado


A. doxiciclina
B. Ofloxacina
C. Metronidazole
D. Ceftriaxone

Usted está en el cuarto de urgencias y evalúa un paciente con cuadro clínico evidente de intoxicación por
Paraquat pero para más seguridad ya que no trajeron el recipiente usted le hace una prueba de ditionita
y el misma da una coloración verde usted pensaría que:
A. Se trata de una intoxicación por diquat
B. Que la Ditionita está oxidada.
C. Que la Ditionita no tiene bicarbonato por esto sale verde.
D. Que da ese color porque el paciente no tiene ni 30 minutos de estar intoxicado.

. Cual de los siguientes es un Carbamato:


A. Baygon
B. Asuntol
C. Baytex
D. Maltox

Un paciente intoxicado con paraquat llega al cto de urgencia con insuficiencia respiratoria, este pacientes
no deben de recibir.
A. Oxigeno
B. Solución salina
C. Dextrosa en salina
D. Corticoide

Todo los siguiente es cierto, excepto:


A. La coral pertenece a la familia Elapidae
B. El efecto biologico del veneno crotalico es exclusivamente neurotoxico
C. La fraccion toxica del veneno de serpiente son proteinas y peptido de 6000-30,000 menes con
receptores específicos
D. El veneno de serpiente se produce en las glandulas homologas a las parotidas
E. El suero antiofidico se obtiene inmunización equina

Del veneno de coral todo lo siguiente es cierto excepto:


A. Es de acción exclusivamente neurotóxica.
B. Ataca el sistema nervioso en la placa mioneural
produce una parálisis muscular progresiva la afección de los trayectos nerviosos es en
sentido centrífugo.
C. Se fija a los tejidos en un período de 4 horas

Es caracteristico de la intoxicación por diquat


A. Convulsiones
B. Infarto agudo del miocardio
C. Insuficiencia hepatica
D. Edema pulmonar
E. Ninguna de la anteriores

Cual de las sig es incorrecta con el consumo de sustancias


R/ El concepto de tolerancia a una sust hace referencia al hecho de que el consumo continuado se
precisan cada vez dosis menores para producir el mismo efecto

En intoxiación por organofosforados los síntomas muscarínicos incluyen todos excepto:


A. Sialorrea
B. Miosis
C. Calambres

En intoxicación por organofosforados los síntomas nicotínicos incluyen:


A. Agitación
B. Parálisis
C. Convulsiones

El tratamiento con atropina da todo :


A. Miosis
B. Sequedad de boca
C. Bradicardia

En paciente intoxicado con OF usted revisa:


A. Hemograma diario con revisión de electrólitos
B. Pseudocolinesterasa diaria niveles en sangre
C. Gases arteriales diarios
D. Urinálisis
TOXICOS

1- Efecto de reversión enzimática de la pralidoxima


- tiempo:________
- dosis: ADULTOS Y > 12 AÑOS: 1-2 g DILUÍDOS EN 100-200 ml DE SUERO GLUCOSADO AL 5%
O SSN. PASAR EN INFUSIÓN NO MAYOR DE 0.2 GRAMOS POR MIN (10-20 ml/min.)
< DE 12 AÑOS: 20-40 mg/Kg de PESO, DILUÍDO EN SOLUCIÓN A RAZÓN DE 10 ml/Kg.

2- Efectos Agudos de los OF:

3. Todos los sintomas son propios de la intoxicacion con inhibidores de la


colinesterasa, exceptos:
a)bradicardia
b)fasciculaciones
c)sequedad de la mucosa
d)miosis

4. Le llega un paciente intoxicado de paraquat al cuarto de urgencias por intento


de suicidio. No se debe administrar:
a)N-acetyl cisteina
b)oxigeno
c)solucion salina
d)acetaminofen
e)Ninguna de las anteriores
5. En el paciente que presentamos para la investigación que tenia un cuadro
similar a la gastroenteritis los receptores más afectados eran:
a)Nicotinicos
b)SNC
c)Sustancia P
d)Muscarinicos
e) Ninguno de los anteriores

Esto es porque el caso de la investigación de los 2 pacientes que mandaron a casa


porque se pensaba que era una gastroenteritis sin saber que era una intoxicación por
Carbamatos… En alguna parte del p esta el caso…

6. Cual de las siguientes entidades se puede confundir con una intoxicación por
organofosforados
a) Constipación
b) Hipotiroidismo
c) Hipertermia maligna
d) Intoxicación por hongos
e) Boca reseca

7.Paciente masculino de 47 años de edad que acude al cuarto de urgencia por


sintomatología de dolor abdominal, nauseas, vómitos, diarrea, sudoración
profuso con mucha ansiedad, 3 horas después de haber ingerido una sopa en su
restaurante favorito.
Tiene como antecedentes que es hipertenso y toma atenolol. Al examen fisico
tiene una PA 110/70 FC: 60 x min. Ojos pupilas con la tendencia a la miosis,
corazon con bradicardia, pulmones con rudeza respiratoria. Usted sospecha que
puede tratarse de cual de las siguientes entidades. Usted sospecha que tiene una
intoxicación por organofosforado. Los síntomas mencionados son la mayoria:
a) Muscarínicos
b) Nicotínicos
c) Del SNC
d) Intoxicación…
e) Ninguna de las anteriores

8.Con respecto a la intoxicación anterior, cual de los siguiente es un síntoma del


SNC
a) Agitación
b) Fasciculaciones (nicotínico)
c) Miosis (muscarinico)
d) Dolor abdominal (muscarinico)
e) Tremor(temblor)
9.Si usted pudiera medir los niveles de paraquat en sangre por
espectrofotometria, … nivel que indica alta probabilidad de muerte es
a) 2 mg/L a las 4 horas
b) 2 mg/L a las 2 horas
c) 1.5 mg/L a las 4 horas
d) 0.9 mg/L a las 6 horas
e) 0.5 mg/L a las 6 horas

10.Es caracteristico de la intoxicación por diquat


a) Convulsiones
b) Infarto agudo del miocardio
c) Insuficiencia hepatica
d) Edema pulmonar
e) Ninguna de la anteriores

11. Cuando usted administra pralidoxine en la intoxicación por


organofosforado… actuando sobre:
a) Síntomas pulmonares
b) Síntomas muscarinicos
c) Reactivacion enzimatica
d) Síntomas del SNC (y síntomas nicotinicos)
e) Ninguna de las anteriores

12. Es la dosis letal media de veneno para un adulto de 70 kg es, excepto


a) 24 mg de veneno crotálico y 150 mg de veneno laquésico
b) 5 mg de veneno de coral
c) 70 mg de veneno bothropico
d) Todas las anteriores
e) Ninguna de las anteriores

13. La dosis inicial calculada de suero antiofidico polivalente seria


a) 1-4 ampollas
b) 5-9 ampollas
c) 10-15 ampollas
d) 15-20 ampollas
e) 25 ampollas

14. Droga que es síntesis de laboratorio:


Tramadol

15. Efectos simpaticomimeticos de cocaína:


Hipoglicemia
*no salía ni en ppt ni en internet*
La toxicidad aguda es dependiente de la dosis y se caracteriza por los efectos
simpaticomiméticos: hipertensión, taquicardia, hipertermia, arritmias y
posteriormente convulsiones, depresión del tronco cerebral y colapso
cardiorespiratorio. Tras una "borrachera" de cocaína se puede producir ataque
cerebral, coma, vasculitis intracraneal, infarto de miocardio o muerte súbita. A
menores dosis se observan breves períodos de paranoia y alucinaciones.

16. Nicotina cierta:


o Tiene receptores específicos
Es cierto pero no se a que se refiere en la nicotina en si o los receptores
nicotínicos

17. Cual de los siguientes síntomas y signos forma parte del Síndrome
muscarínico en intoxicación aguda de órganos fosforados excepto
a. Abdominal. b. Miosis c. Calambres d. Rinorrea. e. Diarrea
Muscarinicos: post ganglionares
Miosis, visión borrosa
Dolor abdominal
Sialorrea
Rinorrea
Broncorrea, Broncoespasmo
Bradicardia
Trastornos gastricos

18.cual de las sig es incorrecta con el consumo de sustancias


R/ el concepto de tolerancia a una sust hace referencia al hecho de que el
consumo continuado se precisan cada vez dosis menores para producir el mismo
efecto
Son dosis mayores para producir el mismo efecto*

19. Todo lo siguiente es cierto, excepto


a. el veneno de serpiente se produce en las glándulas homólogas a las
paratiroides
b. la fracción tóxica del veneno de serpiente son proteínas y péptidos de 5,000 -
30,000
c. el efecto biológico del veneno CROTALICO es estructuralmente reumático
d. la coral pertenece a la familia Elipidae
*El efecto es local con aparición de bulas, necrosis, ampollas, edema, hemorragia
en el punto de la mordida y sistémico mas que todo hematológicos (hemorragias,
necrosis) y neuronales (ptosis palpebral, mareos, visión borrosa, dolor de cabeza)
DHE entre otros

20. Paciente mordido por una culebra no presenta signos al inicio, 24 horas
después presenta la intoxicación crotálica la clasificaría como
a. síndrome bothtropico
b. s. e…
c. s. crotálico
d. s. botulínico
*Según se la familia Vipiridae tiene a los Bothrox y a los Crotalus y aparecen los
síntomas sistémicos horas después que la local

21. La hemodiálisis como tratamiento de intoxicación por barbitúricos esta


indicada en:
a. coma en estadio III
b. cuando el paciente esta en estupor
c. al llegar al cuarto de urgencia para prevenir el coma
d. edema agudo de pulmon
e. se el paciente tiene diarrea
Normalmente es para oliguria o anuria pero encontré en internet que la diuresis
forzada podía aliviarlo

22.Cual de las siguientes entidades se puede confundir con una intoxicación por
organofosforados
a) Constipación
b) Hipotiroidismo
c) Hipertermia maligna
d) Intoxicación por hongos
e) Boca reseca

Diagnóstico diferencial
Debido a que existen algunas patologías o situaciones que pueden ser semejantes
al cuadro presentado por los intoxicados con inhibidores de las colinesterasas, es
necesario conocerlas, para establecer en el menor tiempo posible, el diagnóstico
diferencial que asegure una conducta terapéutica acertada. Podemos mencionar,
como las más relevantes, las siguientes:
• Síndrome convulsivo
• Estado de coma hipo e hiperglicémico
• Estados de coma provocados por otras causas
• Intoxicaciones con otros tóxicos, tales como fluoracetato de sodio, hidrocarburos
clorados, depresores del sistema nervioso central (como los alcoholes etílico y
metílico, los opiáceos).
• Intoxicación paralítica por mariscos (marea roja)
• Cuadros neuropsiquiátricos, como neurosis conversiva
• Enfermedad diarréica aguda
• Edema pulmonar agudo asociado a otras etiologías
• Insuficiencia cardíaca congestiva
• Hiperreactividad bronquial
• Micetismo (intoxicación por hongos)
24) En paciente intoxicado con OF usted revisa:

a) Hemograma diario con revisión de electrólitos


b) Pseudocolinesterasa diaria niveles en sangre
c) Gases arteriales diarios
d) Urinálisis
Esto es para intoxicación aguda*

25) Intervalo de tiempo desde la intoxicación con un organofosforado en el cual


es más beneficiosa y efectiva la administración de una oxima:
a. Antes de transcurridas las 24 a 48 horas
b. Luego de transcurridas las 48 horas.
c. Entre las 72 a 96 horas
d. Su administración es igual de efectiva en cualquier momento.

26. En el manejo de los accidentes ofídicos lo más importante es


a. Determinar que especie de serpiente está involucrada en el accidente
b. Trasladar a la víctima lo antes posible a un centro médico
c. Administrar suero antiofíodico específico
d. Determinar el grado de envenenamiento
e. Todas las anteriores

27. De las escorpionotoxinas solo es cierto


a. Actúan sobre los canales calcio de varios tejidos excitables.
b. Actúan sobre el sistema nervioso simpático produciendo liberación de
acetilcolina lo que determina el cuadro sistémico.
c. La acción de las toxinas escorpiónicas produce hipertensión e
hiperglicemia.
c. los órganos blancos más afectados son: corazón, páncreas y riñón.
d. Ninguna de las anteriores

28. Un paciente de 30 años es mordido en el pie por una serpiente venenosa.


Posteriormente, desarrolla edema de la pierna hasta la mitad del muslo, con signos
inflamatorios y sangrado por encías y hematuria. Podemos decir según la clasificación
clínica que nos encontramos ante un:
a. Síndrome Crotalico b. Síndrome Elapidico c. Síndrome Laquesico d,
Síndrome Bothropico

29) El producto que puede ayudar a la intoxicación por paraquat es el siguiente:

a. Ditionita b. Oxigeno c. N-Acetil-Cisteina d. Antinflamatorios e. Acetominofen

30). Cual de los siguientes síntomas y signos forma parte del Síndrome muscarínico en
intoxicación aguda de órganos fosforados excepto
a. Dolor abdominal. b. Miosis c. Pesadez de piernas d .Rinorrea. e.
Diarrea
31) Las siguientes manifestaciones indican atropinización, excepto:

a. Sed b. bradicardia c. midriasis d. Visión borrosa e. salivación

32) Paciente intoxicado con medicamento para los ojos presenta: rubor, mucosas
secas, intranquilo y retención urinaria su tx incluiría:
a. Fisostigmina b) Atropina c) Propranolol d) Flumazenil

33).. Paciente rociado con un tóxico llamado Malatión podrá presentar los siguientes
síntomas muscarínicos excepto:
a) lagrimeo b) sialorrea c) broncoespasmo d) rubicundez

34) Intervalo de tiempo desde la intoxicación con un organofosforado en el cual es más


beneficiosa y efectiva la administración de una oxima:
a) Antes de transcurridas las 24 a 48 horas
b) Luego de transcurridas las 48 horas.
c) Entre las 72 a 96 horas
d) Su administración es igual de efectiva en cualquier momento.

*Dice que es mejor utilizarlas en las primeras 24 a 36 horas*

35). Paciente mordido por una culebra no presenta signos al inicio, 24 horas
después presenta la toxicación crotálica la clasificaría como
a. síndrome bothtropico
b. s. eliphidae
c. s. crotalico
d. s. botulínico

36.La ruta más importante de intoxicación por tóxicos es:


a. Piel
b. Respiratorio
c. Ojos
d. Digestivo
e. Ninguna de las anteriores

37.El producto que no se debe usar en la intoxicación por paraquat es el siguiente:


Ditionita
Oxigeno
N-Acetil-Cisteina
Antiinflamatorios
Acetaminofen

38. Cuál de los siguientes síntomas y signos forma parte del Síndrome
muscarinico en intoxicación aguda de órganos fosforados excepto:
Dolor abdominal
Miosis
Taquicardia
Rinorrea
Diarrea

39. Cuál de los siguientes síntomas y signos forma parte del Síndrome nicotínico
en intoxicación aguda por órganos fosforados.
Ansiedad
Ataxia
Parálisis flácido
Convulsiones
Cefalea

40. En el manejo terapéutico del paciente intoxicado por órganos fosforados es


mejor guiarse por el siguiente patrón de laboratorio o gabinete.
Niveles de Pseudocolinesterasas diario.
Rx de tórax diario.
Hemograma diario.
Electrolitos y Pruebas funcionales Hepáticas diarias.
Gases arteriales diarios.

41. El tx de las arritmias ventriculares por intoxicación con cocaína incluye:


Digoxina
Amiodarona
Nifedipina
Propanolol

42. Hombre de 24 años que acude al cuarto de urgencias por edema de miembro
superior derecho de 6 horas de evolución y 1 hora de sangrado gingival. El
paciente refiere que vende drogas y que había escondido unos paquetes en un
patio baldió con herbazal en horas de la mañana. Los recogió en la tarde y
cuando introduce la mano en el herbazal, sintió que algo lo “pullo”. No tiene
antecedentes médicos de importancia excepto alergia al yodo y a la penicilina. El
examen físico: PA: 130/80 FC: 100 FR: 20 T: 38.5. Consciente, orientado con
edema de mano, antebrazo y hasta el hombro derecho, con flictenas en la mano y
antebrazo y salida de sangre por orificio pequeño en el dorso de la mano, cerca
de una gran flictena. Pulso radial y cubital derechas ausentes. Palidez de dedos de
mano derecha con pobre llenado capilar. Discreto edema de área supraescapular
derecha. Sangrado gingival mínimo. Cardiopulmonar normal, abdomen blando
no doloroso. No edema de miembros inferiores ni de miembro superior
izquierdo. Neurológico normal.
Con relación a este paciente:
a) El diagnóstico más probable es una fasceitis necrotizante
b) El paciente tiene un síndrome compartimental y el tratamiento inmediato
debe ser fasciotomia
c) El tratamiento de elección seria Vancomicina y Clindamicina IV.
d) Debe administrarse de manera inmediata toxoide tetánico.
e) Ninguna de las anteriores

43. Con respecto a la picadura por alacrán en Panamá:


El veneno de escorpión es una secreción apocrina compuesta de neurotoxinas de
10,000 a 20,000 gramos/mole.
El centruroides margaritatus es la especie más comúnmente involucrada en
accidentes por picadura de escorpión.
El envenenamiento por el género Tityus no produce manifestaciones locales, solo
sistémicas.
Todas son ciertas.

44. En relación los efectos de los venenos todas son ciertas excepto:
El veneno de coral es neurotóxico y llega a las uniones mioneurales donde se
produce un bloqueo sináptico.
El veneno de especies Bothrops produce alteración de la coagulación porque
afecta el fibrinógeno y el factor X de la coagulación.
El veneno de especies crotalus se fija en los tejidos a las 6 horas.
El veneno de escorpión actúa a nivel de terminaciones nerviosas preganglionares
del sistema simpático y parasimpático produciendo despolarización de las
mismas y liberación de catecolaminas y acetilcolina.

45. Paciente de 35 años con historia de haber sido mordido por una serpiente en
el pie derecho hace 10 horas mientras se encontraba cortando hierba en una finca,
posterior a lo cual nota que se le hincha el pie y tiene dolor local que se extiende
hasta la pierna. Desconoce que vacunas ha recibido: No alergias conocidas.
El examen físico muestra PA: 110/60 FC: 100 FR: 18 T: 37.9, escaso sangrado
por encía en arcada inferior derecha. Caries dentales. Cardiopulmonar normal,
abdomen sin alteraciones. Edema, eritema, calor local en pie y pierna derecha y 1
cm por arriba de la rodilla. Con dos puntos sangrantes en maléolo externo.
Biometria con leucocitosis y neutrofilia leves, TP y TPT prolongados en 12
segundos y plaquetas en 90,000. Urinalisis con 10-12 eritrocitos por campo,
Glicemia, creatinina y electrolitos normales.
Debe administrarse Toxoide tetánico 0.5cc I.M. como medida inmediata.
Debe administrarse 10 ampollas de Suero antiofídico polivalente por via
intravenosa.
Los antibióticos de elección para tratar el proceso infeccioso serian Oxacilina y
Clindamicina I.V.
Hay que administrarle al paciente Plasma fresco y plaquetas, conjuntamente con
la administración del suero antiofídico.
Todo lo anterior es cierto.

SEMESTRAL QUE SUBIO EVELYN!! HASTA AQUI


46. La hemodiálisis como tratamiento de intoxicación por barbitúricos está
indicada en:
coma en estadio III
cuando el paciente está en estupor
al llegar al cuarto de urgencia para prevenir el coma
edema agudo de pulmón
se el paciente tiene diarrea

47. En caso de mordedura humana, cuál de las siguientes indicaciones no


tendrían relevancia:
Toxicoide tetánico
Amoxicilina con ácido clavulanico
Metronidazol
Curación de la herida
Penicilina sódica

49. La prueba de Ditionita tiene valor pronostico en


en las primeras 24 horas
en las primeras 36 horas
36 horas en adelante
Después de las 48 horas
48. Desde el punto de vista toxicológico de la vía de absorción más importante es
la intoxicación por paraquat es:
digestivo
respiratorio
piel
conjuntival

49.Órganos afecta el paraquat que puede tener daño irreversible cuando la dosis
es de 20 a 40/kg de peso corporal:
pulmón
hígado
riñón
corazón

50. Complicación tardía del paraquat a nivel de los ojos es:


Desprendimiento de retina
Conjuntiva de retina
Queratitis y Uveítis
Opacidad corneal

51.Niveles de paraquat en sangre que indican alta probabilidad de muerte


0.9 md/L a las 24 horas
1.0 mg/L a las 6 horas
1.5 mg/L a las 4 horas
2.0 mg/L a las 4 horas
52.Todos estos síntomas son propios de la intoxicación por inhibidores de
colinesterasa excepto:
Bradicardia
Fasciculaciones
Sequedad de mucosas
Miosis

53.El diquat es menos toxico que el paraquat y actúan peroxidando la membrana


celular.
Primera parte cierta y segunda parte cierta.
Primera parte cierta y segunda parte falsa.
Primera parte falsa y segunda parte cierta.
Primera parte falsa y segunda parte falsa.
54. La causa más frecuente de enfisema subcutáneo en el estadio II y III de la
intoxicación por Paraquat es:
Perforación pulmonar por el daño directo del Paraquat al pulmón.
Perforación de una bula pulmonar.
La erosión directa en la piel es severa.
Perforación esofágica.

55 Paciente intoxicado con medicamento para los ojos presenta: rubor, mucosas
secas, intranquilo y retención urinaria su tx incluiría:
Fisostigmina b) Atropina c) Propranolol d) Flumazenil

56.Paciente rociado con un tóxico llamado Malatión podrá presentar los


siguientes síntomas muscarínicos excepto:
i. lagrimeo b) sialorrea c) broncoespasmo d) rubicundez

57. Cuando se requiere antibiótico esta indicado


a) doxiciclina b) Ofloxacina c) Metronidazole d) Ceftriaxone

58. En relación con el uso de atropina en la intoxicación por organofosforados,


cuál de las siguientes afirmaciones es falsa:
Debe administrarse en forma precoz
Contrarresta la hipersecreción bronquial y las bradiarritmias.
Actúa sobre los síntomas muscarínicos y nicotínicos
El delirio atropínico es consecuencia de su excesiva administración.

59. Intervalo de tiempo desde la intoxicación con un organofosforado en el cual


es más beneficiosa y efectiva la administración de una oxima:
ii. Antes de transcurridas las 24 a 48 horas
iii. Luego de transcurridas las 48 horas.
iv. Entre las 72 a 96 horas
v. Su administración es igual de efectiva en cualquier momento
60. En el manejo de los accidentes ofídicos lo más importante es
b. Determinar que especie de serpiente está involucrada en el accidente
c. b. Trasladar a la víctima lo antes posible a un centro médico
d. c. Administrar suero antiofíodico específico
e. d. Determinar el grado de envenenamiento
f. e. Todas las anteriores

61. Cuando usted administra pralidoxine en la intoxicación por


organofosforado… actuando sobre:
b. Síntomas pulmonares
c. Síntomas muscarinicos
d. Reactivacion enzimatica
e. Síntomas del SNC (y síntomas nicotinicos)
f. Ninguna de las anteriores

62. Niveles de paraquat en sangre que indican alta probabilidad de muerte


b. 0.9 md/L a las 24 horas
c. 1.0 mg/L a las 6 horas
d. 1.5 mg/L a las 4 horas
e. 2.0 mg/L a las 4 horas

61) Conociendo usted las propiedades bioquímicas del Paraquat. ¿Cuál será el
objetivo de utilizar como tratamiento inicial tierra o tierra fuller?
g. Bloquea la absorción intestinal y se elimina en orina
h. Elimina los metabolitos del mismo por orina.
i. Se inactiva en contacto con minerales arcillosos
j. Ayuda a eliminar el paraquat vía intestinal por heces al unirse a
ciertas proteínas intestinales

62) Usted aunque no lo crea esta esperando los niveles de Paraquat realizado por
cromatografía para dializar. Cuál sería el nivel que se puede beneficiar este
paciente con dicho procedimiento:
f. 2.5 mg/Lt a las 4 horas
g. 1.0 mg/Lt a las 6 horas
h. 1.5 mg/Lt a las 6 horas
i. 1.83mg/Lt a las 4 horas

63) Usted es llamado al cuarto de urgencia por un paciente que está intoxicado
por paraquat, para que le realice lavado gástrico. Usted lo realiza si el paciente:
g. Tiene 6 horas de haber ingerido la sustancia
h. Tiene 7 horas de haber ingerido la sustancia
i. Tiene 4 horas de haber ingerido la sustancia (lei que solo ates de la
primera hora de haber sido expuesto)
j. No sabe el tiempo de haber ingerido, lo importante es salvarlo

64) El manejo apropiado de un paciente con intoxicación por un organofosforado


incluye las siguientes medidas, Excepto:
b. Atropina
c. Nitrito de sodio
d. Cloruro de pralidoxima
e. Descontaminación.

65) Indique el efecto neurológico tardío provocado por la intoxicación aguda


con organofosforado:
a. Degeneración cerebelar b. Convulsiones c. Demencia d.
Polineuropatía motora

66) En relación con el uso de atropina en la intoxicación por organofosforados,


cuál de las siguientes afirmaciones es falsa:
a. Debe administrarse en forma precoz
b. Contrarresta la hipersecreción bronquial y las bradiarritmias.
c. Actúa sobre los síntomas muscarínicos y nicotínicos
d. El delirio atropínico es consecuencia de su excesiva
administración.

67) Todos estos síntomas son propios de la intoxicación con inhibidores de


colinesterasa excepto:
a. Bradicardia b. Fasciculaciones c. Miosis d. Sequedad de
piel y mucosas

68) Intervalo de tiempo desde la intoxicación con un organofosforado en el cual


es más beneficiosa y efectiva la administración de una oxima:
a. Antes de transcurridas las 24 a 48 horas
b. Luego de transcurridas las 48 horas.
c. Entre las 72 a 96 horas
d. Su administración es igual de efectiva en cualquier momento.

69) Paciente masculino de 52 años de edad que es llevado al Cuarto de Urgencia


luego de 6 horas después de tomar una sopa en la fonda, en donde siempre
almorzaba con los compañeros del trabajo. El cuadro se caracterizaba por
nauseas, vómitos, dolor abdominal, y diarrea. Según el médico del cuarto de
urgencia los signos vitales eran estables y solo tenía algo de signos de
deshidratación. Se trata al paciente como una gastroenteritis y se inicia el manejo
con hidratación parenteral y ciproxina I.V.
70) A los 30 minutos es traído el compañero que siempre lo acompaña al
almuerzo con el mismo cuadro, por lo que los médicos del cuarto de Urgencia lo
manejaron como una intoxicación alimentaria. A eso de las 2 horas uno de los
pacientes comienza a presentar broncorrea, sialorrea, cuando se ausculta tiene
estertores subcrepitantes y roncus el médico le suspende la venoclisis porque
piensa que el paciente tiene un edema de pulmón. Usted esta entrando al turno y
le presentan los 2 casos, ¿Cuál seria su abordaje?
a. Iniciar nuevamente la venoclisis pero en forma mas lenta ya que el
paciente mejoro levemente con furosemida y aminofilina.
b. Omitir la ciproxina porque el medicamento desencadeno el cuadro de
edema.
c. Abordar nuevamente a los 2 pacientes e investigar si hay más casos
parecidos.
d. Iniciar mejor hidratación oral.
71) Mientras usted investiga han pasado 3 horas, ya que usted no solo esta
viendo estos pacientes, uno de los pacientes está somnoliento y convulsiona, y el
otro comienza a presentar síndrome confucional, ya definitivamente comienza a
sospechar en otro tipo de intoxicación:
a. Intoxicación por Diquat por las convulsiones y dolor abdominal.
b. Intoxicación por órganos fosforados porque al principio tenían síntomas
nicotínicos.
c. Intoxicación por Paraquat por el dolor abdominal secundario a
Pancreatitis.
d. Intoxicación por órganos fosforados porque al principio tenían
síntomas muscarínicos.

72) Cual seria su primer paso en este momento?


a. lavado gastrico
b. Dar jarabe de ipecacuana
c. Hemodiálisis
d. Cumplir el ABC de los primeros auxilios

73) Como explicaria el ultimo cuadro de los pacientes (convulsiones,


somnolencia), ya que sus síntomas estan relacionados con:
a. efecto del diquat a nivel del SNC
b. cuadro del SNC de organos fosforados
c. Cuadro del paraquat a nivel del SNC
d. Cuadro nicotínico por organos fosforados

74) Cual sería el antídoto para los siguientes tóxicos:


A) Oxigeno y N-acetil cisteina para el paraquat.
B) Oxigeno y Diazepan para el diquat.
C) Atropina y Oxima para intoxicación por órganos fosforados.
D) Tratamiento de sostén para órganos clorados.
75) todo lo siguiente es cierto excepto:

a. El veneno de serpiente se produce en las glándulas homologas a las


parótidas
b. La fracción tóxica del veneno de serpiente son proteínas y péptidos de
6000-30,000 moles con receptores específicos
c. El efecto biológico del veneno crotálico es exclusivamente
neurotóxico
d. La coral pertenece a la familia elapidae

76) Pte mordido por una culebra , no presenta signos al inicio, 24 horas después
presenta ptosis palpebral
La intoxicación ofídica la clasificaría como:
a) Síndrome Botrops b) S. Elapídico c) S. Clotalico d) S. Botulínico
77) Pte mordido por una culebra en el pie , con aparición de equimosis , edema
hasta 1/3 medio de la pierna , con
TPT y TP alterados ligeramente la intoxicación ofídica se clasifica como:
a) Grado 0 b) leve c) moderada d) Severa

78) El tx con suero antiofídico del paciente anterior sería así:


a) < 5frascos b) 5-9 frascos c) 15 frascos d) > 15 frascos.
79) En las características generales de los bipiridilos estos se inactivan al:
a) Se inactivan en presencia de la luz solar en
b) Se inactivan en presencia del agua.
c) Se inactivan con presencia de otro producto mezclado en el envase para
producir emesis.
d) Se inactivan al tener contacto con los minerales arcillosos del suelo.

80). La causa mas frecuente de enfisema subcutáneo en el estadio II y III de la


intoxicación por Paraquat es:
a) Perforación pulmonar por el daño directo del Paraquat al Pulmón.
b) B. Perforación de una bula Pulmonar.
c) C. La erosión directa en la piel que es severa.
d) D. Perforación Esofágica.

81. Usted está en el cuarto de urgencias y evalúa un paciente con cuadro clínico
evidente de intoxicación por Paraquat pero para más seguridad ya que no trajeron
el recipiente usted le hace una prueba de ditionita y el misma da una coloración
verde usted pensaría que:
a) Se trata de una intoxicación por diquat.
b) B. Que la Ditionita está oxidada.
c) C. Que la Ditionita no tiene bicarbonato por esto sale verde.
d) D. Que da ese color porque el paciente no tiene ni 30 minutos de estar
intoxicado.

82. El antídoto confirmado hoy en día del Paraquat es:


a) N-Acetil Cisteina.
b) Oxigeno a 2 litros por minuto con inhaloterapia de N-Acetil cisteina.
c) Ciclofosfamida.
d) No existe en este momento.

83. Cual de los siguientes es un Carbamato:


a) Baygon
b) Asuntol
c) Baytex
d) Maltox.

84. En el paciente que presentamos para la investigación que tenía un cuadro


similar a una Gastroenteritis los receptores más afectado eran:
a) mucarinicos
b) Sistema Nervioso Central.
c) Sustancia P.
d) Nicotinicos.

85. Quien dota material como tierra Fuller al cuarto de Urgencia para el manejo
del Paraquat:
a) Ministerio de Salud
b) Ministerio de Desarrollo Agropecuario
c) La OMS.
d) La casa farmacéutica que produce el Paraquat.

86. El antídoto principal para las manifestaciones Nicotinitas es:


a. Atropina.
b. Protopan.
c. Toxogonin.
d. Ninguna de las anteriores.

87. El antídoto principal para las manifestaciones nicotìnicas es


a. atropina
b. protopam (pralidoxima)
c. terazocin
d. ninguna de las anteriores
e. todas las anteriores

88. Tenemos un paciente en la sala de Medicina y lo llama la enfermera porque


la infusión de Atropina se termino y el paciente intoxicado por Carbamatos está
asintomático cual sería el paso a seguir:
a. No continuar con la atropina.
b. Continuarle a dosis bajas esperando los niveles de
colinesterasas.
c. No continuar la atropina pero administrarla en bolo de atropina si
presenta manifestaciones de Broncorrea.
d. Administrarle Protopan y suspender la Atropina.

89. Un paciente intoxicado con paraquat llega al cto de urgencia con


insuficiencia respiratoria, este pacientes no deben de recibir.

a. Oxigeno
b. Solución salina
c. Dextrosa en salina
d. Corticoide

90. En el caso de la investigación de los 2 pacientes lo mandaron a casa porque


se pensaba que era una gastroenteritis sin saber que era una intoxicación por
Carbamatos regresan por convulsiones y paro cardiorrespiratorio. Usted pensaría
que son manifestaciones típicas de:
a. Síndrome Nicotínico. b. Síndrome muscarínico a nivel pulmonar y cardiaco.
c..Síndrome intermedio.
d. Síndrome del Sistema Nervioso Central. e. Síndrome de la medula
espinal

91. Cual de los siguientes síntomas y signos forma parte del Síndrome
muscarínico en intoxicación aguda de órganos fosforados excepto
a. Abdominal. b. Miosis c. Calambres d. Rinorrea. e. Diarrea

92) En intoxicación por organofosforados los síntomas nicotínicos incluyen:


a) Agitación
b) Parálisis
c) Convulsiones

93) Le llega un paciente intoxicado de paraquat al cuarto de urgencia por intento


de suicidio. Le explica a los familiares que el antídoto confirmado hoy en día del
Paraquat es:
a. N-Acetil Cisteina. b. Oxigeno a 2 litros por minuto con inhaloterapia de
N-Acetil cisteina.
c. Ciclofosfamida. d. No existe en este momento. c. Atropina

94. LAS DIFERENCIAS ENTRE EL SAO( SUERO ANTIOFIDICO)


LIOFILIZADO Y EL LIQUIDO ES
a. EL SAO LIQUIDO DURA MAS QUE EL LIOFILIZADO b. EL SAO
LIOFILIZADO NO REQUIERE REFRIGERACIÓN
c. EL SAO LIQUIDO NO REQUIERE REFRIGERACIÓN,PERO DEBE SER
GUARDADO EN LUGARES FRESCOS
d. NINGUNA DE LAS ANTERIORES e. TODAS LAS
ANTERIORES.
95. Cual de los siguientes síntomas y signos forma parte del Síndrome nicótico
en intoxicación aguda por órganos fosforados.
a. Ansiedad b. Ataxia c.Parálisis flácido d. Convulsiones. e. Cefalea
96) En intoxicación por Paraquat tratamiento es:
a) Ditionita de sodio
b) N-acetil-cisteína
c) Antiinflamatorios+....
d) AINES
97) En paciente intoxicado con OF usted revisa:
a) Hemograma diario con revisión de electrólitos
b) Pseudocolinesterasa diaria niveles en sangre
c) Gases arteriales diarios
d) Urinálisis
98. En el manejo terapéutico del paciente intoxicado por órganos
fosforados es mejor guiarse por el siguiente patrón de laboratorio o
gabinete.
k. Niveles de Pseudocolinesterasas diario.
l. Rx de tórax diario.
m. Hemograma diario.
n. Electrolitos y Pruebas funcionales Hepáticas diarias.
o. Gases arteriales diarios.
99. Efectos simpaticomimeticos de cocaína:
hiperglicemia
100) Con respecto a la adicción de las drogas uno de los siguientes
enunciados es falso:
La cafeína se considera un estimulante mayor
101) Con respecto a la neurobiología de la adicción es correcto:
Las anfetaminas son dopaminergicas
102) Paciente de 30 años mordido por serpiente venenosa y mitad de la
pierna con edema:
o Síndrome bothropico

103) Paciente en coma porque se tomo 30 pastillas de alprazolam, enojado


con su novia. Usted utilizaría el siguientr antídoto:
Flumazenil
104). Paciente en coma porque se tomó 30 pastillas de alprazolam, enojado con
su novia. Usted utilizaría el siguiente antídoto: a) Atropina b) Flumazenil
c) Protamina d) Fisostigmina

105) Cual de las siguientes no es una medida para evitar abrsocion del toxico
Descontaminación oral
D. digestiva
Diuresis forzada
D. cutánea
106) El tratamiento de las arritmias ventriculares por intoxicacion con cocaina
incluye:
a)Digoxina
b)Amiodarona
c)Nifedipina
d)Propanolol
107) Del veneno de coral todo lo siguiente es cierto excepto:
a. Es de acción exclusivamente neurotóxica
b. Ataca el sistema nervioso en la placa mioneural
c. Produce una paralisis muscular progresiva
d. La afeccion de los trayectos nerviosos es en sentido centrifugo
e. Se fija a los tejidos en un periodo de 4 horas.
108) De las escorpionotoxinas solo es cierto:
a. Actuan sobre los canales calcio de varios tejidos excitables
b. Actúan sobre el sistema nervioso simpatico produciendo liberación
de acetilcolina lo que determina el cuadro sistémico.
c. La acción de las toxinas escorpionicas produce hipertensión e
hiperglicemia
d. Los órganos blancos mas afectados son: corazón, páncreas y rinon.
e. Ninguna de las anteriores.

Toxicología

1. En intoxicación por Paraquat tratamiento es:

a) Ditionita de sodio
b) N-acetil-cisteína
c) Antiinflamatorios+....
d) AINES

2. En intoxiación por organofosforados los síntomas muscarínicos


incluyen todos excepto:

a) Sialorrea
b) Miosis
c) Calambres

3. En intoxicación por organofosforados los síntomas nicotínicos


incluyen:

a) Agitación
b) Parálisis
c) Convulsiones

4. El tratamiento con atropina da todo menos:


a) Miosis
b) Sequedad de boca
c) Bradicardia

5. El principal antidoto para la intoxicación por paraquat es:


a. n-acetil cisterna
b. selenio
c. colchicina
d. vitamina E en altas dosis
e. No existe ninguno en este momento

6.La hemodiálisis como tratamiento de intoxicación por barbitúricos esta


indicada en:
a. coma en estadio III
b. cuando el paciente esta en estupor
c. al llegar al cuarto de urgencia para prevenir el coma
d. edema agudo de pulmon
e. se el paciente tiene diarrea

7. Paciente masculino de 47 años de edad que acude al cuarto de urgencia por


sintomatología de dolor abdominal, nauseas, vómitos, diarrea, sudoración
profuso con mucha ansiedad, 3 horas después de haber ingerido una sopa en
su restaurante favorito.
Tiene como antecedentes que es hipertenso y toma atenolol. Al examen fisico
tiene una PA 110/70 FC: 60 x min. Ojos pupilas con la tendencia a la miosis,
corazon con bradicardia, pulmones con rudeza respiratoria. Usted sospecha
que puede tratarse de cual de las siguientes entidades. Usted sospecha que
tiene una intoxicación por organofosforado. Los síntomas mencionados son la
mayoria:
a) Muscarínicos
b) Nicotínicos
c) Del SNC
d) Intoxicación…
e) Ninguna de las anteriores

8. Con respecto a la intoxicación anterior, cual de los siguientes es un síntoma


del SNC
a) Agitación
b) Fasciculaciones (nicotínico)
c) Miosis (muscarinico)
d) Dolor abdominal (muscarinico)
e) Tremor

9. Si usted pudiera medir los niveles de paraquat en sangre por


espectrofotometria, … nivel que indica alta probabilidad de muerte es
a) 2 mg/L a las 4 horas
b) 2 mg/L a las 2 horas
c) 1.5 mg/L a las 4 horas
d) 0.9 mg/L a las 6 horas
e) 0.5 mg/L a las 6 horas

10. Es característico de la intoxicación por diquat


a) Convulsiones
b) Infarto agudo del miocardio
c) Insuficiencia hepatica
d) Edema pulmonar
e) Ninguna de la anteriores

11. Cuando usted administra pralidoxine en la intoxicación por


organofosforado… actuando sobre:
a) Síntomas pulmonares
b) Síntomas muscarinicos
c) Reactivacion enzimatica
d) Síntomas del SNC (y síntomas nicotinicos)
e) Ninguna de las anteriores
12. Cual de las siguientes entidades se puede confundir con una intoxicación
por organofosforados
a) Constipación
b) Hipotiroidismo
c) Hipertermia maligna
d) Intoxicación por hongos
e) Boca reseca

13. Paciente masculino de 52 anos de edad que es llevado al cuarto de


urgencia luego de 6 horas después de tomar una sopa de fonda, en
donde siempre almorzaba con los companeros de trabajo. El cuadro se
caracterizaba por nauseas, vomitos, dolor abdominal, y diarrea. Según el
medico del cuarto de urgencias los signos vitales eran estables y solo
tenia algo de signos de deshidratación. Se trata al paciente como una
gastroenteritis y se inicia el manejo con hidratación parenteral y
ciproxina IV. A los 30 minutos es traido el companero que siempre lo
acompana al almuerzo con el mismo cuadro, por lo que los medicos del
cuarto de urgencias lo manejaron com una intoxicación alimentaria. A
eso de las 2 horas uno de los pacientes comienza a presentar
boncorrea, sialorrea, cuando se ausculta tiene estertores subcrepitantes
y roncus, el medico le suspende la venoclisis porque piensa que el
paciente tiene un edema de pulmon. Usted esta entrando al turno y le
presentan los 2 casos ¿Cuál seria su abordaje?
a. Iniciar nuevamente la venoclisis pero en forma mas lenta ya que
el paciente mejoro levemente con furosemida y aminofilina
b. Omitir la ciproxina porque el medicamento desencadeno el cuadro
de edema
c. Abordar nuevamente a los 2 pacientes e investigar si hay
mas casos parecidos
d. Iniciar mejor hidratación oral

14. Mientras usted investiga han pasado 3 horas, ya que usted no solo esta
viendo estos pacientes, uno de los pacientes esta somnoliento y
convulsiona, y el otro comienza a presentar síndrome confucional, ya
definitivamente comienza a sospechar en otro tipo de intoxicación
a. Intoxicación por Diquat por las convulsiones y el dolor abdominal
b. Intoxicación por organos fosforados porque al principio tenian
síntomas nicotinicos
c. Intoxicación por paraquat por el dolor abdominal secundario a
pancreatitis
d. Intoxicación por organos fosforados porque al principio
tenian síntomas muscarinicos

15. Cual seria su primer paso en este momento?


a. lavado gastrico
b. Dar jarabe de ipecacuana
c. Hemodiálisis
d. Cumplir el ABC de los primeros auxilios
16. Como explicaria el ultimo cuadro de los pacientes (convulsiones,
somnolencia), ya que sus síntomas estan relacionados con:
a. efecto del diquat a nivel del SNC
b. cuadro del SNC de organos fosforados
c. Cuadro del paraquat a nivel del SNC
d. Cuadro nicotínico por organos fosforado

Cual seria el antidoto para los siguientes toxicos:


e. Oxigeno y N-acetil cisterna para el paraquat
f. Oxigeno y Diazepam para el diaquat
g. Atropina y Oxima para intoxicación por organos fosforados
h. Tratamiento de soten para organos clorados

17. La prueba de Ditionita tiene valor pronostico en


a. en las primeras 24 horas
b. en las primeras 36 horas
c. 36 horas en adelante
d. Después de las 48 horas

18. Desde el punto de vista toxicologico de la via de absorción mas importante


es la intoxicación por paraquat es:
a. digestivo
b. respiratorio
c. piel
d. conjuntival

19. Organos afecta el paraquat que puede tener dano irreversible cuando la
dosis es de 20 a 40/kg de peso corporal:
e. pulmon
f. higado
g. rinon
h. corazon

20.Complicación tardia del paraquat a nivel de los ojos es:


a. Desprendimiento de retina
b. Conjuntiva de retina
c. Keratitis y Uveitis
d. Opacidad corneal

21. Niveles de paraquat en sangre que indican alta probabilidad de muerte


a. 0.9 md/L a las 24 horas
b. 1.0 mg/L a las 6 horas
c. 1.5 mg/L a las 4 horas
d. 2.0 mg/L a las 4 horas

22. Conociendo usted las propiedades bioquímicas del Paraquat. ¿Cuál será el
objetivo de utilizar como tratamiento inicial tierra o tierra fuller?
a. Bloquea la absorción intestinal y se elimina en orina
b. Elimina los metabolitos del mismo por orina.
c. Se inactiva en contacto con minerales arcillosos
d. Ayuda a eliminar el paraquat vía intestinal por heces al unirse a
ciertas proteínas intestinales

23. Usted aunque no lo crea esta esperando los niveles de Paraquat realizado
por cromatografía para dializar. Cuál sería el nivel que se puede beneficiar este
paciente con dicho procedimiento:
a. 2.5 mg/Lt a las 4 horas
b. 1.0 mg/Lt a las 6 horas
c. 1.5 mg/Lt a las 6 horas
d. 1.83mg/Lt a las 4 horas

24. Usted es llamado al cuarto de urgencia por un paciente que está intoxicado
por paraquat, para que le realice lavado gástrico. Usted lo realiza si el paciente:
a. Tiene 6 horas de haber ingerido la sustancia
b. Tiene 7 horas de haber ingerido la sustancia
c. Tiene 4 horas de haber ingerido la sustancia
d. No sabe el tiempo de haber ingerido, lo importante es salvarlo

25. El manejo apropiado de un paciente con intoxicación por un


organofosforado incluye las siguientes medidas, Excepto:
a. Atropina
b. Nitrito de sodio
c. Cloruro de pralidoxima
d. Descontaminación.

26. Indique el efecto neurológico tardío provocado por la intoxicación aguda


con organofosforado:
a. Degeneración cerebelar b. Convulsiones c. Demencia d.
Polineuropatía motora

27. En relación con el uso de atropina en la intoxicación por organofosforados,


cuál de las siguientes afirmaciones es falsa:
a. Debe administrarse en forma precoz
b. Contrarresta la hipersecreción bronquial y las bradiarritmias.
c. Actúa sobre los síntomas muscarínicos y nicotínicos
d. El delirio atropínico es consecuencia de su excesiva
administración.

28. Todos estos síntomas son propios de la intoxicación con inhibidores de


colinesterasa excepto:
a. Bradicardia b. Fasciculaciones c. Miosis d. Sequedad de
piel y mucosas

29. Intervalo de tiempo desde la intoxicación con un organofosforado en el cual


es más beneficiosa y efectiva la administración de una oxima:
a. Antes de transcurridas las 24 a 48 horas
b. Luego de transcurridas las 48 horas.
c. Entre las 72 a 96 horas
d. Su administración es igual de efectiva en cualquier momento.

30. La ruta más importante de intoxicación por tóxicos es:

a. Piel b. Respiratorio c. Ojos d. Digestivo e. Ninguna de las


anteriores
31. El producto que puede ayudar a la intoxicación por paraquat es el siguiente:

a. Ditionita b. Oxigeno c. N-Acetil-Cisteina d. Antinflamatorios e.


Acetominofen

32. Cual de los siguientes síntomas y signos forma parte del Síndrome
muscarínico en intoxicación aguda de órganos fosforados excepto
a) Dolor abdominal. b. Miosis c. Pesadez de piernas d .Rinorrea.
e. Diarrea

33. Cual de los siguientes síntomas y signos forma parte del Síndrome nicótico
en intoxicación aguda por órganos fosforados.
a) Ansiedad b. Ataxia c .Parálisis flácido d. Convulsiones. e.
Cefalea

34. En el manejo terapéutico del paciente intoxicado por órganos fosforado es


mejor guiarse por el siguiente patrón de laboratorio o gabinete.

a. Niveles de Pseudocolinesterasas diario.


b. Rx de Tórax diario.
c. Hemograma diario
d. Electrolitos y Pruebas funcionales Hepáticas diarias.
e. Gases arteriales diarios

35. Las siguientes manifestaciones indican atropinización, excepto:

a. Sed b. bradicardia c. midriasis d. Visión borrosa e.


Salivación

36. Paciente intoxicado con medicamento para los ojos presenta: rubor,
mucosas secas, intranquilo y retención urinaria su tx incluiría:
a) Fisostigmina b) Atropina c) Propranolol d) Flumazenil

37. Paciente rociado con un tóxico llamado Malatión podrá presentar los
siguientes síntomas muscarínicos excepto:
a) lagrimeo b) sialorrea c) broncoespasmo d) rubicundez

38. Paciente en coma porque se tomó 30 pastillas de alprazolam, enojado con su


novia. Usted utilizaría el siguiente antídoto: a) Atropina b) Flumazenil c)
Protamina d) Fisostigmina
39. Cuál de las siguientes no es una medida para evitar absorción del tóxico:
a) Descontaminación oral b) D. digestiva c)Diuresis forzada d) D. Cutánea

40. El tx de las arritmias ventriculares por intoxicación con cocaína incluye:


a) Digoxina b) Amiodarona c) Nifedipina d) Propranolol*
*no se cual es, no encuentro info de eso, ya que cada página que busque dice
algo diferente, sale aquí marcado que propranolol y en otro profeta también,
otra pagina lei que arritmia se usa amiodarona, tambien lei algo de propranolol
pero no tan afirmador, y también que lidocaína. En fin no se.
41. Un paciente intoxicado con paraquat llega al cto de urgencia con
insuficiencia respiratoria, este pacientes no deben de recibir.

a. Oxigeno b. Solución salina c. Dextrosa en salina


d.Corticoide

42. Todos estos síntomas son propios de la intoxicación con inhibidores de


colinesterasa excepto:
a. Bradicardia b. Fasciculaciones c. Miosis d. Sequedad de piel y
mucosa

43. Usted está de turno en sala de Medicina y lo llama la enfermera porque la


infusión de Atropina se termino y el paciente intoxicado por Carbamatos está
asintomático cual sería el paso a seguir:
a. No continuar con la atropina.
b. Continuarle a dosis bajas esperando los niveles de
colinesterasas.
c. No continuar la atropina pero administrarla en bolo de atropina si
presenta manifestaciones de Broncorrea.
d. Administrarle Protopan y suspender la Atropina.
e. Continuar la misma infusión para cumplir protocolo

44. En el caso de la investigación de los 2 pacientes lo mandaron a casa


porque se pensaba que era una gastroenteritis sin saber que era una
intoxicación por Carbamatos regresan por convulsiones y paro
cardiorrespiratorio. Usted pensaría que son manifestaciones típicas de:
a. Síndrome Nicotínico. b. Síndrome muscarínico a nivel pulmonar y
cardiaco. c..Síndrome intermedio.
d. Síndrome del Sistema Nervioso Central. e. Síndrome de la medula
espinal

45. La causa mas frecuente de enfisema subcutáneo en el estadio II y III de la


intoxicación por Paraquat es:
a. Perforación pulmonar por el daño directo del Paraquat al Pulmón. b.
Perforación de una bula Pulmonar.
c. La erosión directa en la piel que es severa. d. Perforación
Esofágica
46. Cual de los siguientes síntomas y signos forma parte del Síndrome
muscarínico en intoxicación aguda de órganos fosforados excepto
a. Abdominal. b. Miosis c. Calambres d. Rinorrea. e. Diarrea

47. . Le llega un paciente intoxicado de paraquat al cuarto de urgencia por


intento de suicidio. Le explica a los familiares que el antídoto confirmado hoy en
día del Paraquat es:
a. N-Acetil Cisteina. b. Oxigeno a 2 litros por minuto con inhaloterapia de N-
Acetil cisteina.
c. Ciclofosfamida. d. No existe en este momento. c. Atropina

48. En el paciente que presentamos para la investigación que tenía un cuadro


similar a una Gastroenteritis los receptores más afectado eran:
a. Nicotinicos. b. Sistema Nervioso Central. c. Sustancia P. d,
Muscarinicos. e. Ninguno de los anteriores
**no estoy muy segura de esta, en la respuesta de la 44 está correcto que es
un síndrome del SNC, pero no sé si esto quiere decir que estos receptores son
los más afectados. La respuesta del profeta original era muscarínicos, pero
también el profeta original en la respuesta del 44 ponía Síndrome del SNC.

49. En paciente intoxicado con OF usted revisa:

a) Hemograma diario con revisión de electrólitos


b) Pseudocolinesterasa diaria niveles en sangre
c) Gases arteriales diarios
d) Urinálisis

50. En las características generales de los bipiridilos estos se inactivan al:


a) Se inactivan en presencia de la luz solar en
b) Se inactivan en presencia del agua.
c) Se inactivan con presencia de otro producto mezclado en el envase
para producir emesis.
d) Se inactivan al tener contacto con los minerales arcillosos del
suelo.

51. Usted está en el cuarto de urgencias y evalúa un paciente con cuadro


clínico evidente de intoxicación por Paraquat pero para más seguridad ya que
no trajeron el recipiente usted le hace una prueba de ditionita y el misma da
una coloración verde usted pensaría que:
a) Se trata de una intoxicación por diquat.
b) Que la Ditionita está oxidada.
c) Que la Ditionita no tiene bicarbonato por esto sale verde.
d) Que da ese color porque el paciente no tiene ni 30 minutos de estar
intoxicado.

52. Cual de los siguientes es un Carbamato:


a) Baygon
b) Asuntol
c) Baytex
d) Maltox.

53. Quien dota material como tierra Fuller al cuarto de Urgencia para el manejo
del Paraquat:
a) Ministerio de Salud
b) Ministerio de Desarrollo Agropecuario
c) La OMS.
d) La casa farmacéutica que produce el Paraquat.

54. El antídoto principal para las manifestaciones Nicotinitas es:


a. Atropina.
b. Protopan.
c. Toxogonin.
d. Ninguna de las anteriores.

55. el diquat es menos tóxico que el paraquat y actúan peroxidadndo la


membrana lipídica
a. Primera parte cierta, segunda parte cierta
b. primera parte cierta, segunda falsa
c. Primera falsa, segunda cierta
d. primera falsa, segunda falsa

• Efecto simpaticomimético de cocaía: hiperglicemia

• Con respecto a la adicción de las drogas uno de los siguientes


enunciados es falso: la cafeína se considera un estimulante mayor
(*es menor)

• Con respecto a la neurobiología de la adicción es correcto: las


anfetaminas son dopaminérgicas

• Nicotina cierta: tiene receptores específicos


Resolviendo el Profeta
1. Plasma: creatinina 1mg/dl, 140 mEq/L de Na. Orina: creatinina 150 mg/dl, Na 40
mEq/L la FE Na es:
0.19% (La fórmula es: FE Na= (Na Orina x Creatinina plasmática) / (Na plasmático x Creatinina
Orina) Y lo multiplicas por 100 para que de en %

FE Na= (40 x 1)/ (140 x 150) TODO por 100 = 0.190%


2. Factor determinante en tratamiento de hipokalemia real es:
R: función cardiovascular.
Recordar que el potasio es el principal actor en el potencial de relajación transmembrana así
que el paciente corre el gran riesgo de padecer arritmias; por ende en los pacientes con
insuficiencia cardiaca congestiva se da IECA para atenuar la hipokalemia inducida por
diuréticos y brinda protección contra arritmias cardiacas.
3. Tratamiento de inicio de SIADH es:
R: restricción hídrica
El tratamiento de inicio de la hiponatremia euvolémica es la restricción de H2O hasta <
1L/día, sin embargo a veces es inefectiva o mal tolerada. EXTRA: El tratamiento óptimo es
la corrección del trastorno subyacente.
4. Paciente con alcalosis metabolica e hipokalemia debe ser tratada con:
a) KCl
Esto se debe a que en el LEC hay una disminución de H+ y salen hidrogeniones del LIC al
LEC para compensar la alcalosis metabólica (disminución en [HCO3]) posterior a esto entra
un K+ del LEC para mantener el equilibrio de las cargas en ambos lugares.
Sabiendo esto el tratamiento con KCL es complementado con la corrección patológica
subyacente (diarrea) o retiro del medicamento nocivo (diurético de asa o tiazídico). Entonces
si no es posible o tolerable la complementación oral, puede administrarse KCL I.V. a través
de una vena central con vigilancia cardiaca, a un ritmo < 20 mmol/h. El KCL siempre debe
administrarse en soluciones salinas, no con glucosas porque el aumento en la insulina
inducido por glucosa causa exacerbación aguda de hipopotasemia porque la insulina en su
metabolismo introduce el potasio sérico a nivel intracelular lo que agrava aún más el asunto.
5. Causa de hiperkalemia
a) aumento de volumen arterial eficaz
b) hipercatabolismo tisular
c) hiperaldosteronismo
d) alcalosis metabolica
e) cirrosis hepática
R: Hay una amplia clasificación de las etiologías de la hiperkalemia y aquí redacto las
principales y sus sub-divisiones:
I. Pseudohiperkalemia
Salida de K+ de la célula (trombosis, eritrocitosis, leucocitosis y hemolisis in vitro.
(Aquí es donde está la respuesta de arriba)
Defectos hereditarios en la membrana de transporte eritrocitaria.
II. Desplazamiento del interior al exterior de la célula.
Acidosis
Hiperosmolalidad, medio de contraste radiográfico
Antagonistas adrenérgicos B
Digoxina y Glucósidos relacionados
III. Excreción inadecuada
Inhibición del SRAA. Aumenta el riesgo con IECA y ARA.
Disminución del aporte distal
Hipoaldosteronismo hiporreninémico
Resistencia renal a mineralcorticoides
Insuficiencia renal avanzada con velocidad de filtración glomerular disminuida.
Insuficiencia suprarrenal primaria
6. Causa de hipomagnesemia
a) cirrosis avanzada
b) falla renal cronica
c) falla cardiaca
d) constipación
e) acidosis respiratoria
Esta alteración no es muy común y sus causas son: falta de aporte (desnutrición), trastornos
del tracto digestivo (S. mal absorción, S. intestino corto, etc.) y los alcohólicos crónicos en
donde se presenta cirrosis. Y en estos casos no se presenta de forma aislada, sino junto a la
hipocalemia, hipofostatemia, hiponatremia.
7. Causa renal de hipomagnesemia
a) falla renal cronica
b) esteatorrea
c) desnutrición severa
d) uso de diuréticos de tiazidas. También Alcoholismo
e) infeccion urinaria
R: Las tiazidas causan hipomagnesemia por el siguiente mecanismo: Este medicamento
trabaja a nivel del túbulo contorneado distal inhibiendo el simportador de Na+/CL- en la
membrana apical de la célula; por ende lo que sucede es que al no permitir el paso de sodio
por su canal este se acumula en la membrana apical gormando una barrera catiónica y
obstruyendo a los canales de Mg y Ca por consiguiente todos estos iones son positivos y los
positivos se repelen es por eso que no se puede reabsorber el Mg y Ca por la obstrucción de
la barrera catiónica formada por el sodio y además a este nivel no se reabsorbe Mg ni Ca por
los espacios intercelulares pues carecen de canales y de gradiente eléctrico para poder ser
reabsorbidos.
8. En caso de hipercalcemia pensar en:
a) neoplasia
b) hiperfosfatemia severa
c) nefrolitiasis
d) nefrocalcinosis
e) diureticos
Primero saber que: Hiperparatiroidismo primario (más frecuente en pacientes ambulatorios),
la hipercalcemia de origen tumoral y la hipervitaminosis D (por iatrogenia), abarcan el 90%
de las causas de hipercalcemia.
La hipercalcemia se relaciona con el hiperparatiroidismo secundario a un adenoma de la
glándula tiroides que hipersecreta PTH, entonces es cierto que la PTH sintetiza calcitonina
que su trabajo es disminuir la concentraciones de Ca, sin embargo solo la PTH trabaja a nivel
renal reabsorbiendo calcio y como hay una hipersecreción de la misma el calcio sérico
aumenta y se tiende al riesgo de formar cálculos renales calcio.
9. El síndrome nefrótico puede encontrarse cilindros:
a) cereos
b) hematicos
c) leucocitarios
d) epiteliales
e) mixtos
R: Aparte el síndrome nefrótico se caracteriza por albuminuria (>3.5 g/día) e
hipoalbuminemia (<30g/L) asociado a edema, hiperlipidemia y lipiduria (En respuesta a la
hipoproteinemia, el hígado pone en marcha un mecanismo compensatorio consistente en la
síntesis de una mayor cantidad de proteínas, tales como alfa-2
macroglobulina y lipoproteínas, siendo estas últimas las causantes de la hiperlipemia que
incrementa el riesgo cardiovascular en estos pacientes)
10. U Na 115, Uosm 240. P osm 680, P Na 62
SIADH
Esto está así solo estos datos: Bueno lo que se sabe del SIADH es que el UNa debe ser > 20
mmol/L y que la Uosm debe ser > 600 mosm.
11. Paciente de 84 años que recibe diureticos para la hipertensión arterial. Presenta
diarrea y trastornos de sensoriales y disminución del rubor de la piel. Na 174 U Na 5 U
osm 606.
La hipernatremia se debe a: Perdidas insensibles
12. Hipernatremia hipervolemica
Hiperaldosteronismo primario o Síndrome de Cohn
R: Aumento de Na+ más que el aumento de agua. Corrección de una hiponatremia, NaHCO3,
solución usada en diálisis. Tratamiento Diurético + Agua. Acidosis metabólica con
bicarbonato.
Se presenta un adenoma a nivel de las glándulas suprarrenales que hipersecreta aldosterona,
por consiguiente se retiene más sodio y agua y se da una gran pérdida de potasio. Esto
conlleva hipernatremia con hiperosmolalidad e hipervolemia y trastornos cardiacos como las
arritmias.
13. causa de hipomagnesemia
Alcoholismo. También uso de diuréticos
R: Esta alteración no es muy común y sus causas son: falta de aporte (desnutrición),
trastornos del tracto digestivo (S. mal absorción, S. intestino corto, etc.) y los alcohólicos
crónicos en donde se presenta cirrosis. Y en estos casos no se presenta de forma aislada, sino
junto a la hipocalemia, hipofostatemia, hiponatremia.
14. hipocalcemia
Provoca potencializacion de efectos de hiperkalemia
15. En hipocalcemia
Prolonga QT a expensas de segmento ST
Lo de arriba mencionado solo sucede cuando la hipocalciemia es intensa además se asocia a
convulsiones, espasmo carpopedal, broncoespasmo, laringoespasmo
Aparte la hipocalciemia puede no mostrar síntomas si la disminución del calcio sérico es
relativamente leve y crónica. La hipocalciemia moderada o grave se acompaña de parestesias,
por lo común de dedos de manos y pies y zonas peribucales, y es causada por una mayor
irritabilidad neuromuscular.
16. Tratamiento de urgencia hipercalcemica
Solucion salina isotonica 0.9% 4 a 5 L
Saber además lo siguiente: La hipercalciemia asintomática leve por lo común no necesita
tratamiento inmediato y éste debe depender del diagnóstico principal. A diferencia de la
sintomática que obliga a una intervención terapéutica, sea cual sea el origen del exceso de
calcio. Las medidas iniciales contra la hipercalciemia notable comienzan con la expansión
volumétrica, porque el problema invariablemente culmina en deshidratación se necesitan en
las primeras 24 h 4 a 6 L de solución salina intravenosa, sin olvidar que cuadros coexistentes
primarios (como insuficiencia cardiaca congestiva) obligan a veces a emplear diuréticos con
acción en asa de Henle para intensificar la excreción de sodio y calcio esto pasa por que los
diuréticos de asa actúan inhibiendo el cotransportador de Na/K/2CI lo que forma en una
barrera catiónica en la membrana apical de la célula del asa ascendente de Henle e impide
que el Ca y Mg sean reabsorbidos por la tight junctions (zonas de unión) porque ambos son
positivos y estos con el sodio se repelen evitando así su reabsorción.
31. Síndrome nefrotico
Edema, proteinuria (>3.5 g/día), hipertensión arterial, hematuria (cilindros céreos),
hiperlipidemia y lipiduria
Las últimas dos se deben a lo ya explicado del siguiente proceso hepático: En respuesta a la
hipoproteinemia, el hígado pone en marcha un mecanismo compensatorio consistente en la
síntesis de una mayor cantidad de proteínas, tales como alfa-2
macroglobulina y lipoproteínas, siendo estas últimas las causantes de la hiperlipidemia que
incrementa el riesgo cardiovascular en estos pacientes.

COMPILACIÓN DE LLENAR ESPACIOS

Antibioticos

A usted le llega al consultorio una mujer embarazada de 12 semanas. Le realiza exámenes


y encuentra que tiene infección urinaria con más de 100,000 colonias. Nombre tres grupos
de antibióticos que se pudieran usar en esta paciente. Ejemplo:

• B-lactámicos
• Fluoroquinolonas
• macrolido

Describa el tratamiento para la profilaxis para los contactos con meningitis por
meningococo, dosis y tiempo.

• Rifampicina 4 dosis 10-20 mg/kg cada 12 horas


• Ciprofloxacina 500mg dosis única
• Ceftriaxona intramuscular dosis única

En la meningitis bacteriana, en el LCR, señale lo que espera usted encontrar.

• Hipoglucorraquia
• Proteinorraquia
• Pleocitosis predominantemente polimorfonuclear

Mencione el tratamiento de elección para los siguientes agentes etiológicos:

• Moraxella catarrhalis: cefalosporina de 2° o 3° generación, fluoroquinolonas, amoxicilina +


ácido clavulánico.
• Haemophilus ducreyi: Azitromicina
• Listeria monocytogenes: Ampicilina
• Clostridium difficile: Metronidazole o Vancomicina

Mencione cuatro fluoroquinolonas:

• Ofloxacina
• Ciprofloxacina
• Levofloxacina
• Norfloxacina

Neumonía adquirida en la comunidad:


-Cefalosporina de 3° generación

Infección urinaria no complicada:


Fosfomicina

Uretritis no gonococcica:
-Doxiciclina (acción contra Chlamydia trachomatis)

Tratamiento de Chlamydia trachomatis:

• Doxiciclina
• Azitromicina

Factores predisponentes a candidemia

• Nutrición parenteral
• Neutropenia
• Inmunosuprimidos
• Uso prolongado de antibióticos
modo de transmisión de TBC (5 rayas): orificio en orejas, circuncición, tatuaje, respiración boca
a boca, inyecciones

efectos adversos de los medicamentos:


• Rifampicina: hepatopatia
• Eritromicina: sintomas gastrointestinales

Uso del Ceftriazone: (3 rayas): Tx por gonococo, Ins Renal Aguda y barro biliar

Mencione 2 cefalosporina de 2° generación: cefotetan y cefoxitina

Tx de Acinetobacter—polimixina B 15000-25000 unidades/kg al dia iv dividido c/12h (también en


pseudomona aeruginosa)

Riketsia, mycoplasma, clamydia, menigitis por yersinia y francisella→cloranfenicol 12,5 a


25mg/kg iv c/6h
Clostridium P y D.-, giardia, entamoeba, trichomona, →Metronidazol 250 mg-750mg vo/iv
c/6h-12h y vancomicina 125- 250 mg qid x 10 dias
p. jiroveci→sulfonazol 1g vo c/6h
shíguela, cólera, salmonella→ TMP- SMX
staph aureus→ nafcilina 1.5g oxacilina 2g
STAPH RESISTENTE→ Vanco 250mg iv c/12h , TMP-SMX, eritro, clindam
E. coli, Proteus, klebsiella→ Cefalexina 250 mg a 500 vo c 6h
H. Influenzae→ claritromicina 250-500mg vo c/12h
Legionella→MACROLIDO: eritromicina 250- 500mg vo c/6h
Pseudomona A.→ciprofloxacina 250-750 mg vo c/12h
LISTERIA→ Ampicilina 1-3g iv c4-6h
MORAXELLA→ amoxi 500 y clavulanico 125 vo c 8h x 10 dia

Mencione tres antibióticos que tengan metabolismo hepático

• Tetraciclinas • Ciprofloxacina
• Doxiciclina • Eritromicina
• Isoniazida • Amoxicilina
• Nitrofurantoina • Ácido clavulánico

Nombre dos condiciones de tuberculosis extrapulmonar en donde usaría corticoides:

• TBC cerebral (meníngea)


• TBC pericárdica

Efecto secundario principal de los sig antibioticos:


• Rifampicina : hepatotoxico
• Imipenenm: convulsión
• Anfotericina B: hipokalemia
• Ketoconazol: cefalea, mareo, fotofobia, dif hepatica,
• Fluconazol: alopecia, nausea vomito

Mencione 3 antibióticos que se pueden usar en el embarazo:


• Penicilina • Cefuroxima
• Ampicilina • Ceftriaxona
• Amoxicilina • Erotromicina
• Azitromicina • Claritromicina

En la neumonía; tres signos de distress respiratorio:

• Disnea
• Taquipnea
• Taquicardia

Nombre dos condiciones para que se produzca una endocarditis:

• Lesión en el endocardio
• Bacteremia

Cuando un paciente tiene una sonda Foley puesta, entre la sonda y la uretra se forma un “biofilm”
por el que ascienden los siguientes dos gérmenes que son:

• Cándida
• Gram positivos

Nombre de anaerobios en lesión intracraneal.


• Actinomyces
• Fusobacterium

La actinomicosis es una condición relativamente rara con distribución mundial y sin predilección por
edad, raza, estación del año y ocupación. Los factores predisponentes incluyen: Nombre tres
condiciones que se asocian a esta enfermedad
a- Inmunosupresión
b- Mala higiene dental
c- Uso de dispositivos intrauterinos

Cefalosporina de 1ra generación de uso parenteral:


• Cefalotina
• Cefalexina
• cefazolina

Gastro
1. Mencione los 5 criterios de Child-Pugh.
• TP
• Albumina
• Bilirrubina
• Ascitis
• Encefalopatía hepática

2. Factores de riesgo en hígado graso no alcohólico (Nonalcoholic fatty liver) [3]


• Anastomosis gástrica (Gastric bypass • Síndrome metabólico
surgery) • Obesidad
• Hipercolesterolemia • Síndrome de ovario poliquístico
• Hipertrigliceridemia • Apnea del sueño
• Diabetes tipo 2 • Hipopituitarismo
• Hipotiroidismo

3. El absceso hepatico puede drenar principalmente a los siguientes tres espacios. Nómbrelos:
• Retroperitoneal
• Intratorácico
• Pericárdico

4. Mencione dos enfermedades que produzcan síndrome de malabsorción severo:


• Sprue tropical
• Enfermedad celiaca
• Intolerancia a la lactosa

5. Mencione causas de cáncer de colon:


• Edad mayor a 50 años
• Factores ambientales
• Predisposición genética (mutaciones en APC, K-ras, p-53)
• Enfermedad inflamatoria intestinal (EII, CUCI, CD)
• Poliposis adenomatosa familiar

6. Nombre 2 manifestaciones extrahepaticas de hepatitis viral:


• Crioglobulinemia
• Anemia
• Glomerulonefritis

7. Seudotipo de Vibrio cholerae→ biotipos: tor y clásico???

8. Hepatopatia ALCOHOLICA factores: alcoholismo, genético, ambiental.

9. Fármacos que producen colestasis hepática: paracetamol, isoniacida, ketoconazol, alopurinol,


ampicilina, cimetidina, esteroides anabólicos, estradiol, imipramida, tolbutamina.

10. Dentro del tratamiento farmacológico de la Enfermedad ulcero péptica, hay dos grandes pilares, neutralizar
el ácido gástrico, y actuar como cito protectores. Mencione, tres grupos, y de ejemplos de cada grupo, de
medicamentos que neutraliza con el ácido gástrico:
a. IBP:_OMEPRAZOL
b. ANTI H2_RANITIDINA
c. ANTIACIDOS: HIDROXIDO DE ALUMINIO

Mencione ahora, 2 medicamentos, cito protectores:


a. ___SALES DE BISMUTO_______________________________________.
d. __________SUCRALFATO_________________________________________.

11. A continuación, se muestran diversas características de la EII, si la misma corresponde a la


Enfermedad de Crohn, indique CD, pero si la misma corresponde a la Colitis Ulcerativa
Crónica Idiopática, coloque CUCI:
a. Presencia de moco: ______CUCI___________________________________.
b. Masa abdominal: _______CD____________________________________.
c. Presencia de Fistulas: _____CD______________________________________.
d. Respuesta a los antibióticos: ____CUCI_______________________________________.
e. Recto respetado: _____CD______________________________________.
f. Patrón Empedrado: _____CD______________________________________.
g. Signo de la Cuerda: _______CD____________________________________.
h. Granulomas: ______CD_____________________________________.
i. ANCA +: _____CUCI______________________________________.
j. ASCA +: _________CD__________________________________.

12. La triada clinica clásica del Sd. De Reiter es:


a. ____URETRITIS_______________________________________.
b. _____CONJUNTIVITIS______________________________________.
c. ______ARTRITIS_____________________________________.

13. Las siguientes son manifestaciones extra intestinales de la EII, indique si la misma es más
frecuente en CUCI, en CD o en ambas por igual:
a. Piodermia Gangrenosa: ____CUCI_______________________________________.
b. Eritema Nudoso: ___CD________________________________________.
c. Espondilitis anquilosante: __CD_________________________________________.
d. Sacroileitis: _______________AMBAS____________________________.
e. Epiescleritis: ____________________CD_______________________.

14. Mencione, los diversos grupos de fármacos que se pueden usar para el tratamiento de la EII, y de
al menos un ejemplo para cada grupo de medicamentos:

a. GLUCOCORTICOIDES__PREDNIZONA_____________________________.
b. _______ANTIBIOTICOS____METRONIDAZOLE________________________.
c. _______INMUNOSUPRESORES___CICLOSPORINA__
____D.______ ANAMONOCLONALES____INFLIXIMAB

15. Mencione 2 síndromes hereditarios asociados al CA de colon:


a. TURCOTT________________________________________.
b. GARDNER
c. LYNCH__________________________________________.

16. El H. Pylori puede implicarse en el desarrollo del CA gástrico, pues produce ciertas alteraciones
entre las que podemos mencionar.
17.
a. _______________disminucion de la acides y aumento de ph._____________________.
b. _____________________proiferacion bacteriana__________________________________.
c. ___________________________________gastritis atrofica____________________.

18. Mencione 2 bacterias del grupo HACEK:


a. __HAEMOPHILUS
PARAINFLUENZAE________________________________________________.
b. _EIKINELLA______________________________________________________.

19. Nombre dos síndromes diarréicos tipo secretora:


• Consumo crónico de etanol.
• Obstrucción intestinal incompleta.
• Tumor velloso
20. Nombre 2 sindromes diarréicos motores con aumento de motilidad:
• Hipertiroidismo
• Prostaglandinas.

IRC
Defina Falla Renal Aguda: perdida súbita y usualmente reversible de la función renal que se desarrolla a
lo largo de los días y se acompaña de disminución del volumen urinario

Única causa de diarrea con alcalosis metabólica es:


a. _____adenoma velloso___________________________________________.

Menciones los niveles plasmáticos normales de:


a. Sodio: ________135-145________________________________________.
b. Potasio: __________3.5-4.5______________________________________.
c. Magnesio: _________1.3-2.1_______________________________________.
d. Calcio: __________8.2-10.2______________________________________.
e. Fósforo: ______________2.5-4.5__________________________________.
f. Bicarbonato: __________22-26______________________________________.
g. pH: ______________7.35-7.45__________________________________.
h. pCO2: ________________40________________________________.
i. Hemoglobina: ____________12-16____________________________________.
j. Leucocitos: ______________4000-10 000__________________________________.
k. cloro: 96-110

El Sd. De Barter, se caracteriza clínicamente por:


a. ______HIPERALDOSTERONISMO__________________________________________.
b. __________NORMOTENSION___________________________________.
c. _____________ACIDOSIS METABOLICAS_____________________________.
d. ______________________HIPOKALEMIA______.

Los 4 síndromes básicos de la nefrología son:


a. ______IRA__________________________________________.
b. ______IRC__________________________________________.
c. _________GLOMERULOPATIA_______________________________________.
d. ____ENFERMEDAD TUBULAR INTERSTICIAL_______________________________.

Menciones algunas características físicas, que nos ayudarian a pensar que un paciente, de la
calle, por nuestra experiencia en este módulo, tiene una FRC:
a. _______HIPERPIGMENTACION DE LA PIEL_______________________________________.
b. ____________ALIENTO A ORINA____________________________________.
c. _____________EDEMA___________________________________.
d. _______PRURITO____________
PALIDO
PENSAR EN EDWARD CULLEN_________________.

La Osteodistrofia Renal, se va a caracterizar clínicamente por la presencia de:


a. _________OSTEOMALACIA_______________________________________.
b. ______________________OSTEOPOROSIS__________________________.

En un paciente con FRC, como espera usted encontrar:


a. Sodio: _______AUMENTADO_________________________________________.
b. Calcio: __________________DISMINUIDO______________________________.
c. Fósforo: _____________________AUMENTADO___________________________.
Mencione algunos parámetros de laboratorio, para pensar en una IRC Renal:
a. ___________________BUN_____________________________.
b. __________________ CREATININA_________________________.
c. __________________HEMOGLOBINA_______________.
d. _____________ELECTROLITOS____
__________________ PROTEINURIA _____________.

REUMA

Nombre tres indicaciones para realizar una densitometría

• Menopausia precoz • Inmovilización prolongada


• Menopausia por cirugía, radiación o • Tratamiento con corticoides
quimioterapia • Hiperparatiroidismo primario
• Menopausia desde 60 años en asintomático
pacientes con riesgo o >65 años • Osteopenia radiológica

Para qué se utiliza el análisis de líquido sinovial

• Artritis séptica
• Artritis por microcristales
• Artritis reumatoide*
• Gota*
• Pseudogota*

Microscopía con luz polarizada

• Gota
• Pseudogota

Análisis por artrocentesis se manda para diagnóstico de

• Artritis séptica
• Artritis por cristales

Mencione dos fármacos modificadores de la artritis reumatoidea:

• Metotrexate
• Cloroquina

Un paciente que tiene debilidad muscular proximal de ambos brazos se encuentra en estudio de una polimiositis. ¿Qué
enzimas están indicadas para el diagnóstico de esta enfermedad?

• Creatina cinasa (CK)


• Transaminasa glutámica oxaloacética
• Transaminasa glutámica pirúvica
• Deshidrogenasa láctica
• Aldolasa sérica
• PÁGINA 3514, HARRISON 18° EDICIÓN
Nombre las dos vasculitis de células gigantes.

• Arteritis temporal
• Arteritis de Takayasu

Fármacos que producen osteoporosis son:

• Corticoides • Heparina
• Litio • Tamoxifeno
• Aluminio • Tiroxina
• Metotrexate

Pareo de anticuerpos

• Anti-SS A/Ro Síndrome de Sjogren


• Anti-histona CREST
• Anti jo1 Polimiositis (encontrado en miopatías inflamatorias idiopáticas)
• Anti Sc-70 Esclerodermia

En la fiebre de origen por determinar (OxD), además de la causa más frecuente ser la infecciosa y considerar las
neoplasias, la ficticia enumere otras tres causas.

• Medicamentosa
• Inflamatoria
• ?

El síndrome antifosfolípido es hipercoagulable. Mencione otros 2 síndromes hipercoagulables:

• Embarazo
• Preeclampsia
• Neoplasia
• Anemia falciforme
• Enfermedad de hughes

Mencione factores de riesgo modificables en la osteoporosis: [3]

• Fumar
• Anticonceptivos
• Dieta baja en calcio

Anticuerpos de las enfermedades autoinmunes (pareo)


Anti RO→ sojogren
Anti histonas→por drogas
Anti RNP→ enf mixta de tej conectivo
FR→ Artritis Reuma
Anti CCP→ AR
Anti LA→ NEFRITIS
Anti CENTROMERO→ CREST
Anti scl 70→ esclerodermia
Anti JO I→ poliomiositis
c-ANCA→ vasculitis.
IL→ AR, Arteritis temporal, CA, sifilis.
1- Enfermedad digestiva que puede causar osteoporosis: colestasis por CBP (cirrosis biliar primaria),
malabsorcion ,, pancreatitis, fibrosis quística.
2- Enf hematológica que produce osteoporosis: mieloma multile, talasemia, leucemia

Efecto de reversión enzimática de la pralidoxima


• Tiempo: Tiene efecto antes de que envejezca la acetilcolinesterasa. Esto ocurre de 60 mins hasta semanas.
• Dosis: 1-2 gramos en 100 mL de SSN I.V. durante 20 minutos. Después infusión de 500 mg-h hasta que el
paciente no presente síntomas o sea extubado.

Efectos agudos de los organofosforados (OF):

ANTIBIOTICOS PROFETAS
1) Tx de infección de vías urinarias en el embarazo
a) Amoxicilina 500 mg tid por 7 días
b) Norfloxacina 400 mg bid por 7 días c) Ciprofloxacina 500 mg bid por 7 días d) Tetraciclina 500 mg tid por 7 días
e)
Ninguna de los anteriores

2) Uno de los siguientes antibióticos es bacteriostático


a) Cloranfenicol
b) Cefalosporinas
c) Quinolonas

d) Aminoglucosidos
e) Polimixinas

3) Uno de los siguientes antibioticos no requieren de ajuste en falla renal:


a)Aminoglicosidos

b)Lincomicinas

c)Aminopenicilinas
d)Cefalosporinas
e) Ampicilinas

4)Uno de los siguientes antibioticos no actúa contra las Pseudomonas



a)Ciprofloxacina( Ppt el Dr. Torres slide 18. tiene act. Intermedia pero Cirpo es la que tiene mayor act)

b)Piperacilina

c)Carbenicilina

d)Meropenem

e)Bacampicilina( Es un derivado de ampicilina)

5) La ampicilina o amoxicilina tiene igual espectro que la penicilina G pero cambia en que actúa mejor sobre
algunos Gram negativos

a)Sentencia correcta, complemento correcto (Ppt del Dr. Torres slide 6)

b) Sentencia correcta, complemento falso
c) Sentencia falsa, complemento correcto d)Sentencia falsa, complemento falso
e) Ninguna de las anteriores

6) La carbenicilina no debe usarse en infecciones por:


a) Pseudomona aeruginosa

b) especies de Proteus con excepción de P. mirabilis
c) Algunas cepas de Enterobacter S
d. klebsiella
e) Anaerobios

7) Las penicilinas resistentes a penicilinasa son activa contra excepto:


a) Moraxella Catharralis

b) Entorococo

c)Leptospira
d)Neisseria Gonorreae
e)H. Influenzae

8)Es una cefalosporina de segunda generación:


a) Cefadroxil

b) Cefalexina

c)Ceftazidima
d)Ceftibuten
e) Cefoxitina

9) Los aminoglicosidos son nefrotoxicos y penetran bien el LCR



a) Sentencia correcta, complemento correcto

b)Sentencia correcta, complemento falso (no penetra bien LCR ni próstata)
c)Sentencia falsa, complemento correcto
e)Sentencia falsa, complemento falso

10) Los siguientes aminoglucosidos actúan sobre pseudomonas: Amikacina


a) Kanamicina

b) Neomicina

c) Estreptomicina
d)Garamicina
e) Ninguno de los anteriores

11) La lincomicina tiene un espectro igual a:


a)Penicilina

b)Clindamicina

c)Cefalosporina
d)Meropenem
e)Ciprofloxacina

12)Las tetraciclinas no se deben utilizar en:


a) Gonorrea
b) Ricketsias

c) Vibrio Colera

d) Granuloma inguinal
e) Clamidia

13)Una de las complicaciones importantes del cloranfenicol


a) colitis

b)colestasis

c)Anemia Aplasica
d)Fibrosis Pulmonar
e)Rinitis

14) El metronidazol no debe usarse en:


a)Giardiasis

b)Abcesos Pulmonares

c)Colitis Pseudomembranosa
d)Amebiasis
e)Tricomoniasis

15)La rifampicina:

a)Actuá sobre el ADN: ARN polimerasa
b)Es bacteriostático

c)No atraviesa la barrera hematoencefalica
d)Dosis es de 5mg/kg/peso
e)
Se elimina principalmente por bilis

16) Una de las siguientes drogas para el tratamiento para el TBC se metaboliza por acetilación

a)Isoniacida

b)Rifampicina
c)Etambutol
d)Rifampicina
e)Estreptomicina

17) Cuadro infeccioso que usted requiere de un antibiótico de amplio espectro pero el paciente ha tenido
antecedente de haber convulsionado evitaría:

a)Piperacilina con tazobactam

b)Ertapenem
c)Levofloxacina
d)Meropenem e)Cefotaxima

18) Los siguientes son antibióticos que se eliminan por el riñón, excepto:
a)Metronidazol
b)Aminoglucosido
c)Cefalosporina
d)Vancomicina
e)Fluorquinolonas

19) Uno de los fármacos no actúa sobre el Staphylococcus aureus coagulasa positivo:
a) Rifampicina

b) Ampicilina Sulbactam

c) Tetraciclina
d) Ticarcilina clavulonato
e) Linezolide; NO

MARZO 2015-2014

1) Uno de los siguientes antibióticos no es efectivo para anaerobios:


a) Moxifloxacino
b)Ceftazidina
c)Imipenem
d)Metronidazole
e) Piperazina

2) La teicoplanina es un ab del grupo de : Glucopéptidos

3)Las penicilinas tiene las caracteristicas que son buenas para anaerobios pero no para clostridium difficile:
Sentencia correcta, complemento correcto

4)Problema electrolitico que se tiene que vigilar en los pacientes que toman Anfotericina B:
a)Hipokalemia
b)Hiperkalemia

c)Hipocalcemia

d)Hipercalcemia
e hipermagnesemia

5)Ab que más frecuentemente produce fiebre:


a)Penicilina
b)Quinolonas
c)Eritromicina
d)Carbapenem
e)Fosfomicina

6)La vancomicina es un ab activo contra:


a)Proteus

b)Clostridium difficile

c)E. coli
d)Klebssiela
e)Acinetobacter

7) Cuál de los siguientes ab es el que tiene menor difusión al LCR:


a) Fosfomicina
b) Metronidazole
c) cotrimaxole

d) Penicilina
e)Cefotaxima

8)Los siguientes Ab producen mielosupresión o inmunodepresiónpor lo cual hay que


vigilar con un hemograma, señale aquel que no requiere de esta vigilancia:
a)Eritromicina
b)Cloranfenicol: Anemia Aplasica
c)Ceftriaxona
d)Cotrimoxazol
e) azitromicina

9)Señale el antimicrobiano que no se elimina por vía hepatica


a)Cloranfenicol

b)Clindamicina

c)Cefalosporina
d)Rifampicina
e)Doxiciclina

10) Los betalacatamicos pueden usarse en todos los trimestres de embarazo, mientras las sulfas están
contraindicadas el primer trimestre:
Sentencia correcta, complemento falso ( HARRISON PAG 1142, CUADRO 133-5)( Sulfas estan contraindicadas al
término del embarazo)

13)La garamicina se usa junto con la penicilina en:


a)Meningitis

b)Endocarditis

c)Neumonia: NO
d)Abceso Abdominal
e) Celulitis

14) Uno de los siguientes Ab no es efectivo para Anaerobios: Piperacilina

15) Penicilina contra Pseudomona: Piperacilina

16) La vancomicina es un Ab activo contra:


a) Proteus

b)Enterococos

c)E.coli
d)Klebsiella
e)Acinetobacter

17)Cuál de los siguientes ab es el que tiene menor difusión al LCR:


a)Garamicina
b)Metronidazole
c)Cotrimoxazole
d)Penicilina
e)Cefotaxima

18) Meningitis bacteriana tx de elección:


Cefalosporina de tercera generación

1. Tratamiento en endocarditis sub-aguda:


a)Vancomicina + Gentamicina
b)Rifampicina + Gentamicina
c)Cloramfenicol + Ampicilina
d)PNC GSC + Estreptomicina
e)Trimetropín sulfa + Rifampicina

Tratamiento neumonía comunitaria en paciente ambulatorio; excepto:


a)Azitromicina

b)Claritromicina

c)Doxiciclina
d)Carbepenem

Tratamineto de neumonía comunitaria en paciente hozpitalizado; excepto:


a)Ceftriaxona + Macrólido

b)Doxiciclina + Macrólido

c)Fluoroquinolona
d)Carbepenem

Tratamineto de neumonía comunitaria en paciente que por la gravedad del caso requiere UCI:
Fluroquinolona+ ceftriaxone
Fluoroquinolona + cefotaxima

Tratamiento de neumonía hospitalaria:


Floroquinolona + Carbepenem

Y en sospecha de S. Aureus, agregar...


Vancomicina

Antifímico que no debe ser administrado en el embarazo:


Estreptomicina

Dosis de Rifampicina en Tuberculosis:


600mg VO c/ 24h

Dosis de Etambutol en Tuberculosis:


15 – 25mg/Kg

Anti tuberculoso que produce neuritis óptica:


Etambutol
Anti tuberculoso que produce nefrotoxicidad, tiñe los líquidos corporales color naranja y se usa en el
tratamiento de la BCG “geitis”:
Rifampicina

Anti tuberculoso que produce neuritis periférica, hepatotoxicidad y anemia sideroblástica:


Isoniacida

Único antibiótico usado en TBC con poder bacteriostático:


Etambutol

Antibiotico cuyo mecanismo de acción es fijarse a la polimerasa del RNA bacteriano:


Rifampicina

Debido a la emergencia de cepas de estreptococo pneumoniae resistente, la terapia empírica en meningitis debe
iniciarse a base de:

a)Penicilina + Gentamicina

b)Doxiciclina + Rifampicina
c)Cloramfenicol + Ampicilina
d)Tazobactam + Piperazilina
e)Cefalosporina 3ra gen. + Vancomicina

Tratamiento de elección en meningitis por Listeria monocitógenes:


a)Penicilina

b)Ampicilina

c)Macrólidos
d)Vancomicina
e)Teicoplanina

Tratamiento de elección en Meningitis por N. Menigitidis no resistente:


a)PNC GSC en dosis altas
b)Ampicilina- Sulbactam
c)Ceftriaxona
d)Cefotaxima
e)Vancomicina

Primera elección para meningitis en R. Nacidos:


a)Ampicilina + Gentamicina

b)Vancomicina + Rifampicina

c)PNC GSC en dosis altas
d)Cefotaxima + Ampicilina
e)TMP/ SMT + Ampicilina

Tratamiento empírico en absceso cerebral secundario a extracción dental:


a)PNC GSC + Metronidazol
b)Cloramfenicol + Ampicilina
c)Tazobactam + Piperazilina
d)TMP/ SM + Gentamicina
e)Ampicilina + Gentamicina


Tratamiento en artritis gonocócica una vez demostrado que existe resistencia:


Ceftriaxona

8.La penicilina actua contra los siguientes microbios excepto:


a.neumococo

b.serratia

c. listeria
d.clostridium
45.En la enfermedad por legionella el tratamiento de eleccion es:
a. Macrolidos
b.Tetraciclinas
c.Cefalosporinas
d.Aminoglicosidos
e.Fosfomicinas

26) Paciente con neumonía adquirida en la comunidad, se aislan bacilos gram +, anaerobios. Lo puedo tratar con
monoterapia con:

a) ertapenem

26.-29. Nombre 4 floxacina que conozca:

• levofloxacina
• neofloxacina
• ofloxacina
• ciprofloxacina

30-33.efecto secundario principal de los sig antibioticos:


a. rifampicina : hepatotoxico
b.imipenem: convulsión
c. anfotericina B: hipokalemia
d.ketoconazol: cefalea, mareo, fotofobia, dif hepatica,
e.fluconazol: alopecia, nausea vomito

34.antimicrobiano usado en cualquier trimestre del embarazo:


R/ betalactamico

37.profilaxis para contactos de px con meningococo, se realiza con rifampicina


o: R/ Rifampicina es el de elección pero como alternativa se puede utilizar azitromicina o ceftriaxone.

48. Hombre de 20 años con IVU en urocultivo se aisla cepa sensible de proteus mirabilis antibiótico menos
apropiado para tx:

R/ nitrofurantoina

49.Inhibe la síntesis de proteinas:


R/ Aminoglucosidos

1. Un paciente masculino de 56 años es admitido con un cuadro de diverticulitis del colon, Se le encuentran una
colección de pus en la cavidad peritoneal de la cual se aislaron germenes gram negativos, gram positivos y anaerobios.
Tomando en cuenta que esta infección surgió con el paciente en la comunidad (y no es una infección nosocomial), ¿Cuál
de los siguientes antibioticos usados en monoterapia sería el mas indicado?
a.Gentamicina
b.Aztreonam
c.Cloramfenicol
d.Ertapenem
e.Penicilina Sódica

2. Por las características farmacocinéticas los aminoglicosidos no deben emplearse por vía oral salvo en ¿cuál
de las siguientes situaciones?
a.Infección de Vías urinarias

b.Infección de vías biliares

c.Recontaminación del tracto gastrointestinal
d.Infecciones Dermatológicas
e.Buscando el sinergismo con Vancomicina

3. En un paciente con Insuficiencia Renal moderada, antecedentes de convulsiones y una infección nosocomial
por gérmenes multiresistentes ¿Cuál de los siguientes antibióticos usted evitaria para disminuir el riesgo teorico
de convulsiones?

a.Piperacilina con Tazobactam
b.Ertapenem
c.Imipenem
d.Meropenem e.Gentamicina

4. En cuál de las siguientes situaciones clínicas consideraría usted razonable el uso de cloranfenicol
especialmente ante la falla de un betaláctamico.

a.Infeccion Urinaria por Gram negativos

b.Infeccion del Sistema Nervioso Central por un anaerobio
c.Endocarditis bacteriana por Staphylococus metilcilino resistentes
d.Meningitis por Cryptococus neoformans
e.En combinación con penicilina para buscar sinergismo

21. Cual de las siguientes quinolonas tiene mayor cobertura contra el streptococos pneumoniae?

a.ciprofloaxina

b.levofloxacina
c.pefloxacina
d.norfloxacina

e.ninguna de las anteriores

23. De los siguientes antibioticos cual es seguro durante el embarazo?


a.Tetraciclina

b.Vancomicina
c.Amoxicilina
d.Garamicina

9. Paciente acude al cuarto de urgencias con diarrea, el azul de metileno es positivo por leucocitos y en las heces al dia
siguiente le informan que está creciendo una salmonella. Por este motivo cambia la terapia y decide administrarle:

a.clorafenicol
b.aminoglicosidos
c.penicilina
d.ciprofloxacina
e.Trimetropin con sulfa.

46.La fosfomicina es un antibiotico que ademas de las infecciones de piel tiene indicacion en infecciones:

a.intestinales

b.urinarias
c.vesiculares
d.respiratorias bajas
e.otitis por gram negativo

47.Los macrólidos son antibioticos:


a.Primera eleccion en faringoamigdalitis
b.Bacteriostaticos

c.Tienen cobertura antipseudomonal
d.Se usan en diverticulitis
e.Excelente como monoterapia en el helicobacter sp.

48.En el tratamiento de la meningitis por meningococo en paciente alergico a la penicilina y cefalosporina es:

a.clorafenicol

b.ceftriaxone
c.amikacina
d.fosfomicina
e.clindamicina

51. En caso de mordedura humana, cual de las siguientes indicaciones no tendrían relevancia:

a.Toxicoide tetánico

b.Amoxicilina con ácido clavulánico
c.Metronidazol

D. Curación de la herida
e. Penicilina sódica

83. Concepto falso sobre el metronidazol


a.Alternativa de tratamiento para la prostatitis
b.metabolismo hepatico c.antianaerobico
d.es un tratamiento para las tricomonas
e.penetra la barrera hematoencefalica

60.Indicacion para el uso de Vancomicina


a.Prostatitis por estafilococo
b.Colitis asociada a antibioticos
c.Síndrome del hombre rojo
d.Primera eleccion en las celulitis
e.Neumonía

85. Diferencia de la Teicoplanina en relación a la Vancomicina


a.Mayor actividad contra estafilococo

b.Mayor nefrotoxicidad

c.Uso intramuscular
d.Vida media similar
e.No existe diferencias

86. Para los inhibidores de beta lactamasa es cierto:


a.Excelente actividad antibacteriana

b.Todos cruzan la barrera hematoencefalica

c.Son efectivos contra todas las betalactamasas
d.No producen anafilaxia
e.Ninguna de las anteriores

7. De los siguientes antibióticos, cual tiene la mejor cobertura contra klebsiella:


a.Vancomicina

b.Streptograminas

c.Linezolid
d.Cefotaxima
e.Lincomicina

61. Las siguientes son cefalosporinas que actúan sobre las pseudomonas excepto:
a.ceftriaxona
b.cefepima

c.cefpirome

d.ceftazidima

e. Ninguna de las anteriores

34 Tratamiento de Pneumocystis Carinii:


TMP-SMX

37 Penicilina sigue siendo efectiva para: meningococo y neumococo.


64 Cuál de los siguientes antibióticos tiene mejor cobertura contra klebsiella
cefotaxima

71 Cuál de estos no es betalactámico


Vancomicina
¿Cuál de los siguientes antibióticos es el que tiene menor difusión al LCR?
a) Clindamicina ***
b) Metronidazol
c) Cotromoxazol
d) Penicilina

e) Cefotaxima

¿Cuál de los siguientes antibióticos es el menos eficaz para el tratamiento de la uretritis gonocócica? (Todas son
muy utilizadas para N. gonorreae)

a)Ceftriaxona 500mg IM (125mg)

b)Cefixima 400mg oral en una sola dosis
c)Azitromicina 1 g una dosis

d)Ciprofloxacina 1 g en una dosis (500mg) Por la elevada resistencia.
e)Cefuroxina 750 mg en una sola dosis

¿Cuántos días debe prolongarse el tratamiento antibiótico eficaz de una meningitis meningococcica?

a)1-3

b)4-7
c)8-10
d)12-15
e)18-21

En el tratamiento con metronidazol del absceso amebiano hepático usted considera que puede tratarse de un
absceso bacteriano sino hay respuesta
al tratamiento a:
a)24 horas

b)48 horas

c)72 horas
d)5 dias
e)7 dias

El tratamiento específico del granuloma inguinal es:


a)Ceftriaxone IM

b)Azitromicina una sola dosi
s
c)Eritromicina
d)Tetraciclina
e)Cefepime

Tratamiento de la sífilis secundaria en una paciente embarazada es:


a)Tetraciclina 250mg VO qid

b)Azitromicina 1 gramo cada dia por 3 dias

c)Penicilina desensibilizacon previa
d)Doxiciclina 100mg VO bid por 14 días
e)Claritromicina 500 mg dos veces al día

El tratamiento actual más seguro para salmonella es:


a)Cloranfenicol

b)Aminoglicosidos

c)Penicilina
d)Ciprofloxacina
e)Tetraciclinas

En un paciente que llegue con ictericia de tipo colestasica y está recibiendo tratamiento para TBC usted piensa que la
droga que le produce esto es
:
a)Isoniacida

b)Rifampicina
c)Etambutol
d)Pirazinamida
e)Estreptomicina

12. Pirazinamida

a. Procede del ácido pirrolico
b.Actúa mejor a un pH ácido
c.Hepatotoxico

d. Bacteriostático

e.No atraviesa la BHE

Mencione 3 antibioticos que tengan metabolismo hepatico: isoniacida, doxiciclina, amoxicilina.

Mencione 3 antibioticos que se pueden usar en el embarazo: amipicilina, penicilina, amoxicilina.

Tratamiento de ricketzias
a.doxiciclina

47. Tratamiento de leptospira


a.doxiciclina

b.amoxicilina

c.ampicilina
d.penicilina
e.vancomicina


49. La carbenicilina no debe de usarse en infeccion por:


a. Pseudomona Aeruginosa

b. Especies de Proteus, con excepcion de P. Mirabilis
c. Algunas cepas de enterobacter
d.
d. Klebsiella spp y algunas especies de Serratia

e. Anaerobios

50. Las carboxipenicilinas y las ureidopenicilina difieren de la ampicilina en su mayor eficacia contra muchos:
a. Gram –
b. Anaerobios
c. Gram+

d. Micoplasma
e. No difieren

Pareo
3. Nitrofurantoina----- al final del embarazo produce hemolisis
4. Aminoglicosidos----ototoxicidad y nefrotoxicidad
5. Sulfamidas----- icteria al final del embarazo
6. Trimetropin----- acción antifolica en el primer trimestre
7. Quinolonas----alteración a nivel del cartílago óseo del feto

10. Antibiótico que penetra adecuadamente la próstata:


a. trimetropin

b.cotrimoxazol (Tx para prostatitis bacteriana aguda)

c. garamicina
d. penicilina
e. cefepime

25. Paciente con fiebre de 39° diarrea sanguinolenta más de una semana, positivo para C. yeyuni, fármaco de
elección:
•azitromicina o amoxicilina
De eleccion es eritromicina, pero azitromicina y las fluoroquinolonas se pueden utilizar

27. Indicación de aminoglicósidos por vía oral:


• descontaminación del tracto gastrointestinal

31. En un paciente diabetico con historia de convulsiones y con una infeccion nosocomial por germenes multi
resistentes, usted evitaria el uso de:

R. Imipenem

32. Uso del cloranfenicol, especialmente ante la falla de beta lactamicos:


R. Infección del SNC por anaerobios.

39- Tratamiento de elección del actinomices:


a- Metronidazol

b- Penicilina sodica

c- clindamicina
d- tetraciclina
e- Quinolona

3)Imipenem se da con cilastatina para: inhibir el metabolismo renal para mantener concentraciones altas de
imipenem:

a) La cilastatina tiene sinergismo con él

b) La cilastatina lo ayuda en su metabolismo hepático. Es para la difuncion renal

c) La cilastatina inhibe la B-lactamasa para que imipenem funcione mejor. Cilastina inhibe enzima dihidropeptidasa en
el riñon para la degradación del imipenem
d) N/A

30-33.efecto secundario principal de los sig antibioticos:


a rifampicina : hepatotoxico
b.imipenenm: convulsión
c. anfotericina B: hipokalemia
d. ketoconazol: cefalea, mareo, fotofobia, dif hepatica,
e.fluconazol: alopecia, nausea vomito

36. No es una condicion de uso de antimicrobiano parenteral:


R/ pobre penetración al sistema nervioso central

16. En que condiciones clinica debemos considerar la clindamicina como parte del tratamiento del paciente
neurocronico

a. Frotis de esputo con cocos Gram negativo

b. Falta de respuesta a la cefalosporina
c. Historia de convulsions

d. Frotis de esputo con cocos gram positivos
e. Si ademas de sospecha una meningitis

23. De los siguientes antibioticos cual es seguro durante el embarazo?


a. Tetraciclina

b. Vancomicina

c. Amoxicilina
d. Garamicina

La duracion minima del tratamiento de la TBC cuando no se asocian rifampicina o la isoniacida es de:

 6 meses
.9 meses
12 meses
18 meses
24 meses

Dosis de los fármacos para la TBC (por Kg de peso)


o Izoniacida: 5 mg
o Rifampicina: 10 mg
o Etambutol: 15 a 25 mg
o Pirazinamida: 25 a 30 mg
Pareo:

o Pirazinamida: hiperurecemia y artralgia


o Etambutol: neuritis óptica
o Anfotericina B: hipokalemia
o Izoniacida: hepatitis y colestasis
o Rifampicina: compromiso hepático

4. Un hombre de 45 años acudió al área de Urgencias de un hospital por fiebre elevada y exantema máculo-papuloso
generalizado, incluyendo palmas y plantas. El paciente vive en el campo con perros frecuentemente parasitados por
garrapatas. Señale la enfermedad a la que se refiere, el germen causante y el tratamiento adecuado:
1. Fiebre botonosa, Ricckettsia Conori: Doxiciclina.
2. Kala-azar, Leishmaniae Donovani: Antimoniales.

3. Dengue, Arenaviridae Aegypti: Tratamiento sintomático.
4. Fiebre Q, Coxiella Burnetti, Doxiciclina.

5. Fiebre de Malta, Brucella Mellitensis: Cotrimoxazol.

14. Una joven de 18 años acude al hospital por fiebre y cefalea de varias horas de evolución. Los días previos había
notado dolor de garganta y tos. Se observó tendencia al sueño, rigidez de nuca y petequias en conjuntivitis y
extremidades. El LCR era turbio y contenía 36.000 leucocitos /mm3, 200 mg/dL de proteínas y 20 mg/dL de glucosa
(glucemia simultánea 120 mg/dL). El examen con Gram fue negativo. Señale cuál de las siguientes afirmaciones NO es
correcta:
1. El diagnóstico más probable es meningitis meningocócica.
2. Los meningococos son siempre sensibles a la penicilina y, por tanto, la penicilina G es el
tratamiento de elección.
3. Como un significativo porcentaje de meningococos del grupo C son resistentes a la penicilina, la cefotaxima es un
tratamiento más seguro.

4. El empleo de dexametasona reduce el riesgo de secuelas neurosensoriales en niños con meningitis bacteriana, pero
su uso en un caso como éste es cuestionable.

5. Se debe administrar rifampicina a las personas que conviven estrechamente con la paciente.

2. Un enfermo con neumonía, perteneciente a un brote epidémico de varias personas que ocasionalmente
conviven en un edificio, presenta un cuadro confusional desproporcionado a la fiebre, diarrea, hiponatremia y
ligero ascenso de las enzimas hepáticas. Entre las siguientes opciones, ¿cuál se debe incluir en el tratamiento?:
Cefalosporinas y aminoglucósidos a las dosis adecuadas.
Vancomicina a las dosis adecuadas.
Doxiciclina 100 mg/12 h durante 14 días.

Eritromicina 2-4 g/día durante 14 días.
Penicilina G procaína 0,6 – 1,2x106 U/12 h durante 7 días.

11. Paciente de 64 años, fumador, que acude a urgencias por un cuadro de 48 h. de evolución de fiebre y tos con
expectoración mucopurulenta. La radiografía de tórax muestra una condensación alveolar en lóbulo inferior derecho y un
pequeño infiltrado en el lóbulo inferior izquierdo. La gasometría arterial muestra un pH de 7,39, una pO2 de 54 mmHg y
una
pCO2 de 29 mmHg. ¿Cuál de las siguientes opciones terapéuticas le parece más adecuada?:

Claritromicina 500 mg IV/12h.



Iprofloxacina 200 mg IV/12h
.
Amoxicilina-Ácido clavulánico 1g IV/8h.
Ciprofloxacino 200 mg EV/12h + Claritromicina 500 mg IV/12h.
Ceftriaxona 2g IV/24h + Claritromicina 500 mg IV/12h.

24. El antibiótico de elección en el tratamiento de una neumonía producida por Legionella pneumophila es

Eritromicina.

Penicilina G.
Cotrimoxazol.
Cefotaxima.
Clindamicina.

16. ¿Cuál de los siguientes tratamientos antibióticos no sería suficiente en monoterapia en el manejo del
paciente oncológico en tratamiento quimioterápico con neutropenia febril?:
1. Cefepima.

2. Ceftazidima.
3. Meropenem.

4. Imipenem.

5. Piperacilina-tazobactam.

31. ¿Cuál de los siguientes antimicrobianos utilizaría en primer lugar en monoterapia, para tratar una infección
grave por Staphylococcus Aureus resistente a meticilina?:

1. Eritromicina.

2. Rifampicina.
3. Vancomicina.
4. Levofloxacino.
5. Linezolide.

10. La penicilina G sigue siendo efectiva contra unas de estas bacterias, menciónela:
a. Actinomises israeli

b. Neisseria gonorroheae

c. Treponema pallidum
d. Legionella

11. Medicamento que no se debe modificar en el tratamiento de la insuficiencia renal:


a. Penicilina

b. Amoxicilina

c. Doxicilina
d.Tetraciclina
e.Clindamicina
f. Meropenem

12. Medicamento que se debe modificar en un paciente con insuficiencia hepática:


a. Gentamicina
b. Vancomicina
c. Cefuroxima

d. Rifampicina

e. Ciprofloxacino

13. Neumonía adquirida en la comunidad:


● Cefalosporina de tercera generación

14.Colecistitis aguda: Imipenem


16. Uretritis no gonocócica: Doxiciclina


17. Cefalosporina de 1ra generación de uso parenteral:
a. Cefalotina im/iv
b. Cefradina vo/im

c. Cefaclor – 2nda gen
d. Cefadroxina

19. Antibiótico bacteriostático que trabaja junto con PABA:


● Sulfonamida
● Cefuroxima-acetil

20. Si un paciente es alérgico a la penicilina en que porcentaje sería alérgico a las cefalosporinas de 3ra
generación

● 10 %

43. Cuando se administran aminoglucósidos observar por:


● Falla renal

44. Paciente con hematuria, proteinuria, rash cutáneo y nefritis intersticial. Cuál medicamento lo produce:

● Penicilina y sus derivados (Meticilina)

PROFETON REUMA

• El síndrome antifosfolipido secundario asocia más a


Esclerosis sistémica
A. Lupus eritematoso sistémico
B. Artritis reumatoide
C. Síndrome de sjogren
D. Artritis reactiva

29. Para el lupus eritematoso sistémico todo es cierto menos una:z


a. En un 20% de los pacientes tiene una prueba positiva para sífilis
b. Puede asociarse al anticoagulante lúpico
c. La cuenta de leucocitos generalmente es normal
d. La eritrosedimentacion es elevada cuando hay actividad
e. Los pacientes pueden presentar anemia hemolítica

• En el síndrome de antifosfolipido de la embarazada el tratamiento es con heparina de bajo peso y se evita la


aspira por su efecto teratógenico
A. Primera parte cierta y segunda parte cierta
B. Primera parte cierta y segunda parte falsa
C. Primera parte falsa y segunda parte cierta
D. Primera parte falsa y segunda parte falsa

• Diagnostico de SAF es necesario: 1 criterio clínico y 1 un criterio laboratorio

Criterio clínico: trombosis venosa arterial a repetición, petequias y perdidas de embarazo


Laboratorios: anticardiolipina

• La warfarina en el SAF después del primer episodio: se usa de por vida

• Manifestación clínica hematología del SAF es: TROMBOCITOPENIA

• Manifestación clínica NEOPLASICA EN SAF: Linfoma

• Son manifestaciones clínicas de SAF: tromboembolismo, accidentes cerebrovasculares, livedo reticularis, abortos.
EXCEPTO VASCULITIS
• En el SAF la causa secundaria es mas frecuente pero la causa primaria es también menos frecuente: la primera
parte es cierta y la segunda parte es cieta

• Otra de las causas secundarias de SAF después de las hemorragias son : infecciones

• Parámetro más común de laboratorio en el SAF: trombocitopenia

• Todo es cierto en cuanto al SAF en las manifestaciones clínicas: livedo reticularis, valvulopatía, trombocitopenia

• Usted sabe que el LES es:


a. Una enfermedad degenerativa propia del envejecimiento. MENOS FRECUENTE EN MAYORES DE 65 AÑOS Y EN
LA INFANCIA
b. Una enfermedad autoinmune que afecta sobre todo a mujeres
c. Una artropatía inflamatoria potencialmente deforme. ES SIMETRICA NO DEFORME
d. Todas las anteriores.
• Acerca de factores genéticos que contribuyen en la fisiopatología de LES usted sabe:
a. Las deficiencias en C2, C4 y C1q son rara pero confieren alto riesgo de LES
b. Polimorfismos de un nucleótido en genes que codifican proteínas implicadas en la inducción de IFN tipo I.
c. La presencia de HLA B27 confiere alto riesgo de padecer LES. ES HLA DR2 y DR3
d. a y b son ciertas.

• Acerca del predominio del LES en mujeres se sabe:


a. El predominio en mujeres esta dado por una relación 3 a 1 → 9:1
b. Los estrógenos pueden modular la activación de linfocitos → y prolactina y se expresa en niveles aumentados de
suero en los pacientes con les
c. Prolactina se expresa en niveles aumentados en pacientes con LES
d. b y c son ciertas.

• Entre los factores ambientales implicados en la ocurrencia de LES están:


a. Virus de Ebstein Baar por la alta prevalencia de anticuerpos específicos contra este en niños con LES. Podría ser
esta también pero en las diapos dice asi: alta prevalencia de anticuerpos específicos para antígenos de EBV en
niños con LES
b. Uso de medicamentos como hidralacina, procainamida, inhibidores de factor de necrosis tumoral.
c. Consumo de carnes y productos derivados de la leche.--> es las diapos solo sale alfalfa sprouts and foods
containing canavanine ¿??? No sale nada de lacteos y carnes. NADA QUE VER
d. Todas las anteriores.

• Entre los criterios de 1982 para LES usted sabe que están: aquí también estudiar otros criterios pero poner la misma
pregunta pero con otros enunciados: ras discoide. Serosistis, etc
a. Rash malar
b. Ulceras orales
c. Artritis no erosiva
d. Todas las anteriores

• Usted sabe que la diferencia serológica entre: lupus inducido por medicamentos y LES es:
a. presencia de anticuerpos anti histonas en el 95% del inducido por medicamentos. En LES ES DE 70%
b. ausencia de anticuerpos anti DNA nativo en lupus inducido por medicamentos. LES es de 40%
c. Ausencia de anticuerpos anti SM en lupus inducido por medicamentos. LES es de 30%
d. Todas las anteriores.

• de la afectación renal del LES usted sabe que las clases o tipos de nefropatía lupica son todas excepto: faltan otras
q si son: glomerulonefritis proliferativa difusa y la nefritis intersticialy tubular.
a. Nefritis lupica mesangial
b. Glomerulonefritis focal leve
c. Glomerulonefritis inducida por IgA
d. Glomerulonefritis membranosa

• La siguiente es una droga que produce inhibidor lupico


a. Metildopa
b. Fenitoina
c. Isoniacida
d. Amiodarona

Farmacos que inducen lupus : Algunos de los más comunes son los antiarrítmicos procainamida, disopiramida y
propafenona; los antihipertensivos hidralazina, varios inhibidores de la enzima convertidora de angiotensina y los
antagonistas adrenérgicos β; el antitiroideo propiltiouracilo; los antipsicóticos cloropromazina y litio; los
anticonvulsivos carbamazepina y fenilhidantoína; los antimicrobianos isoniazida, minociclina y macrodantina; el
antirreumático sulfasalazina; el diurético hidroclorotiazida; los antihiperlipidémicos lovastatina y simvastatina; así
como los interferones biológicos y los inhibidores del factor de necrosis tumoral.

• son sindromes hipercoagulables excepto:


a. Embarazo
b. S. antifosfolipido
c. Hiperhomocisteina
d. Neoplasias
e. Nefritis

• Los anticuerpos antifosfolipidos producen sobre las plaquetas:


a. Aumento del numero
b. Aumento de la actividad
c. Aumento de su producción
d. Aumento del secuestro por el bazo.

• en el síndrome antifosfolipido el tratamiento en la embarazada para evitar aborto es con warfarina y en la trombosis
venosa profunda también.
a. Primera parte cierta y segunda cierta
b. Primera parte cierta y segunda parte falsa
c. Primera parte falsa y segunda parte cierta
d. Primera parte falsa y segunda parte falsa

• en la isquemia vascular cerebral en pacientes que viven alejados una alternativa en el tratamiento del síndrome
antifosfolipidos es la aspirina y en estos pacientes el problema es que la recaida es de más del 50% :
a. Primera parte cierta y segunda parte cierta
b. Primera parte cierta y segunda parte falsa
c. Primera parte falsa y segunda parte cierta
d. Primera parte falsa y segunda parte falsa

• Las manifestaciones más frecuentes en el lupus eritematoso sistémico son


A. Renales
B. Neuropsiquiatricas
C. Cardiacas
D. Musculoarticulares

• son síndrome hipercoagulable excepto


A. embarazo
B. sepsis
C. hiperhomocisteina
D. neoplasias

34. Cuál es la forma mas frecuente de presentación del LES


E. a. pericarditis
F. b. anemia
G. c. pleuritis
H. d. convulsiones
I. e. artritis o artralgia

35. Tratamiento de lupus eritematoso sistémico incluye lo siguiente

J. a. omatuximab
K. b. asovaquona
L. c. ciclofosfamida oral
M. d. suplíride

80. Manifestaciones hematológicas del lupus eritematoso incluye lo siguiente excepto


a. linfopenia
b. leucopenia
c. trombocitosis (TROMBOPENIA)
d. anemia hemolítica

99. Para monitorizar la actividad clínica del lupus eritematoso sistémico es útil a
a. la VSG y la cuenta de leucocitos
b. los anticuerpos antinucleares
c. la proteína c
d. las proteínas séricas
e. el C3

31. La artritis en el lupus eritematoso sistémico se caracteriza por


a. ser siempre deformante
b. ser idéntica a la reumatoidea
c. presentar artralgia
d. ser parecida a la espondilitis anquilosante
e. siempre simétrica

1. En el LES: EL ANA es positivo en 95% de los pacientes

Alopecia en LES : (HARRISON Y RITA DICEN 40%)

A. Se da en el 70% de los enfermos


B. Es muy rara
C. Se da en todos los enfermos
D. Solo se manifiesta en mujeres
E. Se da en un 25% de los enfermos

12. en el lupus medicamentoso son muy poco frecuentes


• Nefritis y SNC

13.Una pcte la cual se encuentra es estudio por artralgia, dentro de los lab le sale positivo el anticuerpo Anti-histona;
su preferencia Dx sería: lupus inducido por drogas

13) Es cierto para los anticuerpos antifosfolípidos:


a. El VDRL es falso negativo FALSO POSITIVO
b. TPT esta alargado generalmente
c. Test de Coombs es positivo
d. En la perdida fetales solo hay uno de los dos anticuerpos
e. La causa mas frecuente estadísticamente es la isquemia vascular cerebral
28) El tratamiento de elección en la embarazada con síndrome antifosfolípo es
a. Aspirina y heparina de bajo peso molecular b. Aspirina sola c. Warfarina
d. Heparina sódica e. No tiene tratamiento

33) Las manifestaciones neurológica más frecuente del lupus eritematosos sistémico son:

a. Ataxia cerebelosa b. Depresión y psicosis c. Hemiplejia y paraplejia


d. Polineuritis y mononeuritis múltiple e. Convulsiones

1. TRATAMIENTO DE SINDROME ANTIFOSFOLIPIDICO LO CIERTO:


A.Presencia de anticuerpos antifosfolipido en una embarzada sin antecedentes de trombosis o abortos .
B.La anticoagulacion se realiza en la actualidad con heparina de bajo peso molecular asi no se hacen controles.
C.La anticoagulacion manteniendo un INR alto(-3) es el tratamiento de eleccion en pacientes que ya han
tenido trombosis.
D.En episodios tromboticos emplear corticoides ademas de aspirina.

27.La artritis en LES se caracteriza por en una radiografia : No presenta erosiones

28) Paciente con Lupus , controlo la enfermedad midiendo: b) C3 y C4

55 Criterios de Lupus:

1. Una mujer de 28 años con anticuerpos anticardiolipina y antecedentes de tres abortos en el primer
trimestre del embarazo es evaluada en la sexta semana de un cuarto embarazo. El embarazo actual
transcurre con normalidad. Nunca ha sido tratada por la positividad de los anticuerpos anticardiolipina.
¿Cuál de las siguientes es la conducta más apropiada?:

1. Observación estrecha.
2. Prednisona.
3. Aspirina.
4. Infusión intravenosa de inmunoglobulinas.
5. Heparina y aspirina.
7. Enferma de 32 años que cuando acude a la consulta refiere que hace unos 20 días, después de una
exposición solar, le aparece en la zona externa, hombros, brazos y región escapular, unas lesiones anulares,
eritematoedematosas en su borde y con regresión central, algunas confluentes de dos o tres centímetros de
diámetro que apenas le ocasionan molestias. El diagnóstico sería:

1. Eritema polimorfo.
2. Liquen plano.
3. Porfiria hepatocutánea.
4. Lupus eritematoso cutáneo subagudo.
5. Dermatomiositis.

12. ¿Cuál de los siguientes confirma el diagnóstico de lupus eritematoso sistémico?

1. Unos anticuerpos antinucleares positivos.


2. Una biopsia renal demostrando una glomerulonefritis proliferativa.
3. Unos niveles bajos de complemento sérico y aftosis oral.
4. La presencia de artritis no deformante, fotosensibilidad, leucopenia y pleuritis.
5. Alopecia reciente.

37. ¿Cuál de las siguientes determinaciones de anticuerpos es, de resultar positiva, más específica de lupus
eritematoso sistémico?:

1. Antinucleares.
2. Anti-RNP.
3. Anti-SSA.
4. Anti-SSB.
5. Anti-Sm.

37. Respecto al tratamiento del Lupus eritematoso sistémico señale, entre las siguientes, la respuesta
INCORRECTA:

1. Se deben utilizar glucocorticoides a dosis altas y fármacos citotóxicos en los casos de afectación orgánica severa
potencialmente reversible.
2. Los fármacos antipalúdicos son eficaces para el control de formas leves moderadas de la enfermedad, siendo
recomendable vigilar periódicamente la posible toxicidad retiniana.
3. En períodos de inactividad de la enfermedad es posible prescindir del tratamiento, aunque las remisiones completas
son raras.
4. En caso de insuficiencia renal terminal deben recibir tratamiento con diálisis, estando contraindicado el
transplante renal de cadáver.
5. Las complicaciones infecciosas son un riesgo importante en los pacientes que reciben tratamiento inmunosupresor.

22. En el lupus cutáneo subagudo es caracteríscica de la presencia de anticuerpos:

1. Anti-Sm.
2. Anti-centrómero.
3. Anti-Ro.
4. Anti Jo1.
5. Anti-histona.
6.
30 . Nombre dos enfermedades en la cual los anticuerpos antinucleares poder ser positiva en:

• LES
• Snd de sjogren

32. Usted sabe que es LES es:

• Una enfermedad degenerativa propia de la mujer en edad adulta (fertil)

33. Acerca de los factores genéticos que contribuyen en la fisiopatología de LES usted sabe que:
• Polimorfismos de un nucleótido en genes que codifican proteínas implicadas en la inducción de INF tipo I

34. Acerca del predominio de LES en mujeres se sabe:

• Los estrógenos pueden modular activación de linfocitos

6. Usted sabe que la PCR puede incrementarse en


Infecciones crónicas
IAM
Malignidad
Todas

7. Acerca de los valores de ácido úrico sérico usted sabe que


Valores por encima de 7mg/dl sugieren la posibilidad de Artritis gotosa
Valores normales descartan posibilidad de artritis gotosa
Valores elevados deben tratarse siempre
Todas

8. Del FR es cierto todo menos


Es positivo en el 5% de la población general
Su positividad indica AR
En el 50% de los casos de AR temprana es negativo
Es positivo en enfermedades del tejo conectivo como síndrome de sjogren

9. La prueba de anticuerpos antinucleares puede ser positiva en también en escleroderma polimiositis y


vasculitis
LES
AR
Síndrome de sjogren
Todas

Nefropatía lúpica más grave: LA MAS FRECUENTE Y MAS GRAVE


Tipo I (minima mesangial) b. Tipo II(proliferativa del mesangio) c. Tipo III(proliferativafocal) d.Tipo IV(
proliferativa difusa) e. tipo V membranosa

Un estudiante universitario de 24años de edad se queja de fatiga, artralgias, ulceraciones en la mucosa y


exantema facial (en mariposa) poco de manifiesto depósitos lineales de inmunoglubulina G (IgG) y de
complemento en la unión dermoepidermica. Que de lo siguiente es verdadero de acuerdo a esta
enfermedad?
Las mujeres son afectadas por igual que los varones
Los datos en la biopsia de piel suelen ser patognomonicos
El espécimen de biopsia de piel no afectada es normal
Los valores de complemento en suero disminuyen con la exacerbación de la enfermedad
La tasa de eritrosedimentacion (ESR) es una guía confiable de la actividad de la enfermedad

¿Cuál es la prueba mas especifica para esta enfermedad?


ANA (anticuerpos antinucleares)
Anti-Sm (textualmente del libro dice: en la alteración fisiopatológica +característica del LES es la presencia de
autoanticuerpos como ANA, anti-DNAds, y anti Sm son específicos.
Anti-RNP
Anti-Ro
Antihistona

En el Sd. de sjogren usted sabe que


Puede ocurrir en asociación con artritis reumatoide y cirrosis biliar primaria
La biopsia de glándulas salivales es necesaria para el diagnóstico en todos los casos
En la mayoría de los casos es secundario
Todas las anteriores

En cuanto a la prueba de hemograma en reutomologia usted sabe


Que anemia microcitica heterogénea es característica de la anemia hemolítica autoinmune
Que leucocitosis es característica del Sd. de Felty y el lupus eritematoso sistémico
Que trombocitos es característica de la inflamación sistémica
Ninguna de las anteriores

Que la química sanguínea nos sirven para monitorizar


Efectos secundarios hepáticos de metotrexate
Actividad renal de lupus eritematoso sistémico
Diagnosticar miopatías inflamatorias
Todas las anteriores

Usted sabe que elevación de velocidad de sedimentación globular se puede encontrar en


Embarazo
Altas dosis de esteroides
Policitemia
Ninguna de las anteriores

Señalar que circunstancias no exacerba la enfermedad de lupus eritematoso sistémico


Exposición a microonda
Exposición a radiación ultravioleta
Contraceptivo orales
Penicilinas
el fenómeno de Raynaud es más indicativo de
esclerosis sistémica
lupus eritematoso sistémico
artritis reumatoide
Sd. de sjogren

26. nombre tres indicaciones para realizar una densitrometrìa ( pregunta de llenar espacio, por eso lo pongo) :
Mujeres menores de 65 años de edad postmenopausicas.
Todas las mujeres mayores de 65 años de edad
Pacientes con tratamiento prolongado de corticoides.

38. En medio de consulta màs frecuente en el sìdrome de Sjogren es debido a


a. artritis
b. molestias oculares( QUERATOCOJUNTIVITIS)
c. molestìa orales
d. falta de sudoración
e. dispepsia
9. El anticuerpo anti membrana basal es característico de
a. Granulomatosis de Wegener
b. Poliarteritis Nodosa
c. Síndrome de Cogan
d. Enfermedad de Goodpasture parecida a la vasculitis de pequeños vasos pero en ves de anca+ , tiene anticuerpo-
antimembrana basal.

31. Síndrome de CREST:


a. Calcinosis, raynaud, trastorno esofágico, esclerodactilia y telangectasia.
b. Cristales, rigidez matutina, trastorno esofágico, sinovitis y telangectasia.
c. Colagenosis, Raynaud, esofagitis, serositis y telangectasias.
d. Vasculitis, Raynaud, esofagitis, serositis y telangectasia.

44. El inhibidor lúpico lo encontramos en, excepto:


a. Trombocitopenia inmune b. Arteritis temporal c. Neoplasias d. Nefritis
48. Son síndrome hipercoagulables, excepto:
a. Embarazo b. Hiperhomocisteína c. Anemia d. Neoplasias e. Enf. de Hughes

. En relación al Lupus Eritematoso Sistémico, indique cuál de las siguientes


afirmaciones es FALSA:
1.Es una enfermedad de claro predominio femenino.
2. La afectación renal es frecuente y es uno de los indicadores de agresividad de la enfermedad.
3. La biopsia renal es de particular utilidad dado que el tipo de afectación renal no varía con la evolución de la
enfermedad.
4. Los pacientes pueden presentar lesiones cutáneas en relación a la exposición solar.
5. Los autoanticuerpos más característicos son los antinucleares y los anti-ADN.

Señale cual de las siguientes afirmaciones es INCORRECTA respecto a los anticuerpos que pueden
encontrarse en lupus eritematoso generalizado.
Los antinucleares (ANA) aparecen en el 96- 98% de los pacientes, pero no son disgnósticos.
Los anti-Sm son específicos, pero solo aparecen en el 30% de los casos.
Los anti-Ro(SSA) se relacionan con la presencia de bloqueo cardiaco congénito en los hijos de madres afectadas.
Los antifosfolípidos están siempre presentes y se relacionan con la actividad de la enfermedad .

El rash malar es un Criterio para el lupus eritematoso y otro criterio es la psicosis :


La primera parte es cierta y segunda parte cierta.

Nombre dos medicamentos modificadores de la artritis reumatoidea : cloroquina y metrotexate.

Citar 3 enfermedades que producen espondiloartritis seronegativa :


Espondilitis anquilosante
Espondilitis juvenil
Artritis psoriasica
Sindrome de reiter
Artropatías entericas
En la Nefritis secundaria al LES, se suelen encontrar positivos los:
a. ANA.
b. ANCA c.
c. ANTI LU
d. ANCA p.

Asociación de artritis reumatoidea con esplenomegalia y leucopenia se conoce como:


Sindrome de Felty

Acerca del tx adyuvante en la nefropatía lupica usted sabe que :


Todos los px deben recibir corticosteroides.
Todos los px deben recibir hidroxiclororquina
Todos los px deben recibir estatinas
El bloque del eje renina- angiotensina aldosterona es indicado según las cifras de presión arterial.

7. Enferma de 32 años que cuando acude a la consulta refiere que hace unos 20 días, después de una
exposición solar, le aparece en la zona externa, hombros, brazos y región escapular, unas lesiones anulares,
eritematoedematosas en su borde y con regresión central, algunas confluentes de dos o tres centímetros de
diámetro que apenas le ocasionan molestias. El diagnóstico sería:
1. Eritema polimorfo.
2. Liquen plano.
3. Porfiria hepatocutánea.
4. Lupus eritematoso cutáneo su agudo.
5. Dermatomiositis.

El hecho más importante en relación al pronóstico del enfermo lúpico es la:


Pericarditis
Nefropatía (relacionado con mal pronostico)
Marcados de LES: Anti-DNA y anti-smith

Si una enferma con lupus eritematoso diseminado presenta un infiltrado pulmonar, lo más probable es que
se trate de:
Neumonitis intersticial con fibrosis.
Neumonitis aguda lúpica.
Edema pulmonar.
Hemorragias intraalveolares.
Infección pulmonar (La infección pulmonar es la causa más frecuente de infiltrados pulmonares en el LES).

Señale en cuál de las siguientes manifestaciones clínicas del lupus eritematoso sistémico NO está indicado
el tratamiento con corticosteroides:
Anemia hemolítica.
Glomerulonefritis.
Artritis aislada( cuando no responde a aines ni antipaludicos esta indicado glucocorticoides)
Miositis.
Trombopenia importante.

Necrosis avascular de la cadera con complicación: LES.


La necrosis avascular de la cabeza femoral y humeral puede estar presente en el 30% de los casos, relacionada, en
la mayoría de las ocasiones, con la administración de glucocorticoides.

En el lupus medicamentoso son muy poco frecuentes


o Manif renales y snc.

En la evaluación de las enfermedades reumáticas es cierto que:


los niveles de PCR SSSEEEE correlacionan con la progresión de las lesiones radiológicas articulares.

1. en relación con la determinacion de reactante de fase aguda es cierto que


a. la ESD(eritrosedimentacion) refleja de forma as rápida que la PCR los cambios en la respuesta de fase aguda
b. la cuantificación en unidad de la pcr no es de utilidad clínica, ya que no existe correlacion entre sus niveles y
determinados grupos de patología inflamatoria.
c. la esd depende en gran parte de la contraccion plasmatica de fibrinógeno.
d. la esd de 40 mm/h en una persona anciana indica siempre una patología inflamatoria subyacente.

2. en la evaluación de las enfermedades reumaticas es cierto que


a. los niveles de pcr no se correlacion con la progresión de las lesiones radiológicas articulares.
b. la pcr se eleva habitualmente en los enfermos con les activo
c. una elevación importante de la pcr en un enfermo con les sin sinovitis ni serositis sugiere una complicación por
una infección bacteriana.
d. La ferritina normal en un paciente con ar actiav y anemia descarta déficit de hierro.

OJO ESTA NO ES LA MISMA QUE LA ANTERIOR (2), CUIDADO QUE NOS AGARRAN: los niveles de PCR SE
correlacionan con la progresión de las lesiones radiológicas articulares.

7. De la química sanguínea nos sirve para monitorizar efectos secundarios hepáticos de metotrexate y también
nos ayuda en la activdad renal de lupus eritematoso sistémico

a. sentencia cierta y complemento cierto


b. sentencia cierta y complemento falso.
c. sentencia falsa y complemento cierto
d. sentencia falsa y complemento falso

8. usted sabe que la proteína c reactiva puede incrementarse en infecciones cronicas y además puede
incrementarse en la polimialgia reumatica
a. sentecia cierta y complemento cierto
b. sentica cierta y complmeento falso.
c. sentecia falsa y complmento cierto (no es esta)
d. sentecia falsa y compemlento falso

el siguiente de un paciente con lupus eritematoso sistémico es con C3-C4 RECORDAR

TOXICO
1- Efecto de reversión enzimática de la pralidoxima
- tiempo:___hasta que envejezca la acetilcolinesterasa. De 60 min a semanas. _____
- dosis: ADULTOS Y > 12 AÑOS: 1-2 g DILUÍDOS EN 100-200 ml DE SUERO GLUCOSADO AL 5% O SSN 0.9 % . PASAR
EN INFUSIÓN NO MAYOR DE 0.2 GRAMOS POR MIN (10-20 ml/min.)
< DE 12 AÑOS: 20-40 mg/Kg de PESO, DILUÍDO EN SOLUCIÓN A RAZÓN DE 10 ml/Kg.

2- Efectos Agudos de los OF: bradicardia, hipotension, aumento de las secreciones, alteraciones del estado (fatiga, debilidad muscular)

Efectos intermedios (48h): afectacion de la faringe, paralisis respiratoria, alteracion del 6to par craneal, reflejos ausentes o
disminuidos.

Efectos graves (1 semana): parestesia, neurotoxicidad tardia, debilidad caudal cefalico

3. Todos los sintomas son propios de la intoxicacion con inhibidores de la colinesterasa, exceptos:
a)bradicardia
b)fasciculaciones
c)sequedad de la mucosa
d)miosis

4. Le llega un paciente intoxicado de paraquat al cuarto de urgencias por intento de suicidio. No se debe administrar:
a)N-acetyl cisteina
b)oxigeno
c)solucion salina
d)acetaminofen
e)Ninguna de las anteriores

5. En el paciente que presentamos para la investigación que tenia un cuadro similar a la gastroenteritis los receptores más afectados
eran:
a)Nicotinicos
b)SNC
c)Sustancia P
d)Muscarinicos
e) Ninguno de los anteriores

Esto es porque el caso de la investigación de los 2 pacientes que mandaron a casa porque se pensaba que era una gastroenteritis sin
saber que era una intoxicación por Carbamatos… En alguna parte del p esta el caso…

22.Cual de las siguientes entidades se puede confundir con una intoxicación por organofosforados
a) Constipación
b) Hipotiroidismo
c) Hipertermia maligna
d) Intoxicación por hongos
e) Boca reseca

7.Paciente masculino de 47 años de edad que acude al cuarto de urgencia por sintomatología de dolor abdominal, nauseas, vómitos,
diarrea, sudoración profuso con mucha ansiedad, 3 horas después de haber ingerido una sopa en su restaurante favorito.
Tiene como antecedentes que es hipertenso y toma atenolol. Al examen fisico tiene una PA 110/70 FC: 60 x min. Ojos pupilas con la
tendencia a la miosis, corazon con bradicardia, pulmones con rudeza respiratoria. Usted sospecha que puede tratarse de cual de las
siguientes entidades. Usted sospecha que tiene una intoxicación por organofosforado. Los síntomas mencionados son la mayoria:
a) Muscarínicos
b) Nicotínicos
c) Del SNC
d) Intoxicación…
e) Ninguna de las anteriores

8.Con respecto a la intoxicación anterior, cual de los siguiente es un síntoma del SNC
a) Agitación
b) Fasciculaciones (nicotínico)
c) Miosis (muscarinico)
d) Dolor abdominal (muscarinico)
e) Tremor(temblor)

9.Si usted pudiera medir los niveles de paraquat en sangre por espectrofotometria, … nivel que indica alta probabilidad de muerte es
a) 2 mg/L a las 4 horas
b) 2 mg/L a las 2 horas TIENE DOS RESPUESTAS!!!!!!!!!! PREFERIMOS LA A
c) 1.5 mg/L a las 4 horas
d) 0.9 mg/L a las 6 horas
e) 0.5 mg/L a las 6 horas

10.Es caracteristico de la intoxicación por diquat


a) Convulsiones
b) Infarto agudo del miocardio
c) Insuficiencia hepatica
d) Edema pulmonar
e) Ninguna de la anteriores

11. Cuando usted administra pralidoxine en la intoxicación por organofosforado… actuando sobre:
a) Síntomas pulmonares
b) Síntomas muscarinicos en vista que síntomas nicotínicos no esta en la pregunta consideramos que es C
c) Reactivacion enzimatica
d) Síntomas del SNC (y síntomas nicotinicos)
e) Ninguna de las anteriores

17. Cual de los siguientes síntomas y signos forma parte del Síndrome muscarínico en intoxicación aguda de órganos fosforados
excepto
a. Abdominal. b. Miosis c. Calambres d. Rinorrea. e. Diarrea
Muscarinicos: post ganglionares
Miosis, visión borrosa
Dolor abdominal
Sialorrea
Rinorrea
Broncorrea, Broncoespasmo
Bradicardia
Trastornos gastricos
24) En paciente intoxicado con OF usted revisa:
a) Hemograma diario con revisión de electrólitos
b) Pseudocolinesterasa diaria niveles en sangre
c) Gases arteriales diarios
d) Urinálisis
Esto es para intoxicación aguda*

25) Intervalo de tiempo desde la intoxicación con un organofosforado en el cual es más beneficiosa y efectiva
la administración de una oxima:
a. Antes de transcurridas las 24 a 48 horas
b. Luego de transcurridas las 48 horas.
c. Entre las 72 a 96 horas
d. Su administración es igual de efectiva en cualquier momento.

29) El producto que puede ayudar a la intoxicación por paraquat es el siguiente:

a. Ditionita b. Oxigeno c. N-Acetil-Cisteina d. Antinflamatorios e. Acetominofen

30). Cual de los siguientes síntomas y signos forma parte del Síndrome muscarínico en intoxicación aguda de
órganos fosforados excepto
a. Dolor abdominal. b. Miosis c. Pesadez de piernas d .Rinorrea. e. Diarrea

31) Las siguientes manifestaciones indican atropinización, excepto:

a. Sed b. bradicardia c. midriasis d. Visión borrosa e. salivación

32) Paciente intoxicado con medicamento para los ojos presenta: rubor, mucosas secas, intranquilo y
retención urinaria su tx incluiría:
a. Fisostigmina b) Atropina c) Propranolol d) Flumazenil

33).. Paciente rociado con un tóxico llamado Malatión podrá presentar los siguientes síntomas muscarínicos
excepto:
a) lagrimeo b) sialorrea c) broncoespasmo d) rubicundez

36.La ruta más importante de intoxicación por tóxicos es:


a. Piel
b. Respiratorio
c. Ojos
d. Digestivo
e. Ninguna de las anteriores

38. Cuál de los siguientes síntomas y signos forma parte del Síndrome muscarinico en intoxicación aguda de
órganos fosforados excepto:
Dolor abdominal
Miosis
Taquicardia
Rinorrea
Diarrea

39. Cuál de los siguientes síntomas y signos forma parte del Síndrome nicotínico en intoxicación aguda por
órganos fosforados.
Ansiedad
Ataxia
Parálisis flácido
Convulsiones
Cefalea

52.Todos estos síntomas son propios de la intoxicación por inhibidores de colinesterasa excepto:
Bradicardia
Fasciculaciones
Sequedad de mucosas
Miosis

58. En relación con el uso de atropina en la intoxicación por organofosforados, cuál de las siguientes
afirmaciones es falsa:
Debe administrarse en forma precoz
Contrarresta la hipersecreción bronquial y las bradiarritmias.
Actúa sobre los síntomas muscarínicos y nicotínicos
El delirio atropínico es consecuencia de su excesiva administración.

64) El manejo apropiado de un paciente con intoxicación por un organofosforado incluye las siguientes
medidas, Excepto:
b. Atropina
c. Nitrito de sodio
d. Cloruro de pralidoxima
e. Descontaminación.

65) Indique el efecto neurológico tardío provocado por la intoxicación aguda con organofosforado:
a. Degeneración cerebelar b. Convulsiones c. Demencia d. Polineuropatía motora

69) Paciente masculino de 52 años de edad que es llevado al Cuarto de Urgencia luego de 6 horas después de
tomar una sopa en la fonda, en donde siempre almorzaba con los compañeros del trabajo. El cuadro se
caracterizaba por nauseas, vómitos, dolor abdominal, y diarrea. Según el médico del cuarto de urgencia los
signos vitales eran estables y solo tenía algo de signos de deshidratación. Se trata al paciente como una
gastroenteritis y se inicia el manejo con hidratación parenteral y ciproxina I.V.
A los 30 minutos es traído el compañero que siempre lo acompaña al almuerzo con el mismo cuadro, por lo que
los médicos del cuarto de Urgencia lo manejaron como una intoxicación alimentaria. A eso de las 2 horas uno
de los pacientes comienza a presentar broncorrea, sialorrea, cuando se ausculta tiene estertores subcrepitantes
y roncus el médico le suspende la venoclisis porque piensa que el paciente tiene un edema de pulmón. Usted
esta entrando al turno y le presentan los 2 casos, ¿Cuál seria su abordaje?
a. Iniciar nuevamente la venoclisis pero en forma mas lenta ya que el paciente mejoro levemente con
furosemida y aminofilina.
b. Omitir la ciproxina porque el medicamento desencadeno el cuadro de edema.
c. Abordar nuevamente a los 2 pacientes e investigar si hay más casos parecidos.
d. Iniciar mejor hidratación oral.

71) Mientras usted investiga han pasado 3 horas, ya que usted no solo esta viendo estos pacientes, uno de los
pacientes está somnoliento y convulsiona, y el otro comienza a presentar síndrome confucional, ya
definitivamente comienza a sospechar en otro tipo de intoxicación:
a. Intoxicación por Diquat por las convulsiones y dolor abdominal.
b. Intoxicación por órganos fosforados porque al principio tenían síntomas nicotínicos.
c. Intoxicación por Paraquat por el dolor abdominal secundario a Pancreatitis.
d. Intoxicación por órganos fosforados porque al principio tenían síntomas muscarínicos.

72) Cual seria su primer paso en este momento?


a. lavado gastrico
b. Dar jarabe de ipecacuana
c. Hemodiálisis
d. Cumplir el ABC de los primeros auxilios

73) Como explicaria el ultimo cuadro de los pacientes (convulsiones, somnolencia), ya que sus síntomas estan
relacionados con:
a. efecto del diquat a nivel del SNC
b. cuadro del SNC de organos fosforados
c. Cuadro del paraquat a nivel del SNC
d. Cuadro nicotínico por organos fosforados

74) Cual sería el antídoto para los siguientes tóxicos:


A) Oxigeno y N-acetil cisteina para el paraquat.
B) Oxigeno y Diazepan para el diquat.
C) Atropina y Oxima para intoxicación por órganos fosforados.
D) Tratamiento de sostén para órganos clorados.

83. Cual de los siguientes es un Carbamato:


a) Baygon
b) Asuntol
c) Baytex
d) Maltox.

84. En el paciente que presentamos para la investigación que tenía un cuadro similar a una Gastroenteritis los
receptores más afectado eran:
a) mucarinicos
b) Sistema Nervioso Central.
c) Sustancia P.
d) Nicotinicos.

86. El antídoto principal para las manifestaciones Nicotinitas es:


a. Atropina.
b. Protopan. (FALSO)
c. Togoxin
d. Ninguna de las anteriores.

87. El antídoto principal para las manifestaciones nicotìnicas es


a. atropina
b. protopam
c. terazocin
d. ninguna de las anteriores
e. todas las anteriores

88. Tenemos un paciente en la sala de Medicina y lo llama la enfermera porque la infusión de Atropina se
termino y el paciente intoxicado por Carbamatos está asintomático cual sería el paso a seguir:
a. No continuar con la atropina.
b. Continuarle a dosis bajas esperando los niveles de colinesterasas.
c. No continuar la atropina pero administrarla en bolo de atropina si presenta manifestaciones
de Broncorrea.
d. Administrarle Protopan y suspender la Atropina.

90. En el caso de la investigación de los 2 pacientes lo mandaron a casa porque se pensaba que era una
gastroenteritis sin saber que era una intoxicación por Carbamatos regresan por convulsiones y paro
cardiorrespiratorio. Usted pensaría que son manifestaciones típicas de:
a. Síndrome Nicotínico. b. Síndrome muscarínico a nivel pulmonar y cardiaco. c..Síndrome intermedio.
d. Síndrome del Sistema Nervioso Central. e. Síndrome de la medula espinal

92) En intoxicación por organofosforados los síntomas nicotínicos incluyen:


a) Agitación
b) Parálisis
c) Convulsiones

98. En el manejo terapéutico del paciente intoxicado por órganos fosforados es mejor guiarse por el
siguiente patrón de laboratorio o gabinete.
a. Niveles de Pseudocolinesterasas diario.
b. Rx de tórax diario.
c. Hemograma diario.
d. Electrolitos y Pruebas funcionales Hepáticas diarias.
e. Gases arteriales diarios.

Toxic6. Las siguientes situaciones se presentan en caso de hipocalcemia con hipoalbuminemia,


excepto

a. gastritis

b. enfermedades agudas

c. sepsis

d. síndrome de mala absorciòn

e. nefrosis

9. Todos estos síntomas son propios de la intoxicación con inhibidores de colinesterasa, excepto

a. bradicardia

b. fasiculaciones
c. miosis

d. resequedad de piel y mucosa

16. Usted está de turno en sala de medicina y lo llama la enfermera porque la infusión de atropina
se terminó, intoxicación por carbamatos está asintomático cuàl serìa el paso a seguir

a. no continuar con la atropina

b. continuarle a dosis bajas esperando los niveles de laboratorio

c. no continuar la atropina pero dar un bolo de atropina si presenta síntomas

d. administrarle … y suspender la atropina

e. continuar con la misma infusión para cumplir el protocolo

17. En el caso de la investigación de los dos pacientes lo mandaron a casa porque se pensaba que
… intoxicación por carbamatos regresan por convulsiones y paro cardiorespiratorio

a. sìndrome nicotínico

b. sìndrome muscarìnico a nivel pulmonar y cardiaco

c. sìndrome intermedio

d. sìndrome del sistema nervioso central

e. sìndrome de la mèdula espinal

18. En las caracterìsticas generales de los bipiridilos èstos se inactivan

a. se inactivan en presencia de la luz solar

b. se inactivan en presencia del agua

c. se inactivan con presencia de otro producto mezclado en el envase

d. se inactivan al tener contacto con los minerales arcillosos del suelo

20. Le llega un paciente intoxicado de paraquat al cuarto de urgencias el tratamiento de elección


hoy en día del paraquat es

a. N-acetil cisteína
b. oxìgeno a dos litros por minuto con inahaloterapia de N-acetil cisteìna

c. ciclofosfamida

d. no existe en este momento

e. atropina

21. En el paciente que presentamos para la investigación que tenìa un cuadro similar a una …

a. nicotinicos

b. sistema nervioso central

c. sustancia P

d. muscarìnicos

e. ninguna de las anteriores

22. El antídoto principal para las manifestaciones nicotìnicas es

a. atropina

b. protopan **

c. terazocin

d. ninguna de las anteriores

e. todas las anteriores

24. En el análisis del lìquido sinobial de un paciente usted encuentra que …

a. artritis tuberculosa

b. artritis reumatoidea

c. fiebre reumática

d. artrosis

e. artritis posd…

26. 27. 28. nombre tres indicaciones para realizar una densitrometrìa

a.____________________________________

b.____________________________________

c.____________________________________
29. Los marcadores bioquímicos para la osteoporosis estàn indicados solamente

a. para diagnóstico de osteoporosis

b. para cuando hay dudas con osteopenia radiográfica

c. seguimiento de la enfermedad del tratamiento para la osteoporosis

d. para determinar suspensión del tratamiento de osteoporosis

e. ya no se utiliza

30. Los anticuerpos antifosfolípidos producen sobre las plaquetas

a. aumento en el número de plaquetas

b. aumento de la actividad plaquetaria

c. aumento de la producción de la médula ósea

d. aumento en el secuestro por el vaso

e. no tiene ningún efecto sobre las plaquetas

31. La artritis en el lupus eritematoso sistémico se caracteriza por

a. ser siempre deformante

b. ser idéntica a la reumatoidea

c. presentar artralgia

d. ser parecida a la espondilitis anquilosante

e. siempre simétrica

32. En la artritis reactiva

a. el staphilococo es agente inductor

b. es màs frecuente el gonococo

c. el ANA es de utilidad

d. suele estar asociada a bacterias urogenitales

e. ninguna de las anteriores es cierta


33. Las fracturas mas frecuentes en la osteoporosis son las

a. hombro

b. vertebrales

c. cadera

d. rodillas

e. dedos de la mano

34. Cuál es la forma mas frecuente de presentación del LES

a. pericarditis

b. anemia

c. pleuritis

d. convulsiones

e. artritis o artralgia

35. Un paciente con debilidad muscular proximal de 65 años de edad de inicio que exámenes
pediría

a. electromiografía

b. referograma**

c. enzimas

d. exploraciòn muscular

e. biopsia de músculo

37. El dolor de la artritis reumatoidea

a. cede con el reposo

b. solo aparece con los movimientos

c. empeora en època lluviosa

d. aparece o se reactiva en la noche

e. es siempre muy intenso


38. En medio de consulta màs frecuente en el sìdrome de Sjogren es debido a

a. artritis

b. molestias oculares

c. molestìa orales

d. falta de sudoración

e. dispepsia

39. Cuándo aparece edema en una articulación se puede considerar que se trata de

a. gota

b. pseudogota

c. artritis séptica

d. artritis de microcristales

e. artritis

40. Las manifestaciones clínicas de la artrosis son bien conocidas …

a. eritema

b. tumefacción

c. deformidad

d. perdida de la función

e. crujidos

41. En la osteoporosis los dolores raquídeos se deben a.

a. fracturas vertebrales

b. lesiones discales

c. espondilosis vertebral

d. deformaciones raquìdeas

e. falta de calcio en los huesos


42. Citar una enfermedad que no presenta fenòmeno de RAYNAUD

a. artritis psoriasica

b. artritis reumatoidea

c. sìndrome de Sjogren

d. enfermedad del tejido conjuntivo

e. síndrome de CREST

43. En la artritis infecciosa el germen

a. actúa desde un lugar alejado de la articulación

b. se halla en el seno de la articulación

c. actúa mediante inmuno complejos

d. prepara el terreo para una sinovitis ulterior

e. inflama la articulación por mecanismo desconocido

44. Las dos hormonas fundamentales en el metabolismo fosfocalcico son

a. calcitonina y ACTH

b. paratohormona y vitamina D

c. vitamina D y Calcitonina

d. paratorhormona y calcitonina

e. ACTH y paratorhormona

45. El coco grampositivo que con mayor frecuencia causa artritis infecciosa

a. streptococo pneumoniai

b. streptococo pyogenes

c. streptococo betahemolítico

d. saphylococo epidermidis

e. staphylococo aureus

46. La pseudogota se debe a la precipitación intraarticular de cristales de


a. urato sódico

b. hidró…

c. colestrol

d. pirofosfáto sódico

e. corticoides

47. En el riñón la paratohormona

a. aumenta la reabsorción de calcio y disminuye la de fósforo

b. aumenta la reabsorción de calcio y fósforo

c. disminuye la reabsorción de calcio y fósforo

d. disminuye la reabsorción celular de calcio y aumenta la de fósforo

e. solo aumenta la reabsorción d calcio

48. La forma biológicamente más activa de la vitamina D

a. 1,25 (CH) D2

b. 1,25 (CH)2 D3

c. 2,25 (CH)2 D3

d. colesterol

e. 1,24 (CH)2 D3

50. Todo lo siguiente es cierto, excepto

a. el veneno de serpiente se produce en las glándulas homólogas a las paratiroides

b. la fracción tóxica del veneno de serpiente son proteínas y péptidos de 5,000 -30,000

c. el efecto biológico del veneno CROTALICO es estructuralmente reumático

d. la coral pertenece a la familia …

51. Cuando disminuye el fosfato sérico

a. disminuye la absorción de calcio intersticial

b. aumenta la reabsorción ósea de calcio


c. disminuye la paratorhormona

d. disminuye la 1,25 vit D

e. ninguna de las anteriores

68. Paciente en coma porque se tomó 30 pastillas de alprazolam enojado con su esposa

Tratamiento

a. atropina

b.

c.

d.

69. Cuál de las siguientes no es una medida para evitar absorción del …

a. descontaminación oral

b. D digestiva

c. diuresis forzada

d. D. cutánea

70. Paciente mordido por una culebra no presenta signos al inicio, 24 horas después presenta la
intoxicación crotálica la clasificaría como

a. síndrome bothtropico

b. s. e…

c. s. crotálico

d. s. botulínico

71. Paciente mordido por una culebra en el pie con aparición de edema un tercio de la pierna …

a. grado 0

b. leve

c. intermedio
d. severa

72. El tratamiento con suero antiofídico del paciente anterior sería así

a. menos de 5 frascos

b. 5 a 9 frascos

c. 15 frascos

d. más de 15 frascos

73. El tratamiento de las arritmias ventriculares por intoxicación con cocaína incluye

a. digoxina

b. amiodarona

c. nifedipina

d. propanolol

74. Cuál de los siguientes es causa de hipoglicemia

a. acidosis metabólica

b. síndrome de cushing

c. diarrea

d. cetoacidosis

75. El tratamiento en cuarto de Urgencia de hipercalcemia intensa es

a. bifosforados

b. furosemida y SS isotónica

c. furosemida y bifosforado

d. SS isotónica y calcitriol

76. Causa de hipercalcemia son todas las siguientes excepto

a. hipoparatiroidismo

b. hipertiroidismo
c. insuficiencia renal crónica

d. hipomagnesemia

77. Criterios de artritis reumatoidea incluye todo lo siguiente excepto

a. rigidez matutina

b. nódulo reumatoideo

c. erosión articular

d. artritis asimétrica

78. La poliartritis de la artritis reumatoidea frecuentermente afecta a todo lo siguiente excepto

a. manos

b. columna vertebral

c. brazos

d. pies

79. Tratamiento de lupus eritematoso sistémico incluye lo siguiente

a. omatuximab

b. asovaquona

c. ciclofosfamida oral

d. suplíride

80. Manifestaciones hematológicas del lupus eritematoso incluye lo siguiente excepto

a. linfopenia

b. leucopenia

c. trombocitosis

d. anemia emolítica

81. NO HABÍA EN EL EXÁMEN PREGUNTA #81.


82. Manifestaciones clínicas frecuentes de esclerosis sistémicas son todas las siguientes excepto

a. calcicosis

b. falla cardíaca

c. fenómeno de Raynaud

d. reflujo gastroesofágico

83. La principal causa de muerte en esclerosis sistémica es

a. neuropatía

b. nefropatía

c. hipertensión pulmonar

d. cáncer de esófago

94. Articulación menos afectada en la artritis reumatoidea

a. interfalángicas proximales

b. interfalángicas distales

c. témporomandibulares

d. ………

95. El HLA B-27 tiene utilidad en

a. artritis reumatoide

b. artritis reumatoidea sero negativa

c. espondilitis anquilosante

d. osteoporosis

e. síndrome de la enfermedad sin pulso

96. Señora de 90 años de edad acude a urgencia por hinchazón muy dolorosa de rodilla izquierda,
toma diuréticos por hipertensión arterial. Usted punziona la rodilla y extrae líquido cetrino poco
vizcoso, lo pone en el microscopio y observa cristales birrefrigentes negativos en forma de aguja
dentro de los PMN y cristales romos abundantes con birrefrigencia positiva. La RX de la rodilla
muestra cóndor… diagnóstico
a. artritis séptica

b. artritis por ácido úrico

c. artritis por pirofosfáto sódico

d. gota

e. SJOGRE

97. A ésta paciente usted la trataría con

a. antiinflamatorios no esteroideos

b. colchicina

c. corticoides

d. asa…

e. no se le da tratamiento

98. Paciente que presenta artritis reumatoidea, leucopenia y esplenomegalia se trata de

a. síndrome de hughes

b. síndrome de felty

c. síndrome de SJOGRE

d. síndrome de reiters

e. síndrome de barré

99. Para monitorizar la actividad clínica del lupus eritematoso sistémico es útil a

a. la VSG y la cuenta de leucocitos

b. los anticuerpos antinucleares

c. la proteína c

d. las proteínas séricas

e. el C3

100. Vasculitis primarias c-anca’s anticuerpo contra citoplasma de neutrófilos asociados

a. Tacayasu
b. Wegener

c. Cawasaki

d. células giganteso-reuma-renal

TOXICOLOGIA
Semestral 2015-3

1. Cual de enunciados es correcto:


a) Los opiáceos no tienen receptores endógenos ;
Falso; los alcaloides opioides producen analgesia a través de acciones sobre
receptores en sistema nervioso central, que contiene péptidos con propiedades
farmacológicas similares a opioides. El termino general de uso actual para estas
sustancias endógenas es péptidos opioides endógenos.
b) Las benzodiacepinas tienen receptores dopaminergicos;
falso, su receptor es GABA
c) Las anfetaminas tiene receptores GABAergico
Su receptor es adrenergico
d) Cocaína se una a la membrana presinaptica dopaminergica

2. Los receptores mu de la heroína actúan sobre los receptores:


a) GABAergicos*
Los metabolitos de la heroína se unen a los receptores opioides. Pueden
causar cambios en la excitabilidad de las neuronas, estimulando la liberación
presináptica de ácido gamma-aminobutírico (GABA).
b) Dopaminergicos
c) Colinergicos
d) Serotinoninergico
3. La siguiente no es una acción simpaticomimetica de la cocaína:
a) Taquicardia
b) Miosis*
c) HTA
d) Estreñimiento
La cocaína produce: vasoconstricción por su efecto simpaticomimético periférico y
aumento de la presión arterial por su efecto inotrópico y cronotrópico positivo unido
al efecto vasoconstrictor. Bradicardia a dosis bajas por depresión del nodo sinusal y
más frecuentemente taquicardia por estímulo sinusal, como consecuencia directa del
estímulo simpático. Aumento de la fuerza de contracción y de la frecuencia cardiaca
por un estímulo de los receptores b1 fundamentalmente. También produce
midriasis, temblor y sudoración por estímulo simpático.

4. En el manejo terapéutico del paciente intoxicado por órganos fosforados es mejor guiarse
por el siguiente (no se que dice supongo que es dato) de laboratorio o gabinete:
a) Niveles de pseudocolinesterasas diario*

Tambien denominada butirilcolinesterasa o colinesterasa plasmática. Esta presente


generalmente en forma soluble en todos los tejido, principalmente el hígado, plasma y
en poca concentración SNC y periférico. La medición de su actividad constituye una
ayuda importante para el diagnostico de intoxicaciones agudas.

b) Rx de torax diario
c) Hemograma diario
d) Electrolitos y pruebas funcionales hepáticas diarias
e) Gases arteriales diarios

5. Las siguientes manifestaciones indican atropinizacion, excepto:


a) Sed
b) Bradicardia
c) Miosis
d) Resequedad de boca
Sus efectos secundarios incluyen sequedad de la boca, hipohidrosis, midriasis,
retención urinaria,y estreñimiento. La producción reducida de sudor puede
resultar en la hipertermia.

Semestral junio 2014

6. Paciente en coma porque se tomo 30 pastillas de alprazolam, enojado con su novia. Ustes
utilizaría el siguiente antídoto:
a) Atropina
b) Flumazenil*
c) Protamina
d) Fisostigmina
El antídoto conocido para las benzodiacepinas es el Flumazenil. Su uso se
recomienda en aquellos casos en los cuales hay coma y depresión
respiratoria secundarios al uso de las benzodiacepinas lo cual como ya se
mencionó suele ser raro. El suministro del flumazenil puede implicar
riesgos en aquellos pacientes que se encuentran hipotensos, con arritmias o
alteraciones hemodinámicas.
7. Cual de las siguientes no es una medida para evitar absorción del toxico:
a) Descontaminación oral*
b) D. digestiva
c) Diuresis forzada
d) D. cutánea
8. Pte. Mordido por una culebra, no presenta signos al inicio, 24 horas después presenta
ptosis palpebral. La intoxicaicon ofídica la clasificaría como:
a) Sd. Botrops
b) Sd. Elapidico
c) S. crotalico
d) S. botulínico
Según el ppt de Vietnam: el crotalico produce: DOLOR EN NUCA,

MAREOS, DISMINUCIÓN……PERDIDA DE LA VISIÓN,


PTOSIS OBNUBILACIÓN
CASOS GRAVES:
COMA Y CONVULSIONES

Accidente por Crotalus (Cascabel):


En este caso se combina un síndrome neurotóxico con trastornos hemolíticos.
· El síndrome local es discreto: dolor y edema leve acompañado de parestesias,
pudiendo observarse petequias en el sitio de la mordedura.
· Las manifestaciones neurológicas son de tipo curarizantes debido a la presencia de
fosfolipasa A e incluyen:
• Oftalmoplejía: ptosis palpebral, diplopía, disminución de la agudeza visual.
• Obnubilación.
• Dificultad en articular palabras.
• Trastornos del equilibrio.
La sintomatología neurológica, una vez constituida se mantiene alrededor de 2
semanas, desapareciendo gradualmente sin dejar secuelas aparentes.
• La orina presenta: reacción ácida, albúmina 1 grs./l., intensa hemoglobinuría,
cilindruría.

9. Paciente mordido por una culebra en el pie, con aparición de equimosis, edema hasta 1/3
medio de la pierna con TPT y TP alterados ligeramente, la intoxicación ofídica se clasifica
como:
a) Grado 0
b) Leve
c) Moderado
d) Severa

Según el ppt: la leve produce esto: EFECTOS LOCALES (EDEMA Y DOLOR) EN 1-2
SEGMENTOS

HEMORRAGIA ESCASA EN EL SITIO DE LA MORDEDURA.

SIN ALTERACIONES SISTÉMICAS.

PRUEBAS DE COAGULACIÓN NORMALES O LEVEMENTE ALTERADAS

10. El tratamiento con suero antiofídico del paciente anterior seria asi:
a) 5 frascos*
b) 5 a 9 frascos
c) 15 frascos
d) Mas de 15 frascos

• Según ppt: GRADO I : LEVE……………………………………5 AMPOLLAS

11. El tx de las arritmias ventriculares por intoxicación con cocaína incluye:


a) Digoxina
b) Amiodarona
c) Nifedipina
d) Propanolol *

La taquicardia sinusal severa o sintomática se debe de tratar con betabloqueantes o bloqueantes


de los canales del calcio. Se han utilizado con éxito en animales el esmolol, labetalol, proanolol,
diltiazen y verapamil. Se debe de tomar precaución con la utilización de betabloqueantes solos en
los pacientes con dolor torácico e hipertensión concomitante o hipotensión ya que puede conducir
a una estimulación alfa con paradójico aumento de la TA, aumento del vasoespasmo coronario y
disminución del gasto cardíaco. Por esto se recomienda que se utilice con el propanolol un
alfabloqueante como fentolamina (5 mg intravenoso cada 15 o 30 minutos en infusión intravenosa
rápida) o vasodilatadores como la nitroglicerina o el nitroprusiato para el tratamiento de la
taquicardia asociada con hipertensión o dolor torácico.

12. Del veneno de coral todo lo siguiente es cierto excepto:


a) Es de acción exclusivamente neurotóxica
b) Ataca el sistema nervioso en la placa mioneural
c) Produce una paralisis muscular progresiva
d) La afeccion de los trayectos nerviosos es en sentido centrifugo
e) Se fija a los tejidos en un periodo se 4 horas; sin conducir a daño permanente al
tejido circundante a la herida ni a la discapacidad o amputación del miembro
afectado

• Según ppt: EL VENENO DE CORAL ES NEUROTÓXICO. ATACA EL SISTEMA NERVIOSO EN LA


PLACA MIONEURAL PARÁLISIS MUSCULAR PROGRESIVA.

13. De las escorpionotoxinas solo es cierto:


a) Actúan sobre los canales calcio de varios tejidos excitables
b) Actúan sobre el sistema nervioso simpático produciendo liberaicon de acetilcolina
lo que determina el cuadro sistémico: es falso por que nivel simpático libera
adrenalina
c) La acción de las toxinas escropionicas produce hipertensión e hiperglicemia
d) Los órganos blanco mas afectados son: corazón, páncreas y riñon; es falso por que
es pulmón no riñon
e) N.a.

Toxicologia

11. Cuando usted administra pralidoxine en la intoxicación por organofosforado… actuando sobre:

a) Síntomas pulmonares
b) Síntomas muscarinicos

c) Reactivacion enzimatica

d) Síntomas del SNC (y síntomas nicotinicos)

e) Ninguna de las anteriores

Las oximas (la pralidoxina forma parte de ellos) son útiles para combatir los síntomas nicotínicos en
la IA por IOF.

12. Cual de las siguientes entidades se puede confundir con una intoxicación por organofosforados

a) Constipación

b) Hipotiroidismo

c) Hipertermia maligna

d) Intoxicación por hongos

e) Boca reseca

Diagnóstico diferencial

Debido a que existen algunas patologías o situaciones que pueden ser semejantes al cuadro
presentado por los intoxicados con inhibidores de las colinesterasas, es necesario conocerlas, para
establecer en el menor tiempo posible, el diagnóstico diferencial que asegure una conducta
terapéutica acertada. Podemos mencionar, como las más relevantes, las siguientes:

• Síndrome convulsivo
• Estado de coma hipo e hiperglicémico
• Estados de coma provocados por otras causas
• Intoxicaciones con otros tóxicos, tales como fluoracetato de sodio, hidrocarburos clorados,
depresores del sistema nervioso central (como los alcoholes etílico y metílico, los opiáceos).
• Intoxicación paralítica por mariscos (marea roja)
• Cuadros neuropsiquiátricos, como neurosis conversiva
• Enfermedad diarréica aguda
• Edema pulmonar agudo asociado a otras etiologías
• Insuficiencia cardíaca congestiva
• Hiperreactividad bronquial
• Micetismo (intoxicación por hongos)
13. Paciente masculino de 52 anos de edad que es llevado al cuarto de urgencia luego de 6 horas
después de tomar una sopa de fonda, en donde siempre almorzaba con los companeros de
trabajo. El cuadro se caracterizaba por nauseas, vomitos, dolor abdominal, y diarrea. Según el
medico del cuarto de urgencias los signos vitales eran estables y solo tenia algo de signos
de deshidratación. Se trata al paciente como una gastroenteritis y se inicia el manejo con
hidratación parenteral y ciproxina IV. A los 30 minutos es traido el companero que siempre lo
acompana al almuerzo con el mismo cuadro, por lo que los medicos del cuarto de urgencias lo
manejaron com una intoxicación alimentaria. A eso de las 2 horas uno de los pacientes comienza a
presentar boncorrea, sialorrea, cuando se ausculta tiene estertores subcrepitantes y roncus, el
medico le suspende la venoclisis porque piensa que el paciente tiene un edema de pulmon. Usted
esta entrando al turno y le presentan los 2 casos ¿Cuál seria su abordaje?

a. Iniciar nuevamente la venoclisis pero en forma mas lenta ya que el paciente mejoro levemente
con furosemida y aminofilina

b. Omitir la ciproxina porque el medicamento desencadeno el cuadro de edema

c. Abordar nuevamente a los 2 pacientes e investigar si hay más casos parecidos

d. Iniciar mejor hidratación oral

14. Mientras usted investiga han pasado 3 horas, ya que usted no solo esta viendo estos
pacientes, uno de los pacientes esta somnoliento y convulsiona, y el otro comienza a presentar
síndrome confucional, ya definitivamente comienza a sospechar en otro tipo de intoxicación

a. Intoxicación por Diquat por las convulsiones y el dolor abdominal

b. Intoxicación por organos fosforados porque al principio tenian síntomas nicotinicos


c. Intoxicación por paraquat por el dolor abdominal secundario a pancreatitis

d. Intoxicación por organos fosforados porque al principio tenian síntomas muscarinicos

Los síntomas muscarínicos aparecen en las 4 primeras horas, y revierten con atropina ; los más
comunes son: vómitos, miosis y sialorrea.

15. Cual seria su primer paso en este momento?

a. lavado gastrico

b. Dar jarabe de ipecacuana

c. Hemodiálisis

d. Cumplir el ABC de los primeros auxilios

16. Como explicaria el ultimo cuadro de los pacientes (convulsiones, somnolencia), ya que sus
síntomas estan relacionados con:

a. efecto del diquat a nivel del SNC

b. cuadro del SNC de organos fosforados

c. Cuadro del paraquat a nivel del SNC

d. Cuadro nicotínico por organos fosforado

Cual seria el antidoto para los siguientes toxicos:

e. Oxigeno y N-acetil cisterna para el paraquat

f. Oxigeno y Diazepam para el diaquat


g. Atropina y Oxima para intoxicación por organos fosforados

h. Tratamiento de soten para organos clorados

Para tratar la intoxicación por organofosforados y carbamatos se debe utilizar como antídoto
atropina y en el caso de los organofosforados puede ser necesaria la utilización de oximas
(Pralidoxima u Obidoxima).

17. La prueba de Ditionita tiene valor pronostico en

a. en las primeras 24 horas

b. en las primeras 36 horas

c. 36 horas en adelante

d. Después de las 48 horas

La prueba de la ditionita parece tener un valor pronóstico aproximado cuando la orina se examina
en las primeras 24 horas siguientes a la ingestión:

18. Desde el punto de vista toxicologico de la via de absorción mas importante es la intoxicación
por paraquat es:

a. digestivo

b. respiratorio

c. piel

d. conjuntival
Desde el punto de vista toxicológico, la vía de absorción más importante es la digestiva, puesto
que es la más frecuentemente involucrada en casos de intoxicación (accidental o suicida). La
absorción por la piel intacta y la vía respiratoria es menor.

19. Organos afecta el paraquat que puede tener dano irreversible cuando la dosis es de 20 a 40/kg
de peso corporal:

e. pulmon

f. higado

g. rinon

h. corazon

El profeta dice riñon pero encontré un lugar que decía que daño también era irreversible en
pulmón.

20.Complicación tardia del paraquat a nivel de los ojos es:

a. Desprendimiento de retina

b. Conjuntiva de retina

c. Keratitis y Uveitis

d. Opacidad corneal

El contacto del paraquat con los ojos causa conjuntivitis y si no se retira de inmediato, puede traer
como consecuencia la opacidad tardía de la córnea.

1. En intoxicación por Paraquat tratamiento es:

a) Ditionita de sodio
b) N-acetil-cisteína
c) Antiinflamatorios+....
d) AINES

2. En intoxiación por organofosforados los síntomas muscarínicos incluyen


todos excepto:
a) Sialorrea
b) Miosis
c) Calambres es un efecto nicotínico

3. En intoxicación por organofosforados los síntomas nicotínicos incluyen:

a) Agitación es muscarinico
b) Parálisis
c) Convulsiones es SNC

4. El tratamiento con atropina da todo menos:


a) Miosis
b) Sequedad de boca AQUI HAY DOS RESPUESTAS
c) Bradicardia

5. El principal antidoto para la intoxicación por paraquat es:

a. n-acetil cisterna
b. selenio
c. colchicina
d. vitamina E en altas dosis
e. No existe ninguno en este momento

6. La hemodiálisis como tratamiento de intoxicación por barbitúricos esta indicada en:

coma en estadio III

cuando el paciente esta en estupor

al llegar al cuarto de urgencia para prevenir el coma

edema agudo de pulmon

se el paciente tiene diarrea

7. Paciente masculino de 47 años de edad que acude al cuarto de urgencia por sintomatología de
dolor abdominal, nauseas, vómitos, diarrea, sudoración profuso con mucha ansiedad, 3 horas
después de haber ingerido una sopa en su restaurante favorito.

Tiene como antecedentes que es hipertenso y toma atenolol. Al examen fisico tiene una PA
110/70 FC: 60 x min. Ojos pupilas con la tendencia a la miosis, corazon con bradicardia, pulmones
con rudeza respiratoria. Usted sospecha que puede tratarse de cual de las siguientes entidades.
Usted sospecha que tiene una intoxicación por organofosforado. Los síntomas mencionados son la
mayoria:

a) Muscarínicos
b) Nicotínicos

c) Del SNC

d) Intoxicación…

e) Ninguna de las anteriores

8. Con respecto a la intoxicación anterior, cual de los siguientes es un síntoma del SNC

a) Agitación

b) Fasciculaciones (nicotínico)

c) Miosis (muscarinico)

d) Dolor abdominal (muscarinico)

e) Tremor

9. Si usted pudiera medir los niveles de paraquat en sangre por espectrofotometria, … nivel que
indica alta probabilidad de muerte es

a) 2 mg/L a las 4 horas

b) 2 mg/L a las 2 horas

c) 1.5 mg/L a las 4 horas TAMBIEN HAY DOS RESPUESTAS

d) 0.9 mg/L a las 6 horas

e) 0.5 mg/L a las 6 horas

10. Es característico de la intoxicación por diquat

a) Convulsiones

b) Infarto agudo del miocardio

c) Insuficiencia hepática EN ALGUNAS MENCIONA ESTA TAMBIEN

d) Edema pulmonar

e) Ninguna de la anteriores

41. Un paciente intoxicado con paraquat llega al cto de urgencia con insuficiencia respiratoria,
este pacientes no deben de recibir:

a) Oxigeno
b) Solución salina
c) Dextrosa en salina
d) Corticoide

El paraquat tiene carga, lo que le proporciona la capacidad de producir radicales libres que
reaccionan con oxígeno, formando radicales iónicos como el superóxido y peróxido de
hidrógeno , que al producirse en una cantidad elevada, la superóxido dismutasa no puede
neutralizar. Estos radicales forman hidroperóxidos al reaccionar con la pared de la membrana
celular, produciendo la destrucción de la membrana e interfiriendo con la función pulmonar
surfactante. El radical se forma más rápidamente si hay oxígeno en el medio, por eso son los
pulmones su órgano diana.

42. Todos estos síntomas son propios de la intoxicación con inhibidores de colinesterasa
excepto:

a) Bradicardia
b) Fasciculaciones
c) Miosis
d) Sequedad de piel y mucosa

Tiene acción sobre el sistema parasimpático, por lo cual causará bradicardia, hipotensión,
hipersecreción glandular, broncoconstricción, hipermotilidad del tracto gastrointestinal y
disminución de la presión intraóptica.

43. Usted está de turno en sala de Medicina y lo llama la enfermera porque la infusión de
Atropina se termino y el paciente intoxicado por Carbamatos está asintomático cual sería el
paso a seguir:

a) No continuar con la atropina.


b) Continuarle a dosis bajas esperando los niveles de colinesterasas.
c) No continuar la atropina pero administrarla en bolo de atropina si presenta
manifestaciones de Broncorrea.
d) Administrarle Protopan y suspender la Atropina.
e) Continuar la misma infusión para cumplir protocolo

La mejoría pulmonar puede no ser paralela a otras señales de atropinización. La continuación


o retorno de señales colinérgicas indican la necesidad de más atropina. La dosis podrá ser
disminuida cuando los síntomas se mantengan estables durante por lo menos 6 horas. Una
vez estabilizado el paciente, se debe controlar la cantidad de atropina utilizada por día, debiendo
mantener esa dosis como mínimo 24 a 48 horas,comenzando luego a disminuirla
paulatinamente, hasta pasar a la vía oral, nunca suspenda el tratamiento en forma brusca.
Recién se decidirá suspender la atropina definitivamente cuando el paciente este asintomático
y la colinesterasa se encuentre por encima de la mitad de su valor normal.

44. En el caso de la investigación de los 2 pacientes lo mandaron a casa porque se pensaba


que era una gastroenteritis sin saber que era una intoxicación por Carbamatos regresan por
convulsiones y paro cardiorrespiratorio. Usted pensaría que son manifestaciones típicas de:

a) Síndrome Nicotínico
b) Síndrome muscarínico a nivel pulmonar y cardiaco
c) Síndrome intermedio.
d) Síndrome del Sistema Nervioso Central
e) Síndrome de la medula espinal

Manifestaciones del SNC son: ansiedad, labilidad emocional, inquietud, confusión, ataxia,
temblores, convulsiones y coma

45. La causa mas frecuente de enfisema subcutáneo en el estadio II y III de la intoxicación por
Paraquat es:

a) Perforación pulmonar por el daño directo del Paraquat al Pulmón


b) Perforación de una bula Pulmonar
c) La erosión directa en la piel que es severa
d) Perforación Esofágica

En ocasiones puede encontrarse enfisema subcutáneo secundario a perforación esofágica. La


pancreatitis, si se presenta, produce intenso dolor abdominal. Estas manifestaciones tempranas
no necesariamente son inmediatas, sino que pueden aparecer hasta después de 24 horas. Ha
habido casos fatales de intoxicación que habían sido dados de alta por no presentar lesiones
orales al momento de la consulta inicial

46. Cual de los siguientes síntomas y signos forma parte del Síndrome muscarínico en
intoxicación aguda de órganos fosforados excepto

a) Abdominal
b) Miosis
c) Calambres
d) Rinorrea
e) Diarrea

Efectos muscarínicos de los sistemas de órganos incluyen los siguientes:


Cardiovascular - La bradicardia, hipotensión

Respiratorio - Rinorrea, broncorrea, broncoespasmo, tos, dificultad respiratoria grave

Gastrointestinal - Hipersalivación, náuseas y vómitos, dolor abdominal, diarrea, incontinencia


fecal

Genitourinario – Incontinencia

Ocular - Visión borrosa, miosis

Glándulas - Aumento del lagrimeo, diaforesis

47. Le llega un paciente intoxicado de paraquat al cuarto de urgencia por intento de suicidio. Le
explica a los familiares que el antídoto confirmado hoy en día del Paraquat es:

a) N-Acetil Cisteina
b) Oxigeno a 2 litros por minuto con inhaloterapia de N-Acetil cisteina.
c) Ciclofosfamida
d) No existe en este momento
e) Atropina

No se dispone de antídoto específico y eficaz. La administración de anticuerpos antiparaquat


secuestran el mismo del plasma, pero no previenen el acúmulo en los tejidos. Se han usado
inmunosupresores como la ciclofosfamida sola o con corticoides, para reducir la fibrosis
pulmonar. Algunos autores han obtenido buenos resultados pero no han sido confirmados por
otros autores. El trasplante pulmonar se ha usado en pocos casos y podría ser útil en fibrosis
pulmonar, es de esperar que sea usada en el futuro. El tratamiento irá encaminado a disminuir
la absorción del tóxico, aumentar su excreción y disminuir los efectos del tóxico sobre los
órganos dianas, de todas formas no es muy efectivo.

48. En el paciente que presentamos para la investigación que tenía un cuadro similar a una
Gastroenteritis los receptores más afectado eran:

a) Nicotinicos.
b) Sistema Nervioso Central
c) Sustancia P
d) Muscarinicos
e) Ninguno de los anteriores

Los receptores muscarinicos están asociados a fenómenos gastrointestinales, como:


hipersalivación, náuseas y vómitos, dolor abdominal, diarrea, incontinencia fecal

**no estoy muy segura de esta, en la respuesta de la 44 está correcto que es un síndrome del
SNC, pero no sé si esto quiere decir que estos receptores son los más afectados. La respuesta
del profeta original era muscarínicos, pero también el profeta original en la respuesta del 44
ponía Síndrome del SNC.

49. En paciente intoxicado con OF usted revisa:

a) Hemograma diario con revisión de electrólitos


b) Pseudocolinesterasa diaria niveles en sangre
c) Gases arteriales diarios
d) Urinálisis

La intoxicación por organofosforados es de diagnóstico clínico. Confirmación de la intoxicación


de organofosforado se basa en la medición de la actividad de la colinesterasa; típicamente,
estos resultados no son fácilmente disponibles. Aunque los glóbulos rojos (RBC) y plasma
(pseudo) de la colinesterasa (PChE) pueden ser utilizadas, RBC de la colinesterasa se
correlaciona mejor con el sistema nervioso (CNS) acetilcolinesterasa central (AChE) y es, por
lo tanto, un marcador más útil de la intoxicación de organofosfato.

50. En las características generales de los bipiridilos estos se inactivan al:

a) Se inactivan en presencia de la luz solar en


b) Se inactivan en presencia del agua.
c) Se inactivan con presencia de otro producto mezclado en el envase para producir emesis.
d) Se inactivan al tener contacto con los minerales arcillosos del suelo.

Las arcillas tienen grandes áreas de superficie y alta capacidad para adsorber los herbicidas,
siendo la montmorilonita más adsortiva que la ilita o la caolinita. Los herbicidas cargados
positivamente, como paraquat, no tienen actividad en el suelo, ya que quedan fijados
fuertemente por los coloides de suelo cargados negativamente.

51. Usted está en el cuarto de urgencias y evalúa un paciente con cuadro clínico
evidente de intoxicación por Paraquat pero para más seguridad ya que no trajeron
el recipiente usted le hace una prueba de ditionita y el misma da una coloración
verde usted pensaría que:

a) Se trata de una intoxicación por diquat.

b) Que la Ditionita está oxidada.

c) Que la Ditionita no tiene bicarbonato por esto sale verde.

d) Que da ese color porque el paciente no tiene ni 30 minutos de estar intoxicado.

Al parecer, la prueba de ditionito tiene un valor pronóstico aproximado


cuando se realiza la prueba con orina recolectada en las primeras 24 horas
después de ingerir paraquat: concentraciones menores a un miligramo por
litro (incoloro azul claro) generalmente predicen sobrevivencia, mientras que
las concentraciones superiores a un miligramo por litro (azul marino a azul
oscuro) con frecuencia vaticinan un resultado fatal. En esta prueba, el diquat
en la orina produce un color verde. Aunque existe menos experiencia con la
prueba de ditionito en envenenamientos por diquat,

52. Cual de los siguientes es un Carbamato:

a) Baygon ( contienen la ciflutrina piretroides, transflutrina, praletrina y los


propoxur y organofosforados clorpirifos carbamato, como ingredientes
activos.)

b) Asuntol (pertenece al grupo de pesticidas a base de ésteres del ácido fosfórico. Es


un garrapaticida efectivo que reúne múltiples cualidades que lo han situado como
líder en la familia de los organofosforados.)

c) Baytex (plaguicidas organofosforados)

d) Maltox. (plaguicidas organofosforados)

53. Quien dota material como tierra Fuller al cuarto de Urgencia para el manejo
del Paraquat:

a) Ministerio de Salud

b) Ministerio de Desarrollo Agropecuario

c) La OMS.

d) La casa farmacéutica que produce el Paraquat.

54. El antídoto principal para las manifestaciones Nicotinitas es:


a. Atropina.

b. Protopan.

c. Toxogonin.

d. Ninguna de las anteriores.

55. el diquat es menos tóxico que el paraquat y actúan peroxidadndo la membrana


lipídica

a. Primera parte cierta, segunda parte cierta (El daño pricincipalmente


es la peroxidación lipídica de la membrana y a la disminución del NADPH)

b. primera parte cierta, segunda falsa

c. Primera falsa, segunda cierta

d. primera falsa, segunda falsa

21. Niveles de paraquat en sangre que indican alta probabilidad de muerte

a. 0.9 md/L a las 24 horas

b. 1.0 mg/L a las 6 horas

c. 1.5 mg/L a las 4 horas

d. 2.0 mg/L a las 4 horas

22. Conociendo usted las propiedades bioquímicas del Paraquat. ¿Cuál será el objetivo de utilizar
como tratamiento inicial tierra o tierra fuller?

a. Bloquea la absorción intestinal y se elimina en orina

b. Elimina los metabolitos del mismo por orina.

c. Se inactiva en contacto con minerales arcillosos (el barro)

d. Ayuda a eliminar el paraquat vía intestinal por heces al unirse a ciertas proteínas intestinales
23. Usted aunque no lo crea esta esperando los niveles de Paraquat realizado por cromatografía
para dializar. Cuál sería el nivel que se puede beneficiar este paciente con dicho procedimiento:

a. 2.5 mg/Lt a las 4 horas

b. 1.0 mg/Lt a las 6 horas

c. 1.5 mg/Lt a las 6 horas

d. 1.83mg/Lt a las 4 horas

24. Usted es llamado al cuarto de urgencia por un paciente que está intoxicado por paraquat, para
que le realice lavado gástrico. Usted lo realiza si el paciente:

a. Tiene 6 horas de haber ingerido la sustancia

b. Tiene 7 horas de haber ingerido la sustancia

c. Tiene 4 horas de haber ingerido la sustancia

d. No sabe el tiempo de haber ingerido, lo importante es salvarlo

25. El manejo apropiado de un paciente con intoxicación por un organofosforado incluye las
siguientes medidas, Excepto:

a. Atropina

b. Nitrito de sodio

c. Cloruro de pralidoxima

d. Descontaminación.

26. Indique el efecto neurológico tardío provocado por la intoxicación aguda con
organofosforado:

a. Degeneración cerebelar b. Convulsiones c. Demencia d. Polineuropatía motora

27. En relación con el uso de atropina en la intoxicación por organofosforados, cuál de las
siguientes afirmaciones es falsa:

a. Debe administrarse en forma precoz

b. Contrarresta la hipersecreción bronquial y las bradiarritmias.

c. Actúa sobre los síntomas muscarínicos y nicotínicos ( solo contra los muscarinicos)
d. El delirio atropínico es consecuencia de su excesiva administración.

28. Todos estos síntomas son propios de la intoxicación con inhibidores de colinesterasa excepto:

a. Bradicardia b. Fasciculaciones c. Miosis d. Sequedad de piel y mucosas

29. Intervalo de tiempo desde la intoxicación con un organofosforado en el cual es más


beneficiosa y efectiva la administración de una oxima:

a. Antes de transcurridas las 24 a 48 horas

b. Luego de transcurridas las 48 horas.

c. Entre las 72 a 96 horas

d. Su administración es igual de efectiva en cualquier momento.

30. La ruta más importante de intoxicación por tóxicos es:

a. Piel b. Respiratorio c. Ojos d. Digestivo e. Ninguna de las anteriores

31. El producto que puede ayudar a la intoxicación por paraquat es el siguiente:

a. Ditionita b. Oxigeno c. N-Acetil-Cisteina d.


Antinflamatorios e. Acetominofen

El test cualitativo de la ditionita indica sólo exposición al tóxico pero no tiene valor
pronóstico. Consiste en añadir 2 ml de ditionita sódica al 1% a una muestra de
orina o contenido gástrico, previamente alcalinizado con bicarbonato sódico. La
positividad viene dada por la aparición de una coloración azul-verdosa.
PERO LA PREGUNTA DICE EL PRODUCTO QUE PUEDE AYUDAR ENTONCES SI LA
ENTIENDO DE ESTA MANERA SERIA EL OXIGENO

la terapia de oxígeno que no debe ser administrado a menos que se haya


confirmado la hipoxemia, ya que puede agravar el daño celular mediado por
oxígeno inducida por el ciclo redox.

32. Cual de los siguientes síntomas y signos forma parte del Síndrome muscarínico
en intoxicación aguda de órganos fosforados excepto
a) Dolor abdominal. b. Miosis c. Pesadez de piernas d .Rinorrea. e.
Diarrea

33. Cual de los siguientes síntomas y signos forma parte del Síndrome nicótico en
intoxicación aguda por órganos fosforados.
a) Ansiedad b. Ataxia c .Parálisis flácido d. Convulsiones. e. Cefalea

34. En el manejo terapéutico del paciente intoxicado por órganos fosforado es mejor
guiarse por el siguiente patrón de laboratorio o gabinete.

a. Niveles de Pseudocolinesterasas diario.


b. Rx de Tórax diario.
c. Hemograma diario
d. Electrolitos y Pruebas funcionales Hepáticas diarias.
e. Gases arteriales diarios

35. Las siguientes manifestaciones indican atropinización, excepto:

a. Sed b. bradicardia c. midriasis d. Visión borrosa e.


Salivación
ES INCORRECTO BRADICARDIA PORQUE UNO DE LAS MANIFESTACIONES
ES TAQUICARDIA
36. Paciente intoxicado con medicamento para los ojos presenta: rubor, mucosas
secas, intranquilo y retención urinaria su tx incluiría:
a) Fisostigmina b) Atropina c) Propranolol d) Flumazenil

ESTE LO HICE POR EXCLUSION Y EL TRATAMIENTO POR INTOXICACIONES POR


MEDICAMENTOS ES MAS COMUN EL USO DE LA FISOSTIGMINA QUE LOS
OTROS MEDICAMENTOS.

37. Paciente rociado con un tóxico llamado Malatión podrá presentar los siguientes
síntomas muscarínicos excepto:
a) lagrimeo b) sialorrea c) broncoespasmo d) rubicundez

38. Paciente en coma porque se tomó 30 pastillas de alprazolam, enojado con su


novia. Usted utilizaría el siguiente
antídoto: a) Atropina b) Flumazenil c) Protamina d) Fisostigmina
39. Cuál de las siguientes no es una medida para evitar absorción del tóxico:
a) Descontaminación oral b) D. digestiva c)Diuresis forzada d) D. Cutánea

ESTA NO ME SALE ESPECIFICAMENTE SOLO DICE QUE SE DEBE EVITAR LA


DESCONTAMINACION ORAL

40. El tx de las arritmias ventriculares por intoxicación con cocaína incluye:


a) Digoxina b) Amiodarona c) Nifedipina d) Propranolol*
Toxico
Efecto de reversión enzimática de la pralidoxima
tiempo:___1h a semanas_____
dosis: ADULTOS Y > 12 AÑOS: 1-2 g DILUÍDOS EN 100-200 ml DE SUERO
GLUCOSADO AL 5% O SSN. PASAR EN INFUSIÓN NO MAYOR DE 0.2 GRAMOS POR
MIN (10-20 ml/min.)
< DE 12 AÑOS: 20-40 mg/Kg de PESO, DILUÍDO EN SOLUCIÓN A RAZÓN DE 10
ml/Kg.

Efectos Agudos de los OF: bradicardia, hipotension, aumento de las secreciones,


alteraciones del estado (fatiga, debilidad muscular)

Efectos intermedios (48h): afectacion de la faringe, paralisis respiratoria, alteracion del


6to par craneal, reflejos ausentes o disminuidos.

Efectos graves (1 semana): parestesia, neurotoxicidad tardia, debilidad caudal cefalico


1. Paciente rociado con un tóxico llamado Malatión podrá presentar los siguientes
síntomas muscarínicos excepto:
a) lagrimeo b) sialorrea c) broncoespasmo d) rubicundez
** se produce es cianosis principalmente en labios y uñas los demás si.
2. Cuál de los siguientes síntomas y signos forma parte del Síndrome muscarinico en
intoxicación aguda de órganos fosforados excepto:
Dolor abdominal
Miosis
Taquicardia
Rinorrea
Diarrea

Los síntomas muscarínicos aparecen en las 4 primeras horas, y revierten con atropina ; los
más comunes son: vómitos, miosis y sialorrea.
diaforesis, pupilas puntiformes, sialorrea, epífora, broncoconstríccíón y aumento de la
secreción de las glándulas bronquiales, espasmo abdominal con vómito y diarrea,
bradicardia.
3. Cuál de los siguientes síntomas y signos forma parte del Síndrome nicotínico en
intoxicación aguda por órganos fosforados.
Ansiedad
Ataxia
Parálisis flácido
Convulsiones
Cefalea

Los síntomas nicotinicos aparecen algo más tarde que los anteriores, y no revierten con
atropina pero si con oximas, los más comunes son: fasciculaciones, temblor y debilidad.
taquicardia, fasciculaciones musculares o contracciones espasmódicas de los músculos
finos, en los casos más severos del diafragma y músculos respiratorios.
4. Es la dosis letal media de veneno para un adulto de 70 kg es, excepto
a) 24 mg de veneno crotálico y 150 mg de veneno laquésico
b) 5 mg de veneno de coral (15 mgs)
c) 70 mg de veneno bothropico
d) Todas las anteriores
e) Ninguna de las anteriores
**DE ESTA NO ESTOY SEGURA NO ENCONTRE NADA ESPECIFICO. UN PROFETA
DECIA NINGUNA DE LAS ANTERIORES.
5. La dosis inicial calculada de suero antiofidico polivalente seria

a) 1-4 ampollas

b) 5-9 ampollas

c) 10-15 ampollas

d) 15-20 ampollas

e) 25 ampollas

** A ESTA PREGUNTA SIENTO QUE LE HACE FALTA MAS


DATOS

6. Droga que es síntesis de laboratorio:


Tramadol

7. Nicotina cierta:

o Tiene receptores específicos


Es cierto pero no se a que se refiere en la nicotina en si o los receptores nicotínicos

8. Todo lo siguiente es cierto, excepto

a. el veneno de serpiente se produce en las glándulas homólogas a las paratiroides

b. la fracción tóxica del veneno de serpiente son proteínas y péptidos de 5,000 -30,000

c. el efecto biológico del veneno CROTALICO es estructuralmente reumático

d. la coral pertenece a la familia Elipidae

*El efecto es local con aparición de bulas, necrosis, ampollas, edema, hemorragia en el punto de la mordida y
sistémico mas que todo hematológicos (hemorragias, necrosis) y neuronales (ptosis palpebral, mareos, visión
borrosa, dolor de cabeza) DHE entre otros

9. La hemodiálisis como tratamiento de intoxicación por barbitúricos esta indicada en:

b. coma en estadio III


c. cuando el paciente esta en estupor
d. al llegar al cuarto de urgencia para prevenir el coma
e. edema agudo de pulmon
f. se el paciente tiene diarrea
Normalmente es para oliguria o anuria pero encontré en internet que la diuresis forzada podía aliviarlo

10. En el manejo de los accidentes ofídicos lo más importante es

a. Determinar que especie de serpiente está involucrada en el accidente

b. Trasladar a la víctima lo antes posible a un centro médico

c. Administrar suero antiofíodico específico

d. Determinar el grado de envenenamiento

e. Todas las anteriores

11. Con respecto a la picadura por alacrán en Panamá:


El veneno de escorpión es una secreción apocrina compuesta de neurotoxinas de 10,000 a 20,000
gramos/mole.
El centruroides margaritatus es la especie más comúnmente involucrada en accidentes por picadura de
escorpión.
El envenenamiento por el género Tityus no produce manifestaciones locales, solo sistémicas.
Todas son ciertas.

12. . En relación los efectos de los venenos todas son ciertas excepto:
El veneno de coral es neurotóxico y llega a las uniones mioneurales donde se produce un bloqueo sináptico.
El veneno de especies Bothrops produce alteración de la coagulación porque afecta el fibrinógeno y el factor
X de la coagulación.
El veneno de especies crotalus se fija en los tejidos a las 6 horas.
El veneno de escorpión actúa a nivel de terminaciones nerviosas preganglionares del sistema simpático y
parasimpático produciendo despolarización de las mismas y liberación de catecolaminas y acetilcolina.

13. Paciente de 35 años con historia de haber sido mordido por una serpiente en el pie derecho hace 10
horas mientras se encontraba cortando hierba en una finca, posterior a lo cual nota que se le hincha el
pie y tiene dolor local que se extiende hasta la pierna. Desconoce que vacunas ha recibido: No
alergias conocidas.
El examen físico muestra PA: 110/60 FC: 100 FR: 18 T: 37.9, escaso sangrado por encía en arcada
inferior derecha. Caries dentales. Cardiopulmonar normal, abdomen sin alteraciones. Edema,
eritema, calor local en pie y pierna derecha y 1 cm por arriba de la rodilla. Con dos puntos sangrantes
en maléolo externo. Biometria con leucocitosis y neutrofilia leves, TP y TPT prolongados en 12
segundos y plaquetas en 90,000. Urinalisis con 10-12 eritrocitos por campo, Glicemia, creatinina y
electrolitos normales.
Debe administrarse Toxoide tetánico 0.5cc I.M. como medida inmediata.
Debe administrarse 10 ampollas de Suero antiofídico polivalente por via intravenosa.
Los antibióticos de elección para tratar el proceso infeccioso serian Oxacilina y Clindamicina I.V.
Hay que administrarle al paciente Plasma fresco y plaquetas, conjuntamente con la administración del suero
antiofídico.
Todo lo anterior es cierto.

14. En caso de mordedura humana, cuál de las siguientes indicaciones no tendrían relevancia:
Toxicoide tetánico
Amoxicilina con ácido clavulanico
Metronidazol
Curación de la herida
Penicilina sódica

60. En el manejo de los accidentes ofídicos lo más importante es


f. Determinar que especie de serpiente está involucrada en el accidente
g. b. Trasladar a la víctima lo antes posible a un centro médico
h. c. Administrar suero antiofíodico específico
i. d. Determinar el grado de envenenamiento
j. e. Todas las anteriores

62. Niveles de paraquat en sangre que indican alta probabilidad de muerte


b. 0.9 md/L a las 24 horas

c. 1.5 mg/L a las 4 horas


d. 2.0 mg/L a las 4 horas
** Con exposiciones mayores a 40 mg/kg se observa falla orgánica
multisistémica y muerte en las siguientes 24 - 48 horas
61) Conociendo usted las propiedades bioquímicas del Paraquat. ¿Cuál será el objetivo de utilizar como
tratamiento inicial tierra o tierra fuller?

k. Bloquea la absorción intestinal y se elimina en orina


l. Elimina los metabolitos del mismo por orina.
m. Se inactiva en contacto con minerales arcillosos
n. Ayuda a eliminar el paraquat vía intestinal por heces al unirse a ciertas proteínas intestinales

15. En las características generales de los bipiridilos estos se inactivan al:

a) Se inactivan en presencia de la luz solar en


b) Se inactivan en presencia del agua.
c) Se inactivan con presencia de otro producto mezclado en el envase para producir emesis.
d) Se inactivan al tener contacto con los minerales arcillosos del suelo.

62) Usted aunque no lo crea esta esperando los niveles de Paraquat realizado por cromatografía para dializar.
Cuál sería el nivel que se puede beneficiar este paciente con dicho procedimiento:

e. 2.5 mg/Lt a las 4 horas


f. 1.0 mg/Lt a las 6 horas
g. 1.5 mg/Lt a las 6 horas
h. 1.83mg/Lt a las 4 horas

63) Usted es llamado al cuarto de urgencia por un paciente que está intoxicado por paraquat, para que le
realice lavado gástrico. Usted lo realiza si el paciente:

a. Tiene 6 horas de haber ingerido la sustancia


b. Tiene 7 horas de haber ingerido la sustancia
c. Tiene 4 horas de haber ingerido la sustancia (lei que solo ates de la primera hora de haber sido
expuesto)
d. No sabe el tiempo de haber ingerido, lo importante es salvarlo
16. LAS DIFERENCIAS ENTRE EL SAO( SUERO ANTIOFIDICO) LIOFILIZADO Y EL LIQUIDO
ES

a. EL SAO LIQUIDO DURA MAS QUE EL LIOFILIZADO b. EL SAO LIOFILIZADO NO


REQUIERE REFRIGERACIÓN

c. EL SAO LIQUIDO NO REQUIERE REFRIGERACIÓN,PERO DEBE SER GUARDADO EN


LUGARES FRESCOS

d. NINGUNA DE LAS ANTERIORES e. TODAS LAS ANTERIORES.


1. La vacunación con BCG utiliza:
a. Bacilos vivos
b. Bacilos atenuados
c. Bacilos muertos
d. Productos del bacilo
2. El grupo de bacterias famada HACEK son causantes de las endocarditis en un:
a. 10%
b. 30%
c. 50%
d. 70%
e. 90%
3. La endocarditis aguda generalmente es causada por unas de las siguientes bacterias
excepto:
a. S. aureus group
b. Streptococci viridiam
c. Streptoccos hemolítico
d. Penumococo
e. Gonococo
4. Es un criterio de mayor de DUKE para endocarditis:
a. Serología de Coxiella burneti
b. Fiebre
c. Petequia conjuntival
d. Uso de droga I.V.
e. Factor reumatoideo positivo
5. El tratamiento profiláctico de la endocarditis en manipulaciones orales debe de darse:
a. Una hora antes del procedimiento
b. Cuatro horas antes
c. Series horas antes
d. Ocho horas antes
e. Doce horas antes
6. El chancro en la fase temprana de la sífilis si no es tratado desaparece a los:
a. 10 a 14 días
b. 2 a 4 días
c. 20 a 25 días
d. 6 a 8 días
e. Un mes
7. Periodo de incubación del chancroide:
a. 10 a 14 días
b. 4 a 7 días
c. 14 a 17 días
d. 20 a 21 días
e. Un mes
8. Tratamiento de elección para infección por chlamidia tracomasis:
a. Fluoroquinolona
b. Ceftriaxona
c. Penicilina sódica
d. Tetraciclina
9. Paciente con diarrea sanguinolenta y fiebre de 39C de más de una semana de duración. El
coprocultivo sale positivo para campilobater yeyuni. ¿cual de los siguientes es el
antibiótico de elección?
a. Amoxicilina
b. Ciprofloxacina
c. Azitromicina
d. Cefotaxima
e. Gentamicina
10. Las tetraciclinas no se deben de utilizar en:
a. Gonorrea
b. Rickettsias
c. Vibrión colera
d. Granuloma inguinal
e. Clamidias
11. A usted le llega una paciente femenina de 16 años con un embarazo de 20 semanas, el
examen físico encuentra lesiones en la mucosa compatibles con sífilis secundaria que es
confirmada por test específico para treponema palidum, pero en el expediente tiene un
titulo que advierte que la paciente es alérgica a penicilina. ¿Qué terapéutica iniciaría
usted?
a. Interrumpir embarazo
b. Tratar con tetraciclina vía oral
c. Iniciar protocolo de desensibilización a la penicilina
d. Claritromicina parenteral por 7 días
12. La rifampicina:
a. Actúa sobre el DNA
b. Es bactericida
c. No atraviesa la barrera hematoencefálica
d. Dosis es de 5mg/kg/depeso
e. Se elimina principalmente por el riñón
13. El siguiente no es un fator de riesgo para cáncer de colon
a. antinflamatorios no esteroideos
b. ácidos biliares
c. tabaquismo
d. dietas altas en grasas
14. son factores en la clasificación de Child-Pugh, excepto:
a. Hematocrito
b. Ascitis
c. Bilirrubina
d. Albumina
e. Tiempo de protrombina
15. Un paciente que ingresa a la sala y usted le diagnostica una cirrosis hepática, un examen
que debe realizarse siempre es:
a. Colonoscopia
b. La arteriografía selectiva hepática
c. Serie esófago gastro duodenal
d. La endoscopia alta
16. Un paciente se le encuentra flapping tremor; se considera que:
a. Es un signo de hepatopatía crónica
b. Indica la existencia de un delirium tremens
c. Es un signo de encefalopatía hepática
d. Su ausencia descarta la existencia de una enfermedad hepática
e. Consiste en un temblor intencional de las manos.
17. Un varón de 19 años de edad acude al medico por queja desde hace 4 días de disuria. No
tiene dificultad al inicio de la micción, pero tiene le dolor al orinar. Niega tener fiebre,
escalofrió o dolor testicular. Es heterosexualmente activo, con numerosas parejas, a la
exploración física hay fiebre y el área genital es normal. Al ordenar la uretra del paciente,
produce una cantidad pequeña de material purulento, la tinción de gran revela >5
leucocitos por campo de alto poder sin diplococos intracelulares gram negativos ¿cual de
los siguientes microorganismos es la causa mas probable de los síntomas?
a. Chlamydia trochomatis
b. Escheriquia coli.
c. Micoplasma genitalium
d. Tricomona vaginalis
e. Ureaplasma urealyticum
18. En la encefalopatía hepática existe:
a. Estimulación dopaminérgica
b. Estimulación serotoninérgica
c. Estimulación gabaergica
d. Estimulación noradrenérgica
e. Ninguna de las anteriores.
19. La clasificación de DUKES del cáncer de colon tiene en cuenta los siguientes parámetros:
a. Invasión del tumor en la pared intestinal
b. Estado de los ganglios regionales
c. Afección de órganos a distancia del color
d. Ninguna de las anteriores.
20. Todos estos síntomas son propios de la intoxicación con inhibidores de colinesterasa
excepto:
a. Bradicardia
b. Fasciculaciones
c. Miosis
d. Sequedad de mucosa
e. Poliuria
21. Una de las siguientes bacterias o parásitos se asocia a síndrome de mal absorción.
a. Giardiasis
b. Amebiasis
c. Colera
d. Infección por bacilo dificile
e. Salmonelosis
22. Usted esta de turno en el cuarto de urgencias y lo llama la enfermera porque la infusión de
atropina se termino y el paciente intoxicado por carbamatos esta asintomático cual seria
el paso a seguir:
a. No continuar con la atropina
b. Continuarle a dosis baja esperando los niveles de colinesterasa
c. No continuar la atropina, pero administrarla en bolo de atropina si presenta
manifestaciones de broncorrea.
d. Continuar la misma infusión para cumplir el protocolo.
23. En el caso de la investigación de los 2 pacientes lo mandaron a casa porque se pensaba
que era una gastroenteritis sin saber que era una intoxicación por carbamatos regresan
por convulsiones y paro cardiorrespiratorio, usted pensaría que son manifestaciones de:
a. Síndrome nicotínico
b. Síndrome muscarínico a nivel pulmonar y cardiaco
c. Síndrome intermedio
d. Síndrome del sistema nervioso central
e. Síndrome de la medula espinal.
24. La causa mas frecuente de enfisema subcutáneo en el estado II y III de la intoxicación por
paranquat es:
a. Perforación pulmonar por el daño directo del paraquat al pulmón
b. Perforación de una bula pulmonar
c. La erosión directa en la piel que es severa
d. Perforación esofágica
e. Perforación traqueal
25. Cual de los siguientes síntomas y signos forma parte del síndrome muscarínico en
intoxicación aguda de órganos fosforados excepto:
a. Abdominal
b. Miosis
c. Parestesias
d. Rinorrea
e. Diarrea
26. Indique el efecto neurológico tardío provocado por intoxicación aguda con
organofosforado:
a. degeneración cerebelar
b. convulsiones
c. demencia
d. polineuropatía motora
e. sin efecto tardío
27. el antígeno carcinoembrionario es útil en el estudio de los pacientes con canceres
colorrectales:
a. porque es específico para estos tumores
b. detecta precozmente la aparición del cáncer
c. es eficaz para la detección de recidivas después de la extirpación del tumor
colorrectal
d. aumenta después de estudios endoscópicos
e. aumenta después de estudios radiológicos
28. paciente femenina de 65 años, con dolor súbito en la espalda a nivel de la vertebra T10.
Utiliza corticoides en forma crónica por atopia y asma bronquial. Usted sospecha
Osteoporosis ¿qué haría?
a. Densitometría ósea de cadera y columna
b. USG de calcáneo
c. Radiografías de columna torácica AP y lateral
d. Iniciaría tratamiento con calcio y vitamina D
e. Tratamiento para dolor y la observo.
29. Las fracturas más frecuentes después de las vertebrales y cadera en la osteoporosis son las
de:
a. Hombro
b. Colles
c. Tobillos
d. Rodillas
e. Dedos de la mano
30. Para evitar en lo posible la osteoporosis por corticoides se recomienda administrar uno de
los siguientes:
a. Calcio con vit D
b. Estrógenos
c. Andrógenos
d. Bifosfonato
e. Flúor
31. Las dos hormonas fundamentales en el metabolismo fosfocalcico son:
a. Calcitonina y ACTH
b. Parathormona y Vitamina D
c. Vitamina D y calcitonina
d. Parathormona y calcitonina
e. ACTH y parathormona
32. Y claro está, usted recuerda que en el hemograma la presencia de trombocitosis puede
representar todo lo siguiente excepto:
a. Vasculitis sistémica activa
b. Artritis reumatoide activa
c. Síndrome antifosfolípido
d. Policitemia vera
33. Cuando se le da tratamiento para osteoporosis a una paciente con raloxifeno, usted
siempre tiene pendiente que puede complicarse con:
a. El cáncer de seno
b. Trombosis venosa profunda
c. Hiperplasia cervical
d. Fracturas vertebrales
e. Riesgo de enfermedad coronaria
34. En el cáncer de colon lo siguiente es cierto:
a. La mortalidad por este cáncer ha ido en aumento, pues no se cuenta con terapias
apropiadas.
b. Es poco frecuente en mayores de 30 años
c. La mayoría de estos canceres son hereditarios
d. Los pólipos vellosos tienen mayor riesgo de malignidad.
35. Cual es la principal mutación que se encuentra en los pacientes con poliposis hereditaria
familiar.
a. Mutación en el gen K-ras
b. Mutación en el gen APC
c. Mutación en el gen p-53
d. Mutación en proteínas reparadoras del DNA
36. El Tx de las arritmias ventriculares por intoxicación con cocaína incluye:
a. Digoxina
b. Amiodarona
c. Nifedipina
d. Propanolol
e. Labetalol
37. Acerca de los anticuerpos anti-péptidos citrulinados cíclicos usted sabe que:
a. Prueba de laboratorio que puede usarse de tamizaje para artritis reumatoide en la
población general.
b. Prueba de laboratorio que por su alta sensibilidad es útil en el diagnostico de artritis
reumatoide temprana
c. Prueba de laboratorio que por su baja especificidad tiene a dar falsos positivos.
d. Prueba de laboratorio que por su alta especificidad puede discriminar
eficientemente a individuos sin artritis reumatoide de aquellos que la padecen.
38. Las vasculitis asociadas a ANCA tienen una distribución:
a. Mayoritariamente en el sexo femenino
b. Mayoritariamente en el sexo masculino
c. Distribución igual entre ambos sexos con casos más severos en hombres
d. Distribución igual entre ambos sexos con casos más severos en mujeres.
39. Un ejemplo de síndromes clínicos en los que debe considerar el Dx de vasculitis sistémica
seria:
a. Perdida de peso involuntaria de causa no determinada
b. Anemia hemolítica por anticuerpos calientes
c. Purpura trombocitopenia idiopática
d. Síndrome pulmonar renal
40. Usted sabe que la enfermedad de Kawasaki
a. Es una enfermedad que afecta vasos de pequeño calibre
b. Es una enfermedad que casi exclusivamente afecta a mayores de 50 años
c. Es una enfermedad que afecta preferentemente a niños menores de 5 años
d. Es una vasculitis sistémica asociada a enfermedades del tejido conectivo
41. De la artropatía por cristales de urato monosódico usted sabe:
a. Es una artritis simétrica aditiva crónica
b. Es una artritis asimétrica aditiva de predominio en miembros inferiores
c. Es una mono artritis aguda autolimitada
d. Es una artritis oligoarticular migratoria.
42. Durante un ataque de artritis por Ac. Úrico usted sabe que los niveles serios de Ac. Úrico
a. Están elevados hasta en el 80% de los casos
b. Están normales o bajos hasta en el 80% de los casos
c. Están elevados hasta en el 10% de los casos
d. Están normales o bajos hasta el 40% de los casos.

También podría gustarte